വ്യക്തിജീവിതം

//വ്യക്തിജീവിതം
“കൊള്ളയുദ്ധങ്ങളിൽ കീഴടക്കിയ ഗോത്രങ്ങളിലെ സ്ത്രീകളെ അവരുടെ ഭർത്താക്കൻമാരുടേയും കുട്ടികളുടേയും പിതാക്കൻമാരുടേയും മുന്നിലിട്ട് ബലാൽസംഗം ചെയ്ത ലിസ്റ്റ്…” മറുപടി പെരും നുണകൾ യാതൊരു ലജ്ജയുമില്ലാതെ പടച്ചുവിട്ടിരിക്കുകയാണിവിടെ എന്ന് പ്രത്യേകം പറയേണ്ടതില്ലല്ലൊ. “സ്ത്രീകളോട് പളുങ്കുപാത്രങ്ങൾ പോലെ സൗമ്യതയോടെ ഇടപെടണം” (സ്വഹീഹുൽ ബുഖാരി: 6209) എന്ന് പഠിപ്പിച്ച മുഹമ്മദ് നബി (സ)… “സ്ത്രീകളെ അവർ വെറുക്കുന്നത് ചെയ്യാൻ നിങ്ങൾ നിർബന്ധിക്കരുത്.” എന്ന് പഠിപ്പിച്ച മുഹമ്മദ് നബി (സ)… (മുസ്വന്നഫ് അബ്ദുർ റസാഖ്: 10320) “അല്ലാഹുവാണെ സാക്ഷി, അനാഥ, സ്ത്രീ എന്നീ രണ്ട് ദുർബല വിഭാഗങ്ങളുടെ അവകാശങ്ങളെ (നിറവേറ്റുന്നതിൽ പരാജയപ്പെടുന്നതിലുള്ള പാപത്തെ) സംബന്ധിച്ച് ഞാൻ നിങ്ങൾക്ക് ശക്തമായ താക്കീത് നൽകുന്നു.” എന്ന് സമുദായത്തെ താക്കീത് ചെയ്ത മുഹമ്മദ് നബി (സ)… (മുസ്നദു അഹ്‌മദ്‌: 2/439, റിയാളുസ്സ്വാലിഹീൻ: 146) തന്റെ ഭാര്യയുടെ അടിമസ്ത്രീയുമായി ലൈംഗിക ബന്ധത്തിൽ ഏർപ്പെട്ട ഒരു വ്യക്തിയുടെ കാര്യത്തിൽ വിധി പറയവെ അടിമസ്ത്രീ ബന്ധത്തിന് നിർബന്ധിക്കപ്പെടുകയാണ് ഉണ്ടായതെങ്കിൽ അവൾ സ്വതന്ത്രയാണെന്ന് പ്രഖ്യാപിച്ച മുഹമ്മദ് നബി (സ)… (മുസ്വന്നഫ് അബ്ദുർ റസാഖ്: 13417) “അടിമ സ്ത്രീകളെ പോലും വേശ്യാവൃത്തിക്ക് നിർബന്ധിക്കരുത്…” (ക്വുർആൻ: 24: 33) എന്ന് ലോകത്തെ പഠിപ്പിച്ച മുഹമ്മദ് നബി (സ)… ഈ മുഹമ്മദ് നബി (സ), യുദ്ധത്തിൽ ബന്ധികളാക്കപ്പെട്ട സ്ത്രീകളെ ബലാൽസംഗം ചെയ്യാൻ, അതും അവരുടെ ഭർത്താക്കന്മാരുടേയും കുട്ടികളുടേയും പിതാക്കൻമാരുടേയും മുന്നിൽ വെച്ച് ദ്രോഹിക്കാൻ അനുയായികളെ അനുവദിച്ചു അല്ലെങ്കിൽ അദ്ദേഹം തന്നെ ദ്രോഹിച്ചു എന്നത് ഈ നൂറ്റാണ്ടിലെ ഏറ്റവും വലിയ നുണയാണ്. മനുഷ്യ മനസ്സിൽ ഇന്നേ വരെ വർഗീയ വാദികൾ പാകിയ വിഷ വിത്തുകളിൽ ഏറ്റവും വലിയ വിധ്വേഷ വിത്താണ്. ഈ വിഷയകമായുള്ള കൂടുതൽ വിവരങ്ങൾക്ക് ആറ് ഭാഗങ്ങളായി സ്നേഹ സംവാദം വെബ്സിനിൽ ഈയുള്ളവൻ തന്നെ എഴുതിയ “ഇസ്‌ലാം അടിമസ്ത്രീകളെ ദ്രോഹിച്ചുവോ ?” എന്ന ലേഖനം വായിക്കുക: (https://www.snehasamvadam.org/ഇസ്‌ലാം-അടിമസ്ത്രീകളെ-ദ്/) അടുത്ത ആരോപണം മുഹമ്മദ് നബിക്ക് (സ) മക്കളുണ്ടായിരുന്നില്ല എന്ന കല്ലുവെച്ച നുണയാണ്. “മക്കളുണ്ടായിരുന്നില്ല” എന്ന ഭാഷയല്ല വിമർശകർ “മുഹമ്മദിന്റെ പെണ്ണുങ്ങൾ” എന്ന ലേഖനത്തിൽ പ്രയോഗിച്ചിരിക്കുന്നത് എന്ന് വായനക്കാർ മനസ്സിലാക്കണം. മറിച്ച് വിമർശകരുടെ വിമർശന ഭാഷയിലെ അശ്ലീലത കാരണം അതിന്റെ ശബ്‌ദാന്തര രചനയാണ് ലേഖകൻ ഇവിടെ ചേർത്തിരിക്കുന്നത്. ഏതായാലും, സ്വഹീഹായ നിവേദനങ്ങൾ പ്രകാരം മുഹമ്മദ് നബിക്ക് (സ) ഉണ്ടായിരുന്ന ഏഴു മക്കളുടെ പേരുകൾ അക്കമിട്ട് ഇവിടെ ചേർക്കാം: ആൺമക്കൾ: 1. അൽകാസിം 2. അബ്ദുല്ല 3. ഇബ്രാഹീം പെൺമക്കൾ: 1. സൈനബ് 2. റുക്വിയ്യ 3. ഉമ്മുകുൽസൂം 4. ഫാത്വിമ (സാദുൽ മആദ്: 1/103) മുഹമ്മദ് നബിയോടുള്ള(സ) വെറുപ്പ് സിരയിലും ശിരസ്സിലുമേന്തി നടക്കുന്നവരോട് അവസാനമായി ഒന്നേ പറയാനുള്ളു. പുരുഷോത്തമനായ ഈ തിരുദൂതൻ ലോകത്തിന് മുമ്പിൽ കൊളുത്തി വെച്ച ആദർശ – ധാർമ്മിക പാഠങ്ങളോടുള്ള വെറുപ്പിന്റെ ജ്വരമാണ്, ചരിത്ര നീതിയൊ വൈജ്ഞാനിക ധർമ്മമൊ തൊട്ടു തീണ്ടിയിട്ടില്ലാത്ത, ഈ അധപതിച്ച നിരൂപണ സംസ്കാരത്തിലേക്ക് നിങ്ങളെ എത്തിച്ചിരിക്കുന്നത്. നിഷ്പക്ഷതയോടെ ലോകത്തിന്റെ വിവിധ കോണുകളിലുള്ളവർ മുഹമ്മദ് നബിയെ(സ) കൂടുതൽ പഠിക്കാനും അദ്ദേഹത്തിൽ ആകൃഷ്ടനാകാനും, വിമർശകരുടെ ആദർശ രാഹിത്യം തിരിച്ചറിയാനും മാത്രമെ നിങ്ങളുടെ വിഷം പുരട്ടിയ വിമർശനങ്ങളും അപഹാസങ്ങളും ഉപകരിക്കൂ എന്ന് തിരിച്ചറിയുക… ആ മഹത് വ്യക്തിത്വം പരത്തുന്ന നന്മയുടെ പ്രഭാകിരണങ്ങളുടെ ഊഷ്മളതയിൽ പൊട്ടി പതിക്കുന്ന നീർകുമിളകളുടെ സ്ഥാനമേ മാനവ ചരിത്രത്തിൽ നിങ്ങളെയൊക്കെ കാത്തിരിക്കുന്നുള്ളൂ. { إِنَّ شَانِئَكَ هُوَ ٱلۡأَبۡتَرُ } “(നബിയെ,) തീര്‍ച്ചയായും നിന്നോട് വിദ്വേഷം വെച്ചു പുലര്‍ത്തുന്നവന്‍ തന്നെയാകുന്നു വാലറ്റവന്‍ (ഭാവിയില്ലാത്തവന്‍).” (സൂറത്തുൽ കൗസർ: 3) എല്ലാ നീർക്കുമിളകൾക്കും ‘ഊഷ്മളമായ’ “ഭാവി” നേരുന്നു…

“സ്വശരീരം മുഹമ്മദിന്റെ ലൈംഗികതയിലേക്ക് ദാനം ചെയ്ത സ്ത്രീകൾ ആണ് ബാക്കിയുള്ളവർ….” എന്ന വാചകം ‘ഹിബത്ത് വിവാഹങ്ങൾ’ എന്താണെന്നറിയാത്തതിൽ നിന്നൊ അറിവില്ലായ്മ നടിക്കുന്നതിൽ നിന്നൊ ഉണ്ടായതാണ്. അറബി ഭാഷയൊ ഇസ്‌ലാമിലെ സാങ്കേതിക പദങ്ങളൊ വിമർശകർക്ക് യാതൊരു അറിവുമില്ല. നാവും അസഭ്യങ്ങൾ നിറഞ്ഞ മനസ്സും മാത്രമാണ് ആധാരം. എതായാലും ‘ഹിബത്ത് വിവാഹങ്ങളെ’ സംബന്ധിച്ച് മറ്റൊരു ലേഖനത്തിൽ വ്യക്തമാക്കിയ വിവരം ഇവിടെയും ആവർത്തിക്കാം:

“ശരീരദാനം ചെയ്യുന്നുവെന്ന ആലങ്കാരിക പദപ്രയോഗത്തിലൂടെയുള്ള ‘ഹിബത്ത് വിവാഹങ്ങൾ’ കൊണ്ട് എന്താണ് ഉദ്ദേശിക്കുന്നത്? വിമര്‍ശകന്മാര്‍ ജല്‍പ്പിക്കും പ്രകാരം സ്വന്തം ശരീരത്തെ ഒരു സ്ത്രീ ലൈംഗികാസ്വാദനത്തിനായി നബി(സ്വ)ക്ക് സമര്‍പ്പിക്കുന്ന ഏര്‍പ്പാടാണോ അത്. അല്ലേ അല്ല.!! നാസ്തിക/മിഷനറി/ഷോവനിസ്റ്റ് ഞരമ്പുരോഗികളുടെ ശിഫക്കായി (ശമനം) നമുക്കതു മന്ത്രിച്ചു കൊടുക്കാം.

വിശുദ്ധ ക്വുര്‍ആന്‍ പറഞ്ഞു: “സത്യവിശ്വാസിയായ സ്ത്രീ സ്വന്തത്തെ പ്രവാചകന് ദാനം ചെയ്യുകയും അവളെ വിവാഹം കഴിക്കാനുദ്ദേശിക്കുകയുമാണെങ്കില്‍ അതിനും വിരോധമില്ല.” (ക്വുര്‍ആന്‍: 33: 50)

‘വിവാഹം കഴിക്കാനുദ്ദേശിക്കുകയുമാണെങ്കില്‍’ എന്നാണ് ക്വുര്‍ആന്‍ ഇവിടെ പരാമര്‍ശിച്ചിരിക്കുന്നത്. അഥവാ വിമര്‍ശകന്മാര്‍ ആരോപിക്കും വിധം ലൈംഗികാസ്വാദനത്തിനായി നബി(സ്വ)ക്ക് ഒരു സ്ത്രീ സ്വശരീരത്തെ സമര്‍പ്പിക്കുന്ന ഏര്‍പ്പാടല്ല ഇത്. മറിച്ച് വിവാഹത്തിനായി സ്വന്തത്തെ സമര്‍പ്പിക്കലാണ്. അഥവാ സാധാരണ വിവാഹത്തില്‍ നിന്നും വ്യത്യാസമായി ഇവിടെ സ്ത്രീ മഹ്ര്‍ ഒഴിവാക്കും. ‘ഹിബത്ത്’ വിവാഹം എന്നാണ് ഇതറിയപ്പെടുന്നത്. ഇത് പ്രവാചകനു മാത്രം ബാധകമായ നിയമമാണ്. എല്ലാ പണ്ഡിതന്മാരും അതു വ്യക്തമാക്കിയിട്ടുണ്ട്.

”സത്യവിശ്വാസിനിയായ ഒരു സ്ത്രീ സ്വദേഹം നബിക്ക് ദാനം ചെയ്യുന്ന പക്ഷം നബി അവളെ വിവാഹം കഴിക്കാന്‍ ഉദ്ദേശിക്കുന്നെങ്കില്‍ അതും (അനുവദിച്ചിരിക്കുന്നു.)” അഥവാ മഹ്ര്‍ ഇല്ലാതെ വിവാഹം ചെയ്യാന്‍ ആവശ്യപ്പെട്ടാല്‍. ഹിബത്ത് (സ്വദേഹം ദാനം ചെയ്യുന്നു) എന്ന പദം ഉപയോഗിച്ച് മഹ്ര്‍ ഇല്ലാത്ത നിക്കാഹാണ് ഇവിടെ ഉദ്ദേശം.” (തഫ്‌സീറുല്‍ ജലാലൈനി: 33: 50 ന്റെ വ്യാഖ്യാനം)

“ഇബ്‌നു അബ്ബാസ് (റ) പറഞ്ഞു: അല്ലാഹുവിന്റെ ദൂതന്റെ പത്‌നിമാരില്‍ സാധാരണ ചടങ്ങിലൂടെയൊ വലങ്കൈ ഉടമപ്പെടുത്തിയതോ അല്ലാതെ ആരും ഉണ്ടായിരുന്നില്ല. ഹിബത്ത് (സ്വദേഹം ദാനം ചെയ്യുന്നു) എന്ന പദം ഉപയോഗിച്ച് മഹ്ര്‍ ഇല്ലാത്ത നിക്കാഹിന് തയ്യാറായ ഒരു സ്ത്രീയേയും പ്രവാചകന്‍ (സ) സ്വീകരിച്ചിട്ടില്ല. അവരില്‍ പെട്ട ആരും പ്രവാചകന് (സ) ഉണ്ടായിരുന്നില്ല.” (തഫ്‌സീറുല്‍ കുര്‍തുബി: 33: 50 ന്റെ വ്യാഖ്യാനം)

“മഹ്ര്‍ കൂടാതെ, എന്നെ വിവാഹം ചെയ്യണോ എന്ന വിധി താങ്കള്‍ക്ക് ഞാന്‍ ഇഷ്ടദാനം ചെയ്യുന്നു.” (അല്‍ മുഫ്ഹിം: ഇമാം കുര്‍ത്തുബി: പേജ്: 4/128)

“ഹിബത്ത് (സ്വദേഹം ദാനം ചെയ്യുന്നു) എന്ന പദം ഉപയോഗിച്ച് മഹ്ര്‍ ഇല്ലാത്ത നിക്കാഹിന് തയ്യാറായ സ്ത്രീകളെ വിവാഹം ചെയ്യല്‍ പ്രവാചകന് അനുവദിക്കപ്പെട്ടിട്ടും പ്രവാചകന്‍ (സ) അതു ചെയ്തില്ല എന്നാണ് ഇബ്‌നു അബ്ബാസ് (റ) പറയുന്നത്.” (ഫത്ഹുല്‍ ബാരി: 8: 526)

“ഞാന്‍ എന്നെ താങ്കള്‍ക്കു മുമ്പില്‍ വിവാഹത്തിനായി ഇഷ്ടദാനം ചെയ്യുന്നു’ എന്ന വാചകത്തില്‍ ഒരു മുള്വാഫ് (Possession) (ഭാഷാ പരമായ ഭംഗിക്കായി) വിട്ടുകളഞ്ഞതാണ്. യഥാർത്ഥത്തില്‍ വാചകത്തിന്റെ വിവക്ഷ ഇപ്രകാരമാണ്: ‘ഞാന്‍ എന്നെ അഥവാ എന്റെ വിവാഹ കാര്യത്തെ താങ്കള്‍ക്കു മുമ്പില്‍ ഇഷ്ടദാനം ചെയ്യുന്നു’. കാരണം ഒരു സ്വതന്ത്ര സ്ത്രീ ഉടമപ്പെടുത്തപ്പെടുകയോ ദാനം ചെയ്യപ്പെടുകയോ ഇല്ലല്ലോ. മഹ്ര്‍ ഇല്ലാതെ തന്നെ താങ്കളെ വിവാഹം ചെയ്യാന്‍ ഞാന്‍ തയ്യാറാണ് എന്നാണ് ആ സ്ത്രീ പറഞ്ഞതിന്റെ വിവക്ഷ.” (ഫത്ഹുല്‍ ബാരി: 9/112, ഫത്ഹുല്‍ മുന്‍ഇം: 5/540)

നബി(സ്വ)ക്ക് ലൈംഗികാസ്വാദനത്തിനായി സ്ത്രീകള്‍ അവരുടെ ശരീരം ദാനം ചെയ്യുന്ന ഏര്‍പാടല്ല ‘ഹിബത്ത്’. മറിച്ച് മഹ്ര്‍ (വിവാഹ മൂല്യം) ഇല്ലാതെ തന്നെ അവരെ വിവാഹം ചെയ്യാനുള്ള അവകാശം പ്രവാചകനു സമര്‍പിച്ചുകൊണ്ട് നടത്തപ്പെടുന്ന വിവാഹ രീതിയാണത്. നബി(സ്വ)യുടെ പത്‌നിപദം ആഗ്രഹിച്ചുകൊണ്ടാണവര്‍ അപ്രകാരമുള്ള വിവാഹത്തിനു സന്നദ്ധത അറിയിക്കുന്നത്. എന്നാല്‍ അത്തരത്തിലുള്ള ‘ഹിബത്ത്’ വിവാഹം നബി (സ്വ) ഒരിക്കലും ചെയ്തിട്ടില്ലെന്നാണ് പ്രവാചക ശിഷ്യനും സന്തതസഹചാരിയുമായ ഇബ്‌നു അബ്ബാസ് (റ) പറയുന്നത്.” (നബിപാഠങ്ങളിൽ പെൺ വിരുദ്ധതയില്ല !!! 8)

മറുപടി:

ആറ് വരികളിൽ ആറും ദാക്ഷിണ്യമില്ലാത്ത നുണകളാണ്, മിഷണറിമാരുടെ നുണകളും നവനാസ്തിക തെറികളും കൂട്ടി കുഴച്ചു ചുട്ടെടുത്ത ഈ ലേഖനത്തിൽ നിന്ന് നാം വായിക്കുന്നത്.

ക്വുർആൻ കൊണ്ടോ സ്വഹീഹായ ഹദീസുകൾ കൊണ്ടോ അവലംബനീയമായ സീറ: (നബി ചരിത്രം) കൊണ്ടോ മുഹമ്മദ് നബിക്ക്(സ) 61 പോയിട്ട് ഒരു അവിഹിത ബന്ധമെങ്കിലും ഉണ്ടായിരുന്നെന്ന് തെളിയിക്കാൻ കഴിയാതെ നിരാശരായ മിഷണറിമാർ നുണകളും അർധ സത്യങ്ങളും ദുർവ്യാഖ്യാനങ്ങളും നടത്തി പണികഴിച്ച ചീട്ടു കൊട്ടാരങ്ങളാണ് ഈ ആരോപണങ്ങൾ. പാപസുരക്ഷിതരും വിശുദ്ധരുമായ ദൈവദൂതന്മാരെ പോലും ബലാൽസംഗക്കാരും, സ്ത്രീ പീഢകരും, അഗമ്യഗമനക്കാരുമൊക്കെയായി സ്വന്തം വേദഗ്രങ്ങളിൽ വക്രീകരിച്ച് അവതരിപ്പിക്കാൻ യാതൊരു സങ്കോചവും ഇല്ലായിരുന്ന ഒരു വിഭാഗത്തിൽ നിന്ന് ഇതിൽ കുറവൊന്നും പ്രതീക്ഷിക്കേണ്ടതില്ലല്ലൊ! (ബൈബിൾ: 2 സാമുവൽ 11-12, ഉല്പത്തി 19:30-35, 1 രാജാക്കന്മാർ: 11:3)

ബാലാനുരാഗവും അഗമ്യഗമനവും കുടുംബരതിയും മൃഗരതിയും ശവരതിയും എന്തിനേറെ ബലാത്സംഗം പോലും വസ്തുനിഷ്ടമായ തെറ്റല്ല എന്ന് തുറന്നടിക്കുകയും മോണോഗമി പ്രകൃതി വിരുദ്ധമാണെന്ന് വാദിക്കുകയുമെല്ലാം ചെയ്ത നവനാസ്തികർ (https://youtu.be/FFWkS5nwXjo

https://youtu.be/sDLUYpgzEwo

https://www.washingtonpost.com/national/on-faith/richard-dawkins-under-fire-for-mild-pedophilia-remarks/2013/09 )

ഏത് ധാർമ്മിക ഭൂമികയിൽ ഊന്നി നിന്നു കൊണ്ടാണ് മുഹമ്മദ് നബിയുടെ(സ) (ആരോപിതമായ) ഈ ‘വിവാഹങ്ങളെ’ വിമർശിക്കുന്നത് എന്ന് മനസ്സിലാവുന്നില്ല !!

ബഹുഭാര്യത്വത്തെയും അടിമ സ്ത്രീകളുമായുള്ള ഉപവിവാഹത്തെയും ബൈബിൾ വിശ്വാസികൾ അവമതിക്കുന്നത് ഏറെ ആശ്ചര്യജനകമായ ഒരു നവീന പ്രതിഭാസം തന്നെയാണ്.

അബ്രഹാം പ്രവാചകന് മൂന്ന് ഭാര്യമാരും അടിമ സ്ത്രീകളുമുണ്ടായിരുന്നെന്ന് ബൈബിൾ പഴയ നിയമം വാദിക്കുന്നു. (ഉല്പത്തി: 12:20, 16:3, 1:25, 25:6)

യാകോബ് പ്രവാചകന് നാല് ഭാര്യമാരും അടിമ സ്ത്രീകളുമടക്കം നാല് സ്ത്രീകളുണ്ടായിരുന്നു. (32 : 22).

ദാവൂദിന് ഒമ്പത് സ്ത്രീകളുണ്ടായിരുന്നു. (2 സാമുവേൽ: 3:1-6, 6:23, 11:26, 1 രാജാക്കന്മാർ: 1:1-5)

സോളമൻ രാജാവിന് എഴുന്നൂറ് ഭാര്യമാരും മുന്നൂറ് ഉപഭാര്യമാരും ഉണ്ടായിരുന്നു. (1 രാജാക്കന്മാർ: 3:11)

ഇതൊന്നും പോരാതെ, യുദ്ധം ചെയ്ത് കീഴ്‌പ്പെടുത്തിയ സമൂഹങ്ങളിലെ ചെറിയ പെൺകുട്ടികളെ കണക്കില്ലാതെ കൈവശം വെക്കാം എന്ന വിചിത്രമായ ദൈവകൽപ്പനയും ബൈബിളിൽ കാണാം.

ജൂബിലി ബൈബിൾ, ലിവിംഗ് ബൈബിൾ, വെബ്സ്റ്റേഴ്സ് എന്നീ ബൈബിളിന്റെ ഇംഗ്ലീഷ് പരിഭാഷകൾ ഒന്നിച്ച് വെച്ച് ബൈബിളിലെ സംഖ്യാപുസ്തകത്തിലെ 31 ആം അധ്യായത്തിലെ 17, 18 വാക്യങ്ങളിൽ മോശെയുടെ കൽപ്പനയായി ഇപ്രകാരം നാം വായിക്കുന്നു:

1. But all the female children that have not known a man by lying with him keep alive for yourselves. (Jubilee Bible 2000: Numbers 31:18)

(https://www.biblegateway.com/passage/?search=Numbers%2031%3A18&version=JUB)

2. Only the little girls may live; you may keep them for yourselves. (Living Bible. (TLB) : Numbers 31:18)

(https://www.biblegateway.com/passage/?search=Numbers%2031%3A18&version=JUB)

3. “But all the female children that have not known a man by lying with him, keep alive for yourselves.” (Webster’s Bible: Numbers 31:18)

(https://www.biblestudytools.com/search/?s=bibles&q=Female+children&t=wbt&c=all)

“ഇനി എല്ലാ ആൺകുട്ടികളേയും നിങ്ങൾ കൊല്ലുക. പുരുഷന്മാരുമായി ശയിച്ചിട്ടുള്ള സ്ത്രീകളേയും കൊന്നുകളയുക. പക്ഷെ പുരുഷന്മാരുമായി ഒരിക്കലും ശയിച്ചിട്ടില്ലാത്ത ചെറിയ പെൺകുരുന്നുകളെ നിങ്ങൾക്ക് വേണ്ടി എടുത്തു കൊൾക.” (Numbers 31: 17-18)

മോശെയുടെ ഈ കൽപ്പന പ്രകാരം കന്യകളും ചെറുപ്രായക്കാരുമായ 32,000 പെൺകുഞ്ഞുങ്ങൾ ശത്രു സമൂഹങ്ങളിൽ നിന്ന് അടിമകളായി എടുക്കപ്പെട്ടു. (Numbers 31: 35-40)

പീറ്റർ പെറ്റിന്റെ ബൈബിൾ വിവരണം സംഖ്യ പുസ്തകത്തിലെ വാക്യത്തെ (31: 17-18) ഇപ്രകാരം വിശദീകരിച്ചു:

“അത്തരം പെൺകിടാങ്ങളെ വിവാഹം ചെയ്യാനോ ദാസ്യവേല ചെയ്യാനോ വേണ്ടി സ്വീകരിക്കാമല്ലോ. അങ്ങനെ വരുമ്പോൾ അവർ ഭർത്താവിന്റെ അല്ലെങ്കിൽ യജമാനന്റെ മതം സ്വീകരിക്കുന്നത് പ്രതീക്ഷിക്കാം. അവർ കാലക്രമേണ ഇസ്രാഈൽ സമൂഹത്തിന്റെ ഭാഗമായി തീരുകയും ചെയ്യും.”

ദി ന്യൂ അമേരിക്കൻ കമന്ററി:

“ചെറിയ പെൺകുട്ടികളെ മാത്രമേ ജീവിക്കാൻ അനുവദിക്കാവൂ. അപ്പോഴാണല്ലൊ തോറയിലെ നിയമാവർത്തനത്തിലെ (20: 13-14; 21: 10-14) യുദ്ധ വിധികൾ പ്രകാരം അവരെ ഭാര്യമാരായോ ഇസ്രാഈലി പുരുഷന്മരാരുടെ അടിമകളായോ സ്വീകരിക്കാൻ സാധിക്കൂ. ഇതിലൂടെ അവരെ വിശ്വാസി സമൂഹത്തിന്റെ കുടകീഴിൽ കൊണ്ടുവരാൻ സാധിക്കുമല്ലൊ.”

ജോൺ ഡമ്മലോവിന്റെ ബൈബിൾ വിവരണം:

“ആൺകുഞ്ഞുങ്ങളെ വധിക്കാൻ കൽപ്പിച്ചത് വിഗ്രഹാരാധകരുടെ വർഗത്തെ തന്നെ വേരോടെ പിഴുതുകളയുക എന്ന ലക്ഷ്യത്തോടെയാണ്. മുതിർന്ന സ്ത്രീകളാകട്ടെ വിശ്വാസത്യാഗത്തിന്റെ പ്രധാന കാരണക്കാരായതിനാലും ഭാവിയിൽ ആളുകളെ വഴിതെറ്റിക്കാൻ സാധ്യതയുള്ളതിനാലും കൊല്ലപ്പെടുന്നു. എന്നാൽ ഇളം പ്രായക്കാരായ പെൺകുട്ടികളെ വധശിക്ഷയിൽ നിന്ന് ഒഴിവാക്കിയത്, അവരെ അടിമകളോ ഭാര്യമാരോ ആയി എടുക്കാനും, ഒരുപക്ഷേ എബ്രായ ജനതയിലേക്ക്‌ മതപരിവർത്തനം നടത്താനും സാധിക്കുമെന്നതിനാലുമാണ്.”

ഇനി ഹിന്ദു മതത്തിലേക്ക് വരാം. ആമുഖങ്ങളൊന്നും ഇല്ലാതെ തന്നെ സുലൈമാൻ റസ്‌വിയുടെ ‘ബഹുഭാര്യത്വം ഹിന്ദു ധർമ്മത്തിൽ’ (Polygamy in Hindu Dharma) എന്ന ഒരു ലേഖത്തിലെ പ്രധാന ഭാഗങ്ങൾ ചേർക്കുന്നതിന് ഇവിടെ പ്രസക്തിയുണ്ട്:

“ഗവൺമെന്റിന്റെ 1961 ലെ സെൻസസ് അനുസരിച്ച്, മുസ്‌ലിങ്ങളുടെ 5.7% മായി താരതമ്യം ചെയ്യുമ്പോൾ ഹിന്ദുക്കളിൽ 5.8% ബഹുഭാര്യത്വത്തിൽ ഏർപ്പെടുന്നവരായിരുന്നു. ബുദ്ധമതത്തിലെ പുരുഷന്മാർക്കിടയിൽ ബഹുഭാര്യത്വം 7.9%, ജൈനർ 6.7%, ബഹുഭാര്യത്വം ഏറ്റവും കൂടുതലുള്ളത് ആദിവാസികളിലാണ്, 15.25% പേർ ബഹുഭാര്യത്വം ആചരിക്കുന്നു.

ഇന്ത്യയിലെ അമുസ്‌ലിംകൾക്ക് ഒന്നിലധികം ഭാര്യമാരുള്ളത് നിയമവിരുദ്ധമാണ്. ഇതൊക്കെയാണെങ്കിലും, പല ഹിന്ദുക്കൾക്കും ഒന്നിലധികം ഭാര്യമാരുണ്ട്. 1974-ൽ പുറത്തുവന്ന ഔദ്യോഗിക റിപ്പോർട്ടുകൾ, ഹിന്ദു ദ്വിഭാര്യത്വം നിരോധിച്ച് ഏകദേശം രണ്ട് പതിറ്റാണ്ടുകൾക്ക് ശേഷം, ഹിന്ദുക്കൾക്കിടയിൽ ബഹുഭാര്യത്വം മുസ്‌ലിങ്ങളെക്കാൾ കൂടുതലാണെന്ന ഞെട്ടിക്കുന്ന വസ്തുത എടുത്തുകാട്ടുന്നു (ആദിവാസികൾ: 15 ശതമാനം, ഹിന്ദുക്കൾ: 5.8 ശതമാനം, ജൈനർ: 6.7 ശതമാനം, ബുദ്ധമതക്കാർ: 7.9 ശതമാനം, മുസ്‌ലിങ്ങൾ: 5.6 ശതമാനം). തുടർന്നുള്ള ദശാബ്ദങ്ങളിലെ കണക്കുകൾ ലഭ്യമല്ല. വ്യത്യാസം നിസ്സാരമെന്ന് തോന്നുമെങ്കിലും യഥാർത്ഥത്തിൽ അത് വളരെ വലുതാണ് – ഒരു കോടിയോളം വരുന്ന ഹിന്ദു പുരുഷന്മാർക്ക് ഒന്നിലധികം ഭാര്യമാരുണ്ടായിരുന്നു, വെറും 12 ലക്ഷം മുസ്‌ലിങ്ങൾക്കും.

2006-ൽ നടത്തിയ ദേശീയ കുടുംബാരോഗ്യ സർവേ കാണിക്കുന്നത് 2.5% മുസ്‌ലിം പുരുഷന്മാരും 1.7% ഹിന്ദു പുരുഷന്മാരും 2.1% ക്രിസ്ത്യൻ പുരുഷന്മാരും ബഹുഭാര്യത്വം ആചരിക്കുന്നവരാണെന്നാണ്. ഹിന്ദുക്കളിലും മുസ്‌ലിങ്ങളിലും ബഹുഭാര്യത്വ വിവാഹങ്ങളുടെ എണ്ണം ഗണ്യമായി കുറഞ്ഞു. എന്നാൽ ഇത്തവണ ഹിന്ദുക്കളും മുസ്‌ലിങ്ങളും തമ്മിൽ 0.8% അന്തരമുള്ള മുസ്‌ലിങ്ങൾക്കിടയിൽ ബഹുഭാര്യത്വത്തിന്റെ ശതമാനം കൂടുതലാണ്. എന്നാൽ ഈ 0.8% വിടവ് ഒരു വലിയ സംഖ്യയല്ല. ഇന്ത്യൻ നിയമം മുസ്‌ലിങ്ങൾക്ക് ബഹുഭാര്യത്വം അനുവദിക്കുമ്പോൾ, ഹിന്ദു വിവാഹ നിയമം 1955 ഹിന്ദു പുരുഷന്മാർക്ക് ഒന്നിലധികം ഭാര്യമാരെ ഒരേസമയം വിലക്കുന്നുണ്ട്, ഈ നിരോധനം ഉണ്ടായിരുന്നിട്ടും 1.7% ഹിന്ദുക്കൾ ഇപ്പോഴും ബഹുഭാര്യത്വം ആചരിക്കുന്നു എന്നത് ശ്രദ്ധിക്കേണ്ടതാണ്. ഒന്നിലധികം ഭാര്യമാരുണ്ടാകാൻ ഇന്ത്യൻ നിയമം അനുവദിക്കുന്നെങ്കിൽ ഹിന്ദുക്കളിൽ ബഹുഭാര്യത്വത്തിന്റെ തോത് സങ്കൽപ്പിക്കുക. ഇന്ത്യൻ നിയമകൂടം ശരീഅ: നിയമത്തെ അടിസ്ഥാനമാക്കി മുസ്‌ലിങ്ങൾക്ക് ബഹുഭാര്യത്വം അനുവദിക്കുന്നു, എന്നിട്ടും 2.5% പുരുഷന്മാർ മാത്രമേ ബഹുഭാര്യത്വം അനുഷ്ഠിക്കുന്നുള്ളൂ. 1961-ലെയും 1974-ലെയും സെൻസസ് കൂടുതൽ പ്രാധാന്യമർഹിക്കുന്നു, കാരണം വിവാഹങ്ങളെ മതപരമായ രീതിയിൽ നോക്കിക്കാണുന്ന രണ്ട് ഔദ്യോഗിക സെൻസസ് മാത്രമാണവ.

സ്രോതസ്സുകൾ:-

http://scroll.in/article/669083/muslim-women-and-the-surprising-facts-about-polygamy-in-india

http://timesofindia.indiatimes.com/articleshow/5004493.cms

http://www.cpsindia.org/index.php/pub/87-religious-demography-of-india/reviews/84-indian-currents-lies-half-truths-and-statistics-by-john-dayal

http://indianexpress.com/article/opinion/columns/three-is-a-crowd/

ഹൈന്ദവ ഗ്രന്ഥങ്ങളിൽ ബഹുഭാര്യത്വം:

പദ്‌മ പുരാണ V.57.27-40 പ്രകാരം രാമന്റെ പിതാവ് ദശരഥ രാജാവിന് 3 ഭാര്യമാർ ഉണ്ടായിരുന്നത്രെ. കൗശല്യ, സുമിത്ര, കൈയേയി… പദ്‌മ പുരാണ V.116.42-45 പ്രകാരം അദ്ദേഹത്തിന് നാല് ഭാര്യമാരാണ്.

ശ്രീകൃഷ്ണന് എട്ട് മുഖ്യ ഭാര്യമാരെ കൂടാതെ 16,100 ഉപഭാര്യമാർ ഉണ്ടായിരുന്നു.

വിഷ്ണുപുരാണം പുസ്തകം 5.28.1-5 “ചാരുദേശൻ, സുദേശൻ, ചാരുദേഹം, സുഷേണൻ, ചാരുഗുപ്തൻ, ഭദ്രാചാരു, ചാരുവിന്ദൻ, സുചരു, അതിശക്തനായ ചാരു എന്നീ പുത്രന്മാരെ രുക്മിണി കൃഷ്ണനിൽ പ്രസവിച്ചു; ചാരുമതി എന്ന ഒരു മകളും. കൃഷ്ണന് മറ്റ് ഏഴ് സുന്ദരിമാരായ ഭാര്യമാരുണ്ടായിരുന്നു, കാളിന്ദി, മിത്രവൃന്ദ, സദാചാരിയായ നാഗജിതി, രാജ്ഞി ജാംബവതി; രോഹിണി, മദ്രയിലെ രാജാവായ മാദ്രിയുടെ സ്നേഹനിധിയും മികച്ചതുമായ പുത്രി; സത്യഭാമ, ശത്രുജിത്തിന്റെ മകൾ; മനോഹരമായ പുഞ്ചിരിക്കുടമ ലക്ഷ്മണയും. ഇവരെക്കൂടാതെ അദ്ദേഹത്തിന് പതിനാറായിരം ഭാര്യമാരും ഉണ്ടായിരുന്നു.” (വിവർത്തന: എച്ച്.എച്ച്.വിൽസൺ)

ബ്രഹ്മപുരാണം 95.12-18 “…നരകന്റെ ശേഖരത്തിൽ നിന്ന് സേവകർ കൊണ്ടുവന്ന ആനകളും കുതിരകളും മറ്റ് സമ്പത്തും കൃഷ്ണൻ കൈവശപ്പെടുത്തി. കൃഷ്ണൻ, ഒരു ശുഭദിനത്തിൽ നരകന്റെ വസതിയിൽ നിന്ന് കൊണ്ടുവന്ന യുവതികളെ വിവാഹം കഴിച്ചു. ഹേ ഉത്തമ ബ്രാഹ്മണരേ, ഇവരിൽ ഓരോരുത്തർക്കും പ്രത്യേകമായ ശരീരത്തോടെ, കൃഷ്ണൻ ഭക്തിയോടെ അവരെ വിവാഹം കഴിച്ചു. പതിനാറായിരത്തി നൂറോ അതിലധികമോ സ്ത്രീകൾ ഉണ്ടായിരുന്നു അവർ. ഭഗവാൻ കൃഷ്ണൻ പല രൂപങ്ങൾ സ്വീകരിച്ചു. എന്നാൽ ആ കന്യകമാർ അവനെ തങ്ങളുടെ ഏക ഭർത്താവായി നിനച്ചു; ‘കൃഷ്ണൻ എന്നെ മാത്രം വിവാഹം കഴിച്ചു എന്ന്.’ രാത്രികളിൽ, ബ്രാഹ്മണരേ, പ്രപഞ്ചത്തിന്റെ സ്രഷ്ടാവായ കൃഷ്ണൻ, അവരുടെ എല്ലാവരുടെയും വാസസ്ഥലങ്ങളിൽ താമസിച്ചു.” (വിവർത്തനം: ബോർഡ് ഓഫ് സ്കോളേഴ്സ്, എഡിറ്റ് ചെയ്തത് ജെ.എൽ. ശാസ്ത്രി)

കൃഷ്ണന്റെ പിതാവായ വാസുദേവന് 14 ഭാര്യമാരുണ്ടായിരുന്നു.

ബ്രഹ്മപുരാണം 112.35 “വാസുദേവന് തന്റെ ഭാര്യമാരായി പതിനാല് വിശിഷ്ട സ്ത്രീകളുണ്ടായിരുന്നു. ആദ്യത്തെ അഞ്ച് പേരായിരുന്നു: രോഹിണി, മദിര, വൈശാഖി, ഭദ്ര, സുനാംനി എന്നിങ്ങനെ പേരുള്ള പുരിയുടെ സന്തതികൾ. സഹദേവൻ, ശാന്തിദേവൻ, ശ്രീദേവി, ദേവരക്ഷിത, വൃകാദേവി, ഉപാദേവി, ദേവകി എന്നിവരടങ്ങിയ ഏഴ് സ്ത്രീകളുടെ രണ്ടാമത്തെ ശൃംഖലയും. പതിമൂന്നാമത്തേതും പതിന്നാലാമത്തേതും സുതനുവും യാദവിയും ആയിരുന്നു. ഇവർ രണ്ടുപേരും ആദ്യം വേലക്കാരികളായിരുന്നു. പ്രസിദ്ധനായ സൗരി (കൃഷ്ണൻ) ദേവകിയുടെയും വസുദേവയുടെയും മകനായി ജനിച്ചു…” (വിവർത്തനം: ബോർഡ് ഓഫ് സ്കോളേഴ്സ്, എഡിറ്റ് ചെയ്തത് ജെ.എൽ. ശാസ്ത്രി)

വാസുദേവന് 14 ഭാര്യമാരുണ്ടെന്ന് ഹരിവംശ ​​പുരാണത്തിൽ 1.35.3-ലും പരാമർശിച്ചിട്ടുണ്ട്.

ചന്ദ്രദേവനായ സോമയ്ക്ക് 27 ഭാര്യമാരുണ്ടായിരുന്നു.

ബ്രഹ്മപുരാണം 1.173: “സോമന്റെ ഇരുപത്തിയേഴ് ഭാര്യമാരെന്ന് പരാമർശിക്കപ്പെട്ട പുണ്യ ചടങ്ങുകളിൽ വിവാഹിതരായ സ്ത്രീകളിൽ നിന്ന് അളവറ്റ തേജസ്സുള്ള മിടുക്കരായ കുട്ടികൾ ജനിച്ചു.”

27 ഭാര്യമാരുള്ള സോമയെ സംബന്ധിച്ച് സ്കന്ദപുരാണം V.ii.26.1-6-ലും വരാഹപുരാണം 35.1-2 ലും പരാമർശിക്കപ്പെട്ടിട്ടുണ്ട്.

ബ്രഹ്മ വൈവർത്ത പുരാണം, ബ്രഹ്മ ഖണ്ഡം 9.42: “…ഇപ്പോൾ ഞാൻ ചന്ദ്രദേവന്റെ ഭാര്യമാരുടെ പേരുകളും പുരാണങ്ങളുടെ സാരാംശം ഉൾക്കൊള്ളുന്ന അവരുടെ സ്വഭാവ സവിശേഷതകളും നിങ്ങൾക്ക് വിവരിക്കാൻ പോകുന്നു. അവർ 27 എണ്ണമായിരുന്നു, അവരുടെ പേരുകൾ ഇപ്രകാരമാണ്: അശ്വിനി, ഭ്രാണി, കൃതിക, രോഹിണി, മൃഗ-സിരി, ആർദ്ര, പുനർവസു, പുഷ്യ, അസ്ലേസ, മാഘ, പൂർവ-ഫൽഗുനി, ഉത്തര-ഫൽഗുനി, ഹസ്ത, ചിത്ര, സ്വാതി, വിശാഖം, അനുരാധ, ജ്യേഷ്ഠ, മാഫ, പൂർവാശാദ, പൂർവാസാദ, ഉത്തരാസാദം, ധനിഷ്‌ഠ, ശ്രവണ, ശതാഭിഷ, ഉത്തരാഭിദ്രാപാദി, രേവതി…” (വിവ: രാജേന്ദ്ര നാഥ് സെൻ)

ശ്രീമദ് ഭാഗവതം 6.6.2: — അവൻ പത്തു പെൺമക്കളെ ധർമ്മരാജാവിന് [യമരാജ], പതിമൂന്ന് കശ്യപന് [ആദ്യം പന്ത്രണ്ടും പിന്നെ ഒന്ന് കൂടി], ഇരുപത്തിയേഴ് പേരെ ചന്ദ്രദേവനും, രണ്ട് വീതം അങ്കിരനും കൃഷ്ണാശ്വനും ബുധനും നൽകി. മറ്റ് നാല് പെൺമക്കളെ കശ്യപന് നൽകി. [അങ്ങനെ കശ്യപന് ആകെ പതിനേഴു പുത്രിമാരെ ലഭിച്ചു.]

ഹനുമാന്റെ പിതാവ് കേസരിക്ക് അഞ്ജന, അദ്രിക എന്നീ രണ്ട് ഭാര്യമാരുണ്ടായിരുന്നു. അഞ്ജനയിൽ നിന്നും വായുദേവനിൽ നിന്നുമാണ് ഹനുമാൻ ജനിച്ചത്.

ബ്രഹ്മപുരാണം: ഗൗതമി മാഹാത്മ്യം 14.1-4 “…ഹേ നാരദാ, അഞ്ജന പർവ്വതമുണ്ട്. ആ പർവതത്തിൽ, ഹേ ശ്രേഷ്ഠമായ മഹർഷി, അഞ്ജന എന്ന മഹത്തായ ഒരു ദേവതയുണ്ടായിരുന്നു. ഒരു ശാപത്താൽ അവൾക്ക് ഒരു വീഴ്ച സംഭവിച്ചു. അവളുടെ മുഖം ഒരു കുരങ്ങനോട് സാമ്യമുള്ളതായി മാറി. കേസരി എന്നായിരുന്നു അവളുടെ ഭർത്താവിന്റെ പേര്. കേസരിയുടെ മറ്റൊരു ഭാര്യയായിരുന്നു അദ്രിക. അവളും ഒരു ശാപത്താൽ തകർച്ച നേരിട്ട ഒരു സ്വർഗ്ഗീയ പെൺകുട്ടിയായിരുന്നു. അവളുടെ മുഖവും തലയും പൂച്ചയുടേതിനോട് സാമ്യമുള്ളതായിരുന്നു. അവളും അഞ്ജന പർവതത്തിൽ താമസിച്ചു.” (വിവ: ബോർഡ് ഓഫ് സ്കോളേഴ്സ്, എഡിറ്റർ: ജെ.എൽ. ശാസ്ത്രി)

രുദ്രന് പതിനൊന്ന് ഭാര്യമാരുണ്ടായിരുന്നു

ബ്രഹ്മ വൈവർത്ത പുരാണം, ബ്രഹ്മ ഖണ്ഡം 9.13-22 “അല്ലയോ മഹാനായ സന്യാസി, ഇപ്പോൾ ഞാൻ രുദ്രന്റെ ഭാര്യമാരുടെ പേരുകൾ വായിക്കാൻ പോകുന്നു. (1) കാല (2) കലാവതി (3) കാഷ്ട (4) കാളിക (5) കലഹപ്രിയ (6) കന്ദലി (7) ഭീഷണ (8) ബസ്ന (9) പ്രമോച (10) ഭൂഷണ (11) സുകി എന്നീ പേരുകളിൽ അവ ആഘോഷിക്കപ്പെടുന്നു. അവർ നിരവധി കുട്ടികളെ ജനിപ്പിച്ചു, എല്ലാവരും ശിവയുടെ അനുയായികളായിരുന്നു…” (വിവ: രാജേന്ദ്ര നാഥ് സെൻ)

പതിനൊന്ന് ഭാര്യമാരുള്ള രുദ്രനെ സംബന്ധിച്ച് ശ്രീമദ് ഭാഗവതം 3.12.13-ലും പരാമർശമുണ്ട്.

അഗ്നിക്ക് രണ്ട് ഭാര്യമാരുണ്ടായിരുന്നു.

ദേവീഭാഗവതം 12.10.81-100 പ്രകാരം അഗ്നിക്ക് സ്വാഹ, സ്വധ എന്നിങ്ങനെ രണ്ട് ഭാര്യമാരുണ്ടായിരുന്നു.

ഗണേശന് സിദ്ധി, ബുദ്ധി എന്നീ രണ്ട് ഭാര്യമാരുണ്ടായിരുന്നു, അവർ ഇരുവരും പ്രജാപതി വിശ്വരൂപയുടെ പുത്രിമാരായിരുന്നു.

ശിവപുരാണം, രുദ്ര സംഹിത 2.20.1-10, ഗണേശന് പ്രജാപതി വിശ്വരൂപയുടെ പുത്രിമാരായ സിദ്ധി, ബുദ്ധി എന്നിങ്ങനെ രണ്ട് ഭാര്യമാരുണ്ടായിരുന്നു, അവർ രണ്ട് ആൺമക്കളെ ജനിപ്പിച്ചു എന്ന് വ്യക്തമാക്കുന്നു.

ശിവപുരാണം, രുദ്ര സംഹിത 2.20.13: “ഗണേശന് സന്തോഷത്തോടെ രണ്ട് ഭാര്യമാരെ ലഭിച്ചു. അവർ പ്രജാപതി വിശ്വരൂപന്റെ ഉത്തമപുത്രിമാരാണ്. അവൻ തന്റെ ഐശ്വര്യമുള്ള ശരീരത്തിന് ഉടമകളായ രണ്ട് ഭാര്യമാരിൽ നിന്ന് രണ്ട് പുത്രന്മാരെ ജനിപ്പിച്ചു, സിദ്ധിയുടെ ക്ഷേമ, ബുദ്ധിയുടെ ലാഭ. അവർ എല്ലാവർക്കും സന്തോഷം നൽകുന്നു. ” (വിവ: ജെ.എൽ.ശാസ്ത്രി)

വിഷ്ണുവിന് മൂന്ന് ഭാര്യമാരുണ്ടായിരുന്നു

ബ്രഹ്മ വൈവർത്ത പുരാണം, പ്രകൃതി ഖണ്ഡ 6.13-21: “…ലക്ഷ്മി, സരസ്വതി, ഗംഗ എന്നിവരാണ് ഹരിയുടെ മൂന്ന് ഭാര്യമാർ…” (വിവ: രാജേന്ദ്ര നാഥ് സെൻ)

മറ്റൊരു പതിപ്പ് അനുസരിച്ച്, വിഷ്ണുവിന് നാല് ഭാര്യമാരുണ്ടായിരുന്നു, തുളസിയെ ബലാത്സംഗം ചെയ്തതിന് ശേഷം ഭാര്യമാരിൽ ഉൾപ്പെടുത്തപ്പെടുത്തി കൊണ്ടാണ് ഈ കണക്ക്. ബ്രഹ്മ വൈവർത്ത പുരാണം പ്രകാരം അവൾ തന്റെ ശരീരം ഉപേക്ഷിച്ച് വിഷ്ണുവിന്റെ ഭാര്യയായി എന്നാണുള്ളത്.

ബ്രഹ്മാവിന് രണ്ട് ഭാര്യമാരുണ്ടായിരുന്നു.

സ്കന്ദപുരാണം III.i.41.98-99 പ്രകാരം ബ്രഹ്മാവിന് ഗായത്രിയും സരസ്വതിയും എന്നിങ്ങനെ രണ്ട് ഭാര്യമാരുണ്ടായിരുന്നു.

യമരാജന് പത്തു ഭാര്യമാരുണ്ടായിരുന്നു.

ശ്രീമദ് ഭാഗവതം 6.6.4 പ്രകാരം യമരാജന് നൽകിയ പത്ത് പെൺമക്കളുടെ പേരുകൾ ഭാനു, ലംബ, കകുദ്, യാമി, വിശ്വം, സാധ്യ, മരുത്വതി, വാസു, മുഹൂർത്ത, സംകൽപ എന്നിങ്ങനെയാണ്. ഇപ്പോൾ അവരുടെ ആൺമക്കളുടെ പേരുകൾ കേൾക്കുക.

ഗരുഡന് ധാരാളം ഭാര്യമാരുണ്ടായിരുന്നു.

ബ്രഹ്മാണ്ഡപുരാണം 2.3.7.448-454; വായുപുരാണം 8.319 “…ഗരുഡന്റെ ഭാര്യമാർ മറ്റ് അഞ്ച് പേരായിരുന്നു.-ഭാസി, ക്രൗഞ്ചി, സുകി, ധൃതരാഷ്ട്രി, സയേനി…” (വിവ: ജി.വി. ടാഗരെ)

ബ്രഹ്മാവിന്റെ പുത്രനായ സ്വയംഭുവ മനുവിന് രണ്ട് പുത്രന്മാരുണ്ടായിരുന്നു; പ്രിയവ്രതനും ഉത്താനപാദനും. ഉത്താനപാദന് രണ്ട് ഭാര്യമാരുണ്ടായിരുന്നു; സുരുസിയും സുനിതിയും.

ലിംഗപുരാണം 62.1-5: ഉത്താനപാദന് സുനിതി, സുരുചി എന്നിങ്ങനെ രണ്ട് ഭാര്യമാരുണ്ടായിരുന്നു, ധ്രുവൻ തന്റെ മൂത്ത ഭാര്യയായ സുനിതിയിൽ ജനിച്ചു.

മനുവിന് പത്തു ഭാര്യമാരുണ്ടായിരുന്നു എന്ന് ശ്രീമദ് ഭാഗവതം 4.8.8-ലും പരാമർശിച്ചിട്ടുണ്ട്. മൈത്രായണി സംഹിത 1.5.8 പ്രകാരവും മനുവിന് പത്ത് ഭാര്യമാരുണ്ടായിരുന്നു.

യാജ്ഞവൽക്യ മുനിക്ക് രണ്ട് ഭാര്യമാരുണ്ടായിരുന്നു.

ബൃഹദാരണ്യക ഉപനിഷത്ത് 4.5.1 പ്രകാരം യാജ്ഞവൽക്യന് മൈത്രേയി, കാത്യായൻ എന്നീ രണ്ട് ഭാര്യമാരുണ്ടായിരുന്നു.

മന്ദാർകിണി മഹർഷിക്ക് അഞ്ച് ഭാര്യമാരുണ്ടായിരുന്നു.

വാല്മീകി രാമായണം അനുസരിച്ച്, (ആരണ്യകാണ്ഡം 3, അദ്ധ്യായം 11 ) മന്ദകർണി മഹർഷിക്ക് അഞ്ച് അപ്സരസുകൾ (Nymphs) ഭാര്യമാരായിരുന്നു.

പ്രജാപതി അംഗിരയ്ക്ക് രണ്ട് ഭാര്യമാരുണ്ടായിരുന്നു.

ശ്രീമദ് ഭാഗവതം 6.6.19: പ്രജാപതി അംഗിരയ്ക്ക് സ്വധ എന്നും സതി എന്നും പേരുള്ള രണ്ട് ഭാര്യമാരുണ്ടായിരുന്നു. സ്വധ എന്ന ഭാര്യ എല്ലാ പിതാക്കന്മാരെയും പുത്രന്മാരായി സ്വീകരിച്ചു, സതി അഥർവാംഗിരസ വേദത്തെ പുത്രനായി സ്വീകരിച്ചു.

സൗഭരി മുനിക്ക് അമ്പത് ഭാര്യമാരുണ്ടായിരുന്നു.

സൗഭരി മുനിക്ക് അമ്പത് ഭാര്യമാരുണ്ടെന്ന് ശ്രീമദ് ഭാഗവതം 9.6.52 പറയുന്നു.

വിശ്രവാസന് നാല് ഭാര്യമാരുണ്ടായിരുന്നു.

വായുപുരാണം 9.32-34: “വിശ്രവാസൻ എന്ന മുനി ഇടവിടയിൽ ആണ് ജനിച്ചത്. പുലസ്ത്യ കുടുംബത്തെ അഭിവൃദ്ധിപ്പെടുത്താൻ അദ്ദേഹത്തിന് നാല് ഭാര്യമാരുണ്ടായിരുന്നു. ദേവാധ്യാപകനായ ബൃഹസ്പതിക്ക് ദേവിണിനി എന്ന പ്രശസ്തയായ ഒരു മകളുണ്ടായിരുന്നു. അവൻ (വിശ്രവാസൻ) ആ പെൺകുട്ടിയെ വിവാഹം കഴിച്ചു. അവൻ (വിശ്രവാസൻ) മാല്യവന്റെ പുത്രിമാരായ പുഷ്പ്ത്കതയെയും വാകയെയും മാലിൻ്റെ മകളായ കൈകസിയെയും വിവാഹം കഴിച്ചു. അവരുടെ സന്തതികളെ കേൾക്കുക.” (വിവ: ജി.വി. ടാഗാരെ, എഡിറ്റർ: ജി.പി. ഭട്ട്)

അത്രിക്ക് പത്ത് ഭാര്യമാരുണ്ടായിരുന്നു.

വായുപുരാണം 9.64: “മൂന്നാം പ്രജാപതിയായ അത്രിയുടെ വംശപരമ്പരയെ ഞാൻ ഇപ്പോൾ വിവരിക്കാം. അദ്ദേഹത്തിന് ശുദ്ധരും സുന്ദരികളുമായ പത്ത് ഭാര്യമാരുണ്ടായിരുന്നു. (വിവ: ജി.വി. ടാഗാരെ, എഡിറ്റർ: ജി.പി. ഭട്ട്)

ഭൃഗുവിന് രണ്ട് ഭാര്യമാരുണ്ടായിരുന്നു.

ബ്രഹ്മാണ്ഡ പുരാണം 2.3.1.74-76: “ഭ്രഗുവിന്റെ രണ്ട് ഭാര്യമാരും ജന്മനാ ഉത്തമ കുലീനരായിരുന്നു. അവർ സമാനതകളില്ലാത്തവരും ഗംഭീരവുമായിരുന്നു. (അവരിൽ ഒരാൾ) ദിവ്യ എന്ന പേരിൽ അറിയപ്പെടുന്ന ഹിരണ്യകശിപുവിന്റെ മകളായിരുന്നു. രണ്ടാമത്തേത് പൗലോമി ആയിരുന്നു, പുലോമന്റെ മകൾ. (ജി.വി. ടാഗരെ)

മാരീസി മുനിക്ക് നാല് ഭാര്യമാരുണ്ടായിരുന്നു, അവർ 60,000 കുട്ടികളെ ജനിപ്പിച്ചു.

ബ്രഹ്മപുരാണം 1.195-8: “ഉപാദാനവ, ഹയശിരസിന്റെ മകളും ശർമ്മിഷ്ഠ, വ്രസപർവ്വന്റെയും പുലോമനും കലകയും, വൈശ്വാനരന്റെ രണ്ടു പുത്രിമാരും ആയിരുന്നു. അവർ മാരീസിയുടെ ഭാര്യമാരായിരുന്നു. അവർക്ക് വലിയ ശക്തി ഉണ്ടായിരുന്നു, അവർ ധാരാളം കുട്ടികളെ പ്രസവിച്ചു. ദാനവനെ സന്തോഷിപ്പിച്ച അറുപതിനായിരം പുത്രന്മാർ അവർക്കുണ്ടായിരുന്നു…” (ബോർഡ് ഓഫ് സ്കോളേഴ്സ്, എഡിറ്റർ: ജെ.എൽ. ശാസ്ത്രി)

ബ്രഹ്മാണ്ഡപുരാണം 2.3.6.26-ലും പരാമർശിച്ചിട്ടുണ്ട്.

ബ്രഹ്മാവിന്റെ മകൻ മാരീചി, മാരീചിയുടെ മകൻ കശ്യപൻ. കശ്യപ ഋഷിക്ക് പതിമൂന്ന് ഭാര്യമാരുണ്ടായിരുന്നു, അവരെല്ലാം സഹോദരിമാരായിരുന്നു, കശ്യപിന് ദിതി, അദിതി എന്നിങ്ങനെ രണ്ട് പ്രധാന ഭാര്യമാരുണ്ടായിരുന്നു.

ശ്രീമദ് ഭാഗവതം 6.6.24-26: “…ഹേ പരീക്ഷിത് രാജാവേ, ഇപ്പോൾ കശ്യപന്റെ ഭാര്യമാരുടെ പേരുകൾ എന്നിൽ നിന്ന് കേൾക്കൂ, അവരുടെ ഗർഭപാത്രങ്ങളിൽ നിന്ന് പ്രപഞ്ചത്തിലെ മുഴുവൻ ജനസംഖ്യയും ഉണ്ടായി. അവർ പ്രപഞ്ചത്തിലെ മിക്കവാറും എല്ലാ ജനസംഖ്യയുടെയും അമ്മമാരാണ്, അവരുടെ പേരുകൾ കേൾക്കുന്നത് വളരെ ശുഭകരമാണ്. അവ അദിതി, ദിതി, ദനു, കാഷ്ട, അരിഷ്ട, സുരസ, ഇല, മുനി, ക്രോധവശ, താമ്ര, സുരഭി, സാരമ, തിമി… ആകുന്നു. ”

ദേവീഭാഗവതം 4.3.21-22: വ്യാസൻ പറഞ്ഞു:– ദക്ഷ പ്രജാപതിക്ക് ദിതി, അദിതി എന്നീ രണ്ട് പെൺമക്കളുണ്ടായിരുന്നു. ഉയർന്ന പദവിയിലുള്ള ഈ രണ്ടുപേരും കശ്യപനെ വിവാഹം കഴിച്ചു; അവർ അദ്ദേഹത്തിന് പ്രിയപ്പെട്ടവരായിരുന്നു. അദിതി ദേവരാജാവായ അതിശക്തനായ ഇന്ദ്രനെ പ്രസവിച്ചു. ദിതിയും ഇന്ദ്രന്റെ അതേ ശക്തിയും പ്രതാപവുമുള്ള ഒരു പുത്രനെ ആവശ്യപ്പെട്ടു.”

ബ്രഹ്മപുരാണം 1.164-5: “ഹേ പ്രമുഖ ബ്രാഹ്മണരേ, ഇപ്പോൾ പ്രജാപതി കശ്യപന്റെ ഭാര്യമാരുടെ പേരുകൾ ശ്രദ്ധിക്കുക. അദിതി, ദിതി, ദനു, അരിസ്ത, സുരസ, ഖസ, സുരഭി, വിനത, താമ്ര, ക്രോധവാസ, ഇര, കദ്രു, മുനി എന്നിവയാണ് അവ. ഹേ ബ്രാഹ്മണരേ, അവരിൽ നിന്ന് ജനിച്ച കുട്ടികളെ അറിയുക. (വിവ: ബോർഡ് ഓഫ് സ്കോളേഴ്സ്, ജെ.എൽ. ശാസ്ത്രി എഡിറ്റുചെയ്തത്.)

ബ്രഹ്മാണ്ഡ പുരാണത്തിൽ 2.3.3.56-ലും ഇത് പരാമർശിച്ചിട്ടുണ്ട്.

മത്സ്യപുരാണം 4.53-54: “താൻ സൃഷ്ടിച്ച പെൺകുട്ടികളിൽ നിന്ന് പത്ത് പേർ ധർമ്മത്തിനും പതിമൂന്ന് കശ്യപർക്കും…” (വിവ: താലുഖ്ദാർ ഓഫ് ഔദ്, എഡിറ്റർ: ബി.ഡി. ബസു)

അദിതിയുടെ മകൻ ആദിത്യന് നാല് ഭാര്യമാരുണ്ടായിരുന്നു.

കൂർമ്മപുരാണം I.20.1-2: “അദിതി തന്റെ മകനായ ഭഗവാൻ ആദിത്യനെ (സൂര്യദേവൻ) കശ്യപനിൽ നിന്ന് പ്രസവിച്ചു. ഈ ആദിത്യന് നാല് ഭാര്യമാരുണ്ടായിരുന്നു. അവർ സംജ്ഞ, രജനി, പ്രഭ, ഛായ എന്നിവരായിരുന്നു…” (വിവ: ജി.വി. ടാഗരെ)

ആദിത്യയ്ക്ക് മൂന്ന് ഭാര്യമാരുണ്ടായിരുന്നു, നാലാമത്തെ ഭാര്യ ഛായ, സംജ്ഞയുടെ നിഴലായിരുന്നു.

അദിതിയുടെ ഏഴാമത്തെ പുത്രനും നാല് ഭാര്യമാരുണ്ടായിരുന്നു.

ശ്രീമദ് ഭാഗവതം 6.18.3-4: അദിതിയുടെ ഏഴാമത്തെ പുത്രനായ ധാതയ്ക്ക് കുഹു, സിനിവാലി, രാക, അനുമതി എന്നിങ്ങനെ നാല് ഭാര്യമാരുണ്ടായിരുന്നു. ഈ ഭാര്യമാർ യഥാക്രമം സായം, ദർശ, പ്രാതഃ, പൂർണ്ണമാസ എന്നീ നാല് പുത്രന്മാരെ ജനിപ്പിച്ചു. അദിതിയുടെ എട്ടാമത്തെ പുത്രനായ വിധാതയുടെ ഭാര്യയുടെ പേര് ക്രിയ എന്നാണ്. അവളുടെ വിധാതയിൽ പുരിഷ്യൻമാർ എന്ന് പേരുള്ള അഞ്ച് അഗ്നിദേവന്മാരെ ജനിപ്പിച്ചു. അദിതിയുടെ ഒമ്പതാമത്തെ പുത്രനായ വരുണന്റെ ഭാര്യയുടെ പേര് കർഷാണി എന്നാണ്. ബ്രഹ്മാവിന്റെ പുത്രനായ ഭൃഗു അവളുടെ ഉദരത്തിൽ വീണ്ടും ജനിച്ചു.

ക്ഷത്രിയർക്കിടയിൽ ബഹുഭാര്യത്വം കൂടുതൽ പ്രബലമായിരുന്നു, അത് ഹിന്ദു വാക്താക്കൾ തന്നെ അംഗീകരിക്കുന്നു. ഋഗ്വേദവും അതുതന്നെയാണ് കാണിക്കുന്നത്.

ഋഗ്വേദം 6.18.2: “തന്റെ ഭാര്യമാരിൽ ഒരു രാജാവിനെപ്പോലെ നീ വസിക്കുന്നു: മഹത്വങ്ങളോടെ, ഒരു മഹർഷിയെപ്പോലെ, ഞങ്ങളെ വളയുകയും സഹായിക്കുകയും ചെയ്യുക…”

ദേവീഭാഗവതം പുസ്തകം 2, അദ്ധ്യായം 7 അനുസരിച്ച്, സാധാരണ ഭാര്യയായ ദ്രൗപതിയെ കൂടാതെ, അർജുനന് കൃഷ്ണന്റെ സഹോദരിയായ സുഭദ്ര എന്ന പേരിൽ ഒരു ഭാര്യ കൂടി ഉണ്ടായിരുന്നു. കൃഷ്ണന്റെ സമ്മതപ്രകാരം അർജുനൻ അവളെ ബലപ്രയോഗത്തിലൂടെ തട്ടിക്കൊണ്ടുപോയി.

വേദപുത്രനായ ഹരിശ്ചന്ദ്ര രാജാവിന് നൂറ് ഭാര്യമാരുണ്ടായിരുന്നു.

ഐതരേയ ബ്രാഹ്മണൻ, അദ്ധ്യായം 3, ഖണ്ഡിക 13, ഇക്ഷവാകു വംശത്തിൽപ്പെട്ട വേധസിന്റെ മകൻ ഹരിശ്ചന്ദ്ര, പുത്രനില്ലാത്ത രാജാവായിരുന്നു. അദ്ദേഹത്തിന് നൂറ് ഭാര്യമാരുണ്ടായിട്ടും അവർക്ക് ഒരു പുത്രൻ ജനിച്ചില്ല.

പാണ്ഡുവിന് കുന്തിയും മാദ്രിയും എന്നീ രണ്ട് ഭാര്യമാരുണ്ടായിരുന്നു.

പത്മപുരാണം II.79.1-2 പ്രകാരം യയാതി രാജാവിന് മൂന്ന് ഭാര്യമാരുണ്ടെന്ന് പറയപ്പെടുന്നു.

രാജാക്കന്മാർക്ക് നിരവധി ഭാര്യമാരുണ്ടെന്നതിന് ഡസൻ കണക്കിന് ഉദാഹരണങ്ങളുണ്ട്, പക്ഷേ ഞാൻ അത് കുറച്ച് മാത്രമായി പരിമിതപ്പെടുത്തട്ടെ.

ബഹുഭാര്യത്വം അനുവദിക്കുന്ന ഹിന്ദു ഗ്രന്ഥങ്ങൾ

വിഖ്യാത ഹിന്ദു പണ്ഡിതനായ വിജ്ഞാനേശ്വരൻ എഴുതുന്നു:

“വർഗങ്ങളുടെ ക്രമമനുസരിച്ച്, ബ്രാഹ്മണർക്ക് മൂന്ന്, ക്ഷത്രിയർക്ക് രണ്ട്, വൈശ്യർക്ക് ഒരു ഭാര്യ എന്നാണ് വിധി. ഒരു ശൂദ്രന് ഒരേ ജാതിയിൽ ജനിച്ച ഒരു ഭാര്യ മാത്രമേ ഉണ്ടാകാവൂ. (യാജ്ഞവൽക്യ സ്മൃതിയുടെ മിതാക്ഷര വ്യാഖ്യാനത്തിൽ വിജ്ഞാനേശ്വരൻ 3: 57)

ബഹുഭാര്യത്വത്തിൽ ഒരു പ്രശ്നവുമില്ലെന്ന് സ്വാമി വിവേകാനന്ദൻ അഭിപ്രായപ്പെട്ടു:

“സ്ത്രീകളെ സംബന്ധിച്ചിടത്തോളം, അവർ പവിത്രതയെ ഏറ്റവും പ്രധാനപ്പെട്ട പുണ്യമായി കണക്കാക്കുന്നു, സംശയമില്ല. ഒരേ സമയം ഒന്നിലധികം ഭർത്താക്കന്മാരുള്ള ഒരു സ്ത്രീ എന്നതോളം ഒരു പുരുഷൻ ഒന്നിലധികം ഭാര്യമാരെ വിവാഹം ചെയ്യുന്നത് സമൂഹത്തിന് അത്ര ദോഷകരമല്ല, കാരണം ഒന്നാമത്തേത് വംശത്തിന്റെ ക്രമാനുഗതമായ അപചയത്തിലേക്ക് നയിക്കുന്നു.”

The Complete Works of Swami Vivekananda/Volume 5/Writings: Prose and Poems/The East and The West/France-Paris https://en.wikisource.org/wiki/The_Complete_Works_of_Swami_Vivekananda/Volume_5/Writings:_Prose_and_Poems/The_East_and_The_West/France-Paris

ബഹുഭാര്യത്വത്തെക്കുറിച്ചുള്ള ISKCON സ്ഥാപകൻ സ്വാമി പ്രഭുപാദ

“…നമ്മുടെ വേദപ്രക്രിയ അനുസരിച്ച് ബഹുഭാര്യത്വം അനുവദനീയമാണ്. ഉദാഹരണത്തിന്, കൃഷ്ണൻ 16,000 ഭാര്യമാരെ വിവാഹം കഴിച്ചു, അർജുനൻ 3 അല്ലെങ്കിൽ 4 ഭാര്യമാരെ വിവാഹം കഴിച്ചു, കൃഷ്ണന്റെ പിതാവ് വാസുദേവൻ, 16 അല്ലെങ്കിൽ 18 ഭാര്യമാരെ വിവാഹം കഴിച്ചു. അതിനാൽ വൈദിക സമ്പ്രദായമനുസരിച്ച് ബഹുഭാര്യത്വം നിഷിദ്ധമല്ല…” (Swami Prabhupada, Letter to Karandhara written from Bombay http://vanisource.org/wiki/730109_-_Letter_to_Karandhara_written_from_Bombay)

“ഒരു വശത്ത് ബഹുഭാര്യത്വത്തെ എതിർക്കുകയും മറുവശത്ത് അവർ സ്ത്രീകളെ പല തരത്തിൽ വേട്ടയാടുകയും ചെയ്യുന്ന തരത്തിൽ ഈ കാലഘട്ടത്തിൽ ആളുകൾ വളരെ അധഃപതിച്ചിരിക്കുന്നു. ഈ ക്ലബ്ബിലോ ആ കടയിലോ ടോപ്‌ലെസ്സ് പെൺകുട്ടികൾ ലഭ്യമാണെന്ന് പല ബിസിനസ്സ് സംഘങ്ങളും പരസ്യമായി പരസ്യം ചെയ്യുന്നു. അങ്ങനെ സ്ത്രീകൾ ആധുനിക സമൂഹത്തിൽ ഇന്ദ്രിയ ആസ്വാദനത്തിന്റെ ഉപകരണങ്ങളായി മാറിയിരിക്കുന്നു. ബ്രാഹ്മണർ, ക്ഷത്രിയർ, വൈശ്യർ, എന്തിനേറെ ശൂദ്രർ പോലും ഒരു ഉയർന്ന സാമൂഹിക ക്രമത്തിലുള്ള പുരുഷന്മാർ എന്ന നിലയിൽ, ഒന്നിലധികം ഭാര്യമാരെ ആസ്വദിക്കാനുള്ള പ്രവണത പുരുഷനുണ്ടാകാൻ ഇടയുള്ളത് പോലെ ഉണ്ടായാൽ, കൂടുതൽ വിവാഹം കഴിക്കാൻ വേദങ്ങൾ അനുശാസിക്കുന്നു. ഒന്നിലധികം ഭാര്യമാരെ അവന് വിവാഹം ചെയ്യാൻ അനുവദിച്ചു. വിവാഹം എന്നാൽ ഒരു സ്ത്രീയുടെ പൂർണ്ണമായ ഉത്തരവാദിത്തം ഏറ്റെടുക്കുകയും വ്യഭിചാരത്തെ സമീപിക്കാതെ സമാധാനത്തോടെ ജീവിക്കുകയും ചെയ്യുക എന്നതാണ്. ഇന്ന്, വ്യഭിചാരം അനിയന്ത്രിതമായി നടക്കുന്നു. അതേസമയം, ഒന്നിലധികം ഭാര്യമാരെ വിവാഹം ചെയ്യാൻ പാടില്ലെന്ന നിയമം സമൂഹം ഉണ്ടാക്കുന്നു. ഇത് ഒരു പൈശാചിക സമൂഹത്തിന്റെ സവിശേഷതയാണ്. ”

(Swami Prabhupada on Srimad Bhagavatam 4.26.6 http://vanisource.org/wiki/SB_4.26.6)

അദ്ദേഹം എഴുതുന്നു,

“ഭാര്യ ഗർഭിണിയായിരിക്കുമ്പോൾ ലൈംഗികത ആസ്വദിക്കാൻ കഴിയാത്തതിനാൽ ഒരു പുരുഷന് ഒന്നിലധികം ഭാര്യമാരെ സൂക്ഷിക്കാൻ അനുവാദമുണ്ട്. അത്തരമൊരു സമയത്ത് ലൈംഗികത ആസ്വദിക്കണമെങ്കിൽ അയാൾ ഗർഭിണിയല്ലാത്ത മറ്റൊരു ഭാര്യയുടെ അടുത്തേക്ക് പോയേക്കാം. മനു-സംഹിതയിലും മറ്റു ഗ്രന്ഥങ്ങളിലും പറഞ്ഞിരിക്കുന്ന നിയമങ്ങളാണിവ.”

(Swami Prabhupada on Srimad Bhagavatam 4.27.5 http://vanisource.org/wiki/SB_4.27.5)

ഹിന്ദുമതത്തിലെ ഏറ്റവും പൗരാണിക വേദഗ്രന്ഥമായ വേദങ്ങൾ ബഹുഭാര്യത്വത്തെ അനുവദിക്കുകയോ നിരോധിക്കുകയോ ചെയ്യുന്നില്ല. എന്നാൽ ബഹുഭാര്യത്വത്തിന്റെ അടയാളങ്ങൾ വേദങ്ങളിൽ കാണാം, അത് ബഹുഭാര്യത്വം വേദ കാലഘട്ടത്തിൽ പ്രയോഗിക തലത്തിൽ ഉണ്ടായിരുന്നുവെന്ന് സൂചിപ്പിക്കുന്നു, ഞാൻ ശ്രീരാം ശർമ്മയുടെ ഹിന്ദി വിവർത്തനമാണ് ഉപയോഗിക്കുന്നത്.

ഋഗ്വേദം 5.42.12 ശ്ലോകത്തിൽ ഇന്ദ്രന് മൂന്ന് ഭാര്യമാരുണ്ടെന്ന് പറയുന്നു.

മൈത്രായണി സംഹിത 3.11.1 [140, 10-11] ഇന്ദ്രന്റെ ഭാര്യമാരായ സരസ്വതിയെയും, ഇഡയെയും, ഭാരതിയെയും സംബന്ധിച്ച് പരാമർശിക്കപ്പെടുന്നു. (വിവ. കാതറിൻ ലുഡ്വിക്)

വേദങ്ങൾ, ബഹുഭർതൃത്വത്തെയും അംഗീകരിക്കുന്നതായി തോന്നുന്നു, ഋഗ്വേദം 6.49.7 സരസ്വതിയെ ഇന്ദ്രന്റെ ഭാര്യയായി കണക്കാക്കുന്നു, ഇത് വൈരുദ്ധ്യമല്ലെങ്കിൽ മുകളിലുള്ള വാക്യങ്ങൾ (ബ്രഹ്‌മാവിന്റെ ഭാര്യയാണ് സരസ്വതി എന്ന് വാദിക്കുന്ന വാക്യങ്ങൾ) വൈരുദ്ധ്യമാണെന്ന് പറയേണ്ടിവരും. യജുർവേദം 19.94-ൽ സരസ്വതിയെ അശ്വിനി സഹോദരന്മാരുടെ ഭാര്യ എന്നും വിളിക്കുന്നു, ഇത് വ്യക്തമായും ബഹുഭർതൃത്വത്തിന്റെ കേസാണ്.

ഋഗ്വേദം 10.43.1. “സ്വർഗ്ഗത്തിന്റെ വെളിച്ചം കണ്ടെത്തുന്ന, എന്റെ എല്ലാ സ്തുതികളും തികഞ്ഞ ഐക്യത്തോടെ ഇന്ദ്രനെ സ്തുതിച്ചു. ഭാര്യമാർ തങ്ങളുടെ യജമാനനായ സുന്ദരനായ വരനെ ആലിംഗനം ചെയ്യുന്നതുപോലെ, അവർ മഘവാനെ സഹായത്തിനു വേണ്ടി ആലിംഗനം ചെയ്യുന്നു.”

https://vedkabhed.files.wordpress.com/2016/05/rig-veda-10-43-1.png?w=517&h=93

ബഹുഭാര്യത്വത്തെ സ്തുതിക്കുന്ന മറ്റൊരു വാക്യത്തിന്റെ വ്യക്തമായ പരിഭാഷ:

ഋഗ്വേദം 4.58.8 “ഗിയുടെ പ്രവാഹങ്ങൾ, പുഞ്ചിരിക്കുന്ന, അർപ്പണബോധമുള്ള ഭാര്യമാരായി അഗ്നിയിലേക്ക് ചായുന്നു: അവർ ഇന്ധനം പോലെ (ജ്വാലയെ) പോഷിപ്പിക്കുന്നു, കൂടാതെ ജാതവേദങ്ങൾ അവരെ പ്രീതിപ്പെടുത്തി സ്വീകരിക്കുന്നു.” (വിവ: എച്ച്.എച്ച്. വിൽസൺ)

കൃഷ്ണ യജുർവേദം 6.5.1.4 … “അതിനാൽ ഒരാൾ പോകുമ്പോൾ പലരും പിന്തുടരുന്നു; അതിനാൽ ഒരുവൻ പലരിലും ശ്രേഷ്ഠനാകുന്നു; അതിനാൽ ഒരാൾ ധാരാളം ഭാര്യമാരെ നേടുന്നു …”

ഒരു പഴയ ഋഷിയായ ച്യവന താൻ പുനരുജ്ജീവിപ്പിച്ചതിന് ശേഷം നിരവധി പെൺകുട്ടികളെ വിവാഹം കഴിച്ചിരുന്നു എന്നാണ് ഋഗ്വേദത്തിലെ ഒരു വാക്യം വ്യക്തമായി കാണിക്കുന്നത്.

ഋഗ്വേദം 1.116.10 “ഹേ നാസത്യരേ, പഴയ ച്യവനത്തിൽ നിന്ന്, ശരീരത്തിലെ തൊലി ഉരിഞ്ഞുകളഞ്ഞു, എല്ലാവരും അവനെ നിസ്സഹായനാക്കിയപ്പോൾ അവന്റെ ആയുസ്സ് നീട്ടപ്പെട്ടു, ദാസരേ! അവനെ യൗവനക്കാരിയായ കന്യകമാരുടെ അധിപതിയാക്കി.”

https://vedkabhed.files.wordpress.com/2016/05/rig-veda-1-116-10.png?w=554&h=120

മുകളിൽ ശ്രീരാം ശർമ്മയുടെ ഹിന്ദി വേദവിവർത്തനവും തുടർന്നുള്ള വിവർത്തനം പണ്ഡിറ്റ് രാം ഗോവിന്ദ് ത്രിവേദിയുടേതുമാണ്.

https://vedkabhed.files.wordpress.com/2016/05/rig-veda-1-116-10-govind-trivedi.png?w=501&h=122

നിരുക്തം 4.19, മഹാഭാരതം ആദിപർവ്വം 1.177, പഞ്ചവിംസ ബ്രാഹ്മണം 14.6.10 എന്നിവയിലും ച്യവന പുനരുജ്ജീവിപ്പിക്കപ്പെട്ടതായി പരാമർശിക്കപ്പെടുന്നു, എന്നാൽ അത് അവന്റെ വിവാഹത്തെക്കുറിച്ച് ഒന്നും പറയുന്നില്ല. ഇവിടെ കന്യകകൾ എന്നത് നിരവധി കന്യകമാരെയോ രണ്ട് കന്യകമാരെയോ സൂചിപ്പിക്കാം. ഹിന്ദു ഗ്രന്ഥങ്ങളിൽ അദ്ദേഹത്തിന്റെ രണ്ട് ഭാര്യമാരെ മാത്രമേ പരാമർശിക്കുന്നുള്ളൂ. ച്യവനന്റെ ഭാര്യമാരെക്കുറിച്ച് തിരുവെഴുത്തുകൾ കൂടുതൽ വിശദാംശങ്ങൾ നൽകുന്നില്ല, കൂടാതെ സരയാതി രാജാവിന്റെ മകളായ സുകന്യയെ കൂടുതൽ ഊന്നിപ്പറയുന്നു. ശ്രീമദ് ഭാഗവതം 9.3-ൽ പരാമർശിച്ചിരിക്കുന്നതുപോലെ സുകന്യ ച്യവനയുടെ ഭാര്യയായി, മറ്റ് ഗ്രന്ഥങ്ങൾ മനുവിന്റെ മകളായ ആരുഷി എന്ന മറ്റൊരു ഭാര്യയെക്കുറിച്ച് സംസാരിക്കുന്നു, അദ്ദേഹത്തിന് ഔർവ എന്നൊരു മകനെ പ്രസവിച്ചു, ഇത് മഹാഭാരതം ആദിപർവ്വം 1.66.47 ൽ പരാമർശിച്ചിരിക്കുന്നുണ്ട്.

മറ്റ് ഹിന്ദു ഗ്രന്ഥങ്ങൾ ബഹുഭാര്യത്വത്തെക്കുറിച്ച് വ്യക്തമായ നിലപാടാണ് സ്വീകരിക്കുന്നത്. അവ ഒരു സംശയവുമില്ലാതെ ബഹുഭാര്യത്വം അനുവദിക്കുന്നു.

മനു സ്മൃതി 3.12-13: പുരുഷന്മാരോട് തുല്യ ജാതിയിൽ നിന്നുള്ള ആദ്യ വിവാഹത്തിന് ശുപാർശ ചെയ്യുന്നു; എന്നാൽ (വീണ്ടും വിവാഹം കഴിക്കാൻ) ആഗ്രഹിക്കുന്നവർ താഴെപ്പറയുന്ന (ജാതികളിൽ പെട്ട) സ്ത്രീകളെ ക്രമപ്രകാരം വിവാഹം കഴിക്കുന്നത് അംഗീകരിക്കുന്നു. ഒരു ശൂദ്ര സ്ത്രീക്ക് ഒറ്റയ്ക്ക് ശൂദ്രന്റെ ഭാര്യയാകാം, അവളെയും വൈശ്യന്റെ സ്വന്തം ജാതിയിൽപ്പെട്ട ഒരാളെയും വൈശ്യന് വിവാഹം ചെയ്യാം, ആ രണ്ടുപേരും ഒപ്പം തന്റെ സ്വന്തം ജാതിയിൽപ്പെട്ട ഒരാളെയും ക്ഷത്രിയന് വിവാഹം ചെയ്യാം. ഈ മൂന്ന് പേരേയും, സ്വന്തം ജാതിയിൽ പെട്ട ഒരാളെയും ബ്രാഹ്മണന് വിവാഹം ചെയ്യാം.

അഗ്നിപുരാണം 285:63-67 “…നിരവധി ഭാര്യമാരുള്ള ഒരു ഭർത്താവ്, എല്ലാ ദിവസവും, ത്രിഫല, പിപ്പലി, തേൻ, വെണ്ണ, പൊടിച്ച അമലാക്കി, അതേ പഴത്തിന്റെ നീര് എന്നിവ ചേർത്ത ഒരു സിറപ്പ് നാവ് കൊണ്ട് നക്കി കഴിക്കണം. ശേഷം വെള്ളം കുടിക്കൂ…” (വിവ: എം.എൻ. ദത്ത്)

മനു സ്മൃതി 9.85: “ദ്വിജാതിയനായി ജനിച്ച പുരുഷന്മാർ അവരുടേതും മറ്റേ (താഴ്ന്ന ജാതിയിലുള്ള) സ്ത്രീകളെയും വിവാഹം കഴിച്ചാൽ, അവരുടെ (ഭാര്യമാരുടെ) സീനിയോറിറ്റി, ബഹുമാനം, വാസസ്ഥലം എന്നിവ ജാതികളുടെ (വർണ്ണ) ക്രമം അനുസരിച്ചായിരിക്കണം നിശ്ചയിക്കേണ്ടത്.”

മഹാഭാരതം 1.160.36: “ഇതിൽ പാപമില്ല. പുരുഷനെ സംബന്ധിച്ചിടത്തോളം ബഹുഭാര്യത്വം ഒരു പുണ്യ പ്രവൃത്തിയാണ്, എന്നാൽ ഒരു സ്ത്രീയെ സംബന്ധിച്ചിടത്തോളം ആദ്യ ഭർത്താവിന് ശേഷം രണ്ടാമത്തെ ഭർത്താവിനെ സ്വീകരിക്കുന്നത് വളരെ പാപമാണ്. (വിവ. കെ.എം. ഗാംഗുലി)

മഹാഭാരതം 14.80.12-18: “ഓ, വിജയാ, ഗുഡകേശൻ എന്ന് വിളിക്കപ്പെടുന്നവൻ, ചുവന്ന കണ്ണുകളുള്ള ഈ വീരൻ മടങ്ങിവരട്ടെ, ജീവിതമേ. ഹേ ഭാഗ്യവതി, ബഹുഭാര്യത്വം പുരുഷന്മാരുടെ കുറ്റമല്ല. ഒന്നിലധികം ഭർത്താക്കന്മാരെ സ്വീകരിക്കുന്നതിലൂടെ മാത്രമേ സ്ത്രീകൾക്ക് തെറ്റ് സംഭവിക്കൂ.” (വിവ: കെ.എം. ഗാംഗുലി)

മഹാഭാരതം 1.197.27-28: “ദ്രുപദൻ മറുപടി പറഞ്ഞു, ‘ഓ കുരുവിന്റെ വംശത്തിലെ സന്തതി, ഒരു പുരുഷന് അനേകം ഭാര്യമാരുണ്ടാകാമെന്ന് നിർദ്ദേശിക്കപ്പെട്ടിരിക്കുന്നു. എന്നാൽ ഒരു സ്‌ത്രീക്ക്‌ അനേകം ഭർത്താക്കന്മാരുണ്ടാകുമെന്ന്‌ ഇതുവരെ കേട്ടിട്ടില്ല!” (വിവ: കെ.എം. ഗാംഗുലി)

ഭർത്താവിന് നിരവധി ഭാര്യമാരുണ്ടെങ്കിൽ ഏത് ഭാര്യയാണ് അവനോടൊപ്പം മതപരമായ കർത്തവ്യങ്ങൾ ചെയ്യേണ്ടതെന്ന് ഹിന്ദു ഗ്രന്ഥങ്ങളും നിർദ്ദേശങ്ങൾ നൽകുന്നുണ്ട്.

വിഷ്ണു സ്മൃതി 26.1-4: “ഒരു പുരുഷന് സ്വന്തം ജാതിയിൽ പെട്ട നിരവധി ഭാര്യമാരുണ്ടെങ്കിൽ, അവൻ മൂത്ത (അല്ലെങ്കിൽ ആദ്യ വിവാഹിത) ഭാര്യയോടൊപ്പം തന്റെ മതപരമായ കർത്തവ്യങ്ങൾ നിർവഹിക്കണം. (അവന് നിരവധി) വിവിധ ജാതികളിൽപ്പെട്ട ഭാര്യമാർ ഉണ്ടെങ്കിൽ സ്വജാതിയിൽ പെട്ടവളുമായി അവൻ അത് അനുഷ്ഠിക്കണം; അവൾ ഇളയഭാര്യ ആണെങ്കിൽ പോലും. സ്വന്തം ജാതിയിലുള്ള ഒരു ഭാര്യ അനുഷ്ഠാനത്തിൽ പരാജയപ്പെട്ടാൽ (അവൻ അവ നിർവഹിക്കേണ്ടത്) തന്റെ സ്വന്തം ജാതിയുടെ തൊട്ട് താഴെയുള്ള ജാതിയിൽപ്പെട്ടവളുമായിട്ടാണ്…

കാത്യായന സംഹിത 8.6: “ഒരേ ജാതിയിൽപ്പെട്ടവരും നിലവിലുള്ള മറ്റ് ജാതികളിൽപ്പെട്ടവരുമായ അനേകം ഭാര്യമാർ ഉണ്ടാകുന്ന സന്ദർഭത്തിൽ, അഗ്നിയെ ഉൽപ്പാദിപ്പിക്കുന്നതിനുള്ള ചടങ്ങ്, ജന്മത്തിന്റെ ശ്രേഷ്ഠത കണക്കിലെടുത്ത്, ഒരേ ജാതിയിലുള്ള ശുദ്ധിയുള്ള ഭാര്യമാരാൽ ചെയ്യണം.” (വിവ: മന്മഥ നാഥ് ദത്ത്)

നാല് ഭാര്യമാർ:

ഹിന്ദു ഗ്രന്ഥങ്ങളിലെ ബഹുഭാര്യത്വത്തെ സംബന്ധിച്ച ഏക ആശയക്കുഴപ്പം ചില ഹിന്ദു ഗ്രന്ഥങ്ങൾ ഒരു ബ്രാഹ്മണന് നാല് സ്ത്രീകളെ വിവാഹം കഴിക്കാൻ അനുവാദം നൽകുന്നു, ചിലത് മൂന്ന് സ്ത്രീകളെ മാത്രമേ അനുവദിക്കൂ എന്നതാണ്. ഒരു ബ്രാഹ്മണന് നാല് ഭാര്യമാരും, ഒരു ക്ഷത്രിയന് മൂന്ന് ഭാര്യമാരും, ഒരു വൈശ്യന് രണ്ട്, ശൂദ്രന് ഒന്ന് മാത്രം അനുവദനീയമാണ് എന്നിങ്ങനെ നിർദ്ദേശിക്കുന്ന ശ്ലോകങ്ങൾ നിരവധിയാണ്.”

മഹാഭാരതം 13.47.4:

“അല്ലയോ മുത്തശ്ശി, ഒരു ബ്രാഹ്മണന് നാല് ഭാര്യമാരെ എടുക്കാം. അതായത്, സ്വന്തം ക്രമത്തിൽ പെട്ടവളെ, ഒരു ക്ഷത്രിയയെ, ഒരു വൈശ്യയെ, ഒരു ശൂദ്ര സ്ത്രീയെ എന്നിങ്ങനെയുള്ള നാല് ഭാര്യമാരെ സ്വീകരിക്കാം എന്ന് പറഞ്ഞിരിക്കുന്നു; ബ്രാഹ്മണന് ലൈംഗികബന്ധത്തിൽ ഏർപ്പെടാൻ ആഗ്രഹമുണ്ടെങ്കിൽ.” (വിവ: കെ.എം. ഗാംഗുലി)

മഹാഭാരതം 13.48.4 : “ബ്രാഹ്മണന് നാല് ജാതികളിൽ നിന്ന് ഓരോരുത്തർ വീതം നാല് ഭാര്യമാരെ സ്വീകരിക്കാം. അവയിൽ രണ്ടിൽ (അതായത്, ഭാര്യ തന്റെ സ്വന്തം ജാതിയിൽ നിന്ന് എടുത്തതും താഴെയുള്ളതിൽ നിന്ന് എടുത്തതും), അവൻ തന്നെ ജനിക്കുന്നു (അവരിൽ ജനിക്കുന്ന കുട്ടികൾ തന്റേതിന് സമാനമായ പദവിയിൽ നിക്ഷേപിക്കപ്പെട്ടവരായി കണക്കാക്കപ്പെടുന്നു)… ഒരു ക്ഷത്രിയൻ മൂന്ന് ഭാര്യമാരെ സ്വീകരിക്കാം… വൈശ്യന് രണ്ട് ഇണകളെ സ്വീകരിക്കാം…ശൂദ്രന് ഒരു ഭാര്യയെ മാത്രമേ സ്വീകരിക്കാൻ കഴിയൂ, അതായത്, സ്വന്തം ജാതിയിൽ നിന്ന് സ്വീകരിക്കപ്പെട്ടവൾ. അവളിൽ അവനാൽ ജനിച്ച മകൻ ശൂദ്രനാകുന്നു…” (വിവ: കെ.എം. ഗാംഗുലി)

അഗ്നിപുരാണം 153.1: “പുഷ്കരൻ പറഞ്ഞു:- ഒരു ബ്രാഹ്മണന് നാല് ഭാര്യമാരെയും, ഒരു ക്ഷത്രിയൻ മൂന്നും, ഒരു വൈശ്യന് രണ്ട് ഭാര്യമാരെയും, ശൂദ്ര ജാതിയിൽപ്പെട്ട ഒരാൾക്ക് ഒരു ഭാര്യയിൽ കൂടുതലും പാടില്ല.” (വിവ: എം.എൻ. ദത്ത്)

മനു സ്മൃതി 9.149: “ജാതികളുടെ നേരിട്ടുള്ള ക്രമത്തിൽ ഒരു ബ്രാഹ്മണന്റെ നാല് ഭാര്യമാരുണ്ടെങ്കിൽ, അവരിൽ ജനിച്ച പുത്രന്മാർക്കിടയിൽ (ഭൂമി) വിഭജിക്കാനുള്ള നിയമം ഇപ്രകാരമാണ്:… ”

വിഷ്ണു സ്മൃതി 24.1-5: “ഇപ്പോൾ ഒരു ബ്രാഹ്മണന് (നാല്) ജാതികളുടെ നേരിട്ടുള്ള ക്രമത്തിൽ നാല് ഭാര്യമാരെ എടുക്കാം; ഒരു ക്ഷത്രിയൻ, മൂന്ന്; ഒരു വൈശ്യന് രണ്ട്; ഒരു ശൂദ്രൻ, ഒരാൾ മാത്രം.”

ബൗധ്യാന ധർമ്മ ശാസ്ത്രം, പ്രശ്നം I, അധ്യായ 8, കണ്ടിക 16, വാക്യങ്ങൾ 1-5: ബ്രാഹ്മണർ, ക്ഷത്രിയർ, വൈശ്യർ, ശൂദ്രർ എന്നിങ്ങനെ നാല് ജാതികളുണ്ട്. (പുരുഷന്മാർ) ജാതികളുടെ ക്രമമനുസരിച്ച് ഭാര്യമാരെ സ്വീകരിക്കാം, (അതായത്) ഒരു ബ്രാഹ്മണൻ നാല്, ഒരു ക്ഷത്രിയൻ മൂന്ന്, ഒരു വൈശ്യൻ രണ്ട്, ഒരു ശൂദ്രൻ ഒന്ന്.

മൂന്ന് ഭാര്യമാർ:

പരസ്കര ഗൃഹ്യസൂത്രം I കാണ്ഡ, 4 കാണ്ഡിക, 8-11:

ബ്രാഹ്മണന് മൂന്ന് (ഭാര്യമാർ അനുവദനീയമാണ്), ജാതികളുടെ ക്രമം അനുസരിച്ച്, രാഗണ്യന് രണ്ട്, വൈശ്യന് ഒരാൾ…

യാജ്ഞവൽക്യ സ്മൃതി 3.57:

“ജാതിയുടെ ക്രമമനുസരിച്ച്, മൂന്ന്, രണ്ട്, ഒന്ന് യഥാക്രമം ബ്രാഹ്മണനും ക്ഷത്രിയനും വൈശ്യനും (ഭാര്യമാരായിരിക്കാം). ശൂദ്രനായി ജനിച്ച ഒരാൾക്ക് സ്വന്തം ജാതിയിൽപ്പെട്ട പെൺകുട്ടിയാണ് ഭാര്യ.” ശ്രീസ ചന്ദ്ര വാസു

ഇതേ കാര്യം തന്നെ ശംഖ സംഹിത 4.7,വസിഷ്ഠ സംഹിത 1.24, മഹാഭാരതം 13.44.11-12 എന്നിവയിലും ആവർത്തിക്കുന്നു…”

(https://vedkabhed.com/index.php/2018/05/05/polygamy-in-hindu-dharma/)

(ഹിന്ദു ധർമ്മശാസ്ത്രത്തിലെ ബഹുഭാര്യത്വത്തെ സംബന്ധിച്ച് ഇത്രയും വിവരിച്ചത് സുലൈമാൻ റസ്‌വിയുടെ polygamy in Hindu Dharma എന്ന ലേഖനത്തിൽ നിന്നാണ്. കൂടുതൽ വിശദാംശങ്ങളും പരിശോധനകളും പ്രസ്തുത ലേഖനത്തിൽ നിന്ന് നേരിട്ട് കാണുക.)

ഇതൊക്കെ ന്യായീകരിച്ചും വെളുപ്പിച്ചും ക്ഷീണിതരായി, രണ്ട് ദിവസം നന്നായി ഉറങ്ങി, എഴുന്നേറ്റിട്ട് പോരെ മുഹമ്മദ് നബിയുടെ(സ) പരിശുദ്ധവും മാതൃകാപരവുമായി ദാമ്പത്യങ്ങളെ കാമ പ്രേരിതമായ രതിവേട്ടകളായി വക്രീകരിക്കാൻ ?!

മുഹമ്മദ് നബിയെ (സ) സംബന്ധിച്ചിടത്തോളം ഒരു സ്ത്രീയേയും അദ്ദേഹം ബലം പ്രയോഗിച്ച് വിവാഹം ചെയ്തിട്ടില്ല. വിവാഹം ചെയ്തവരോട് തന്നെ വേണമെങ്കിൽ മാത്രം തന്റെ കൂടെ ജീവിച്ചാൽ മതിയെന്ന് പരസ്യമായി പ്രഖ്യാപിച്ചിട്ടുമുണ്ട്:

“നബിയേ, നിന്‍റെ ഭാര്യമാരോട് നീ പറയുക: ഐഹികജീവിതവും അതിന്‍റെ അലങ്കാരവുമാണ് നിങ്ങള്‍ ഉദ്ദേശിക്കുന്നതെങ്കില്‍ നിങ്ങള്‍ വരൂ! നിങ്ങള്‍ക്ക് ഞാന്‍ ജീവിതവിഭവം നല്‍കുകയും, ഭംഗിയായ നിലയില്‍ ഞാന്‍ നിങ്ങളെ മോചിപ്പിച്ച് അയച്ചുതരികയും ചെയ്യാം.” (ക്വുർആൻ:33: 28)

വലിച്ചിഴച്ച് പുറത്തിടുമെന്നല്ല പ്രഖ്യാപനം, “ഭംഗിയായ നിലയില്‍ ഞാന്‍ നിങ്ങളെ മോചിപ്പിച്ച് അയച്ചുതരികയും ചെയ്യാം” എന്നാണ്. തന്നിൽ നിന്ന് ശരണം തേടിയ ഉമൈമ എന്ന ഭാര്യയെ സമ്മാനങ്ങൾ നൽകി ആദരിച്ച് വീട്ടിലേക്ക് തിരിച്ചു പൊയ്കൊള്ളാൻ പറഞ്ഞ മുഹമ്മദ് നബിയുടെ(സ) മാതൃക ഈ ക്വുർആൻ വാക്യത്തിന്റെ ജീവിത വ്യാഖ്യാനമായിരുന്നു. (സ്വഹീഹുൽ ബുഖാരി: 5254, 5255)

أيها الرجل إن كان إنما بك الحاجة جمعنا لك حتى تكون أغنى قريش رجلا، وإن كان إنما بك الباه فاختر أي نساء قريش شئت فلنزوجك عشرا. ” മുഹമ്മദെ, നിനക്ക് ധനമാണ് ആവശ്യമെങ്കിൽ ഞങ്ങൾ ധനം ഒരുമിച്ചു കൂട്ടി നിന്നെ ക്വുറൈശികളിലെ ഏറ്റവും പണക്കാരനാക്കി ഞങ്ങൾ മാറ്റാം. നിനക്ക് രതിയാണാവശ്യമെങ്കിൽ ക്വുറൈശികളിലെ നീ തിരഞ്ഞെടുക്കുന്ന പത്തു സ്ത്രീകളെ ഞങ്ങൾ നിനക്ക് വിവാഹം കഴിപ്പിച്ചു തരാം..” (അൽ ബിദായ വന്നിഹായ: ഇബ്നുകസീർ: 3: 80) എന്ന ക്വുറൈശികളുടെ വാഗ്ദാനം യാതൊരു സങ്കോചവുമില്ലാതെ നിമിഷാർദ്ധം കൊണ്ട് തിരസ്കരിച്ച വ്യക്തിയാണ് മുഹമ്മദ് നബി (സ).

സ്വഹീഹായ ഹദീസുകളുടെ വെളിച്ചത്തിൽ പരിശോധിച്ചാൽ പതിനൊന്ന് ഭാര്യമാരായിരുന്നു നബിക്ക്(സ) ഉണ്ടായിരുന്നത് എന്ന് വ്യക്തമാവുന്നതാണ്. 1. ഖദീജ, 2. സൗദ, 3. ആഇശ, 4. ഹഫ്‌സ, 5. സൈനബ് ബിൻത് ഖുസൈമ, 6. ഉമ്മു സലമ, 7. ഉമ്മു ഹബീബ, 8. ജുവൈരിയ, 9. മൈമൂന, 10. സ്വഫിയ്യ, 11. സൈനബ് ബിൻത് ജഹ്ശ്… എന്നിവരാണവർ. റെെഹാന ബിൻത് സൈദിന്റെ വിഷയത്തിൽ അഭിപ്രായ വ്യത്യാസമുണ്ട്. (സാദുൽ മആദ്: 1: 79)

عن ابن عباس، قال: «توفي رسول الله صلى الله عليه وسلم وعنده تسع نسوة (നബി (സ) ജീവിത കാലത്തു തന്നെ മരണമടഞ്ഞ ഖദീജ, സൈനബ് ബിൻത് ഖുസൈമ എന്നീ രണ്ട് ഭാര്യമാരെ കൂട്ടാതെ) നബി (സ) മരണപ്പെടുമ്പോൾ അദ്ദേഹത്തിന് ഒമ്പത് ഭാര്യമാരെ ഉണ്ടായിരുന്നുള്ളു എന്ന് പ്രവാചക ശിഷ്യൻ ഇബ്നു അബ്ബാസ് (റ) വ്യക്തമാക്കുന്ന സ്വഹീഹായ ഹദീസ് നിലവിലുണ്ട്. (സുനനുന്നസാഈ: 3197)

വിമർശകർ ഉദ്ധരിക്കുന്ന ഈ അറുപതോളം വരുന്ന വിവാഹങ്ങളുടെ കെട്ടുകഥകൾ ഇസ്‌ലാമിക പ്രമാണങ്ങളായ ക്വുർആനിൽ നിന്നൊ സ്വഹീഹായ ഹദീസുകളിൽ നിന്നൊ തെളിയിക്കാൻ സാധിക്കുമൊ? ഒരിക്കലുമില്ല. എല്ലാം എടുക്കപ്പെട്ടിരിക്കുന്നത് ചരിത്ര ഗ്രന്ഥങ്ങളിൽ നിന്നാണ്; അതിൽ ഭൂരിഭാഗത്തിലും തങ്ങളുടേതായ (ദുർ) വ്യാഖ്യാനങ്ങളും അഭിപ്രായങ്ങളും സാങ്കൽപ്പിക അശ്ലീലങ്ങളും വിമർശകരുടെ വകയായി തിരുകി കയറ്റുകയും ചെയ്തിട്ടുണ്ട്. വിമർശകർ ദുർബല നിവേദനങ്ങൾ ചികയാൻ നൽകുന്ന ഈ ചരിത്ര ഗ്രന്ഥങ്ങളൊ സീറയൊ ഇസ്‌ലാമിക പ്രമാണങ്ങളല്ല. നബിയുടെ(സ) സമകാലികരായ സ്വഹാബികൾ നിവേദനം ചെയ്ത ഹദീസുകൾ നിലനിൽകെ പ്രവാചക വിയോഗത്തിന് കാലങ്ങൾക്ക് ശേഷം പല സ്രോതസ്സുകളിൽ നിന്നും ആധികാരികത പരിശോധിക്കാതെ വാരിക്കൂട്ടിയ കഥകളുടെ സമാഹാരങ്ങളായ സീറകളിൽ വിമർശകരുടെ കണ്ണു പതിയാൻ കാരണം മറ്റൊന്നുമല്ല. തങ്ങളെ പോലെ വ്യാജ വാർത്തകളും അർദ്ധ സത്യങ്ങളും പ്രചരിപ്പിച്ചു നടക്കുന്ന ദുർബല നിവേദകരുടെ ഒരു ശൃംഖല തന്നെ സീറാ ഗ്രന്ഥങ്ങളിൽ സ്ഥാനം പിടിച്ചിട്ടുണ്ട് എന്ന് മുമ്പ് വായിച്ച ‘ദുർബല ഹദീസുകളും കള്ളകഥകളും’ ഭാഗങ്ങളിൽ വ്യക്തമാക്കപ്പെട്ടിട്ടുണ്ട്. മുഹമ്മദ് നബിയുമായി(സ) ബന്ധപ്പെടുത്തി ഈ വ്യാജ വിവാഹ കഥകൾ പ്രചരിപ്പിക്കുന്നതിൽ ഈ ദുർബലരും, വ്യാജഹദീസ് നിർമ്മാതാക്കളുമായ നിവേദകർക്ക് പല ലക്ഷ്യങ്ങളുമാണ് ഉണ്ടായിരുന്നത്. മുഹമ്മദ് നബിയുടെ (സ) വ്യക്തിത്വത്തെ വികൃതമായി ചിത്രീകരിക്കുക എന്നതായിരിക്കാം പ്രധാന ലക്ഷ്യം. മറ്റു ചില നിവേദകരാകട്ടെ തീർത്തും വിപരീതമായ ഉദ്ദേശത്തോടെ വ്യാജ വിവാഹങ്ങൾ ആരോപിച്ചവരാണ്. ഉദാഹരണതിന് വ്യാജ നിവേദനങ്ങൾ നിർമ്മിക്കുന്നവനാണെന്ന് ഹദീസ് നിദാനശാസ്ത്ര പണ്ഡിതർ ഒരു പോലെ അഭിപ്രായപ്പെട്ട കൽബ് ഗോത്രക്കാരനായ ഒരു നിവേദകൻ കൽബ് ഗോത്രത്തിലെ പല സ്ത്രീകളിലേക്കും ചേർത്ത് പ്രവാചകന് (സ) വിവാഹങ്ങൾ ആരോപിക്കുന്നത് വായനക്കാർ ശ്രദ്ധിച്ചിരിക്കുമല്ലൊ. തങ്ങളുടെ ഗോത്രത്തിൽ നിന്നും പ്രവാചകൻ (സ) വിവാഹം ചെയ്തുവെന്ന ആദരവും ശ്രേഷ്ടതയും പിടിച്ചു പറ്റാനുള്ള വിഫല ശ്രമമാണിത് എന്ന് ന്യായമായും കരുതാം.

ഇത്തരം വ്യാജന്മാരെ കൂട്ടു പിടിക്കാതെ സ്വഹീഹായ സനദുകൾ (നിവേദക പരമ്പര) ഉള്ള അവലംബനീയമായ സ്രോതസ്സുകളിൽ നിന്ന്, മുഹമ്മദ് നബി (സ) 61 സ്ത്രീകളുമായി രമിച്ചെന്നോ പത്തൊമ്പത് സ്ത്രീകളെ വിവാഹം ചെയ്ത് ഭോഗിച്ചുവെന്നോ തെളിയിക്കാൻ ലോകത്ത് ഇന്ന് ജീവിച്ചിരിക്കുന്ന സർവ്വ നാസ്തികരേയും വെല്ലുവിളിക്കുന്നു.

അപ്പോൾ പിന്നെ സീറ രചയിതാക്കളായ മുസ്‌ലിം പണ്ഡിതർ എന്തിന് ഈ വ്യാജ കഥകൾ തങ്ങളുടെ ഗ്രന്ഥങ്ങളിൽ രേഖപ്പെടുത്തി എന്ന ചോദ്യം ഉയർന്നേക്കാം.

അതിനുള്ള ഉത്തരം എത്രയോ ആവർത്തി നാം വ്യക്തമാക്കി കഴിഞ്ഞു. എങ്കിലും മറുപടിയുടെ രത്നചുരുക്കം മറ്റൊരു ലേഖനത്തിൽ നിന്നും ഇവിടെ പേർക്കാം:

“ഹദീസ് ഗ്രന്ഥങ്ങൾക്കു പുറമെ സീറ (നബി ചരിത്ര) ഗ്രന്ഥങ്ങളുടേയും ഇസ്‌ലാമിക ചരിത്ര ഗ്രന്ഥങ്ങളുടേയും കാര്യവും സമാനമാണ്. പൗരാണിക ഹദീസ് – സീറ ഗ്രന്ഥങ്ങളിലെല്ലാം ഓരോ നിവേദനങ്ങൾക്കുമൊപ്പം സനദും കണിശമായി ചേർക്കപ്പെട്ടിരിക്കും. അവയിൽ എഴുതപ്പെട്ട ഹദീസുകളും നിവേദനങ്ങളുമെല്ലാം സ്വീകാര്യമാണെന്ന് ആ ഗ്രന്ഥകാരന്മാരായ പണ്ഡിതർക്കു തന്നെ അഭിപ്രായമില്ലെന്നാണ് ഇത് തെളിയിക്കുന്നത്. മറിച്ച് ഏത് നിവേദനങ്ങളുടെ സനദുകളാണോ സ്വഹീഹ് അവ മാത്രമാണ് -പ്രസ്തുത ഗ്രന്ഥകർത്താക്കളുടെ അടുക്കൽ തന്നെ – സ്വീകാര്യം, ദഈഫ് ആയ നിവേദനങ്ങൾ അസ്വീകാര്യങ്ങളും. ഈ ഗ്രന്ഥങ്ങളിൽ സ്വഹീഹായ നിവേദനങ്ങൾ മാത്രമല്ല പ്രസ്തുത പണ്ഡിതന്മാർ ഉൾപ്പെടുത്തിയിരിക്കുന്നത്. ഒരു വിഷയത്തെയൊ കാലഘട്ടത്തെയൊ ചർച്ച ചെയ്യുമ്പോൾ അതുമായി ബന്ധപ്പെട്ട് അവർക്കു കിട്ടിയ സർവ്വ നിവേദനങ്ങളും അവർ അവരുടെ ഗ്രന്ഥങ്ങളിൽ ഉൾപ്പെടുത്തിയിട്ടുണ്ട്. ആ നിവേദനങ്ങളിൽ ഏതൊക്കെയാണ് സത്യസന്ധം ഏതൊക്കെയാണ് വ്യാജം എന്നത് വേർതിരിച്ച് മനസ്സിലാക്കൽ ആ ഗ്രന്ഥങ്ങൾ പഠനവിധേയമാക്കുന്നവരുടെ ബാധ്യതയായാണ് ആ ഗ്രന്ഥങ്ങളുടെ രചയിതാക്കൾ മനസ്സിലാക്കിയത്. ഉദാഹരണത്തിന് ഇസ്‌ലാമിക ലോകത്തെ ഏറ്റവും പൗരാണികവും പ്രശസ്തവുമായ ചരിത്ര ഗ്രന്ഥത്തിന് ഉടമയായ ഇമാം ഇബ്നു ജരീർ ത്വബ്‌രി തന്റെ ചരിത്ര ഗ്രന്ഥത്തിന്റെ ആമുഖത്തിൽ പറയുന്നത് കാണുക:

“നമ്മുടെ ഈ ഗ്രന്ഥം വായിക്കുന്നവർ അറിയേണ്ട ഒരു വസ്തുതയുണ്ട്. അതായത് നാമിവിടെ കൊണ്ട് വന്നതും പറഞ്ഞതുമായ കാര്യങ്ങളിൽ എന്റെ അവലംബം, ഞാൻ തന്നെ നേരിട്ട് അറിയുന്ന ചില കാര്യങ്ങളും, നിവേദകന്മാരിലേക്ക് ഞാൻ ചേർത്തി ഉദ്ധരിച്ച സംഭവങ്ങളുമാണ്.

എന്നാൽ, ഭൂതകാലത്തെ പറ്റി നാം സംസാരിക്കവെ, വായനക്കാരന് അപരിചിതമായതോ കേൾക്കുന്നവന് വെറുപ്പുണ്ടാക്കുന്നതോ ആയ എന്തെങ്കിലുമുണ്ടെങ്കിൽ – അഥവാ ഒരു തരത്തിലും സ്വീകാര്യമല്ലാത്തവയോ വസ്തുതകൾക്ക് നിരക്കാത്ത ആശയത്തിലുള്ളതോ ഒക്കെ – അത് നമ്മുടെ അടുക്കൽ നിന്നുള്ളതല്ലെന്നും നിവേദകന്മാരിൽ നിന്നും സംഭവിച്ചതാണെന്നും അറിയണം. (ഏതെങ്കിലും വിഷയാസ്പദമായി) നമുക്ക് ലഭിച്ച നിവേദനങ്ങൾ അപ്പടി ഉദ്ധരിക്കുക മാത്രമാണ് നാം (ഈ ഗ്രന്ഥത്തിൽ) ചെയ്തിരിക്കുന്നത്. (അവയിലെ നെല്ലും പതിരും വ്യവഛേദിക്കൽ ഗ്രന്ഥ പഠിതാക്കളുടെ ബാധ്യതയാണ്)” (മുഖദ്ദിമ: താരീഖുത്വബ്‌രി: 5) ”

ചരിത്രത്തിൽ എവിടെയും രേഖപ്പെടുത്താൻ കഴിയാത്ത പീഢനകഥകൾ ‘ഇനിയും ഉണ്ടായേക്കാം ‘അവ നമുക്ക്’ നഷ്ടമായതായിരിക്കാം ‘എന്നൊക്കെ മുതലക്കണ്ണീരൊഴുക്കുന്നതിനും’ ആഞ്ഞ് ‘നിഗമിക്കുന്നതിനും യാതൊരു സാധുതയുമില്ല എന്ന് ഇതിൽ നിന്നും വ്യക്തമാവുന്നു. കാരണം ഉള്ളതെല്ലാം – നല്ലതും ചീത്തയും നോക്കാതെ, വിശ്വസ്‌തരോ വ്യാജരോ എന്ന് വേർതിരിക്കാതെ – തങ്ങളുടെ ഗ്രന്ഥത്തിൽ കൂട്ടി കുഴക്കുന്നത് ശീലമാക്കിയവരായിരുന്നു സീറക്കാർ എന്ന് വരുമ്പോൾ അവർക്ക് കിട്ടാതെ പോയ ചരിത്ര ‘വിവരങ്ങളെ’ ഓർത്ത് വിലപിക്കുന്ന ചരിത്രപരമായ അജ്ഞത മച്ചി പെറ്റ കുട്ടിക്ക് ചരമഗീതം പാടുന്നതു പോലെ മൂഢത്വമാണ്. “അതിന്റെ ബാഹുല്യം കാരണമാവാം… എവിടേയും പറയുന്നില്ല… ഒരു പക്ഷേ അത് പിൻ കാലത്ത് പതിവു പോലെ ചരിത്രത്തിൽ നിന്ന് നീക്കം ചെയ്തതാവാം…” എന്ന വാചകം ഒരു നിരാശാജനകമായ നിശ്വാസം മാത്രമാണ്. കഴിവിന്റെ പരമാവധി ന്യൂനതകൾ മുഹമ്മദ് നബിക്കെതിരെ(സ) കുത്തിപ്പൊക്കാൻ ശ്രമിച്ചിട്ടും സാധിക്കാതിരിക്കുന്നതിന്റെ ഗാഢമായ ദുഃഖം മുറ്റിയ മോഹഭംഗം മാത്രമാണത്.

മുഹമ്മദ് നബിയുടെ (സ) വിവാഹങ്ങളൊന്നും കാമ പ്രേരിതമായിരുന്നില്ല. ഓരോന്നിന്നും മതപരവും സാമൂഹികവും രാഷ്ട്രീയവുമായ മഹ്ത്‌ലക്ഷ്യങ്ങളും കാരണങ്ങളുമുണ്ടായിരുന്നു.

സാമൂഹികമായ കാരണങ്ങൾ:

ആദ്യ വിവാഹം നാൽപതുകാരിയായ ഖദീജയെയായിരുന്നു. അദ്ദേഹത്തേക്കാൾ പതിനാല് വയസ്സ് കൂടുതൽ പ്രായമുള്ള അവരെ വിവാഹം കഴിക്കുകയും അവർ മരണമടയുന്നതു വരെ 25 വർഷം അദ്ദേഹം അവരെ മാത്രം ഭാര്യയായി സ്വീകരിച്ച് ജീവിച്ചു. ബുദ്ധിമതിയും പക്വമതിയുമായ അവരുടെ വ്യക്തിത്വത്തിലും സ്നേഹ സമ്പന്നതയിലും അദ്ദേഹം ആകൃഷ്ടനായിരുന്നു. വ്യക്തിത്വ സമ്പൂർണ്ണതയെത്തിയ ഒരു സ്ത്രീയായിരുന്നു അവരെന്ന് മുഹമ്മദ് നബി (സ) തന്നെ വ്യക്തമാക്കുന്നുണ്ട്. അദ്ദേഹത്തിന്റെ സൽസ്വഭാവത്തിലും സത്യസന്ധതയിലും അവരും ആകൃഷ്ടയായിരുന്നു.

ഖദീജയുടെ(റ) മരണത്തിന് ശേഷം അദ്ദേഹം വിവാഹം കഴിച്ചത് സൗദയെയാണ്. നാല് മക്കളുള്ള വിധവയായിരുന്നു സൗദാ ബീവി (റ). അവരുടെ സംരക്ഷണവും പരിപാലനവും എളുപ്പമാക്കുന്നതായിരുന്നു ഈ വിവാഹത്തിലൂടെ.

ഹഫ്‌സ ബിൻത് ഉമറിനെ (റ) അവർ വിധവയായപ്പോൾ നബി (സ) വിവാഹം ചെയ്തു. അവർക്ക് താങ്ങും തണലുമാകാനും അവരുടെ പിതാവിന് ആദരവായി കൊണ്ടുമായിരുന്നു ഈ വിവാഹം.

ഉഹ്ദ് യുദ്ധത്തിൽ ഭർത്താവ് രക്തസാക്ഷിയായി, വിധവയായ സൈനബ് ബിൻത് ഖുസൈമയെ (റ) വിവാഹം ചെയ്ത് നബി (സ) അവർക്ക് അഭയം നൽകി.

ഒരുപാട് മക്കളും പ്രാരാബ്ദവുമുണ്ടായിരുന്ന പ്രവാചകന്റെ പ്രിയ അനുചരനായ അബൂസലമയുടെ (റ) വിധവയായ ഉമ്മു സലമയെ നബി (സ) ഏറ്റെടുത്തു.

മുസ്‌ലിംകൾക്കായി സ്വന്തം ജീവൻ ത്യാഗം ചെയ്ത ഒരുപറ്റം മനുഷ്യരുടെ വിധവകളെയാണ് പ്രവാചകൻ (സ) വിവാഹം ചെയ്തതിൽ അധികവും. അവരെ ആശ്വസിപ്പിക്കാനും, അവരുടെ കണ്ണീരൊപ്പാനും അവരുടെ സന്താനങ്ങളുടെ പരിപാലനം അന്യത്വമില്ലാതെ ഏറ്റെടുക്കാനും, നിരാലംബരായ അവരുടെ പ്രാർത്ഥനകൾക്ക് മറുപടിയെന്നോണമെല്ലാമാണ് ഈ വിവാഹങ്ങൾ എല്ലാം നടന്നത്.

മത നിയമ സ്ഥാപനത്തിന്റെ ഭാഗമായ വിവാഹങ്ങൾ:

ചെറുപ്പക്കാരിയും ബുദ്ധിമതിയുമായ ആഇശയാണ് (റ) പ്രവാചകന്റെ ജീവിത സന്ദേശങ്ങൾ ഹദീസുകളായി ഏറ്റവും കൂടുതൽ ഉദ്ധരിച്ച പ്രവാചക പത്നി; അല്ല സ്ത്രീകളിൽ തന്നെ ഏറ്റവും കൂടുതൽ ഉദ്ധരിച്ചിട്ടുള്ളത്. പുരുഷന്മാരേയും കൂട്ടി ഏറ്റവുമധികം ഹദീസുകൾ ഉദ്ധരിച്ച അഞ്ചാമത്തെ പ്രവാചകാനുചര. ദാമ്പത്യം, കുടുംബം, സ്ത്രീ വിഷയങ്ങൾ എന്നിവയിലെല്ലാം അവരാണ് ഇസ്‌ലാമിക കർമ്മശാസ്ത്ര വിജ്ഞാനങ്ങൾക്ക് അവലംബം. പ്രവാചകൻ (സ) മരണപ്പെടുമ്പോൾ ചെറുപ്പക്കാരിയായിരുന്നതിനാൽ പിന്നീട് ജീവിച്ച ദീർഘമായ കാലഘട്ടം വ്യത്യസ്‌ത സന്ദർഭങ്ങളിൽ അനവധി വിജ്ഞാന കുതുകികൾക്കും സ്ത്രീ ജനങ്ങൾക്കും ഈ ഹദീസ് പാഠങ്ങൾ പകരാനും ആവർക്ക് സാധിച്ചു. ഇതെല്ലാം കൊണ്ട് തന്നെയാണ് വഹ്‌യ് അഥവാ ദിവ്യ ബോധനം വഴി ആഇശയെ(റ) വിവാഹം ചെയ്യാൻ ദൈവകൽപ്പന ഉണ്ടായത്.

സൈനബ് ബിൻത് ജഹ്ശ്നെ(റ), പ്രവാചകന്റെ വളർത്ത് പുത്രനും അടിമ വംശജനുമായിരുന്ന സൈദിന് (റ) പ്രവാചകൻ (സ) വിവാഹം ചെയ്ത് കൊടുത്തതാണ്. ഈ ദമ്പതികൾക്ക് പരസ്പരം യാതൊരു തൃപ്തിയുമുണ്ടായിരുന്നില്ലെങ്കിലും, ക്ഷമ ദീക്ഷിക്കാനുള്ള പ്രവാചകന്റെ (സ) കൽപ്പന മാനിച്ചു കൊണ്ട് മാത്രമായിരുന്നു അവർ പരസ്പരം സഹജീവിച്ചത്. വളർത്തു പുത്രന്മാരെ സ്വപുത്രന്മാരായി പരിഗണിക്കുന്ന അറബികളിലെ സമ്പ്രദായം അവസാനിപ്പിച്ചു കൊണ്ട് നിയമം അവതരിപ്പിക്കപ്പെട്ടപ്പോൾ (ക്വുർആൻ: 33:5) ആ നിയമം മുസ്‌ലിങ്ങളുടെ മനസ്സിൽ സുസ്ഥാപിതമാക്കാനായി, മുസ്‌ലിംകളുടെ മനസ്സിലെ ധാർമ്മിക മാതൃകയും വിശുദ്ധിയുടെ പ്രതീകവുമായ മുഹമ്മദ് നബി(സ)യോട് തന്നെ വളർത്തു “പുത്രനായ” സൈദിന്റെ വിവാഹ മോചിതയായ ഭാര്യയെ വിവാഹം ചെയ്യാൻ അല്ലാഹു കൽപ്പന നൽകി. ഇതിലൂടെ അവരുടെ മനസ്സിലെ തെറ്റായ പുത്ര സങ്കൽപ്പത്തെ അല്ലാഹു വേരോടെ പിഴുതെറിഞ്ഞു.

രാഷ്ട്രീയ കാരണങ്ങൾ:

ശത്രുത ഉന്മൂലനം ചെയ്യാനും ഹൃദയങ്ങളെ കൂട്ടിയിണക്കാനും യുദ്ധാനന്തര ബന്ധികളെ അടിമത്തത്തിൽ നിന്ന് മോചിപ്പിക്കാനുമെല്ലാം ഏറ്റവും സ്നേഹജനകമായ മാധ്യമമായി വിവാഹത്തെ പരിഗണിച്ചു കൊണ്ട് നടന്നതാണ് മറ്റു പല വിവാഹങ്ങളും. നബിയോട് (സ) യുദ്ധം ചെയ്ത് പരാജിതരായ ബനൂ മുസ്ത്വലക് ഗോത്രക്കാരിൽ നിന്നും ബന്ധിയാക്കപ്പെട്ട ജുവൈരിയയെ(റ) നബി (സ) വിവാഹം ചെയ്തത് ഈ ലക്ഷ്യത്തോടെയായിരുന്നു. ബനൂ മുസ്ത്വലക് ഗോത്രക്കാരുടെ നേതാവിന്റെ പുത്രിയായിരുന്നു അവർ. ഈ വിവാഹത്തിലൂടെ ശത്രുത ഇല്ലാതാവുകയും പ്രവാചകന്റെ(സ) സ്നേഹ ബന്ധുക്കളായി ബനൂ മുസ്ത്വലക് ഗോത്രക്കാർ പരിണമിക്കുകയും ചെയ്തു.

പ്രവാചകന്റേയും ഇസ്‌ലാമിന്റെയും കൊടിയ ശത്രുവായിരുന്ന കുറൈശി നേതാവ് അബൂസുഫ്‌യാന്റെ(റ) – പിന്നീടദ്ദേഹം ഇസ്‌ലാം സ്വീകരിച്ചു – മകൾ ഉമ്മു ഹബീബയും (റ) അവരുടെ ഭർത്താവും മുസ്‌ലിംകളായിരുന്നു. എന്നാൽ പിൻകാലത്ത് അവരുടെ ഭർത്താവ് അഭയാർത്ഥിയായി ചെന്ന അബിസീനിയലിൽ വെച്ച് മരണപ്പെട്ടു. ഈ സന്ദർഭത്തിൽ നിരാലംബയും അഭയാർത്ഥിയുമായ ഉമ്മു ഹബീബയെ(റ) നബി (സ) ഏറ്റെടുത്തു. ഇത് അബൂസുഫ്‌യാന്റെ(റ) മനസ്സിലെ ശത്രുതയെ കെടുത്താനും പ്രവാചകന്റെ(സ) വ്യക്തിത്വത്തോട് ബഹുമാനം ജനിക്കാനും കാരണമായി. കൂടാതെ ഇസ്‌ലാമിൽ ഉറച്ചു നിന്നതിന്റെ പേരിൽ കൂടുതൽ പ്രതിഫലമല്ലാതെ നഷ്ടമൊന്നും ഒരു വിശ്വാസിക്കുണ്ടാവില്ല എന്ന പൊതുവായ ഗുണപാഠവും ഈ വിവാഹം വിശ്വാസികൾക്ക് പകർന്നു നൽകി.

സമാനമായ ലക്ഷ്യങ്ങളോടെ തന്നെയാണ് സ്വഫിയ (റ), മൈമൂന (റ) എന്നിവരുമായുള്ള വിവാഹവും നടന്നത്.

മുമ്പ് സൂചിപ്പിച്ചതു പോലെ നബിയുടെ (സ) ബഹുഭാര്യത്വം കൃത്യവും വ്യക്തവുമായ യുക്തിയുടേയും ന്യായത്തിന്റെയും അടിസ്ഥാനത്തിലായിരുന്നു. അവയ്ക്ക് സാമൂഹികവും ധാർമ്മികവും രാഷ്ട്രീയവുമായ ധർമ്മങ്ങളുണ്ടായിരുന്നു. അവ കാമ പ്രേരിതങ്ങളായിരുന്നില്ല.

കുറൈശികളിലെ സുന്ദരികളും കുലീനകളും മികച്ചവരുമായ സ്ത്രീകളെ തരാമെന്ന ക്വുറൈശികളുടെ വാഗ്ദാനങ്ങൾ തിരസ്കരിച്ച നബി (സ) ഈ പ്രയാസങ്ങളും പ്രാരാബ്ധങ്ങളും നിറഞ്ഞ വിധവകളേയും വിവാഹ മോചിതകളേയും മാതാക്കളേയും തന്നേക്കാൾ പ്രായം കൂടിയവരേയുമെല്ലാം വിവാഹം ചെയ്തത് കാമ പ്രേരിതനായി കൊണ്ടാണെന്ന് വിവേകവും നീതി ബോധവുമുള്ള ആർക്കാണ് വാദിക്കാൻ കഴിയുക ?!

പ്രവാചകന്റെ(സ) കൂടെ നിന്ന സ്ത്രീകൾ ഓരോരുത്തരും അദ്ദേഹത്തിന്റെ സാന്നിധ്യത്തിനായി കൊതിച്ചു. അദ്ദേഹത്തിന്റെ സഹവാസത്തിൽ ആനന്ദിച്ചു. ഭൂമിയിൽ തനിക്ക് കിട്ടാവുന്നതിൽ ഏറ്റവും നല്ല ഭർത്താവായി അദ്ദേഹത്തെ മനസ്സിലാക്കി. മറിച്ചൊരു ചിത്രം ഏത് സീറ പൊടിതട്ടിയെടുത്ത് ദുർബല നിവേദനങ്ങൾ ചികഞ്ഞ് കൊണ്ടുവന്നാലും തെളിയിക്കാനാവില്ല. പള്ളിയിൽ അബ്സീനിയക്കാർ കുന്തപ്പയറ്റ് കളിക്കുന്ന പ്രദർശനം കാണാൻ നബി (സ) ആഇശ ബീവിയെ വിളിച്ചു. നബി (സ) നിൽക്കുകയായിരുന്നു, ആഇശ (റ) പിന്നിലൂടെ വന്ന് അദ്ദേഹത്തിന്റെ തോളിൽ താടി വെച്ച് കവിളിൽ കവിളു ചേർത്ത് വാതിൽക്കൽ നിന്ന് കളി കണ്ടു കൊണ്ടിരുന്നു. കുറച്ചു കഴിഞ്ഞപ്പോൾ പ്രവാചകൻ (സ) മതിയായില്ലെ എന്നു ആഇശയോട് (റ) തിരക്കി. അവർ പറഞ്ഞു: അല്ലാഹുവിന്റെ ദൂതരേ, നിങ്ങൾ തിരക്കല്ലെ… കുറച്ചു കഴിഞ്ഞപ്പോൾ വീണ്ടും പ്രവാചകൻ (സ) മതിയായില്ലെ എന്നു ആഇശയോട്(റ) തിരക്കി. അവർ പറഞ്ഞു: അല്ലാഹുവിന്റെ ദൂതരേ, നിങ്ങൾ തിരക്കല്ലെ… അപ്പോൾ അദ്ദേഹം ആഇശക്ക്(റ) വേണ്ടി അങ്ങനെ നിന്നു കൊടുത്തു. ആഇശ (റ) പറയുന്നു: “എനിക്ക് അവരുടെ കളി കാണാനുള്ള ആഗ്രഹം കൊണ്ടല്ല ഞാൻ പ്രവാചകനെ പിടിച്ചു നിർത്തിയത്. മറിച്ച്, അദ്ദേഹത്തിന്റെ മനസ്സിൽ എനിക്കുള്ള സ്ഥാനവും എന്റെ മനസ്സിൽ അദ്ദേഹത്തിനുള്ള സ്ഥാനവും സ്ത്രീകൾ അറിയട്ടെ എന്ന കൊതിയായിരുന്നു അത്…” (സുനനുൽ കുബ്റാ: നസാഈ: 8:181, മുശ്കിലുൽ ആസാർ: ത്വഹാവി: 292, അഹ്കാമുന്നദ്ർ: 360, സിൽസിലത്തു സ്വഹീഹ: 7/818) പ്രവാചക പത്നി ഉമ്മു സലമ (റ) പറയുന്നു: അബൂ സലമ (റ) മരണപ്പെട്ടപ്പോൾ ഞാൻ നബിയുടെ (സ) അടുക്കൽ ചെന്ന് വിവരം അറിയിച്ചു. അദ്ദേഹം എനിക്ക് ഒരു പ്രാർത്ഥന പഠിപ്പിച്ചു തന്നു. اللَّهمَّ اغفِر لي ولَهُ وأعقِبني منْهُ عقبى حسَنةً “അല്ലാഹുവേ, എനിക്കും അദ്ദേഹത്തിനും (മരണപ്പെട്ട വ്യക്തിക്കും) നീ പാപങ്ങൾ പൊറുത്തു തരേണമേ. അദ്ദേഹത്തിൽ നിന്ന് ഏറ്റവും നല്ല പര്യവസാനവും നൽകേണമേ ”

ഞാൻ ചിന്തിച്ചു: “അബൂ സലമയേക്കാൾ നല്ല പര്യവസാനം (അല്ലെങ്കിൽ അദ്ദേഹത്തേക്കാൾ ഉത്തമായ ഇണ) ആരാണ് ?!” എങ്കിലും ഞാൻ ആ പ്രാർത്ഥന പ്രാർത്ഥിച്ചു.

അങ്ങനെ അബൂ സലമയേക്കാൾ ഉത്തമനായ ഇണയെ അല്ലാഹു എനിക്ക് നൽകി. അത് അല്ലാഹുവിന്റെ ദൂതനായിരുന്നു. (സ്വഹീഹു മുസ്‌ലിം: 919, 918, തുർമുദി: 977)

പ്രവാചക പത്നി ഉമ്മു ഹബീബ (റ), പ്രവാചകനോടൊപ്പമുള്ള തന്റെ ഊഷ്മളമായ ദാമ്പത്യ ജീവിത്തിൽ ആനന്ദ പ്രേരിതയായി തന്റെ സഹോദരിയെ കൂടി അദ്ദേഹം വിവാഹം കഴിച്ചിരുന്നെങ്കിൽ എന്ന് കൊതിക്കുകയുണ്ടായി. أَحَبُّ مَن شَرِكَنِي في الخَيْرِ أُخْتِي “എന്നോടൊപ്പം ഈ സൗഖ്യത്തിൽ പങ്കാളിയാവാൻ ഞാൻ ഏറ്റവും ഇഷ്ടപ്പെടുന്നത് എന്റെ സഹോദരിയാണ്…” എന്ന് അവർ പ്രവാചകനോട് അപേക്ഷിച്ചു. രണ്ട് സഹോദരിമാരെ ഒരുമിച്ച് വിവാഹം കഴിക്കുന്നത് അനുവദനീയമല്ല എന്ന് പഠിപ്പിച്ചു കൊണ്ട് അദ്ദേഹം ആ വിവാഹാഭ്യർത്ഥന നിരസിച്ചു. എന്നു മാത്രമല്ല തങ്ങളുടെ ഭാര്യമാരോട്, (فلا تَعْرِضْنَ عَلَيَّ بَنَاتِكُنَّ، وَلَا أَخَوَاتِكُنَّ) “നിങ്ങളുടെ മക്കളുടേയൊ സഹോദരിമാരുടേയോ വിവാഹാഭ്യർത്ഥനയുമായി എന്റെ അടുത്ത് വരരുത്” എന്ന് വിലക്കുക കൂടി ചെയ്തു. (സ്വഹീഹു മുസ്‌ലിം: 1449)

ഇതും ഇതല്ലാത്തതുമായ മറ്റു പല സന്ദർഭങ്ങളിലും പ്രവാചകൻ (സ) വിവാഹാഭ്യർത്ഥനകൾ വിലക്കുന്നതും നിരസിക്കുകയും ചെയ്യുന്നതായി സ്വഹീഹായ ഹദീസുകളിൽ കാണാം. ഒരു പക്ഷെ സ്വീകരിച്ച വിവാഹാഭ്യർത്ഥനകളേക്കാൾ അദ്ദേഹം നിരസിച്ച വിവാഹാഭ്യർത്ഥനകളാണ് എണ്ണത്തിൽ കൂടുതലും. ഒരു സ്ത്രീലംബടനും കാമവെറിയനും ഇപ്രകാരം ചെയ്തതായി നമുക്കിതുവരെ അറിവില്ല.

പ്രവാചക പത്നി ഖദീജക്ക്(റ) പ്രവാചകനെ പറ്റിയുള്ള അഭിപ്രായം നോക്കൂ:

كَلَّا وَاللَّهِ مَا يُخْزِيكَ اللَّهُ أَبَدًا ، إِنَّكَ لَتَصِلُ الرَّحِمَ ، وَتَحْمِلُ الكَلَّ ، وَتَكْسِبُ المَعْدُومَ ، وَتَقْرِي الضَّيْفَ ، وَتُعِينُ عَلَى نَوَائِبِ الحَقِّ

“അല്ലാഹുവാണേ, അല്ലാഹു നിങ്ങളെ ഒരിക്കലും അപമാനിക്കുകയില്ല. തീർച്ചയായും നിങ്ങൾ കുടുംബ ബന്ധങ്ങൾ ചേർക്കുന്ന വ്യക്തിയാണ്, മറ്റുള്ളവരുടെ ദുഖഭാരം ഏറ്റെടുക്കുന്നവനും, പാവപ്പെട്ടവർക്ക് വേണ്ടി പണിയെടുക്കുന്നവനും, അതിഥികളെ തീറ്റുന്നവനും, സാമൂഹത്തിന് ദുരന്തം നേരിടുമ്പോൾ സഹായത്തിൽ മുഴുകുന്നവനുമാണ്…” (സ്വഹീഹുൽ ബുഖാരി: 3)

പ്രവാചകപത്നി സ്വഫിയ്യയുടെ(റ) അടിമത്ത മോചനവുമായി ബന്ധപ്പെട്ട അനുഭവം തന്നെ പരിശോധിക്കുക. ഖൈബർ യുദ്ധത്തിൽ ബന്ദികളാക്കപ്പെട്ടവരുടെ കൂട്ടത്തിൽ സ്വഫിയ്യയും ഉണ്ടായിരുന്നു. പ്രവാചകൻ (സ) അവരെ സ്വന്തത്തിനായി തിരഞ്ഞെടുക്കുകയും അവർക്ക് മുമ്പിൽ രണ്ട് വഴികളിലൊന്ന് തിരഞ്ഞെടുക്കാനായി അവസരം നൽകുകയും ചെയ്തു. ഒന്നുകിൽ മോചിതയാവുകയും തന്റെ ഭാര്യയാവുകയും ചെയ്യുക. അല്ലെങ്കിൽ മോചിതയാവുകയും കുടുംബത്തിലേക്ക് ചെന്നുചേരുകയും ചെയ്യാം. മോചിതയാവുകയും പ്രവാചകന്റെ പത്നിയാവുകയും ചെയ്യുക എന്നതാണ് സ്വഫിയ്യ സ്വമനസ്സാൽ തിരഞ്ഞെടുത്തത്. (സ്വഹീഹു ഇബ്നുഹിബ്ബാൻ: 4613)

വിമർശനം:

“പരിപൂർണ്ണതയിലെത്താത്ത മറ്റു ഡിവോഴ്സുകൾ”

ഉമ്രാ ബിൻത് റിഫാ… മേൽ പറഞ്ഞ റിഫയുടെ സഹോദരി…. (ഇബ്നു സാദ് 8: 107)

മറുപടി:

ഈ നാമത്തിലുള്ള ഒരു സ്ത്രീയെ സംബന്ധിച്ച വിവാഹ കഥ ഈയുള്ളവന്റെ പരിശോധനയിൽ എവിടെയും കണ്ടെത്താൻ കഴിഞ്ഞില്ല. കിട്ടിയ പേര് വെച്ച് ഒരു സാങ്കൽപിക ഭാര്യയെ ഭാവനാത്മകമായി രൂപകൽപ്പന ചെയ്തെടുത്തതാവാം. അങ്ങനെയാണെങ്കിൽ ഭൗതികവാദികളെ സംബന്ധിച്ച് അത് ഒരു അത്ഭുതമൊ പുതിയതൊ അല്ല.

വിമർശനം:

ബിൻത് ജുദൂബ് ഇബ്ന് ദാമ്രാ ഓഫ് ജൻദ്രാ…. കെട്ടി … ഒഴിവാക്കി..(ഇബ്നു സാദ് 8: 106)

മറുപടി:

ജന്ദഇയ്യ ഇബ്നതു ജുന്ദുബ് ഇബ്നു ദംറയെ നബി (സ) വിവാഹം അന്വേഷിച്ചു എന്നത് പോലും സ്ഥിരപ്പെട്ടിട്ടില്ല.

ഇബ്നു സഅ്ദ് തന്റെ ത്വബകാത്തിൽ കഥ ഉദ്ധരിച്ച നിവേദക പരമ്പരകൾ രണ്ടും കണ്ണി മുറിഞ്ഞതും ദുർബലരായ നിവേദകരാൽ നിർഭരവുമാണ്. എല്ലാം ഉദ്ധരിക്കുന്നത് മുഹമ്മദ് ഇബ്നു ഉമർ ആണ്:

أخبرنا محمد بن عمر قال حدثني محمد بن عبد الله عن الزهري مثل ذلك بنت جندب ابن ضمرة الجندعي أخبرنا محمد بن عمر حدثني عبد الله بن جعفر عن يزيد بن بكر أن رسول الله صلى الله عليه وسلم تزوج بنت جندب بن ضمرة الجندعي

മുമ്പത്തെ ലേഖനങ്ങളിൽ വിശദീകരിച്ചതു പോലെ മുഹമ്മദ് ഇബ്നു ഉമർ ദുർബലനാണ്. എന്ന് മാത്രമല്ല ഈ കഥ ഉദ്ധരിച്ച മുഹമ്മദ് ഇബ്നു ഉമർ തന്നെ ഈ കഥ കെട്ടുകഥയാണെന്നും നബി (സ) ഒരു കിനാന ഗോത്രക്കാരിയെയും ഒരിക്കലും വിവാഹം ചെയ്തിട്ടില്ലെന്നും പറയുന്ന നിവേദനവും ഇബ്നു സഅ്ദിന്റെ ത്വബക്കാത്തിൽ (8:149), കഥയുടെ തൊട്ട് താഴെയുണ്ടെങ്കിലും നാസ്തികർ അത് കണ്ട മട്ടില്ല.

قال محمد بن عمر وأصحابنا ينكرون ذلك ويقولون لم يتزوج رسول الله صلى الله عليه وسلم كنانية قط

വിമർശനം:

ജാമ്രാ ബിൻത് അൽ ഹരിത്…. മുഹമ്മദ് വിഹാഹ കരാറിൽ ഒപ്പിട്ട ശേഷം അറിഞ്ഞു അവൾക്ക് രോഗമുണ്ട് എന്ന്… ഒഴിവാക്കി (തബാരി v9, P 140- 141)

മറുപടി:

‘വിവാഹ കരാർ ചെയ്തു’, ‘വിവാഹ കരാറിൽ ഒപ്പിട്ടു’ എന്നൊക്കെ എല്ലാ കഥകളിലും നാസ്തികർ ആവർത്തിച്ചു കൊണ്ടിരിക്കുന്ന പദങ്ങൾ മിഷണറി പരിഭാഷകളിൽ നിന്ന് കടമെടുത്തവയാണ്. അറബിയിൽ ഖത്വബ (خطب) അഥവാ വിവാഹം അന്വേഷിച്ചു എന്നെയുള്ളു. അതിൽ ഒരു പഞ്ച് പോരാത്തതു കൊണ്ടും നബിയെ(സ) ഒരു കരാർ ലംഘകനാക്കാനും വേണ്ടിയാണ് ഈ പദങ്ങളിലെ ബോധപൂർവ്വമുള്ള തിരിമറികൾ. ഏതായാലും കഥ എല്ലായ്‌പ്പോഴും പോലെ വ്യാജമാണ്. സനദു പോലുമില്ലാത്ത രണ്ട് വരി!!. (താരീഖുത്വബ്‌രി: 3: 169)

വിമർശനം:

അൽ ഷാൻബാ ബിൻത് അമ്ര്… പ്രവാചകനല്ല എന്ന് ആദ്യ ദിവസം തന്നെ കളിയാക്കിയത് കൊണ്ട് ഒഴിവാക്കി (തബാരി 9, P 136)

മറുപടി:

ശൻബാഅ് ബിൻത് ഉമർ അൽ ഗിഫാരി. കഥ ഇപ്രകാരമാണ്:

ശൻബാഅ്നെ പ്രവാചകൻ (സ) വിവാഹം ചെയ്ത ആദ്യ രാത്രി അദ്ദേഹത്തിന്റെ പുത്രൻ ഇബ്രാഹിം മരണപ്പെട്ടു. അപ്പോൾ ശൻബാഅ് പറഞ്ഞു: അദ്ദേഹം പ്രവാചകനായിരുന്നെങ്കിൽ അദ്ദേഹത്തിന് ഏറ്റവും പ്രിയപ്പെട്ടവർ എന്തുകൊണ്ട് മരണപ്പെട്ടു? ഇസ്‌ലാമിക പാഠങ്ങൾ തീരെ അറിവില്ലാത്തതിനാലും ദുർബല വിശ്വാസി ആയതിനാലും പ്രതിസന്ധി ഘട്ടത്തിൽ തന്നെ ആശ്വസിപ്പിക്കാനുള്ള പക്വത ഇല്ലാത്തതിനാലുമെല്ലാമാവാം നബി (സ) അവരെ വിവാഹ മോചനം ചെയ്തത് എന്ന് അനുമാനിക്കാം. പക്ഷെ നാസ്തികൻ സ്വന്തം വക കാരണം കണ്ടെത്തി.

“പ്രവാചകനല്ല എന്ന് ആദ്യ ദിവസം തന്നെ ‘കളിയാക്കിയതിനാൽ’ ഒഴിവാക്കി”

ഒരു തമാശ പറഞ്ഞതിന് ഒഴിവാക്കി എന്ന് വരുത്തി തീർത്തു. മകൻ മരിച്ച കാര്യം സൂചിപ്പിച്ചുമില്ല. അതിന് മറ്റൊരു കാരണം കൂടി ഉണ്ട്. “മുഹമ്മദിന്റെ പെണ്ണുങ്ങൾ” എന്ന ലേഖനത്തിന്റെ അവസാനത്തിൽ “ഇത്രയൊക്കെയായിട്ടും മുഹമ്മദിന് കുട്ടികളില്ല” എന്ന ഒരു കല്ലുവെച്ച നുണ പറഞ്ഞാണ് ലേഖനം അവസാനിപ്പിക്കുന്നത്. അപ്പോൾ പിന്നെ നിവേദനം മുഴുവനായും ഉദ്ധരിക്കാൻ കഴിയില്ലല്ലൊ.

ഏതായാലും ഈ നിവേദനവും വ്യാജമാണ്. സനദ് (നിവേദക പരമ്പര) പോലും കഥയ്ക്കില്ല.

വിമർശനം:

“പരിപൂർണ്ണതയിലെത്താത്ത മറ്റു ഡിവോഴ്സുകൾ”

സനാ അൽ-നഷാത്ത് ബിൻത് റിഫാ… മുഹമ്മദിന്റെ ഒരു സാദാ പടയാളിയുടെ മകൾ… സ്ഥാനക്കയറ്റത്തിന് വേണ്ടി മുഹമ്മദിന് മകളെ വിവാഹം ചെയ്തു കൊടുത്തു… മുഹമ്മദ് കരാറിൽ ഒപ്പിട്ട ശേഷം മരിച്ചു (ആത്മഹത്യയാവാം) (അൽ തബാരി v9, P 135-136, അൽ തബാരി v 39 P 166)

(മുഹമ്മദിന്റെ പെണ്ണുങ്ങൾ: നാസ്‌തിക സോഷ്യൽ മീഡിയ തെറിമാല)

മറുപടി:

നാസ്‌തിക സോഷ്യൽ മീഡിയ തെറിമാലകളിൽ ഒന്നായ “മുഹമ്മദിന്റെ പെണ്ണുങ്ങൾ” എന്ന കുറിപ്പിൽ നിന്നുള്ള ചില വരികളെയാണ് നാം തുടർച്ചയായി നിരൂപണം ചെയ്‌തു കൊണ്ടിരിക്കുന്ന്. മുമ്പ് സൂചിപ്പിച്ചതു പോലെ, സ്ത്രീവിമോചകനായ നബിയെ (സ), സ്ത്രീ പീഢകനും ലമ്പടനുമായി പ്രചരിപ്പിക്കാൻ വേണ്ടി കല്ലുവച്ച നുണകളും, അർദ്ധ സത്യങ്ങളും, ദുർവ്യാഖ്യാനങ്ങളും, വൈരുദ്ധ്യങ്ങളും കൂട്ടി കുഴച്ചുണ്ടാക്കിയ വിധ്വേഷ കഷായമാണ് ലേഖനം.

ആരോപണ വിധേയമായ വിഷയത്തിലേക്ക് കടന്നു വരാം…

സനാഅ് ബിൻത്ത് സ്വൽത്, നഷാത്ത് ബിൻത് രിഫാഅ എന്നിങ്ങനെ രണ്ട് സ്ത്രീകളുടെ നാമങ്ങൾ കൂട്ടിയോജിപ്പിച്ച് നാസ്‌തികരുണ്ടാക്കിയ ഒരു പുതിയ ഭാര്യയാണ് “സനാ അൽ-നഷാത്ത് ബിൻത് റിഫാ… ” !!

ഇതിനൊരു കാരണമുണ്ട്. ഇത് രണ്ടും രണ്ട് സ്ത്രീകളാണൊ ? അതൊ ഒരു സ്ത്രീയെ തന്നെ ഉദ്ദേശിച്ച് രണ്ട് പേര് പറയപ്പെട്ടതാണൊ ? എന്നതിലൊന്നും ആർക്കും ഒരു നിശ്ചയവുമില്ല. ഇങ്ങനെയൊരു സ്ത്രീയുണ്ടൊ എന്നതിന് തന്നെ കൃത്യമായ തെളിവൊന്നും ഇല്ല എന്നത് മറ്റൊരു വസ്തുത.

ഇവരുടെ നാമത്തിലൊ അസ്തിത്വത്തിലൊ യാതൊരു തീർച്ചയുമില്ല എന്ന് നിവേദനം ഉദ്ധരിച്ച ത്വബ്‌രി തന്നെ വ്യക്തമാക്കിയിട്ടുമുണ്ട്. (താരീഖു ത്വബ്‌രി: 2: 416)

ഇനി, നിവേദനത്തിന്റെ പരമ്പരകളിലേക്ക് വരാം. ത്വബ്‌രി തന്റെ താരീഖിൽ ഒരു പരമ്പരയുമില്ലാതെ ചില അഭിപ്രായങ്ങൾ രേഖപ്പെടുത്തുകയാണ് ചെയ്തിരിക്കുന്നത്. എന്നാൽ ത്വബ്‌രിയുടെ മറ്റൊരു ഗ്രന്ഥത്തിൽ അദ്ദേഹത്തിന് ഈ വ്യാജ വാർത്ത ലഭിച്ച നിവേദക പരമ്പര അദ്ദേഹം വ്യക്തമാക്കുന്നുണ്ട്:

قال هشام بن محمد الكلبي حدثني رجل من رهط عبد الله بن خازم السلمي أن رسول الله صلى الله عليه وسلم تزوج سنا بنت الصلت بن حبيب السلمية فماتت قبل أن يصل إليها

“ഹിശാമിബ്നു മുഹമ്മദ് അൽ കൽബി പറഞ്ഞു: അബ്ദുല്ലാഹിബ്നു ഖാസിം അസ്സുലമിയുടെ സംഘത്തിൽ പെട്ട’ ഏതോ ഒരാൾ’ എന്നോട് പറഞ്ഞു: ദൈവദൂതൻ (സ) സനാഅ് ബിൻത്ത് സ്വൽത് ഇബ്നു ഹബീബ് അസ്സുലമിയയെ വിവാഹം ചെയ്തു. അവരുടെ അടുത്തെത്തുന്നതിന് മുമ്പ് തന്നെ അവർ മരണമടഞ്ഞു…” (അൽ മുൻതഖബ് മിൻ ദൈലിൽ മുദയ്യൽ: ത്വബ്‌രി: 90)

സനദ് (നിവേദക പരമ്പര):

പരമ്പരയിലെ ഹിശാമിബ്നു മുഹമ്മദ് അൽ കൽബിയെ സംബന്ധിച്ച വിശദ വിവരങ്ങൾ മുൻ കഴിഞ്ഞ ലേഖനങ്ങളിൽ വന്നിട്ടുണ്ടല്ലൊ. ഹിശാമിബ്നു മുഹമ്മദ് അൽ കൽബിയും അദ്ദേഹത്തിന്റെ പിതാവും നുണ പറയുന്നവരും ദുർബലരുമാണെന്നതിൽ ഹദീസ് നിദാന ശാസ്ത്ര പണ്ഡിതർക്കിടയിൽ ഇരു പക്ഷമില്ല.

ഹിശാമിന് ഈ കഥ കിട്ടിയതാവട്ടെ “അബ്ദുല്ലാഹിബ്നു ഖാസിം അസ്സുലമിയുടെ സംഘത്തിൽ പെട്ട ‘ഏതോ ഒരാൾ’ എന്നോട് പറഞ്ഞു” കൊണ്ടാണ്. ഏതോ ആളുകൾ പറഞ്ഞ കഥകളൊക്കെ തെളിവിന് നിരക്കുന്നതാണൊ ?!

ഇനി നിവേദനത്തിന്റെ മത്‌ന് അഥവാ ഉള്ളടക്കത്തിലേക്ക് വരാം: “ദൈവദൂതൻ (സ) സനാഅ് ബിൻത്ത് സ്വൽത് ഇബ്നു ഹബീബ് അസ്സുലമിയയെ വിവാഹം ചെയ്തു. അവരുടെ അടുത്തെത്തുന്നതിന് മുമ്പ് തന്നെ അവർ മരണമടഞ്ഞു…” ഇത്രയെ നിവേദനത്തിലുള്ളു. പക്ഷെ നബിയെ സ്ത്രീ പീഢകനും ലമ്പടനുമാക്കാൻ വേണ്ടിയുള്ള ലേഖനമായതിനാൽ ഈ കഥക്ക് ഒരു പഞ്ച് പോര !! അപ്പോൾ നാസ്‌തിക ബുദ്ധിയിലെ പാഷാണം ഉറവ പൊട്ടി ഒഴുകാൻ തുടങ്ങി…..

“മുഹമ്മദിന്റെ ഒരു സാദാ പടയാളിയുടെ മകൾ… സ്ഥാനക്കയറ്റത്തിന് വേണ്ടി മുഹമ്മദിന് മകളെ വിവാഹം ചെയ്തു കൊടുത്തു… മുഹമ്മദ് കരാറിൽ ഒപ്പിട്ട ശേഷം മരിച്ചു (ആത്മഹത്യയാവാം)….”

ഇപ്പോഴാണ് ഒരു പഞ്ച് വന്നത് !! നബിക്ക് ഒരു വില്ലൻ പരിവേഷം കിട്ടി ആശ്വാസമായി !! ചരിത്ര ധർമ്മവും വൈജ്ഞാനിക നീതിയും സത്യസന്ധതയുമൊക്കെ അവിടെ നിൽക്കട്ടെ എന്നങ്ങ് തീരുമാനിച്ചു. അല്ലെങ്കിലും ധാർമ്മികതക്ക് ഈ വിഡ്ഢി പരിഷകളുടെ അടുക്കൽ വല്ല അസ്തിത്വമൊ അടിത്തറയൊ ഉണ്ടൊ ?!

നഷാത്ത് ബിൻത് രിഫാഅ എന്ന നാമം -ചില നിവേദനങ്ങിൽ – ലോപിച്ച് ‘ഷാത്ത്’ ബിൻത് രിഫാഅ എന്ന് വന്നതായി കാണാം.

മുഫദ്ദൽ അൽ ഗസ്സാനി തന്റെ താരീഖിൽ സൂചിപ്പിക്കുകയും ഇബ്നുൽ കയ്യിം തന്റെ ‘സബീലുൽ ഹുദാ വർറശാദ് ഫീ സീറതി ഖൈറിൽ ഇബാദ് എന്ന ഗ്രന്ഥത്തിൽ ക്വോട്ട് ചെയ്യുകയും ചെയ്ത നിവേദനത്തിന്റെ പരമ്പര ഇപ്രകാരമാണ്:

روى المفضل بن غسان العلائي في تاريخه من طريق سيف بن عمر عن أبي عمر عثمان بن مقسم عن قتادة قال:

മുഫദ്ദൽ ബിൻ ഗസ്സാൻ അൽ അലാഈ തന്റെ താരീഖിൽ ഉദ്ധരിച്ചു: സൈഫ് ഇബ്നു ഉമർ വഴി: അബൂ ഉമർ ഉസ്മാനിബ്നു മിക്സം ൽ നിന്ന്: കത്താദയിൽ നിന്ന്, അദ്ദേഹം പറഞ്ഞു:…

നിവേദനം ഉദ്ധരിച്ച ഇബ്നുൽ കയ്യിം തന്നെ പറയുന്നു:

وعثمان بن مقسم متروك

നിവേദകനായ ഉസ്മാനിബ്നു മിക്സം മത്റൂക് (അതി ദുർബലൻ) ആകുന്നു. പ്രവാചക ശിഷ്യൻമാരെ തെറി വിളിക്കുന്ന, മീസാൻ പോലെയുള്ള ഇസ്‌ലാമിലെ വിശ്വാസ കാര്യങ്ങൾ നിഷേധിക്കുന്ന, കദരിയ്യായ വ്യക്തിയാണ് ഉസ്മാനിബ്നു മിക്സം എന്ന് ഇമാം ദഹബി വ്യക്തമാക്കിയിട്ടുണ്ട്. വ്യാജ ഹദീസുകൾ നിർമ്മിക്കാറുള്ള വ്യക്തിയാണിയാൾ എന്ന് യഹ്‌യബ്നു മഈൻ വ്യക്തമാക്കി. ധാരാളം കളവുകൾ പറയുന്ന വ്യക്തിയാണിയാൾ എന്ന് ജൂസജാനി പറഞ്ഞു. (മീസാനുൽ ഇഅ്തിദാൽ: 3: 56, അൽ ജർഹു വതഅ്ദീൽ: 6: 176, അദ്ദുഅഫാഉൽ കബീർ: 3: 220)

മറ്റൊരു നിവേദകനായ കത്താദ താബിഈ ആകുന്നു. ആരിൽ നിന്നാണ് സംഭവം കേട്ടതെന്ന് വ്യക്തമാക്കാത്തതിനാൽ പരമ്പര മുറിഞ്ഞതാണ്.

സൈഫിബ്നു ഉമർ അത്തീമി എന്ന നിവേദകനും ദുർബലനാണ്. أبو حاتم بن حبان البستي : يروي الموضوعات عن الأثبات، اتهم بالزندقة، قالوا إنه كان يضع الحديث

ഇബ്നു ഹിബ്ബാൻ പറഞ്ഞു: വിശ്വസ്‌തരിൽ നിന്ന് വ്യാജ നിവേദനങ്ങൾ ഉദ്ധരിക്കുമായിരുന്നു. ഉള്ളിൽ അവിശ്വാസം ഒളിപ്പിച്ചിരുന്നവരുടെ കൂട്ടത്തിൽ ആണെന്ന് സംശയിക്കപ്പെട്ടിരുന്ന വ്യക്തി. അദ്ദേഹം വ്യാജ ഹദീസുകൾ നിർമ്മിക്കുമായിരുന്നു.

വിമർശനം:

“പരിപൂർണ്ണതയിലെത്താത്ത മറ്റു ഡിവോഴ്സുകൾ”

ശറാഫ് ബിൻത് കലീഫ … മേൽ പറഞ്ഞ ഖവ്‌ല മരിച്ചതിന് ശേഷം അവരുടെ കുടുംബം മുഹമ്മദുമായി ബന്ധം പുനസ്ഥാപിച്ച് തടി രക്ഷിക്കാൻ വേണ്ടി അയച്ചു കൊടുത്ത സ്ത്രീ… ഇവരെ ഡിവോഴ്സ് ചെയ്ത് നബി ആ ഗോത്രവും ആയുള്ള കരാർ ഇല്ലാതാക്കി.. (തബാരി v9, P 138; ഇബ്നു സാദ് v 8, P 116 -117)

(മുഹമ്മദിന്റെ പെണ്ണുങ്ങൾ: നാസ്‌തിക സോഷ്യൽ മീഡിയ തെറിമാല)

മറുപടി:

നാസ്‌തിക സോഷ്യൽ മീഡിയ തെറിമാലകളിൽ ഒന്നായ “മുഹമ്മദിന്റെ പെണ്ണുങ്ങൾ” എന്ന കുറിപ്പിൽ നിന്നുള്ള ചില വരികളെയാണ് നാം തുടർച്ചയായി നിരൂപണം ചെയ്‌തു കൊണ്ടിരിക്കുന്ന്. മുമ്പ് സൂചിപ്പിച്ചതു പോലെ, സ്ത്രീവിമോചകനായ നബിയെ (സ), സ്ത്രീ പീഢകനും ലമ്പടനുമായി പ്രചരിപ്പിക്കാൻ വേണ്ടി കല്ലുവച്ച നുണകളും, അർദ്ധ സത്യങ്ങളും, ദുർവ്യാഖ്യാനങ്ങളും, വൈരുദ്ധ്യങ്ങളും കൂട്ടി കുഴച്ചുണ്ടാക്കിയ വിധ്വേഷ കഷായമാണ് ലേഖനം.

വിമർശനത്തിലേക്ക് വരാം…

ശറാഫ് ബിൻത് ഖലീഫയെ ദൈവദൂതൻ വിവാഹം ചെയ്തു എന്ന് സൂചിപ്പിക്കുന്ന നിവേദനം ഇപ്രകാരമാണ്:

أَخْبَرَنَا هِشَامُ بْنُ مُحَمَّدِ بْنِ السَّائِبِ ، قَالَ : حَدَّثَنَا الشَّرْقِيُّ بْنُ الْقَطَّامِيُّ قَالَ : لَمَّا هَلَكَتْ خَوْلَةُ بِنْتُ الْهُذَيْلِ تَزَوَّجَ رَسُولُ اللَّهِ صَلَّى اللَّهُ عَلَيْهِ وَسَلَّمَ شَرَافَ بِنْتَ خَلِيفَةَ أُخْتَ دِحْيَةَ وَلَمْ يَدْخُلْ بِهَا

ഹിശാമിബ്നു മുഹമ്മദ് ഇബ്നുസ്സാഇബ് അറിയിച്ചു: നമ്മോട് ശർക്വിയ്യിബ്നു കത്വാമി പറയുകയുണ്ടായി: ഖൗല ബിൻത് ഹുദൈൽ മരണപ്പെട്ടപ്പോൾ ദൈവദൂതൻ (സ) ശറാഫ് ബിൻത് ഖലീഫയെ – ദിഹ്‌യയുടെ സഹോദരി – വിവാഹം ചെയ്തു. അവരോടൊപ്പം താമസിച്ചില്ല (അതിനു മുമ്പ് വിവാഹ മോചനം നടന്നു)

1. സനദിലെ ഹിശാമിബ്നു മുഹമ്മദ് അൽ കൽബി നുണയനും ദുർബലനുമാണെന്ന, ഹദീസ് നിദാന ശാസ്ത്ര പണ്ഡിതന്മാരുടെ അഭിപ്രായങ്ങൾ മുമ്പത്തെ ലേഖനത്തിൽ ചേർത്തിട്ടുണ്ട്.

2. മറ്റൊരു നിവേദകനായ ‘ശർകിയ്യിബ്നു ക്വത്വാമി’ യും ദുർബലനാണ്. ഇതും മുമ്പത്തെ ലേഖനത്തിൽ വിശദമായി പ്രതിപാദിച്ചിട്ടുണ്ട്.

3. ശർകിയ് പ്രവാചകാലഘട്ടക്കാരനല്ല; താൻ ആരിൽ നിന്നാണ് ഈ കഥ കേട്ടതെന്ന് വ്യക്തമാക്കിയിട്ടുമില്ല. അഥവാ പരമ്പര കണ്ണി മുറിഞ്ഞതാണ്.

ഈ മൂന്ന് കാരണങ്ങളാൽ തന്നെ ഇത്തരമൊരു വിവാഹം പ്രവാചക ജീവിതത്തിൽ നടന്നിട്ടില്ലെന്നും, ചില തൽപര കക്ഷികൾ കെട്ടിച്ചമച്ചുണ്ടാക്കിയ വ്യാജകഥ മാത്രമാണിതെന്നും സുതരാം വ്യക്തമാണ്.

ബനൂ കൽബ് ഗോത്രക്കാരനായ ഹിശാമിബ്നു മുഹമ്മദ് അൽ കൽബി എന്ന റാവി തന്റെ ഗോത്രത്തിലെ ഒരുപാട് സ്ത്രീകളുമായി നബി(സ)ക്ക് വിവാഹ ബന്ധമുണ്ടായിരുന്നു എന്ന് ഗോത്ര മഹിമ പ്രചരിപ്പിക്കാൻ ചമച്ചുണ്ടാക്കിയതാവാം ഈ കഥകൾ മുഴുവൻ. എന്നിട്ട് ആരും ഈ വിവാഹങ്ങൾ അറിയാതിരുന്നതെന്തെ? എന്ന് നബി ചരിത്രത്തെയും നബി കുടുംബത്തെയും സംബന്ധിച്ച് വിവരമുള്ളവർ ചോദിക്കാനുള്ള സാധ്യത ഇല്ലാതാക്കാൻ, പെട്ടെന്ന് തന്നെ വിവാഹ മോചനങ്ങളും നടന്നു എന്നും കഥയിൽ റ്റ്വിസ്റ്റുണ്ടാക്കി പ്രചരിപ്പിച്ചു എന്ന് കരുതാനെ വഴിയുള്ളു.

സ്വമേധയാ ഇസ്‌ലാം സ്വീകരിച്ച ഗോത്രമാണ് കൽബ് ഗോത്രം. ആദ്യ കാലഘട്ടത്തിൽ തന്നെ ആവേശത്തോടെ ഇസ്‌ലാം ആശ്ലേഷിച്ച വ്യക്തിയായിരുന്നു – ശറാഫിന്റെ സഹോദരനായ – ദിഹ്‌യ ഇബ്നു ഖലീഫ. (സിയറു അഅ്ലാമിന്നുബലാഅ്: 2: 551)

പിന്നെ എവിടെ നിന്നാണ് “മുഹമ്മദുമായി ബന്ധം പുനസ്ഥാപിച്ച് തടി രക്ഷിക്കാൻ വേണ്ടി അയച്ചു കൊടുത്ത സ്ത്രീ… ” “ഇവരെ ഡിവോഴ്സ് ചെയ്ത് നബി ആ ഗോത്രവും ആയുള്ള കരാർ ഇല്ലാതാക്കി..” എന്ന് തുടങ്ങിയ നാസ്‌തികർ വകയായുള്ള വാചകങ്ങൾ നിവേദനത്തിൽ തിരുകി കയറ്റപ്പെട്ടത് ? വാചകം കേട്ടാൽ തോന്നുക മുഹമ്മദ് നബി (സ) ആ ഗോത്രത്തെ സായുധശക്തിയാൽ കീഴ്‌പ്പെടുത്തി സ്ത്രീകളെ അയച്ചു കൊടുക്കാൻ വല്ല കരാറും ഉണ്ടാക്കിയിട്ടുണ്ട് എന്ന്. ഇങ്ങനെ ചരിത്ര വസ്തുതകൾക്ക് നിരക്കാത്ത കുപ്രചരണങ്ങൾ നിവേദനങ്ങൾക്കിയിൽ തിരുകി കയറ്റി വളരെ സാധാരണമായ ഒരു വിവാഹത്തെ, ഭയത്തിന്റെ നിഴൽപ്പാടിൽ നടന്ന ‘പെണ്ണയച്ച് കൊടുക്കൽ’ ആക്കി മാറ്റാനുള്ള കുൽസിത ശ്രമമാണിത്. യുദ്ധമൊ കലാപമൊ ഒന്നും നിലനിൽക്കാത്ത, നബിയുടെ(സ) ഏറ്റവും പഴയ ആദർശ സ്നേഹികളും സുഹൃത്തുക്കളും നബിക്ക് കൊണ്ടുവന്ന വിവാഹാലോചന, എന്തിൽ നിന്ന് തടി രക്ഷിക്കാനാണ് എന്ന് തെളിവ് സഹിതം വ്യക്തമാക്കണം. വ്യംഗ്യമായ വാചകങ്ങളിലൂടെ ഇല്ലാക്കഥ നിർമ്മിക്കുന്ന സ്ഥിരം പരിപാടി തന്നെയാണ്, നടക്കാത്ത കല്യാണത്തിൽ നാസ്‌തികർ വക ഒരു ഇല്ലാത്ത കരാറും ‘പെണ്ണയച്ചു കൊടുക്കലും’.

അവസാനമായി സൂചിപ്പിക്കട്ടെ, ശറാഫ് ബിൻത് ഖലീഫയെ നബി (സ) വിവാഹം ചെയ്തിട്ടില്ല എന്ന നിവേദനവും ‘ത്വബകാത്തി’ ൽ ഉണ്ട് (ത്വബകാത്തു ഇബ്നു സഅ്ദ്: 8: 161) എങ്കിലും നാസ്‌തികർ അത് കാണില്ല. കാരണം, ‘ത്വബകാത്ത്’ കണ്ടിട്ടുള്ള, വായിച്ചിട്ടുള്ള ഒരു നാസ്‌തികനുമില്ലെന്ന് ഏവർക്കുമറിയാമല്ലൊ. എല്ലാത്തിന്റെയും സ്രോതസ്സുകൾ ഇസ്‌ലാമോഫോബിക്ക് വെബ്സൈറ്റുകൾ മാത്രമാണല്ലൊ.

വിമർശനം:

“പരിപൂർണ്ണതയിലെത്താത്ത മറ്റു ഡിവോഴ്സുകൾ”

ലൈല ബിൻത് ഖുത്യാം… മക്കയിലെ മുഹമ്മദിന്റെ ആദ്യകാലത്ത് ഇസ്ലാം സ്വീകരിച്ച സ്ത്രീ… പിന്നീട് മുഹമ്മദിനെ വിവാഹം കഴിക്കാൻ ആഗ്രഹം പ്രകടിപ്പിച്ചു… മുഹമ്മദ് പതിവു പോലെ അത് സ്വീകരിച്ചു… പക്ഷേ പിന്നീട് മുഹമ്മദിന്റെ ഭാര്യക്കൂട്ടത്തിൽ നിനക്ക് ഒത്തുപോവാൻ സാധിക്കില്ല എന്ന സ്വന്തം കുടുംബത്തിന്റെ വാക്കു കേട്ട് പിൻ വാങ്ങി (തബാരി v9, P 139, ഇബ്നു സാദ് 8:7, P 108 – 109, 231)

(മുഹമ്മദിന്റെ പെണ്ണുങ്ങൾ: നാസ്‌തിക സോഷ്യൽ മീഡിയ തെറിമാല)

മറുപടി:

നാസ്‌തിക സോഷ്യൽ മീഡിയ തെറിമാലകളിൽ ഒന്നായ “മുഹമ്മദിന്റെ പെണ്ണുങ്ങൾ” എന്ന കുറിപ്പിൽ നിന്നുള്ള ചില വരികളെയാണ് നാം തുടർച്ചയായി നിരൂപണം ചെയ്‌ത് കൊണ്ടിരിക്കുന്നത്. മുമ്പ് സൂചിപ്പിച്ചതു പോലെ, സ്ത്രീവിമോചകനായ നബിയെ(സ), സ്ത്രീ പീഢകനും ലമ്പടനുമായി പ്രചരിപ്പിക്കാൻ വേണ്ടി കല്ലുവച്ച നുണകളും, അർദ്ധ സത്യങ്ങളും, ദുർവ്യാഖ്യാനങ്ങളും, വൈരുദ്ധ്യങ്ങളും കൂട്ടി കുഴച്ചുണ്ടാക്കിയ വിധ്വേഷ കഷായമാണ് ലേഖനം.

ലൈല ബിൻത് അൽ ഖുതൈമിനെ വിവാഹം ചെയ്‌തതുമായി ബന്ധപ്പെട്ട് വന്ന കഥ ഇപ്രകാരമാണ്:

ലൈല ബിൻത് അൽ ഖുതൈം വിവാഹാഭ്യർത്ഥനയുമായി നബിയുടെ(സ) അടുക്കൽ വരുകയുണ്ടായി: ഞാൻ എന്റെ സ്വന്തത്തെ വിവാഹത്തിനായി അഭ്യർത്ഥിച്ചു കൊണ്ട് വന്നതാണ്. അതിനാൽ എന്നെ താങ്കൾ വിവാഹം ചെയ്‌താലും. നബി (സ) ചെയ്യാമെന്ന് അംഗീകരിച്ചു. ഉടനെ ലൈല സന്തോഷത്തോടെ തന്നെ നാട്ടുകാരുടെ അടുത്തു ചെന്ന് വിളമ്പരം ചെയ്‌തു: എന്നെ ദൈവദൂതൻ വിവാഹം ചെയ്‌തിരിക്കുന്നു. ഇതു കേട്ടപ്പോൾ നാട്ടുകാർ പറഞ്ഞു: നീ ചെയ്‌തത് എത്ര മോശം. നീ വളരെ അഭിമാന രോഗമുള്ള ഒരു സ്ത്രീയാണ്. നബിക്കാകട്ടെ വേറെയും ഭാര്യമാരുണ്ട്. അവരോട് നീ വഴക്കിലാവുക തന്നെ ചെയ്യും. അപ്പോൾ അദ്ദേഹം നിനക്കെതിരെ പ്രാർത്ഥിച്ചാലോ? അതിനാൽ നീ അദ്ദേഹത്തിൽ നിന്ന് മോചനം വാങ്ങുക. അങ്ങനെ ലൈല നബി(സ)യുടെ അടുത്ത് വന്ന് പറഞ്ഞു: ദൈവദൂതരേ, എന്നെ വിവാഹ മോചനം ചെയ്‌താലും. നബി (സ) പറഞ്ഞു: ശരി, ഞാനിതാ നിന്നെ മോചിപ്പിച്ചിരിക്കുന്നു. അങ്ങനെ ലൈല പിന്നീട് മസ്ഊദിബ്നു ഔസിനെ വിവാഹം ചെയ്യുകയുണ്ടായി…”

(ത്വബകാത്തു ഇബ്നു സഅ്ദ്: 8/150)

നിവേദനം സ്വഹീഹ് ആയ (സ്വീകാര്യയോഗമായ) പരമ്പരയിലൂടെ വന്നിട്ടില്ല. രണ്ട് സ്രോതസ്സുകളാണ് കഥക്കുള്ളത്.

ഒന്ന്:

أخبرنا هشام بن محمد بن السائب عن أبيه عن أبي صالح عن بن عباس قال أقبلت ليلى بنت الخطيم…..

രണ്ട്:

أخبرنا محمد بن عمر حدثنا عبد الله بن جعفر عن بن أبي عون أن ليلى بنت الخطيم

ഹിശാമിബ്നു മുഹമ്മദ് ഇബ്നു സാഇബ് തന്റെ പിതാവിൽ നിന്ന്… ഉദ്ധരിക്കുന്നതാണ് ഒന്നാമത്തെ നിവേദനം. ഹിശാമിബ്നു മുഹമ്മദ് അൽ കൽബിയും അദ്ദേഹത്തിന്റെ പിതാവും നുണ പറയുന്നവരും ദുർബലരുമാണെന്നതിൽ ഹദീസ് നിദാന ശാസ്ത്ര പണ്ഡിതർക്കിടയിൽ ഇരു പക്ഷമില്ല.

ഇമാം ദഹബി എഴുതി: …കൂഫക്കാരനായ ഇദ്ദേഹം ശിഈയും കളവു പറയുന്ന വ്യക്തിയുമായിരുന്നു; അദ്ദേഹത്തിന്റെ പിതാവും തഥൈവ. അഹ്മദിബ്നു ഹമ്പൽ പറഞ്ഞു: …അദ്ദേഹത്തിൽ നിന്ന് ആരും ഹദീസ് ഉദ്ധരിക്കുമെന്ന് ഞാൻ കരുതുന്നില്ല. ഇമാം ദാറകുത്നിയും മറ്റു പണ്ഡിതരും പറഞ്ഞു: ഇദ്ദേഹം കളവു പറയുന്നതായി ആരോപിക്കപ്പെട്ടിരിക്കുന്നതിനാൽ ഇദ്ദേഹത്തിന്റെ ഹദീസുകൾ തള്ളപ്പെട്ടവയാണ്. ഇബ്നു അസാകിർ പറഞ്ഞു: അദ്ദേഹം റാഫിദിയാണ്, വിശ്വസ്തനല്ല. ക്വുർആൻ മൂന്ന് ദിവസം കൊണ്ട് താൻ മനപാഠമാക്കി എന്നെല്ലാം അസത്യ വീര വാദങ്ങൾ മുഴക്കുമായിരുന്നു.

(സിയറു അഅ്ലാമിന്നുബലാഅ്: 10: 101, 102, മീസാനുൽ ഇഅ്തിദാൽ)

മുഹമ്മദ് ഇബ്നു ഉമറാണ് രണ്ടാമത്തെ നിവേദനത്തിന്റെ മൂല നിവേദകൻ. അദ്ദേഹം ദുർബലനാണ്. അഹ്മദിബ്നു ഹമ്പൽ പറഞ്ഞു: മുഹമ്മദിബ്നു ഉമർ വാക്കിദുൽ അസ്‌ലമി നുണയനാണ്; അയാൾ ഹദീസുകളിൽ കോട്ടിമാട്ടുമായിരുന്നു.

യഹ്‌യ പറഞ്ഞു: അയാൾ വിശ്വസ്തനല്ല. അയാളുടെ ഹദീസുകൾ എഴുതിവെക്കാൻ കൊള്ളാത്തത്രയും അവിശ്വസനീയമാണ്.

ഇമാം ബുഖാരി, റാസി, നസാഈ എന്നിവർ പറഞ്ഞു: അയാൾ കളവു കൊണ്ട് ആരോപിതനാണ്. റാസി, നസാഈ എന്നിവർ പറഞ്ഞു: അയാൾ വ്യാജ ഹദീസുകൾ ഉണ്ടാക്കുന്ന വ്യക്തിയായിരുന്നു. ഇമാം ദാറക്കുത്നി പറഞ്ഞു: അയാളിൽ ദൗർബല്യമുണ്ട്. ഇസ്ഹാകിബ്നു റാഹൂയ പറഞ്ഞു: അയാൾ നുണയനാണ്.

(അദ്ദുഅഫാഉ വൽ മത്റൂകീൻ: ഇബ്നുൽ ജൗസി: 3/ 87, അദ്ദുഅഫാഉ സ്സ്വഗീർ: ബുഖാരി: 334, അൽ ജർഹുവതഅദീൽ: അബൂഹാതിം: 8/ 21, അൽ കാമിൽ ഇബ്നു അദിയ്യ്: 7/ 481)

എന്ന് മാത്രമല്ല, ലൈലയുമായുള്ള വിവാഹ കഥ നിവേദനം ചെയ്‌ത മുഹമ്മദ് ഇബ്നു ഉമർ അൽവാക്വിദി എന്ന നിവേദകൻ തന്നെ അത്തരമൊരു വിവാഹം നടന്നിട്ടില്ല എന്ന് നിവേദനത്തിന് തൊട്ടു തുടർച്ചയായി വിശദീകരിച്ചിട്ടുണ്ട്.

أخبرنا محمد بن عمر حدثنا عبد الله بن جعفر عن بن أبي عون أن ليلى بنت الخطيم وهبت نفسها للنبي صلى الله عليه وسلم ووهبن نساء أنفسهن فلم يسمع أن النبي صلى الله عليه وسلم قبل منهن أحدا

(ത്വബകാത്തു ഇബ്നു സഅ്ദ്: 8: 151)

അഥവാ കഥ ഉദ്ധരിച്ച നിവേദകൻ തന്നെ കഥയുടെ സത്യത നിഷേധിച്ചിട്ടുണ്ട് എന്നർത്ഥം. പക്ഷെ ഇബ്നു സഅ്ദിന്റെ ത്വബകാത്തിൽ നിന്ന് വിവാഹ കഥ മാത്രം കാണുകയും തൊട്ടുടനെ അതിന്റെ സത്യതയെ നിരാകരിച്ചു കൊണ്ടുള്ള പ്രസ്‌താവന നാസ്‌തികർ കാണാതെ പോയത് എങ്ങനെയാണ് ? ഉത്തരം ലളിതം, മിഷണറിമാരുടെ പഴയ വിധ്വേഷ വീഞ്ഞ് നാസ്‌തികൻ പുതിയ കുപ്പിയിൽ പുതിയ സ്റ്റിക്കറൊട്ടിച്ച് അവതരിപ്പിച്ചതാണ്. സ്വന്തമായി ഒരു പഠനമോ പരിശോധനയോ നടത്താനുള്ള മാന്യതയൊ കഴിവോ ഉള്ള ഒരാളും ഈ വ്യാജ ഗവേഷകർക്കിടയിൽ ഇല്ല.

ഈ നിവേദനം സ്വഹീഹ് ആണെന്ന് കരുതുക. സ്ത്രീ പീഢകനും ലൈംഗിക കൊതിയനുമായ ഒരാളുടെ ചിത്രമല്ല ഈ കഥയിൽ ആർക്കും ദർശിക്കാൻ കഴിയുന്നത് എന്ന് സാന്ദർഭികമായി നാം ശ്രദ്ധയിലേക്ക് കൊണ്ടു വരട്ടെ. മുഹമ്മദ് നബിയല്ല(സ) ലൈലയാണ് വിവാഹം ആഗ്രഹിച്ചു കൊണ്ട് അഭ്യർത്ഥനയുമായി വന്നത്. തന്റെ ന്യൂനതകളും കോപ പ്രകൃതിയും പരിഗണിച്ച് വിവാഹ മോചനത്തെ സംബന്ധിച്ച് ആലോചിച്ചതും ലൈല തന്നെ. തനിക്ക് വിവാഹ മോചനം വേണമെന്ന് ആവശ്യപ്പെട്ട ഉടനെ വിവാഹ മോചനം നൽകിയത് ആ അറബ് ഉപഭൂഖണ്ഡത്തിന്റെ ആത്മീയവും ഭൗതികവുമായ നേതാവായ പ്രവാചകൻ (സ) !! ഒഴികഴിവുകൾ പറഞ്ഞില്ല, ബലാൽകാരങ്ങളില്ല, പ്രകോപനങ്ങളൊ ഭീഷണികളൊ ഇല്ല, കേസും കോടതി വിചാരണകളും ഇല്ല. തന്റെ ഇഷ്ടാനുസാരം സ്വാതന്ത്ര്യത്തിന്റെ ചക്രവാള വിശാലതയിൽ പാറി പറക്കുന്ന സ്ത്രീകളുടെ ചിത്രം മാത്രം… വിവാഹവും വിവാഹ മോചനവും ഇണയും തുണയുമെല്ലാം അവളുടെ തൃപ്തിക്ക് വിട്ടു കൊടുത്ത ഒരു സ്ത്രീവിമോചനത്തിന്റെ സാമ്രാട്ട്; അതായിരുന്നു മുഹമ്മദ് നബി (സ).

വിമർശനം:

“പരിപൂർണ്ണതയിലെത്താത്ത മറ്റു ഡിവോഴ്സുകൾ”

ഖൗല ബിൻത് ഹുധ്യാൽ… ഒരു ക്രിസ്ത്യൻ ഗോത്രത്തിലെ രാജകുമാരി… മുഹമദുമായി വിവാഹം നടത്തിയതിന് ശേഷം മദീനയിലേക്കുള്ള യാത്രാമധ്യേ അവർ മരിച്ചു (ആത്മഹത്യയാവാം) (അൽ തബാരി – വോളിയം 9, പേജ് 139, 166; ഇബ്നു സാദ് 8:116)

(മുഹമ്മദിന്റെ പെണ്ണുങ്ങൾ: നാസ്‌തിക സോഷ്യൽ മീഡിയ തെറിമാല)

മറുപടി:

നാസ്‌തിക സോഷ്യൽ മീഡിയ തെറിമാലകളിൽ ഒന്നായ “മുഹമ്മദിന്റെ പെണ്ണുങ്ങൾ” എന്ന കുറിപ്പിൽ നിന്നുള്ള ചില വരികളെയാണ് നാം തുടർച്ചയായി നിരൂപണം ചെയ്ത് കൊണ്ടിരിക്കുന്നത്. മുമ്പ് സൂചിപ്പിച്ചതു പോലെ, സ്ത്രീവിമോചകനായ നബിയെ(സ), സ്ത്രീ പീഢകനും ലമ്പടനുമായി പ്രചരിപ്പിക്കാൻ വേണ്ടി കല്ലുവച്ച നുണകളും, അർദ്ധ സത്യങ്ങളും, ദുർവ്യാഖ്യാനങ്ങളും, വൈരുദ്ധ്യങ്ങളും കൂട്ടി കുഴച്ചുണ്ടാക്കിയ വിധ്വേഷ കഷായമാണ് ലേഖനം.

ഖൗല ബിൻത് അൽ ഹുദൈലുമായുള്ള നബിയുടെ(സ) വിവാഹത്തെ സംബന്ധിച്ച് ഇബ്നു സഅ്ദ് തന്റെ ത്വബകാത്തിൽ ഉദ്ധരിച്ച സനദ് (നിവേദക പരമ്പര) ഇപ്രകാരമാണ്:

أخبرنا هشام بن محمد حدثني الشرقي بن القطامي

1. സനദിലെ ഹിശാമിബ്നു മുഹമ്മദ് അൽ കൽബി നുണയനും ദുർബലനുമാണെന്നതിൽ ഹദീസ് നിദാന ശാസ്ത്ര പണ്ഡിതർക്കിടയിൽ ഇരു പക്ഷമില്ല.

ഇമാം ദഹബി എഴുതി: …കൂഫക്കാരനായ ഇദ്ദേഹം ശിഈയും കളവു പറയുന്ന വ്യക്തിയുമായിരുന്നു; അദ്ദേഹത്തിന്റെ പിതാവും തഥൈവ. അഹ്മദിബ്നു ഹമ്പൽ പറഞ്ഞു: …അദ്ദേഹത്തിൽ നിന്ന് ആരും ഹദീസ് ഉദ്ധരിക്കുമെന്ന് ഞാൻ കരുതുന്നില്ല. ഇമാം ദാറകുത്നിയും മറ്റു പണ്ഡിതരും പറഞ്ഞു: ഇദ്ദേഹം കളവു പറയുന്നതായി ആരോപിക്കപ്പെട്ടിരിക്കുന്നതിനാൽ ഇദ്ദേഹത്തിന്റെ ഹദീസുകൾ തള്ളപ്പെട്ടവയാണ്. ഇബ്നു അസാകിർ പറഞ്ഞു: അദ്ദേഹം റാഫിദിയാണ്, വിശ്വസ്തനല്ല. ക്വുർആൻ മൂന്ന് ദിവസം കൊണ്ട് താൻ മനപാഠമാക്കി എന്നെല്ലാം അസത്യ വീര വാദങ്ങൾ മുഴക്കുമായിരുന്നു. (സിയറു അഅ്ലാമിന്നുബലാഅ്: 10:101,102, മീസാനുൽ ഇഅ്തിദാൽ)

2. മറ്റൊരു നിവേദകനായ ‘ശർകിയ്യിബ്നു ക്വത്വാമി’ യും ദുർബലനാണ്.

ഇമാം ദഹബി എഴുതി: സ്വഹീഹായ ഹദീസുകൾക്ക് വിരുദ്ധമായി പത്തോളം അതി ദുർബലമായ (മുൻകറായ) ഹദീസുകൾ ശർകിയ് ഉദ്ധരിക്കുമായിരുന്നു. അദ്ദേഹം ദുർബലനാണെന്ന് സകരിയ്യ അസ്സാജിയും അഭിപ്രായപ്പെട്ടിട്ടുണ്ട്. (മീസാനുൽ ഇഅ്തിദാൽ: 2: 268)

وقال النديم في الفهرست: اسمه الوليد بن الحصين قرأت بخط اليوسفي كان كذابا ويكنى أبا المثنى.))

ഇബ്നു നദീം തന്റെ ഫഹ്റസത്തിൽ പറഞ്ഞിരിക്കുന്നത് ശർകിയ് കളവു പറയുന്ന വ്യക്തിയാണ് എന്നാണ്. (ലിസാനുൽ മീസാൻ: ഇബ്നു ഹജർ:4: 241)

3. മാത്രമല്ല, ശർകിയ് പ്രവാചകാലഘട്ടകാരനല്ല; താൻ ആരിൽ നിന്നാണ് ഈ കഥ കേട്ടതെന്ന് വ്യക്തമാക്കിയിട്ടുമില്ല. അഥവാ പരമ്പര കണ്ണി മുറിഞ്ഞതാണ്.

ഈ മൂന്ന് കാരണങ്ങളാൽ തന്നെ ഇത്തരമൊരു വിവാഹം പ്രവാചക ജീവിതത്തിൽ നടന്നിട്ടില്ലെന്നും, ചില തൽപര കക്ഷികൾ കെട്ടിച്ചമച്ചുണ്ടാക്കിയ വ്യാജകഥ മാത്രമാണിതെന്നും സുതരാം വ്യക്തമാണ്.

സാങ്കൽപ്പിക വിവാഹത്തിലെ, ഭാവനാത്മകമായ “പ്രവാചക പത്നി”, വഴിയിൽ വെച്ച് മരണപ്പെട്ടു (فهلكت في الطريق) എന്ന ഒരു വ്യാജ നിവേദനം പൊക്കിപ്പിടിച്ച്, “ആത്മഹത്യയാവാം” എന്ന മനസ്സിലെ വിഷവിത്ത് തിരുകി കയറ്റി, മുതല കണ്ണീരൊഴുക്കി… നബി വിധ്വേഷം ഞെക്കി തുറിപ്പിച്ചുണ്ടാക്കാനുള്ള കഷ്ടപ്പാടെത്രയാണ്!!

വിമർശനം:

“…വിവാഹം ചെയ്ത് ഭോഗിച്ചു… 4 എണ്ണത്തിനെ ബാലിശമായ കാരണങ്ങൾ പറഞ്ഞ് ഡിവോഴ്സ് ചെയ്തു…. അധികം പേർക്കും അറിയാത്ത (മുഹമ്മദ് നബി) മൊഴിചൊല്ലിയ സ്ത്രീകളുടെ പേരുകൾ ഇവയാണ്…

3) ഫാത്തിമ അൽ അലിയാ ബിൻത് സാബിയാൻ അൽ ദഹാക്ക്…. മറ്റൊരു പുരുഷനെ ഒളിഞ്ഞു നോക്കി എന്നും പറഞ്ഞ് മൊഴി ചൊല്ലി (തബാരി v9, P 138; തബാരി v39 P 186 – 188)

4) അമ്ര ബിൻത് യാസിദ്…. (ഇബ്നു ഇഷാഖ്, സീറത്തുൽ റസൂലള്ളാ P 155)”

(മുഹമ്മദിന്റെ പെണ്ണുങ്ങൾ: നാസ്തിക സോഷ്യൽ മീഡിയ തെറിമാല)

മറുപടി:

നാസ്തിക സോഷ്യൽ മീഡിയ തെറിമാലകളിൽ ഒന്നായ “മുഹമ്മദിന്റെ പെണ്ണുങ്ങൾ” എന്ന കുറിപ്പിൽ നിന്നുള്ള ചില വരികളെയാണ് നാം തുടർച്ചയായി നിരൂപണം ചെയ്ത് കൊണ്ടിരിക്കുന്നത്. മുമ്പ് സൂചിപ്പിച്ചതു പോലെ, കല്ലുവച്ച നുണകളും, അർദ്ധ സത്യങ്ങളും, ദുർവ്യാഖ്യാനങ്ങളും, വൈരുദ്ധ്യങ്ങളും നിറഞ്ഞതാണ് ലേഖനം.

“കിലാബ് ഗോത്രക്കാരിയായ ഒരു സ്ത്രീയെ നബി (സ) വിവാഹം ചെയ്തു എന്ന് പറയപ്പെടുന്നു… എന്താണ് ആ സ്ത്രീയുടെ നാമം എന്ന കാര്യത്തിൽ പണ്ഡിതന്മാർക്കിടയിൽ ധാരാളം അഭിപ്രായ വ്യത്യാസങ്ങളുണ്ട്. ചിലർ പറഞ്ഞു: അവരുടെ നാമം ഫാത്വിമ ബിൻത് ദഹ്ഹാക് ഇബ്നു സുഫ്യാൻ അൽ കിലാബിയ്യ എന്നാണ്. വേറെ ചിലർ പറഞ്ഞു: അവരുടെ നാമം അംറ ബിൻത് യസീദ് ബിൻ ഉബൈദ് ബിൻ റുവാസ് ബിൻ കിലാബ് ബിൻ റബീഅ ബിൻ ആമിർ എന്നാണ്. മറ്റൊരാൾ പറയുന്നത് അവരുടെ നാമം ആലിയ ബിൻത് ളബ്യാൻ ബിൻ അംറ് ബിൻ ഔഫ് ബിൻ കഅ്ബ് ബിൻ അബ്ദ് ബിൻ അബൂബകർ ബിൻ കിലാബ് എന്നാണ്. ഒരാൾ പാഞ്ഞു: അവർ സബാ ബിൻത് സുഫ്യാൻ ബിൻ ഔഫ് ബിൻ കഅ്ബ് ബിൻ അബ്ദ് ബിൻ അബൂബകർ ബിൻ കിലാബ് എന്നാണ്. ഇക്കാര്യത്തിൽ നാം കേട്ട നിവേദനങ്ങളെല്ലാം നാം ഇവിടെ എഴുതി എന്നു മാത്രം. ചിലർ പറയുന്നു: ഒരൊറ്റ കിലാബ് കാരിയെ മാത്രമെ നബി (സ) വിവാഹം കഴിച്ചിട്ടുള്ളു. അവരുടെ പേരിന്റെ കാര്യത്തിൽ സംശയങ്ങളുണ്ടായതാണ്…” (ത്വബകാതു ഇബ്നു സഅ്ദ്: 8/112)

ഇതാണ് കിലാബ് കാരിയായ സ്ത്രീയുമായി ബന്ധപ്പെട്ട കഥയുടെ അവസ്ഥ !! അത്തരമൊരു വിവാഹം നടന്നിട്ടുണ്ടൊ ഇല്ലേ ? നടന്നെങ്കിൽ അവരുടെ പേരെന്താണ്? ആ പേരുകൾ എല്ലാം ഒരേ ആളാണൊ അതൊ വ്യത്യസ്തരായ സ്ത്രീകളാണൊ എന്നൊന്നും ഉള്ളതിന് കൃത്യമായ ഒരു തെളിവുമില്ല. വിമർശകർ തന്നെ – മുഹമ്മദിന്റെ പെണ്ണുങ്ങൾ എന്ന ലേഖനത്തിൽ – ഉദ്ധരിച്ച പേര് “ഫാത്തിമ അൽ അലിയാ ബിൻത് സാബിയാൻ അൽ ദഹാക്ക്” എന്നാണ് !! ഇത് രണ്ട് പേരുകൾ കൂട്ടിയോജിപ്പിച്ച ഒരു പേരാണ്. 1. ഫാത്വിമ ബിൻത് ദഹ്ഹാക് 2. ആലിയ ബിൻത് ളബ്യാൻ എന്ന രണ്ടു പേരുകൾ ചേർത്ത് “ഫാത്തിമ അൽ അലിയാ ബിൻത് സാബിയാൻ അൽ ദഹാക്ക്” എന്നായി.

കഥയുടെ ഉള്ളടക്കം ഉമൈമയുടെ കഥക്ക് സമാനം തന്നെ ! പ്രവാചകൻ (സ) ‘ഫാത്വിമ ബിൻത് ദഹ്ഹാക്’ അല്ലെങ്കിൽ ‘ആലിയ ബിൻത് ളബ്യാനെ’ അല്ലെങ്കിൽ ‘അംറ ബിൻത് യസീദിനെ’ അതോ ‘സബാ ബിൻത് സുഫ്യാനെ’ (!) വിവാഹം ചെയ്യുകയും അവരുടെ അടുത്ത് പ്രവേശിക്കുകയും ചെയ്തപ്പോൾ അവർ അദ്ദേഹത്തിൽ നിന്ന് അല്ലാഹുവിൽ ശരണം തേടി. അപ്പോൾ നബി (സ) അവരെ വീട്ടിലേക്ക് പൊയ്കൊള്ളാൻ നിർദ്ദേശിച്ച് സ്വതന്ത്രരാക്കി.

മുമ്പ് പല ലേഖനങ്ങളിൽ സൂചിപ്പിച്ചതു പോലെ ഉമൈമ എന്ന ഭാര്യയുടെ കഥ പല നാമങ്ങളിലായി പുനർ നിർമ്മിച്ചെടുത്ത വ്യാജ കഥകളും വ്യാജ ഭാര്യമാരുമാണ് ഇവ/ ഇവർ എല്ലാം.

وأشار ابن سعد إلى أنها واحدة اختلف في اسمها ، والصحيح أن التي استعاذت منه هي الجونية . وروى ابن سعد من طريق سعيد بن عبد الرحمن بن أبزى قال : لم تستعذ منه امرأة غيرها . ഇബ്നു ഹജർ (റ) പറഞ്ഞു: ഒരൊറ്റ ഭാര്യയുടെ കാര്യത്തിൽ നടന്നതാണ് ഈ ശരണ തേട്ടവും വിവാഹ മോചനവും; അവരുടെ പേരെന്താണ് എന്ന കാര്യത്തിൽ അഭിപ്രായ വ്യത്യാസം വന്നു എന്ന് മാത്രം. നബിയിൽ(സ) നിന്ന് ശരണം തേടിയത് (ഉമൈമ) ജുവനിയ ആണെന്നതാണ് സ്വഹീഹ് (വിശ്വസ്‌തമായ ഹദീസിലൂടെ സ്ഥാപിതമായ വസ്തുത). സഈദിബ്നു അബ്ദുർറഹ്‌മാൻ ബിൻ അബ്സാ പറഞ്ഞതായി ഇബ്നു സഅ്ദ് ഉദ്ധരിച്ചിരിക്കുന്നു: ഉമൈമയല്ലാതെ മറ്റൊരു സ്ത്രീയും നബിയിൽ നിന്ന് അല്ലാഹുവിൽ ശരണം തേടിയിട്ടില്ല. (ഫത്ഹുൽ ബാരി: 9:269)

ത്വബ്‌രി, ഇബ്നു സഅ്ദ്, ദഹബി പോലുള്ള എല്ലാ ചരിത്രകാരന്മാരും ഈ കഥകൾ ഉദ്ധരിക്കുന്നത് മുഹമ്മദ് ഇബ്നു ഉമർ ഇബ്നുൽ വാക്വിദ് എന്ന ചരിത്രകാരനിൽ നിന്നാണ്. കഥകളുടെ വ്യത്യസ്ത നിവേദനങ്ങളുടെ സനദുകൾ കാണുക:

1. أَخْبَرَنَا مُحَمَّدُ بْنُ عمر. حَدَّثَنَا مُحَمَّدُ بْنُ عَبْدِ اللَّهِ عَنِ الزُّهْرِيِّ قَالَ: … 2. أَخْبَرَنَا مُحَمَّدُ بْنُ عُمَرَ. حَدَّثَنِي مُحَمَّدُ بْنُ عَبْدِ اللَّهِ عَنِ الزُّهْرِيِّ عَنْ عُرْوَةَ عَنْ عَائِشَةَ قَالَتْ: … 3. أَخْبَرَنَا مُحَمَّدُ بْنُ عُمَرَ. حَدَّثَنَا عَبْدُ اللَّهِ بْنُ جَعْفَرٍ عَنْ عَبْدِ الْوَاحِدِ بن أبي عون عن ابْنِ مَنَّاحٍ قَالَ: … 4. أَخْبَرَنَا مُحَمَّدُ بْنُ عُمَرَ. حَدَّثَنَا عَبْدُ اللَّهِ بْنُ سُلَيْمَانَ عَنْ عَمْرِو بْنِ شُعَيْبٍ عَنْ أَبِيهِ عَنْ جَدِّهِ قَالَ: … 5. أَخْبَرَنَا مُحَمَّدُ بْنُ عُمَرَ. أَخْبَرَنَا عَبْدُ اللَّهِ بْنُ جَعْفَرٍ عَنْ مُوسَى بْنِ سَعِيدٍ وَابْنِ أَبِي عَوْنٍ قَالا: …

എല്ലാ നിവേദക പരമ്പരകളും ആരംഭിക്കുന്നത് തന്നെ മുഹമ്മദ് ഇബ്നു ഉമർ ഇബ്നുൽ വാക്വിദ് എന്ന ചരിത്രകാരനിൽ നിന്നാണ്. അദ്ദേഹത്തെ സംബന്ധിച്ച ഹദീസ് – ചരിത്ര നിദാന ശാസ്ത്ര പണ്ഡിതന്മാരുട അഭിപ്രായം ഇവിടെ ആവർത്തിക്കട്ടെ:

അഹ്മദിബ്നു ഹമ്പൽ പറഞ്ഞു: മുഹമ്മദിബ്നു ഉമർ വാക്കിദുൽ അസ്‌ലമി നുണയനാണ്; അയാൾ ഹദീസുകളിൽ കോട്ടിമാട്ടുമായിരുന്നു.

യഹ്‌യ പറഞ്ഞു: അയാൾ വിശ്വസ്തനല്ല. അയാളുടെ ഹദീസുകൾ എഴുതിവെക്കാൻ കൊള്ളാത്തത്രയും അവിശ്വസനീയമാണ്.

ഇമാം ബുഖാരി, റാസി, നസാഈ എന്നിവർ പറഞ്ഞു: അയാൾ കളവു കൊണ്ട് ആരോപിതനാണ്.

റാസി, നസാഈ എന്നിവർ പറഞ്ഞു: അയാൾ വ്യാജ ഹദീസുകൾ ഉണ്ടാക്കുന്ന വ്യക്തിയായിരുന്നു.

ഇമാം ദാറക്കുത്നി പറഞ്ഞു: അയാളിൽ ദൗർബല്യമുണ്ട്.

ഇസ്ഹാകിബ്നു റാഹൂയ പറഞ്ഞു: അയാൾ നുണയനാണ്.

(അദ്ദുഅഫാഉ വൽ മത്റൂകീൻ: ഇബ്നുൽ ജൗസി: 3 / 87, അദ്ദുഅഫാഉ സ്സ്വഗീർ: ബുഖാരി: 334, അൽ ജർഹുവതഅദീൽ: അബൂഹാതിം: 8/21, അൽ കാമിൽ ഇബ്നു അദിയ്യ്: 7/ 481)

സാന്ദർഭികമായി ചില ചോദ്യങ്ങൾ ആവർത്തിക്കട്ടെ ?

കഥ യാഥാർഥ്യമാണെങ്കിൽ തന്നെ നബി (സ) പെണ്ണുങ്ങളെ കെട്ടി ഭോഗിച്ചു, ബാലിശമായ കാരണങ്ങൾ പറഞ്ഞ് മൊഴി ചൊല്ലി എന്നതെല്ലാം നാസ്തിക നുണകൾ മാത്രമല്ലെ ?! ലൈംഗിക ബന്ധം നടന്നിട്ടില്ല എന്നതും നബിയല്ല, നവവധുവാണ് ഇഷ്ടമില്ലായ്മ പ്രകടിപ്പിച്ചത് എന്നതും എന്തുകൊണ്ട് മറച്ചു പിടിക്കുന്നു ?! താൽപര്യമില്ലാത്ത വിവാഹ ബന്ധത്തിൽ നിന്ന് സ്വഭാര്യയെ മോചിപ്പിച്ച നബി, സ്ത്രീകളോട് പുലർത്തിയ മാന്യതയും ദയാപരതയും ലൈംഗിക വിശുദ്ധിയും എന്തുകൊണ്ട് കട്ടുമുക്കുന്നു ?!!

ചില (വ്യാജ) നിവേദനങ്ങളിൽ, ഫാതിമയെ വിവാഹ മോചനം ചെയ്യാനുണ്ടായ സാഹചര്യം അവർ സ്ഥിരമായി പുരുഷന്മാരെ ഒളിഞ്ഞു നോക്കാറുണ്ടായിരുന്നു എന്നും, ഇത് നബി (സ) നേരിട്ട് കാണുകയുണ്ടായി എന്നും വന്നിട്ടുണ്ട്. ഇതിനെ നിസ്സാര കാര്യമെന്നോണം നാസ്തികൻ ഇപ്രകാരം കുറിച്ചിട്ടിരിക്കുന്നത് വായിക്കാം: “മറ്റൊരു പുരുഷനെ ഒളിഞ്ഞു നോക്കി എന്നും പറഞ്ഞ് മൊഴി ചൊല്ലി”… ! പുരുഷന്മാരെ ഒളിഞ്ഞു നോക്കുന്ന ഒരു സ്ത്രീയെ ഭാര്യയായി വെക്കാൻ നാസ്തികർ തയ്യാറാകുമോ എന്നു കൂടി നാസ്തിക കോപ്പിയടിയന്മാർ വ്യക്തമാക്കേണ്ടതുണ്ട്.

വിമർശനം:

” …വിവാഹം ചെയ്ത് ഭോഗിച്ചു… 4 എണ്ണത്തിനെ ബാലിശമായ കാരണങ്ങൾ പറഞ്ഞ് ഡിവോഴ്സ് ചെയ്തു…. അധികം പേർക്കും അറിയാത്ത (മുഹമ്മദ് നബി) മൊഴിചൊല്ലിയ സ്ത്രീകളുടെ പേരുകൾ ഇവയാണ്…

2) മുലൈഖ… തന്റെ പിതാവിനേയും മറ്റും മുഹമ്മദ് നേരിട്ടാണ് കൊന്നത് എന്ന് തിരിച്ചറിഞ്ഞതിനു ശേഷം ഡിവോഴ്സ് ആവശ്യപ്പെട്ടു.. മുഹമ്മദ് കൊടുത്തു,… (തബാരി v39, P 165)”

(മുഹമ്മദിന്റെ പെണ്ണുങ്ങൾ: നാസ്‌തിക സോഷ്യൽ മീഡിയ തെറിമാല)

മറുപടി:

നാസ്‌തിക സോഷ്യൽ മീഡിയ തെറിമാലകളിൽ ഒന്നായ “മുഹമ്മദിന്റെ പെണ്ണുങ്ങൾ” എന്ന കുറിപ്പിൽ നിന്നുള്ള ചില വരികളാണ് ഈ വായിച്ചത്. മുമ്പ് സൂചിപ്പിച്ചതു പോലെ, കല്ലുവച്ച നുണകളും, അർദ്ധ സത്യങ്ങളും, ദുർവ്യാഖ്യാനങ്ങളും, വൈരുദ്ധ്യങ്ങളും നിറഞ്ഞതാണ് ലേഖനം.

1. മുലൈഖ ബിൻത് കഅ്ബിനെ വിവാഹ മോചനം ചെയ്തതുമായി ബന്ധപ്പെട്ട് വിമർശകർ അവലംബിക്കുന്ന നിവേദനം വളരെയേറെ ദുർബലമാണ്. നിവേദനത്തിന്റെ ഉള്ളടക്കം ഇപ്രകാരമാണ്:

تزوج النبي صلى الله عليه وسلم مليكة بنت كعب، وكانت تُذكر بجمال بارع، فدخلَت عليها عائشة، فقالت لها: أما تستحين أن تنكحي قاتِلَ أبيك، فاستعاذت من رسول الله صلى الله عليه وسلم، فطلَّقها، فجاء قومها إلى النبي صلى الله عليه وسلم، فقالوا: يا رسول الله، إنها صغيرة، وإنها لا رأيَ لها وإنها خدعت فارتجعها، فأبى رسول الله صلى الله عليه وسلم، وكان أبوها قتل في يوم فتح مكة، قتَله خالد بن الوليد بالخندمة

മക്കാ വിജയ വേളയിൽ നബിയുടെ(സ) ശിഷ്യൻ ഖാലിദിബ്നു വലീദ് കഅ്ബിനെ വധിക്കുകയുണ്ടായി. അദ്ദേഹത്തിന്റെ മകൾ മുലൈഖ ബിൻത് കഅ്ബിനെ നബി (സ) വിവാഹം ചെയ്തു. അവരോട് അസൂയ തോന്നിയ ആഇശ (റ) അവരോട് ഇപ്രകാരം പറഞ്ഞു: നിങ്ങളുടെ പിതാവിന്റെ ഘാതകനെ വിവാഹം ചെയ്യാൻ നിങ്ങൾക്ക് മടിയില്ലെ ? അങ്ങനെ അവരുടെ അടുത്തേക്ക് നബി (സ) വന്നപ്പോൾ മുലൈഖ, നബിയിൽ (സ) നിന്ന് ശരണം തേടി. അപ്പോൾ നബി (സ) അവരെ വിവാഹ മോചനം നൽകി സ്വാതന്ത്രയാക്കി. (ത്വബകാതു ഇബ്നു സഅ്ദ്: 8 /148, താരീഖു ദ്ദഹബി: 1 /335)

ഈ നിവേദനത്തിൽ എവിടെയാണ് – നാസ്‌തികർ വാദിക്കുന്നതു പോലെ – നബി മുലൈഖയെ ഭോഗിച്ചു എന്നുള്ളത് ?! ഭോഗിക്കുക പോയിട്ട് സ്വന്തം ഭാര്യയായിരുന്നിട്ടും, തന്നെ ഇഷ്ടമില്ലാത്തതു കൊണ്ട് നബി (സ) മുലൈഖയെ സ്പർശിക്കുക പോലുമുണ്ടായില്ല. മുലൈഖയെ പിടിച്ചു വെക്കുകയൊ ബലം പ്രയോഗിക്കുകയൊ ചെയ്തില്ല. സൗമ്യതയോടെ വീട്ടിലേക്ക് പൊയ്കൊള്ളാൻ പറഞ്ഞു! ഇതാണൊ ഒരു സ്ത്രീപീഢകന്റെ ചിത്രം ?!

തനിക്ക് ഭർത്താവിനെ ഇഷ്ടമില്ലെന്ന് പറയുന്ന സ്ത്രീക്ക് യാതൊരു സങ്കോചമോ വൈമനസ്യമോ കൂടാതെ വീട്ടിലേക്ക് പോകാനും വിവാഹ മോചനം ചെയ്യാനുമുള്ള അവകാശം നൽകുകയാണ് നബി (സ) ഇവിടെ ചെയ്യുന്നതായി നിവേദനത്തിൽ പ്രസ്‌താവിക്കുന്നത്. ഇത് സ്ത്രീ പീഢകരുടെ സ്വഭാവമല്ല, സ്ത്രീവിമോചകന്റേതാണ്.

“ബാലിശമായ കാരണങ്ങൾ പറഞ്ഞ് ഡിവോഴ്സ് ചെയ്തു” എന്ന ആരോപണവും നുണ തന്നെയെന്ന് ഈ നിവേദനവും വ്യക്തമായി തെളിയിക്കുന്നു. മുലൈഖയാണ് നബിയിൽ (സ) താൽപര്യമില്ലെന്ന അഭിപ്രായം അവതരിപ്പിച്ചത്, നബിയല്ല (സ). തന്നെ ഇഷ്ടമല്ലാത്ത ഒരാളെ മോചിപ്പിക്കുക മാത്രമാണ് നബി (സ) ചെയ്തത്.

2. ത്വബ്‌രിയും ഇബ്നു സഅ്ദും എല്ലാം ഈ കഥ ഉദ്ധരിച്ചിരിക്കുന്നത് ചരിത്രകാരനായ വാക്വിദിയിൽ നിന്നാണ്. നിവേദനം ഉദ്ധരിച്ച തൊട്ടുടനെ തന്നെ കഥയുടെ അസത്യത വാക്വിദിയും, അദ്ദേഹത്തിൽ നിന്ന് ഇമാം ത്വബ്‌രിയും ഇബ്നു സഅ്ദും എല്ലാവരും വ്യക്തമാക്കിയിട്ടുമുണ്ട്. അതു പക്ഷെ നാസ്‌തിക പകർത്തെഴുത്തുകാർ കണ്ടിട്ടില്ല. കാരണം, ഈച്ച കോപ്പിയടിക്കാൻ ഉപയോഗിച്ച മിഷണറി ദുർവ്യാഖ്യാന ലേഖനങ്ങളിൽ അതു കാണില്ല; അതവർ ഉദ്ധരിക്കില്ല എന്നതിൽ അത്ഭുതപ്പെടാനൊന്നുമില്ല. നിവേദനത്തിന്റെ തൊട്ടുടനെ നിവേദകൻ തന്നെ പ്രസ്‌താവിക്കുന്നത് ഇപ്രകാരമാണ്.

“വാക്വിദി പറഞ്ഞു: നമ്മുടെ സഹചരിത്രകാരന്മാർ ഇതിനെ നിശിതമായി നിഷേധിക്കുന്നു. നബി (സ) ഒരിക്കലും ഒരു കിനാനക്കാരിയെയും വിവാഹം ചെയ്തിട്ടില്ല എന്നതാണ് വസ്തുത.” (മുൻതഖബു മിൻ ദൈലിൽ മിദ്‌യൽ 89, താരീഖുത്വബ്‌രി: 11:596, അൽ ഇസ്വാബ: ഇബ്നു ഹജർ: 8: 320)

3. مما يضعف هذا الحديث، ذِكر عائشة أنها قالت: ألا تستحين، وعائشةُ لم تكن مع النبي صلى الله عليه وسلم عام الفتح.

നിവേദനം ദുർബലമാണെന്നതിനുള്ള മറ്റൊരു തെളിവാണ്, അവരോട് അസൂയ തോന്നിയ ആഇശ (റ) അവരോട് ഇപ്രകാരം ചോദിച്ചു എന്ന ഭാഗം: “നിങ്ങളുടെ പിതാവിന്റെ ഘാതകനെ വിവാഹം ചെയ്യാൻ നിങ്ങൾക്ക് മടിയില്ലെ ?”. മക്കാ വിജയ ദിവസം ആഇശ (റ) നബിയോടൊപ്പം (സ) ഉണ്ടായിരുന്നില്ല. പിന്നെ എങ്ങനെയാണ് മുലൈഖ ബിൻത് കഅ്ബിനോട് ആഇശ (റ) ഇപ്രകാരം ചോദിക്കുക ?! 500 കിലോമീറ്ററുകൾക്കപ്പുറം മദീനയിലുള്ള ആഇശ (റ), നവ വധുവായ മുലൈഖയുമായി എങ്ങനെ സംഭാഷണം നടത്തും ?! ഇതിന്റെ അടിസ്ഥാനത്തിൽ തന്നെ നിവേദനം ദുർബലമാണെന്ന് പണ്ഡിതന്മാർ വ്യക്തമാക്കിയിട്ടുണ്ട്.

4. നബിയുടെ (സ) പത്നിമാരിൽ, നബിയിൽ നിന്നും ശരണം തേടിയത് ഒരൊറ്റ പത്നി മാത്രമാണ്. അത് ഉമൈമ ബിൻത് നുഅ്മാനാണ് (ബുഖാരി: 5254).

ഉമൈമയുടെ ചരിത്രം വിശദമായി മറ്റൊരു ലേഖനത്തിൽ പ്രതിപാദിച്ചിട്ടുള്ളതിനാൽ ഇവിടെ ആവർത്തിക്കുന്നില്ല:

ഉമൈമയുടെ ഈ സംഭവം മുലൈഖ ബിൻത് കഅ്ബിലേക്കും മറ്റു പല സ്ത്രീകളിലേക്കും ചേർത്തി കൊണ്ട് ചില നിവേദനങ്ങൾ രേഖപ്പെടുത്തപ്പെട്ടിട്ടുണ്ട്. ഇത് നിവേദകന്മാരിൽ നിന്നും സംഭവിച്ച ഓർമ്മ പിശകു മാത്രമാണ്. നാമങ്ങൾ പരസ്പരം കൂടി കലർന്നു കൊണ്ടുള്ള ചില ആശയക്കുഴപ്പങ്ങൾ ചില ചരിത്രകാരന്മാരിൽ നിന്ന് സംഭവിച്ചു എന്നതിനുള്ള വ്യക്തമായ തെളിവാണ് മുലൈഖ ബിൻത് കഅ്ബിന്റെയും, ഉമൈമ ബിൻത് നുഅ്മാന്റെയും കഥകളുടെ ഉള്ളടക്കത്തിലെ സാമ്യത.

5. നിവേദനത്തിന്റെ പരമ്പരകൾ ദുർബലമാണ് എന്നതും ഈ കഥയുടെ അസത്യതയെ തെളിയിക്കുന്നതാണ്.

ഒന്നാമത്തെ പരമ്പര: ذكر ابن عمر أن عبد العزى بن الجندعي حدثه عن أبيه عن عطاء بن يزيد الجندعي قال

രണ്ടാമത്തെ പരമ്പര: قال ابن عمر وحدثني محمد بن عبد الله عن الزهري مثل ذلك

മൂന്നാമത്തെ പരമ്പര: أخبرنا محمد بن عمر حدثني أبو معشر قال

മൂന്നു പരമ്പരകളിലേയും റാവിയായ മുഹമ്മദിബ്നു ഉമർ വാക്കിദുൽ അസ്‌ലമി (സത്യസന്ധതയിൽ) ദുർബലനാണ്. അഹ്മദിബ്നു ഹമ്പൽ പറഞ്ഞു: മുഹമ്മദിബ്നു ഉമർ വാക്കിദുൽ അസ്‌ലമി നുണയനാണ്; അയാൾ ഹദീസുകളിൽ കോട്ടിമാട്ടുമായിരുന്നു.

യഹ്‌യ പറഞ്ഞു: അയാൾ വിശ്വസ്ഥനല്ല. അയാളുടെ ഹദീസുകൾ എഴുതിവെക്കാൻ കൊള്ളാത്തത്രയും അവിശ്വസനീയമാണ്. ഇമാം ബുഖാരി, റാസി, നസാഈ എന്നിവർ പറഞ്ഞു: അയാൾ കളവു കൊണ്ട് ആരോപിതനാണ്. റാസി, നസാഈ എന്നിവർ പറഞ്ഞു: അയാൾ വ്യാജ ഹദീസുകൾ ഉണ്ടാക്കുന്ന വ്യക്തിയായിരുന്നു. ഇമാം ദാറക്കുത്നി പറഞ്ഞു: അയാളിൽ ദൗർബല്യമുണ്ട്. ഇസ്ഹാകിബ്നു റാഹൂയ പറഞ്ഞു: അയാൾ നുണയനാണ്. (അദ്ദുഅഫാഉ വൽ മത്റൂകീൻ : ഇബ്നുൽ ജൗസി: 3 / 87, അദ്ദുഅഫാഉ സ്സ്വഗീർ: ബുഖാരി: 334, അൽ ജർഹുവതഅദീൽ: അബൂഹാതിം: 8/21, അൽ കാമിൽ ഇബ്നു അദിയ്യ്: 7/ 481)

പുറമെ മൂന്നു സനദുകൾക്കും നബിയിലേക്കെത്തുന്ന നിവേദകരുടെ കണ്ണികളില്ല, പരമ്പര മുറിഞ്ഞതാണ്. രണ്ടു സനദുകൾക്കും മറ്റു പല ന്യൂനതകളുമുണ്ട്. بو معشر: نجيح ابن عبد الرحمن السِّندي،. ضعَّفه أيوب بن عبد الرحمن صعصعة،: لم يوثقه غير ابن حبان മൂന്നാമത്തെ സനദിലെ അബൂ മഅ്ഷർ: നജീഹിബ്നു അബ്ദുർറഹ്‌മാൻ അസ്സിന്ദി ദുർബലനാണ്; അയ്യൂബിബ്നു അബ്ദുർറഹ്മാൻ സ്വഅ്സ്വഅ അദ്ദേഹത്തെ ദുർബലനാണെന്ന് വ്യക്തമാക്കിയിട്ടുണ്ട്, ഇബ്നു ഹിബ്ബാൻ അല്ലാത്ത ആരും അബൂ മഅ്ഷർ വിശ്വസ്‌തനാണെന്ന് അഭിപ്രായപ്പെട്ടിട്ടില്ല.

ചുരുക്കത്തിൽ മൂന്ന് പരമ്പരകളും ദുർബലമാണ്.

വിമർശനം:

” …വിവാഹം ചെയ്ത് ഭോഗിച്ചു… അതിൽ 4 എണ്ണത്തിനെ ബാലിശമായ കാരണങ്ങൾ പറഞ്ഞ് ഡിവോഴ്സ് ചെയ്തു…. അധികം പേർക്കും അറിയാത്ത (മുഹമ്മദ് നബി) മൊഴിചൊല്ലിയ സ്ത്രീകളുടെ പേരുകൾ ഇവയാണ്…

1) അസ്മ ബിൻത് അൽ നുമാൻ… ആയിഷ കുതത്രം പ്രയോഗിച്ച് ഡിവോഴ്സ് ചെയ്യിച്ചു.. (ഇബ്നു സാദ് 8: 101 -105, 153)”

(മുഹമ്മദിന്റെ പെണ്ണുങ്ങൾ: നാസ്‌തിക സോഷ്യൽ മീഡിയ തെറിമാല)

മറുപടി:

നാസ്‌തിക സോഷ്യൽ മീഡിയ തെറിമാലകളിൽ ഒന്നായ “മുഹമ്മദിന്റെ പെണ്ണുങ്ങൾ” എന്ന കുറിപ്പിൽ നിന്നുള്ള ചില വരികളാണ് ഈ വായിച്ചത്. കല്ലുവച്ച നുണകളും, അർദ്ധ സത്യങ്ങളും, ദുർവ്യാഖ്യാനങ്ങളും, വൈരുദ്ധ്യങ്ങളും നിറഞ്ഞ ഒരു മിഷനറി ലേഖനം ഫോട്ടോസ്റ്റാറ്റ് എടുത്ത് അരികുകൾ ചെത്തി കളഞ്ഞുണ്ടാക്കിയതാണ് ഈ നാസ്‌തിക അക്ഷര ഗരളം.

മറുപടിയിലേക്ക് വരാം…

1. അസ്‌മാഅ് ബിൻത് അന്നുഅ്മാൻ എന്ന സ്ത്രീയെ മുഹമ്മദ് നബി (സ) വിവാഹം ചെയ്യുകയും, “ഭോഗി”ക്കുകയും ബാലിശമായ കാരണങ്ങൾ പറഞ്ഞ് ഡിവോഴ്സ് ചെയ്യുകയുമുണ്ടായി എന്നാണ് ആരോപണം. ആരോപണത്തിനായി വിമർശകർ അവലംബിച്ച നിവേദനത്തിന്റെ ഉള്ളടക്കം ഇതാണ്:

അസ്‌മാഅ് ബിൻത് അന്നുഅ്മാനെ നബി (സ) വിവാഹം ചെയ്തു. വിവാഹ ശേഷം നബി (സ) അവരുടെ അടുത്തു ചെന്നു. അപ്പോൾ അവർ പറഞ്ഞു: “നിങ്ങളിൽ നിന്നും ഞാൻ അല്ലാഹുവിൽ ശരണം തേടുന്നു”. അപ്പോൾ നബി (സ) തന്റെ വസ്ത്രം കൊണ്ട് മുഖം മറച്ചു കൊണ്ട് പറഞ്ഞു: നീ ശരണം തേടിയിരിക്കുന്നത് ഏറ്റവും നല്ല രക്ഷകനിലാണ്… നീ നിന്റെ വീട്ടിലേക്ക് പൊയ്‌ക്കൊള്ളു…”

സംഭവം വായിച്ചാൽ പ്രവാചകനോട് വെറുപ്പല്ല ആദരവാണ് ഉണ്ടാവുക എന്ന് മിഷണറിമാർക്കറിയാം. അതുകൊണ്ട് സംഭവം മുഴുവൻ കൊടുക്കാതെ “വിവാഹം ചെയ്ത് ഭോഗിച്ചു… അതിൽ 4 എണ്ണത്തിനെ ബാലിശമായ കാരണങ്ങൾ പറഞ്ഞ് ഡിവോഴ്സ് ചെയ്തു…. അസ്മ ബിൻത് അൽ നുമാൻ… ആയിഷ കുതത്രം പ്രയോഗിച്ച് ഡിവോഴ്സ് ചെയ്യിച്ചു..” എന്നങ്ങു പറഞ്ഞു നിർത്തി !!

എവിടെ നിന്ന് ഭോഗിച്ച വിവരം നാസ്‌തികർക്ക് കിട്ടി ?! ഭോഗിയ്ക്കുക പോയിട്ട് സ്വന്തം ഭാര്യയായിരുന്നിട്ടും ഇഷ്ടമില്ലാത്തതു കൊണ്ട് സ്പർശിക്കുക പോലുമുണ്ടായില്ല. തന്നെ ഇഷ്ടമില്ലാത്തതിനാൽ തന്നെ മുഖം മറച്ച് ദൃഷ്ടി പോലും അവരിൽ നിന്നും മറച്ചുവെച്ചു. പിടിച്ചു വെക്കുകയൊ ബലം പ്രയോഗിക്കുകയൊ ചെയ്തില്ല. സൗമ്യതയോടെ വീട്ടിലേക്ക് പൊയ്കൊള്ളാൻ പറഞ്ഞു! ഇതാണൊ ഒരു സ്ത്രീപീഢകന്റെ ചിത്രം ?!

തനിക്ക് ഭർത്താവിനെ ഇഷ്ടമില്ലെന്ന് പറയുന്ന സ്ത്രീക്ക് യാതൊരു സങ്കോചമോ വൈമനസ്യമോ കൂടാതെ വീട്ടിലേക്ക് പോകാനും വിവാഹ മോചനം ചെയ്യാനുമുള്ള അവകാശം നൽകിയ ഒരാളെ “ബലാൽസംഗ വീരനും” ” സ്ത്രീ പീഢകനും” ആയി ചിത്രീകരിക്കാൻ വർഗീയ വിഷ വാഹകർക്കെ സാധിക്കു. അല്ലാത്തവർക്ക് അദ്ദേഹത്തെ സ്ത്രീവിമോചകനായാണ് മനസ്സിലാവുക. തന്നെ സ്നേഹത്തോടെ തിരഞ്ഞെടുത്തവരെ മാത്രമെ നബി (സ) ഭാര്യയാക്കിയിട്ടുള്ളു, സ്നേഹിച്ചിട്ടുള്ളു, സ്പർശിച്ചിട്ടുള്ളു എന്നത് അദ്ദേഹത്തിന്റെ മാന്യ വ്യക്തിത്വത്തിനും ലൈംഗിക വിശുദ്ധിക്കും മികച്ച തെളിവാണ്.

“ബാലിശമായ കാരണങ്ങൾ പറഞ്ഞ് ഡിവോഴ്സ് ചെയ്തു” എന്ന ആരോപണവും നുണ തന്നെ. അസ്‌മാഅ് ആണ് നബിയിൽ (സ) താൽപര്യമില്ലെന്ന അഭിപ്രായം അവതരിപ്പിച്ചത്, നബിയല്ല (സ). അവരുടെ അഭിപ്രായത്തെ മാനിച്ചു എന്നത് ഒരു തെറ്റായി മനസ്സിലാക്കാൻ മാത്രം ബുദ്ധിക്ക് വിധ്വേഷജ്വരം ബാധിച്ചോ നാസ്‌തികരെ നിങ്ങൾക്ക് ?!

2. അസ്‌മാഅ് ബിൻത് അന്നുഅ്മാൻ എന്ന സ്ത്രീയെ മുഹമ്മദ് നബി (സ) വിവാഹ മോചനം ചെയ്തു എന്ന് പോയിട്ട് വിവാഹം ചെയ്തു എന്നു പോലും സ്ഥിരപ്പെട്ടിട്ടില്ല എന്നതാണ് യാഥാർഥ്യം. ഈ പ്രസ്‌താവനക്ക് ഉപോൽബലകമായി രണ്ട് കാരണങ്ങൾ ഇവിടെ ഉദ്ധരിക്കാം.

ഒന്നാമത്തെ കാരണം:

നബിയുടെ (സ) പത്നിമാരിൽ, നബിയിൽ നിന്നും ശരണം തേടിയത് ഒരൊറ്റ പത്നി മാത്രമാണ്. അത് ഉമൈമ ബിൻത് നുഅ്മാനാണ് (ബുഖാരി: 5254). നബിയെ (സ) സംബന്ധിച്ചും അദ്ദേഹത്തിന്റെ മഹനീയമായ വ്യക്തിത്വത്തെ സംബന്ധിച്ചും പൂർവ്വ അറിവില്ലാത്ത ഉമൈമ പ്രവാചകനിൽ അനിഷ്ടം പ്രകടിപ്പിക്കുകയുണ്ടായി. താന്‍ വിവാഹം ചെയ്ത സ്ത്രീക്ക് തന്നോടൊപ്പം ജീവിക്കുവാന്‍ താല്‍പര്യമില്ലെന്നറിഞ്ഞപ്പോള്‍, നിര്‍ബന്ധിച്ച് കൂടെ താമസിപ്പിക്കാതെ മാന്യമായി അവരെ സ്വഗൃഹത്തിലേക്ക് യാത്രയാക്കുകയും വേര്‍പിരിയും മുമ്പ് അവര്‍ക്ക് സമ്മാനങ്ങള്‍ നല്‍കുകയും ചെയ്ത മാതൃകാപരമായ ഒരു നപടിയാണ് നബിയിൽ നിന്നുണ്ടായത്.

ﻻَ ﺗَﺤْﻤِﻠُﻮا اﻟﻨِّﺴَﺎءَ ﻋَﻠَﻰ ﻣَﺎ ﻳَﻜْﺮَﻫْﻦَ

“സ്ത്രീകളെ അവർക്ക് വെറുക്കുന്നത് ചെയ്യാൻ നിങ്ങൾ നിർബന്ധിക്കരുത്.” (മുസ്വന്നഫ് അബ്ദുർ റസാഖ്: 10320) എന്ന് അനുചരന്മാരെ പഠിപ്പിക്കുക മാത്രമല്ല കാരുണ്യ മൂർത്തിയായ പ്രവാചകൻ (സ) ചെയ്തത്, പ്രത്യുത ഉമൈമയോട് അനുവർത്തിച്ച നിലപാടിലൂടെ തന്റെ ആദർശനിഷ്ഠത സ്വജീവിതത്തിൽ പ്രാവർത്തികമായി തെളിയിക്കുക കൂടി അദ്ദേഹം ചെയ്തു.

പ്രവാചകന്‍ (സ) ഉദ്ദേശിച്ചിരുന്നെങ്കില്‍ അവരെ നിര്‍ബന്ധപൂര്‍വ്വം കൂടെ താമസിപ്പിക്കുവാന്‍ യാതൊരു തടസ്സവുമുണ്ടായിരുന്നില്ല. കാരണം അവിടുന്ന് ഇസ്‌ലാമിക സാമ്രാജ്യത്തിന്റെ അധിപനായിരുന്നു. രാജാക്കന്മാരും ചക്രവര്‍ത്തിമാരും ഒരു പെണ്ണിനെ ആഗ്രഹിച്ചു കഴിഞ്ഞാല്‍ അവളുടെ താല്‍പര്യം അന്വേഷിക്കുന്ന പതിവില്ലെന്ന് എല്ലാവര്‍ക്കുമറിയാവുന്ന വസ്തുതയാണ്. അതിനെതിരെ ഒരു ശബ്ദവുമവിടെ ഉയരുകയില്ല. ഇവിടെ പ്രവാചകന്‍ (സ) മാതൃകയാവുകയാണ്. താന്‍ വിവാഹം ചെയ്ത ഒരു സ്ത്രീക്ക് തന്നോടൊപ്പം ജീവിക്കുവാന്‍ താല്‍പര്യമില്ലെന്നറിഞ്ഞ നിമിഷം അവളെ ആശ്വസിപ്പിക്കുകയും സമാധാനിപ്പിക്കുകയും നിര്‍ഭയത്വത്തോടെ സ്വഗൃഹത്തിലേക്ക് മടങ്ങാന്‍ അവസരമൊരുക്കുകയും ചെയ്യുന്ന പ്രവാചകന്‍, ഒരു രാഷ്ട്രത്തിന്റെ ചോദ്യം ചെയ്യപ്പെടാത്ത ഭരണാധിപനാണെന്ന വസ്തുതയും ചേര്‍ത്തു മനസ്സിലാക്കുമ്പോള്‍ എത്രമാത്രം ആദരവും താല്‍പര്യവുമാണ് ആ വ്യക്തിത്വത്തിനോട് തോന്നേണ്ടത്. ഇസ്‌ലാം വിമര്‍ശകര്‍ക്ക് പക്ഷെ അത്തരം ഊഷ്മളമായ ചിന്തയും വികാരവുമൊന്നും ഉണ്ടാവുകയില്ല. കാരണം അവരുടെ ഹൃദയം കടുത്തു പോയിരിക്കുന്നു. ഊഷരമായ ചിന്തയും വികാരവുമാണ് അവരെ നയിക്കുന്നത്. വെറുപ്പും വിദ്വേഷവും മാത്രമാണ് അവരെ ഭരിക്കുന്നത്.

ഉമൈമയുടെ ചരിത്രം വിശദമായി മറ്റൊരു ലേഖനത്തിൽ പ്രതിപാദിച്ചിട്ടുള്ളതിനാൽ ഇവിടെ ആവർത്തിക്കുന്നില്ല:

ഉമൈമയുടെ ഈ സംഭവം അസ്‌മാഅ് ബിൻത് നുഅ്മാനടക്കം മറ്റു പല സ്ത്രീകളിലേക്കും ചേർത്തി കൊണ്ട് ചില നിവേദനങ്ങൾ രേഖപ്പെടുത്തപ്പെട്ടിട്ടുണ്ട്. ഇത് നിവേദകന്മാരിൽ നിന്നും സംഭവിച്ച ഓർമ്മ പിശകു മാത്രമാണ്. ഉമൈമ ബിൻത് നുഅ്മാൻ, അസ്‌മാഅ് ബിൻത് നുഅ്മാൻ എന്നീ പേരുകളിലെ സാമ്യതയാണ് ഇവിടെ അതിനു കാരണം. നാമങ്ങൾ പരസ്പരം കൂടി കലർന്നു കൊണ്ടുള്ള ചില ആശയക്കുഴപ്പങ്ങൾ ചില ചരിത്രകാരന്മാരിൽ നിന്ന് സംഭവിച്ചു എന്നതിനുള്ള വ്യക്തമായ തെളിവാണ് ഉമൈമ ബിൻത് നുഅ്മാന്റെയും, അസ്‌മാഅ് ബിൻത് നുഅ്മാന്റെയും കഥകളുടെ ഉള്ളടക്കത്തിലെ സാമ്യത.

രണ്ടാമത്തെ കാരണം:

അസ്‌മാഅ് ബിൻത് നുഅ്മാനെ നബി (സ) വിവാഹം ചെയ്തു, നബി (സ) അവരെ സന്ദർശിച്ചപ്പോൾ അവർ വെറുപ്പു പ്രകടിപ്പിച്ചു, (“നിങ്ങളിൽ നിന്നും ഞാൻ അല്ലാഹുവിൽ ശരണം തേടുന്നു” എന്ന് നബിയോട് പറഞ്ഞു കൊള്ളാൻ അസ്‌മായോട് ഉപദേശിച്ചു കൊണ്ട്) ആഇശ ബീവി തന്ത്രം പ്രയോഗിച്ചു എന്നൊക്കെ സൂചിപ്പിക്കുന്ന നിവേദനങ്ങളുടെ പരമ്പരകൾ ആസകലം ദുർബലമാണ് എന്നതാണ് മറ്റൊരു കാരണം.

പ്രസ്തുത നിവേദനങ്ങളുടെ പരമ്പരകളും അവയെ സംബന്ധിച്ച നിരൂപണങ്ങളും താഴെ ചേർക്കാം:

ഒന്നാമത്തെ പരമ്പര:

ഇബ്നു സഅ്ദ് (ത്വബകാത് 8/143-148) നിവേദനം ഉദ്ധരിച്ചിരിക്കുന്നത് മുഹമ്മദ് ഇബ്നു ഉമർ വാകിദിയിൽ നിന്നാണ് അദ്ദേഹം ഹദീസിന്റെ മേഖലയിൽ (ദഈഫ്) ദുർബലമാണ്.

രണ്ടാമത്തെ പരമ്പര:

أخبرنا محمد بن عمر حدثني عبد الله بن جعفر عن عمرو بن صالح عن سعيد بن عبد الرحمن بن أبزى قال

ഇബ്നു സഅ്ദ് (ത്വബകാത്: 8/144), സഈദിബ്നു അബ്ദുർറഹ്മാൻ ബിൻ അബ്സയിൽ നിന്ന് ഉദ്ധരിച്ച പരമ്പര പൂർണമല്ല, മുർസലാണ്. പരമ്പരയിലെ മുഹമ്മദിബ്നു ഉമർ വാകിദ് ഹദീസിന്റെ വിഷയത്തിൽ ദുർബലനാണ്. നിവേദനത്തിലെ അംറിബ്നു സ്വാലിഹ് എന്ന റാവി മജ്ഹൂൽ (അജ്ഞാതൻ) ആണ്. ഇബ്നു അദിയ്യ് തന്റെ ‘അൽ കാമിൽ ഫീ ദുഅഫാഇർ രിജാലിൽ’, അംറിബ്നു സ്വാലിഹ് എന്ന റാവിയെ ദുർബലനായി പരിഗണിച്ചിരിക്കുന്നു.

മൂന്നാമത്തെ പരമ്പര:

أخبرنا هشام بن محمد بن السائب ، عن أبيه ، عن أبي صالح ، عن بن عباس قال…

ഇബ്നു സഅ്ദ് (ത്വബകാത്: 8/145) ഉദ്ധരിച്ച ഈ പരമ്പരയും അങ്ങേയറ്റം ദുർബലമാണ്. നിവേദനത്തിലെ ഹിശാമിബ്നു മുഹമ്മദിബ്നുസ്സാഇബിനെ സംബന്ധിച്ച ഹദീസ് നിദാനശാസ്ത്ര പണ്ഡിതരുടെ അഭിപ്രായം ഇപ്രകാരമാണ്. ഇബ്നു ഇറാക്: കളവു പറയുന്നവനായി ആരോപിതൻ ഇബ്നു അസാകിർ പറഞ്ഞു: അദ്ദേഹം റാഫിളിയാണ്, വിശ്വസ്തനല്ല. ദാറകുത്നി പറഞ്ഞു: മത്റൂക് (നുണയനെന്ന് ആരോപിക്കപ്പെട്ടതിനാൽ ഹദീസ് സ്വീകരിക്കപ്പെടാത്ത വ്യക്തി) ദഹബി പറഞ്ഞു: അദ്ദേഹം വിശ്വസ്‌തനല്ല. ( http://hadith.islam-db.com/narrators/33196/ )

നിവേദനത്തിലെ മുഹമ്മദിബ്നുസ്സാഇബിനെ സംബന്ധിച്ചു ഹദീസ് നിദാനശാസ്ത്ര പണ്ഡിതരുടെ അഭിപ്രായം:

കള്ള ഹദീസുകൾ നിർമ്മിക്കുന്ന വ്യക്തിയാണയാൾ എന്നാണ് ബുർഹാനുദ്ദീൻ അൽ ഹലബിയുടെ അഭിപ്രായം (അൽ കശ്ഫുൽ ഹസീസ് അമ്മൻ റുമിയ ബി വദ്ഇൽ ഹദീസ്)

ഇബ്നുൽ ജവ്സി തന്റെ ‘മൗദൂആത്ത്’ ൽ, പ്രസ്തുത റാവി കള്ള ഹദീസ് നിർമ്മാതാക്കളുടെ നേതാവാണെന്ന് പ്രസ്ഥാവിക്കുന്നു. ‘അലി മരണപ്പെട്ടിട്ടില്ലെന്നും, മേഘങ്ങൾക്കു മുകളിൽ വസിക്കുകയാണ്…’ എന്നെല്ലാം വിശ്വസിക്കുന്ന തീവ്ര ശീഈ വിഭാഗത്തിൽ പെട്ട വ്യക്‌തിയാണെന്ന് ഇബ്നു ഹിബ്ബാൻ, റാവിയെ പറ്റി അഭിപ്രായപ്പെടുന്നു.

ദാറകുത്നി തന്റെ ‘അദ്ദുഅഫാഉ വൽ മത്റൂ കീൻ’ ലും, അബു നുഐം അൽ അസ്ബഹാനി തന്റെ ‘ദുഅഫാഅ്’ ലും, റാവി കള്ള ഹദീസുകൾ പടച്ചുണ്ടാക്കുന്ന വ്യക്തിയായി തന്നെ വിശദീകരിക്കുന്നുണ്ട്.

മറ്റൊരു സനദ് ഇപ്രകാരമാണ്: وروى أيضا قال : أخبرنا هشام بن محمد ، حدثني ابن الغسيل ، عن حمزة بن أبي أسيد الساعدي ، عن أبيه – وكان بدريا – قال :

പരമ്പരയിലെ ഹിശാമിബ്നു മുഹമ്മദ് കളവു പറയുന്ന വ്യക്തിയാണെന്ന വിശദീകരണം മുമ്പ് പ്രസ്താവിക്കപ്പെട്ടിട്ടുണ്ട്. നിവേദനത്തിലെ ഇബ്നുൽ ഗസീൽ വിശ്വസ്തനാണെങ്കിലും ഓർമ്മശക്തി കുറഞ്ഞ വ്യക്തിയാണെന്ന് ഇബ്നു ഹജറും വ്യക്തമാക്കിയിട്ടുണ്ട്.

വിമർശനം:

വിവസ്ത്രരായ കുറേ ആളുകൾ മുഹമ്മദ് നബിയുമായി ലൈംഗിക ബന്ധത്തിൽ ഏർപ്പെട്ടു, തന്നിമിത്തം അദ്ദേഹം തളർന്നുവെന്നുമെല്ലാം ഹദീസിൽ ഉണ്ട്. ഇത് സ്വവർഗ്ഗ സംഘരതിയിൽ നബി ഏർപ്പെട്ടു എന്നതിന് തെളിവല്ലെ ?

മറുപടി:

ആരോപിക്കുന്നതെന്ത് ?! ആരോപണത്തിനു പിൻബലമായി ഉദ്ധരിക്കുന്ന ഹദീസിന്റെ വിഷയമെന്ത്?!! പുലബന്ധം പോലുമില്ലാത്ത എന്തോ സംഭവത്തിലെ, ഏതോ വാചകം ലജ്ജയുടെ ലാഞ്ചന പോലും ഇല്ലാത്ത വിധം ദുർവ്യാഖ്യാനിക്കുക… എന്നിട്ട് മുഹമ്മദ് നബിക്കെതിരെ(സ) ഒരു അശ്ലീല കഥ പടച്ചുവിടുക. ഈ മിഷണറി ജീർണ്ണതകൾ പുതിയ ചായം പൂശി അവതരിപ്പിക്കുന്ന നാസ്തികരുടെ ഗതികേട് നോക്കണേ.

ലൈംഗികതയുമായി ബന്ധപ്പെട്ട ഒരു പരാമർശം പോലുമില്ലാത്ത ഒരു ഹദീസിനെ ഓർജിയാക്കി മാറ്റണമെങ്കിൽ എത്രമാത്രം രോഗാതുരമായിരിക്കണം ഇവരുടെയൊക്കെ മനസ്സും ജീവിതവും !? ധൈഷണികമായി എത്രമാത്രം ശൂന്യവും ഹീനവുമായിരിക്കണം ഇവരുടെയൊക്കെ ചിന്താമണ്ഡലം !!?

വിമർശകർ ഉദ്ധരിക്കുന്ന ഹദീസ് ഇപ്രകാരം നമുക്ക് വായിക്കാം:

اسْتَتبْعَنِي رَسُولُ اللهِ صَلَّى اللهُ عَلَيْهِ وَسَلَّمَ، قَالَ: فَانْطَلَقْنَا، حَتَّى أَتَيْتُ مَكَانَ كَذَا وَكَذَا فَخَطَّ لِي خِطَّةً، فَقَالَ لِي: ” كُنْ بَيْنَ ظَهْرَيْ هَذِهِ لَا تَخْرُجْ مِنْهَا، فَإِنَّكَ إِنْ خَرَجْتَ هَلَكْتَ ” . قَالَ: فَكُنْتُ فِيهَا، قَالَ: فَمَضَى رَسُولُ اللهِ صَلَّى اللهُ عَلَيْهِ وَسَلَّمَ، حَذَفَةً ، أَوْ أَبْعَدَ شَيْئًا، أَوْ كَمَا قَالَ: ثُمَّ إِنَّهُ ذَكَرَ هَنِينًا كَأَنَّهُمْ الزُّطُّ (قَالَ عَفَّانُ: أَوْ كَمَا قَالَ عَفَّانُ: إِنْ شَاءَ اللهُ) : لَيْسَ عَلَيْهِمْ ثِيَابٌ، وَلَا أَرَى سَوْآتِهِمْ، طِوَالًا، قَلِيلٌ لَحْمُهُمْ ، قَالَ: فَأَتَوْا، فَجَعَلُوا يَرْكَبُونَ رَسُولَ اللهِ صَلَّى اللهُ عَلَيْهِ وَسَلَّمَ . قَالَ: وَجَعَلَ نَبِيُّ اللهِ صَلَّى اللهُ عَلَيْهِ وَسَلَّمَ يَقْرَأُ عَلَيْهِمْ . قَالَ: وَجَعَلُوا يَأْتُونِي فَيُخَيِّلُونَ، أَوْ يَمِيلُونَ حَوْلِي، وَيَعْتَرِضُونَ لِي . قَالَ عَبْدُ اللهِ: فَأُرْعِبْتُ مِنْهُمْ رُعْبًا شَدِيدًا . قَالَ: فَجَلَسْتُ، أَوْ كَمَا قَالَ . قَالَ: فَلَمَّا انْشَقَّ عَمُودُ الصُّبْحِ جَعَلُوا يَذْهَبُونَ، أَوْ كَمَا قَالَ . قَالَ: ثُمَّ إِنَّ رَسُولَ اللهِ صَلَّى اللهُ عَلَيْهِ وَسَلَّمَ جَاءَ ثَقِيلًا وَجِعًا، أَوْ يَكَادُ أَنْ يَكُونَ وَجِعًا، مِمَّا رَكِبُوهُ .

അബ്ദുല്ലാഹിബ്നു മസ്ഊദ് (റ) പറഞ്ഞു: അല്ലാഹുവിന്റെ ദൂതൻ എന്നെ ഒരു യാത്രക്കായി കൂട്ട് വിളിച്ചു. അങ്ങനെ ഞങ്ങൾ പുറപ്പെട്ടു. ഇന്ന സ്ഥലമെത്തിയപ്പോൾ അദ്ദേഹം എനിക്ക് നിലത്ത് ഒരു വരവരച്ചു തന്നു. എന്നിട്ട് എന്നോട് പറഞ്ഞു: നീ എന്റെ പിറകിൽ ഈ വരക്കപ്പുറം നിൽക്കണം, അതിനപ്പുറം വരരുത്. ഈ വരവിട്ട് നീ മുന്നോട്ടു വന്നാൽ നീ മരിച്ചേക്കും. അങ്ങനെ ഞാൻ അവിടെ തന്നെ നിന്നു.

എന്നിട്ട് അല്ലാഹുവിന്റെ ദൂതൻ (സ) കുറച്ച് മുന്നോട്ട് നിന്നു. കുറച്ച് കഴിഞ്ഞപ്പോൾ ഒരു സംഘം പ്രാകൃതരായ ആളുകൾ വന്നു. അവരുടെ മേൽ വസ്ത്രമുണ്ടായിരുന്നില്ല, എന്നാൽ അവരുടെ നഗ്നത എനിക്ക് കാണാൻ കഴിഞ്ഞില്ല. അവർ വളരെ നീളമുള്ളവരും മാംസം കുറഞ്ഞവരുമായിരുന്നു. അങ്ങനെ അവർ അദ്ദേഹത്തിന്റെ അടുത്തു വന്നു. അവർ അല്ലാഹുവിന്റെ ദൂതരെ തിക്കിതിരക്കാൻ തുടങ്ങി.

നബിയാകട്ടെ അവർക്ക് ക്വുർആൻ പാരായണം ചെയ്തു കൊണ്ടിരിക്കാനും തുടങ്ങി. പിന്നീടവർ എന്റെയടുത്തു വരാൻ തുടങ്ങി, എന്നെ വലയം വെക്കാനും എനിക്ക് തടസ്സമുണ്ടാക്കാനും തുടങ്ങി. ഞാനാകെ പേടിച്ചരണ്ടു. അങ്ങനെ ഞാനവിടെ ഇരുന്നു. നേരം പുലരുന്നതിനനുസരിച്ച് അവർ പിരിഞ്ഞു പോവാൻ തുടങ്ങി. ശേഷം അല്ലാഹുവിന്റെ ദൂതൻ ക്ഷീണിതനും അവശനുമായി വന്നു; തിക്കും തിരക്കും കാരണം…” (സുനനു തുർമുദി: 2861)

ആത്മീയ ജീവികളായ ജിന്നുകളുമായി നബി (സ) നടത്തിയ സംവാദവും പ്രബോധനവുമാണ് ഹദീസിന്റെ ഇതിവൃത്തം. സംഭവത്തെ സംബന്ധിച്ച് ക്വുർആനും പ്രസ്താവിക്കുന്നുണ്ട്. “ജിന്നുകളില്‍ ഒരു സംഘത്തെ നാം നിന്‍റെ അടുത്തേക്ക് ഖുര്‍ആന്‍ ശ്രദ്ധിച്ചുകേള്‍ക്കുവാനായി തിരിച്ചുവിട്ട സന്ദര്‍ഭം (ശ്രദ്ധേയമാണ്‌.) അങ്ങനെ അവര്‍ അതിന് സന്നിഹിതരായപ്പോള്‍ അവര്‍ അന്യോന്യം പറഞ്ഞു: നിങ്ങള്‍ നിശ്ശബ്ദരായിരിക്കൂ. അങ്ങനെ അത് കഴിഞ്ഞപ്പോള്‍ അവര്‍ തങ്ങളുടെ സമുദായത്തിലേക്ക് താക്കീതുകാരായിക്കൊണ്ട് തിരിച്ചുപോയി.” (ക്വുർആൻ: 46:29)

“ഒരു സംഘം പ്രാകൃതരായ ആളുകൾ വന്നു…” എന്ന് ഹദീസിൽ പ്രസ്താവിച്ചിരിക്കുന്നത് ആത്മീയ ജീവികളായ ജിന്നുകളെ ഉദ്ദേശിച്ചാണ്. ചില നിവേദനങ്ങളിൽ ഇത് വ്യക്തമായി പ്രസ്താവിക്കപ്പെട്ടിട്ടുമുണ്ട്. “مَن هؤلاءِ يا رسولَ اللهِ؟ قال: هؤلاء جِنُّ نَصِيبينَ، جاؤوا يَختصِمون إليَّ في أمورٍ كانتْ بينهم”. “ഇബ്നു മസ്ഊദ് ചോദിച്ചു: അല്ലാഹുവിന്റെ ദൂതരേ, ആരാണവർ ? അപ്പോൾ അദ്ദേഹം (സ) പറഞ്ഞു: അവർ ജിന്നുകളാണ്. അവർക്കിടയിലുള്ള ചില വിഷയങ്ങളെ സംബന്ധിച്ച് എന്നോട് സംവദിക്കാൻ വന്നതാണ് അവർ.”

“നീ എന്റെ പിറകിൽ ഈ വരക്കപ്പുറം നിൽക്കണം, അതിനപ്പുറം വരരുത്. ഈ വരവിട്ട് നീ മുന്നോട്ടു വന്നാൽ നീ മരിച്ചേക്കും” എന്ന് നബിയും(സ), “പിന്നീടവർ എന്റെയടുത്തു വരാൻ തുടങ്ങി, എന്നെ വലയം വെക്കാനും എനിക്ക് തടസ്സമുണ്ടാക്കാനും തുടങ്ങി. ഞാനാകെ പേടിച്ചരണ്ടു.” എന്ന് അബ്ദുല്ലാഹിബ്നു മസ്ഊദും (റ) പറയാനുള്ള കാരണം ആ സംഘം അദൃശ്യ ലോകത്തു നിന്നുള്ള അഭൗതിക ജീവവർഗമായതു കൊണ്ടും അവരുമായി ഭൗതികമായി സന്ധിക്കുവാൻ അല്ലാഹു ഒരുക്കിയ അസുലഭവും അത്ഭുതകരവുമായ ഒരു സംഗമമായിരുന്നു അത് എന്നതു കൊണ്ടുമാണ്.

ആത്മീയ ജീവികളായ ജിന്നുകൾ മുഹമ്മദ് നബിയുമായി സ്വവർഗരതിയിൽ ഏർപ്പെട്ടു എന്ന് നാസ്തികർക്ക് എങ്ങനെയാണ് വാദിക്കാനാവുക എന്നത് മറ്റൊരു കാര്യം.

ഭീമാകാരമായ ശരീരമുള്ള, മാംസമില്ലാത്ത ഈ ആത്മീയ ജീവവർഗ്ഗം (ഒരു തരം ജിന്ന് വിഭാഗം) വസ്ത്രം ധരിച്ചതായി കാണപ്പെട്ടില്ലെങ്കിലും അവരുടെ നഗ്നതയും കാണപ്പെട്ടിരുന്നില്ലെന്ന് നിവേദനത്തിൽ തന്നെ പ്രസ്‌താവിക്കുന്നുണ്ട്. ഒരു നിവേദനത്തിൽ لا أرَى عَورةً ولا أرَى قِشرًا “അവരുടെ നഗ്നതയൊ തൊലിയൊ ഞാൻ കണ്ടില്ല” എന്ന് വന്നിരിക്കുന്നു. അഥവാ ഗുഹ്യാവയവങ്ങളൊ ലൈംഗികാവയവങ്ങളൊ പോയിട്ട് ശരീരത്തിൽ തൊലി പോലും കാണപ്പെടാത്ത വിധമാണ് അവരുടെ ശരീര സൃഷ്ടി ! ലൈംഗികാവയവങ്ങളില്ലാതെ പിന്നെ എങ്ങനെയാണ് സ്വവർഗരതിയൊക്കെ നടന്നത് എന്ന് നാസ്തികർ തന്നെ വ്യക്തമാക്കണം.

ഇസ്‌ലാം പഠിക്കാൻ വേണ്ടി വന്ന ജിന്നുകളായിരുന്നു അവർ. പ്രവാചകനെ(സ) ആദ്യമായി കണ്ട ആവേശത്തിൽ അവർ അദ്ദേഹത്തിന് ചുറ്റും തിങ്ങി കൂടി… “അങ്ങനെ അവർ അദ്ദേഹത്തിന്റെ അടുത്തു വന്നു. അവർ അല്ലാഹുവിന്റെ ദൂതരെ തിക്കിതിരക്കാൻ തുടങ്ങി. നബിയാകട്ടെ അവർക്ക് ക്വുർആൻ പാരായണം ചെയ്തു കൊണ്ടിരിക്കാനും തുടങ്ങി..” എന്ന ഹദീസിലെ വരികൾ ശ്രദ്ധിക്കുക. അവർ തിക്കിതിരക്കി വന്നത് ‘ഓർജി’ക്കായിരുന്നില്ല. ഇസ്‌ലാം പഠിക്കാനായിരുന്നു. അവർ വന്നപ്പോൾ പ്രവാചകൻ (സ) ചെയ്തത് ക്വുർആൻ പാരായണമായിരുന്നു; ഏതെങ്കിലും തരത്തിലുള്ള ലൈംഗിക ചേഷ്ടകളായിരുന്നുമില്ല.

ഇവിടെ ലൈംഗികതയുമായി എങ്ങിനെയാണ് ഈ സംഭവം ബന്ധപ്പെട്ടു കിടക്കുന്നത് എന്ന് ഒരു സാധാരണക്കാരൻ ആശ്ചര്യപ്പെട്ടേക്കാം. പക്ഷെ, ഒരു മാനസിക രോഗിയുടെ ചിന്തകളുടെ സഞ്ചാരപഥം എത്ര സങ്കീർണ്ണമാണ് എന്ന് മിഷണറിമാരുടെയും നാസ്തികരുടെയും ദുർവ്യഖ്യാനങ്ങൾ നമുക്ക് ബോധ്യപ്പെടുത്തി തരും. ദുർവ്യാഖ്യാനം ഇപ്രകാരമാണ്:

1. ജിന്നുകളുടെ ശരീര പ്രകൃതിയെ വർണ്ണിച്ചു കൊണ്ട് “അവരുടെ മേൽ വസ്ത്രമുണ്ടായിരുന്നില്ല, എന്നാൽ അവരുടെ നഗ്നത എനിക്ക് കാണാൻ കഴിഞ്ഞില്ല.” എന്ന് ഇബ്നു മസ്ഊദ് പറഞ്ഞ വാചകം ആദ്യം ദുർവ്യാഖ്യാനിച്ചു വഷളാക്കി. വസ്ത്രം അഴിച്ചു വന്ന ഒരു പറ്റം മനുഷ്യരായി അവരെ വിമർശകർ വ്യാഖ്യാനിച്ചു. യഥാർത്ഥത്തിൽ പ്രവാചകന്റെ(സ) അടുത്തു വന്ന ജിന്നു വിഭാഗത്തിന്റെ ശരീരഘടനയാണ് ഇവിടെ പ്രസ്‌താവിക്കുന്നത്. മനുഷ്യർക്ക് സമാനമായ ശരീരമല്ലായിരുന്നു അവരുടെത്. മനുഷ്യേതര ജീവജാലങ്ങളെ പോലെ വസ്ത്രം ധരിക്കാത്ത ജീവികൾ; എന്നാൽ അവരുടെ നഗ്നത കാണാനും കഴിയില്ല. മൃഗങ്ങളിലെ രോമകുപ്പായം (fur) പോലെ വേണമെങ്കിൽ ആ ശരീര പ്രകൃതിയെ – മനുഷ്യർക്ക് മനസ്സിലാക്കാക്കാനായി – വ്യാഖ്യാനിക്കാം. ഇങ്ങനെ അത്ഭുതകരമായ ശരീര പ്രകൃതിയെ സൂചിപ്പിച്ചു കൊണ്ടുള്ള വരികളെ ഓർജിക്കു തയ്യാറായി വരുന്ന ഒരു സംഘം സ്വവർഗരതിക്കാരായി വ്യാഖ്യാനിക്കാൻ നിസ്സാര തൊലിക്കട്ടി ഒന്നും പോര.

2. “അങ്ങനെ അവർ അദ്ദേഹത്തിന്റെ അടുത്തു വന്നു. അവർ അല്ലാഹുവിന്റെ ദൂതരെ തിക്കിതിരക്കാൻ തുടങ്ങി. നബിയാകട്ടെ അവർക്ക് ക്വുർആൻ പാരായണം ചെയ്തു കൊണ്ടിരിക്കാനും തുടങ്ങി..” എന്ന ഹദീസിലെ വരികളാണ് അടുത്തതായി ലൈംഗികവൽകരിക്കപ്പെട്ടിരിക്കുന്നത്. ഇവിടെ “അവർ അല്ലാഹുവിന്റെ ദൂതരെ തിക്കിതിരക്കാൻ തുടങ്ങി…” എന്ന വാചകത്തിലെ ‘യർകബൂന’ (അവർ തിക്കിതിരക്കുന്നു يركبون) എന്ന അറബി പദത്തിനെ ആദ്യം ഇംഗ്ലീഷിലേക്ക് വിവർത്തനം ചെയ്തു. മുപ്പതിലേറെ അർത്ഥങ്ങളുള്ള ഈ ഒരു അറബി പദത്തിന് ദുർവ്യാഖ്യാനത്തിനുതകുന്ന ഒരു ഇംഗ്ലിഷ് അർത്ഥം അവർ കണ്ടെത്തുന്നു; Drive/Ride. Ride എന്ന ഇംഗ്ലിഷ് പദത്തിനെ അനൗപചാരികമായി (colloquial) ലൈംഗികമായി ബന്ധപ്പെടുക എന്ന ഒരു അർത്ഥത്തിൽ ഉപയോഗിക്കാറുണ്ടല്ലൊ. അപ്പോൾ ജിന്നുകൾ നബിയെ Ride ചെയ്തു; ലൈംഗികമായി ബന്ധപ്പെട്ടു എന്നർത്ഥം വരുന്നു !! എങ്ങനെയുണ്ട് കണ്ടെത്തൽ?!

പണ്ട് സ്കൂളിൽ പഠിക്കുന്ന കാലത്ത് കലോത്സവത്തിനിടയിൽ “ഇന്നയിന്ന വിദ്യാർത്ഥികൾ ഇന്ന സാറുമായി എത്രയും പെട്ടെന്ന് ‘ബന്ധപ്പെടുക ‘…. ജഡ്ജസുമായി ‘ബന്ധപ്പെടുക ‘… സ്റ്റേജുമായി ‘ബന്ധപ്പെടുക ‘…” അനൗൺസ്മെന്റ് കേൾക്കുമ്പോൾ ഒരു പറ്റം അലവലാതി ചെക്കന്മാർ പൊട്ടിച്ചിരിക്കുമായിരുന്നത് ഓർമ്മ വന്നു പോവുകയാണ്. അവരുടെ മനസ്സിലെ ‘ബന്ധപ്പെടുക’ എന്ന് പറഞ്ഞാൽ അത് ലൈംഗിക അർത്ഥത്തിലായിരുന്നു. അവരൊക്കെ “പഠിച്ച്” “മിടുക്കന്മാരായി” യുക്തിവാദി സംഘത്തിന്റെ നേതൃത്വത്തിൽ എത്തി എന്ന് ഇപ്പോൾ മനസ്സിലാവുന്നു. സന്തോഷം.

ഏതായിരുന്നാലും ഹദീസിലെ فَأَتَوْا، فَجَعَلُوا يَرْكَبُونَ رَسُولَ اللهِ صَلَّى اللهُ عَلَيْهِ وَسَلَّمَ എന്ന വാചകത്തിനർത്ഥം “അവർ അല്ലാഹുവിന്റെ ദൂതരുടെ അടുത്ത് തിക്കിതിരക്കാൻ തുടങ്ങി” എന്നാണ് എന്നതിൽ യാതൊരു സംശയവും ഇല്ല. സംഭവത്തിന്റെ മറ്റൊരു നിവേദനത്തിൽ ‘യർകബൂന’ (അവർ തിക്കിതിരക്കുന്നു يركبون) എന്ന പദത്തിന്റെ വിവക്ഷയെന്താണെന്ന് വ്യക്തമായും വന്നിട്ടുണ്ട്:

انْطَلَقْتُ مَعَ النَّبِيِّ صَلَّى اللهُ عليه وآله وَسَلَّمَ لَيْلَةَ الْجِنِّ حَتَّى إِذَا أَتَى الْحَجُونَ، فَخَطَّ عَلَيَّ خَطًّا ثُمَّ تَقَدَّمَ إليهم فَازْدَحَمُوا عَلَيْهِ …

” …ശേഷം അദ്ദേഹം (നബി (സ)) ജിന്നുകളുകളുടെ അടുത്തേക്ക് ചെന്നു. അവർ അദ്ദേഹത്തിന് ചുറ്റും തിങ്ങി തിരക്കുകൂട്ടി.” (ദലാഇലുന്നുബുവ്വ: 2:232)

ഈ നിവേദനത്തിൽ ‘യർകബൂന’ (يركبون) എന്ന പദത്തിന് പകരം ‘ഇസ്ദഹമൂ’ (ازْدَحَمُوا crowded) എന്ന് തന്നെ അർത്ഥശങ്കക്കിടയില്ലാത്ത വിധം വന്നിരിക്കുന്നു. (https://translate.google.com/?sl=ar&tl=en&text=%D8%A7%D8%B2%D8%AF%D8%AD%D9%85%D9%88%D8%A7%0A&op=translate )

يركبون، أي: يزحمونه ويقربون منه ഭാഷാ പണ്ഡിതർ പറഞ്ഞിരിക്കുന്നു: “യർകബൂൻ (يركبون) എന്നാൽ ചുറ്റും തിങ്ങി തിരക്കുകൂട്ടുകയും അടുക്കുകയും ചെയ്യുക എന്നാണ്…” (തഹ്കീക്: മുസ്നദു അഹ്മദ്: 6:337 )

ക്വുർആനും ഈ സംഭവത്തെ സംബന്ധിച്ച് സംസാരിക്കുന്നതും ഇതേ അർത്ഥത്തിൽ തന്നെയാണ്: “അല്ലാഹുവിന്‍റെ ദാസന്‍ (നബി) അവനോട് പ്രാര്‍ത്ഥിക്കുവാനായി എഴുന്നേറ്റ് നിന്നപ്പോള്‍ അവര്‍ (ജിന്നുകൾ) അദ്ദേഹത്തിന് ചുറ്റും ‘തിങ്ങിക്കൂടുവാനൊരുങ്ങി’ എന്നും…” (ക്വുർആൻ: 72: 19)

3. ഈ തിക്കും തിരക്കും കഴിഞ്ഞ് പ്രവാചകൻ (സ) ക്ഷീണിച്ച് വന്ന് ഉറങ്ങി എന്ന് ഹദീസിൽ സൂചിപ്പിക്കപ്പെട്ടതാണ് അടുത്ത ദുർവ്യാഖ്യാനം. രതിക്കു ശേഷമുള്ള ക്ഷീണമായിരുന്നത്രെ അത്.?!

ഭീകരവും ഭീമാകാര രൂപികളുമായ ഒരു വലിയ സംഘം ജിന്നുകൾക്ക് ഒരു ദിവസം മുഴുവൻ ക്വുർആൻ ഓതി കൊടുത്ത്, സമ്മേളനം കഴിഞ്ഞ് വന്നു കിടന്നുറങ്ങിയ നിഷ്കളങ്കമായ ആ ഉറക്കത്തെ ലൈംഗികവൽകരിക്കാൻ ലജ്ജയുടെ ഒരു കണിക പോലും ബാക്കിയില്ലെ ഇവരുടെയൊന്നും മനസ്സിൽ. തലയിൽ മുഴുവൽ ലൈംഗിക ചിന്തകൾ മാത്രമാണൊ അവശേഷിക്കുന്നത് ?!

ഇനി നിവേദനത്തിന്റെ പരമ്പരയും സ്വീകാര്യതയും നിരൂപണ വിധേയമാക്കാം:

നിവേദനത്തിന്റെ സനദും (പരമ്പര) മത്നും (ഉള്ളടക്കവും) ദുർബലമാണെന്ന് ഒരുപാടു പണ്ഡിതർ വീക്ഷിച്ചിട്ടുണ്ട്.

നിവേദനത്തിന്റെ സനദ് ഇപ്രകാരമാണ്:

حَدَّثَنَا عَارِمٌ، وَعَفَّانُ، قَالَا: حَدَّثَنَا مُعْتَمِرٌ، قَالَ: قَالَ أَبِي: حَدَّثَنِي أَبُو تَمِيمَةَ، عَنْ عَمْرٍو – لَعَلَّهُ أَنْ يَكُونَ قَدْ قَالَ: الْبِكَالِيَّ يُحَدِّثُهُ عَمْرٌو – عَنْ عَبْدِ اللهِ بْنِ مَسْعُودٍ – قَالَ عَمْرٌو إِنَّ عَبْدَ اللهِ -، قَالَ:

ഇബ്നു മസ്ഊദിൽ നിന്ന് അംറ് അൽ ബക്കാലിയാണ് സംഭവം ഉദ്ധരിക്കുന്നത്.

അംറ് അൽ ബക്കാലിയാകട്ടെ ഇബ്നു മസ്ഊദിൽ നിന്ന് ഹദീസൊന്നും കേട്ടിട്ടില്ല. ഇമാം ബുഖാരി പറഞ്ഞു: ولا يُعرف لعمرو سماعٌ من ابن مسعود അംറ് അൽ ബക്കാലി, ഇബ്നു മസ്ഊദിൽ നിന്ന് ഹദീസൊന്നും കേട്ടതായി അറിയപ്പെട്ടിട്ടില്ല. (അത്താരീഖുൽ കബീർ: 2: 200)

അംറ് അൽ ബക്കാലിക്ക് ‘സുഹ്ബത്ത്’ (പ്രവാചകാനുചരനാകാനുള്ള അവസരം) ഉണ്ടായിട്ടുണ്ട് എന്ന ഒരു അഭിപ്രായം ചില പണ്ഡിതർക്കുണ്ട്. ഇത് ശരിയാണെങ്കിൽ തന്നെ ഈ സംഭവം അംറ് അൽ ബക്കാലി, ഇബ്നു മസ്ഊദിൽ നിന്ന് ഒരു മധ്യവർത്തി മുഖേനയാണ് ഉദ്ധരിക്കുന്നത് എന്ന് വരുന്നു. ആ മധ്യവർത്തി മജ്ഹൂൽ (അറിയപ്പെടാത്ത നിവേദകൻ) ആണ്. അപ്പോൾ നിവേദനം ദുർബലമാണെന്ന് വരുന്നു.

കൂടാതെ നിവേദനം “വളരെ ഒറ്റപ്പെട്ടതാണെന്ന്” ഇബ്നു കസീറും, “പരമ്പര ദുർബലമാണെന്ന്” ശുഐബ് അൽ അർനാവൂത്വും, ഡോ. തൈസീറിബ്നു സഅ്ദ് അബൂ ഹൈമദും പ്രസ്‌താവിക്കുന്നു. (തഫ്സീറു ഇബ്നു കസീർ: 7:293, തഹ്കീക്: മുസ്നദു അഹ്മദ്: 6: 334, തഹ്കീക്: താരീഖുൽ ഔസത്: 2:1072)

(ശൈഖ് അബൂ ഉമർ അൽ ബാഹിസിന്റെ ലേഖനത്തിൽ നിന്ന് https://www.antishubohat.net/2012/09/08/3orah/)

ജിന്നുകളുമായുള്ള നബിയുടെ (സ) ഈ സംവാദം അബ്ദുല്ലാഹിബ്നു മസ്ഊദ് (റ) സാക്ഷിയായതായിട്ടാണല്ലൊ വിമർശന വിധേയമായ നിവേദനം ഉദ്ധരിക്കപ്പെട്ടിരിക്കുന്നത്. വളരെ പ്രബലമായ നിവേദനത്തിൽ അബ്ദുല്ലാഹിബ്നു മസ്ഊദ് (റ) ഈ സംഭവത്തിന് സാക്ഷിയായിട്ടില്ല എന്ന് അദ്ദേഹം തന്നെ പ്രസ്താവിക്കുന്നുണ്ട്.

عَنْ عَبْدِ اللهِ قَالَ: لَمْ أَكُنْ لَيْلَةَ الْجِنِّ مَعَ رَسُولِ اللهِ صَلَّى اللهُ عَلَيْهِ وَسَلَّمَ، وَوَدِدْتُ أَنِّي كُنْتُ مَعَهُ

അബ്ദുല്ലാഹിബ്നു മസ്ഊദ് (റ) പറഞ്ഞു: “ജിന്നുകൾ സന്ദർശിച്ച രാത്രി ഞാൻ അല്ലാഹുവിന്റെ ദൂതനോടൊപ്പം ഉണ്ടായിരുന്നില്ല, ഞാൻ അദ്ദേഹത്തോടൊപ്പം ആയിരുന്നെങ്കിൽ എന്ന് ഞാൻ ആഗ്രഹിച്ചു പോയി.” (സ്വഹീഹു മുസ്‌ലിം: 152)

പിന്നെ എങ്ങനെയാണ് ചർച്ചാ വിഷയകമായ നിവേദനം സ്വഹീഹ് (സ്വീകാര്യമാവുക) എന്ന് ചില ഹദീസ് പണ്ഡിതർ ചോദ്യം ചെയ്യുന്നുണ്ട്.

മൃഗങ്ങളെ നാണിപ്പിക്കുന്ന മൃഗരതി കളവുകൾ !! വിമർശനം:

Sahih al-Bukhari 2, 357:

Ibn Sharib narrated, Ibn Abdul Talib said:

“Always when his wives had their period, I saw the Prophet (pbuh) near by his camel herd. There he had lovingly intercourse with the female animals, sometimes he also turned towards the young animals of both sexes.”

സ്വഹീഹുൽ ബുഖാരി 2, 357:

ഇബ്നു ശരീബ് വിവരിച്ചു, ഇബ്നു അബ്ദുൽ താലിബ് പറഞ്ഞു:

“എല്ലായ്‌പ്പോഴും അദ്ദേഹത്തിന്റെ ഭാര്യമാർക്ക് ആർത്തവം വരുമ്പോൾ നബി(സ)യെ അദ്ദേഹത്തിന്റെ ഒട്ടകക്കൂട്ടത്തിനരികിൽ ഞാൻ കാണാറുണ്ട്. അവിടെ അവൻ പെൺ മൃഗങ്ങളുമായി സ്നേഹപൂർവ്വം ലൈംഗിക ബന്ധത്തിൽ ഏർപ്പെട്ടു, ചിലപ്പോൾ അവൻ രണ്ട് ലിംഗത്തിലുള്ള മൃഗങ്ങളിലേക്കും തിരിഞ്ഞു.

മറുപടി:

സോഷ്യൽ മീഡിയയിലൂടെ പ്രചരിക്കപ്പെടുന്ന ഈ കള്ള കഥയുടെ ഇംഗ്ലീഷ് രൂപവും അത് കേരളത്തിലെ ഭൗതികവാദികൾ ‘ഗൂഗിൾ ട്രാൻസ്ലേറ്റർ’ ഉപയോഗിച്ച് (ഇല്ലാ കഥയാണെന്ന് അറിയാവുന്നതു കൊണ്ട് മിനക്കെട്ട് പരിഭാഷ ചെയ്യെണ്ടന്ന് തീരുമാനിച്ചതായിരിക്കാം) ഉണ്ടാക്കിയെടുത്ത് പ്രചരിപ്പിക്കുന്ന മലയാള തിരകഥയുമാണ് മുകളിൽ കൊടുത്തിരിക്കുന്നത്.

സ്വഹീഹുൽ ബുഖാരി 2, 357 എന്ന നമ്പറിട്ട് കൊടുത്തിട്ടുള്ള ഈ മൃഗരതി ഉള്ളടക്കമുള്ള ഒരു ഹദീസും സ്വഹീഹുൽ ബുഖാരിയിൽ എവിടെയുമില്ല! ഗൂഗിളിൽ ഈ നമ്പറും ഹദീസിന്റെ ഇംഗ്ലീഷ് ടെക്സ്റ്റും സെർച്ച് ചെയ്താൽ ഒരിക്കലും തുറക്കാൻ കഴിയാത്ത ചില വ്യാജ വെബ്സൈറ്റുകളുടെ പേരുകൾ പ്രത്യക്ഷപ്പെട്ടേക്കാം…. പല ഇസ്‌ലാമിക സൈറ്റുകളിലും വീഡിയൊകളുടെ കമന്റ് ബോക്സുകളിലെ നാസ്തിക വിധ്വേഷ പ്രചാരകരുടെ കമന്റുകളിലും മാത്രം ഈ കള്ള കഥയും കള്ള നമ്പറുകളും കാണാം. അതല്ലാതെ സ്വഹീഹുൽ ബുഖാരിയിൽ നേരിട്ടൊ, ബുഖാരിക്ക് മുസ്‌ലിം പണ്ഡിതന്മാർ എഴുതിയ പരിഭാഷകളിലൊ ഇങ്ങനെയൊരു കള്ള കഥ കാണാൻ സാധ്യമെ അല്ല. അഥവാ യാതൊരു അടിസ്ഥാനവുമില്ലാത്ത, മൃഗങ്ങളെ പോലും ലജ്ജിപ്പിക്കാനുതകുന്ന ഒരു വൃത്തികെട്ട പോൺ സ്റ്റോറി എഴുതി, തോന്നിയ നമ്പറിട്ട് അത് നബിയുടെ (സ) പേരിൽ പ്രചരിപ്പിക്കുകയാണിവിടെ.

നവനാസ്തികരുടെ ലൈംഗിക സ്വാതന്ത്ര്യകാംക്ഷയുടെ സ്‌മൃതിലഹരിയിൽ ഉന്മത്തനായി, സ്വഹീഹുൽ ബുഖാരിയുടെ ഇംഗ്ലീഷ് വിവർത്തനം വായിച്ചപ്പോൾ സംഭവിച്ച ഭാവനാവിലാസമാണ് ഈ കള്ള ഹദീസ്. അല്ലാതെ സ്വഹീഹുൽ ബുഖാരിയടക്കം ഒരു ഹദീസ് ഗ്രന്ഥത്തിലും ഇത്തരം, നാസ്തിക രതി വൈകൃത ചിത്രങ്ങൾ കാണാൻ ആർക്കും കഴിയില്ല. ഒരു ഹദീസ് ഗ്രന്ഥത്തിന്റെ പേരെഴുതി, എന്തെങ്കിലും നമ്പർ കുറിച്ചിട്ട്, സ്വന്തം കാമാഭിലാഷങ്ങളും സ്വന്തം ഗൃഹാന്തര കാഴ്ച്ചകളും നബിയുടെയും സ്വഹാബികളുടേയും പേരിൽ എഴുതി പിടിപ്പിക്കുക എന്ന നാസ്തിക-മിഷണറി ശൈലിയാണ് ഇവിടെയും ആവർത്തിക്കപ്പെട്ടിരിക്കുന്നത്. ഏതോ ഒരു നാസ്തിക മൃഗാനുരാഗിയുടെ രോഗാതുരമായ മനസ്സിലെ -മൃഗങ്ങളെ പോലും നാണിപ്പിക്കുന്ന- ഭോഗലാലസതയുടെ വിരൂപസൃഷ്ടി !

കളവിന്റെ ആഴവും അശ്ലീലതയുടെ തീക്ഷ്ണതയും ഈ വർഗീയവാദികളുടെ അന്തരാളങ്ങളിൽ നുരഞ്ഞു പൊന്തുന്ന വെറുപ്പിന്റെ ആധിക്യമാണ് തെളിയിക്കുന്നത്. എന്തും എങ്ങനെയും നേടിയെടുക്കുക എന്ന നാസ്തിക ധർമ്മരാഹിത്യത്തിന്റെ പ്രകടമായ ഉദാഹരണങ്ങളാണ് ഇത്തരം കള്ള കഥാ നിർമ്മാണങ്ങൾ. ഉള്ളിലെ ജീർണത തുളുമ്പി തെറിപ്പിച്ച് മറ്റുള്ളവരെയും തങ്ങളെ പോലെ മലിനമാക്കുക എന്ന ഉപജാപത്തിന്റെ ഭാഗമാണിതും. ഇത്തരക്കാരോട് ഒന്നേ പറയാനുള്ളു:

“…നിങ്ങളുടെ അരിശം കൊണ്ട് നിങ്ങള്‍ മരിച്ചുകൊള്ളൂ. തീര്‍ച്ചയായും അല്ലാഹു മനസ്സുകളിലുള്ളത് അറിയുന്നവനാകുന്നു.” (ക്വുർആൻ 3: 119)

വിമർശനം:

In Islam, sex with animals and sex with dead bodies is halal.

Book: Sunan Abu Dawood (Ifa), Chapter: 33 / Provision of Punishment, Hadith Number: 4406

4406. Narrated from Ahmad Ibn Yunus (R): Ibn Abbas (R). He said: There is no punishment for having intercourse with an animal.

There are two opinions if a woman inserts the penis of an animal (into her vagina), and if she inserts a separate penis; The most accurate is that the woman’s genitals should be washed.

MuslimSahih Muslim – Book of Menstruation – hadith # 525 – Commentary

Comment: The sex or separate male genitalia of any animal of Momina can be inserted into its female genitalia.

According to some Sunni Islamic scholars, sexual intercourse with an animal does not invalidate fasting or Hajj.

(വിമർശകരുടെ തന്നെ (ഗൂഗിൾ) പരിഭാഷ:)

ഇസ്‌ലാമിൽ മൃഗങ്ങളുമായുള്ള ലൈംഗികതയും മൃതദേഹവുമായുള്ള ലൈംഗികതയും ഹലാലാണ്.

പുസ്തകം: സുനൻ അബു ദാവൂദ് അധ്യായം: 33 / ശിക്ഷയുടെ വ്യവസ്ഥ, ഹദീസ് നമ്പർ: 4406 4406.

അഹ്മദ് ഇബ്നു യൂനുസ് (റ): ഇബ്നു അബ്ബാസ് (റ) ൽ നിന്ന് നിവേദനം. അദ്ദേഹം പറഞ്ഞു: മൃഗവുമായി ഇണചേരുന്നതിന് ശിക്ഷയില്ല. ഒരു സ്ത്രീ ഒരു മൃഗത്തിന്റെ ലിംഗം (അവളുടെ യോനിയിൽ) പ്രവേശിപ്പിക്കുകയാണെങ്കിൽ, അവൾ ഒരു ച്ഛേദിക്കപ്പെട്ട ലിംഗം പ്രവേശിപ്പിക്കുകയൊ ആണെങ്കിൽ അക്കാര്യത്തിൽ രണ്ട് അഭിപ്രായങ്ങളുണ്ട്; സ്ത്രീയുടെ ജനനേന്ദ്രിയങ്ങൾ കഴുകണം എന്നതാണ് ഏറ്റവും കൃത്യമായത്.

സഹിഹ് മുസ്ലിം – ഹദീസ് # 525 – വ്യാഖ്യാനം അഭിപ്രായം: ഏതെങ്കിലും മൃഗത്തിന്റെ ലിംഗഭേദം അല്ലെങ്കിൽ പ്രത്യേക പുരുഷ ജനനേന്ദ്രിയം അതിന്റെ സ്ത്രീ ജനനേന്ദ്രിയത്തിൽ ചേർക്കാവുന്നതാണ്.

ചില സുന്നി ഇസ്ലാമിക പണ്ഡിതരുടെ അഭിപ്രായത്തിൽ, മൃഗവുമായുള്ള ലൈംഗികബന്ധം നോമ്പിനെയോ ഹജ്ജിനെയോ അസാധുവാക്കില്ല.

മറുപടി:

1. ഇസ്‌ലാമിലെ പ്രമാണങ്ങൾ ക്വുർആനും സ്വഹീഹായ ഹദീസുകളുമാണ്. അല്ലാതെ പണ്ഡിതാഭിപ്രായങ്ങളല്ല. പണ്ഡിത വ്യാഖ്യാനങ്ങൾ പ്രമാണങ്ങളെ മനസ്സിലാക്കി തരുന്ന ഗൈഡൻസുകളാണ്. സുനനു അബൂദാവൂദ് അടക്കം ഹദീസ് ഗ്രന്ഥങ്ങളിൽ ഹദീസുകൾക്ക് പുറമെ ഫിക്ഹ് (കർമ്മശാസ്ത്രമുണ്ട്), ക്വുർആൻ വ്യാഖ്യാനമുണ്ട്, പണ്ഡിതാഭിപ്രായങ്ങളുണ്ട്, ഭാഷാ ചർച്ചകൾ… എന്നിങ്ങനെ പലതുമുണ്ട്. അങ്ങനെ പല വിവരങ്ങളും ഉൾകൊള്ളുന്ന ഒരു ഗ്രന്ഥമാണത്. അതിലെ ഹദീസുകളാണ് മുസ്‌ലിംകൾ പ്രമാണമായി കാണുന്നത്.

( https://www.snehasamvadam.org/ഇസ്‌ലാമിലെ-രണ്ട്-അടിസ്ഥാ/ )

പ്രവാചകന്റെ(സ) വാക്കുകളുടേയും പ്രവർത്തനങ്ങളുടേയും ക്രോഡീകരണമാണ് ഹദീസ്. (നുസ്ഹത്തുന്നദ്ർ: 1:36, മുഖദ്ദിമ ഫീ ഉസൂലുൽ ഹദീസ്: 1:33, അൽഫദ്‌ലുൽ മുബീൻ അലാ അക്ദി ജൗഹരി സ്സമീൻ: 61)

ഇവിടെ, ഇസ്‌ലാം മൃഗരതിക്ക് അനുവാദം നൽകുന്നു എന്നതിന് തെളിവായി വിമർശകർ എടുത്തുദ്ധരിച്ചിരിക്കുന്നത് ക്വുർആനൊ, സ്വഹീഹായ ഹദീസുകളോ അല്ല. പണ്ഡിതാഭിപ്രായങ്ങളാണ്. ഇത്തരം പണ്ഡിതാഭിപ്രായങ്ങൾ ഇസ്‌ലാമിൽ പ്രമാണങ്ങളല്ല. അവ ഫിക്ഹ് (കർമ്മശാസ്ത്രം) ആകുന്നു. ഫിക്ഹ് എന്നത് ഒരു പഠനശാഖയാണ് എന്ന് ഏവർക്കുമറിയാം. അതിൽ പ്രമാണങ്ങളോട് യോജിക്കുന്നവ മുസ്‌ലിംകൾ സ്വീകരിക്കുകയും പ്രമാണങ്ങളോട് യോജിക്കാത്തവ തിരസ്കരിക്കുകയും ചെയ്യും. ഹദീസുകൾക്ക് പുറമെ, ഇത്തരത്തിലുള്ള ഫിക്‌ഹും ഹദീസ് ഗ്രന്ഥങ്ങളിലും അവക്ക് പണ്ഡിതന്മാർ രചിച്ച വിശദീകരണങ്ങളിലുമുണ്ട്. ആ ഫിക്ഹുകൾ മുസ്‌ലിംകളുടെ അടുക്കലൊ ഇസ്‌ലാമിലോ അലംഘനീയമായ പ്രമാണങ്ങൾ അല്ല. (ഉലൂമുൽ ഹദീസ്: ഇബ്നു സ്വലാഹ്: 22, 23)

2. ഇസ്‌ലാമിക പ്രമാണങ്ങൾ മൃഗരതിയെ എങ്ങനെ കാണുന്നു എന്ന് തിരിച്ചറിയാൻ ക്വുർആനിലൂടെയും സ്വഹീഹായ ഹദീസുകളിലൂടെയും ഒന്ന് കണ്ണോടിച്ചാൽ പോരെ ?! ഫിക്ഹിന്റെ അന്തരാളങ്ങളിൽ ഊളിയിടണമെന്നുണ്ടോ ?!!

ക്വുർആൻ പറയുന്നു: “തങ്ങളുടെ ലൈംഗികവിശുദ്ധി കാത്തുസൂക്ഷിക്കുന്നവരുമാണ്. തങ്ങളുടെ ഭാര്യമാരുമായോ, തങ്ങളുടെ അധീനത്തിലുള്ള അടിമസ്ത്രീകളുമായോ ഉള്ള ബന്ധം ഒഴികെ. അപ്പോള്‍ അവര്‍ ആക്ഷേപാര്‍ഹരല്ല. എന്നാല്‍ അതിന്നപ്പുറം ആരെങ്കിലും ആഗ്രഹിക്കുന്ന പക്ഷം അവര്‍ തന്നെയാണ് അതിക്രമകാരികള്‍.” (ക്വുർആൻ: 23: 5-7)

ക്വുർആനിൽ ഇത്ര വ്യക്തമായി പറഞ്ഞിരിക്കെ പിന്നെ മൃഗരതിയെ ഇസ്‌ലാമിന്റെ പേരിൽ അവതരിപ്പിക്കുന്നതിൽ എന്ത് അർത്ഥമാണുള്ളത് ?!

നബി (സ) പറഞ്ഞു: لعن اللهُ مَن وقع على بهيمةٍ لعن اللهُ من عمِل عملَ قومِ لوطٍ “മൃഗങ്ങളുമായി രതിയിൽ ഏർപ്പെട്ടവനെ ദൈവം ശപിച്ചിരിക്കുന്നു. ലൂത്തിന്റെ ജനതയുടെ പ്രവർത്തനത്തിൽ (സ്വവർഗരതി) ഏർപ്പെട്ടവനെ അല്ലാഹു ശപിച്ചിരിക്കുന്നു.” (മുസ്നദ് അഹ്മദ്: 2816, ഇബ്നുഹിബ്ബാൻ: 4417, ഹാകിം: 8052 )

ملعونٌ مَنْ وقعَ على بهيمَةٍ ، ملعونٌ مَنْ عمِلَ بعمَلِ قومِ لوطٍ

“മൃഗങ്ങളുമായി രതിയിൽ ഏർപ്പെട്ടവൻ ശപിക്കപ്പെട്ടിരിക്കുന്നു. ലൂത്തിന്റെ ജനതയുടെ പ്രവർത്തനത്തിൽ (സ്വവർഗരതി) ഏർപ്പെട്ടവൻ ശപിക്കപ്പെട്ടിരിക്കുന്നു.” (മുസ്നദു അഹ്മദ്: 1875, ഇബ്നുഹിബ്ബാൻ: 4417, ത്വബ്റാനി: 11546 )

3. പ്രവാചകാനുചരന്മാരുടെ കാലഘട്ടത്തിലെ അവസാന സന്ധിയിൽ ഇസ്‌ലാമിന്റെ വളർച്ച ദ്രുതഗതിയിലാവുകയും ലക്ഷോപലക്ഷങ്ങൾ ഇസ്‌ലാമിലേക്ക് കടന്നുവരികയും ചെയ്യാൻ തുടങ്ങിയപ്പോൾ ദുർബല വിശ്വാസികളും കപട വിശ്വാസികളും ഇസ്‌ലാമിക സമൂഹത്തിൽ സ്ഥാനം പിടിച്ചു. വളർച്ച പ്രാപിച്ച മറ്റേത് സമുദായങ്ങളിലേതുമെന്ന പോലെ മുസ്‌ലിംകൾക്കിടയിലും ജീർണതകളും അധാർമികതകളും വർധിച്ചു. മദ്യം, വ്യഭിചാരം, കളവ്, മോഷണം, കൊള്ള, കൊല തുടങ്ങിയ സാമൂഹിക- വൈയക്തിക തിന്മകൾ എല്ലാ സമൂഹങ്ങളിലും ഇടതടവില്ലാതെ നടക്കുന്നുണ്ട്. മുസ്‌ലിം സമൂഹത്തിൽ ദുർബല വിശ്വാസികളും കപട വിശ്വാസികളും സ്ഥാനം പിടിച്ചതോടെ അവരിലും ഇത്തരം ദൂഷ്യങ്ങൾ പെരുകുക സ്വഭാവികം മാത്രം. അതുകൊണ്ട് തന്നെ ഈ സാമൂഹിക ഭൂമികയെ പരിഗണിച്ച് കർമ്മശാസ്ത്ര ചർച്ചകളെ വാർത്തെടുക്കാൻ കർമശാസ്ത്ര പണ്ഡിതർ നിർബന്ധിതരായി. അല്ലാതെ സാമൂഹിക യഥാർത്ഥ്യങ്ങളെ അവഗണിച്ച് എങ്ങനെ ഫിക്ഹ് (കർമ്മശാസ്ത്ര) ചർച്ച ചെയ്യും? അധാർമിക പ്രവണതകളെയും ആ പ്രവണതകളുടെ അനന്തരഫലമായുണ്ടാകുന്ന സാമൂഹിക- കുടുംബ- വൈയക്തിക പ്രശ്നങ്ങളെയുമെല്ലാം കർമ്മശാസ്ത്ര പണ്ഡിതർ ചർച്ച ചെയ്തിട്ടുണ്ട്. അതിൽ ഒരു വിഷയവും അറപ്പു കൊണ്ട് മാറ്റി വെച്ചിട്ടില്ല. ഒരു കൈപ്പുറ്റ യാഥാർത്ഥ്യത്തെയും അവഗണിച്ച് ഫിക്ഹിനെ (കർമ്മശാസ്ത്ര) വാർത്തെടുക്കാൻ ശ്രമിച്ചിട്ടുമില്ല, ശ്രമിക്കാനും പാടില്ല. സ്വാഭാവികമായും നിഷിദ്ധമായ രതി വൈകൃതങ്ങൾ സമൂഹത്തിൽ നടക്കാനുള്ള സാധ്യത മുന്നിൽ കണ്ട് അത്തരം പ്രശ്നങ്ങളിലെ കർമ്മശാസ്ത്ര വിധികളെ പറ്റി ചർച്ച ചെയ്തിട്ടുണ്ട്. അതിനർത്ഥം അത്തരം അവിഹിതങ്ങളെയൊ രതി വൈകൃതങ്ങളെയൊ ഇസ്‌ലാമിക കർമ്മശാസ്ത്രമോ, കർമ്മശാസ്ത്ര ചർച്ചകളുടെ അവലംബമായ ഇസ്‌ലാമിക പ്രമാണങ്ങളൊ അംഗീകരിക്കുന്നു എന്നല്ല. ഒരു രാജ്യത്ത് കൊലപാതകവും കലാപവും ബലാത്സംഗവുമെല്ലാം നടന്നാലുള്ള അനന്തര നടപടികളെ സംബന്ധിച്ച നിയമങ്ങൾ ഉണ്ടെന്നതും ഭരണഘടന അത് ചർച്ച ചെയ്യുന്നുണ്ട് എന്നതും ആ രാജ്യവും ഭരണഘടനയും ഈ ദ്രോഹങ്ങൾക്ക് അംഗീകാരം നൽകുന്നു എന്നതിന് തെളിവാണോ ? അല്ലല്ലൊ. ഈ വസ്തുത മനസ്സിലാക്കിയതിന് ശേഷമാവണം ഹദീസ് ഗ്രന്ഥങ്ങളിലെ കർമ്മശാസ്ത്ര ചർച്ചയെ നാം സമീപിക്കേണ്ടത്.

“In Islam, sex with animals and sex with dead bodies is halal… ” “ഇസ്‌ലാമിൽ മൃഗങ്ങളുമായുള്ള ലൈംഗികതയും മൃതദേഹവുമായുള്ള ലൈംഗികതയും ഹലാലാണ്…” എന്ന പെരും നുണ പറഞ്ഞു കൊണ്ടാണ് വിമർശകർ പ്രചരിപ്പിക്കുന്ന ഈ വിമർശനം ആരംഭിക്കുന്നത്. തുടർന്ന് അതിന് തെളിവായി ഉദ്ധരിക്കുന്നത് ഇതാണ്:

Book: Sunan Abu Dawood (Ifa), Chapter: 33 / Provision of Punishment, Hadith Number: 4406

4406. Narrated from Ahmad Ibn Yunus (R): Ibn Abbas (R). He said: There is no punishment for having intercourse with an animal.

“പുസ്തകം: സുനൻ അബു ദാവൂദ് അധ്യായം: 33 / ശിക്ഷയുടെ വ്യവസ്ഥ, ഹദീസ് നമ്പർ: 4406 4406.

അഹ്‌മദ്‌ ഇബ്നു യൂനുസ് (റ): ഇബ്നു അബ്ബാസ് (റ) ൽ നിന്ന് നിവേദനം. അദ്ദേഹം പറഞ്ഞു: മൃഗവുമായി ഇണചേരുന്നതിന് ശിക്ഷയില്ല.”

സുനനു അബൂദാവൂദിൽ നിന്ന് ദുർവ്യാഖ്യാനിക്കാൻ സാധ്യത കാണുന്ന ഒരു വാചകമങ്ങെടുത്ത് ചേർത്തു! അതിന്റെ മുന്നും പിന്നുമൊന്നുമില്ല.!! അധ്യായം മുഴുവനായും ഇവിടെ ഉദ്ധരിച്ചിട്ടാവാം നമ്മുടെ ചർച്ച:

30 – باب فيمن أتى بهيمةً

4464 – حدَّثنا عبدُ الله بنُ محمَّد النُّفيليُّ، حدَّثنا عبدُ العزيز بنُ محمدٍ، حدَّثني عَمرُو بنُ أبي عَمرو، عن عِكرمة

عن ابنِ عباس، قال: قال رسولُ الله – صلَّى الله عليه وسلم -: “مَن أتى بهيمةً، فاقتلوهُ واقتلوها مَعَهم” قال: قُلتُ له: ما شأنُ البهيمةِ؟ قال: ما أُراه قال ذلك إلا أنه كَرِهَ أن يُؤكَلَ لحمُها، وقد عُمِلَ بها ذلك العملُ…

4465 – حدَّثنا أحمدُ بنُ يونسَ، أن شريكاً وأبا الأحوصِ وأبا بكر بنَ عياشٍ حدَّثوهم، عن عاصِمٍ، عن أبي رزينٍ

عن ابنِ عباس، قال: ليس على الذي يأتي البهيمة حدٌّ.

قال أبو داود: وكذا قال عطاءٌ، وقال الحكم: أرى أن يُجلَدَ ولا يُبْلَغَ به الحدّ، وقال الحسنُ: هو بمنزلةِ الزَّاني

“അധ്യായം: മൃഗവുമായി രതിയിൽ ഏർപ്പെട്ടവന് എന്ത് ശിക്ഷാ നടപടിയാണ് സ്വീകരിക്കേണ്ടത്.” എന്ന അധ്യായത്തിന്റെ നാമം തന്നെ വായിച്ചാൽ കാര്യം സ്പഷ്ടമാണ്. മൃഗരതി അനുവദനീയമാണൊ അല്ലെ എന്നല്ല അധ്യായം. മൃഗരതി എന്ന വൻപാപം ഒരാൾ ചെയ്താൽ അയാളുടെ മേൽ ഒരു ഇസ്‌ലാമിക ഭരണകൂടം ഭൗതികമായ ശിക്ഷാ നടപടികൾ എന്തെങ്കിലും നടപ്പാക്കണൊ വേണ്ടെ എന്നത് മാത്രമാണ് ചർച്ച. മൃഗരതി എന്ന വൻപാപത്താൽ പരലോകത്ത് നൽകപ്പെടുന്ന ദൈവികമായ ശിക്ഷയൊ, ഭൗതികമായ പ്രായശ്ചിത്തമൊ ‘തഅ്സീറൊ’ മാത്രമാണ് അയാളുടെ മേൽ ഉള്ളു, വധശിക്ഷയൊ സമാനമായ ഭൗതിക ശിക്ഷാ നടപടികൾ ഇല്ല എന്ന് ചില പണ്ഡിതന്മാർ അഭിപ്രായപ്പെട്ടിട്ടുണ്ട്. നബിയുടെ (സ) കാലഘട്ടത്തിൽ മൃഗരതിയിൽ ആരും ഏർപ്പെടാത്തതിനാൽ ശിക്ഷാ നടപടികൾ ഒന്നും നടപ്പാക്കിയതായി ഇസ്‌ലാമിക പ്രമാണങ്ങളിൽ രേഖപ്പെടുത്തപ്പെട്ടിട്ടില്ല എന്നതാണ് ഈ ഒറ്റപ്പെട്ട അഭിപ്രായത്തിന് കാരണം. ഹദ്ദ് അഥവാ ശിക്ഷക്ക് പകരം കഫ്ഫാറത്ത് അഥവാ പ്രായശ്ചിത്തം ആണ് വിധിയെന്ന് അവർ വാദിക്കുന്നു.

മൃഗരതിക്ക് ഭൗതിക ശിക്ഷാ നടപടികൾ ഒന്നുമില്ല എന്ന ഇബ്നു അബ്ബാസിന്റെ അഭിപ്രായം സുനനു അബൂദാവൂദിൽ നിന്നും കോട്ടിമാട്ടിയ വിമർശകർ ആ വാചകത്തിന്റെ തൊട്ടു മുകളിലും താഴെയുമുള്ള വാചകങ്ങൾ കട്ടു മറച്ചു.

مَن أتى بهيمةً، فاقتلوهُ واقتلوها مَعَهم

“മൃഗരതിയിൽ ഏർപ്പെടുന്നവന് വധശിക്ഷയുണ്ട്” എന്ന ഇബ്നു അബ്ബാസിന്റെ(റ) തന്നെ അഭിപ്രായം വിമർശകർ മുക്കി.

രണ്ടാമതായി വിമർശകർ മുക്കിയ വാചകമിതാണ്: وقال الحكم: أرى أن يُجلَدَ ولا يُبْلَغَ به الحدّ، وقال الحسنُ: هو بمنزلةِ الزَّاني

“ഹകം പറഞ്ഞു: മൃഗരതിയിൽ ഏർപ്പെട്ടവന് മർദ്ദന ശിക്ഷ നടപ്പാക്കണം. അങ്ങേയറ്റം ശിഷക്ക് വിധേയനാക്കണം. ഹസൻ പറഞ്ഞു: അവൻ വ്യഭിചാരിക്ക് സമാനമായതിനാൽ വ്യഭിചാരത്തിനുള്ള ശിക്ഷ അവനിൽ നടപ്പാക്കണം.”

ഇതൊന്നും കാണാത്ത മട്ടിൽ ഭൗതിക ശിക്ഷാവിധി ഉണ്ടെന്നും ഇല്ലെന്നും ഒരു പണ്ഡിതൻ തന്നെ അഭിപ്രായപ്പെട്ട രണ്ട് അഭിപ്രായത്തിൽ ശിക്ഷാവിധി ഇല്ലെന്ന വാചകം മാത്രമെടുത്തു വെച്ച്, ഫ്രെയ്‌മിട്ടു. ചർച്ച ശിക്ഷാവിധി ഇല്ല എന്ന ആശയത്തിൽ നിന്നും അനുവദനീയം എന്ന ആശയത്തിലേക്ക് തള്ളി മാറ്റി… എങ്ങനെയെങ്കിലും തങ്ങളുടെ മനസ്സുകളിലെ ലൈംഗിക വൈകൃതങ്ങൾ മറ്റുള്ളവരിൽ പ്രതിഫലിപ്പിച്ച് കാണിക്കുന്ന ഈ മ്ലേച്ഛത എത്ര ഭീകരം !!

ഇസ്‌ലാമിൽ മൃഗരതി അനുവദനീയമാണ് എന്ന് കെട്ടിച്ചമക്കാൻ വിമർശകർ ഉദ്ധരിക്കുന്ന മറ്റൊരു വാചകമിതാണ്:

There are two opinions if a woman inserts the penis of an animal (into her vagina), and if she inserts a separate penis; The most accurate is that the woman’s genitals should be washed.

MuslimSahih Muslim – Book of Menstruation – hadith # 525 – Commentary

According to some Sunni Islamic scholars, sexual intercourse with an animal does not invalidate fasting or Hajj.

“ഒരു സ്ത്രീ ഒരു മൃഗത്തിന്റെ ലിംഗം (അവളുടെ യോനിയിൽ) പ്രവേശിപ്പിക്കുകയാണെങ്കിൽ, അവൾ ഒരു ച്ഛേദിക്കപ്പെട്ട ലിംഗം പ്രവേശിപ്പിക്കുകയൊ ആണെങ്കിൽ അക്കാര്യത്തിൽ രണ്ട് അഭിപ്രായങ്ങളുണ്ട്; സ്ത്രീയുടെ ജനനേന്ദ്രിയങ്ങൾ കഴുകണം എന്നതാണ് ഏറ്റവും കൃത്യമായത്.

സഹിഹ് മുസ്ലിം – ഹദീസ് # 525 – വ്യാഖ്യാനം അഭിപ്രായം: ഏതെങ്കിലും മൃഗത്തിന്റെ ലിംഗഭേദം അല്ലെങ്കിൽ പ്രത്യേക പുരുഷ ജനനേന്ദ്രിയം അതിന്റെ സ്ത്രീ ജനനേന്ദ്രിയത്തിൽ ചേർക്കാവുന്നതാണ്.

ചില സുന്നി ഇസ്ലാമിക പണ്ഡിതരുടെ അഭിപ്രായത്തിൽ, മൃഗവുമായുള്ള ലൈംഗികബന്ധം നോമ്പിനെയോ ഹജ്ജിനെയോ അസാധുവാക്കില്ല.”

മുകളിൽ ഉദ്ധരിക്കപ്പെട്ടത് ക്വുർആനൊ നബിയുടെ ഹദീസൊ അല്ല. അറബിയിൽ എഴുതപ്പെടുന്നതെല്ലാം ഇസ്‌ലാമാണെന്ന തെറ്റിദ്ധാരണ നാം മാറ്റി വെക്കുക.

ക്രിസ്താബ്ദം 1277 ൽ അഥവാ ഹിജ്റാബ്ദം 676 ൽ നിര്യാതനായ ഇമാം നവവി എന്ന ശാഫിഈ കർമ്മശാസ്ത്ര പണ്ഡിതന്റെ വാചകമാണ് മുകളിലെ ആദ്യത്തെ പാരഗ്രാഫ്. പ്രവാചക വിയോഗത്തിന് ശേഷം ആറ് നൂറ്റാണ്ട് കഴിഞ്ഞ് വന്ന ഒരു പണ്ഡിതൻ, ഹദീസിന്റെ വ്യാഖ്യാനത്തിൽ കുറിച്ചിട്ട തന്റെ കർമ്മശാസ്ത്ര സംബന്ധമായ അഭിപ്രായങ്ങങ്ങളെ ഇസ്‌ലാമിന്റെ പ്രമാണമൊ വിശുദ്ധ ഗ്രന്ഥമൊ ആയി അവതരിപ്പിക്കുന്നതു തന്നെ അബദ്ധമാണ്.

മാത്രമല്ല, ഇമാം നവവി കുറിച്ചിട്ട ചർച്ചയും ശ്രദ്ധ അർഹിക്കുന്നുണ്ട്. മൃഗരതി എന്ന വൻപാപം അനുവദനീയമൊ നിഷിദ്ധമൊ എന്നതല്ല അദ്ദേഹവും ചർച്ച ചെയ്യുന്നത്!! അത്തരമൊരു മ്ലേച്ഛത ഒരാൾ ചെയ്താൽ അതിനെ തുടർന്നു വരുന്ന മറ്റു കർമ്മശാസ്ത്ര വിധികളെ പറ്റി മാത്രമാണ് ഇവിടെയും ചർച്ച.

മൃഗരതിയെ സംബന്ധിച്ച ധാർമ്മിക വിധി എന്താണെന്ന് വളരെ വ്യക്തമായി ഇമാം നവവി തന്നെ രേഖപ്പെടുത്തുന്നത് കാണുക:

ﻭﻳﺤﺮﻡ اﺗﻴﺎﻥ اﻟﺒﻬﻴﻤﺔ ﻟﻘﻮﻟﻪ ﻋﺰ ﻭﺟﻞ (ﻭاﻟﺬﻳﻦ ﻫﻢ ﻟﻔﺮﻭﺟﻬﻢ ﺣﺎﻓﻈﻮﻥ اﻻ ﻋﻠﻰ ﺃﺯﻭاﺟﻬﻢ ﺃﻭ ﻣﺎ ﻣﻠﻜﺖ ﺃﻳﻤﺎﻧﻬﻢ ﻓﺈﻧﻬﻢ ﻏﻴﺮ ﻣﻠﻮﻣﻴﻦ) ﻓﺈﻥ ﺃﺗﻰ اﻟﺒﻬﻴﻤﺔ ﻭﻫﻮ ﻣﻤﻦ ﻳﺠﺐ ﻋﻠﻴﻪ ﺣﺪ اﻟﺰﻧﺎ ﻓﻔﻴﻪ ﺛﻼﺛﺔ ﺃﻗﻮاﻝ…

“മൃഗങ്ങളുമായുള്ള രതി നിഷിദ്ധമാണ്. കാരണം, അല്ലാഹു ഇപ്രകാരം പറഞ്ഞിരിക്കുന്നു: ‘തങ്ങളുടെ ഗുഹ്യാവയവങ്ങളെ കാത്തുസൂക്ഷിക്കുന്നവരുമത്രെ അവര്‍. തങ്ങളുടെ ഭാര്യമാരുമായോ, തങ്ങളുടെ അധീനത്തിലുള്ള അടിമസ്ത്രീകളുമായോ ഉള്ള ബന്ധം ഒഴികെ. അപ്പോള്‍ അവര്‍ ആക്ഷേപാര്‍ഹരല്ല. എന്നാല്‍ അതിന്നപ്പുറം ആരെങ്കിലും ആഗ്രഹിക്കുന്ന പക്ഷം അവര്‍ തന്നെയാണ് അതിക്രമകാരികള്‍.’ (ക്വുർആൻ: 23:5-7). ഇനി ആരെങ്കിലും മൃഗങ്ങളെ സമീപിച്ചാൽ അയാളിൽ നടപ്പാക്കേണ്ട ശിക്ഷാവിധിയിൽ മൂന്ന് അഭിപ്രായങ്ങൾ പണ്ഡിതന്മാർക്കിടയിലുണ്ട്…” (അൽ മജ്മൂഉ ശർഹുൽ മുഹദ്ദബ്: 20:30)

തുടർന്ന് ഇമാം നവവി മൂന്ന് അഭിപ്രായങ്ങൾ വിശദീകരിച്ചു.

(ﺃﺣﺪﻫﺎ) ﺃﻧﻪ ﻳﺠﺐ ﻋﻠﻴﻪ اﻟﻘﺘﻞ

അതിൽ പ്രഥമവും പ്രധാനവുമായ അഭിപ്രായം അയാൾക്ക് വധശിക്ഷ നൽകപ്പെടണം എന്നതാണ്.

രണ്ടാമത്തെ അഭിപ്രായം മൃഗരതിക്കുള്ള ശിക്ഷ വ്യഭിചാരത്തിനുള്ള ശിക്ഷ പോലെയാണ് എന്നാണ്. വിവാഹിതൻ വ്യഭിചരിച്ചാൽ വധശിക്ഷയും അവിവാഹിതനാണെങ്കിൽ മർദ്ദനവുമാണ് ശിക്ഷ. മൂന്നാമത്തെ അഭിപ്രായം അയാൾക്ക് ഭൗതികമായ ശിക്ഷാവിധികളൊന്നും ഭരണാധികാരി നടപ്പാക്കേണ്ടതില്ല എന്നതാണ്. (അൽ മജ്‌മൂഉ ശർഹുൽ മുഹദ്ദബ്: 20:30)

അത്തരമൊരു രതി വൈകൃതത്തിൽ ഏർപ്പെട്ടാൽ പിന്നീടു മറ്റു കർമ്മങ്ങൾ നിർവ്വഹിക്കാൻ അംഗശുദ്ധിയും കുളിയും നിർബന്ധമാണൊ എന്നതാണ് ഒരു ചർച്ച.

‏وَلَوْ اسْتَدْخَلَت الْمَرْأَة ذَكَرَ بَهِيمَة وَجَبَ عَلَيْهَا الْغُسْل , وَلَوْ اسْتَدْخَلَت ذَكَرًا مَقْطُوعًا فَوَجْهَانِ أَصَحّهمَا يَجِب عَلَيْهَا الْغُسْل

“ഒരു സ്ത്രീ, മൃഗത്തിന്റെ ലിംഗം തന്റെ ഗുഹ്യാവയവത്തിൽ പ്രവേശിപ്പിച്ചാൽ കുളിച്ച് ശുദ്ധി വരുത്തൽ അവൾക്ക് നിർബന്ധമാണ്. ഒരു സ്ത്രീ, ച്ഛേദിക്കപ്പെട്ട ഒരു ലിംഗം തന്റെ ഗുഹ്യാവയവത്തിൽ പ്രവേശിപ്പിച്ചാലുള്ള വിധിയിൽ രണ്ടഭിപ്രായമുണ്ട്. കുളിച്ച് ശുദ്ധി വരുത്തൽ നിർബന്ധമാണ് എന്നതാണ് ശരിയായ അഭിപ്രായം.” ഇതാണ് ഇമാം നവവിയുടെ വാചകം. ഭൗതികവാദി അത് ഇംഗ്ലീഷിലേക്ക് മാറ്റിയപ്പോൾ പരിഭാഷയിൽ വേണ്ടുവോളം വെള്ളം ചേർത്തു എന്ന് സാന്ദർഭികമായി സൂചിപ്പിക്കട്ടെ. ഭൗതികവാദിയുടെ പരിഭാഷ ശ്രദ്ധിക്കുക: There are two opinions if a woman inserts the penis of an animal (into her vagina), and if she inserts a separate penis; The most accurate is that the woman’s genitals should be washed.

അത്തരമൊരു രതി വൈകൃതത്തിൽ ഏർപ്പെട്ടാൽ മഹാപാപിയായി പരിണമിക്കുമെങ്കിലും ആ വൻപാപം മൂലം വ്രതമോ ഹജ്ജ് കർമ്മമോ അസാധുവായി പോവുമോ എന്ന ചർച്ചയും ചില കർമ്മശാസ്ത്ര പണ്ഡിതർ നടത്തിയിട്ടുണ്ടാവാം. അതാണ് ഈ വാചകത്തിന്റെ പൊരുൾ: “According to some Sunni Islamic scholars, sexual intercourse with an animal does not invalidate fasting or Hajj.” മൃഗരതിയുടെ ധാർമ്മിക വിധിയല്ല ഇവിടെയും അന്വേഷിക്കപ്പെടുന്നത്.

പൈശാചിക സ്വാധീനത്താൽ ഒരു ദുർബല നിമിഷത്തിൽ മൃഗ രതിയിൽ ഒരാൾ ഏർപ്പെട്ടു എന്ന് കരുതുക. ചിലപ്പോൾ, മൃഗരതി ലൈംഗിക അവകാശത്തിന്റെ ഭാഗമായി കാണുന്ന ഒരു എക്സ് മുസ്‌ലിമിന്റെയൊ ഫ്രീതിങ്കേഴ്സിന്റെയൊ അവകാശ വാദങ്ങളിൽ സ്വാധീനിക്കപ്പെട്ട് കൊണ്ട് അങ്ങനെയൊന്ന് ഒരാളിൽ നിന്ന് സംഭവിച്ചു എന്ന് കരുതുക. ശേഷം, അയാൾ, “മൃഗങ്ങളുമായി രതിയിൽ ഏർപ്പെട്ടവനെ ദൈവം ശപിച്ചിരിക്കുന്നു…”(മുസ്നദ് അഹ്മദ്: 2816) എന്ന നബിയുടെ (സ) ഹദീസിൽ നിന്ന് ആ പാപത്തിന്റെ ഭയാനകത മനസ്സിലാക്കുകയും ചെയ്തു. ഇത്തരമൊരു സാഹചര്യത്തിൽ തുടർന്ന് അയാൾ പാലിക്കേണ്ട ശുദ്ധി, ആരാധനാ കർമ്മങ്ങൾ എന്നിങ്ങനെയുള്ള പശ്ചാത്താപ-ആത്മീയ അനുഷ്ഠാനങ്ങളെ സംബന്ധിച്ച വിധിവിലക്കുകൾ കർമ്മശാസ്ത്രം ചർച്ച ചെയ്യേണ്ടതില്ലെ ?! അത്തരം തുടർന്നടപടികൾ ചർച്ച ചെയ്താൽ അതിനർത്ഥം ആ വൻപാപത്തെ ആ കർമ്മശാസ്ത്ര പണ്ഡിതർ അനുകൂലിക്കുന്നു എന്നാണെന്ന് വർഗീയ തിമിരം ബാധിച്ചവരല്ലാതെ വാദിക്കുകയില്ല.

ചുരുക്കത്തിൽ, മൃഗരതി നന്മയാണോ തിന്മയാണോ എന്നതല്ല ഇവിടെയൊന്നും ചർച്ച. ആ തിന്മ ഒരാൾ ചെയ്താൽ മറ്റു കർമ്മങ്ങളെ അത് എപ്രകാരം ബാധിക്കും എന്നാണ് ചർച്ച. അങ്ങനെ സംഭവിച്ചാൽ തുടർന്നുള്ള വിധികളെ സംബന്ധിച്ച ചില പണ്ഡിതരുടെ ചർച്ചകൾ “ഇസ്‌ലാമിന്റെ വിശുദ്ധ പ്രമാണങ്ങൾ” എന്ന വ്യാജേന അവതരിപ്പിക്കൽ ഭൗതികവാദികളുടെ സ്ഥിരം ഗവേഷക സംസ്കാരമാണ്.

Comment: The sex or separate male genitalia of any animal of Momina can be inserted into its female genitalia.

മൃഗരതിയുടെ ധാർമ്മിക വിധി ചർച്ച ചെയ്തു കൊണ്ട് ഇമാം നവവിയൊ മറ്റൊരു മുസ്‌ലിം പണ്ഡിതനൊ ഇത്തരമൊരു അഭിപ്രായം കുറിച്ചിട്ടിട്ടില്ല. ഇത്തരമൊരു വാചകം ഭൗതീകവാദികൾക്ക് കിട്ടിയത് “ഇമാം” ഡോകിൻസിൽ നിന്നൊ “ഇമാം” സാം ഹാരിസിൽ നിന്നൊ ഒക്കെയാവാനാണ് സാധ്യത.!!

വിമർശനം:

പ്രവാചക പത്നി ഹഫ്‌സയുടെ വീട്ടിൽ വെച്ച് മാരിയത്ത് എന്ന അടിമ സ്ത്രീയുമായി മുഹമ്മദ് നബി ലൈംഗിക ബദ്ധത്തിൽ ഏർപ്പെട്ടു. ഇത് ഹഫ്‌സ അറിഞ്ഞപ്പോൾ അവരോട് അക്കാര്യം ആരുമായും പങ്കു വെക്കരുത് എന്ന് നബി അപേക്ഷിക്കുകയും ചെയ്തു.

മറുപടി:

ഒരു വ്യാജ നിവേദനത്തിൻമേൽ കെട്ടിപടുത്ത കാൽപനിക സൗധമാണ് ഈ കഥ. പ്രവാചകൻ (സ), തന്റെ പത്നി ഹഫ്‌സയുടെ വീട്ടിൽ വെച്ച് “അവിഹിത”ത്തിൽ ഏർപ്പെടുകയും ഭാര്യ “പിടികൂടിയപ്പോൾ” ഈ “അവിഹിതം” രഹസ്യമാക്കി വെക്കാൻ ഭാര്യയോട് കേണപേക്ഷിച്ചു എന്നൊക്കെയാണ് വിമർശകരുടെ തിരക്കഥ!! അതിനായി അവർ അവലംബിക്കുന്ന വ്യാജവും അങ്ങേയറ്റം ദുർബലവുമായ നിവേദനങ്ങളിലാകട്ടെ വിമർശകരുടെ ഈ ഭാവനാചിത്രണങ്ങൾ ഒന്നും ഉള്ളടങ്ങിയിട്ടില്ല എന്നതാണ് യാഥാർത്ഥ്യം.

വിഷയകമായ ദുർബല ഹദീസിന്റെ ഹ്രസ്വരൂപം ആദ്യം നമുക്ക് പരിചയപ്പെടാം: പ്രവാചകന്റെ(സ) അടിമസ്ത്രീയായിരുന്നു മാരിയ. ഒരിക്കൽ ഹഫ്‌സയുടെ വീട്ടിൽ വെച്ച് പ്രവാചകൻ (സ) അവരുമായി ശാരീരിക ബന്ധത്തിലേർപ്പെട്ടു. ഇത് മനസ്സിലാക്കിയപ്പോൾ ഹഫ്‌സ (റ) പ്രവാചകനോട് ഇപ്രകാരം പരാധിപ്പെട്ടു:

“താങ്കൾ എന്റെ വീട്ടിൽ വെച്ച് ‘മാരിയ’യുമായി ബന്ധപ്പെട്ടു എന്നത് (എന്നോട് താങ്കൾക്കുള്ള അതൃപ്തിയായി) മറ്റുള്ള ഭാര്യമാർക്കിടയിൽ (തെറ്റിദ്ധാരണ ജനിപ്പിച്ചേക്കാം എന്നതിനാൽ) എനിക്ക് അപമാനത്തിന് കാരണമായേക്കും.”

അപ്പോൾ പ്രവാചകൻ (സ) ‘ഈ സംഭവം ആരോടും പങ്കു വെക്കേണ്ട’ എന്ന് അവരെ ഉപദേശിച്ചു. തുടർന്ന് ‘മാരിയ’യുമായി മേലിൽ ശാരീരിക ബന്ധം സ്ഥാപിക്കില്ലെന്നും പ്രവാചകൻ (സ) പ്രസ്ഥാവിച്ചു. അപ്പോൾ ഹഫ്‌സ ചോദിച്ചു: ( وكيف تحرم عليك وهي جاريتك) “അവരുമായുള്ള ശാരീരിക ബന്ധം താങ്കൾക്കെങ്ങനെ നിഷിദ്ധമാകും; അവർ താങ്കളുടെ അടിമ സ്ത്രീ ആയിരിക്കെ ?!”

‘മാരിയ’യെ സമീപിക്കില്ലെന്ന് ഹഫ്‌സയോട് ശപഥം ചെയ്ത പ്രവാചകന്റെ(സ) നിലപാട് തിരുത്തി കൊണ്ട് വിശുദ്ധ ക്വുർആനിൽ ചില വചനങ്ങൾ അവതരിപ്പിക്കപ്പെടുന്നു: “നബിയേ, നീയെന്തിനാണ് ഭാര്യമാരുടെ പ്രീതി കാംക്ഷിച്ച് അല്ലാഹു അനുവദനീയമാക്കിയത് നിഷിദ്ധമാക്കുന്നത്?”(സൂറത്തു തഹ്‌രീം: 1). (മുഅ്ജമുൽ അവ്സത്: ത്വബ്റാനി: 2316, സുനനു ദാറകുത്നി: 122)

മാരിയ പ്രവാചകന്റെ അടിമസ്ത്രീയാണ്. അടിമസ്ത്രീകളുമായി അവരുടെ ഹിതപ്രകാരം ബന്ധം സ്ഥാപിക്കാൻ ഇസ്‌ലാം പുരുഷന്മാർക്ക് അനുവാദം നൽകിയിട്ടുണ്ട്. (അടിമ സ്ത്രീകളുമായുള്ള ലൈംഗികബന്ധം ഇസ്‌ലാം അനുവദിച്ചതിലെ യുക്തിയും നീതിയും വിശാലമായ മറ്റൊരു ചർച്ചയാണ്. അതിന് ഇവിടെ മുതിരുന്നില്ല. ചുരുങ്ങിയ വായനക്കായി: (https://www.snehasamvadam.org/responds-to-criticism/അടിമസ്ത്രീകളുമായി-ബന്ധം/)

ഈ ബന്ധം ഒരു “അവിഹിത”മായി അക്കാലഘട്ടത്തിലെ ഒരു രാജ്യത്തിലേയും, ഒരു മതവും, ഒരു മത സമൂഹവും, മതരഹിതരുമടക്കം ആരും കരുതിയിട്ടില്ല എന്നിരിക്കെ ഇസ്‌ലാമിക സമൂഹത്തിൽ മാത്രം നിലനിന്നിരുന്ന “അവിഹിത”മായി ഈ ബന്ധത്തെ വീക്ഷിക്കുന്നതിലെ വർഗീയ വിവേചനം വ്യക്തമാണ്. പ്രവാചകന്റെ നാട്ടിലെ അമുസ്‌ലിംകൾ പോലും വിഹിതവും വിവാഹ തുല്യവുമായി കണ്ട ഒന്നാണ് അടിമസ്ത്രീ – യജമാന ബന്ധം എന്നതിന് ഏറ്റവും നല്ല തെളിവാണ് പ്രവാചക പത്നി ഹഫ്‌സയുടെ(റ) (ഹദീസിലെ) വാചകം: “‘മാരിയ’യുമായുള്ള ശാരിരിക ബന്ധം താങ്കൾക്കെങ്ങനെ നിഷിദ്ധമാകും; അവർ താങ്കളുടെ അടിമ സ്ത്രീ ആയിരിക്കെ ?!”.

പ്രവാചകൻ (സ) അവിഹിതത്തിൽ ഏർപ്പെട്ടുവെന്നോ, തന്റെ വീട്ടിൽ വെച്ച് അവിഹിതത്തിൽ മുഴുകിയെന്നോ ഒന്നും ഹഫ്‌സ (റ) പരാതിപ്പെട്ടിട്ടില്ല എന്നും (ദുർബല) ഹദീസിൽ നിന്ന് സുവ്യക്തം. തന്റെ വീട്ടിൽ വെച്ച് ‘മാരിയ’യുമായി ബന്ധപ്പെട്ടു എന്നത് തന്നോട് പ്രവാചകനുള്ള അതൃപ്തിയായി മറ്റുള്ള ഭാര്യമാർ തെറ്റിദ്ധരിക്കാൻ ഇടയാകുമെന്നും ഇത് തനിക്ക് അപമാനത്തിന് കാരണമായേക്കും എന്ന് മാത്രമാണ് ഹഫ്‌സയുടെ (റ) പരാതി. എങ്കിൽ ഇക്കാര്യം ആരുമായും പങ്കു വെക്കേണ്ടതില്ല എന്ന് പ്രവാചകൻ (സ) മറുപടി പറഞ്ഞു. അല്ലാതെ – വിമർശകർ ദുർവ്യാഖ്യാനിക്കുന്നതു പോലെ- തന്റെ “അവിഹിതം” ആരോടും വെളിപ്പെടുത്തരുതേ എന്ന ഭാര്യയോടുള്ള അപേക്ഷയൊന്നുമായിരുന്നില്ല അത്.

എല്ലാത്തിനുമുപരി വിമർശകർ ഓർക്കേണ്ട ഏറ്റവും സുപ്രധാനമായ കാര്യം മാരിയയുടെ ഈ കഥ സ്ഥിരപ്പെട്ട ഒരു സംഭവമൊ ഇസ്‌ലാമിന്റെയൊ പ്രവാചക ചരിത്രത്തിന്റെ ഭാഗമൊ അല്ല എന്നതാണ്.

മാരിയയുമായി ബന്ധപ്പെട്ട കഥയുടെ സനദ് അഥവാ നിവേദക പരമ്പര:

ഹഫ്‌സയുടെ (റ) വീട്ടിൽ വെച്ച് മാരിയയുമായി (റ) പ്രവാചകൻ (സ) ബന്ധത്തിലേർപ്പെട്ടു എന്ന വ്യാജ കഥയുടെ ഇരുപതോളം സനദുകൾ (നിവേദക പരമ്പര) ഓരോ മുടിനാരിഴ കീറി മുറിച്ച് പഠനം ത്വാഹിർ ഇബ്നു സഈദ് ഇബ്നു അബ്ദുർ റഊഫ് അസ്സയിൽഹീതി നടത്തിയിട്ടുണ്ട്. മാരിയയുടെ കഥയുമായി ബന്ധപ്പെട്ട എല്ലാ സനദുകളും വ്യാജങ്ങളൊ (الباطل), കള്ള ഹദീസുകളൊ (الموضوع) അങ്ങേയറ്റം ദുർബലമൊ (ضعيف جدا) ഒക്കെയാണ് എന്ന് തെളിവു സഹിതം സമർത്തിക്കുന്നുണ്ട് അദ്ദേഹം. സനദു സമ്പന്ധമായ ആ ദീർഘമായ ചർച്ച പഠനവിധേയമാക്കാൻ ആഗ്രഹിക്കുന്നവർ ആ ലേഖനം വായിക്കുക. (http://darhadith.com/pubs/asbab-nuzl-sura-tahrim-maria/)

ആയത്തിന്റെ ശരിയായ അവതരണ പശ്ചാത്തലം:

മാരിയയുമായി ബന്ധപ്പെട്ട് ചർച്ചാ വിഷയകമായ നിവേദനങ്ങൾ വ്യാജവും അങ്ങേയറ്റം ദുർബലവുമാണ് എന്നതിന് പുറമെ ഇസ്‌ലാമിക പ്രമാണങ്ങൾക്കും സ്വഹീഹായ ഹദീസുകൾക്കും എതിരുമാണ്. മാരിയയുമായി ബന്ധപ്പെട്ട (ദുർബല) നിവേദനത്തിൽ പ്രസ്ഥാവിക്കുന്നത്, സൂറത്തു തഹ്‌രീമിലെ ആയത്തുകൾ (ദിവ്യ വചനങ്ങൾ) അവതരിപ്പിക്കപ്പെടാനുള്ള പശ്ചാത്തലം മാരിയയുമായുള്ള ശാരീരിക ബന്ധം പ്രവാചകൻ (സ) സ്വന്തത്തിനു മേൽ നിഷിദ്ധമായി പ്രഖ്യാപിക്കുകയും, ശപഥം ചെയ്യുകയും ചെയ്തു എന്നതുമാണ്. ഇത് സ്വഹീഹായ ഒട്ടനവധി നിവേദനങ്ങൾക്കും ഹദീസുകൾക്കും വിരുദ്ധമാണ്.

സൂറത്തു തഹ്‌രീമിലെ ആയത്തുകളുടെ യഥാർത്ഥ അവതരണ പശ്ചാത്തലം ഇപ്രകാരമാണ്: മുഹമ്മദ് നബി (സ) തന്റെ പത്നിയായ സൈനബ് ബിൻത് ജഹ്ശിന്റെ (റ) അടുക്കൽ താമസിക്കുമ്പോൾ തേൻ കുടിക്കുക പതിവായിരുന്നു. ഇത് ഇഷ്ടപ്പെടാതിരുന്ന ആഇശ (റ), ഹഫ്‌സ (റ) എന്നീ പത്നിമാർ, തേൻ കുടിക്കുന്നത് മൂലം ‘മഗാഫിർ’ കുടിച്ചാലുണ്ടാവുന്ന ദുർഗന്ധം പോലെ അനുഭവപ്പെടുന്നു എന്ന് അഭിപ്രായപ്പെട്ടു. എങ്കിൽ താൻ ഇനി തേൻ കുടിക്കില്ലെന്ന് പ്രവാചകൻ (സ) ശപഥം ചെയ്തു. പ്രവാചകന്റെ(സ) ഈ നിലപാടിനെ തിരുത്തി കൊണ്ട് ക്വുർആൻ വചനം അവതരിപ്പിക്കപ്പെട്ടു: “നബിയേ, നീയെന്തിനാണ് ഭാര്യമാരുടെ പ്രീതി കാംക്ഷിച്ച് അല്ലാഹു അനുവദനീയമാക്കിയത് നിഷിദ്ധമാക്കുന്നത്?”(സൂറത്തു തഹ്‌രീം: 1). പ്രവാചക പത്നി ആഇശയിൽ (റ) നിന്ന് ഒട്ടനവധി ഹദീസ് ഗ്രന്ഥങ്ങളിൽ, സ്വഹീഹായ സനദോടു(നിവേദക പരമ്പര)കൂടെ ഉദ്ധരിക്കപ്പെട്ട ഹദീസ് ആകുന്നു ഇത്. (സ്വഹീഹുൽ ബുഖാരി: 4966, സ്വഹീഹു മുസ്‌ലിം: 2787, സുനനു അബൂദാവൂദ്: 3281, തുർമുദി: 1834, ഇബ്നുമാജ: 3342, മുസ്നദു അഹ്മദ്: 23769, ഇബ്നു ഹിബ്ബാൻ: 5344, നസാഈ: 3645, ഹാകിം: 7174, ദാരിമി: 2062)

മാരിയയുമായി പ്രവചകൻ (സ) ഹഫ്‌സയുടെ വീട്ടിൽ വെച്ച് ബന്ധപ്പെട്ടു എന്ന കഥ വ്യാജമൊ അങ്ങേയറ്റം ദുർബലമൊ ആകുന്നു എന്നാണ് ഹദീസ്-ചരിത്ര പണ്ഡിതന്മാർ അഭിപ്രായപ്പെട്ടത്:

ഉകൈലി (മരണം: ഹി: 322) പറഞ്ഞു: “(‘മാരിയ’യുമായി ബന്ധപ്പെട്ട) ഈ കഥ പ്രമാണ വിരുദ്ധവും വ്യാജവുമാണ്.” (മീസാനുൽ ഇഅ്തിദാൽ: 4:201)

ഇമാം ദഹബി (ജനനം: ഹി: 673) ഈ കഥ പ്രമാണ വിരുദ്ധവും നുണ കഥയുമാണെന്നും വിധി പറഞ്ഞിരിക്കുന്നു. (ലിസാനുൽ മീസാൻ: 8:191)

നൂറുദ്ദീൻ അൽ ഹൈസമി (ജനനം: ഹി: 735) ഈ കഥ ദുർബലമാണെന്ന് തന്റെ ‘മജ്‌മഉസ്സവാഇദ്’ (5:10) എന്ന ഗ്രന്ഥത്തിൽ മൂന്നിടങ്ങിൽ വ്യക്തമാക്കിയിരിക്കുന്നു.

ഇബ്നുൽ അറബി (ജനനം: ഹി: 558) പറഞ്ഞതായി ഇമാം ക്വുർതുബി രേഖപ്പെടുത്തുന്നു: “(‘മാരിയ’യുമായി ബന്ധപ്പെട്ട കഥയുടെ) ദുർബലത, അതിന്റെ നിവേദകന്മാർ വിശ്വസ്തരും നീതിമാന്മാരുമല്ല എന്നതാണ്.” (തഫ്സീറുൽ ക്വുർതുബി: 18: 179)

കാദി ഇയാദ് (ജനനം: ഹി: 476) പറഞ്ഞു: “നബിയേ, നീയെന്തിനാണ് ഭാര്യമാരുടെ പ്രീതി കാംക്ഷിച്ച് അല്ലാഹു അനുവദനീയമാക്കിയത് നിഷിദ്ധമാക്കുന്നത്?” എന്ന ക്വുർആൻ വചനം (സൂറത്തു തഹ്‌രീം: 1) അവതരിപ്പിക്കപ്പെടാനുള്ള പശ്ചാത്തലത്തെ സംബന്ധിച്ച് വ്യത്യസ്ത നിവേദനങ്ങൾ ഉദ്ധരിക്കപ്പെട്ടിട്ടുണ്ട്. പ്രവാചകൻ തേൻ കുടിച്ചതുമായി ബന്ധപ്പെട്ട സംഭവത്തിലാണ് വചനം ഇറക്കപ്പെട്ടത് എന്നാണ് പ്രവാചക പത്നി ആഇശ (റ) വ്യക്തമാക്കുന്നത്… സ്വഹീഹായ (സ്വീകാര്യയോഗ്യം) ഒരു പരമ്പരയിലൂടെയും സ്ഥാപിതമായിട്ടില്ലാത്ത ‘മാരിയ’യുമായി ബന്ധപ്പെട്ട കഥയല്ല അവതരണ പശ്ചാത്തലം.” (അൽ ബദറുൽ മുനീർ: ഇബ്നു മുലക്കിൻ: 8:79)

ഇമാം സ്വൻആനി ഈ അഭിപ്രായം തന്നെ ആവർത്തിക്കുന്നു. (അത്തഹ്‌രീറുൽ ഈദാഹ് മആനി തയ്സീർ: 2: 418)

പരമ്പര മുറിഞ്ഞ കാരണത്താൽ ഇമാം മിസ്സിയും (ജനനം: ഹി 654) ഈ കഥ ദുർബലമാണെന്ന് വ്യക്തമാക്കുന്നു. (തുഹ്ഫതുൽ അശ്റാഫ് ബി മഅ്’രിഫതിൽ അത്റാഫ്: 12: 170)

ഇബ്നു കുദാമ (മുഗ്നി: 9:509), ഇബ്നു ആശൂർ (അത്തഹ്‌രീർ വത്തൻവീർ: 28: 345), ഇബ്നു ഉസൈമീൻ (ഫത്ഹു ദിൽ ജലാലി വൽ ഇക്റാം: 5:71) എന്നീ പണ്ഡിതരും മാരിയയുമായ ബന്ധപ്പെട്ട ഈ കഥ ദുർബലമാണെന്ന് അഭിപ്രായപ്പെടുന്നുണ്ട്.

ശൈഖ് സ്വുഹൈബ് ഹസൻ ഇബ്നു അബ്ദുൽ ഗഫ്ഫാറിനോട് ഈ കഥയെ സംബന്ധിച്ച് ചോദിക്കപ്പെട്ടപ്പോൾ അദ്ദേഹം പൊട്ടിച്ചിരിച്ചു കൊണ്ട് പറഞ്ഞു: “ഒരു പരിഗണനയും അർഹിക്കാത്തതാണ് ഈ കഥ.”

മുബാറക്ഫൂരി രേഖപ്പെടുത്തി: “ഖാസിൻ (ജനനം: ഹി 678) തന്റെ തഫ്സീറിൽ പറഞ്ഞു: പണ്ഡിതന്മാർ ഇപ്രകാരം പ്രസ്ഥാവിച്ചിരിക്കുന്നു: ക്വുർആൻ വചനം (സൂറത്തു തഹ്‌രീം: 1) അവതരിപ്പിക്കപ്പെടാനുള്ള പശ്ചാത്തലത്തെ സംബന്ധിച്ച സ്വഹീഹായ (സ്വീകാര്യയോഗ്യം) നിവേദനം, പ്രവാചകൻ തേൻ കുടിച്ചതുമായി ബന്ധപ്പെട്ട സംഭവത്തിലാണ്, അല്ലാതെ മാരിയയുമായി ബന്ധപ്പെട്ട കഥയല്ല അവതരണ പശ്ചാത്തലം. മാരിയയുമായി ബന്ധപ്പെട്ട കഥ ഒരു സ്വഹീഹായ പരമ്പരയിലൂടെയും വന്നിട്ടില്ല. ഇമാം നസാഈ പറഞ്ഞു: തേൻ കുടിച്ചതുമായി ബന്ധപ്പെട്ട, പ്രവാചക പത്നി ആഇശ ഉദ്ധരിച്ച സംഭവം അങ്ങേയറ്റം സ്വഹീഹ് ആണ്.” (തുഹ്‌ഫതുൽ അഹ്‌വദി: 9 : 162)

ഇമാം നവവി പറഞ്ഞു: “ക്വുർആൻ വചനം (സൂറത്തു തഹ്‌രീം: 1) അവതരിപ്പിക്കപ്പെടാനുള്ള പശ്ചാത്തലത്തെ സംബന്ധിച്ച നിവേദനങ്ങളിൽ സ്വഹീഹായ (സ്വീകാര്യയോഗ്യം) നിവേദനം, പ്രവാചകൻ തേൻ കുടിച്ചതുമായി ബന്ധപ്പെട്ട സംഭവത്തിലാണ് വചനം ഇറക്കപ്പെട്ടത് എന്നാണ്. അല്ലാതെ സ്വഹീഹുൽ ബുഖാരിയിലൊ മുസ്‌ലിമിലൊ വന്നിട്ടില്ലാത്ത, ‘മാരിയ’യുമായി ബന്ധപ്പെട്ട കഥയല്ല അവതരണ പശ്ചാത്തലം. മാരിയയുമായി ബന്ധപ്പെട്ട കഥ ഒരു സ്വഹീഹായ പരമ്പരയിലൂടെയും വന്നിട്ടില്ല. ഇമാം നസാഈ പറഞ്ഞു: തേൻ കുടിച്ചതുമായി ബന്ധപ്പെട്ട പ്രവാചക പത്നി ആഇശ ഉദ്ധരിച്ച സംഭവം അങ്ങേയറ്റം സ്വഹീഹ് ആണ്. ഇതാണ് ഇമാം കാദി ഇയാദിന്റെയും അഭിപ്രായം.” (ശർഹു മുസ്‌ലിം: 1:77)

ശൈഖ് അബൂ ഹംസ പറഞ്ഞു: “മാരിയയുമായി ബന്ധപ്പെട്ട കഥ സ്വഹീഹായ ഒരു പരമ്പരയിലൂടെയും ഉദ്ധരിക്കപ്പെട്ടിട്ടില്ല എന്ന് കാദി ഇയാദ് പറഞ്ഞിരിക്കുന്നു.”

ഇമാം ഇബ്നുൽ അറബി പറഞ്ഞു: “മാരിയുമായുള്ള ലൈംഗികത പ്രവാചകൻ (സ) സ്വയം നിരോധിച്ചുവെന്ന കഥ സ്വഹീഹായ ഒരു സനദിലൂടെയും ക്രോഡീകരിക്കപ്പെട്ടിട്ടില്ല, നീതിമാന്മാരും വിശ്വസ്തരുമായ നിവേദകന്മാരല്ല അത് ഉദ്ധരിച്ചിരിക്കുന്നത്. പരമ്പര മുറിഞ്ഞ നിലയിലും ഈ കഥ ഉദ്ധരിക്കപ്പെട്ടിട്ടുണ്ട്… സ്വഹീഹായ നിവേദനം ക്വുർആൻ വചനം (സൂറത്തു തഹ്‌രീം: 1) അവതരിപ്പിക്കപ്പെടാനുള്ള പശ്ചാത്തലം പ്രവാചകന്റെ(സ) തേൻ കുടിയുമായി ബന്ധപ്പെട്ടതാണ്. സൈനബിന്റെ അടുക്കൽ വെച്ചാണ് അദ്ദേഹമത് കുടിച്ചത്.” (അഹ്കാമുൽ ക്വുർആൻ: 4:252)

ഖത്വാബി (ജനനം: ഹി 319) പറഞ്ഞു: “പ്രവാചകൻ (സ) തന്റെ അടിമ സ്ത്രീയായ മാരിയയുമായുളള ശാരീരിക ബന്ധം സ്വയം വർജിച്ചതുമായി ബന്ധപ്പെട്ട കഥയിലാണ് ‘പ്രവാചക ശപഥം’ നടന്നത് എന്ന ചിലരുടെ വാദം ശരിയല്ലെന്നും, തേൻ കുടിക്കുന്നത് പ്രവാചകൻ (സ) സ്വയം നിഷിദ്ധമാക്കിയതുമായി ബന്ധപ്പെട്ടാണ് ‘പ്രവാചക ശപഥം’ നടന്നത് എന്നും ഈ ഹദീസ് തെളിയിക്കുന്നു”. (മആലിമുസുനന് : 4:272)

വിമർശനം: ഒറ്റ ദിവസം തന്നെ എല്ലാ ഭാര്യമാരോടൊപ്പവും ലൈംഗിക ബന്ധം സ്ഥാപിച്ചുവെന്നത് മുഹമ്മദ് നബി ഒരു സ്ത്രീലമ്പടനായിരുന്നു എന്നതിന് തെളിവല്ലെ ? മറുപടി: മുഹമ്മദ് നബിക്കെതിരെ(സ) ഈ വ്യാജ ആരോപണം അഴിച്ചു വിടാൻ വിമർശകർ അവലംബിക്കുന്ന ഹദീസ് ആദ്യം നമുക്കൊന്ന് വായിക്കാം: അനസ് (റ) പറഞ്ഞു: പ്രവാചകൻ (സ), പകലിൽ നിന്നും രാത്രിയിൽ നിന്നും ഒരൊറ്റ ചുറ്റലിൽ തന്റെ ഭാര്യമാരെയെല്ലാം സന്ദർശിക്കാറുയുണ്ടായിരുന്നു. അവർ പതിനൊന്നു പേരുണ്ടായിരുന്നു. ഞാൻ (കത്താദ) അനസിനോട് ചോദിച്ചു: അദ്ദേഹത്തിന് (പ്രവാചകന്) അത് സാധിക്കുമായിരുന്നോ ? അപ്പോൾ അനസ് (റ) പറഞ്ഞു: അദ്ദേഹത്തിന് മുപ്പതു പേരുടെ കരുത്ത് നൽകപ്പെട്ടിട്ടുണ്ടെന്ന് ഞങ്ങൾ പരസ്പരം സംസാരിക്കാറുണ്ടായിരുന്നു. (സ്വഹീഹുൽ ബുഖാരി: 268) പ്രവാചകൻ (സ) ഒരു സ്ത്രീലമ്പടനുമായിരുന്നു എന്ന് ആക്ഷേപിക്കാനായി വിമർശകർ വികലമായ അർത്ഥം നൽകി വിവാദവൽക്കരിക്കാറുള്ള ഒരു ഹദീസാണ് ഇത്. ഒരു ദിവസം തന്നെ പതിനൊന്ന് ഭാര്യമാരുമായി ലൈംഗിക ബന്ധത്തിൽ ഏർപ്പെടുക എന്നത് അദ്ദേഹത്തിന്റെ അമിതാസക്തിയും സ്‌ത്രീലോലുപത്വവുമാണ് തെളിയിക്കുന്നത് എന്ന് അസഭ്യഭാഷയിൽ അവതരിപ്പിക്കുകയാണ് നബിവിമർശകർ ചെയ്യാറുള്ളത്. വാസ്ഥവത്തിൽ, ഹദീസിൽ പ്രസ്ഥാവിക്കപ്പെട്ട സംഭവത്തിന്റെ യാഥാർത്ഥ്യം മനസ്സിലാക്കിയാൽ നബി വിമർശകർ സൃഷ്ടിച്ചെടുക്കുന്ന അശ്ലീല വ്യാഖ്യാനത്തിനൊന്നും യാതൊരു അടിത്തറയുമില്ലെന്ന് ആർക്കും വ്യക്തമാകുന്നതാണ്. 1. പ്രവാചകന്റെ(സ) പത്നിമാർക്കെല്ലാം അദ്ദേഹം പ്രിയങ്കരനായിരുന്നു. കേവല ശരീരങ്ങൾ തമ്മിലുള്ള ബന്ധം മാത്രമായിരുന്നില്ല അവർ തമ്മിലുള്ള ദാമ്പത്യം. അദ്ദേഹത്തോടൊപ്പമുള്ള സഹവാസവും സമ്പർക്കവും ആത്മീയമായ നേട്ടത്തിന് പുറമെ അവർക്ക് മാനസികവും ഭൗതീകവുമായ ആസ്വാദനവും സമാധാനവും അവലംബവുമായിരുന്നു. അതുകൊണ്ട് തന്നെ അദ്ദേഹത്തിന്റെ സാന്നിധ്യത്തിനായി അവർ ഓരോരുത്തരും അതിയായി ആഗ്രഹിച്ചിരുന്നു. അവരുടെ ആഗ്രഹം കണക്കിലെടുത്തു കൊണ്ടും ഭാര്യമാർക്കിടയിൽ നീതി പാലിക്കുക എന്ന ആദർശനിഷ്ട പ്രയോഗവൽകരിച്ചു കൊണ്ടും ഓരോ ഭാര്യക്കും ഓരോ ദിവസം പ്രവാചകൻ (സ) വീതിച്ചിരുന്നു എന്നും ചില യാത്രകളിൽ കൂടെ കൂട്ടേണ്ടതാരെയാണെന്ന് തിരഞ്ഞെടുക്കാനായി അവർ നറുക്കിട്ടിരുന്നു എന്നും പ്രവാചക പത്നിമാർ തന്നെ പ്രസ്ഥാവിച്ചിട്ടുണ്ട്. (സ്വഹീഹുൽ ബുഖാരി: 2593) ഓരോരുത്തർക്കായും വീതം വെച്ച ദിവങ്ങൾക്കു പുറമെ ചില ദിവസങ്ങളും ഉണ്ടാകാറുണ്ട്. ദീർഘയാത്രകൾ, ആരാധനാ- തീർത്ഥാടന കർമ്മങ്ങൾ തുടങ്ങി പലതിനുമായും ഒഴിച്ചു വെക്കപ്പെട്ട ഇത്തരം ദിവസങ്ങൾ പ്രത്യേകിച്ച് ഒരു ഭാര്യക്കും അവകാശപ്പെട്ടതല്ലാത്തവയാണ്. ഇത്തരം അവസരങ്ങളിൽ ഭാര്യമാരുമായി പിരിയുകയോ അവരിലേക്ക് തിരിച്ചു വരുകയോ ഒക്കെ ചെയ്യുന്ന സന്ദർഭത്തിൽ എല്ലാവരുമായി പ്രവാചകൻ (സ) അൽപ്പനേരം സമ്പർക്കം പുലർത്തുകയും സഹവസിക്കുകയും ചെയ്യുമായിരുന്നു. ഇതാണ്, “പ്രവാചകൻ (സ) ഒറ്റ ദിവസം തന്നെ എല്ലാ ഭാര്യമാരേയും ചുറ്റിസഞ്ചരിക്കുമായിരുന്നു” എന്ന ഹദീസുകളുടെ ഉള്ളടക്കത്തിന്റെ വിവക്ഷ. ഒരു സ്ത്രീലോലുപതയും അമിതാസക്തിയുമില്ലാത്ത സ്വാഭാവിക സന്ദർശനം!! പ്രവാചകന്റെ(സ) ‘ഒറ്റ ദിവസത്തെ കൂട്ട സന്ദർശനത്തിന്റെ’ ലക്ഷ്യമെന്തായിരുന്നെന്ന് അദ്ദേഹത്തിന്റെ പത്നിമാർ തന്നെ വ്യക്തമായി വിശദീകരിച്ചിട്ടുണ്ട്. പ്രവാചകന്റെ (സ) ഭാര്യമാരുടെ അടുത്തേക്കുള്ള അദ്ദേഹത്തിന്റെ ഈ സന്ദർശനത്തിന്റെ അർത്ഥവും ലക്ഷ്യവും എന്താണെന്ന് ഏറ്റവും നന്നായി അറിയുക പ്രവാചകന്റെ ഭാര്യമാർക്ക് തന്നെയാണല്ലൊ; അല്ലാതെ അനസിനെ(റ) പോലെയുള്ള പ്രവാചകന്റെ ഏതെങ്കിലും ശിഷ്യർക്കല്ല. عائشة رضي الله عنها، قالت: ( كان رسولُ اللهِ صلَّى الله عليه وسلم لا يُفضِّلُ بعضنا على بعضٍ في القَسمِ ، من مُكثه عِندنا ، وكان قلَّ يومٌ إلا وهو يَطُوفُ علينا جميعاً ، فيدنو مِنْ كُلِّ امرأة ، مِن غير مَسِيسٍ ، حتى يَبْلُغَ إلى التي هو يَوْمُها فيبيتُ عندها ) . പ്രവാചക പത്നി ആഇശ (റ) പറയുകയുണ്ടായി: “ഞങ്ങളുടെ അടുക്കൽ താമസിക്കുന്നതിനായി വിഭജിച്ച ദിവസങ്ങളിൽ ഒരു ഭാര്യക്ക് മറ്റൊരാളേക്കാൾ പ്രമാഖ്യം അല്ലാഹുവിന്റെ ദൂതൻ (സ) കാണിച്ചിരുന്നില്ല. ഞങ്ങളുടെ എല്ലാവരുടേയും അടുക്കൽ അദ്ദേഹം ചുറ്റിസഞ്ചരിക്കാത്ത ദിവസങ്ങൾ കുറവായിരുന്നു.’ലൈംഗിക ബന്ധത്തിൽ ഏർപ്പെടാതെ’ അദ്ദേഹം എല്ലാ ഭാര്യമാരുടെ അടുത്തേക്കും ചെല്ലുമായിരുന്നു. അവസാനം,(അടുത്ത് താമസിക്കുന്നതിനായി ഭാര്യമാർക്കിടയിൽ വിഭജിച്ച ദിവസങ്ങളിൽ) ഏതു ഭാര്യയുടെ ദിവസമാണോ ആ ഭാര്യയുടെ അടുക്കൽ രാപ്പാർക്കും.” (മുസ്നദ് അഹ്മദ്: 24765, സുനനു അബൂദാവൂദ്: 2135, സ്വഹീഹു അബൂദാവൂദ്: അൽബാനി: 1852) فيدنو مِنْ كُلِّ امرأة ، مِن غير مَسِيسٍ “ലൈംഗിക ബന്ധത്തിൽ ഏർപ്പെടാതെ അദ്ദേഹം എല്ലാ ഭാര്യമാരുടെ അടുത്തേക്കും ചെല്ലുമായിരുന്നു…” എന്ന് ഹദീസിൽ ആഇശ (റ) വളരെ വ്യക്തമായി, പ്രത്യേകം എടുത്തു പറയുന്നുണ്ട് എന്ന് കണ്ടുവല്ലൊ. പ്രവാചകൻ (സ) ഒറ്റ ദിവസം തന്നെ എല്ലാ ഭാര്യമാരേയും ‘സന്ദർശിക്കാറുണ്ടായിരുന്നു’, എല്ലാ ഭാര്യമാരുടേയും അടുത്ത് ‘ചുറ്റിസഞ്ചരിക്കാറുണ്ടായിരുന്നു’ (يَدُورُ – يَطُوفُ) എന്നിങ്ങനെയുള്ള പദപ്രയോഗങ്ങൾ കൊണ്ടുള്ള ഉദ്ദേശം സമ്പർക്കത്തിലൂടെയും സഹവാസത്തിലൂടെയും “അവരുമായി ബന്ധം പുതുക്കുകയാണ്” (ﻳُﺮَاﺩَ ﺑِﻪِ ﺗَﺠْﺪِﻳﺪُ اﻟْﻌَﻬْﺪِ ﺑِﻬِﻦَّ). അഥവാ, ദീർഘയാത്രകൾക്ക് മുമ്പോ ശേഷമോ “എല്ലാവരുമായി സമയം ചെലവഴിക്കുക” എന്ന സ്വാഭാവികമായ സന്ദർശനമാണ് ഉദ്ദേശ്യമെന്ന് പൗരാണികരും ആധുനികരുമായ പല പണ്ഡിതന്മാരും ആഇശയുടെ (റ) ഹദീസുമായി ബന്ധിപ്പിച്ചു കൊണ്ട് വ്യാഖ്യാനിച്ചിട്ടുണ്ട്. ذَكَرْتُهُ لِعَائِشَةَ فَقَالَتْ : يَرْحَمُ اللَّهُ أَبَا عَبْدِ الرَّحْمَنِ كُنْتُ أُطَيِّبُ رَسُولَ اللَّهِ صَلَّى اللَّهُ عَلَيْهِ وَسَلَّمَ فَيَطُوفُ عَلَى نِسَائِهِ ، ثُمَّ يُصْبِحُ مُحْرِمًا يَنْضَخُ طِيبًا ആഇശ (റ) പറഞ്ഞു: …ഞാൻ അല്ലാഹുവിന്റെ ദൂതന് (അദ്ദേഹത്തിന്റെ ശരീരത്തിൽ) സുഗന്ധം തേച്ച് കൊടുക്കുമായിരുന്നു. എന്നിട്ട് അദ്ദേഹം തന്റെ എല്ലാ ഭാര്യമാരേയും അടുത്ത് ‘ചുറ്റി സന്ദർശിക്കും’ (فَيَطُوفُ عَلَى نِسَائِهِ). പിന്നീട് രാവിലെ – (ഞാൻ തേച്ചു കൊടുത്ത) സുഗന്ധം, വമിക്കുന്ന നിലയിൽ അദ്ദേഹം ഇഹ്റാമിൽ പ്രവേശിക്കും. (സ്വഹീഹുൽ ബുഖാരി:267, സ്വഹീഹു മുസ്‌ലിം: 1192) ആഇശ (റ) ഈ പ്രസ്ഥാവിച്ച, ഒരു ദിവസം കൊണ്ട് എല്ലാ ഭാര്യമാരെയുമുള്ള സന്ദർശനത്തിൽ അവരുമായുള്ള ലൈംഗിക ബന്ധം ഉൾപ്പെട്ടിരുന്നെങ്കിൽ ദിവസത്തിന്റെ അന്ത്യത്തിൽ അദ്ദേഹത്തിന് കുളി നിർബന്ധമാകുമായിരുന്നു. കുളിച്ചിരുന്നെങ്കിൽ പിറ്റേന്ന് ഇഹ്റാമിൽ പ്രവേശിച്ചപ്പോൾ ശരീരത്തിൽ തേച്ച സുഗന്ധം അവശേഷിക്കില്ലായിരുന്നു. അത് പിറ്റേന്നും വമിക്കുന്നുണ്ടായിരുന്നു എന്ന ആഇശയുടെ പ്രസ്ഥാവനയിൽ നിന്നും പ്രസ്ഥുത സന്ദർശനം ലൈംഗിക ബന്ധത്തിന് വേണ്ടിയായിരുന്നില്ല, അവരുമായുള്ള സമ്പർക്കത്തിനും സഹവാസത്തിനും വേണ്ടി മാത്രമായിരുന്നു എന്ന് ഇസ്മാഈലി (ജനനം ഹിജ്റ 277) തന്റെ ‘സ്വഹീഹി’ൽ രേഖപ്പെടുത്തി. ഇസ്മാഈലിയിൽ നിന്നും ഈ വ്യാഖ്യാനം ഇബ്നു ഹജറും (ഫത്ഹുൽ ബാരി: 1:377), ബദറുദ്ദീൻ അൽഐനിയും (ഉംദത്തുൽ ക്വാരി: 3:213, 214) ഇബ്നു റജബും (ഫത്ഹുൽ ബാരി: 1:297) കസ്ത്വല്ലാനിയും (ഇർശാദുസ്സാരി: 1:325), ശൻക്വീതിയും (കൗസറുദ്ദുറാറി അൽ മആനി:5:418) എല്ലാം രേഖപ്പെടുത്തിയിട്ടുണ്ട്. 2. പക്ഷെ അനസിന്റെ ഹദീസിൽ “അദ്ദേഹത്തിന് മുപ്പത് ആളുകളുടെ കരുത്ത് നൽകപ്പെട്ടിട്ടുണ്ട്…” എന്നടങ്ങുന്ന പരാമർശത്തിൽ ഒറ്റ ദിവസം കൊണ്ട് എല്ലാ ഭാര്യമാരുമായുള്ള ‘ചുറ്റി സന്ദർശനത്തിൽ’ ലൈംഗിക ബന്ധവും ഉള്ളടങ്ങുന്നതായ സൂചനയുണ്ട് എന്നതാണ് വിവാദങ്ങൾക്ക് നിദാനം. സ്വഭാവം, ബുദ്ധി, ഭക്തി, ശരീരം തുടങ്ങി വ്യക്തിത്വത്തിന്റെ സർവ്വഭാവങ്ങളിലും സമ്പൂർണ്ണത നൽകപ്പെട്ടവരാണ് ദൈവദൂതന്മാർ. പുറമെ മറ്റു മനുഷ്യരിൽ നിന്ന് വ്യത്യസ്ഥമായി അവരിലൂടെ അല്ലാഹു പ്രകടമാക്കുന്ന മുഅ്ജിസത്തിൽ (അമാനുഷിക ദൃഷ്ടാന്തങ്ങൾ) ഉൾകൊള്ളുന്നതാണ് അവരുടെ അസാധാരണമായ കായിക ശക്തിയും ശാരീരിക ശേഷിയും. സ്വാഭാവികമായും മുഹമ്മദ് നബിക്കും(സ) അസാധാരണ ആരോഗ്യവും, അപാരമായ കായികശേഷിയും നൽകപ്പെട്ടിട്ടുണ്ടായിരുന്നു. സൂറത്തുൽ ബക്വറ, സൂറത്തു ആലു ഇംറാൻ, സൂറത്തുന്നിസാഅ് തുടങ്ങിയ ക്വുർആനിലെ നീളൻ അധ്യായങ്ങൾ പാരായണം ചെയ്ത് രാത്രി ഭൂരിഭാഗവും നിന്ന് നമസ്കരിക്കുമായിരുന്നു പ്രവാചകൻ (സ). (സ്വഹീഹു മുസ്‌ലിം: 772) ഇടമുറിയാതെ രണ്ടു ദിവസം ചേർത്ത് വ്രതം അനുഷ്ടിക്കുന്ന രീതിയായ ‘വിസ്വാൽ’ പ്രവാചകൻ (സ) അനുഷ്ടിച്ചിരുന്നു. നിങ്ങൾക്ക് അതിന് ശാരീരികമായി സാധിക്കില്ലെന്നും ഇത് തനിക്ക് മാത്രം നിശ്ചയിക്കപ്പെട്ട വ്രതത്തിന്റെ രീതിയാണെന്നും അദ്ദേഹം തന്റെ അനുചരന്മാരോട് ഉപദേശിക്കുകയും ചെയ്തിരുന്നു.(സ്വഹീഹുൽ ബുഖാരി:7299) ഖന്ദക്ക് യുദ്ധ സന്ദർഭത്തിൽ പ്രവാചകാനുചരന്മാർ തങ്ങളുടെ മഴു കൊണ്ട്, സംഘം ചേർന്ന് തകർക്കാൻ ശ്രമിച്ചിട്ടും തകരാതിരുന്ന ഒരു ഭീമൻ പാറ പ്രവാചകൻ (സ) തന്റെ മഴു കൊണ്ട് വെട്ടി തകർക്കുകയുണ്ടായി. (സുനനുൽ കുബ്റാ: നസാഈ: 8858, മുസ്നദു റൂയാനി: 410) പ്രവാചകന്(സ) അസാധാരണ കായിക ശക്തി ഉണ്ടായിരുന്നു എന്നത് ശരിയാണെന്ന് ചുരുക്കം. അദ്ദേഹം ഇടക്ക്, ഒറ്റ ദിവസം അദ്ദേഹത്തിന്റെ എല്ലാ ഭാര്യമാരേയും സന്ദർശിക്കാറുണ്ടായിരുന്നു എന്നതും വാസ്തവം. പക്ഷെ ഈ രണ്ടു കാര്യങ്ങൾക്കു തമ്മിലുള്ള ബന്ധിപ്പിക്കുന്ന ഘടകം ലൈംഗിക വേഴ്ച്ചയാകണമെന്ന് യാതൊരു നിർബന്ധവുമില്ല. കാരണം, ഒമ്പതു ഭാര്യമാരേയും ഒരു ദിവസം തന്നെ സന്ദർശിക്കുകയാണെങ്കിൽ അവരുമായി ഹൃദ്യമായി സല്ലപിക്കുകയും, അവരുടെ സുഖ ദുഖങ്ങളിൽ പങ്കാളിയാവുകയും, എന്നിട്ട് വഴക്കോ- അസ്വാരസ്യങ്ങളോ ഉണ്ടാകാത്ത നിലയിൽ- അവരുടെയെല്ലാം ഭർതൃ സങ്കൽപങ്ങളെ തൃപ്തിപ്പെടുത്തി വിടപറയുകയും എല്ലാം വേണമല്ലൊ. ലൈംഗിക ബന്ധമില്ലാതെ തന്നെ ഇത് അങ്ങേയറ്റം ശ്രമകരമായ യത്നം തന്നെയാണ്. ഒരു സാധാരണ മനുഷ്യന് കഴിയാൻ സാധ്യതയില്ലാത്ത അനൽപമായ ക്ഷമയും, സഹാനുഭൂതിയും, നീതി ബോധവും, ത്യാഗവും, സ്നേഹവുമെല്ലാം അനിവാര്യമായ ഒരു മഹാ ഉദ്യമമാണത്. മതം, രാഷ്ട്രീയം, സാമൂഹിക സേവനം, യുദ്ധം, ആരാധന, പ്രബോധനം തുടങ്ങി കൃത്യാന്തര ബാഹുല്യത്തിനിടയിലും ഒമ്പതു ഭാര്യമാരും മക്കളും മരുമക്കളം, പേരക്കുട്ടികളുമെല്ലാമടങ്ങുന്ന കുടുംബ-ദാമ്പത്യ ജീവിതത്തിലും പ്രവാചകനെ പോലെ മാതൃകയും തുല്യനീതിയും ആദർശനിഷ്ടയും പുലർത്താൻ അശക്തരായ അനുചരന്മാർ പ്രവാചകന്റെ ഈ ബഹുമുഖ മാതൃകയെ ആശ്ചര്യത്തോടെ ആവിഷ്‌കരിച്ചതാണ് “അദ്ദേഹത്തിന് മുപ്പത് ആളുകളുടെ കരുത്ത് നൽകപ്പെട്ടിട്ടുണ്ട്…” എന്ന വാചകം. ഒരാൾക്ക്, ഒരു ആയുഷ്ക്കാലം കൊണ്ട് ചെയ്തു തീർക്കാൻ കഴിയുന്നതിനുമപ്പുറമായിരുന്നു അദ്ദേഹത്തിലർപ്പിതമായതും അദ്ദേഹം പൂർണതയോടെ നിറവേറ്റിയതുമായ -കുടുംബ ജീവിതമുൾപ്പെടെയുള്ള- ദൗത്യങ്ങൾ എന്നേ അനസിന്റെ ആ വാചകം കൊണ്ട് മനസ്സിലാക്കേണ്ടതുള്ളു. അല്ലാതെ മുപ്പത് പുരുഷന്മാരുടെ ലൈംഗിക ശക്തി അദ്ദേഹത്തിനുണ്ടായിരുന്ന എന്ന് ആ വാചകത്തെ ദുർവ്യാഖ്യാനിക്കുന്നത് ന്യായമില്ല. പ്രവാചകന്റെ(സ) ഈ ‘കൂട്ട സന്ദർശനത്തിന്റെ’ അർത്ഥവും ലക്ഷ്യവും സ്വാഭാവികമായ സഹവാസവും സമ്പർക്കവും മാത്രമായിരുന്നെന്ന് അദ്ദേഹത്തിന്റെ സന്ദർശനത്താൽ അനുഗ്രഹിക്കപ്പെട്ട ഭാര്യമാർ വ്യക്തമാക്കിയിരിക്കെ അനസിന്റെ(റ) വാചകം ദുർവ്യാഖ്യാനിച്ചുള്ള അശ്ലീല ആരോപണങ്ങളുടെ പ്രസക്തിയെന്താണ്?! “അദ്ദേഹത്തിന് മുപ്പത് ആളുകളുടെ കരുത്ത് നൽകപ്പെട്ടിട്ടുണ്ട്…” എന്ന വാചകത്തിന്റെ ഉദ്ദേശ്യം ലൈംഗിക ബന്ധത്തിനുള്ള കരുത്താണ് എന്ന വ്യാഖ്യാനം വാദത്തിന് അംഗീകരിച്ചാൽ തന്നെ അത് ആ സന്ദർശനത്തെ സംബന്ധിച്ച പ്രവാചക ശിഷ്യൻ അനസിന്റെ(റ) ഒരു തെറ്റിദ്ധാരണയാണെന്നേ സ്ഥാപിക്കപ്പെടുന്നുള്ളു. കാരണം, പ്രവാചകന്റെ(സ) ഈ ‘കൂട്ട സന്ദർശനത്തിന്റെ’ അർത്ഥവും ലക്ഷ്യവും, സ്വാഭാവികമായ സഹവാസവും സമ്പർക്കവും മാത്രമായിരുന്നെന്ന് അദ്ദേഹത്തിന്റെ സന്ദർശനത്താൽ അനുഗ്രഹിക്കപ്പെട്ട ഭാര്യമാർ വ്യക്തമാക്കിയിരിക്കെ അനസിന്റെ(റ) ധാരണ യഥാർത്ഥ്യമാകാൻ വല്ല നിർവ്വാഹവുമുണ്ടോ?! അനസാണോ(റ) ആ സന്ദർശനത്തെ സംബന്ധിച്ച് സൂക്ഷ്മമായി അറിയുക ? അതോ പ്രവാചകപത്നിമാരോ ?!കൂടാതെ, അനസിന്റെ(റ) ആ വാചകത്തിൽ നിന്നും എല്ലാ ഭാര്യമാരുമായി അദ്ദേഹം ലൈംഗികവേഴ്ച്ചയിൽ ഏർപ്പെട്ടു എന്നും സ്ഥാപിതമാകുന്നില്ല. കൂടി വന്നാൽ ഒന്നിലേറെ പേരുമായി ബന്ധപ്പെട്ടു എന്നേ ഈ വ്യാഖ്യാനം ശരിവച്ചാലും സ്ഥാപിതമാകുന്നുള്ളു. “ആർക്കെങ്കിലും രണ്ട് ഭാര്യമാർ ഉണ്ടായിട്ട് അവരിൽ ഒരാളിലേക്ക് (പക്ഷപാതപരമായി) അയാൾ താൽപര്യം കാണിച്ചാൽ അന്ത്യനാളിൽ തന്റെ ശരീരത്തിന്റെ ഒരു ഭാഗം ചെരിഞ്ഞതായി/ സ്വാധീനമറ്റതായിട്ടാണ് അയാൾ വരിക” (മുസ്നദു അഹ്മദ്: 2:295, സുനനു അബൂദാവൂദ്: 2:601) എന്ന് ഉണർത്തി, ഭാര്യമാർക്കിടയിൽ നീതിയോടെ വർത്തിക്കണമെന്ന് സ്വന്തം അനുചരൻമാരെ അനുശാസിക്കുക മാത്രമല്ല പ്രവാചകൻ (സ) ചെയ്തത്. ഉപദേശത്തിന് പുറമെ, മരണം വരെയുള്ള സ്വന്തം ദാമ്പത്യജീവിതത്തിലൂടെ അതിന് മാതൃക ലോകത്തിന് സമ്മാനിക്കുക കൂടി ചെയ്തു അദ്ദേഹം. വിവാദ വിധേയമായ ഹദീസ് പ്രവാചകന്റെ നീതിയേയും സമത്വത്തെയുമാണ് ജ്വലിപ്പിക്കുന്നത്.

വിമർശനം:

വഹ്‌യ് (ദിവ്യബോധനം) ദീർഘകാലം തടയപ്പെടുമ്പോഴെല്ലാം വിഷാദം ബാധിച്ച് മുഹമ്മദ് നബി മലമുകളിൽ നിന്ന് ചാടി ആത്മഹത്യ ചെയ്യാൻ ശ്രമിക്കുക പതിവായിരുന്നു. അല്ലാഹുവിന്റെ മലക്കായ ജിബ്‌രീൽ അതിൽ നിന്നും അദ്ദേഹത്തെ തടഞ്ഞു കൊണ്ടുമിരുന്നു. ഇത് ‘സ്വഹീഹുൽ ബുഖാരി’യിൽ രേഖപ്പെടുത്തപ്പെട്ടിരിക്കുന്നു.

മറുപടി:

1. പ്രവാചകനിലേക്ക്(സ) ചേർക്കപ്പെടുന്ന ഈ “ആത്മഹത്യകഥ”ക്ക് യാതൊരു വിധ പ്രാമാണിക സ്രോതസ്സും അടിത്തറയും ഇല്ല. ദുർബലമായ ചില ചരിത്ര നിവേദങ്ങളിൽ നിന്നും ഒരു നിവേദകന്റെ കേട്ടറിവ് മാത്രമാണ് ഈ അവാസ്തവികമായ “കഥ”യുടെ ആധാരം. ഇസ്‌ലാമിക പ്രമാണങ്ങൾ ക്വുർആനും സ്വഹീഹായ ഹദീസുകളുമാണ്. പ്രവാചക ചരിത്രത്തിന്റെ സ്രോതസ്സും ഇവ രണ്ടും തന്നെ. അല്ലാതെ ഏതെങ്കിലും ഹദീസ് നിവേദകന്റെ – തെളിവുകളുടെ പിൻബലമില്ലാത്ത- “കേട്ടറിവോ” ദുർബല നിവേദനങ്ങളൊ അല്ല.

2. ദിവ്യബോധനത്തിന്റെ ആരംഭം എങ്ങനെയായിരുന്നു എന്ന് വിവരിക്കുന്ന ഒരു ഹദീസുമായി ബന്ധപ്പെട്ടാണ് ഈ കഥ കടന്നുവരുന്നത്. ദിവ്യബോധനത്തിന്റെ ആരംഭത്തെ സംബന്ധിച്ച ആഇശയുടെ(റ) ഹദീസ് സ്വഹീഹുൽ ബുഖാരിയിൽ മൂന്ന് സ്ഥലങ്ങളിൽ (ഹദീസ് നമ്പറുകൾ: 3, 4953, 6581) ആവർത്തിക്കപ്പെടുന്നുണ്ട്. ദീർഘമായ ഈ ഹദീസിന്റെ ചുരുക്കം ഇപ്രകാരമാണ്:

പ്രവാചകന് വഹ്‌യുകളുടെ (ദിവ്യബോധനത്തിന്റെ) ആരംഭം സ്വപ്നങ്ങളിലൂടെയായിരുന്നു. അദ്ദേഹം കാണുന്ന സ്വപ്നങ്ങളൊക്കെ പ്രഭാതോദയം പോലെ പുലരുമായിരുന്നു. പിന്നീട് ഹിറാ ഗുഹയില്‍ ആരാധനയില്‍ മുഴുകി കഴിയുമ്പോൾ അദ്ദേഹത്തിന്റെ അടുത്ത് മലക്ക് – ജിബ്‌രീല്‍ വന്ന് അല്ലാഹുവിൽ നിന്നുള്ള വഹ്‌യ് ആദ്യമായി നേരിട്ട് നൽകി. (ക്വുർആനിലെ 96 ആം അധ്യായത്തിലെ ഒന്നു മുതൽ അഞ്ച് വരെയുള്ള വാക്യങ്ങൾ പ്രവാചകന് ഓതി കേൾപ്പിച്ചു.) ഉടനെ പ്രവാചകൻ (സ) പേടിച്ചുവിറയ്ക്കുന്ന ഹൃദയത്തോടെ ഈ സൂക്തങ്ങളുമായി പത്നി ഖദീജയുടെ അടുക്കല്‍ ചെന്നു. തനിക്കെന്തെങ്കിലും കുഴപ്പം ബാധിച്ചോ എന്ന് ഭയപ്പെട്ട പ്രവാചകനെ പത്നി ആശ്വസിപ്പിച്ചു. ഖദീജ (റ) പറഞ്ഞു: “ഇല്ല അല്ലാഹുവാണെ, അവനൊരിക്കലും അങ്ങയെ അപമാനിക്കുകയില്ല. അങ്ങ് കുടുംബബന്ധം പുലര്‍ത്തുന്നു. നിരാലംബരുടെ ഭാരം ചുമക്കുന്നു. അഗതികള്‍ക്ക് (അവകാശങ്ങള്‍) നേടിക്കൊടുക്കുന്നു. അതിഥികളെ സൽക്കരിക്കുന്നു. ദുരിതബാധിതരെ സഹായിക്കുന്നു.” പിന്നീടവര്‍ പ്രവാചകനേയും കൂട്ടി തന്റെ പിതൃവ്യപുത്രന്‍ വറകതിബ്‌നു നൗഫലിനെ സമീപിച്ചു. വറക്വ ജാഹിലിയ്യാ കാലത്ത് ക്രിസ്ത്യാനിയായിരുന്നു. അദ്ദേഹത്തിന് ഹീബ്രു എഴുത്ത് അറിയാമായിരുന്നു. അന്ധനായ പടുവൃദ്ധനായി കഴിഞ്ഞിരുന്ന അദ്ദേഹത്തോട് ഖദീജ (റ) പറഞ്ഞു. പിതൃവ്യപുത്രാ താങ്കളുടെ സഹോദരപുത്രന് പറയാനുള്ളത് കേള്‍ക്കുക, വറക്വ ചോദിച്ചു നീ കണ്ടെതെന്താണ്? കണ്ട വിശേഷങ്ങളെല്ലാം പ്രവാചകൻ (സ) പറഞ്ഞുകേൾപ്പിച്ചു. വറക്വ പ്രതിവചിച്ചു: “മൂസാ(അ)യുടെ അടുത്തേക്ക് അല്ലാഹു അയച്ച രഹസ്യ സൂക്ഷിപ്പുകാരന്‍ (ജിബ്‌രീല്‍) ആണത്. (താങ്കള്‍ പ്രവാചകനാവുമ്പോള്‍) ഞാനൊരു യുവാവായിരുന്നെങ്കില്‍? താങ്കളുടെ ജനത താങ്കളെ ബഹിഷ്‌കരിക്കുന്ന ഘട്ടത്തില്‍ ഞാന്‍ ജീവിച്ചിരുന്നെങ്കിലെന്ന് ആശിക്കുന്നു.” ഉടനെ പ്രവാചകൻ (സ) ചോദിച്ചു: “അവരെന്നെ പുറത്താക്കുമെന്നോ?” വറക്വ പറഞ്ഞു: “അതെ, താങ്കള്‍ കൊണ്ടുവന്നതു പോലെയുള്ളത് കൊണ്ടുവന്നവരൊക്കെ എതിര്‍ക്കപ്പെട്ടിട്ടുണ്ട്. ആ നാളുകളില്‍ ഞാനുണ്ടെങ്കില്‍ സുശക്തമായ പിന്‍ബലം നല്‍കി സഹായിക്കും.” ഏറെ കഴിയുംമുമ്പേ വറക്വ മരണമടഞ്ഞു. കുറച്ച് കാലത്തേക്ക് വഹ്‌യും നിലച്ചു. പ്രവാചകനെ (സ) ഈ രണ്ടു സംഭവങ്ങളും ദുഖിപ്പിച്ചു.

സ്വഹീഹുൽ ബുഖാരിയിൽ മൂന്ന് സ്ഥലങ്ങളിൽ (ഹദീസ് നമ്പറുകൾ: 3, 4953, 6581) ആവർത്തിക്കപ്പെടുന്ന സ്വഹീഹായ ഹദീസിന്റെ ഉള്ളടക്കം ഇതാണ്. ഇതിൽ പ്രവാചകൻ (സ) പിന്നീട് വിഷാദത്തിൽ ആണ്ടു പോയെന്നോ ആത്മഹത്യ ചെയ്യാൻ ശ്രമിച്ചു എന്നോ ഒന്നുമില്ല.!!

3. എന്നാൽ 6581 ആം നമ്പർ ഹദീസിൽ നാം വിവരിച്ച സംഭവത്തിന് തുടർച്ചയായി ഹദീസ് ഉദ്ധരിക്കുന്ന റാവിമാരിൽ ഒരാളായ ഇബ്നു ശിഹാബ് അസ്സുഹ്‌രി ഈ സംഭവവുമായി ബന്ധപ്പെട്ട് തനിക്കു കിട്ടിയ ഒരു ചരിത്ര നിവേദനം സാന്ദർഭികമായി കൂട്ടി ചേർത്ത് പറയുന്നുണ്ട്. അതിന്റെ ചുരുക്കം ഇപ്രകാരമാണ്:

വറക്വയുടെ മരണവും കുറച്ചു കാലത്തേക്ക് വഹ്‌യ് നിലച്ചതും പ്രവാചകന് തീവ്രമായ വിഷാദമുണ്ടാക്കി. അദ്ദേഹം മലയിൽ നിന്ന് ചാടി മരിക്കാൻ തുനിഞ്ഞു. അപ്പോൾ അല്ലാഹുവിന്റെ മലക്കായ ജിബ്‌രീൽ അദ്ദേഹത്തെ തടയുകയും “താങ്കൾ അല്ലാഹുവിന്റെ സത്യദൂതൻ തന്നെ” എന്ന് ഉറപ്പ് നൽകുകയും ചെയ്തു.

ഈ കഥ, ദിവ്യബോധനത്തിന്റെ ആരംഭത്തെ സംബന്ധിച്ച, ആഇശയുടെ (റ) സ്വഹീഹായ ഹദീസിന്റെ ഭാഗമല്ല. ആയിരുന്നെങ്കിൽ ആഇശയുടെ (റ) ഹദീസ് ആവർത്തിക്കപ്പെടുന്നിടത്തെല്ലാം (ഹദീസ് നമ്പറുകൾ: 3, 4953, 6581) ഈ “ആത്മഹത്യ കഥ”യും ആവർത്തിക്കപ്പെടുമായിരുന്നു. സ്വഹീഹുൽ ബുഖാരിയിലെ ഹദീസ് നമ്പർ: 3, ഹദീസ് നമ്പർ: 4953 എന്നിവിടങ്ങളിൽ ഈ “ആത്മഹത്യ കഥ”യുടെ വാൽകഷ്‌ണം നമുക്ക് കാണാൻ കഴിയില്ല. 6581 ആം നമ്പറായി ഉദ്ധരിക്കപ്പെട്ടിടത്ത് മാത്രമാണ്, ഹദീസിന് അനുബന്ധമെന്നോണം “ആത്മഹത്യ കഥ” ചേർക്കപ്പെട്ടിരിക്കുന്നത്.

“നമ്മുടെ അടുക്കൽ എത്തിയ ഒരു വിവരമനുസരിച്ച്” (فيما بلغنا) എന്ന് സൂചിപ്പിച്ച് കൊണ്ട് ഇബ്നു ശിഹാബ് അസ്സുഹ്‌രിയാണ് ആത്മഹത്യാ ശ്രമത്തെ സംബന്ധിച്ച ഈ കഥ ഒരു അനുബന്ധമായി കൂട്ടി ചേർത്തിട്ടുള്ളത്. സ്വഹീഹുൽ ബുഖാരിയിൽ 6581 ആം നമ്പർ ഹദീസിന്റെ അവസാന ഭാഗത്ത് ഇത് നമുക്ക് പ്രത്യേകം കാണാൻ സാധിക്കുന്നതാണ്.

അതല്ലാതെ സ്വഹീഹുൽ ബുഖാരിയിൽ മൂന്ന് സ്ഥലങ്ങളിൽ ആവർത്തിക്കപ്പെട്ടിട്ടുള്ള -ദിവ്യബോധനത്തിന്റെ ആരംഭത്തെ സംബന്ധിച്ച- ആഇശയുടെ (റ) ഹദീസിന്റെ ഭാഗമല്ല അത്. ഹദീസുകൾ ഉദ്ധരിക്കുമ്പോഴും കൈമാറുമ്പോഴും ഹദീസുകളുടെ അനുബന്ധമായി അൽപം ചില റാവിമാർ (നിവേദകന്മാർ) ചില അധിക വിവരങ്ങൾ അനുബന്ധമായി ചേർത്തെന്നു വരാം. ഇത്തരം അധികരിച്ച അനുബന്ധങ്ങളെ ഹദീസ് നിദാനശാസ്ത്രത്തിന്റെ സാങ്കേതിക ഭാഷയിൽ ‘മുദ്റജ്’ (المدرج) എന്നാണ് വിളിക്കപ്പെടുക. നിവേദകന്റെ ഈ അധികരിച്ച അനുബന്ധം അഥവാ ‘മുദ്റജ്’ ആ നിവേദനത്തിന്റെ ഭാഗമല്ല. അത് നിവേദകന്റെ വ്യക്തിപരമായ അഭിപ്രായം മാത്രമായാണ് പരിഗണിക്കപ്പെടുക. അത്തരം അഭിപ്രായങ്ങൾ ഹദീസുമല്ല, സ്വഹീഹുമല്ല, ഇസ്‌ലാമിൽ പ്രമാണവുമല്ല.

അതുകൊണ്ട് തന്നെ, “നമ്മുടെ അടുക്കൽ എത്തിയ ഒരു വിവരമനുസരിച്ച്” ( فيما بلغنا ) എന്ന് സൂചിപ്പിച്ചുകൊണ്ട് ഇബ്നു ശിഹാബ് അസ്സുഹ്‌രി അനുബന്ധമായി കൂട്ടി ചേർത്ത, ഈ “ആത്മഹത്യാ ശ്രമത്തിന്റെ കഥ”, ദിവ്യബോധനത്തിന്റെ ആരംഭത്തെ സംബന്ധിച്ച, ആഇശയുടെ (റ) ഹദീസിന്റെ ഭാഗമല്ല; സ്വഹീഹായ (സ്ഥിരപ്പെട്ടതും വിശ്വസനീയവുമായ) സംഭവവുമല്ല.

ശൈഖ് സ്വാലിഹ് അൽ മുനജ്ജിദ് പറഞ്ഞു: ആത്മഹത്യയുമായി ബന്ധപ്പെട്ട അധികഭാഗം ആഇശയുടെ (റ) സംസാരത്തിലോ ഹദീസിലോ പെട്ടതല്ല. മറിച്ച്, അത് (നിവേദകന്മാരിൽ ഒരാളായ) സുഹ്‌രിയുടെ വാചകമാണ്. അദ്ദേഹം ‘താബിഈ’കളിൽ(പ്രവാചകാനുചരന്മാരുടെ ശിഷ്യന്മാരായ പിൻതലമുറക്കാർ)പെട്ട വ്യക്തിയാണ്. അതുകൊണ്ട് തന്നെ ഈ സംഭവത്തിന് അദ്ദേഹം സാക്ഷിയല്ല. പ്രവാചക ശിഷ്യന്മാരിൽ ആരിൽ നിന്നെങ്കിലുമാണ് ഈ കഥ ഉദ്ധരിക്കുന്നത് എന്ന് സുഹ്‌രി പറഞ്ഞിട്ടുമില്ല. ഇങ്ങനെ, കൃത്യമായ അടിത്തറകളിൽ ഊന്നി കൊണ്ടല്ല ഈ കഥ ഉദ്ധരിക്കുന്നത് എന്നതിനാലാണ് “നമ്മുടെ അടുക്കൽ എത്തിയതനുസരിച്ച്” (فيما بلغنا) എന്ന് സൂചിപ്പിച്ചുകൊണ്ട് ഇബ്നു ശിഹാബ് അസ്സുഹ്‌രി, ആത്മഹത്യയുമായി ബന്ധപ്പെട്ട അധികഭാഗം ഉദ്ധരിച്ചത്.” (ഇസ്‌ലാം സുആൽ വൽജവാബ്: 152611)

ثم إن القائل ” فيما بَلَغَنا ” هو الزهري ، ومعنى الكلام : أن في جملة ما وصل إلينا من خبر رسول الله صلى الله عليه وسلم في هذه القصة . وهو من بلاغات الزهري وليس موصولاً ، وقال الكرماني : هذا هو الظاهر .

ഇബ്നു ഹജർ അൽ അസ്കലാനി പറഞ്ഞു: ” “നമ്മുടെ അടുക്കൽ എത്തിയ ഒരു വിവരമനുസരിച്ച്” (فيما بلغنا) എന്ന് പറഞ്ഞിരിക്കുന്നത് സുഹ്‌രിയാണ്. ആ പറഞ്ഞതിന്റെ അർത്ഥം: ഈ കഥയുമായി ബന്ധപ്പെട്ട് നമ്മുടെ അടുക്കൽ എത്തിപ്പെട്ട മൊത്തം വിവരങ്ങളിൽ പെട്ടതാണ് (പ്രവാചകൻ (സ) ആത്മഹത്യ ചെയ്യാൻ തുനിഞ്ഞു എന്ന കഥ.) ഈ കഥയാകട്ടെ സുഹ്‌രിയുടെ കേവല കേട്ടറിവുകളിൽ പെട്ടത് മാത്രമാണ്. പ്രവാചകനിലേക്കെത്തുന്ന കൃത്യമായ ഒരു നിവേദക പരമ്പരയും അദ്ദേഹത്തിനടുക്കൽ ഇല്ല. കർമാനി പറഞ്ഞു: ഇതാണ് പ്രത്യക്ഷ സത്യം.” (ഫത്ഹുൽ ബാരി: 12:359)

നിവേദകപരമ്പര മുറിഞ്ഞ അല്ലെങ്കിൽ നിവേദകപരമ്പര പറയാത്ത സുഹ്‌രിയുടെ ഇത്തരം അധികരിച്ച അനുബന്ധാഭിപ്രായം മുർസൽ (مرسل) എന്ന ദുർബല നിവേദനങ്ങളിലാണ് ഉൾപ്പെടുക എന്നും ഒരു കാറ്റു പോലെ അടിത്തറയില്ലാത്തവയാണ് സുഹ്‌രിയുടെ ഇത്തരം മുർസലുകൾ എന്നും ഇമാം യഹ്‌യബ്നു സഈദ് അൽകത്വാൻ വ്യക്തമാക്കുന്നുണ്ട്. (ജാമിഉത്തഹ്സ്വീൽ: സ്വലാഹുദ്ദീൻ അലാഈ: 78, തദ്കിറത്തുൽ ഹുഫ്ഫാദ്: 1:1-5)

هذا من كلام الزهري أو غيره ، غير عائشة ، والله أعلم ؛ لقوله : ” فيما بلغنا ” ، ولم تقل عائشة في شيء من هذا الحديث ذلك . അബൂ ശാമ അൽ മക്ദസി പറഞ്ഞു: ” “നമ്മുടെ അടുക്കൽ എത്തിയ ഒരു വിവരമനുസരിച്ച്” (فيما بلغنا) എന്ന വാചകം സുഹ്‌രിയുടേതോ മറ്റൊരു റാവിയുടെതോ ആകുന്നു; അത് ആഇശയുടെ (റ) ഹദീസിൽ പെട്ടതല്ല. ഹദീസിലെവിടെയും “നമ്മുടെ അടുക്കൽ എത്തിയ ഒരു വിവരമനുസരിച്ച്” എന്ന് ആഇശ (റ) പറയുന്നില്ല…” (ശർഹുൽ ഹദീസിൽ മുക്തഫാ : 177)

4. സ്വഹീഹുൽ ബുഖാരി എന്ന ഗ്രന്ഥത്തിന്റെ യഥാർത്ഥ നാമം “അല്ലാഹുവിന്റെ ദൂതന്റെ കാര്യവിവരങ്ങൾ- ചര്യകൾ- യുദ്ധങ്ങൾ എന്നിവയുടെ ഹ്രസ്വ വിവരണങ്ങളായ ‘സ്വഹീഹും’ ‘മുസ്നദു’മായ ഹദീസുകളുടെ സമാഹാരം” (الجامع المسند الصحيح المختصر من أُمور رسول الله صلى الله عليه وسلّم وسننه وأيامه) എന്നാണ്.

ഗ്രന്ഥത്തിന്റെ നാമം സൂചിപ്പിക്കുന്നതു പോലെ പ്രവാചക ഹദീസുകളിലെ ‘സ്വഹീഹും’ ‘മുസ്നദു’മായവ മാത്രമാണ് ഇസ്‌ലാമിലെ പ്രമാണം. മുസ്നദ് (المسند) എന്നാൽ പ്രവാചകനിലേക്ക് എത്തുന്ന സനദ് (നിവേദക പരമ്പര) ഉള്ളത് എന്നാണ് അർത്ഥം. സ്വഹീഹ് (الصحيح) എന്നാൽ ഈ നിവേദക പരമ്പര ഹദീസ് നിദാന ശാസ്ത്രത്തിന്റെ നിബന്ധനകൾ പൂർത്തീകരിക്കപ്പെട്ട സ്വീകാര്യ യോഗ്യമായ ഹദീസുകളുമാണ്.

സ്വഹീഹുൽ ബുഖാരിയിൽ ‘മുസ്നദായി’ ഉദ്ധരിക്കപ്പെട്ട ‘ഹദീസുകൾ’ എല്ലാം സ്വഹീഹാണെന്നും പ്രമാണമാണെന്നും മാത്രമാണ് മുസ്‌ലിംകൾ വിശ്വസിക്കുന്നത്. എന്നാൽ സ്വഹീഹുൽ ബുഖാരിയിൽ ഇത്തരം മുസ്നദായ ഹദീസുകൾക്ക് പുറമെ ധാരാളം ഭാഷാ ചർച്ചകളും, കർമ്മശാസ്ത്ര ചർച്ചകളും, മുഅല്ലക്കായ (പരമ്പര കളഞ്ഞ) നിവേദനങ്ങളും, പണ്ഡിതാഭിപ്രായങ്ങളും തുടങ്ങി അറബി കവിതകൾ വരെ ഉൾപ്പെട്ടിട്ടുണ്ട്… അവയെല്ലാം സ്വഹീഹ് ആണെന്നോ ഇസ്‌ലാമിൽ പ്രമാണമാണെന്നോ മുസ്‌ലിംകളാരും മനസ്സിലാക്കുന്നില്ല; ഇമാം ബുഖാരിയും അഭിപ്രായപ്പെട്ടിട്ടുമില്ല. ഇമാം ബുഖാരി തന്നെയും വാദിച്ചത് തന്റെ ഈ ഗ്രന്ഥത്തിലെ ‘മുസ്നദായ ഹദീസുകളെല്ലാം സ്വഹീഹാണെന്ന്’ മാത്രമാണ്. ആത്മഹത്യക്കായി പ്രവാചകൻ (സ) ചിന്തിച്ചു എന്ന കഥയാകട്ടെ മുസ്നദായ (സനദോടു കൂടിയ) ഒരു ഹദീസ് അല്ല. ആഇശയുടെ ഹദീസിന് അനുബന്ധമായി, സുഹ്‌രി ഉദ്ധരിച്ച സനദില്ലാത്ത ഒരു കേവലാഭിപ്രായമാണ്.

5. വറക്വയുടെ മരണവും ദീർഘ കാലത്തേക്ക് വഹ്‌യ് നിലച്ചതും പ്രവാചകനിലുണ്ടാക്കിയ (സ) തീവ്രമായ വിഷാദത്താൽ പ്രേരിതനായി അദ്ദേഹം (സ) മലമുകളിൽ നിന്ന് ചാടി മരിക്കുന്നതിനെ സംബന്ധിച്ച് ചിന്തിച്ചുവെന്ന് സൂചിപ്പിക്കുന്ന ഈ നിവേദനം സ്വഹീഹുൽ ബുഖാരിയിലല്ലാതെ മറ്റു ചില ചരിത്ര ഗ്രന്ഥങ്ങളിലും വന്നിട്ടുണ്ട് എങ്കിലും പ്രസ്തുത നിവേദനങ്ങളെല്ലാം സംശയലേശമന്യേ ദുർബലവും കെട്ടു കഥകളും തന്നെയാണ്.

a) ഇബ്‌നു മർദൂയയുടെ തഫ്‌സീറിൽ ഉദ്ധരിക്കപ്പെട്ട, കഥയുടെ നിവേദക പരമ്പരയിൽ മുഹമ്മദിബ്‌നു കസീർ എന്ന റാവിയുണ്ട്. ഇയാൾ ദുർബലനും ഓർമ്മക്കുറവുള്ള വ്യക്തിയുമാണ്. അതുകൊണ്ട് തന്നെ “നമ്മുടെ അടുക്കൽ എത്തിയ ഒരു വിവരമനുസരിച്ച്” (فيما بلغنا) എന്ന സുഹ്‌രിയുടെ വാചകം ഒഴിവാക്കി കൊണ്ടാണ് ഇയാൾ കഥ ഉദ്ധരിക്കുന്നതും. (സിൽസിലത്തു ദ്ദഈഫ: 10:453)

b) ഇബ്‌നു സഅ്ദിന്റെ ‘ത്വബക്കാത്തി’ൽ ഉദ്ധരിക്കപ്പെട്ട, കഥയുടെ നിവേദനം വ്യാജമാണ് (മൗദൂഅ്). നിവേദക പരമ്പര ഇപ്രകാരമാണ്.

أخبرنا محمد بن عمر قال حدثني إبراهيم بن محمد بن أبي موسى عن داود بن الحصين عن أبي غطفان بن طريف عن ابن عباس أن رسول الله صلى الله عليه وسلم…

നിവേദക പരമ്പരയിലെ മുഹമ്മദിബ്‌നു ഉമർ എന്ന റാവി, നിവേദനങ്ങൾ വ്യാജമായി നിർമ്മിച്ചുണ്ടാക്കുന്ന വ്യക്തിയാണെന്ന് ആരോപിക്കപ്പെടുന്നു. മറ്റൊരു റാവിയായ ഇബ്രാഹിം ഇബ്‌നു മുഹമ്മദിബ്‌നു അബീ മൂസാ, ‘മത്റൂക്’ (متروك) അഥവാ കളവു പറയുന്നതായി ആരോപിക്കപ്പെടുന്ന വ്യക്തിയാണ്. (സിൽസിലത്തു ദ്ദഈഫ: 10:451)

c) ത്വബ്‌രി ഉദ്ധരിച്ച നിവേദനത്തിന്റെ പരമ്പര:

حدثنا ابن حميد قال حدثنا سلمة عن محمد بن إسحاق قال حدثني وهب بن كيسان مولى آل الزبير قال سمعت عبد الله بن الزبير وهو يقول لعبيد بن عمير بن قتادة الليثي : حدِّثنا يا عبيد كيف كان بدء ما ابتدئ به رسول الله صلى الله عليه و سلم من النبوة حين جاء جبريل عليه السلام فقال عبيد…

ഈ നിവേദകപരമ്പരയിലെ പ്രധാനപ്പെട്ട മൂന്ന് ന്യൂനതകൾ ഇവയാണ്:

ഒന്ന്, നിവേദകപരമ്പരയിലെ റാവിയായ ഉബൈദ് ഇബ്‌നു ഉമർ പ്രവാചക ശിഷ്യനല്ലാത്തതിനാൽ സംഭവത്തിന് സാക്ഷിയല്ല. അപ്പോൾ പ്രവാചകനിലേക്കെത്തുന്ന പരമ്പരയില്ല. രണ്ട്, സലമ ഇബ്‌നുൽ ഫദ്ൽ അൽ അബ്റശ് എന്ന റാവി തീരെ ഓർമ്മ പിശകുള്ള വ്യക്തിയാണ്. മൂന്ന്, ഇബ്‌നു ഹുമൈദ് അങ്ങേയറ്റം ദുർബലനാണ്. അബൂ സർഅ അർറാസിയെ പോലെയുള്ള ഒരു സംഘം ഹദീസ് പണ്ഡിതർ അയാൾ കളവു പറയുന്ന വ്യക്തിയാണെന്ന് പോലും അഭിപ്രായപ്പെട്ടിരിക്കുന്നു. (സിൽസിലത്തു ദ്ദഈഫ: 10:455-457)

പ്രവാചകൻ (സ) മലയിൽ നിന്ന് ചാടി മരിക്കാൻ തുനിഞ്ഞപ്പോൾ അല്ലാഹുവിന്റെ മലക്കായ ജിബ്‌രീൽ അദ്ദേഹത്തെ തടയുകയും “താങ്കൾ അല്ലാഹുവിന്റെ സത്യദൂതൻ തന്നെ” എന്ന് ഉറപ്പ് നൽകുകയും ചെയ്തു എന്ന കഥയിലെ പ്രസ്ഥാവനയിൽ നിന്നു തന്നെ കഥ വാസ്തവ വിരുദ്ധമാണെന്ന് ഏതൊരാൾക്കും മനസ്സിലാക്കാവുന്നതേയുള്ളു. കാരണം, താൻ പ്രവാചകനാണെന്ന സത്യത്തിൽ ഒരു നിമിഷം പോലും മഹാനായ പ്രവാചകൻ മുഹമ്മദ് (സ) ശങ്കിച്ചതായി ക്വുർആനോ സ്വഹീഹായ ഹദീസുകളോ നമ്മുക്ക് അറിവു നൽകുന്നില്ല. താൻ ജീവിക്കുന്ന സമൂഹത്തിലെ അന്ധവിശ്വാസ അനാചാരങ്ങളോടുള്ള തന്റെ വിമുഖതയുടെ കാരണമറിയാതെ വിഷാദനായി ഹിറാ ഗുഹയിലേക്ക് ധ്യാനത്തിന് പോയ പ്രവാചകന് (സ) വഹ്‌യിലൂടെ ജീവിത ലക്ഷ്യവും പ്രവാചകത്വ ദൗത്യവും ഏൽപ്പിക്കപ്പെടുകയാണ് ഉണ്ടായത്. ഇത് ഒരാളുടെ വിഷാദത്തെ ആവേശത്തിലേക്കും നിരാശയെ ജീവിതോന്മുഖതയിലേക്കുമാണ് നയിക്കുക. “മനുഷ്യനായതു കൊണ്ട് തന്നെ വഹ്‌യ് ആദ്യം നൽകപ്പെട്ടപ്പോൾ ഉടലെടുത്ത ആകസ്‌മിക ഭയപ്പാടിൽ നിന്ന് പൂർണ്ണമായ വിമുക്തി കൈവരിക്കാൻ സാവകാശം നൽകി കൊണ്ടാണ് രണ്ടാം വഹ്‌യ് നൽകുന്നതിൽ കാലതാമസമുണ്ടായത്…” എന്ന് ഇബ്‌നു ത്വൂലൂൻ നിരീക്ഷിക്കുന്നുണ്ട്. (സുബുലുൽ ഹുദാ വർറശാദ്: 2:272)

അപ്പോൾ, വറക്വയുടെ മരണവും കുറച്ച് കാലത്തേക്ക് വഹ്‌യ് നിലച്ചതും പ്രവാചകനെ (സ) ദുഖിപ്പിച്ചുവെങ്കിലും, ആത്മഹത്യയിലേക്ക് നയിക്കാൻ മാത്രം ദുർബലമായിരുന്നില്ല ഹിറായിലെ ദിവ്യബോധനത്തിൽ നിന്ന് പുതുതായി അദ്ദേഹത്തിന് ലഭ്യമായ ലക്ഷ്യബോധവും വിശ്വാസ ദർശനവും. ഹൃദയാന്തരാളങ്ങളിലേക്ക് പെയ്തിറങ്ങിയ ആ പുതുമഴ, ഇതിനേക്കാളൊക്കെ എത്രയോ ഭയാനകരമായ പരീക്ഷണാനുഭവങ്ങളെ അതിജീവിക്കാനുള്ള കുളിർമ്മ അദ്ദേഹത്തിന്റെ ജീവിതത്തിന് സമ്മാനിച്ചിട്ടുണ്ട്. അസഭ്യങ്ങൾ, പരിഹാസങ്ങൾ, കല്ലേറുകൾ, കുറ്റപ്പെടുത്തലുകൾ, കൊലവിളികൾ, കൂക്കുവിളികൾ എന്നിവയിൽ തുടങ്ങി… ശത്രുക്കൾ അദ്ദേഹത്തേയും അനുചരന്മാരേയും ക്രൂരമായി പീഡിപ്പിച്ചു, പലരേയും കൊന്നു കളഞ്ഞു, പട്ടിണിക്കിട്ടു, ബഹിഷ്ക്കരിച്ചു, യുദ്ധങ്ങൾ അഴിച്ചു വിട്ടു… ഇങ്ങനെ ആയിരമായിരം പരീക്ഷണങ്ങളുടെ അലമാലകൾക്കുമുമ്പിൽ കണ്ണു ചിമ്മാത്ത, ആദർശ അചഞ്ചലതയുടെ ഗിരിവീര്യമായിരുന്നു മുഹമ്മദ് നബി (സ).

വിമർശനം:

അബ്ബാസിന്റെ മകളായ ഉമ്മു ഹബീബ് ഓടിക്കളിക്കുന്നതു കണ്ട് “അവൾ പ്രായപൂർത്തിയാകുന്ന കാലം വരെ ഞാൻ ജീവിച്ചിരുന്നാൽ ഞാൻ അവളെ വിവാഹം കഴിക്കും” എന്നു മുഹമ്മദ് നബി പറഞ്ഞു. ഇത് നബി കുട്ടികളോട് ആസക്തിയുള്ള വ്യക്തിയായിരുന്നു (pedophile) എന്ന് തെളിയിക്കുന്നു.

മറുപടി:

ഒരു വ്യാജ നിവേദനത്തെ അടിസ്ഥാനപ്പെടുത്തി പ്രചരിക്കപ്പെടുന്ന കള്ളകഥയാണ് ഇത്.

നിവേദനം വ്യാജമാണ് എന്ന വസ്തുതയുടെ തെളിവുകൾ ചർച്ച ചെയ്യുന്നതിന് മുമ്പ് ഈ കഥ, പ്രവാചകൻ ‘പീഡൊഫൈൽ’ (കുട്ടികളോട് ലൈംഗീകാസക്തിയുള്ള വ്യക്തി) ആയിരുന്നു എന്ന് തെളിയിക്കുന്നുണ്ടോ എന്നതാണ് ആദ്യം ചർച്ച ചെയ്യേണ്ട ചോദ്യം. ഇല്ല, എന്നു മാത്രമല്ല പ്രവാചകൻ (സ) പീഡൊഫൈൽ ആയിരുന്നില്ല എന്നാണ് ഈ കഥ തെളിയിക്കുന്നത്. കാരണം, ഒരു പീഡോഫൈലും ഒരു കുഞ്ഞ് വളർന്നു വലുതായിട്ട് അവളെ വിവാഹം ചെയ്യാനല്ലല്ലൊ ആഗ്രഹിക്കുക; അവളുടെ ചെറുപ്രായത്തിൽ തന്നെ അവളെ ലഭിക്കണമെന്നല്ലെ ആശിക്കുക ?! ഉമ്മു ഹബീബിനെ വിവാഹം കഴിക്കാൻ പ്രവാചകനെ പ്രേരിപ്പിച്ച ഘടകം അവളുടെ ചെറുപ്രായമായിരുന്നു എങ്കിൽ വലുതായിട്ട് വിവാഹം ചെയ്യാമെന്നായിരുന്നോ പ്രവാചകൻ (സ) ആഗ്രഹിക്കുക ?! വിമർശകർ സാധാരണയായി വാദിക്കുന്നതു പോലെ പ്രവാചകൻ (സ) ഒരു സ്വേച്ഛാധിപതിയും അവിഹിത ബന്ധക്കാരനും ഒന്നും ആയിരുന്നില്ല എന്ന് കൂടി ഈ കഥ തെളിയിക്കുന്നു. കാരണം ഒരു ഏകാധിപതി, അയാൾ പീഡൊഫൈൽ ആയിരുന്നെങ്കിൽ ആ കുട്ടി വലുതാവാൻ കാത്തിരിക്കുമായിരുന്നോ, ഉടനെ ആഗ്രഹം സഫലീകരിക്കുമായിരുന്നില്ലേ?! ആസക്തി ശമിപ്പിക്കാൻ എന്തിന് ‘വിവാഹം’ ചെയ്യാൻ ആഗ്രഹിക്കുന്നു ?! അവിവാഹിത ബന്ധത്തിൽ ഏർപ്പെടുകയല്ലെ ഒരു പീഡോഫൈൽ ചെയ്യുക?! യുക്തിക്കും ബുദ്ധിക്കും മനശ്ശാസ്ത്രത്തിനും എതിരായ ഇത്തരം പീഡോഫൈൽ ആരോപണം വിമർശകരുടെ ഉള്ളിലെ പീഡോഫീലിയയെ മാത്രമാണ് പ്രതിബിംബിപ്പിക്കുന്നത്.

വിവാഹങ്ങളിലൂടെ ബന്ധങ്ങൾ ഊട്ടി ഉറപ്പിക്കുക എന്നത് അക്കാലഘട്ടത്തിൽ വളരെ പ്രബലമായ ഒരു സാമൂഹിക ശൈലിയായിരുന്നു. തന്റെ പ്രിയ സുഹൃത്തുക്കളും സന്തതസഹചാരികളുമായിരുന്ന അബൂബക്കറിന്റെയും(റ) ഉമറിന്റെയും(റ) മക്കളായ ആഇശയേയും ഹഫ്സയേയും പ്രവാചകൻ (സ) വിവാഹം ചെയ്തത് ഇതേ സാമൂഹിക കാരണങ്ങളിലും സൗഹൃദ പ്രചോദനങ്ങളിലും പ്രേരിതമായി കൊണ്ടായിരുന്നു. അല്ലാതെ അവരുടെ പ്രായമോ സൗന്ദര്യമോ പരിഗണിച്ചു കൊണ്ടായിരുന്നില്ല. ആ കാലഘട്ടത്തിലെ സാമൂഹിക ശീലങ്ങൾ മനസ്സിലാക്കിക്കൊണ്ട് നബി ചരിത്രം പഠിച്ച നിഷ്പക്ഷരായ അമുസ്‌ലിം ചരിത്രകാരന്മാർ പോലും ഈ വസ്തുത വ്യക്തമാക്കിയിട്ടുണ്ട്.

സ്കോട്ടിഷ് ഒറിയന്റലിസ്റ്റും, ചരിത്ര പണ്ഡിതനും, എഡിംഗ്ബർഗ് സർവ്വകലാശാലയിലെ പ്രൊഫസറും, ആംഗ്ലിക്കൻ (കത്തോലിക്കൻ) പുരോഹിതനുമായ വില്യം മോണ്ട്ഗോമറി വാട്ട് (W. Montogomory Watt) പറഞ്ഞു:

“ഇവിടെ നാം ഓർക്കേണ്ട ഒരു വസ്തുത, തീർച്ചയായും ഏഴാം നൂറ്റാണ്ടിലെ അറേബ്യയിലെ പെൺകുട്ടികൾ വളരെ നേരത്തെ പ്രായപൂർത്തി എത്തിയിരുന്നു എന്നതാണ്… ഒരു ബഹുഭാര്യത്വ കുടുംബത്തിൽ ഒരു പെൺകുട്ടിക്ക് ഉണ്ടാകാവുന്നതിൽ വെച്ച് ഏറ്റവും സന്തുഷ്ടയായിരുന്നു ആഇശ എന്ന് നാം കാണുന്നു. അവളുടെ വിവാഹം വ്യക്തമായും അബുബക്കറിനെയും മുഹമ്മദിനെയും തമ്മിൽ ബന്ധിപ്പിക്കുന്നതിനുള്ള രാഷ്ട്രീയ കാരണത്താലായിരുന്നു, അബുബക്കർ മുഹമ്മദിന്റെ ചീഫ് ലെഫ്റ്റനന്റ് ആയിരുന്നതുപോലെ, ആഇശ അദ്ദേഹത്തിന്റെ ഭാര്യമാരിൽ മുഖ്യയായിരുന്നു…” (W. Montgomary Watt, Muhammad: Prophet and Statesman, p.102, ഉദ്ധരണം: ibnehashim.blogspot.com)

മത താരതമ്യ ഗവേണപുസ്തകങ്ങൾക്ക് പേരുകേട്ട ബ്രിട്ടീഷ് എഴുത്തുകാരിയും ഐറിഷ് കത്തോലിക്കാ ബൈബിൾ വ്യാഖ്യാതാവുമായിരുന്ന കാരെൻ ആംസ്ട്രോംഗ് എഴുതി:

“ഭൗമിക ആനന്ദങ്ങളുടെ ഒരു പൂന്തോട്ടത്തിൽ അത്യന്തം സുഖലോലുപനായി മുഹമ്മദ്‌ വെയിൽ കായുകയായിരുന്നു എന്ന് സങ്കൽപ്പിക്കുന്നത് തികച്ചും അനർത്ഥമാണ്… നാം രണ്ട് കാര്യങ്ങൾ ശ്രദ്ധിക്കണം. ഒന്ന്, സൗദയെയും ആഇശയെയും അവരുടെ ലൈംഗിക ആകർഷണീയതയാലല്ല തിരഞ്ഞെടുക്കപ്പെട്ടത്. ആഇശ ഒരു കൊച്ചു പെൺകുട്ടി മാത്രമായിരുന്നു, മുപ്പതാം വയസ്സിൽ, സൗദ തന്റെ ആദ്യ യൗവനത്തെ മറികടന്ന് തടികൂടാൻ തുടങ്ങിയിരുന്നു. (രണ്ടു പേരിലും ലൈംഗീക ആകർഷണീയതയൊന്നുമുണ്ടായിരുന്നില്ല) രണ്ട്, രണ്ട് വിവാഹങ്ങൾക്കും ഒരു രാഷ്ട്രീയ മാനമുണ്ടായിരുന്നു: മുഹമ്മദ് പ്രധാനപ്പെട്ട പല ബന്ധങ്ങളും സൃഷ്ടിക്കുകയായിരുന്നു ആ വിവാഹങ്ങളിലൂടെ.” (Karen Armstrong, Muhammad: A Biography, p.145)

പ്രവാചകന്റെ പിതൃവ്യനായിരുന്നു അബ്ബാസ്. അദ്ദേഹത്തിന്റെ മകൾ ഉമ്മു ഹബീബ് ഏതോ ഒരു പെൺകുട്ടിയായിരുന്നില്ല… പ്രവാചകന്റെ ‘മുറപ്പെണ്ണി’ന്റെ സ്ഥാനമായിരുന്നു അവൾക്കുണ്ടായിരുന്നത്. “വഴിയിൽ ഓടി കളിക്കുന്ന ഏതോ പെൺകുട്ടിയെ പ്രവാചകൻ (സ) വിവാഹം ചെയ്യാൻ ആഗ്രഹിച്ചു” എന്ന രൂപേണ കഥയെ അവതരിപ്പിക്കുന്ന വിമർശകരുടെ ശൈലി ദുർവ്യാഖ്യാനമാണെന്നർത്ഥം.

“അബ്ബാസിന് നൽകുന്ന സ്ഥാനമോ ആദരവോ മറ്റൊരാൾക്കും പ്രവാചകൻ (സ) നൽകുന്നതായി ഞാൻ കണ്ടിട്ടില്ല..” എന്ന് ആഇശ (റ) പറയുകയുണ്ടായി. (അൽ ഫവാഇദ്: അബൂബക്കർ അൽബസ്സാസ്: 266 )

പ്രവാചകൻ (സ) അങ്ങേയറ്റം ആദരിക്കുകയും സ്നേഹിക്കുകയും ചെയ്തിരുന്ന പിതൃവ്യൻ അബ്ബാസിന്റെ മകളെ, അവൾ വലുതായാൽ -വിവാഹങ്ങളിലൂടെ ബന്ധങ്ങൾ ഊട്ടി ഉറപ്പിക്കുക എന്ന അക്കാലഘട്ടത്തിലെ സാമൂഹിക സമ്പ്രദായമനുസരിച്ച് – വിവാഹം ചെയ്യണമെന്ന് പ്രവാചകൻ (സ) ആഗ്രഹിച്ചു. ഇതിൽ ആസക്തിയും പീഡോഫീലിയയുമൊക്കെ കുത്തി കയറ്റപ്പെടുന്നത് വിമർശകരിലൂടെ മാത്രണ്. വിമർശകരുടെ രോഗാതുര മനസ്സുകളിലെ ലൈംഗീക ഭാവനകളിൽ നിന്നാണ് ഇത്തരം അശ്ലീല വ്യാഖ്യാനങ്ങൾ ഉടലെടുക്കുന്നത്. വസ്തുനിഷ്ഠമായ ഒരു തെളിവിന്റേയും അടിസ്ഥാനത്തിലല്ല.

ഇനി ആദ്യത്തെ പോയന്റിലേക്ക് മടങ്ങി വരാം. ഒരു വ്യാജ നിവേദനത്തെ ലൈംഗിക വ്യാഖ്യാനങ്ങളുടെ മസാല തേച്ച് അവതരിപ്പിക്കപ്പെട്ടതാണ് ഈ കഥ എന്ന് പറഞ്ഞുവല്ലൊ. നിവേദനം വ്യാജമാണെന്നതിന് ഒരുപാട് തെളിവുകളുണ്ട്. അവയിൽ പ്രധാനപ്പെട്ടവ ഇവിടെ ഹ്രസ്വമായി സൂചിപ്പിക്കാം:

1. ഇമാം അഹ്‌മദാകട്ടെ മറ്റാരുമാകട്ടെ ഈ കഥ ഉദ്ധരിക്കുന്നത് ‘ഇബ്നു ഇസ്ഹാക്കി’ൽ നിന്നാണ്. കഥയുടെ നിവേദനങ്ങളുടെയെല്ലാം മൂല സ്രോതസ്സ് ഇബ്നു ഇസ്ഹാക്കിന്റെ നബിചരിത്ര കൃതിയായ (സീറ) ‘സീറത്തുന്നബവിയ്യ’യിൽ ഉദ്ധരിച്ച ഒരു വ്യാജ നിവേദനമാണ്.

ഇബ്നു ഇസ്ഹാകിൽ നിന്ന് ഈ നിവേദനം ഉദ്ധരിച്ച ഇമാം അഹ്‌മദ് തന്നെ ഇബ്നു ഇസ്ഹാകിനെ സംബന്ധിച്ച് അഭിപ്രായപ്പെട്ടത് ഇപ്രകാരമാണ്: “ഹദീസിന്റെ വിഷയത്തിൽ അദ്ദേഹം പ്രമാണമല്ല. വിശ്വസ്ഥരല്ലാത്ത പലരിൽ നിന്നും ഇബ്നു ഇസ്ഹാക് നിവേദനങ്ങൾ ഉദ്ധരിച്ചിരുന്നു…” (സിയറു അഅ്ലാമിന്നുബലാഅ്: 7:46)

ഇബ്നു ഇസ്ഹാക് ഒരു ‘സീറ’ക്കാരനാണ്; ഹദീസ് പണ്ഡിതനല്ല. ആണെന്ന് അദ്ദേഹം സ്വയം വാദിച്ചിട്ടുമില്ല. അദ്ദേഹത്തിന്റേയും മറ്റു ചരിത്രകാരന്മാരുടേയും ചരിത്ര ഗ്രന്ഥങ്ങളിൽ ഏത് സംഭവം ഉദ്ധരിക്കുമ്പോഴും അതിനോടൊപ്പം സനദും (നിവേദക പരമ്പര) ഉദ്ധരിച്ചിരിക്കും. ഇബ്നു ഇസ്ഹാകിന്റെ സീറ ആരംഭിക്കുന്നതു തന്നെ ഇപ്രകാരം പറഞ്ഞു കൊണ്ടാണ്: ﻛﻞ ﺷﻲء ﻣﻦ ﺣﺪﻳﺚ اﺑﻦ ﺇﺳﺤﻖ ﻣﺴﻨﺪ “ഇബ്നു ഇസ്ഹാക്കിന്റെ എല്ലാ നിവേദനങ്ങളും സനദ് (നിവേദക പരമ്പര) ഉള്ളതാണ്.”

ഈ സനദുകൾ (നിവേദക പരമ്പരകൾ) എന്തിനാണ് ഇബ്നു ഇസ്ഹാക്കടക്കമുള്ള ചരിത്രകാരന്മാർ ഓരോ നിവേദനത്തിനൊപ്പവും എഴു വെച്ചിരിക്കുന്നത് എന്ന് വിമർശകർ ചിന്തിച്ചിട്ടുണ്ടോ ? മൈലാഞ്ചി എഴുത്തു പോലെയോ കാലിഗ്രഫി പോലെയോ ചന്തം കൂട്ടാനല്ല ഇത്. എഴുതപ്പെട്ട സംഭവം സത്യമാണോ നുണയാണോ, സ്ഥാപിതമാണോ അതോ കേവല ആരോപണമാണോ എന്ന് വായനക്കാർക്ക് പരിശോധിക്കാനാണ് ഈ സനദുകൾ. “ഈ സനദുകളിലെ നിവേദകർ വിശ്വസ്ഥരല്ലെങ്കിൽ അവർ പറഞ്ഞ കഥയും വിശ്വസ്ഥമല്ലെന്നും അവ സ്വീകരിക്കരുതെന്നും” ചരിത്രകാരന്മാർ തന്നെ വ്യക്തമാക്കിയിട്ടുണ്ട്. (മുഖദ്ദിമി: താരീഖുത്വബ്‌രി: 5)

വിശ്വാസ്യതയിൽ ദുർബലരോ, വ്യാജന്മാരോ ആയിട്ടുള്ള നിവേദകർ ഉദ്ധരിച്ച നിവേദനങ്ങൾ വല്ലതും ഇത്തരം ഗ്രന്ഥങ്ങളിൽ ചേർക്കപ്പെട്ടിട്ടുണ്ടെങ്കിൽ അവ സ്ഥാപിത വസ്തുതകളല്ലെന്നും കേവല കള്ള കഥകളാണെന്നും അവ തള്ളിക്കളയണമെന്നും ഈ ചരിത്ര ഗ്രന്ഥകാരന്മാർ തന്നെ വായനക്കാരോട് അഭ്യർത്ഥിക്കുന്നുണ്ട് എന്നർത്ഥം.

ഒരു വിഷയത്തെയൊ ഒരു കാലഘട്ടത്തെയൊ ചർച്ച ചെയ്യുമ്പോൾ അവയുമായി ബന്ധപ്പെട്ട്, തങ്ങൾക്കു ലഭിച്ച സർവ്വ നിവേദനങ്ങളും ചരിത്രകാരന്മാർ അവരുടെ ഗ്രന്ഥങ്ങളിൽ ഉൾപ്പെടുത്തിയിട്ടുണ്ട്. ആ നിവേദനങ്ങളിൽ ഏതൊക്കെയാണ് സത്യസന്ധം ഏതൊക്കെയാണ് വ്യാജം എന്നത് വേർത്തിരിച്ച് മനസ്സിലാക്കൽ ആ ഗ്രന്ഥങ്ങൾ പഠനവിധേയമാക്കുന്നവരുടെ ബാധ്യതയായാണ് ആ ഗ്രന്ഥങ്ങളുടെ രചയിതാക്കൾ മനസ്സിലാക്കിയത്.

2. ഇനി, ചർച്ച ചെയ്യപ്പെടുന്ന നിവേദനത്തിന്റെ ഉള്ളടക്കവും (മത്‌ന്) അതിന്റെ നിവേദക പരമ്പരയും (സനദ്) കാണുക:

സനദ്: ﻧﺎ ﺃﺣﻤﺪ: ﻧﺎ ﻳﻮﻧﺲ ﻋﻦ اﺑﻦ ﺇﺳﺤﻖ ﻗﺎﻝ: ﺣﺪﺛﻨﻲ اﻟﺤﺴﻴﻦ ﺑﻦ ﻋﺒﺪ اﻟﻠﻪ ﺑﻦ ﻋﺒﻴﺪ اﻟﻠﻪ ﺑﻦ ﻋﺒﺎﺱ ﻋﻦ ﻋﻜﺮﻣﺔ ﻋﻦ اﺑﻦ ﻋﺒﺎﺱ ﻗﺎﻝ:

ഇബ്നു ഇസ്ഹാക് പറഞ്ഞു: എന്നോട് ഹസനിബ്നു അബ്ദുല്ലാഹിബ്നു ഉബൈദുല്ലാഹിബ്നു അബ്ബാസ് പറഞ്ഞു: അദ്ദേഹം ഇക്‌രിമയിൽ നിന്ന് ഉദ്ധരിക്കുന്നു: അദ്ദേഹം അബ്ബാസിൽ നിന്ന് ഉദ്ധരിക്കുന്നു: അദ്ദേഹം പറഞ്ഞു:…

മത്‌ന്: ﻧﻈﺮ ﺭﺳﻮﻝ اﻟﻠﻪ ﺻﻠﻰ اﻟﻠﻪ ﻋﻠﻴﻪ ﻭﺳﻠﻢ ﺇﻟﻰ ﺃﻡ ﺣﺒﻴﺐ اﺑﻨﺔ ﻋﺒﺎﺱ ﻭﻫﻲ ﺑﺪﺭ ﺑﻴﻦ ﻳﺪﻳﻪ ﻓﻘﺎﻝ ﺭﺳﻮﻝ اﻟﻠﻪ ﺻﻠﻰ اﻟﻠﻪ ﻋﻠﻴﻪ ﻭﺳﻠﻢ: ﻟﺌﻦ ﺑﻠﻐﺖ ﻫﺬﻩ ﻭﺃﻧﺎ ﺣﻲ ﻷﺗﺰﻭﺟﻨﻬﺎ…

അബ്ബാസിന്റെ മകൾ ഉമ്മു ഹബീബ് ചെറുതായിരിക്കെ അവളെ നോക്കി അല്ലാഹുവിന്റെ ദൂതൻ (സ) പറഞ്ഞു: ഇവൾക്ക് പ്രായപൂർത്തിയാവുമ്പോൾ ഞാൻ ജീവിച്ചിരിക്കുന്നുവെങ്കിൽ ഞാൻ ഇവളെ വിവാഹം ചെയ്യും… (സീറത്തു ഇബ്നു ഇസ്ഹാക്: 1: 268)

ഇബ്നു ഇസ്ഹാകിനോട് ഈ കഥ ഉദ്ധരിക്കുന്നത്, ഹസനിബ്നു അബ്ദുല്ലാഹിബ്നു ഉബൈദുല്ലാഹിബ്നു അബ്ബാസ് എന്ന റാവിയാണ് (നിവേദകൻ) എന്ന് സനദിൽ നിന്നും വ്യക്തം. ഈ റാവിയുടെ സത്യസന്തതയെയും വിശ്വാസ്യതയേയും സംബന്ധിച്ച് ഹദീസ് നിദാന ശാസ്ത്ര പണ്ഡിതന്മാരും ചരിത്രകാരന്മാരും വിശദീകരിക്കുന്നത് ഇപ്രകാരമാണ്:

ഇമാം നസാഈ പറഞ്ഞു: ഹസനിബ്നു അബ്ദുല്ലാഹിബ്നു ഉബൈദുല്ലാഹിബ്നു അബ്ബാസ് നുണയനാണെന്ന് ആരോപിതനാണ്. (അൽ കാമിൽ ഫി ദുഅഫാഉർ രിജാൽ: ഇബ്നു അദിയ്യ: 3:214)

ഇമാം ബുഖാരി പറയുന്നു: ഹസനിബ്നു അബ്ദുല്ലാഹിബ്നു ഉബൈദുല്ലാഹിബ്നു അബ്ബാസ് വ്യാജനാണ്. (അദ്ദുഅഫാഉ വൽമത്റൂകൂൻ: 145)

ഹസനിബ്നു അബ്ദുല്ലാഹിബ്നു ഉബൈദുല്ലാഹിബ്നു അബ്ബാസ് വ്യാജനാണ് എന്ന് ഇബ്നു ഹജറും (തഹ്ദീബുൽ കമാൽ: 6:385, ഉകൈലിയും (അദ്ദുഅഫാ: 111) സ്വാലിഹ് ഇബ്നു അലി നൗഫലിയും വ്യക്തമാക്കുന്നുണ്ട്.

ഹാകിം പറഞ്ഞു: അയാൾ ശക്തനല്ല.

അബൂ ബിശ്ർ അദ്ദൗലാബി പറഞ്ഞു: അയാളുടെ ഹദീസുകൾ വളരെയധികം മുൻകർ (വാസ്തവ വിരുദ്ധം) ആകുന്നു.

* ബൈഹകി, അബൂഹാതിം അർറാസ്സി, ദഹബി, ഇബ്നു ഹജർ, ഇബ്നുൽ മദീനി എന്നിവർ പറഞ്ഞു: അയാൾ ദുർബലനാണ്.

* ഇബ്നുഹിബ്ബാൻ പറഞ്ഞു: അയാൾ സനദുകൾ കോട്ടിമാട്ടുമായിരുന്നു.

* അഹ്‌മദ് പറഞ്ഞു: വിശ്വസ്ഥർക്കെതിരായ നിവേദനങ്ങൾ ഉദ്ധരിക്കുന്ന വ്യക്തിയാണയാൾ.

* ജൊസ്ജാനി പറഞ്ഞു: അയാളുടെ നിവേദനങ്ങൾ വായിച്ച് സമയം കളയരുത്. (https://hadith.islam-db.com/narrators/1334/)

അപ്പോൾ, ഒരു ചരിത്രകാരൻ, ആയിരക്കണക്കിന് നിവേദനങ്ങളുള്ള തന്റെ ഒരു ഗ്രന്ഥത്തിൽ, ഏതോ ഒരു മൂലയിൽ ഒരു നാലു വരി നീളത്തിൽ എഴുതി ചേർത്ത, ‘നുണയനെന്ന് ആരോപിക്കപ്പെട്ട ഒരു റാവി’യിൽ നിന്ന് ഉദ്ധരിക്കപ്പെട്ട ഒരു ‘വ്യാജ നിവേദന’ത്തിൽ, പ്രവാചകൻ (സ) തന്റെ ‘മുറപ്പെണ്ണായ’ ഉമ്മു ഹബീബ് ചെറുതായിരിക്കുമ്പോൾ, ‘വലുതായാൽ’ ഞാൻ ഇവളെ വിവാഹം ചെയ്യുമെന്ന് പറഞ്ഞതിനെ മസാല തേച്ച് കുട്ടികളോടുള്ള ആസക്തിയാക്കി ദുർവ്യാഖ്യാനിക്കുകയാണ് വിവർശകർ ചെയ്തിരിക്കുന്നത് !! ആരോപണത്തിന് വസ്തുതകളുമായി അകന്ന ബന്ധം പോലുമില്ല.

വിമർശനം:

അനസിന്റെ വീട്ടിനടുത്ത് സ്ഥിതിചെയ്യുന്ന കിണറ്റിൽ മുഹമ്മദ് നബി മൂത്രമൊഴിച്ചു. അതിനു ശേഷം കിണറ്റിലെ വെള്ളത്തിൽ അനുഗ്രഹം നൽകപ്പെട്ടുവെന്നും മധുരകരമായ ആ വെള്ളം ശിഷ്യന്മാർ കുടിക്കാറുണ്ടായിരുന്നെന്നും അനസ് പറയുന്നു.

മറുപടി:

പ്രവാചകൻ (സ) കിണറ്റിൽ മൂത്രമൊഴിച്ചു എന്നത് യഥാർത്ഥ്യവുമായി പുലബന്ധം പോലുമില്ലാത്ത ഒരു വ്യാജകഥയാണ്. മുസ്‌ലിം സമുദായത്തിലെ ചില അന്ധവിശ്വാസികൾ ഇത്തരം കഥകളെ കൊണ്ടു നടക്കുന്നു എന്നത് കൊണ്ട് അവ ഇസ്‌ലാമിന്റേയോ, ഇസ്‌ലാമിക പ്രമാണങ്ങളുടേയോ, പ്രവാചക ചരിത്രത്തിന്റേയോ ഭാഗമാണെന്ന് സ്ഥാപിതമാകില്ല.

കാരണങ്ങൾ:

1. കിണർ ഉൾപ്പെടെ ഒഴുകാതെ കെട്ടി നിൽക്കുന്ന ജലത്തിൽ മല-മൂത്ര വിസർജനം നടത്തുന്നത് പ്രവാചകൻ (സ) ശക്തമായി നിരോധിച്ച കാര്യമാണ്.

“ഒഴുകാതെ കെട്ടി നിൽക്കുന്ന വെള്ളത്തിൽ മൂത്രമൊഴിക്കപ്പെടുന്നത് അല്ലാഹുവിന്റെ ദൂതൻ (സ) വിലക്കിയിരിക്കുന്നു” (സ്വഹീഹു മുസ്‌ലിം: 281) എന്ന് പ്രവാചകാനുചരൻ ജാബിർ (റ) പറഞ്ഞതായി സ്വഹീഹായ ഹദീസുകളിൽ വന്നിരിക്കുന്നു. ഇതാണ് ഇസ്‌ലാമിലെ, ഈ വിഷയത്തിലെ സ്ഥിരപ്പെട്ട പ്രവാചക അധ്യാപനം.

“നിശ്ചലമായ അല്ലെങ്കിൽ ഒഴുകാത്ത വെള്ളത്തിൽ മൂത്രമൊഴിക്കുകയും ശേഷം അതിൽ നിന്ന് അംഗശുദ്ധി വരുത്തുകയോ കുളിക്കുകയോ ചെയ്യുന്നത് പ്രവാചകൻ (സ) നിരോധിച്ചു.” (സ്വഹീഹു മുസ്‌ലിം: 282, സുനനു നസാഈ: 399, മുസ്നദു അഹ്‌മദ്‌: 7868, സുനനു ഇബ്നുമാജ: 344)

ഒഴുകാത്ത ജലത്തിൽ ‘ജനാബത്തു'(വലിയ അശുദ്ധി)കാരനായിരിക്കെ കുളിക്കുന്നത് പോലും പ്രവാചകൻ (സ) നിരോധിച്ചു. അതിൽ നിന്ന് വെള്ളം കോരിയെടുത്തേ കുളിക്കാവൂ എന്ന് നിർദ്ദേശിക്കുകയും ചെയ്തു. (സ്വഹീഹു മുസ്‌ലിം)

ജനാബത്തുകാരനായിരിക്കെ ഒഴുകാത്ത ജലസ്രോതസ്സിൽ മുങ്ങി കുളിച്ചാൽ ജലത്തെ അശുദ്ധമാക്കിയേക്കാം എന്നതു പോലും ശ്രദ്ധയോടെ മനസ്സിലാക്കി, അത് നിരോധിച്ച പ്രവാചകൻ (സ) എങ്ങനെയാണ് കിണറ്റിൽ മൂത്രിക്കുക ?!

2. അനസിന്റെ(റ) വീടിനടുത്ത കിണറ്റിൽ പ്രവാചകൻ (സ) മൂത്രമൊഴിച്ചു എന്ന കള്ള കഥയുടെ സ്രോതസ്സ് അബൂ നുഐമിന്റെ ‘ദലാഇലുന്നുബുവ്വ’ എന്ന ഗ്രന്ഥത്തിലെ ഒരു നിവേദനത്തെ അടിസ്ഥാനപ്പെടുത്തി നിർമ്മിക്കപ്പെട്ടതാണ്.

അബൂ നുഐമിന്റെ ‘ദലാഇലുന്നുബുവ്വ’യിലെ ഈ നിവേദനത്തിൽ പ്രവാചകൻ (സ), ആ കിണറ്റിൽ ‘തുപ്പി’ എന്നാണ് വന്നിട്ടുള്ളത്.

أن النبي صلى الله عليه وسلم بزق في بئر داره

നിവേദനത്തിലെ ‘ബസക’ (بزق) അഥവാ ‘തുപ്പി’ എന്ന പദത്തിന് പകരം ‘ബാല’ (بال) അഥവാ ‘മൂത്രിച്ചു’ എന്ന്, അബൂ നുഐമിന്റെ ‘ദലാഇലുന്നുബുവ്വ’ എന്ന ഗ്രന്ഥത്തിന്റെ ചില പ്രതികളിൽ തെറ്റി ചേർക്കപ്പെട്ടിട്ടുണ്ട്. ഇത് കൈയ്യെഴുത്തു പ്രതികൾ ഉണ്ടാക്കുന്ന ചിലരിൽ നിന്ന് ഉടലെടുത്ത് പിന്നീട് പ്രചരിച്ചതാവാം. എന്നാൽ ‘ദലാഇലുന്നുബുവ്വ’ എന്ന ഗ്രന്ഥത്തിന്റെ ശരിയായ പ്രതിയിൽ നിന്ന് നിവേദനം വായിച്ച, ഉദ്ധരിച്ച ഒരുപാടു പണ്ഡിതന്മാർ ഇത് വ്യക്തമാക്കിയിട്ടുണ്ട്. ചില ഉദാഹരണങ്ങൾ കാണുക:

കാദി ഇയാദ് പറഞ്ഞു: “അദ്ദേഹം അനസിന്റെ വീട്ടിലെ കിണറിൽ ‘തുപ്പി’…” (കിതാബു ശിഫാ: 1/246)

ഹാഫിദ് ഇബ്നു ഹജർ പറഞ്ഞു: “ബുജൈരിയുടെ ഹദീസിൽ പ്രവാചകൻ (സ) അനസിന്റെ വീട്ടിലെ കിണറിൽ ‘തുപ്പി’ എന്ന് പ്രസ്ഥാവിക്കുന്നു…” (ഇത്ഹാഫുൽ മഹ്റ: 1/575: 784)

ജലാലുദ്ദീൻ സുയൂത്വി പറഞ്ഞു: “അബൂ നുഐം അനസിൽ നിന്നും ഉദ്ധരിക്കുന്ന ഒരു നിവേദനത്തിൽ പ്രവാചകൻ (സ) അദ്ദേഹത്തിന്റെ വീട്ടിലെ കിണറ്റിൽ ‘തുപ്പി’യെന്ന് പറയുന്നു…” (ഖസാഇസുൽ കുബ്റാ: 1/105)

അബൂ നുഐം അനസിൽ നിന്നും ഉദ്ധരിക്കുന്ന നിവേദനത്തിൽ പ്രവാചകൻ (സ) അദ്ദേഹത്തിന്റെ വീട്ടിലെ കിണറ്റിൽ ‘തുപ്പി’യെന്നാണ് പറയുന്നത് എന്ന് അലിയ്യുബ്നു അബ്ദുല്ല അസ്സംഹൂദിയും (ഖുലാസത്തുൽ വഫാ: 268) മുഹമ്മദ് ഇബ്നു യൂസുഫ് അസ്സ്വാലിഹിയും വ്യക്തമാക്കുന്നുണ്ട്. (സുബുലുൽ ഹുദാ വർറശാദ്: 7/223)

അപ്പോൾ അബൂ നുഐമിന്റെ ‘ദലാഇലുന്നുബുവ്വ’ എന്ന ഗ്രന്ഥത്തിന്റെ ശരിയായ പ്രതികളിൽ കിണറ്റിൽ ‘തുപ്പി’ എന്നാണുള്ളത്. ‘തുപ്പി’ എന്ന പദത്തിന് പകരം ‘മൂത്രിച്ചു’ എന്ന് വന്നത്, ഗ്രന്ഥത്തിന്റെ ചില പ്രതികളിൽ ഉണ്ടായ അക്ഷര തെറ്റോ, പകർത്തി എഴുത്തുകാരുടെ (വർറാക്കുകൾ) ഓർമ്മ പിശകോ മാത്രമാണ്.

3. ഇനി, കിണറ്റിൽ തുപ്പി എന്ന തെറ്റില്ലാത്ത നിവേദനം തന്നെ സ്വഹീഹ് (ഹദീസ് നിദാനശാസ്ത്ര മാനദണ്ഡങ്ങൾ പ്രകാരം സ്വീകാര്യമായത്) ആണോ എന്ന് അടുത്തതായി നമുക്ക് പരിശോധിക്കാം.

കഥയുടെ നിവേദക പരമ്പര ഇപ്രകാരമാണ്: حدثنا علي بن هارون ثنا موسى بن هارون قال: ثنا عبيد الله بن النعمان المنقري قال: ثنا محمد بن عبد الله الأنصاري قال: حدثني أبي، عن ثمامة، عن أنس قال: كان رسول الله صلى الله عليه وسلم يصلي…

ഈ സനദ് അഥവാ നിവേദക പരമ്പര അങ്ങേയറ്റം ദഈഫ് (ദുർബലം ضعيف) ആകുന്നു. പരമ്പരയിലെ അലിയ്യിബ്നു ഹാറൂൻ എന്ന ‘റാവി’ (നിവേദകൻ) ഓർമ്മക്കുറവുള്ള വ്യക്തിയും നിവേദനങ്ങളുടെ ആശയങ്ങളിൽ കൂടി കലരൽ സംഭവിക്കുന്ന വ്യക്തിയുമാണ്. പരമ്പരയിലെ മറ്റൊരു റാവി (നിവേദകൻ) ഉബൈദുല്ലാഹിബ്നു നുഅ്മാൻ അൽ മിൻകരി ‘മജ്ഹൂൽ’ (مجهول) അഥവാ അറിയപ്പെടാത്ത റാവിയാകുന്നു. പരമ്പരയിലെ മൂന്നാമതൊരു റാവിയായ അബ്ദുല്ലാഹിബ്നുൽ മുസന്നാ അൽ അൻസ്വാരി: നിവേദനത്തിൽ ധാരാളം സ്ഖലിതങ്ങൾ സംഭവിക്കുന്ന റാവിയാണ്. (അൽ ജർഹു വതഅ്ദീൽ: 5/ 177)

ചുരുക്കത്തിൽ, കിണറ്റിൽ ‘തുപ്പി’ എന്ന നിവേദനം തന്നെ ‘ദഈഫ്’ (ഹദീസ് നിദാനശാസ്ത്ര പ്രകാരം ദുർബലമായ നിവേദനം) ആകുന്നു. ഇസ്‌ലാം മതത്തിന്റെയും പ്രവാചക ചരിത്രത്തിന്റേയും അടിത്തറ ക്വുർആനും ‘സ്വഹീഹായ ഹദീസുകളും’ മാത്രമാണ്. ദഈഫ് (ദുർബലം) ആയ ഹദീസുകൾ ഇസ്‌ലാമിക പ്രമാണമല്ല.

വിമര്‍ശനം:

നബിയുടെ മാതൃസഹോദരി ഖൗല ബിന്‍ത് ഹകീം എന്ന സ്ത്രീ സ്വന്തം ശരീരം നബിക്കു ദാനം നല്‍കുകയുണ്ടായി എന്നും അതിനെതിരെ സ്വപത്‌നി ആഇശ പോലും നബിയോട് ‘ഒരു സ്വതന്ത്ര സ്ത്രീ സ്വദേഹം ദാനം ചെയ്യുകയോ’ എന്ന് പ്രതികരിച്ചുവെന്നും ഹദീഥ് ഗ്രന്ഥങ്ങളില്‍ കാണാം. എന്നാല്‍ നബി ഒരിക്കലും അതിനെ മോശമായി കണ്ടിരുന്നില്ല എന്നും ഹദീഥ് ഗ്രന്ഥങ്ങള്‍ തന്നെ മനസ്സിലാക്കി തരുന്നുണ്ട്. സ്ത്രീകളെ കൊണ്ട് സ്വശരീരങ്ങള്‍ തനിക്കായി ദാനം ചെയ്യാന്‍ പ്രേരിപ്പിക്കുകയും സ്വന്തം മാതൃസഹോദരിയെ പോലും അതില്‍ നിന്നും ഒഴിവാക്കാതിരിക്കുകയും ചെയ്ത പ്രവാചക നടപടിയെ അല്‍പമെങ്കിലും ധാര്‍മികബോധമുള്ളവര്‍ക്ക് അംഗീകരിക്കാനാകുമോ?

മാത്രമല്ല, ഉമ്മു ഹറാം, ഉമ്മു സുലൈം എന്നീ സ്ത്രീകളെ നബി സ്വകാര്യ സന്ദര്‍ശനം നടത്താറുണ്ടായിരുന്നെന്നും അവരുടെ മടിയില്‍ പലപ്പോഴും തലവെച്ചു കിടക്കാറുണ്ടായിരുന്നു എന്നും ഹദീഥ് ഗ്രന്ഥങ്ങള്‍ തന്നെ വ്യക്തമാക്കുന്നുണ്ട്. അവിഹിത ബന്ധങ്ങള്‍ക്ക് പ്രവാചക ജീവിതത്തിലിടമുണ്ടായിരുന്നു എന്നു വ്യക്തമാക്കുന്ന സംഭവങ്ങളാണിതെല്ലാം. ഇത്തരത്തിലുള്ള ഒരു വ്യക്തിയെ എങ്ങനെയാണ് മാര്‍ഗദര്‍ശകനായി സ്വീകരിക്കാനാവുക.!?

മറുപടി:

അറബിയില്‍ ഒരു പഴമൊഴിയുണ്ട് ‘നാട്ടിയത് വളഞ്ഞാല്‍ നിഴല്‍ നേരെയാവുമോ?’. അക്ഷരാര്‍ഥത്തില്‍ ഈ അറേബ്യന്‍ പഴമൊഴിയെ ഓര്‍മപ്പെടുത്തുകയാണീ വിമര്‍ശനങ്ങള്‍. വളഞ്ഞ വസ്തു നാട്ടിയിട്ട് അതിന്റെ നിഴലിനെ നേരെയാക്കാന്‍ നോക്കിയിട്ട് കാര്യമുണ്ടോ!?. നിഴല്‍ നേരെയാകണമെങ്കില്‍ ആദ്യം വസ്തു നേരെയാക്കണം. തലമുടി മുതല്‍ കാല്‍നഖം വരെ ലൈംഗികവൈകൃതം പേറുന്നവരില്‍ ഇസ്‌ലാംവിരോധം ഇണചേര്‍ന്നാല്‍ ഇതല്ല ഇതിനപ്പുറവും നബിജീവിതത്തില്‍ നിന്നും ‘ഉല്‍ഘനനം’ ചെയ്‌തെടുത്തുകളയുമവര്‍. ചികിത്സിക്കേണ്ടത്, ലൈംഗിക വൈകൃതാനുരക്ത ഇസ്‌ലാമോഫോബിക്കുകളുടെ (Sexually Deviant Islamophobes) ഹൃദയങ്ങളില്‍ കട്ടപിടിച്ചു കിടക്കുന്ന ഇസ്‌ലാംവെറുപ്പും ലൈംഗിക വൈകൃതാസക്തിയുമാണ്. അപ്പോള്‍ മാത്രമേ അവരുടെ ഹൃദയങ്ങളില്‍ നിന്നും പ്രതിഫലിപ്പിക്കപ്പെടുന്ന വിമര്‍ശനങ്ങള്‍ക്ക് സത്യസന്ധതയും മാന്യതയുമുണ്ടാവുകയുള്ളൂ. ആരായിരുന്നു പ്രവാചകരെന്നും എന്തായിരുന്നു അവിടുത്തെ ജീവിതവും സന്ദേശവുമെന്നും നേര്‍ക്കുനേരെ ഇസ്‌ലാമിക പ്രമാണങ്ങളില്‍ നിന്നും നിഷ്പക്ഷമായി പഠിക്കാന്‍ തയ്യാറാകാതെ, വെറുപ്പു കച്ചവടത്തിന്റെ വിപണന സാധ്യതകള്‍ മാത്രം തേടി നബിചരിത്രം പരിശോധിക്കുന്നവരില്‍ നിന്നും ഒരിക്കലും സത്യസന്ധമായ ചരിത്ര വായന കേള്‍ക്കാന്‍ സാദ്ധ്യമല്ലെന്ന് ബധിരത ബാധിക്കാത്തവര്‍ക്കെല്ലാം അറിയാവുന്ന വസ്തുതയാണ്.

ഇസ്‌ലാംപൂര്‍വ കാലഘട്ടത്തില്‍ നിലനിന്നിരുന്ന സകല വിധ ലൈംഗിക അധാര്‍മികതകള്‍ക്കും നീചവൃത്തികള്‍ക്കുമെതിരെ സമൂഹത്തെ ബോധവല്‍ക്കരിക്കുകയും വിശുദ്ധമായ ലൈംഗിക ജീവിതത്തിലേക്ക് തന്റെ ജനതയെ വഴിനടത്തുകയും ചെയ്തത് പ്രവാചകരായിരുന്നു എന്ന് അദ്ധേഹത്തിന്റെ സമകാലികരില്‍ – ശത്രു മിത്ര വ്യത്യാസമില്ലാതെ- ഒരാള്‍ പോലും അംഗീകരിക്കാതിരുന്നിട്ടില്ല എന്ന ചരിത്ര വസ്തുതയെങ്കിലും മുന്നില്‍ വെച്ചുകൊണ്ടായിരുന്നു വിമര്‍ശകന്മാര്‍ നബിചരിത്രം വായിക്കേണ്ടിയിരുന്നത്. ബദ്ധശത്രുക്കള്‍ പോലും പ്രവാചകനെതിരെ വ്യഭിചാരാരോപണമോ ലൈംഗികവൈകൃതമോ ഉന്നയിക്കാതിരുന്നത് എന്തുകൊണ്ടാണെന്ന് നബിവിമര്‍ശകന്മാര്‍ ഏറെ ചിന്തിക്കേണ്ട വസ്തുതയാണ്. ഇന്ന് നബിവൈരികള്‍ ദുര്‍വ്യാഖ്യാനിച്ച് ദുഷിപ്പിച്ചവതരിപ്പിക്കുന്ന ചരിത്ര സന്ദര്‍ഭങ്ങളും ഹദീഥ് നിവേദനങ്ങളും അവര്‍ പ്രചരിപ്പിക്കുന്ന വിധമായിരുന്നുവെങ്കില്‍ എത്രമാത്രം ഒച്ചപ്പാടുകളും ബഹളകോലാഹലങ്ങളും പ്രവാചക കാലഘട്ടത്തില്‍ തന്നെ അതുമൂലം സൃഷ്ടിക്കപ്പെടുമായിരുന്നു. പക്ഷെ ഇസ്‌ലാമിക ചരിത്ര ഗ്രന്ഥങ്ങളിലൊരിടത്തും അത്തരത്തിലുള്ള ഒച്ചപ്പാടുകളെ പറ്റിയോ ബഹളകോലാഹലങ്ങളെ പറ്റിയോ ഉള്ള യാതൊരു സൂചനയും കാണുകയില്ല. നബിയെ പറ്റിയുള്ള വിവരങ്ങള്‍ അന്വേഷിച്ച ഹിര്‍ക്കിലിനെ പേലെയുള്ള ആ കാലഘട്ടത്തിലെ ചക്രവര്‍ത്തിമാരോടും രാജാക്കന്മാരോടും പരമാവധി പ്രവാചകനെ പ്രതികൂലിച്ചു വിവരങ്ങള്‍ പങ്കുവെക്കാന്‍ ശ്രമിച്ച ശത്രുക്കളിലൊരാള്‍ പോലും നബിക്കെതിരെ വ്യഭിചാരാരോപണമോ ലൈംഗികവൈകൃതമോ ഉന്നയിച്ചിട്ടില്ല എന്നതും വളരെ ശ്രദ്ധേയമായ വസ്തുതയാണ്.

മാത്രമല്ല, പ്രവാചകനെതിരെ ഈ ലൈംഗിക വൈകൃതാനുരക്ത ഇസ്‌ലാമോഫോബിക്കുകള്‍ (Sexually Deviant Islamophobes) ആരോപിക്കുന്ന വ്യഭിചാരാരോപണങ്ങളും ലൈംഗികവൈകൃതങ്ങളും ഇസ്‌ലാമില്‍ ഹദ്ദ് (ശിക്ഷ) നല്‍കപ്പെടുന്ന കുറ്റകൃത്യങ്ങളാണ്. ഇവര്‍ ആരോപിക്കും വിധം നബി(സ്വ) തന്നെ അത്തരം കുറ്റങ്ങള്‍ ചെയ്‌തെന്നിരിക്കെ എന്തു ന്യായം പറഞ്ഞാണ് ഇസ്‌ലാമിക രാഷ്ട്രങ്ങള്‍ പൗരന്മാര്‍ക്കുമേല്‍ പ്രസ്തുത തെറ്റുകള്‍ക്ക് ശിക്ഷ നടപ്പാക്കുക; കര്‍മശാസ്ത്ര ഗ്രന്ഥങ്ങള്‍ എങ്ങനെയാണ് പ്രസ്തുത തെറ്റുകള്‍ക്ക് വിധിക്കപ്പെടേണ്ട ശിക്ഷകളെ പറ്റി ചര്‍ച്ച ചെയ്യുക. എന്തിനധികം പറയണം നബി (സ്വ) തന്നെ അത്തരം കുറ്റകൃത്യങ്ങള്‍ക്ക് ഭരണാധികാരി എന്ന നിലയില്‍ ഹദ്ദ് (ശിക്ഷ) നല്‍കിയിട്ടുണ്ട്. വിമര്‍ശകര്‍ ആരോപിക്കും വിധമാണ് കാര്യങ്ങളെങ്കില്‍, നബി തന്നെ ചെയ്തിട്ടുള്ള അത്തരം കുറ്റങ്ങളുടെ പേരില്‍ മറ്റുള്ളവരെ ശിക്ഷിക്കാന്‍ എന്തു ന്യായമായിരിക്കും അദ്ധേഹത്തിനു പറയാനുണ്ടാവുക. വ്യഭിചാരത്തിന്റെ പേരില്‍ ശിക്ഷിക്കപ്പെട്ട കുറ്റവാളികളില്‍ ഒരാള്‍ പേലും പ്രവാചക ജീവിതത്തെ ചൂണ്ടി കാണിച്ച് തങ്ങളുടെ ചെയ്തികളെ ന്യായീകരിക്കാന്‍ ശ്രമിച്ചിട്ടില്ല. ചൂണ്ടിക്കാണിക്കുവാന്‍ അത്തരം ന്യായങ്ങളുണ്ടായിരുന്നെങ്കില്‍ തീര്‍ച്ചയായും അവരതു ചെയ്യുമായിരുന്നു. അത്തരം ചെയ്തികളുടെ പേരില്‍ ശിക്ഷക്കപ്പെട്ടവരില്‍ മുസ്‌ലിംകളും ജൂതന്മാരുമടക്കം നാനാവിധത്തിലുള്ളവരുണ്ടായിരുന്നിട്ടും ഒരാള്‍ പോലും അതു പറഞ്ഞില്ലെങ്കില്‍ അതിനര്‍ഥമെന്താണ്?. നബിവൈരികള്‍ ആരോപിക്കും വിധം ഒരു പുഴുക്കുത്തും നബിജീവിതത്തില്‍ നിന്നും കണ്ടെടുക്കാനാവില്ല എന്നുതന്നെ. ഒരു ചരിത്ര സംഭവത്തിലേക്കു ദൃഷ്ടി തിരിക്കാം നമുക്ക്:

”അബ്ദുല്ലാഹിബ്‌നു ഉമര്‍ (റ) നിവേദനം: വ്യഭിചാരത്തിലുള്‍പ്പെട്ട ഒരു യഹൂദനേയും യഹൂദ സ്ത്രീയേയും നബി(സ്വ)യുടെ മുന്നില്‍ ഹാജരാക്കി. നബി (സ്വ) യഹൂദന്മാരുടെ അടുക്കല്‍ ചെന്നു ചോദിച്ചു: ‘വ്യഭിചാരത്തിന്റെ ശിക്ഷ തൗറാത്തില്‍ എങ്ങനെയാണ് കാണുന്നത്?’ അവര്‍ പറഞ്ഞു: ‘അവര്‍ രണ്ടുപേരുടേയും മുഖം കറുപ്പിക്കും. കഴുതപ്പുറത്ത് കയറ്റി പുറം തിരിച്ചിരുത്തും. അവരെ പ്രദക്ഷിണം ചെയ്യിക്കും.’ നബി(സ്വ) പറഞ്ഞു: ‘നിങ്ങള്‍ തൗറാത്ത് കൊണ്ടു വരിക; നിങ്ങള്‍ സത്യം പറയുന്നവരാണെങ്കില്‍.’ അവര്‍ തൗറാത്തുമായി വന്നു വായിക്കാന്‍ തുടങ്ങി. അങ്ങനെ എറിഞ്ഞു കൊല്ലുന്ന ശിക്ഷാവിധി അടങ്ങിയ ഭാഗത്തെത്തിയപ്പോള്‍ അത് വായിച്ചിരുന്ന ചെറുപ്പക്കാരന്‍ ആ വിധികളടങ്ങിയ സൂക്തങ്ങളുടെ മേല്‍ കൈവച്ചു. അതിന്റെ അപ്പുറവും ഇപ്പുറവും വായിച്ചു. നബി(സ്വ)യുടെ കൂടെയുണ്ടായിരുന്ന അബ്ദുല്ലാഹി ബ്‌നു സലാം (റ) പറഞ്ഞു: ‘അയാളോട് കൈ മാറ്റാന്‍ ആവശ്യപ്പെടുക.’ അയാള്‍ കൈ ഉയര്‍ത്തി. അതില്‍ എറിഞ്ഞുകൊല്ലണമെന്ന വിധി അടങ്ങിയ സൂക്തമുണ്ടായിരുന്നു. നബി (സ്വ) അവര്‍ രണ്ടുപേരേയും എറിഞ്ഞുകൊല്ലാന്‍ കല്‍പ്പിക്കുകയും അങ്ങനെ എറിഞ്ഞു കൊല്ലുകയും ചെയ്തു,” (സ്വഹീഹ് മുസ്‌ലിം: 1699) ഈ ചരിത്ര സംഭവം എന്താണ് നമുക്ക് വിവരിച്ചുതരുന്നത്. വ്യഭിചാരത്തിലേര്‍പ്പെട്ട യഹൂദന്‍ ഉന്നത സ്ഥാനത്തിനുടമയായതുകൊണ്ട് അയാളെ രക്ഷപ്പെടുത്താന്‍ സ്വന്തം വേദഗ്രന്ഥത്തിലുള്ള വിധിപോലും മറച്ചു പിടിച്ച് പ്രവാചകനെ തെറ്റിദ്ധരിപ്പിക്കാന്‍ പരിശ്രമിക്കുകയാണ് യഹൂദന്മാര്‍. പക്ഷെ തൗറാത്ത് നന്നായി അറിയുന്ന, യഹൂദ പണ്ഡിതനായിരുന്ന അബ്ദുല്ലാഹി ബ്‌നു സലാം (റ) അത് കൈയ്യോടെ പിടികൂടുകയും അങ്ങനെ കുറ്റക്കാരനായ യഹൂദനെ രക്ഷിക്കാന്‍ മറ്റു വഴികളില്ലാതെ യഹൂദന്മാര്‍ക്ക് തൗറാത്തിലെ വിധി നടപ്പാക്കേണ്ടി വരികയും ചെയ്യുന്നു. നബിവൈരികള്‍ ആരോപിക്കും വിധം പ്രവാചക ജീവിതം തന്നെ അവിഹിത ബന്ധങ്ങള്‍ നിറഞ്ഞതോ ലൈംഗിക വൈകൃതങ്ങള്‍ പേറിയതോ ആയിരുന്നെങ്കില്‍ യഹൂദന്മാര്‍ നബിയുടെ നിയമനടപടിക്കെതിരെ ആദ്യമായി ഉന്നയിക്കുന്ന തുരുപ്പുചീട്ട് അതാകുമായിരുന്നു. താങ്കള്‍ തന്നെ ചെയ്തുകൊണ്ടിരിക്കുന്ന തെറ്റ് ഞങ്ങളുടെ കൂട്ടത്തില്‍ ഒരാള്‍ ചെയ്‌തെന്നിരിക്കെ അതിനെതിരെ ശിക്ഷാ നടപടി സ്വീകരിക്കാന്‍ താങ്കള്‍ക്കെന്തധികാരം എന്നവര്‍ ചോദിക്കുമായിരുന്നു. പക്ഷെ ഒരു യഹൂദനും അതിനു സാധിച്ചില്ല. നിശബ്ദരായി തൗറാത്തിലെ വിധി നടപ്പാക്കേണ്ടി വന്നു അവര്‍ക്ക്. എന്തുകൊണ്ട്!? ഉത്തരം വളരെ ലളിതമാണ്. നബിവിരോധികള്‍ കച്ചകെട്ടിയുണ്ടാക്കിയ ഈ കളവുകള്‍ അവര്‍ക്കറിയില്ലായിരുന്നു. സ്വന്തം വേദഗ്രന്ഥത്തില്‍ വരെ കളവു കാണിക്കുന്ന യഹൂദനെ പോലും തോല്‍പ്പിക്കുന്ന ഈ പുതിയ കാല മഹാ കള്ളന്മാരെ അന്ന് ആ യഹൂദന്മാര്‍ പരിചയപ്പെട്ടിട്ടില്ലായിരുന്നു.

മാത്രമല്ല, ലൈംഗിക വിശുദ്ധിയുമായി ബന്ധപ്പെട്ട നബിയുടെ ഉപദേശങ്ങള്‍ അതീവ കാര്‍ക്കശ്യ സ്വഭാവമുള്ളവയാണെന്ന് അതില്‍ നിന്നും അല്‍പമെങ്കിലും കേട്ടിട്ടുള്ളവര്‍ക്കെല്ലാം അറിയാവുന്ന വസ്തുതയാണ്. ഒരു സ്ത്രീയും പുരുഷനും ഒറ്റക്കായാല്‍ മൂന്നാമന്‍ പിശാചാണ്, അന്യ സ്ത്രീയുടെ നേര്‍ക്കുള്ള രണ്ടാമത്തെ നേട്ടം പുരുഷനു നിഷിദ്ധമാണ്, അന്യസ്ത്രീയെ സ്പര്‍ശിക്കുന്നതിനേക്കാള്‍ പുരുഷനു നല്ലത് തലയില്‍ ഇരുമ്പാണി തറക്കുന്നതാണ്, മനുഷ്യരെ ഏറ്റവുമധികം നരകാര്‍ഹരാക്കുന്നത് നാവിന്റെയും ലൈംഗികാവയവങ്ങളുടെയും വിശുദ്ധിയില്ലായ്മയാണ്, ആര്‍ ലൈംഗികാവയവങ്ങളെ കാത്തു സൂക്ഷിക്കുന്നുവേ അവനു ഞാന്‍ സ്വര്‍ഗം ജാമ്യം നില്‍ക്കുന്നു, ലൈംഗിക വിശുദ്ധി കാത്തു സൂക്ഷിക്കുവാനായി വിവാഹം ചെയ്യാന്‍ കഴിവില്ലാത്തവന്‍ വ്രതമനുഷ്ഠിക്കട്ടെ, വ്രതം ലൈംഗിക വിശുദ്ധിയുടെ പരിചയാണ്, അന്യസ്ത്രീയാല്‍ ആകര്‍ഷിക്കപ്പെട്ടവന്‍ സ്വന്തം ഭാര്യയുടെ അരികില്‍ പേകട്ടെ കാരണം അവള്‍ക്കുള്ളതു തന്നെയാണ് ഇവള്‍ക്കുള്ളതും, വൃദ്ധനായ വ്യഭിചാരിയോട് ദൈവം മുഖം തിരിക്കും, അത്തരക്കാര്‍ക്ക് ദൈവത്തിന്റെ കാരുണ്യ കടാക്ഷം ലഭിക്കില്ല, അയല്‍ വാസിയുടെ ഭാര്യയെ വ്യഭിചരിക്കല്‍ വന്‍ പാപമാകുന്നു, ദമ്പധിമാര്‍ കിടപ്പറ രഹസ്യങ്ങള്‍ പരസ്യപ്പെടുത്തുന്നത് പാപമാണ്, കുടുംബത്തെ മ്ലേഛ വൃത്തിക്കു കൂട്ടികൊടുക്കുന്നവന്‍ സ്വര്‍ഗത്തില്‍ പ്രവേശിക്കുകയില്ല, പരപുരുഷന്മാരെ വശീകരിക്കാനായി സുഖന്ധമുപയോഗിക്കുന്ന സ്ത്രീകള്‍ നരകത്തിലാണ്; അവളെ നോക്കുന്ന കണ്ണുകളും നരകത്തിലാണ്, സ്വവര്‍ഗ സംഭോഗിയെ അല്ലാഹു ശപിച്ചിരിക്കുന്നു, മൃഗരതിയിലേര്‍പ്പെടുന്നവനെയും മൃഗത്തെയും വധിക്കണം, വിവാഹിതനായ വ്യഭിചാരിയെ ചാട്ടവാറുകൊണ്ട് നൂറടിയടിക്കണം; എറിഞ്ഞുകൊല്ലണം, അവിവാഹിതനായ വ്യഭിചാരിയെ ചാട്ടവാറുകൊണ്ട് നൂറടിയടിക്കണം; നാടുകടത്തണം തുടങ്ങി പാരാവാരം കണക്കെ രേഖപ്പെടുത്തപ്പെട്ടിരിക്കുന്നു ലൈംഗിക വിശുദ്ധിയുടെ നബിപാഠങ്ങള്‍. വിമര്‍ശകര്‍ ആരോപിക്കും വിധം അവിഹിത ബന്ധങ്ങളും ലൈംഗികവൈകൃതങ്ങളും നിറഞ്ഞതായിരുന്നു നബിജീവിതമെങ്കില്‍ പിന്നെ എങ്ങനെയാണ് പ്രവാചകന്‍ തന്റെ ജനതക്ക് ഇത്രമാത്രം ലൈംഗിക വിശുദ്ധിയുടെ ജീവിത പാഠങ്ങള്‍ പകര്‍ന്നു നല്‍കിയത്. പ്രവാചകനില്‍ നിന്നും വിശുദ്ധിയുടെ ജീവിത പാഠങ്ങള്‍ കേട്ട് സ്വജീവിതത്തെ അതിനനുസൃതമായി ചിട്ടപ്പെടുത്താന്‍ അങ്ങേയറ്റത്തെ ത്യാഗ പരിശ്രമങ്ങള്‍ നടത്തിയ ഒരു ജനത, തങ്ങള്‍ക്കു നല്‍കിയ ഉപദേശങ്ങള്‍ക്കു കടക വിരുദ്ധമായി ജീവിക്കുന്ന ഉപദേശകനെ ജീവനേക്കാള്‍ അധികമായി സ്‌നേഹിച്ചുവെന്ന് കരുതാന്‍ ബുദ്ധിഭ്രമം പിടിപെടുക തന്നെ വേണം. സ്വന്തം ജീവനും ജീവിതവും സമ്പാദ്യവും സന്തോഷവുമെല്ലാം ആ ഉപദേശകന്റെ കാല്‍കീഴില്‍ കൊണ്ടു സമര്‍പ്പിക്കുവാന്‍ ആ ജനത തയ്യാറായത്, കേട്ട ഉപദേശങ്ങളെക്കാള്‍ മഹത്തരമായ ജീവിതത്തെ നേരില്‍ കണ്ടതുകൊണ്ടാണെന്ന് തിരിച്ചറിയാന്‍ സത്യസന്ധതക്ക് ജീവിതത്തില്‍ ചെറിയ ഒരിടം കൊടുത്താല്‍ മാത്രം മതി.

ഇമാം അഹ്‌മദ്‌ മുസ്‌നദിലും (5/256) ത്വബ്‌റാനി മുഅ്ജമുല്‍ കബീറിലും (8/162, 183) ബൈഹക്വി ശുഅ്ബുല്‍ ഈമാനിലും (4/362(5415)) ഉദ്ദരിച്ച ഒരു ഹദീഥ് കാണുക: ”അബൂ ഉമാമഃ(റ)യില്‍ നിന്നും നിവേദനം: ഒരു യുവാവ് നബി(സ്വ)യുടെ അടുക്കലേക്ക് വന്നു. എന്നിട്ടവന്‍ പറഞ്ഞു: അല്ലാഹുവിന്റെ ദൂതരെ എനിക്കു വ്യഭിചരിക്കാന്‍ അനുവാദം നല്‍കിയാലും. അപ്പോള്‍ ജനങ്ങള്‍ അയാളുടെ അടുത്തു ചെന്ന് അയാളെ തടഞ്ഞുകൊണ്ട് മിണ്ടാതിരിക്കാന്‍ ആവശ്യപ്പെട്ടു. അപ്പോള്‍ നബി(സ്വ) പറഞ്ഞു: അദ്ധേഹത്തെ എന്റടുത്തേക്ക് വിടൂ. അപ്പോള്‍ ആ യുവാവ് നബി(സ്വ)യുടെ അരികിലേക്ക് വന്നിരുന്നു. എന്നിട്ട് നബി(സ്വ) അയാളോട് ചോദിച്ചു:

‘നിന്റെ മാതാവിനു നീ അത് (വ്യഭിചാരം) ഇഷ്ടപ്പെടുന്നുണ്ടോ? അയാള്‍ പറഞ്ഞു: ഇല്ല. അല്ലാഹുവാണേ, അല്ലാഹു എന്നെ താങ്കള്‍ പ്രായശ്ചിത്തമാക്കട്ടെ. അപ്പോള്‍ നബി(സ്വ) പറഞ്ഞു: ജനങ്ങളും അവരുടെ മാതാക്കള്‍ക്ക് അത് (വ്യഭിചാരം) ഇഷ്ടപ്പെടുന്നില്ല.’ ‘നബി(സ്വ) ചോദിച്ചു: നിന്റെ മകള്‍ക്ക് നീ അത് (വ്യഭിചാരം) ഇഷ്ടപ്പെടുന്നുണ്ടോ? അയാള്‍ പറഞ്ഞു: ഇല്ല. അല്ലാഹുവാണേ, അല്ലാഹു എന്നെ താങ്കള്‍ പ്രായശ്ചിത്തമാക്കട്ടെ. അപ്പോള്‍ നബി(സ്വ) പറഞ്ഞു: ജനങ്ങളും അവരുടെ പുത്രിമാര്‍ക്ക് അത് (വ്യഭിചാരം) ഇഷ്ടപ്പെടുന്നില്ല.’ ‘എന്നിട്ട് വീണ്ടും നബി(സ്വ) അയാളോട് ചോദിച്ചു: നിന്റെ സഹോദരിക്ക് നീ അത് (വ്യഭിചാരം) ഇഷ്ടപ്പെടുന്നുണ്ടോ? അയാള്‍ പറഞ്ഞു: ഇല്ല. അല്ലാഹുവാണേ, അല്ലാഹു എന്നെ താങ്കള്‍ പ്രായശ്ചിത്തമാക്കട്ടെ. അപ്പോള്‍ നബി(സ്വ) പറഞ്ഞു: ജനങ്ങളും അവരുടെ സഹോദരിമാര്‍ക്ക് അത് (വ്യഭിചാരം) ഇഷ്ടപ്പെടുന്നില്ല.’

‘നബി(സ്വ) വീണ്ടും ചോദിച്ചു: നിന്റെ അമ്മായിക്ക് നീ അത് (വ്യഭിചാരം) ഇഷ്ടപ്പെടുന്നുണ്ടോ? അയാള്‍ പറഞ്ഞു: ഇല്ല. അല്ലാഹുവാണേ, അല്ലാഹു എന്നെ താങ്കള്‍ പ്രായശ്ചിത്തമാക്കട്ടെ. അപ്പോള്‍ നബി(സ്വ) പറഞ്ഞു: ജനങ്ങളും അവരുടെ അമ്മായിമാര്‍ക്കും അത് (വ്യഭിചാരം) ഇഷ്ടപ്പെടുന്നില്ല.’

‘വീണ്ടും അവിടുന്ന് ചോദിച്ചു: നിന്റെ മാതൃസഹോദരിക്ക് നീ അത് (വ്യഭിചാരം) ഇഷ്ടപ്പെടുന്നുണ്ടോ? അയാള്‍ പറഞ്ഞു: ഇല്ല. അല്ലാഹുവാണേ, അല്ലാഹു എന്നെ താങ്കള്‍ പ്രായശ്ചിത്തമാക്കട്ടെ. അപ്പോള്‍ നബി(സ്വ) പറഞ്ഞു: ജനങ്ങളും അവരുടെ മാതൃസഹോദരിമാര്‍ക്കും അത് (വ്യഭിചാരം) ഇഷ്ടപ്പെടുന്നില്ല.’

എന്നിട്ട് നബി (സ്വ) അദ്ധേഹത്തിന്റെ കൈകള്‍ അയാളുടെ ശരീരത്തില്‍ വെച്ചുകൊണ്ട് പ്രാര്‍ഥിച്ചു: ‘അല്ലാഹുവേ ഇദ്ധേഹത്തിന്റെ പാപം നീപൊറുത്തുകൊടുക്കുകയും ഹൃദയം ശുദ്ധീകരിക്കുകയും ലൈംഗികാവയവങ്ങളെ സംരക്ഷിക്കുകയും ചെയ്യേണമേ.’ അതിനു ശേഷം ആ യുവാവ് ഒന്നിലേക്കും (ലൈംഗികമായ ദുര്‍വൃത്തിയിലേക്ക്) തിരിഞ്ഞു നോക്കിയിരുന്നില്ല.”

വ്യഭിചാരത്തിന് അനുവാദം നല്‍കണമെന്നാവശ്യപ്പെട്ടുകൊണ്ട് പ്രവാചക സന്നിധിയിലേക്ക് കടന്നു വന്ന ഒരു യുവാവിനെ മനഃശാസ്ത്രപരമായി സമീപിക്കുകയാണിവിടെ പ്രവാചകന്‍ (സ്വ). അവനോടായി അദ്ധേഹം പറയുകയാണ്, നിന്റെ മാതാവും മകളും സഹോദരിയും അമ്മായിയും മാതൃസഹോദരിയുമൊക്കെ വ്യഭിചരിക്കുന്നത് നീ ഇഷ്ടപ്പെടുന്നില്ല. എന്നാല്‍ നീ വ്യഭിചരിക്കുന്ന സ്ത്രീയാകട്ടെ തീര്‍ച്ചയായും മറ്റൊരുവന്റെ മാതാവോ മകളോ സഹോദരിയോ അമ്മായിയോ മാതൃസഹോദരിയോ ആയിരിക്കും. അവനും നിന്നെപ്പോലെ സ്വന്തം മാതാവും മകളും സഹോദരിയും അമ്മായിയും മാതൃസഹോദരിയുമൊക്കെ വ്യഭിചരിക്കുന്നത് ഇഷ്ടപ്പെടുന്നവനല്ല. സ്വന്തക്കാരായ സ്ത്രീകള്‍ വ്യഭിചരിക്കുന്നത് വെറുക്കുന്ന നീ മറ്റുള്ളവരുടെ സ്വന്തക്കാരായ സ്ത്രീകളെ വ്യഭിചരിക്കാനനുവാദം ചോദിക്കുന്നത് എത്രമാത്രം അക്രമമാണ്. അയാള്‍ക്ക് ബോധോദയമുണ്ടായി ആ നികൃഷ്ഠമായ ആഗ്രഹം വെടിഞ്ഞ് വിശുദ്ധമായ ജീവിതം അയാള്‍ തിരഞ്ഞെടുത്തു. ഇനി നാം ചിന്തിക്കുക, നബിവൈരികള്‍ ആരോപിക്കും വിധം പ്രവാചക ജീവിതം തന്നെ വ്യഭിചാരബന്ധങ്ങള്‍ നിറഞ്ഞതോ ലൈംഗികവൈകൃതങ്ങള്‍ പോറിയതോ ആയിരുന്നെങ്കില്‍ ആ യുവാവിനെ നബി എങ്ങനെയാണ് ഉപദേശിക്കുക. അയാള്‍ അത് എങ്ങനെയാണ് ഉള്‍കൊള്ളുക. എന്നെ നന്മയും സദാചാരവും ഉപദേശിക്കാന്‍ താങ്കള്‍ക്കെന്തര്‍ഹത എന്നയാള്‍ തിരിച്ചു ചോദിക്കുമായിരുന്നില്ലേ.? എന്തുകൊണ്ടയാള്‍ അതു ചോദിച്ചില്ല.! കാരണം അയാള്‍ നബിയെ പഠിച്ചതും അറിഞ്ഞതും അനുഭവങ്ങളില്‍ നിന്നും നേര്‍ക്കുനേരെയായിരുന്നു; അല്ലാതെ എതെങ്കിലും നവനാസ്തിക ഞരമ്പുരോഗികളില്‍ നിന്നോ മിഷണറി നുണഫാക്ടറികളില്‍ നിന്നോ ഷോവനിസ്റ്റ് ചാണക പുരകളില്‍ നിന്നോ ആയിരുന്നില്ല.

ഇത്രയും നാം പറഞ്ഞത്, ലൈംഗികവൈകൃതാനുരക്ത ഇസ്‌ലാമോഫോബിക്കുകള്‍ (Sexually Deviant Islamophobes) നബിജീവിതത്തിനെതിരെ ഉന്നയിച്ചുകൊണ്ടിരിക്കുന്ന ഏതൊരു ആരോപണത്തെയും പഠനവിധേയമാക്കുന്നതിനു മുമ്പ് ചിന്തയിലേക്ക് കൊണ്ടുവരേണ്ട ആമുഖ ചിന്താധാരകളെ പറ്റിയാണ്. കല്ലുവെച്ച കള്ളങ്ങള്‍ മാത്രമാണ് ഏതൊരു നബിനിന്ദാ പ്രമേയങ്ങളുമെന്ന ബോധ്യം പകര്‍ന്നു നല്‍കാന്‍ പര്യാപ്തമാണ് ആ വിശുദ്ധ ജീവിതത്തെപ്പറ്റിയുള്ള പ്രാഥമിക ചരിത്രപാഠങ്ങള്‍ പോലും. ഇനി നമുക്കു പരിശോധിക്കുവാനുള്ളത് ഉന്നയിക്കപ്പെട്ട ആരോപണങ്ങളുടെ നിജസ്ഥിതികളാണ്. അക്കമിട്ടു നമുക്കതു പരിശോദിക്കാം.

1, നബിയുടെ മാതൃസഹോദരി ഖൗല ബിന്‍ത് ഹകീം എന്ന സ്ത്രീ സ്വന്തം ശരീരം നബിക്കു ദാനം നല്‍കുകയുണ്ടായി എന്നും അതിനെതിരെ സ്വപത്‌നി ആഇശ പോലും നബിയോട് ‘ഒരു സ്വതന്ത്ര സ്ത്രീ സ്വദേഹം ദാനം ചെയ്യുകയോ’ എന്ന് പ്രതികരിച്ചുവെന്നും ഹദീഥ് ഗ്രന്ഥങ്ങളില്‍ കാണാം. എന്നാല്‍ നബി ഒരിക്കലും അതിനെ മോശമായി കണ്ടിരുന്നില്ല എന്നും ഹദീഥ് ഗ്രന്ഥങ്ങള്‍ തന്നെ മനസ്സിലാക്കി തരുന്നുണ്ട്. സ്ത്രീകളെ കൊണ്ട് സ്വശരീരങ്ങള്‍ തനിക്കായി ദാനം ചെയ്യാന്‍ പ്രേരിപ്പിക്കുകയും സ്വന്തം മാതൃസഹോദരിയെ പോലും അതില്‍ നിന്നും ഒഴിവാക്കാതിരിക്കുകയും ചെയ്ത പ്രവാചക നടപടിയെ അല്‍പമെങ്കിലും ധാര്‍മികബോധമുള്ളവര്‍ക്ക് അംഗീകരിക്കാനാകുമോ?

നാം നിരൂപണവിധേയമാക്കേണ്ട വിമര്‍ശനങ്ങള്‍ ഇവയാണ്:

a) എന്താണ് ശരീരദാനം കൊണ്ട് ഉദ്ദേശിക്കുന്നത്? വിമര്‍ശകന്മാര്‍ ജല്‍പ്പിക്കും പ്രകാരം സ്വന്തം ശരീരത്തെ ഒരു സ്ത്രീ ലൈംഗികാസ്വാദനത്തിനായി നബി(സ്വ)ക്ക് സമര്‍പ്പിക്കുന്ന ഏര്‍പ്പാടാണോ അത്. അല്ലേ അല്ല.!! നാസ്തിക/മിഷനറി/ഷോവനിസ്റ്റ് ഞരമ്പുരോഗികളുടെ ശിഫക്കായി (ശമനം) നമുക്കതു മന്ത്രിച്ചു കൊടുക്കാം.

വിശുദ്ധ ക്വുര്‍ആന്‍ പറഞ്ഞു: ”സത്യവിശ്വാസിയായ സ്ത്രീ സ്വന്തത്തെ പ്രവാചകന് ദാനം ചെയ്യുകയും അവളെ വിവാഹം കഴിക്കാനുദ്ദേശിക്കുകയുമാണെങ്കില്‍ അതിനും വിരോധമില്ല.” (ക്വുര്‍ആന്‍: 33: 50)

‘വിവാഹം കഴിക്കാനുദ്ദേശിക്കുകയുമാണെങ്കില്‍’ എന്നാണ് ക്വുര്‍ആന്‍ ഇവിടെ പരാമര്‍ശിച്ചിരിക്കുന്നത്. അഥവാ വിമര്‍ശകന്മാര്‍ ആരോപിക്കും വിധം ലൈംഗികാസ്വാദനത്തിനായി നബി(സ്വ)ക്ക് ഒരു സ്ത്രീ സ്വശരീരത്തെ സമര്‍പ്പിക്കുന്ന ഏര്‍പ്പാടല്ല ഇത്. മറിച്ച് വിവാഹത്തിനായി സ്വന്തത്തെ സമര്‍പിക്കലാണ്. അഥവാ സാധാരണ വിവാഹത്തില്‍ നിന്നും വ്യത്യാസമായി ഇവിടെ സ്ത്രീ മഹ്ര്‍ ഒഴിവാക്കും. ‘ഹിബത്ത്’ വിവാഹം എന്നാണ് ഇതറിയപ്പെടുന്നത്. ഇത് പ്രവാചകനു മാത്രം ബാധകമായ നിയമമാണ്. എല്ലാ പണ്ഡിതന്മാരും അതു വ്യക്തമാക്കിയിട്ടുണ്ട്.

”സത്യവിശ്വാസിനിയായ ഒരു സ്ത്രീ സ്വദേഹം നബിക്ക് ദാനം ചെയ്യുന്ന പക്ഷം നബി അവളെ വിവാഹം കഴിക്കാന്‍ ഉദ്ദേശിക്കുന്നെങ്കില്‍ അതും (അനുവദിച്ചിരിക്കുന്നു.)” അഥവാ മഹ്ര്‍ ഇല്ലാതെ വിവാഹം ചെയ്യാന്‍ ആവശ്യപ്പെട്ടാല്‍. ഹിബത്ത് (സ്വദേഹം ദാനം ചെയ്യുന്നു) എന്ന പദം ഉപയോഗിച്ച് മഹ്ര്‍ ഇല്ലാത്ത നിക്കാഹാണ് ഇവിടെ ഉദ്ദേശം.” (തഫ്‌സീറുല്‍ ജലാലൈനി: 33:50 ന്റെ വ്യാഖ്യാനം)

”ഇബ്‌നു അബ്ബാസ് (റ) പറഞ്ഞു: അല്ലാഹുവിന്റെ ദൂതന്റെ പത്‌നിമാരില്‍ സാധാരണ ചടങ്ങിലൂടെയൊ വലങ്കൈ ഉടമപ്പെടുത്തിയതോ അല്ലാതെ ആരും ഉണ്ടായിരുന്നില്ല. ഹിബത്ത് (സ്വദേഹം ദാനം ചെയ്യുന്നു) എന്ന പദം ഉപയോഗിച്ച് മഹ്ര്‍ ഇല്ലാത്ത നിക്കാഹിന് തയ്യാറായ ഒരു സ്ത്രീയേയും പ്രവാചകന്‍ (സ) സ്വീകരിക്കരിച്ചിട്ടില്ല. അവരില്‍ പെട്ട ആരും പ്രവാചകന് (സ) ഉണ്ടായിരുന്നില്ല.” (തഫ്‌സീറുല്‍ കുര്‍തുബി: 33:50 ന്റെ വ്യാഖ്യാനം)

”മഹ്ര്‍ കൂടാതെ, എന്നെ വിവാഹം ചെയ്യണോ എന്ന വിധി താങ്കള്‍ക്ക് ഞാന്‍ ഇഷ്ടദാനം ചെയ്യുന്നു.” (അല്‍ മുഫ്ഹിം: ഇമാം കുര്‍ത്തുബി: പേജ്: 4/128)

”ഹിബത്ത് (സ്വദേഹം ദാനം ചെയ്യുന്നു) എന്ന പദം ഉപയോഗിച്ച് മഹ്ര്‍ ഇല്ലാത്ത നിക്കാഹിന് തയ്യാറായ സ്ത്രീകളെ വിവാഹം ചെയ്യല്‍ പ്രവാചകന് അനുവദിക്കപ്പെട്ടിട്ടും പ്രവാചകന്‍ (സ) അതു ചെയ്തില്ല എന്നാണ് ഇബ്‌നു അബ്ബാസ് (റ) പറയുന്നത്.” (ഫത്ഹുല്‍ ബാരി: 8:526)

”ഞാന്‍ എന്നെ താങ്കള്‍ക്കു മുമ്പില്‍ വിവാഹത്തിനായി ഇഷ്ടദാനം ചെയ്യുന്നു’ എന്ന വാചകത്തില്‍ ഒരു മുള്വാഫ് (Possession) (ഭാഷാ പരമായ ഭംഗിക്കായി) വിട്ടുകളഞ്ഞതാണ്. യഥാർത്ഥത്തില്‍ വാചകത്തിന്റെ വിവക്ഷ ഇപ്രകാരമാണ്: ‘ഞാന്‍ എന്നെ അഥവാ എന്റെ വിവാഹ കാര്യത്തെ താങ്കള്‍ക്കു മുമ്പില്‍ ഇഷ്ടദാനം ചെയ്യുന്നു’. കാരണം ഒരു സ്വതന്ത്ര്യ സ്ത്രീ ഉടമപ്പെടുത്തപ്പെടുകയോ ദാനം ചെയ്യപ്പെടുകയോ ഇല്ലല്ലോ. മഹ്ര്‍ ഇല്ലാതെ തന്നെ താങ്കളെ വിവാഹം ചെയ്യാന്‍ ഞാന്‍ തയ്യാറാണ് എന്നാണ് ആ സ്ത്രീ പറഞ്ഞതിന്റെ വിവക്ഷ.” (ഫത്ഹുല്‍ ബാരി: 9/112, ഫത്ഹുല്‍ മുന്‍ഇം: 5/540)

നബി(സ്വ)ക്ക് ലൈംഗികാസ്വാദനത്തിനായി സ്ത്രീകള്‍ അവരുടെ ശരീരം ദാനം ചെയ്യുന്ന ഏര്‍പാടല്ല ‘ഹിബത്ത്’. മറിച്ച് മഹ്ര്‍ (വിവാഹ മൂല്യം) ഇല്ലാതെ തന്നെ അവരെ വിവാഹം ചെയ്യാനുള്ള അവകാശം പ്രവാചകനു സമര്‍പിച്ചുകൊണ്ട് നടത്തപ്പെടുന്ന വിവാഹ രീതിയാണത്. നബി(സ്വ)യുടെ പത്‌നിപദം ആഗ്രഹിച്ചുകൊണ്ടാണവര്‍ അപ്രകാരമുള്ള വിവാഹത്തിനു സന്നദ്ധത അറിയിക്കുന്നത്. എന്നാല്‍ അത്തരത്തിലുള്ള ‘ഹിബത്ത്’ വിവാഹം നബി (സ്വ) ഒരിക്കലും ചെയ്തിട്ടില്ലെന്നാണ് പ്രവാചക ശിഷ്യനും സന്നദ്ധസഹചാരിയുമായ ഇബ്‌നു അബ്ബാസ് (റ) പറയുന്നത്. എങ്കില്‍ എന്തുകൊണ്ടാണ് പ്രവാചക പത്‌നി ആഇശ (റ) ‘ഒരു സ്വതന്ത്ര സ്ത്രീ സ്വദേഹം ദാനം ചെയ്യുകയോ’?! എന്ന് പ്രവാചകനോട് പ്രതികരിച്ചത്? ഒരു സ്വതന്ത്ര സ്ത്രീയുടെ കുലീനതക്കു യോജിച്ച കാര്യമായി അവര്‍ക്കതിനെ ഉള്‍കൊള്ളാന്‍ പ്രയാസം തോന്നിയതുകൊണ്ടു മാത്രമാണ് അവരപ്രകാരം നബി(സ്വ)യോടു പ്രതികരിച്ചത്. കാരണം മഹ്ര്‍ എന്നത് സ്ത്രീയുടെ കുലീനതയുടെ ഭാഗമായാണ് അന്നും ഇന്നും അറേബ്യന്‍ സ്ത്രീകള്‍ കണക്കാക്കി പോരുന്നത്. അതിനാല്‍ സ്വാഭാവികമായും ഉണ്ടായ ഒരാശ്ചര്യം പ്രകടിപ്പിക്കുക മാത്രമായിരുന്നു അവര്‍. എന്നാല്‍ നബി (സ്വ) അത് മോശമായി കാണാതിരുന്നത് ‘ഹിബത്ത്’ വിവാഹത്തിന് സന്നദ്ധരായ സ്ത്രീകള്‍ അതുകൊണ്ടാഗ്രഹിച്ചത് പ്രവാചകന്റെ പത്‌നിപദം മാത്രമാണെന്ന് അവിടുത്തേക്ക് അറിയാവുന്നതു കൊണ്ടായിരുന്നു. ആദര്‍ശ പ്രണയമാണ് ശരീര പ്രണയമല്ല അതിനു പിറകിലുള്ള ചേദോവികാരമെന്ന് പ്രവാചകന് അറിയാമായിരുന്നു.

b) ‘നബിയുടെ മാതൃസഹോദരി ഖൗല ബിന്‍ത് ഹകീം എന്ന സ്ത്രീ സ്വന്തം ശരീരം നബിക്കു ദാനം നല്‍കുകയുണ്ടായി’. വിമര്‍ശനമിതാണ്; സ്വന്തം മാതൃസഹോദരിയെ കൊണ്ടു പോലും ശരീരം ദാനം (ഹിബത്ത്) ചെയ്യപ്പിച്ചു പ്രവാചകന്‍.!! ആരായിരുന്നു വാസ്തവത്തില്‍ ഖൗല ബിന്‍ത് ഹകീം എന്ന സ്ത്രീ. നബിയുടെ മാതൃസഹോദരിയായിരുന്നോ?. അല്ല. മറിച്ച് അവര്‍ പ്രസിദ്ധ താബിഈയായ (പ്രവാചകാനുചരന്മാരുടെ ശിഷ്യന്‍) സഈദിബ്‌നുല്‍ മുസ്വയ്യിബി(റ)ന്റെ മാതൃസഹോദരിയായിരുന്നു. പിന്നെ എന്തുകൊണ്ടാണവര്‍ നബിയുടെ മാതൃസഹോദരിയാണെന്ന തെറ്റിദ്ധാരണയുണ്ടായി?. ആദ്യം നാം പരിശോധിക്കുന്നത് അവര്‍ സഈദിബ്‌നുല്‍ മുസ്വയ്യിബി(റ)ന്റെ മാതൃസഹോദരിയായിരുന്നു എന്ന വസ്തുതയാണ്. അതോടൊപ്പം, എന്തുകൊണ്ടാണവര്‍ നബിയുടെ മാതൃസഹോദരിയാണെന്ന തെറ്റിദ്ധാരണയുണ്ടായതെന്നും പരിശോധിക്കാം.

”അബുല്‍ വലീദ് അത്ത്വയാലിസി പറഞ്ഞു: ശുഅ്ബ നമ്മോട് പറഞ്ഞു: അതാഅ് അല്‍ ഖുറാസാനിയില്‍ നിന്ന്, അദ്ദേഹം പറഞ്ഞു: സഈദിബ്‌നുല്‍ മുസ്വയ്യിബ് പറയുന്നതായി ഞാന്‍ കേട്ടു: ‘എന്റെ മാതൃസഹോദരി’ ഖൗല ബിന്‍ത്ത് ഹകീം അസ്സുലമിയ്യ അല്ലാഹുവിന്റെ തിരുദൂതരോട് സ്ത്രീകളുടെ സ്‌കലനത്തെ സംബന്ധിച്ച് ചോദിച്ചു. അപ്പോള്‍ അത് സംഭവിച്ചാല്‍ കുളിക്കുവാന്‍ പ്രവാചകന്‍ കല്‍പ്പന നല്‍കി.” (സുനനു ദാരിമി: 762)

എന്നാല്‍ ഇതേ ഹദീഥിന്റെ ചില നിവേദനങ്ങളില്‍ പ്രവാചകന്റെ മാതൃസഹോദരിയാണ് ഖൗല ബിന്‍ത്ത് ഹകീം എന്ന് വന്നത് ഹദീസ് നിവേദകന് സംഭവിച്ച ഓര്‍മ പിശക് മാത്രമാണ്. നിവേദക പരമ്പരയിലെ ശുഅ്ബ എന്ന നിവേദകനാണ് ഈ ഓര്‍മ പിശക് സംഭവിച്ചത് എന്നും ചില പണ്ഡിതര്‍ അഭിപ്രായപ്പെടുന്നു. (https://al-maktaba.org/book/31621/19145#p35)

നിവേദക പരമ്പരയിലെ ‘അതാഅ് അല്‍ ഖുറാസാനി’ യില്‍ നിന്ന് പലപോഴും ഹദീഥിന്റെ പദപ്രയോഗങ്ങളില്‍ വഹ്‌മ് അഥവാ ധാരണ പിശക് സംഭവിക്കാറുണ്ട്. (സിയറു അഅ്‌ലാമിന്നുബലാഅ്: 6/140, തക്‌രീബു തഹ്ദീബ്: 1/392, തഹ്ദീബുല്‍ കമാല്‍: 20/107)

അത്തരത്തില്‍ സംഭവിച്ച ധാരണ പിശകാണ്, സഈദിബ്‌നുല്‍ മുസ്വയ്യിബ്, തന്റെ മാതൃസഹോദരിയാണ് ഖൗല ബിന്‍ത്ത് ഹകീം എന്ന് പറഞ്ഞത് പ്രവാചകന്റെ മാതൃസഹോദരിയാണ് ഖൗല ബിന്‍ത്ത് ഹകീം എന്ന് തെറ്റായി മനസ്സിലാക്കിയത്. ഇസ്ഹാകിബ്‌നു റാഹൂയ പറഞ്ഞു: ഖൗല ബിന്‍ത്ത് ഹകീം വഴി അല്ലാഹുവിന്റെ ദൂതനില്‍ നിന്ന് ഉദ്ധരിക്കപ്പെട്ട ഹദീഥ്. അബൂ യഅ്കൂബ് പറഞ്ഞു: ഖൗല ബിന്‍ത്ത് ഹകീം, സഈദിബ്‌നു മുസ്വയ്യിബിന്റെ മാതൃസഹോദരിമാരില്‍ ഒരാളാണ്. (മുസ്‌നദു ഇസ്ഹാകിബ്‌നു റാഹൂയ: 5:44: ഹദീസ് നമ്പര്‍: 2147)

ഇനി നാം പരിശോധിക്കുന്നത് നബി(സ്വ)യുടെ മാതാവ് ആമിനയുടേയും ഖൗല ബിന്‍ത്ത് ഹകീമിന്റേയും കുടുംബ പരമ്പരകളാണ്. അപ്പോള്‍ വ്യക്തമാകുന്ന കാര്യമാണ് ഖൗല ബിന്‍ത്ത് ഹകീം നബി(സ്വ)യുടെ മാതൃസഹോദരിയല്ലെന്ന വസ്തുത.

നബി(സ്വ)യുടെ മാതാവ് ആമിനയുടെ കുടുംബ പരമ്പര കാണുക:

സഹ്‌റയുടെ പുത്രന്‍ അബ്ദു മനാഫിന്റെ പുത്രന്‍ വഹബിന്റെ പുത്രി ആമിന. അബ്ദു യഗൂസിബ്‌നു വഹബ് എന്ന ഒരു മാതൃസഹോദരന്‍ മാത്രമേ പ്രവാചകന് ഉണ്ടായിരുന്നുള്ളു. (അസ്സികാത്ത്: ഇബ്‌നു ഹിബ്ബാന്‍: 1/26)

ഖൗല ബിന്‍ത്ത് ഹകീമിന്റെ കുടുംബ പരമ്പര കാണുക: സുലൈമിന്റെ പുത്രന്‍ ബഹ്തയുടെ പുത്രന്‍ ഇംറുല്‍ കൈസിന്റെ പുത്രന്‍ ദക്വാന്റെ പുത്രന്‍ സഅ്‌ലബയുടെ പുത്രന്‍ ഫാലിജിന്റെ പുത്രന്‍ ഹിലാലിന്റെ പുത്രന്‍ മുര്‍റയുടെ പുത്രന്‍ അവ്കസിന്റെ പുത്രന്‍ ഉമയ്യയുടെ പുത്രന്‍ ഹകീമിന്റെ പുത്രി ഖൗല. (ത്വബകാത്തു ഇബ്‌നു സഅ്ദ്: 8/158)

ഇനി നബിയുടെ മാതാവ് ആമിനയുടെയും സഈദിബ്‌നുല്‍ മുസ്വയ്യിബി(റ)ന്റെ മാതൃസഹോദരിയായ ഖൗല ബിന്‍ത്ത് ഹകീം അസ്സുലമിയ്യയുടെയും മാതാക്കളുടെ പരമ്പരയും കൂടി കാണുക:

“ആമിനയുടെ മാതാവ്, 'ബർറ' ബിൻത്ത് അബ്ദുൽ ഉസ്സായാണ്; കിലാബിന്റെ മകൻ കുസ്വയ്യിന്റെ മകൻ അബ്ദുദ്ദാറിന്റെ മകൻ ഉസ്മാന്റെ മകൻ അബ്ദുൽ ഉസ്സായുടെ മകൾ ബർറ. (ത്വബകാതുൽ കുബ്റാ: ഇബ്നു സഅ്ദ്: 1:59, സീറത്തു ഇബ്നു ഹിശാം: 1:110)

ഖൗലയുടെ മാതാവ് 'ദഈഫ'യാണ്. അബ്ദുശ്ശംസിന്റെ മകൻ ഉമയ്യയുടെ മകൻ ആസിന്റെ മകൾ ദഈഫ. (ത്വബകാതുൽ കുബ്റാ: ഇബ്നു സഅ്ദ്: 8:158, സിയറു അഅ്ലാമിന്നുബലാഅ്: 1:164)

ഇനി സാക്ഷാല്‍ ഖൗല ബിന്‍ത്ത് ഹകീമില്‍ നിന്ന് ഹദീഥ് ഉദ്ധരിച്ച സഈദിബ്‌നുല്‍ മുസ്വയ്യിബിന്റെ മാതാവ് ആരാണെന്ന് കാണുക:

അവ്കസിന്റെ പുത്രന്‍ ഉമയ്യയുടെ പുത്രന്‍ ഹകീമിന്റെ പുത്രി ഉമ്മു സഈദ്. (ത്വബകാത്തു ഇബ്‌നു സഅ്ദ്: 5/119)

സഈദിബ്‌നുല്‍ മുസ്വയ്യിബിന്റെ മാതാവും ഖൗല ബിന്‍ത്ത് ഹകീമും ഉപ്പയും ഉമ്മയുമൊത്ത സഹോദരിമാരാണ്. അല്ലാതെ നബി(സ്വ)യുടെ മാതൃസഹോദരിയല്ല ഖൗല ബിന്‍ത്ത് ഹകീം എന്നര്‍ഥം.

നബി(സ്വ)യോട് ‘ഹിബത്ത്’ വിവാഹത്തിന് താല്‍പര്യമറിയിച്ച ഖൗല ബിന്‍ത്ത് ഹകീമിന്റെ പിതാവിന്റെ പേര് ഹകീം എന്നാണെങ്കില്‍ നബി(സ്വ)യുടെ മാതാവ് ആമിനയുടെ പിതാവിന്റെ പേര് വഹബ് എന്നാണ്. അഥവാ ഖൗല ബിന്‍ത്ത് ഹകീം ആമിന ബിന്‍ത്ത് വഹബിന്റെ സഹോദരിയല്ല. അതുകൊണ്ടു തന്നെ നബിയുടെ മാതുലയുമല്ല. ഇസ്‌ലാമിക ചരിത്ര രേഖകളില്‍ വളരെ വ്യക്തമായി ഇതു രേഖപ്പെടുത്തപ്പെട്ടിട്ടും പ്രവാചകനു മേല്‍ ബന്ധുരതി (Incest) ആരോപിക്കുവാന്‍ പരിശ്രമിക്കുന്നവരില്‍ നിന്നും ഒരു നന്മയും നാം പ്രതീക്ഷിക്കേണ്ടതില്ല. ഹദീഥുകളെയും ഇസ്‌ലാമിക ചരിത്രത്തെയും പച്ചയായി ദുര്‍വ്യാഖ്യാനിച്ച് പ്രവാചകനെ തെറിയഭിഷേകം ചെയ്യുന്നവര്‍, ക്രൂരമായ മതനിന്ദയാണോ നിര്‍വഹിക്കുന്നത് അതല്ല വിമര്‍ശന സ്വാതന്ത്ര്യം ഉപയോഗപ്പെടുത്തുകയാണോ ചെയ്യുന്നതെന്ന് മാന്യതയുള്ളവര്‍ മൂന്നുവട്ടം ആലോചിക്കട്ടെ. ആവിഷ്‌കാര സ്വാതന്ത്ര്യമെന്നത് തെറിവിളിക്കാനുള്ള സ്വാതന്ത്ര്യമായി പരിണമിച്ചാല്‍ പരിക്കേല്‍ക്കുന്നത് നാം പടുത്തുയര്‍ത്തിയ നമ്മുടെ ബഹുസ്വര സംസ്‌കാരത്തിനു മേലായിരിക്കും. അതുതന്നെയാണ് ഈ മനോരോഗികളുടെ ലക്ഷ്യമെന്ന് വിവരമുള്ളവരെല്ലാം തിരിച്ചറിയേണ്ടിയിരിക്കുന്നു. സ്വന്തം മാതുലയെ തനിക്കു വേണ്ടി ശരീരം ദാനം ചെയ്യാന്‍ പ്രേരിപ്പിച്ച വ്യക്തിയായി പ്രവാചകനെ ചിത്രീകരിക്കാന്‍ പരിശ്രമിച്ചവര്‍ വായിക്കേണ്ട ഒരു ഹദീഥ് ഇവിടെ കുറിക്കട്ടെ.

” അബൂസുഫ്‌യാന്റെ പുത്രി ഉമ്മുഹബീബ പറയുന്നു: ഞാന്‍ പറഞ്ഞു: ‘അല്ലാഹുവിന്റെ ദൂതരേ, അബൂസുഫ്‌യാന്റെ പുത്രിയായ എന്റെ സഹോദരിയെക്കൂടി അങ്ങ് വിവാഹം കഴിച്ചാലും’. നബി ചോദിച്ചു: ‘നിനക്ക് അതിഷ്ടമാണോ?’ ഞാന്‍ പറഞ്ഞു: ‘അതേ. ഞാന്‍ അങ്ങയുടെ ഏക പത്‌നിയല്ല. നന്മകളില്‍ എന്നോടൊപ്പം പങ്കുകൊള്ളുന്നത് എന്റെ സഹോദരി ആയിരിക്കുന്നതാണ് എനിക്കേറ്റവും ഇഷ്ടം’. നബി പറഞ്ഞു: ‘അത് എനിക്ക് അനുവദനീയമല്ല’……’നിങ്ങളുടെ പെണ്‍കുട്ടികളെയും സഹോദരിമാരെയും വിവാഹം കഴിക്കാന്‍ നിങ്ങള്‍ എന്നോടാവശ്യപ്പെടരുത്.” (സ്വഹീഹുല്‍ ബുഖാരി: 1755) ഇനി അടുത്ത ആരോപണത്തിന്റെ നിജസ്ഥിതികൂടി നമുക്കു പരിശോദിക്കാം.

2, ‘ഉമ്മു ഹറാം, ഉമ്മു സുലൈം എന്നീ സ്ത്രീകളെ നബി സ്വകാര്യ സന്ദര്‍ശനം നടത്താറുണ്ടായിരുന്നെന്നും അവരുടെ മടിയില്‍ പലപ്പോഴും തലവെച്ചു കിടക്കാറുണ്ടായിരുന്നു എന്നും ഹദീഥ് ഗ്രന്ഥങ്ങള്‍ തന്നെ വ്യക്തമാക്കുന്നുണ്ട്. അവിഹിത ബന്ധങ്ങള്‍ക്ക് പ്രവാചക ജീവിതത്തിലിടമുണ്ടായിരുന്നു എന്നു വ്യക്തമാക്കുന്ന സംഭവങ്ങളാണിതെല്ലാം.’

ആരായിരുന്നു ഉമ്മു ഹറാമും, ഉമ്മു സുലൈമും.? സ്വകാര്യ സന്ദര്‍ശനം നടത്താനും മടിയില്‍ തലവെച്ചു കിടക്കാനും മാത്രം എന്തു ബന്ധമായിരുന്നു പ്രവാചകന് അവരോടുണ്ടായിരുന്നത്.? നമുക്കു പരിശോധിക്കാം.

ഉമ്മു സുലൈം, യഥാർത്ഥ നാമം സഹ്‌ല. ജുന്‍ദുബിന്റെ പുത്രന്‍ ഹറാമിന്റെ പുത്രന്‍ സൈദിന്റെ പുത്രന്‍ ഖാലിദിന്റെ പുത്രന്‍ മല്‍ഹാന്റെ പുത്രിയാണ് ഉമ്മു സുലൈം. അനസ് ഇബ്‌നു മാലികിന്റെ മാതാവ്. (അല്‍ ഇസ്വാബ: 8/227)

ഉമ്മു ഹറാം ബിന്‍ത്ത് മല്‍ഹാന്‍, ഉമ്മു സുലൈമിന്റെ സഹോദരി, അനസ് ഇബ്‌നു മാലികിന്റെ മാതൃസഹോദരി. (തഹ്ദീബുല്‍ കമാല്‍: 35/338: നമ്പര്‍: 7962)

ഇമാം നവവി പറഞ്ഞു: അവര്‍ പ്രവാചകന് വിവാഹം നിഷിദ്ധമായ കുടുംബക്കാരായിരുന്നു എന്നതില്‍ പണ്ഡിതന്മാര്‍ക്കിടയില്‍ ഏകാഭിപ്രായമുണ്ട്. എന്നാല്‍ ഈ കുടുംബ ബന്ധം ഏത് രൂപത്തിലാണ് എന്നതില്‍ മാത്രമാണ് അഭിപ്രായ വ്യത്യാസം. ഇബ്‌നു അബ്ദുല്‍ ബിര്‍റും മറ്റു പണ്ഡിതന്മാരും പറഞ്ഞത് പ്രവാചകന്റെ മുലകുടി ബന്ധത്തിലെ മാതൃസഹോദരിയാണ് അവര്‍ എന്നാണ്. മറ്റു പല പണ്ഡിതന്മാരും അഭിപ്രായപ്പെടുന്നത്, അവര്‍ പ്രവാചകന്റെ പിതാവിന്റെയൊ പിതാമഹന്റെയൊ മാതൃസഹോദരിയാണ് എന്നാണ്. കാരണം പ്രവാചകന്റെ പിതാമഹന്‍ അബ്ദുല്‍മുത്വലിബിന്റെ മാതാവ് ബനൂനജ്ജാര്‍ ഗോത്രത്തില്‍ നിന്നാണ്. (ഉമ്മു ഹറാം, ഉമ്മു സുലൈം എന്നിവരും ബനൂനജ്ജാറുകാരാണ്). (ശര്‍ഹുന്നവവി അലാ മുസ്ലിം: 13/50, ഔനുല്‍ മഅ്ബൂദ്: 7/122, സിറാജുല്‍ വഹ്ഹാജ്: 4/332, ഫത്ഹുല്‍ ബാരി: 11/78, തന്‍വീറുല്‍ ഹവാലിക്: 1/309, ശര്‍ഹു സുയൂതി: 4/505, ഇര്‍ശാദുസ്സാരി: 5/36, നള്‌റത്തു നഈം: 1/549, മിര്‍കാത്തുല്‍ മഫാത്തീഹ്: 9/3478)

താബിഉകളിലെ (പ്രവാചകാനുചരന്മാരുടെ ശിഷ്യര്‍) ഇളതലമുറക്കാരുമായി സഹവസിച്ചിട്ടുള്ള ഇബ്‌നു വഹബ് പറയുന്നു: ഉമ്മു ഹറാം പ്രവാചകന്റെ മുല കുടിബന്ധത്തിലെ മാതൃസഹോദരിയാണ്. (ഫത്ഹുല്‍ ബാരി: 11/78)

നബി(സ്വ)യുടെ മാതൃസഹോദരിമാരാണ് ഉമ്മു ഹറാമും, ഉമ്മു സുലൈമും. അവരെ മകനായ പ്രവാചകന്‍ സന്ദര്‍ശിച്ചതിനെയും ആ മാതാക്കളുടെ മടിയില്‍ തലവെച്ചു കിടന്നതിനെയുമാണ് നബിവൈരികള്‍ ലൈംഗിക ഛായം പൂശിയവതരിപ്പിച്ചത്. എത്രമാത്രം നികൃഷ്ഠമാണ് നബിവിമര്‍ശകന്മാരുടെ അന്തരംഗം. മാതൃ-പുതൃ ബന്ധങ്ങളില്‍ പോലും അശ്ലീലതകള്‍ തിരയുന്നവരെ ഏതു മുക്കാലിയില്‍ കെട്ടിയിട്ടാണാവോ അടിക്കേണ്ടത്. മൃഗരതിയും ശവരതിയും സ്വവര്‍ഗരതിയും ബന്ധുരതിയും ശിശുരതിയുമെല്ലാം അവകാശവും സ്വാതന്ത്ര്യവുമായി ഉദ്‌ഘോഷിക്കുന്ന, അതിനു വേണ്ടിയുള്ള പോരാട്ടങ്ങളെ ആക്ടിവിസമായി നെറ്റിയിലൊട്ടിച്ചു നടക്കുന്ന നവനാസ്തികരില്‍ നിന്നും ഇത്രയൊന്നും പ്രതീക്ഷിച്ചാല്‍ പോരാ. ഇതിലും വലിയ വൃത്തികേടുകള്‍ക്ക് അടയിരിക്കുന്നവരാണവര്‍. വിശുദ്ധരും സ്വാത്തികരുമായവര്‍ക്കെതിരെ ഇനിയുമെന്തെല്ലാം വൃത്തികേടുകള്‍ അവരില്‍ നിന്നും കേള്‍ക്കാനിരിക്കുന്നു. കാരണം അവര്‍ക്ക് വെറുപ്പ് വെളിച്ചത്തിലേക്ക് വിളിച്ചവരോടാണ്. വിശുദ്ധിയിലേക്ക് നയിച്ചവരോടാണ്. മാനവികതക്ക് ജീവിത മൂല്ല്യങ്ങളിലേക്ക് വഴിവെട്ടിയവരോടാണ്. പക്ഷെ മോക്ഷത്തെപ്പറ്റിയും വിശുദ്ധിയെപ്പറ്റിയും മൂല്ല്യങ്ങളെപ്പറ്റിയും നിരന്തരം ഗിരിപ്രഭാഷണങ്ങളും സുവിശേഷഭാഷണങ്ങളും നടത്തുന്ന ഹിന്ദുത്വ വാദികളും മിഷനറിമാരും, ഇസ്‌ലാമിനും പ്രവാചകനുമെതിരെയാണെങ്കില്‍ ഏതു നവനാസ്തുക ഞരമ്പുരോഗിയേയും പേറുമെന്നാണെങ്കില്‍ വിസ്മരിക്കുരുത്; നാറിയവനെ പേറിയാല്‍ പേറിയവനും നാറുമെന്ന യാഥാര്‍ഥ്യത്തെ.

വിമർശനം:

(വിമർശകർ ഹദീസിന് നൽകുന്ന പരിഭാഷ അതേപടി ഇവിടെ ചേർക്കുന്നു:)

മുഹമ്മദ് നബിയുടെ മദ്യപാനത്തെ കുറിച്ച് ഗാബെർബിൻ അബ്ദുള്ള റിപ്പോർട്ട് ചെയ്തത്: അല്ലാഹുവിൻറെ ദൂതന് സമാധാനം ഉണ്ടാവട്ടെ. കൂടെ ആയിരുന്നപ്പോൾ, നബി അല്പം മദ്യം ആവശ്യപ്പെട്ടു. ഇതുകേട്ട് അനുചരന്മാരിൽ ഒരാൾ ചോദിച്ചു: അല്ലയോ അല്ലാഹുവിന്റെ ദൂതരെ ഞങ്ങൾ അങ്ങേയ്ക്ക് മദ്യം നൽകാൻ പാടുണ്ടോ..? തിരുമേനി: പറഞ്ഞു ഉവ്വ്. അയാൾ മദ്യം അന്വേഷിച്ചു പുറത്തേക്ക് പോയി ഒരു കോപ്പ മദ്യവുമായി തിരികെ വന്നു. മുഹമ്മദ് നബി പറഞ്ഞു: നീ മരച്ചില്ല കുത്തനെ പിടിച്ചു കൊണ്ട് (മദ്യം) മറച്ചിരുന്നോ, അവൻ (ഗാബെർ) പറഞ്ഞു ഉവ്വ്. അപ്പോൾ നബി അതു പാനം ചെയ്തു. sahih muslim,Hadith-3753. മറുപടി:

ഹദീസ് സ്വഹീഹ് ആണെന്നതിൽ സംശയമില്ല. പക്ഷെ ഹദീസിന്റെ വിവർത്തനത്തിൽ ധാരാളം കൃത്രിമം നടത്തി കൊണ്ടാണ് പ്രവാചകൻ (സ) മദ്യപിച്ചിരുന്നു എന്ന കള്ള പ്രചാരണത്തിന് ദുർവ്യാഖ്യാനക്കാർ ‘തെളിവ്’ മെനഞ്ഞെടുത്തിരിക്കുന്നത്.

ഹദീസും ഹദീസിനോട് പരമാവധി നീതി പുലർത്തിക്കൊണ്ടുള്ള വിവർത്തനവും ശ്രദ്ധിക്കുക:

ﻋﻦ ﺟﺎﺑﺮ ﺑﻦ ﻋﺒﺪ اﻟﻠﻪ، ﻗﺎﻝ: ﻛﻨﺎ ﻣﻊ ﺭﺳﻮﻝ اﻟﻠﻪ ﺻﻠﻰ اﻟﻠﻪ ﻋﻠﻴﻪ ﻭﺳﻠﻢ ﻓﺎﺳﺘﺴﻘﻰ، ﻓﻘﺎﻝ ﺭﺟﻞ: ﻳﺎ ﺭﺳﻮﻝ اﻟﻠﻪ ﺻﻠﻰ اﻟﻠﻪ ﻋﻠﻴﻪ ﻭﺳﻠﻢ، ﺃﻻ ﻧﺴﻘﻴﻚ ﻧﺒﻴﺬا؟ ﻓﻘﺎﻝ: «ﺑﻠﻰ»، ﻗﺎﻝ: ﻓﺨﺮﺝ اﻟﺮﺟﻞ ﻳﺴﻌﻰ، ﻓﺠﺎء ﺑﻘﺪﺡ ﻓﻴﻪ ﻧﺒﻴﺬ، ﻓﻘﺎﻝ ﺭﺳﻮﻝ اﻟﻠﻪ ﺻﻠﻰ اﻟﻠﻪ ﻋﻠﻴﻪ ﻭﺳﻠﻢ: «ﺃﻻ ﺧﻤﺮﺗﻪ ﻭﻟﻮ ﺗﻌﺮﺽ ﻋﻠﻴﻪ ﻋﻮﺩا»، ﻗﺎﻝ: ﻓﺸﺮﺏ

ജാബിർ ഇബ്നു അബ്ദുല്ല (റ) പറഞ്ഞു: (ഒരിക്കൽ) ഞങ്ങൾ അല്ലാഹുവിന്റെ ദൂതന്റെ (സ) കൂടെയായിരുന്നപ്പോൾ അദ്ദേഹം വെള്ളം ആവശ്യപ്പെട്ടു. അപ്പോൾ ഒരാൾ ചോദിച്ചു: അല്ലാഹുവിന്റെ ദൂതരേ, താങ്കൾക്ക് ഞങ്ങൾ നബീദ് (ഈത്തപ്പഴച്ചാറ്) കുടിക്കാൻ തരട്ടെ ? അദ്ദേഹം അതെ എന്ന് പറഞ്ഞു. അങ്ങനെ അയാൾ പുറപ്പെടുകയും ഒരു പാത്രത്തിൽ ഈത്തപ്പഴച്ചാറ് കൊണ്ട് വരികയും ചെയ്തു. അപ്പോൾ അല്ലാഹുവിന്റെ ദൂതൻ (സ) ചോദിച്ചു: ഒരു മരക്കമ്പ് വെച്ചു കൊണ്ടെങ്കിലും നീ പാത്രം മൂടിയിരുന്നില്ലേ ? (അയാൾ അതെ എന്ന് അറിയിച്ചപ്പോൾ) പ്രവാചകൻ (സ) അത് കുടിച്ചു. (സ്വഹീഹു മുസ്‌ലിം: 2011)

വിമർശകരുടെ വിവർത്തനവും ഹദീസിന്റെ ശരിയായ വിവർത്തനവും തമ്മിലുള്ള അന്തരം നോക്കൂ !! എത്രമാത്രം കൃത്രിമങ്ങളും ദുർവ്യാഖ്യാനങ്ങളുമാണ് വിമർശകർ പടച്ചുണ്ടാക്കിയ ‘മദ്യപാനിയായ മുഹമ്മദ്’ എന്ന നാടകത്തിന്റെ ‘തിരകഥയിൽ’ തിരുകി കയറ്റിയിരിക്കുന്നത് !!! അവ ഓരോന്നും അക്കമിട്ട് വിശദീകരിക്കാം.

1. അല്ലാഹുവിന്റെ ദൂതന്റെ(സ) “വെള്ളം ആവശ്യപ്പെട്ടു” (ﻓﺎﺳﺘﺴﻘﻰ) എന്നതിന് പകരം “നബി അല്പം മദ്യം ആവശ്യപ്പെട്ടു.” എന്നതാണ് ആദ്യത്തെ ‘കള്ളകടത്ത്’. ഹദീസിലെ ‘ഇസ്തസ്ക്വാ’ (اﺳﺘﺴﻘﻰ) എന്ന പദം കുർആനിൽ പോലും വന്നിട്ടുള്ള ഒന്നാണ്:

وَإِذِ ٱسۡتَسۡقَىٰ مُوسَىٰ لِقَوۡمِهِۦ فَقُلۡنَا ٱضۡرِب بِّعَصَاكَ ٱلۡحَجَرَۖ فَٱنفَجَرَتۡ مِنۡهُ ٱثۡنَتَا عَشۡرَةَ عَیۡنࣰاۖ … “മൂസാ നബി തന്റെ ജനതയ്ക്കുവേണ്ടി വെള്ളത്തിനപേക്ഷിച്ച സന്ദര്‍ഭവും (ശ്രദ്ധിക്കുക.) അപ്പോള്‍ നാം പറഞ്ഞു: നിന്റെ വടികൊണ്ട് പാറമേല്‍ അടിക്കുക. അങ്ങനെ അതില്‍ നിന്ന് പന്ത്രണ്ട് ഉറവുകള്‍ പൊട്ടി ഒഴുകി… ” (കുർആൻ: 2:60)

‘ഇസ്തസ്ക്വാ’ (ٱسۡتَسۡقَىٰ) അഥവാ വെള്ളത്തിനപേക്ഷിച്ചു എന്ന പദത്തിന് അൽപ്പം മദ്യം ആവശ്യപ്പെട്ടു എന്ന ഇല്ലാത്ത അർത്ഥം വിമർശകർ സൃഷ്ടിച്ചത് ഹദീസിന്റെ ഉള്ളടക്കം മൊത്തം അപനിർമിക്കാൻ വേണ്ടിയാണ്.

2. വെള്ളം ചോദിച്ച നബിയോട് “താങ്കൾക്ക് ഞങ്ങൾ നബീദ് (ഈത്തപ്പഴച്ചാറ്) കുടിക്കാൻ തരട്ടെ ?” (ﺃﻻ ﻧﺴﻘﻴﻚ ﻧﺒﻴﺬا؟) എന്ന് അനുചരന്മാരിൽ ഒരാൾ പ്രതികരിച്ച സ്വഭാവിക പ്രതികരണത്തെയാണ് രണ്ടാമതായി ദുർവ്യാഖ്യാന വീരന്മാർ അർത്ഥം മാറ്റിയിരിക്കുന്നത്. “താങ്കൾക്ക് ഞങ്ങൾ നബീദ് (ഈത്തപ്പഴച്ചാറ്) കുടിക്കാൻ തരട്ടെ ?” എന്ന വാചകത്തെ വളച്ചൊടിച്ച് “അല്ലയോ അല്ലാഹുവിന്റെ ദൂതരെ ഞങ്ങൾ അങ്ങേയ്ക്ക് മദ്യം നൽകാൻ പാടുണ്ടോ..?” എന്ന ഇല്ലാത്ത അർത്ഥം നൽകി !! ഈ ‘പാടുണ്ടോ’ എന്ന വിവർത്തനമൊക്കെ ‘അലാ നുസ്കീക നബീദാ” എന്ന അറബി വാചകത്തിലെ ഏത് പദത്തിന്റെ പരിഭാഷയാണെന്ന് വിമർശകർക്ക് പറയാനൊക്കുമോ ?!! യാതൊരു പുലബന്ധവുമില്ലാത്ത തോന്യാസങ്ങൾ ഹദീസിന്റെ ഇംഗ്ലീഷ് പരിഭാഷകളിൽ കള്ളക്കടത്തു നടത്തുന്ന മിഷണറി അപനിർമാണ സൈറ്റുകളിലും, ഹദീസ് വിരുദ്ധ ശിഈ സൈറ്റുകളിലും ഓടുന്ന ഹദീസുകളുടെ നിഴലുകൾ മാത്രമാണ് ഇത്തരം വിവർത്തനങ്ങൾ. യഥാർത്ഥ ഹദീസുകളിൽ മാറ്റം വരുത്തി നിർമ്മിക്കപ്പെടുന്ന ഫേക്ക് ഹദീസുകൾ.

3. അടുത്ത കൃത്രിമം ‘നബീദ്’ (النَّبِيذُ) എന്ന പദത്തിൻമേലാണ്. പ്രവാചകനോട് കുടിക്കാൻ തരട്ടെയോ? എന്ന് അനുചരന്മാർ ചോദിച്ചതും പ്രവാചകൻ (സ) കുടിച്ചതുമായി ഹദീസിൽ പ്രസ്ഥാവിക്കുന്നത് നബീദ് എന്ന മധുര പാനീയമാണ്. ആ പദത്തിന് മദ്യമെന്ന് വിവർത്തനം നൽകിയുള്ള ഹദീസ് തിരിമറികൾ ഒരുപാട് നാളായി നടന്നു വരുന്നു. മദ്യത്തിന്റെ കാര്യത്തിൽ പത്തുപേരെ/ കാര്യങ്ങളെ അല്ലാഹുവിന്റെ ദൂതൻ (സ) ശപിച്ചിട്ടുണ്ട്. മദ്യത്തെയും, അത് കുടിക്കുന്നവനേയും കുടിപ്പിക്കുന്നവനേയും വിൽക്കുന്നവനേയും ആർക്കുവേണ്ടി വിൽക്കപ്പെടുന്നുവൊ അവനേയും അതു ഉണ്ടാക്കുന്നവനേയും ആർക്കുവേണ്ടി ഉണ്ടാക്കുന്നവോ അവനേയും അത് വഹിച്ചു കൊണ്ടു പോകുന്നവനേയും അത് അരുടെ അടുത്തേക്ക് വഹിച്ചു കൊണ്ട് പോകുന്നുവൊ അവനേയും അത് വിറ്റു കിട്ടുന്ന പ്രതിഫലം ഭക്ഷിക്കുന്നവനേയും. (മുസ്നദു അഹ്‌മദ്: 4787, ഹാകിം: 7311)

ഇതേ പ്രവാചകൻ മദ്യപാനിയാണെന്ന് വരുത്തി തീർക്കാനുള്ള ദുർവ്യാഖ്യാനം അതീവ സാഹസികത തന്നെയാണെന്നതിൽ സംശയമില്ല. അതിനായി നബീദ് എന്ന ‘പഴം ജ്യൂസ്’ മദ്യമായി മാറ്റുന്ന ‘വാറ്റിയെടുക്കൽ’ പ്രക്രിയയാണ് ഈ വിവർത്തനം.

പ്രവാചകൻ (സ) കുടിച്ച നബീദ് എന്ന മധുര പാനീയം എന്താണെന്ന് ഹദീസുകളിൽ നിന്ന് തന്നെ നമ്മുക്ക് മനസ്സിലാക്കാം.

ﻋَﻦْ ﻗَﺮْﺻَﺎﻓَﺔَ ﺑِﻨْﺖِ ﻋُﻤَﺮَ ﻗَﺎﻟَﺖْ: ﺩَﺧَﻠْﺖُ ﻋَﻠَﻰ ﻋَﺎﺋِﺸَﺔَ ﻓَﻄَﺮَﺣَﺖْ ﻟِﻲ، ﻭَﺳَﺎﺩَﺓً ﻓَﺴَﺄَﻟَﺘْﻬَﺎ اﻣْﺮَﺃَﺓٌ ﻋَﻦِ اﻝﻧﺒﻴﺬ ﻓَﻘَﺎﻟَﺖْ: «ﻧَﺠْﻌَﻞُ اﻟﺘَّﻤْﺮَﺓَ ﻓِﻲ اﻟْﻜُﻮﺯِ ﻓَﻨَﻄْﺒُﺨُﻪُ ﻓَﻨَﺼْﻨَﻌَﻪُ ﻧﺒﻴﺬا ﻓَﻨَﺸْﺮَﺑُﻪُ» ﻓَﻘَﺎﻟَﺖْ: «اﺷْﺮَﺑِﻲ ﻭَﻻَ ﺗَﺸْﺮَﺑِﻲ ﻣُﺴْﻜِﺮًا»

ഒരു സ്ത്രീ നബീദിനെ സംബന്ധി ആഇശയോട്(റ) പറഞ്ഞു: ഈത്തപ്പഴം മൊന്തയിൽ ഇട്ട് പാകം ചെയ്ത് നബീദ് ഞങ്ങൾ കുടിക്കാറുണ്ട്. ആഇശ (റ) പറഞ്ഞു: നീ അത് കുടിച്ചു കൊള്ളുക. എന്നാൽ ലഹരി വന്നത് കുടിക്കരുത്. (മുസ്വന്നഫ് അബ്ദുർറസാക്: 16952)

ആഇശ (റ) പറയുന്നു: ഞങ്ങൾ അല്ലാഹുവിന്റെ ദൂതന് തോൽപാത്രത്തിൽ നബീദ് നൽകാറുണ്ടായിരുന്നു. ഞങ്ങൾ ഒരു പിടി ഈത്തപ്പഴവും ഒരു പിടി ഉണക്കമുന്തിരിയുമെടുത്ത് തോൽ പാത്രത്തിൽ ഇടും. ശേഷം അതിൽ വെള്ളമൊഴിക്കും. എന്നിട്ട് രാവിലെ ഉണ്ടാക്കിയ ആ പഴച്ചാറ് അദ്ദേഹം വൈകുന്നേരം കുടിക്കും. വൈകുന്നേരമാണ് ഉണ്ടാക്കുന്നതെങ്കിൽ രാവിലെ കുടിക്കും. (ഇബ്നുമാജ: 3398)

എന്താണ് പ്രവാചകരും അനുചരന്മാരും കുടിച്ചിരുന്ന നബീദ് എന്ന് ഈ രണ്ടു ഹദീസുകളിൽ നിന്ന് സുതരാം വ്യക്തമാണ്. തോൽപാത്രത്തിൽ ഈത്തപ്പഴമോ മറ്റു പഴവർഗങ്ങളൊ ഇട്ട് വെള്ളം ഒഴിച്ച് വെച്ച് അതിൽ നിന്നും ലഭിക്കുന്ന മധുരമുള്ള ചാറ് അല്ലെങ്കിൽ ജ്യൂസ്… ഇതാണ് നബീദ്.

ഈ പഴച്ചാറ് അല്ലെങ്കിൽ ഫ്രൂട്ട് ജ്യൂസ് ഇസ്‌ലാമിൽ നിഷിദ്ധമല്ല. കാരണം അത് മദ്യമല്ല. മദ്യം എന്നാൽ “ബുദ്ധിയെ തിരിച്ചു കളയുന്ന” ലഹരിയുള്ള വസ്തുക്കളും പാനീയങ്ങളുമാണ് എന്ന് പ്രവാചകൻ (സ) പഠിപ്പിച്ചിട്ടുണ്ട്. (സ്വഹീഹുൽ ബുഖാരി:5266)

നബീദ് എന്ന പഴച്ചാറ് അല്ലെങ്കിൽ ഫ്രൂട്ട് ജ്യൂസ് ലഹരിയുള്ളതല്ലാത്തതു കൊണ്ടാണ് പ്രവാചകൻ (സ) അതു കുടിക്കുന്നത് തെറ്റായി കാണാതിരുന്നത്.

സ്വഹീഹു മുസ്‌ലിമിലെ വിവാദ വിധേയമായ ഹദീസിനെ സംബന്ധിച്ച് ഇമാം നവവി ഇപ്രകാരം പറഞ്ഞു: “ജാബിർ (റ) പ്രവാചകന് കൊണ്ടുവന്നു കൊടുത്ത നബീദ് മൂപ്പെത്തിയിട്ടില്ലാത്തതും ലഹരി ബാധിച്ചിട്ടില്ലാത്തതുമായ പാനീയമാണ്.” (ശർഹു മുസ്‌ലിം: 13: 173)

എന്നാൽ ഈ പഴച്ചാറ് തണുപ്പുള്ള പാത്രത്തിൽ സൂക്ഷിക്കാതിരിക്കുകയോ മൂന്നിൽ അധിക ദിവസം എടുത്തു വെക്കുകയോ ചെയ്താൽ അതിൽ ലഹരി വന്നുചേരും. ഇത്തരം ലഹരി വന്ന നബീദ് പ്രവാചകനോ അനുചരന്മാരോ കുടിച്ചിരുന്നില്ല. “എന്നാൽ ലഹരി വന്നത് കുടിക്കരുത് “എന്ന് ആഇശ (റ) പ്രത്യേകം ഉണർത്തിയതായും മുകളിൽ ഉദ്ധരിച്ച ഹദീസിൽ (മുസ്വന്നഫ് അബ്ദുർറസാക്: 16952) തന്നെ കാണാം. ലഹരി വന്നാൽ അവ ‘ഖംറ്’ (الخمر) അഥവാ മദ്യമായി പരിണമിക്കുന്നു.

പ്രവാചകൻ (സ) തന്നെയും നബീദ് (അഥവാ ഈത്തപ്പഴച്ചാറിൽ) ലഹരിയുടെ ലാഞ്ചനയെങ്കിലും വന്നെന്ന് മനസ്സിലാക്കിയാൽ അത് വലിച്ചെറിയുമായിരുന്നു എന്ന് ഒട്ടനവധി ഹദീസുകളിൽ കാണാം:

باب في النبيذ إذا غلى

‘നബീദ് ലഹരിയുള്ളതായി മാറിയാൽ’ എന്ന അധ്യായത്തിൽ ഇബ്നുമാജ ഉദ്ധരിച്ച ഹദീസ് കാണുക:

ﻋﻦ ﺃﺑﻲ ﻫﺮﻳﺮﺓ، ﻗﺎﻝ: ﻋﻠﻤﺖ ﺃﻥ ﺭﺳﻮﻝ اﻟﻠﻪ ﺻﻠﻰ اﻟﻠﻪ ﻋﻠﻴﻪ ﻭﺳﻠﻢ ﻛﺎﻥ ﻳﺼﻮﻡ، ﻓﺘﺤﻴﻨﺖ ﻓﻄﺮﻩ ﺑﻧﺒﻴﺬ ﺻﻨﻌﺘﻪ ﻓﻲ ﺩﺑﺎء ﺛﻢ ﺃﺗﻴﺘﻪ ﺑﻪ ﻓﺈﺫا ﻫﻮ ﻳﻨﺶ، ﻓﻘﺎﻝ: «اﺿﺮﺏ ﺑﻬﺬا اﻟﺤﺎﺋﻂ، ﻓﺈﻥ ﻫﺬا ﺷﺮاﺏ ﻣﻦ ﻻ ﻳﺆﻣﻦ ﺑﺎﻟﻠﻪ ﻭاﻟﻴﻮﻡ اﻵﺧﺮ

അബൂഹുറൈറ (റ) പറഞ്ഞു: അല്ലാഹുവിന്റെ ദൂതൻ (സ) നോമ്പുകാരനാണെന്ന് ഞാൻ അറിഞ്ഞു. അപ്പോൾ ഞാൻ നോമ്പുതുറക്കായുള്ള നബീദ്, ദുബ്ബാഅ് പാത്രത്തിൽ ഒരുക്കി. ശേഷം അദ്ദേഹത്തിന്റെ അടുത്തേക്ക് ഞാൻ അത് കൊണ്ടുപോയപ്പോൾ അത് ലഹരിയുള്ളതായി മാറിയിരുന്നു. ആ പാത്രം അടുപ്പിച്ച് പിടിക്കാൻ അദ്ദേഹം പറഞ്ഞു. ഞാൻ അത് അടുപ്പിച്ച് പിടിച്ചപ്പോൾ അദ്ദേഹമത് മണത്ത് നോക്കി. അപ്പോൾ (മണത്തിൽ നിന്ന്) അതിന് ലഹരി ബാധിച്ചിട്ടുണ്ടെന്ന് മനസ്സിലാക്കിയപ്പോൾ പ്രവാചകൻ (സ) പറഞ്ഞു: ഇത് വലിച്ചെറിയുക. തീർച്ചയായും ഈ പാനീയം അല്ലാഹുവിലും അന്ത്യദിനത്തിലും വിശ്വസിക്കാത്തവരുടേതാകുന്നു. (മുസ്‌ലിമിനുള്ളതല്ല). (സുനനു ഇബ്നുമാജ: 3716, സുനനു നസാഈ: 5100, സുനനു ദാറകുത്‌നി: 4642) ﻋﻦ ﺃﺑﻲ ﻣﻮﺳﻰ، ﺭﺿﻲ اﻟﻠﻪ ﻋﻨﻪ ” ﺃﻧﻪ ﺃﺗﻰ اﻟﻨﺒﻲ ﺻﻠﻰ اﻟﻠﻪ ﻋﻠﻴﻪ ﻭﺳﻠﻢ ﺑﻧﺒﻴﺬ ﺟﺮ ﻳﻨﺶ، ﻓﻘﺎﻝ «اﺿﺮﺏ ﺑﻬﺬا اﻟﺤﺎﺋﻂ، ﻓﺈﻧﻪ ﻻ ﻳﺸﺮﺑﻪ ﻣﻦ ﻳﺆﻣﻦ ﺑﺎﻟﻠﻪ ﻭاﻟﻴﻮﻡ اﻵﺧﺮ»

അബൂ മൂസൽ അശ്അരിയും ലഹരിയുള്ളതായി മാറിയ നബീദ് പ്രവാചകന്റെ അടുത്തേക്ക് കൊണ്ടുപോയപ്പോൾ അദ്ദേഹം (സ) പറഞ്ഞു: ഇത് വലിച്ചെറിയുക. തീർച്ചയായും ഈ പാനീയം അല്ലാഹുവിലും അന്ത്യദിനത്തിലും വിശ്വസിക്കാത്ത ജനതയുടേതാകുന്നു. (മുസ്നദുൽ ബസ്സാർ: 3192)

പ്രവാചകൻ (സ) പറഞ്ഞു: “നിങ്ങൾ നബീദ് കുടിച്ചു കൊള്ളുക. എന്നാൽ ലഹരിയുള്ളത് ഞാൻ അനുവദിക്കില്ല…” (മുസ്നദു അഹ്‌മദ്: 11329)

അപ്പോൾ പ്രവാചകൻ (സ) കുടിച്ചതായി വിവാദ വിഷയകമായ ഹദീസിൽ പറയുന്ന നബീദ് ലഹരി വന്നിട്ടില്ലാത്ത കേവലം ഈത്തപ്പഴച്ചാറാണ്. അഥവാ തനിക്കു നൽകപ്പെടുന്ന നബീദിൽ ലഹരിയുടെ വാസനയൊ ലാഞ്ചനയോ മനസ്സിലാക്കിയാൽ അവ പ്രവാചകൻ (സ) വലിച്ചെറിയുകയാണ് പതിവ് എന്നർത്ഥം.

ഈ പ്രവാചകനെയാണോ നിങ്ങൾ മദ്യപാനിയായി ചിത്രീകരിക്കുന്നത് ?!!

അപ്പോൾ നബീദ് സ്വമേധയാ അനുവദനീയമായ ഒരു മധുര പാനീയമാണ്. അവ ഒരുപാട് ദിവസം സൂക്ഷിച്ചു വെച്ച് പാകപ്പെടുത്തിയെടുത്ത് ലഹരിയുള്ളതാക്കി മാറ്റുമ്പോളാകട്ടെ ഇസ്‌ലാമിൽ അത് നിഷിദ്ധമാണ്. ഈ ലഹരിയുള്ള നബീദ് അഥവാ വീഞ്ഞായി പരിണമിച്ച നബീദ് പ്രവാചകൻ (സ) കുടിച്ചതായി ഒരു ഹദീസും നമുക്ക് പറഞ്ഞു തരുന്നില്ല. ഈ വസ്തുത അമുസ്‌ലിംകളായ ഗവേഷകർ പോലും രേഖപ്പെടുത്തിയിട്ടുണ്ട്:

ബ്രിട്ടൺ ഗവൺമെന്റിന്റെ ‘നാവൽ ഇന്റലിജൻസ് ഡിവിഷൻ’ (Naval Intelligence Division) തയ്യാറാക്കുകയും ഓക്സ്ഫോർഡ് യൂനിവേഴ്സിറ്റി പ്രസ്സ് പുറത്തിറക്കുകയും ചെയ്ത ‘western Arabia and the Red Sea’ എന്ന പുസ്തകത്തിൽ (പേജ്: 489) ഇപ്രകാരം രേഖപ്പെടുത്തപ്പെട്ടിരിക്കുന്നു: “പുതിയ ഈത്തപ്പഴങ്ങളുടെ മേൽ വെള്ളം ഒഴിച്ച് ഒറ്റരാത്രികൊണ്ട് ഉണ്ടാക്കപ്പെട്ട നബീദ് എന്നറിയപ്പെടുന്ന ഒരു ലഹരി ബാധിതമല്ലാത്ത പാനീയം മുഹമ്മദിന്റെ കാലം മുതൽ കുടിക്കപ്പെട്ടിരുന്നു.” (അവലംബം: https://dawatetowheed.wordpress.com)

നബീദിൽ ലഹരി കടന്നുവരാനുള്ള സാധ്യതയുള്ളതിനാൽ തന്നെ അത്തരം സാധ്യതകളെ പോലും പ്രവാചകൻ (സ) വിലക്കുകയാണുണ്ടായത് എന്നും ഹദീസുകൾ നമ്മെ പഠിപ്പിക്കുന്നു.

ദുബ്ബാഅ്, നകീർ, മുസഫ്ഫത്ത്, ഹൻത്തം എന്നീ പാത്രങ്ങളിൽ നബീദ് സൂക്ഷിക്കുന്നതും കുടിക്കുന്നതും പ്രവാചകൻ (സ) സ്വയം വർജിക്കുകയും അനുചരന്മാരെ വിലക്കുകയും ചെയ്തത് ഇക്കാരണത്താലാണ്. ഇത്തരം പാത്രങ്ങൾക്ക് താഴ്ന്ന താപനില നിലനിർത്തി പഴച്ചാറുകളുടെ ലഹരിവൽകരണത്തെ മന്ദഗതിയിലാക്കാൻ കഴിയില്ല. അതേസമയം തോൽ പാത്രം താഴ്ന്ന താപനില നിലനിർത്തുന്നതിന് സഹായിക്കുന്നു. അതിനാൽ ലഹരി വൽക്കരണം തടയപ്പെടുന്നു. രണ്ടോ മൂന്നോ ദിവസം ഈത്തപ്പഴ വെള്ളം ‘ഫ്രെഷായി’ അവശേഷിക്കുന്നു. മറ്റൊരർത്ഥത്തിൽ പറഞ്ഞാൽ അന്നത്തെ ഒരു ‘മിനി ഫ്രിഡ്ജാ’യിരുന്നു തോൽ പാത്രം. ഇക്കാര്യം വ്യക്തമാക്കുന്ന ഹദീസുകൾ കാണുക:

പ്രവാചകൻ (സ) പറഞ്ഞു: “ദുബ്ബാഅ്, ഹൻത്തം, നകീർ… തുടങ്ങിയ പാത്രങ്ങളിൽ നബീദ് കുടിക്കുന്നത് ഞാൻ നിങ്ങളെ വിലക്കുന്നു. എന്നാൽ തോൽ പാത്രത്തിലുള്ള നബീദ് കുടിച്ചു കൊള്ളുക.” (സ്വഹീഹു മുസ്‌ലിം: 33 – 1993)

“ദുബ്ബാഅ്, നകീർ, മുസഫ്ഫത്ത്, ഹൻത്തം എന്നീ പാത്രങ്ങളിൽ നബീദ് കുടിക്കുന്നത് പ്രവാചകൻ (സ) വിലക്കി.” (സ്വഹീഹു മുസ്‌ലിം: 1995)

പഴച്ചാറുകളുടെ ലഹരിവൽക്കരണത്തിൽ അഥവാ ഫെർമെന്റേഷനിൽ (fermentation) താപനിലക്കും വായുവിനും വളരെ വലിയ സ്വാധീനമുണ്ട്. താപനില കുറയുന്നത് പദാർത്ഥങ്ങളെ പുളിപ്പിച്ച് ലഹരിപാനീയങ്ങളാക്കി മാറ്റുന്ന പ്രക്രിയയെ മന്ദഗതിയിലാക്കുന്നു. ഉയർന്ന താപനിലയിൽ പഴച്ചാറുകളുടെ പുളിക്കൽ, അതിവേഗത്തിൽ ആകുന്നു. തണുത്ത താപനില നൽകാനും നിലനിർത്താനും തോൽപാത്രം സഹായിക്കുന്നു.

ഇബ്നു ഹജർ പറഞ്ഞു: “ദുബ്ബാഅ്, ഹൻത്തം, നകീർ തുടങ്ങിയ പാത്രങ്ങളിൽ നബീദ് കുടിക്കരുത് എന്ന് ഹദീസുകളിൽ ഉദ്ധരിക്കപ്പെട്ടിട്ടുണ്ട്. തോൽ പാത്രവും ഇത്തരം പാത്രങ്ങളും തമ്മിലുള്ള വ്യത്യാസം തോൽ പാത്രമല്ലാത്ത മറ്റു പാത്രങ്ങളിലെ സൂക്ഷ്മായ വിടവുകളിലൂടെ വായു ഉള്ളിലേക്ക് പ്രവേശിക്കും എന്നതാണ്. ഇത് മൂലം തോൽ പാത്രത്തിലുള്ള നബീദിൽ നിന്ന് വ്യത്യസ്ഥമായി മറ്റു പാത്രങ്ങളിലേത് കേടു വരാനും ലഹരി ബാധിക്കാനും ഇടയാകുന്നു. അതുകൊണ്ടാണ് അത്തരം പാത്രങ്ങളിൽ നബീദ് സൂക്ഷിക്കുന്നത് പ്രവാചകൻ (സ) വിരോധിച്ചത്. കൂടാതെ, തോൽപാത്രം ഭദ്രമായി കെട്ടിവെച്ചാൽ ലഹരി ബാധയിൽ നിന്നും സുരക്ഷിതമാവുന്നു. കാരണം നബീദ് പുളിപ്പ് വരികയും സ്ഥിതിഭേദം സംഭവിക്കുകയും ചെയ്താൽ അത് ലഹരിയുള്ളതായി മാറും…” (ഫത്ഹുൽ ബാരി: 10: 61)

ഇമാം നവവി പറഞ്ഞു: “പണ്ഡിതന്മാർ പറഞ്ഞിരിക്കുന്നു: തോൽപാത്രം ഭദ്രമായി കെട്ടിവെച്ചാൽ ലഹരി ബാധയിൽ നിന്നും സുരക്ഷിതമാവുന്നു. കാരണം നബീദ് പുളിപ്പ് വരികയും സ്ഥിതിഭേദം സംഭവിക്കുകയും ചെയ്താൽ അത് ലഹരിയുള്ളതായി മാറും… ദുബ്ബാഅ്, ഹൻത്തം, മുസഫ്ഫത്ത് തുടങ്ങിയ പാത്രങ്ങളിൽ നബീദ് സൂക്ഷിച്ചാൽ അവ ലഹരിയുള്ളതായി മാറും…” (ശർഹു മുസ്‌ലിം: 13: 160)

ദുബ്ബാഅ് (اﻟﺪُّﺑَّﺎء) എന്നാൽ മത്തങ്ങ തുരന്ന് ഉണ്ടാക്കിയ പാത്രമാണ്. നകീർ (اﻟﻨَّﻘﻴﺮ), ഹൻത്തം (اﻟﺤﻨَﺎﺗِﻢ) എന്നിവ ഈത്തപ്പനയുടെ മുരട്, നടുവിലെ കഴമ്പ് കളഞ്ഞ് ഉപയോഗിക്കുന്ന പാത്രങ്ങളാണ്. മുസഫ്ഫത്ത് (اﻟﻤُﺰَﻓَّﺖ) ടാറ് പൂശിയ മരപാത്രം. (മുസ്നദു ശാഫിഈ: 309, 310)

ഇത്തരം പാത്രങ്ങൾ നബീദിന്റെ പുതുമയും ശുദ്ധതയും കുളിർമയും നിലനിർത്താൻ ഉതകുന്നവയല്ലാത്തത് കൊണ്ട് അവയിൽ സൂക്ഷിക്കപ്പെടുന്ന നബീദ് ലഹരി ബാധിതമാകുമായിരുന്നു. അതുകൊണ്ട് തന്നെ ഇത്തരം പാത്രങ്ങളിൽ നബീദ് കുടിക്കുന്നത് പ്രവാചകൻ (സ) വിലക്കുകയും തോൽപാത്രത്തിൽ മാത്രം നബീദ് കുടിക്കുന്നത് അദ്ദേഹം (സ) അനുവദിക്കുകയും ചെയ്തു. എന്നാൽ ഈ യുക്തി തിരിച്ചറിയാതിരുന്ന ചിലർ ഈ പാത്രങ്ങൾ സ്വമേധയാ നിഷിദ്ധമാണെന്ന് തെറ്റിദ്ധരിച്ചു. ചിലർ ഇതല്ലാത്ത പാത്രങ്ങളുടെ ലഭ്യതയുടെ കുറവിനെ സംബന്ധിച്ച് പ്രവാചകനോട് പരാതിപ്പെട്ടു. തദവസരത്തിൽ ആ പാത്രങ്ങൾ പ്രവാചകൻ (സ) അവർക്ക് അനുവദിച്ച് ഇളവു നൽകി. എന്നാൽ അന്നേരവും അത്തരം പാത്രങ്ങളിൽ സൂക്ഷിച്ച നബീദുകളിൽ ലഹരി വന്നാൽ കുടിക്കരുത് എന്ന് പ്രത്യേകം ഉണർത്തുകയും ചെയ്തു.

പ്രവാചകൻ (സ) പറഞ്ഞു: “ചില പാത്രങ്ങളിലെ നബീദിൽ നിന്നും ഞാൻ നിങ്ങളെ വിലക്കിയിരുന്നു. എന്നാൽ നിങ്ങൾ അറിയുക ആ പാത്രങ്ങളിൽ ഉള്ളതിനാൽ മാത്രം അവ ഒന്നിനേയും നിഷിദ്ധമാക്കുന്നില്ല. എന്നാൽ എല്ലാ ലഹരിയുള്ളവയുമാണ് നിഷിദ്ധം.” (അൽ ജാമിഉ ലി ഇബ്നു വഹബ്: 26)

പ്രവാചകൻ (സ) പറഞ്ഞു: “തോൽപാത്രത്തിൽ ഉള്ളതല്ലാത്ത എല്ലാ നബീദും ഞാൻ നിങ്ങൾക്ക് വിരോധിച്ചിരുന്നു. എന്നാൽ (ഇനിമേൽ) എല്ലാ പാത്രത്തിലും നിങ്ങൾ നബീദ് കുടിച്ചു കൊള്ളുക. എന്നാൽ ലഹരിയുള്ളത് കുടിക്കരുത്.” (സ്വഹീഹു മുസ്‌ലിം: 977,1977, മുസ്നദുൽ ബസ്സാർ : 4435, സുനനു തുർമുദി: 1054, ത്വയാലിസി: 807, ഹാകിം: 1/375, മുസ്നദു അഹ്മദ്: 5/259, 261, സുനനു അബൂദാവൂദ്: 3235, ബൈഹകി: 4/76, ബഗ്‌വി: 1553, അബ്ദുർറസാക്: 6708)

പ്രവാചകൻ (സ) പറഞ്ഞു: “കളിമൺ പാത്രത്തിൽ സൂക്ഷിച്ച നബീദ് കുടിക്കുന്നതിൽ നിന്നും ഞാൻ നിങ്ങളെ വിലക്കിയിരുന്നു. എന്നാൽ ഇനിമേൽ എല്ലാ പാത്രത്തിലും നിങ്ങൾ നബീദ് കുടിച്ചു കൊള്ളുക. (എന്നാൽ) എല്ലാ ലഹരിയുള്ള (നബീദും) ഉപേക്ഷിക്കുക.” (മുസ്വന്നഫ് അബ്ദുർറസാക്: 6708, 6714, മുസ്വന്നഫ് ഇബ്നു അബീ ശൈബ: 23942,)

തബൂക്ക് യുദ്ധ സന്ദർഭത്തിൽ ചിലർക്ക്, ദുബ്ബാഅ്, ഹൻതം, മുസഫ്ഫത്ത് എന്നീ പാത്രങ്ങളിൽ നബീദ് കുടിക്കുന്നത് പ്രവാചകൻ (സ) നിരോധിച്ചു. പിന്നീട് അവർക്ക് വയറിന് അസുഖം ബാധിച്ചതായി അവർ പരാധിപ്പെട്ടപ്പോൾ ആ പാത്രങ്ങളിൽ കുടിക്കുന്നത് പ്രവാചകൻ (സ) ഇളവു നൽകി. എന്നാൽ ലഹരിയുള്ള (നബീദ്) കുടിക്കുന്നത് അദ്ദേഹം പ്രത്യേകം വിലക്കി. (അൽ ആസാർ: അബൂ യൂസുഫ്: 997)

ഇത്തരം പാത്രങ്ങൾ അനുവദിച്ച അവസരത്തിലും നബീദുകളിൽ ലഹരി വന്നാൽ കുടിക്കരുത് എന്ന് പ്രവാചകൻ (സ) പ്രത്യേകം ഉണർത്തുന്നത് ശ്രദ്ധിക്കുക. അതു പോലും ശ്രദ്ധിച്ച പ്രവാചകനെയാണോ നിങ്ങൾ മദ്യപാനിയായി ചിത്രീകരിക്കാൻ ശ്രമിക്കുന്നത് ?!!

നബീദ് ഒരുപാടു ദിവസം സൂക്ഷിച്ചാലും മത്തും ലഹരിയും ബാധിക്കാൻ സാധ്യതയുണ്ട്. ഈ സാധ്യത അറിയാവുന്നതു കൊണ്ട് തന്നെ പരമാവധി മൂന്ന് ദിവസ കാലയളവിനപ്പുറം – തോൽപാത്രത്തിൽ സൂക്ഷിച്ച നബീദാണെങ്കിൽ പോലും – അത് കുടിക്കാതിരിക്കാൻ പ്രവാചകൻ (സ) കാർക്കശ്യം കാണിച്ചിരുന്നു. മൂന്ന് ദിവസ കാലയളവിനപ്പുറമുള്ള നബീദ് കുടിക്കരുതെന്ന് മറ്റുള്ളവരോട് ഗൗരവകരമായ ശാസനയും നൽകി. ഈ പ്രവാചകനെയാണോ നിങ്ങൾ മദ്യപാനിയായി ചിത്രീകരിക്കാൻ ശ്രമിക്കുന്നത് ?!!

ഇബ്നു അബ്ബാസ് (റ) പറഞ്ഞു: അല്ലാഹുവിന്റെ ദൂതന്(സ) തോൽപാത്രത്തിൽ നബീദ് നൽകപെട്ടിരുന്നു. ഞായറാഴ്ച്ച രാത്രി നൽകപ്പെട്ട നബീദ് അദ്ദേഹം തിങ്കളാഴ്ച്ച രാവിലെ അദ്ദേഹം കുടിക്കുമായിരുന്നു. ചെവ്വാഴ്ച്ച അസ്ർ വരെയും കുടിക്കും. എന്നിട്ടും വല്ലതും അവശേഷിച്ചാൽ ഭൃത്യന് നൽകുകയോ അയാൾക്കും വേണ്ടെങ്കിൽ ഒഴിച്ച് കളയുകയോ ചെയ്യും (സ്വഹീഹു മുസ്‌ലിം: 2004)

അല്ലാഹുവിന്റെ ദൂതൻ (സ) ഒരു യാത്ര പുറപ്പെട്ടു. ശേഷം മടങ്ങിവന്നു. അപ്പോൾ ചില അനുചരർ ഹനാതിം, നകീർ, ദുബ്ബാഅ് എന്നീ പാത്രങ്ങളിൽ നബീദ് തയ്യാറാക്കിയിരുന്നു. അപ്പോൾ (അവയിൽ ലഹരി വന്നതിനാൽ) അത് ഒഴുക്കി കളയാൻ പ്രവാചകൻ (സ) കൽപ്പിച്ചു. ശേഷം തോൽ പാത്രത്തിൽ ഉണക്കമുന്തിരിയും വെള്ളവും നിക്ഷേപിക്കാൻ പറഞ്ഞു. രാത്രി ഉണ്ടാക്കിയത് പിറ്റേന്ന് പകലും വരുന്ന രാത്രിയും കുടിച്ചു. പിറ്റേ ദിവസം വൈകുന്നേരം വരെ കുടിക്കുകയും കുടിപ്പിക്കുകയും ചെയ്തു. അതിന് പിറ്റേന്ന് രാവിലെ ഭാക്കിയായവ ഒഴുക്കി കളയാൻ കൽപ്പിച്ചു. (സ്വഹീഹു മുസ്‌ലിം: 83- 2004)

ഈ ഹദീസുകളെ വ്യാഖ്യാനിച്ചു കൊണ്ട് ഇമാം നവവി (റ) പറഞ്ഞു:

“മൂന്ന് ദിവസം വരെ പ്രവാചകൻ (സ) നബീദ് കുടിക്കുകയും എന്നിട്ടും വല്ലതും അവശേഷിച്ചാൽ ഒഴിച്ച് കളയുകയും ചെയ്യുമായിരുന്നു (സ്വഹീഹു മുസ്‌ലിം: 2004) എന്ന് ഇബ്നു അബ്ബാസ് പറഞ്ഞ ഹദീസും സമാനമായ ഹദീസുകളും തെളിയിക്കുന്നത് നബീദ് ഉണ്ടാക്കുകയും, നബീദ് മധുര പാനീയമായി അവശേഷിക്കുകയും, മാറ്റം വരുകയോ ലഹരി ബാധിക്കുകയോ ചെയ്യാത്തിടത്തോളം കുടിക്കുകയും ചെയ്യൽ അനുവദനീയമാണെന്ന കാര്യത്തിൽ മുസ്‌ലിം സമുദായം മുഴുവൻ ഏകോപിച്ചിരിക്കുന്ന കാര്യമാണ്. മൂന്ന് ദിവസം കഴിഞ്ഞ് പ്രവാചകൻ (സ) അത് കുടിക്കാതെ ഒഴിച്ച് കളയാൻ കാരണം മൂന്ന് ദിവസത്തിനപ്പുറം നബീദിൽ മാറ്റം സംഭവിക്കുകയും ലഹരി ബാധിക്കുകയും ചെയ്യാൻ സാധ്യത ഉള്ളതിനാലാണ്. ലഹരിയുടെ പ്രാധമികമായ വല്ല ലക്ഷണങ്ങളോ മാറ്റമോ നബീദിൽ പ്രത്യക്ഷപ്പെട്ടാൽ അത് പ്രവാചകൻ (സ) ഒഴുക്കി കളയുമായിരുന്നു. കാരണം, ലഹരിയുള്ളതായി മാറിയാൽ നബീദ് ഹറാമും അശുദ്ധവുമാണ്. ഇത്തരം അവസ്ഥയിൽ പ്രവാചകൻ (സ) അവ ഒഴുക്കി കളയും, ഭൃത്യന്മാരെയും കുടിപ്പിക്കില്ല. കാരണം ലഹരിയുള്ള നബീദ് കുടിക്കലും കുടിപ്പിക്കലും അനുവദനീയമല്ല. എന്നാൽ തോൽ പാത്രത്തിൽ സൂക്ഷിച്ച, മൂന്ന് ദിവസ പരിധിക്കുള്ളിലുള്ള നബീദിൽ മാറ്റം സംഭവിക്കുകയോ ലഹരി ബാധിക്കുകയോ ചെയ്യില്ല. ” (ശർഹു മുസ്‌ലിം: 13: 173)

ഇതേ കാര്യം തന്നെ ഇമാം ഇബ്നു കുദാമ തന്റെ ‘മുഗ്നി'(3/144) എന്ന ഗ്രന്ഥത്തിൽ പറയുന്നു.

നബീദ് രണ്ടു വിധമുണ്ടെന്നും, സ്വമേധയാ അവ കേവല പഴച്ചാറും അനുവദനീയമായ പാനീയവുമാണെന്നും, ലഹരി വന്ന നബീദ് (അഥവാ മദ്യം) നിഷിദ്ധമാണെന്നും തെളിയിക്കുന്ന, പ്രവാചകന്റെ(സ) സമകാലികരുടെ വാചകങ്ങൾ ശ്രദ്ധിക്കുക: അലി (റ) പറഞ്ഞു: “ഖംറ് (الخمر) അഥവാ മദ്യമോ ലഹരിയുള്ള നബീദോ കുടിച്ച ആരെയെങ്കിലും എന്റെ അടുത്ത് കൊണ്ട് വരപ്പെട്ടാൽ അയാൾക്ക് (ഭരണാധികാരിയെന്ന നിലക്ക്) ഞാൻ അടി ശിക്ഷയായി നടപ്പാക്കുന്നതാണ്.” (മുസ്നദു ശാഫിഈ: 1322)

പ്രവാചക പൗത്രൻ ഹസനിബ്നു അലിയോട് നബീദിനെ സംബന്ധിച്ച് ചോദിക്കപ്പെട്ടു. അപ്പോൾ അദ്ദേഹം പറഞ്ഞു: “അത് കുടിച്ചു കൊള്ളു. എന്നാൽ അത് ലഹരിയുള്ളതായി മാറാൻ തുടങ്ങിയാൽ ഉപേക്ഷിക്കുക.” (മുസ്വന്നഫ് ഇബ്നു അബീ ശൈബ: 23872)

മുഖ്താർ ബിൻ ഫുൽഫുൽ പറഞ്ഞു: ഞാൻ പ്രവാചകാനുചരൻ അനസ് ബിൻ മാലികിനോട് നബീദിനെ പറ്റി ചോദിച്ചു. അപ്പോൾ അദ്ദേഹം പറഞ്ഞു: “അവയിൽ ലഹരിയുള്ളതെല്ലാം ഉപേക്ഷിക്കുക…” (മുസ്വന്നഫ് ഇബ്നു അബീ ശൈബ: 23803)

ആമിർ പറഞ്ഞു: “നിങ്ങൾ കല്യാണ സദ്യയിലെ നബീദ് കുടിക്കുക. ലഹരിയുള്ള നബീദ് കുടിക്കരുത്.” (മുസ്വന്നഫ് ഇബ്നു അബീ ശൈബ: 23886)

അബുൽ ആലിയ പറഞ്ഞു: “നിങ്ങളുടെ നബീദ് ലഹരിയുള്ളതിനാൽ വളരെ വ്യത്തികെട്ടതായി ഞങ്ങൾ കാണുന്നു. പണ്ടത്തെ നബീദ് ഈത്തപ്പഴം ഇട്ട മധുര വെള്ളമായിരുന്നു.” (സുനനുൽ കുബ്റാ: 34)

നബീദ് സ്വമേധയാ കേവല മധുര പാനീയമാണെന്നും അതിൽ ലഹരി വന്നാൽ പ്രവാചകൻ (സ) അവ സ്വയം വർജിക്കുകയും മറ്റുള്ളവർക്ക് വിലക്കുകയും ചെയ്തിട്ടുണ്ട് എന്ന് തെളിയിക്കുന്ന മറ്റു ചില ഹദീസുകൾ കൂടി കാണുക:

പ്രവാചകൻ (സ) പറഞ്ഞു: “ചോളം കൊണ്ടും തേൻ കൊണ്ടും ലഹരിയുള്ള നബീദ് കുടിക്കുന്നതിനെ സംബന്ധിച്ച് പ്രവാചകനോട് ചോദിച്ചപ്പോൾ അദ്ദേഹം പറഞ്ഞു: ലഹരിയുള്ളത് നിങ്ങൾ കുടിക്കരുത്.” (ബുഖാരി: 4343, മുസ്നദു അഹ്‌മദ്: 19598, 24652,)

സ്വിഹാർ ഇബ്നു സ്വഖ്റുൽ അബ്ദി, നബീദ് കുടിക്കാമോ എന്ന് ചോദിച്ചപ്പോൾ പ്രവാചകൻ (സ) മറുപടി പറഞ്ഞു: “നിങ്ങളുടെ ബോധത്തെ മത്തുപിടിപ്പിക്കാത്ത നബീദ് നിങ്ങൾക്ക് കുടിക്കാം…” (അൽമുഅ്ജമുൽ കബീർ: ത്വബ്റാനി: 7405)

അബ്ദുൽ കൈസ് നിവേദക സംഘത്തോട് പ്രവാചകൻ (സ) പറഞ്ഞു: (നകീർ (പാത്രത്തിൽ) സൂക്ഷിക്കപ്പെട്ട നബീദ് പെട്ടെന്ന് ലഹരി ബാധിക്കുന്നതിനാൽ) നിങ്ങൾ നകീർ (പാത്രത്തിൽ) നബീദ് കുടിക്കരുത്. കുടിച്ചാൽ നിങ്ങൾ (ലഹരി മൂലം) പരസ്പരം അടിപിടി കൂടുന്നതാണ്. (അൽമുഅ്ജമുൽ കബീർ: ത്വബ്റാനി: 122)

ഹദീസിൽ പരാമർശിക്കപ്പെട്ട നബീദിനെ സംബന്ധിച്ച് ചർച്ച ചെയ്ത പൗരാണികരായ ഹദീസ് പണ്ഡിതന്മാരും മനസ്സിലാക്കിയതും പഠിപ്പിച്ചതും ലഹരിയില്ലാത്ത പഴച്ചാറാണ് പ്രവാചകൻ (സ) കുടിച്ചതും അനുവദിച്ചതുമായ നബീദ് എന്നാണ്. അഥവാ അവയിൽ ലഹരി ബാധിച്ചാൽ അത് മദ്യമായി പരിണമിക്കുമെന്നും തദവസരത്തിൽ പ്രവാചകൻ (സ) അത് വലിച്ചെറിയുകയും ചെയ്തിരുന്നു. ഹദീസ് പണ്ഡിതന്മാരുടെ വിശദീകരണങ്ങൾ കാണുക:

1. ‘സ്വഹീഹു മുസ്‌ലിമി’ലെ വിവാദ വിധേയമായ ഹദീസിന്റെ തൊട്ടു മുമ്പത്തെ അധ്യായത്തിന്റെ പേര് തന്നെ ഇപ്രകാരമാണ്.

ﺑﺎﺏ ﺇﺑﺎﺣﺔ اﻝﻧﺒﻴﺬ اﻟﺬﻱ ﻟﻢ ﻳﺸﺘﺪ ﻭﻟﻢ ﻳﺼﺮ ﻣﺴﻜﺮا “മത്ത് വരാത്ത, ലഹരിയുള്ളതായി മാറിയിട്ടില്ലാത്ത നബീദ് അനുവദനീയമാണ്.”

2. ഇമാം നവവി പറഞ്ഞു: “ഉണക്കമുന്തിരിയും അല്ലാത്തതുമായ പഴങ്ങൾ ഇട്ട മധുരമുള്ള വെള്ളമാണ് ഹദീസിൽ പറഞ്ഞിരിക്കുന്ന നബീദ്. ഈ നബീദ് നല്ല രുചിയുള്ളതും എന്നാൽ ലഹരിയില്ലാത്തതുമാണ്. എന്നാൽ ഈ നബീദ് ദീർഘകാലം കൊണ്ട് ലഹരിയുള്ളതായി പരിണമിക്കും. ഇത് ഹറാം (നിഷിദ്ധം) ആകുന്നു.” (ശർഹു മുസ്‌ലിം: 9: 64)

3. ഇമാം ഇബ്നു ഹിബ്ബാന്റെ ‘സ്വഹീഹിലെ’ ചില അധ്യായങ്ങളുടെ പേരുകൾ കാണുക: “ബാർലിയും തേനും കലർത്തിയ മധുര പാനീയമായമായ നബീദ്, ലഹരിയുള്ളതായാൽ അത് നിഷിദ്ധമായി മാറുന്നതാണ്” (സ്വഹീഹു ഇബ്നു ഹിബ്ബാൻ: 12:190)

ലഹരി ബാധിച്ചിട്ടില്ലാത്ത നബീദ് കുടിക്കൽ ഒരാൾക്ക് അനുവദനീയമാണ് (സ്വഹീഹു ഇബ്നു ഹിബ്ബാൻ: 12:207)

“നബീദ് ലഹരി ബാധിച്ച അവസ്ഥയിൽ എത്തിയാൽ പ്രവാചകൻ (സ) അത് കുടിക്കുകയല്ല, വലിച്ചെറിയുകയാണ് ചെയ്തിരുന്നത്.” (സ്വഹീഹു ഇബ്നു ഹിബ്ബാൻ: 12:208)

4. ഇബ്നു അബ്ദി റബ്ബി പറഞ്ഞു: “നിഷിദ്ധമായ കാര്യങ്ങൾക്ക് പകരം അല്ലാഹു അനുവദനീയമായ കാര്യങ്ങൾ ബദലായി നമ്മുക്ക് നൽകിയിട്ടുണ്ട്. പലിശ നിഷിദ്ധമാക്കുകയും കച്ചവടം അനുവദനീയമാക്കുകയും ചെയ്തു. വ്യഭിചാരം നിഷിദ്ധമാക്കുകയും വിവാഹം അനുവദനീയമാക്കുകയും ചെയ്തു. പട്ട് നിഷിദ്ധമാക്കുകയും ചിത്രത്തുന്നലുകൾ അനുവദനീയമാക്കുകയും ചെയ്തു. മദ്യം നിഷിദ്ധമാക്കുകയും ലഹരിയില്ലാത്ത നബീദ് അനുവദനീയമാക്കുകയും ചെയ്തു; എന്നാൽ നബീദ് ലഹരിയുള്ളതായാൽ നിഷിദ്ധമാണ്.” (അൽ ഇക്ദുൽ ഫരീദ്: 8: 67)

5. അബ്ദുർറഹ്മാൻ മുബാറക് പുരി പറഞ്ഞു:

“ഈന്തപ്പഴത്തിൽ നിന്നും മുന്തിരിയിൽ നിന്നും നബീദ് ഉണ്ടാക്കി എന്ന് പറഞ്ഞാൽ അതിനർത്ഥം ആ പഴങ്ങൾ വെള്ളത്തിൽ ഇട്ട് പഴച്ചാറ് എടുത്തു എന്നാണ്. നബീദ് എന്നാൽ ഈ പഴച്ചാറാണ്; അത് ലഹരി ഉള്ളതാവാം ഇല്ലാത്തതാവാം. പഴത്തിൽ നിന്ന് പിഴിഞ്ഞെടുത്ത (ചാറ് കൂടുതൽ കാലം സൂക്ഷിക്കുന്നതിലൂടെ) ലഹരിയുള്ള നബീദിന് ഖംറ് (മദ്യം) എന്നാണ് വിളിക്കപ്പെടുക.” (തുഹ്‌ഫത്തുൽ അഹ്‌വദി: 1: 245)

“ജസ്‌രി നിഹായയിൽ പറഞ്ഞു: ഈത്തപ്പഴം, ഉണക്കമുന്തിരി, തേൻ, ഗോതമ്പ്, ബാർലി തുടങ്ങിയവയിൽ നിന്ന് ഉണ്ടാക്കപ്പെടുന്ന മധുര പാനീയങ്ങളാണ് നബീദ്. ഈന്തപ്പഴത്തിൽ നിന്നും മുന്തിരിയിൽ നിന്നും നബീദ് ഉണ്ടാക്കി എന്ന് പറഞ്ഞാൽ അതിനർത്ഥം ആ പഴങ്ങൾ വെള്ളത്തിൽ ഇട്ട് പഴച്ചാറ് എടുത്തു എന്നാണ്. നബീദ് എന്നാൽ ഈ പഴച്ചാറാണ്; അത് ലഹരി ഉള്ളതാവാം ഇല്ലാത്തതാവാം. രണ്ടിനേയും നബീദ് എന്ന് പറയും. മധുര പാനീയമായിരിക്കുന്നേടത്തോളം, ലഹരിയുടെ അവസ്ഥയിലേക്ക് എത്താത്തിടത്തോളം നബീദ് അനുവദനീയമാണെന്നതിൽ മുസ്‌ലിംകൾക്കിടയിൽ ഏകാഭിപ്രായമുണ്ട്.” (തുഹ്‌ഫത്തുൽ അഹ്‌വദി: 5: 494)

പൗരാണികരായ അറബി ഭാഷാ പണ്ഡിതർ തങ്ങളുടെ നിഖണ്ഡുക്കളിൽ നബീദിനെ സംബന്ധിച്ച് ചർച്ച ചെയ്യുന്നുണ്ട്. നബീദ് സാധാരണ ഗതിയിൽ ഒരു മധുര പാനീയമാണെന്നും അത് ലഹരിയുള്ളതായി പരിവർത്തിപ്പിക്കപ്പെട്ടാൽ മാത്രമാണ് മദ്യമായി മാറുന്നതെന്നും അവരും വ്യക്തമാക്കുന്നു. നബീദ് ലഹരിയുള്ളതും ഇല്ലാത്തതുമുണ്ടെന്നും ലഹരിയുള്ളത് ഖംറ് (മദ്യം الخمر) എന്ന് വിളിക്കപ്പെടുന്നുവെന്നും അറബി ഭാഷാ നിഖണ്ഡുക്കൾ വിശദീകരിക്കുന്നത് കാണുക:

1. “നബദ (ﻧﺒﺬ) എന്നാൽ ഒരു സാധനത്തെ മുമ്പിലോട്ടോ പിന്നിലോട്ടോ ഇടുക എന്നാണ്. നബദ്തു (ﻧَﺒَﺬْﺕُ) എന്നാൽ ഞാൻ ഒരു കാര്യത്തെ എറിഞ്ഞു, ഞാൻ അകലെ ഇട്ടു എന്നൊക്കെയാണർത്ഥം. നബ്ദ് (اﻟﻨَّﺒْﺬُ) എന്നാൽ ഇടുക (اﻟﻄَّﺮْﺡُ) എന്നാണ്. (പഴങ്ങൾ വെള്ളത്തിൽ ‘ഇട്ട്’ ഉണ്ടാക്കപ്പെടുന്ന ഈ നബീദ്) ലഹരി ബാധിക്കാത്തിടത്തോളം ഹലാൽ (അനുവദനീയം) ആകുന്നു. ലഹരി ബാധിച്ചാൽ ഹറാം (നിഷിദ്ധം) ആകുന്നു. നബീദ് എന്ന പദം ഹദീസുകളിൽ ധാരാളമായി ആവർത്തിക്കപ്പെട്ടിട്ടുണ്ട്. ഈത്തപ്പഴം, ഉണക്കമുന്തിരി, തേൻ, ഗോതമ്പ്, ബാർലി തുടങ്ങിയവയിൽ നിന്ന് ഉണ്ടാക്കപ്പെടുന്ന മധുര പാനീയങ്ങളാണ് നബീദ്. നബീദ് എന്നാൽ ഈ പഴച്ചാറാണ്; അത് ലഹരി ഉള്ളതാവാം ഇല്ലാത്തതാവാം. രണ്ടിനേയും നബീദ് എന്ന് പറയും. പഴത്തിൽ നിന്ന് പിഴിഞ്ഞെടുത്ത ചാറ് (നബീദ്) ലഹരിയുള്ളതായാൽ ഖംറ് (മദ്യം) എന്നാണ് വിളിക്കപ്പെടുക.” (ലിസാനുൽ അറബ്: 3: 511)

2. “നബദ എന്നാൽ അകലെ ഇട്ടു, എറിഞ്ഞു എന്നിവയാണ് (ഭാഷാ പരമായ) അർത്ഥം. ഹദീസുകളിൽ പറയപ്പെട്ട നബീദ് ഈത്തപ്പഴം, ഉണക്കമുന്തിരി, തേൻ, ഗോതമ്പ്, ബാർലി തുടങ്ങിയവയിൽ നിന്ന് ഉണ്ടാക്കപ്പെടുന്ന മധുര പാനീയങ്ങളാണ്. ഈന്തപ്പഴത്തിൽ നിന്നും മുന്തിരിയിൽ നിന്നും നബീദ് ഉണ്ടാക്കി എന്ന് പറഞ്ഞാൽ അതിനർത്ഥം ആ പഴങ്ങൾ വെള്ളത്തിൽ ഇട്ട് പഴച്ചാറ് എടുത്തു എന്നാണ്. നബീദ് എന്നാൽ ഈ പഴച്ചാറാണ്; അത് ലഹരി ഉള്ളതാവാം ഇല്ലാത്തതാവാം. രണ്ടിനേയും നബീദ് എന്ന് പറയും. പഴത്തിൽ നിന്ന് പിഴിഞ്ഞെടുത്ത (ജ്യൂസ് കൂടുതൽ കാലം സൂക്ഷിക്കുന്നതിലൂടെ ഉണ്ടാവുന്ന) ലഹരിയുള്ള നബീദിന് ഖംറ് (മദ്യം) എന്നാണ് വിളിക്കപ്പെടുക.” (അന്നിഹായ ഫീ ഗരീബിൽ ഹദീസ്: 5:7)

3. “നബ്ദ് (اﻟﻨَّﺒْﺬُ) എന്നാൽ ഇടുക (اﻟﻄَّﺮْﺡُ) എന്നാണ് അർത്ഥം. നബീദ് ലഹരി ബാധിക്കാത്തിടത്തോളം അനുവദനീയവും ലഹരി ബാധിച്ചാൽ നിഷിദ്ധവുമാകുന്നു. നബീദ് എന്ന പദം ഹദീസുകളിൽ ധാരാളമായി ആവർത്തിക്കപ്പെട്ടിട്ടുണ്ട്. നബീദ് എന്നാൽ ഈ പഴച്ചാറാണ്; അത് ലഹരി ഉള്ളതാവാം ഇല്ലാത്തതാവാം. രണ്ടിനേയും നബീദ് എന്ന് പറയും.” (താജുൽ ഉറൂസ്: 9: 480)

4. “നബ്ദ് (اﻟﻨَّﺒْﺬُ) എന്നാൽ ഒരു കാര്യം സന്നാഹങ്ങൾ ഒരുക്കാതെ ഇടുക എന്നാണ് അർത്ഥം… നബീദ്: ഈത്തപ്പഴം, ഉണക്കമുന്തിരി എന്നിവ വെള്ളം നിറച്ച പാത്രത്തിൽ ഇടുകയാണ്. പിന്നീടത് പ്രത്യേക പാനീയത്തിന്റെ നാമമായി പരിണമിച്ചു.” (അൽ മുഫ്റദാത്ത്: റാഗിബ് അൽ ഇസ്ബഹാനി: 788)

5. “ഈത്തപ്പഴം, ഉണക്കമുന്തിരി തുടങ്ങിയവ രുചി ലഭിക്കാനായി വെള്ളത്തിൽ ഇടുന്നതാണ് നബീദ്; എന്നാൽ മൂന്ന് ദിവസത്തിനപ്പുറം കടന്നുപോകരുത് എന്ന നിബന്ധനയോടെ. മൂന്ന് ദിവസത്തിനപ്പുറം കടന്നുപോയാൽ അത് നിഷിദ്ധമായ മദ്യമായി മാറുന്നു.” (അൽ മുഅ്ജമുൽ വസീത്വ്: പദം: നബദ نبذ)

6. الكلمة: النبيذ. الجذر: نبذ. الوزن: فَعِيل .[النبيذ]: المنبوذ .والنبيذ معروف، وهو من نبذ الشيءَ: إِذا ألقاه، لأنه يُلقى في الإِناء ثم يصب عليه الماء

“നബീദ് എന്നത് പ്രസിദ്ധമായ പദമാണ്. നബ്ദ എന്നാൽ ഒരു വസ്തു ദൂരെ ഇടുക എന്നാണ് അർത്ഥം… നബീദിന് ആ പേര് വരാൻ കാരണം പഴങ്ങൾ പാത്രത്തിൽ ഇടുകയും വെള്ളം ഒഴിക്കുകയും ചെയ്യലാണ്.” (ശംസുൽ ഉലൂം: നിശ്വാൻ ബിൻ സഈദ് അൽ ഹിംയരി: ഹിജ്റാബ്ദം: 573)

7. النبيذ: الشراب المتخذ من عصير التمر والعنب وغيرهما، ج أنبذة (ز) وهو نبيذ سواء كان مسكرا أو غير مسكر ويقال للخمر المعتصرة من العنب نبيذ. كما يقال للنبيذ خمر.

നബീദ്: (النبيذ) ഈത്തപ്പഴം മുന്തിരി പോലുള്ള പഴങ്ങളുടെ ചാറിൽ നിന്ന് ഉണ്ടാക്കുന്ന പാനീയം. നബീദ് എന്നതിന്റെ ബഹുവചനം അൻബിദ (أنبذة) എന്നാണ്. നബീദ് എന്നാൽ ഈ പഴച്ചാറാണ്; അത് ലഹരിയുള്ളതാകട്ടെ അല്ലാത്തതാകട്ടെ. മുന്തിരിയിൽ നിന്ന് ഉണ്ടാക്കപ്പെട്ട വൈനിനെ നബീദ് എന്ന് വിളിക്കപ്പെടാറുണ്ട്. (ലഹരിയുള്ള) നബീദിനെ ഖംറ് (മദ്യം) എന്നും വിളിക്കപ്പെടാറുണ്ട്. (മുഅ്ജമു മത്നുല്ലുഗ: അഹ്മദ് രിദാ)

8. “നബീദ്: ഈത്തപ്പഴം പോലെയുള്ള പഴങ്ങളിൽ നിന്ന് ഉണ്ടാക്കപ്പെടുന്ന എല്ലാ പാനീയത്തിന്റെയും പേരാണ് നബീദ്. ഈത്തപ്പഴ നബീദ് ഞാൻ നൽകി എന്ന ഹദീസിൽ ഉപയോഗിച്ച പദത്തിന്റെ അർത്ഥം ഈത്തപ്പഴം ഇട്ട വെള്ളമാണ്…” (അദുർറു നകിയ് ഫീ ശർഹി അൽഫാദിൽ ഖർകി: 1524: ഇബ്നുൽ മുബർരിദ്: ഹിജ്റാബ്ദം: 909)

9. نبيذ: 1- ملقى، منبوذ. 2- و 3- الشراب عموما.

“നബീദ്: ഇടപ്പെട്ടത്, നിക്ഷേപിക്കപ്പെട്ടത്. (പഴച്ചാറു കൊണ്ടുള്ള) പാനീയങ്ങളെ മൊത്തത്തിൽ നബീദ് എന്ന് പറയുന്നു.” (അർ റാഇദ്: ജിബ്റാൻ മസ്ഊദ്)

10. النبيذ: ما يُتَّخذ التمر والزبيب والعسلِ من غير غليان واشتداد.

“നബീദ്: ഈത്തപ്പഴം, ഉണക്കമുന്തിരി, തേൻ എന്നിവയിൽ നിന്ന് ഉണ്ടാക്കപ്പെടുന്ന -മത്തു വരികയോ ലഹരി ബാധിക്കുകയോ ചെയ്തിട്ടില്ലാത്ത- പാനീയമാണിത്.” (അത്തഅ്’രീഫാത്തുൽ ഫിക്ഹിയ്യ: മുഹമ്മദ് അമീം അൽ ഇഹ്സാൻ)

11. النبيذ: بفتح فسكون فعيل بمعنى مفعول، الملقى والمطروح. الماء الذي ينبذ فيه التمر أو الزبيب أو نحوهما ما لم ينقلب إلى مسكر، فإذا صار مسكرا فهو خمر… (Wine (of grapes, dates وعند الحنفية: الخمر هو النئ من ماء العنب إذا غلا واشتد وقذف بالزبد، وما عداه فهو نبيذ كله.

“നബ്ദ് (اﻟﻨَّﺒْﺬُ) എന്നാൽ ഒരു കാര്യം സന്നാഹങ്ങൾ ഒരുക്കാതെ ഇടുക എന്നാണ് അർത്ഥം… ഈത്തപ്പഴം, ഉണക്കമുന്തിരി തുടങ്ങിയവ ഇട്ട വെള്ളമാണ് നബീദ്; ലഹരിയുള്ളതായി പരിണമിക്കാത്തിടത്തോളം. ലഹരിയുള്ളതായി പരിണമിച്ചാൽ ഈ പഴച്ചാറിനെ ഖംറ് (മദ്യം Wine) എന്ന് വിളിക്കപ്പെടുന്നു.

ഹനഫി പണ്ഡിതർ പറയുന്നു: ഖംറ് എന്നാൽ മുന്തിരി ലഹരിയുള്ളതാവുകയും നുരയുള്ള കലർപ്പാവുകയും ചെയ്ത പാനീയമാണ്. ലഹരി ബാധിക്കാത്തവയാണ് നബീദ്.” (മുഅ്ജമു ലുഗത്തിൽ ഫുകഹാഅ്: മുഹമ്മദ് റവാസ് കൽഅ, ഹാമിദ് സ്വാദിക്)

ഇത്രയും ചർച്ച ചെയ്ത വിഷയത്തിന്റെ സംക്ഷിപ്തമായി ശൈഖുൽ ഇസ്‌ലാം ഇബ്നു തീമിയ്യയുടെ ഒരു വിശദീകരണം കൂടി ഉദ്ധരിച്ചു കൊണ്ട് നമ്മുക്ക് മറുപടിയിൽ നിന്ന് വിരമിക്കാം:

“നബീദ് അഥവാ പഴച്ചാറ് (النبيذ) കുടിക്കുന്നതിൽ ഇളവ് അനുവദിക്കുന്നതായി വന്നിട്ടുള്ള ചില നിവേദനങ്ങൾ ലഹരിയുള്ള പാനീയത്തെ സംബന്ധിച്ച് വന്നതാണെന്ന് ചിലർ തെറ്റിദ്ധരിക്കുകയുണ്ടായി. പഴങ്ങൾ വെള്ളത്തിൽ ഇട്ട് കുതിർത്ത് ലഹരി വരുന്നതിനു മുമ്പുള്ള അവസ്ഥയെയാണ് ‘നബീദ് ‘ കൊണ്ട് മുൻകാലക്കാർ ഉദ്ദേശിച്ചത്. അതല്ലാതെ പിൻകാലത്തോ, മറ്റു നാടുകളിലോ അറിയപ്പെടുന്നത് പോലെ ലഹരിയുള്ള വീഞ്ഞല്ല ‘നബീദ്’ എന്നതിന്റെ ഉദ്ദേശം.

ഇത് പോലെ മറ്റൊരു പദമാണ് ഖംറ് (الخمر) അഥവാ മദ്യം. പല പണ്ഡിതരും വിശ്വസിക്കുന്നത് പോലെ മുന്തിരിയിൽ നിന്നുമുള്ള മദ്യത്തെയല്ല ഇത് സൂചിപ്പിക്കുന്നത്; ഭാഷയിൽ അതാണ് അർത്ഥമെങ്കിലും. മത്തുണ്ടാകുന്ന എല്ലാ പാനീയങ്ങളെയും ആണ് ഖംറ് കൊണ്ട് ഉദ്ദേശിക്കുന്നത് എന്ന് ഹദീസുകൾ വിശദീകരിക്കുന്നു. (ബുഖാരി: 4619, മുസ്‌ലിം: 3032) (റഫ്ഉൽ മലാം അനിൽ അഇമ്മത്തിൽ അഅ്ലാം: 1:27)

സ്വന്തം നാട്ടിൽ നിന്നും മുഹമ്മദ് പുറത്താക്കിയതിന്റെ വിദ്വേഷത്താൽ ഖൈബറിലെ ജൂതന്മാർ മുഹമ്മദ് നബിയെ വിഷം കൊടുത്ത് കൊന്നതാണ്. മുഹമ്മദ് നബിയുടെ ക്രൂരത അസഹ്യമായതിനാലാണ് അപ്രകാരം അവർ ചെയ്തത്. (സ്വഹീഹുൽ ബുഖാരി) മറുപടി:

ഇസ്‌ലാം മതത്തെ സംബന്ധിച്ച പ്രാധമിക വിവരം പോലും വിമർശകർക്കില്ല എന്നതാണ് ഈ ഹദീസ് ദുർവ്യാഖ്യാനത്തിൽ അദിഷ്ടിതമായ കള്ള കഥ തെളിയിക്കുന്നത്. ഹദീസ് സ്വഹീഹ് (സ്വീകാര്യതയുടെ മാനദണ്ഡങ്ങൾ പൂർത്തിയായത്) ആണെന്നതിൽ സംശയമില്ല. പക്ഷെ പ്രവാചകൻ (സ) വിഷബാധയേറ്റാണ് മരിച്ചത് എന്നത് വിവരമില്ലായ്മയിൽ നിന്ന് ഉരുത്തിരിഞ്ഞ ഒരു അത്ഭുതാവഹമായ വാദമാണ്.

ഹിജ്റാബ്ദം ഏഴിനാണ് ഖൈബർ യുദ്ധം നടക്കുന്നത്.

ഇബ്നു ഇസ്ഹാക് (സീറത്തു ഇബ്നു ഇസ്ഹാക്), വാകിദി, ഇബ്നു സഅ്ദ് (ത്വബകാത്ത്) ഇബ്നു ഹജർ അൽ അസ്കലാനി (ഫത്ഹുൽ ബാരി) തുടങ്ങിയ ചരിത്ര പണ്ഡിതരും ഹദീസ് പണ്ഡിതരും ഇക്കാര്യം തെളിവു സഹിതം തങ്ങളുടെ ഗ്രന്ഥങ്ങളിൽ രേഖപ്പെടുത്തിയിട്ടുണ്ട്. (ഗസ്വത്ത് ഖൈബർ ദുറൂസ് വ ഇബർ: അമീർ ബിൻ മുഹമ്മദ് അൽ മുദ്‌രി : 8-9)

പ്രവാചകൻ (സ) മരണപ്പെടുന്നതാകട്ടെ ഹിജ്റാബ്ദം 11 ന് റബിഉൽ അവ്വൽ 12 നാണ്. അപ്പോൾ ഖൈബറിലെ ജൂത സ്ത്രീ ഒരുക്കിയ വിഷക്കെണിക്ക് ശേഷം നാലോ അഞ്ചോ വർഷം പ്രവാചകൻ (സ) വീണ്ടും ജീവിച്ചിട്ടുണ്ട്.

പ്രവാചകൻ (സ) മരണപ്പെടുന്നത് 63 വയസ്സിലാണ് (സ്വഹീഹുൽ ബുഖാരി: 4466)

പ്രവാചക ശിഷ്യൻ ഇബ്നു അബ്ബാസ് (റ) പറഞ്ഞു: അല്ലാഹുവിന്റെ ദൂതൻ (സ) നാൽപതാമത്തെ വയസ്സിൽ പ്രവാചകനായി നിയോഗിതനായി. ദിവ്യബോധനം നൽകപ്പെട്ടുകൊണ്ട് മക്കയിൽ 13 വർഷം താമസിച്ചു. ശേഷം മദീനയിലേക്ക് പലായനം ചെയ്യാൻ കൽപ്പിക്കപ്പെട്ടപ്പോൾ പത്ത് വർഷം മദീനയിൽ താമസിച്ചു. അറുപത്തിമൂന്നാമത്തെ വയസ്സിൽ മരണപ്പെട്ടു. (സ്വഹീഹുൽ ബുഖാരി: 3902, സ്വഹീഹു മുസ്‌ലിം: 2351)

ഈ പത്തു വർഷത്തെ മദീന ജീവിതത്തിൽ ഖൈബറിന് ശേഷം എത്രയെത്ര ചരിത്ര മൂഹൂർത്തങ്ങളിൽ പ്രവാചകൻ (സ) സന്നിഹിതനായി. മരണ സന്ദർഭത്തിൽ പ്രവാചകൻ (സ) ഇപ്രകാരം പറഞ്ഞതായി ചില ഹദീസുകളിൽ കാണാം:

مازالتْ أكْلَةُ خيبرَ تعاوِدُنِي كلَّ عامٍ ، حتى كان هذا أوانُ قطْعِ أَبْهَرِي

ഖൈബറിൽ കഴിച്ച വിഷം ‘എല്ലാ വർഷത്തിലും’ എനിക്ക് ബുദ്ധിമുട്ട് ഉണ്ടാക്കി കൊണ്ടിരുന്നു. എന്റെ രക്തധമനിയെ മുറിക്കുന്ന ഈ (മരണ) അവസരം എത്തിയതു വരെ. (സ്വഹീഹുൽ ജാമിഅ്: 5629, ത്വിബ്ബുന്നബി: അബൂ നുഐം: 83, ത്വിബ്ബുന്നബി: ഇബ്നു സുന്നി)

വിഷത്തിന്റെ സ്വാധീനത്താലുള്ള ചില ബുദ്ധിമുട്ടുകൾ ‘എല്ലാ വർഷത്തിലും’ പ്രവാചകന്(സ) അനുഭവപ്പെട്ടിരുന്നു എന്ന് മരണ സന്ദർഭത്തിൽ അദ്ദേഹം പറയുന്നതിൽ നിന്ന് എന്താണ് മനസ്സിലാവുന്നത്. ഖൈബർ വിഷബാധക്ക് ശേഷം ഒരുപാട് വർഷം അദ്ദേഹം ജീവിച്ചു എന്നു തന്നെ. (മരണ സന്ദർഭത്തിൽ ഈ വിഷത്തിന്റെ സ്വാധീന ഫലം ശക്തമായി അനുഭവപ്പെട്ടിരുന്നു എന്ന് മാത്രം)

തനിക്കു നൽകിയ ഭക്ഷണത്തിൽ ജൂതന്മാർ വിഷം പൂഴ്ത്തിയിരുന്നു എന്ന് ദിവ്യബോധനത്തിലൂടെ അറിഞ്ഞ മുഹമ്മദ് നബി (സ) ഖൈബറിലെ ജൂതന്മാരെ വിളിച്ചു കൂട്ടി നടത്തിയ പ്രഭാഷണമൊക്കെ സ്വഹീഹുൽ ബുഖാരിയിൽ തന്നെയുണ്ട് (ഹദീസ് നമ്പർ: 5441) പിന്നീട് എത്രയോ വർഷങ്ങൾ ജീവിച്ച് എത്രയോ പ്രഭാഷണങ്ങൾ അദ്ദേഹം നടത്തി. വിഷബാധയേറ്റാണ് അദ്ദേഹം മരിക്കുന്നതെങ്കിൽ അഞ്ച് വർഷം കഴിഞ്ഞാണോ മരിക്കുക ?! ഖൈബറിൽ നിന്ന് 180 കിലോമീറ്റർ ദൂരയുള്ള മദീനയിൽ, സ്വപത്നി ആഇശയുടെ വീട്ടിൽ, അവരുടെ മടിയിൽ കിടന്നാണ് പ്രവാചകൻ (സ) മരണപ്പെടുന്നത്. (സ്വഹീഹുൽ ബുഖാരി: 5959) പതിനാല് നൂറ്റാണ്ടുകൾക്കപ്പുറം ഊഷര മരുഭൂമിയിലൂടെ ഒട്ടകപ്പുറത്ത് സഞ്ചരിച്ചാൽ ഖൈബറിനും മദീനക്കുമിടയിൽ ദിവസങ്ങളോളം വരുന്ന യാത്രാ ദൂരം ഉണ്ട് എന്നതും ഖൈബറിലെ വിഷബാധയാണ് പ്രവാചകന്റെ (സ) മരണ കാരണമെന്ന വാദം മഹാവിഡ്ഢിത്തമാണെന്ന് തെളിയിക്കുന്നു.

ഇബ്നു മുഫ്‌ലിഹ് പറഞ്ഞു: പ്രവാചകനോടൊപ്പം ആട്ടിൻ മാംസം കഴിച്ച ചിലർ വിഷത്താൽ മരിക്കുകയുണ്ടായി. മുഹമ്മദ് പ്രവാചകൻ അല്ലെങ്കിൽ വിഷത്താൽ മരിക്കട്ടെ എന്നായിരുന്നു ജൂതന്മാരുടെ ഉദ്ദേശം. എന്നാൽ പ്രവാചകൻ (സ) വർഷങ്ങളോളം ജീവിച്ചു… ജൂതൻമാർക്ക് മറ്റുള്ളവരെ കൊല്ലാൻ കഴിഞ്ഞതു പോലെ പ്രവാചകനെ കൊല്ലാൻ കഴിഞ്ഞില്ല. (അൽ ആദാബു ശറഇയ്യ: 3:190-205)

ഇബ്നു കസീർ പറഞ്ഞു: പ്രവാചകന് (സ) അല്ലാഹു നൽകിയ സംരക്ഷണത്തിൽ പെട്ടതാണ്… ജൂതന്മാർ ഖൈബറിൽ ആടിൻ മാംസത്തിൽ വിഷം തേച്ച് നൽകിയപ്പോൾ അല്ലാഹു ആ വിഷത്തെ പറ്റി അദ്ദേഹത്തിന് അറിയിച്ചു കൊടുക്കുകയും അദ്ദേഹത്തിന് അതിൽ നിന്ന് സംരക്ഷണം നൽകുകയും ചെയ്തു എന്നത്… (തഫ്സീറു ഇബ്നു കസീർ: 2:81)

ശക്തമായ പനിയായിരുന്നു പ്രവാചകന്റെ മരണരോഗം. ഇത് ഒരുപാടു ഹദീസുകളിലൂടെ പ്രവാചകാനുചരന്മാർ പങ്കു വെച്ച വിവരമാണ്. (സ്വഹീഹുൽ ബുഖാരി: 198, 4437, 4436, 5660, സ്വഹീഹു മുസ്‌ലിം: 418) ആ രോഗാവസ്ഥയിൽ വർഷങ്ങൾക്ക് മുമ്പ് ജൂത വഞ്ചനയുടെ ഭാഗമായി കഴിച്ച വിഷത്തിന്റെ സ്വാധീന ഫലം ശക്തമായി അനുഭവപ്പെട്ടിരുന്നു എന്നത് കൊണ്ട് അക്കാരണത്താലാണ് മരിച്ചത് എന്ന് പറയുന്നത് എന്തൊരു വിഡ്ഢിത്തമാണ്. വിഷം കഴിച്ച് ആത്മഹത്യക്ക് ശ്രമിക്കുകയും രക്ഷിക്കപ്പെടുകയും ചെയ്തവർക്ക് ആ വിഷത്തിന്റെ സ്വാധീനഫലങ്ങൾ ജീവിതത്തിൽ പല ഘട്ടങ്ങളിലുമായി അനുഭവപ്പെടാറുള്ളത് ഒരു പൊതു വിവരമാണ്. ഇതു തന്നെയാണ് പ്രവാചകനും അനുഭവപ്പെട്ടത്. ഈ വേദനാനുഭവം പങ്കുവെക്കുക മാത്രമാണ് പ്രവാചകൻ (സ) ചെയ്തത്. അല്ലാതെ വിഷബാധയായിരുന്നു മരണ കാരണമെങ്കിൽ അഞ്ച് വർഷം കഴിഞ്ഞല്ല മരണപ്പെടുക എന്ന് മനസ്സിലാക്കാൻ സെമിനാരിയിൽ പഠിച്ചവർക്ക്‌ പ്രയാസമുണ്ടാകാം. ഖൈബറിലെ ജൂത സ്ത്രീ നൽകിയ വിഷം പ്രവാചകനോടൊപ്പം കഴിച്ച അനുചരൻ ബദ്ർ ഇബ്നു ബറാഅ് (റ) ഉടനെ മരണപ്പെടുകയാണുണ്ടായത് എന്നും ഓർക്കുക. മരണ വേളയിൽ ഒരു രോഗി അസഹ്യമായ തലവേദന അനുഭവിച്ചാൽ തലവേദനയാൽ മരിച്ചു എന്ന് ആരെങ്കിലും വാദിക്കുമോ ?! ജീവിതത്തിൽ ഏതോ ദശയിൽ ഒരു അപകടത്തിൽ പരിക്കേറ്റ വ്യക്തിക്ക് മരണ സമയത്ത് പരിക്കുണ്ടായിരുന്ന ഭാഗത്ത് വേദന അനുഭവപ്പെട്ടാൽ അപകടത്തിൽ മരിച്ചു എന്ന് വ്യാഖ്യാനിക്കാമോ?

സ്വഹീഹുൽ ബുഖാരിയിലെ ഹദീസിലേക്ക് തിരിച്ചു വരാം. സ്വഹീഹുൽ ബുഖാരിയിലെ ഹദീസ് പ്രവാചകന്റെ ക്രൂരതയെ തെളിയിക്കുന്നു എന്നതാണ് അടുത്ത വാദം. ഇതും ശുദ്ധമായ നുണയാണെന്നതിന് പുറമെ പ്രവാചകന്റെ കാരുണ്യമാണ് ഹദീസിൽ സ്ഫുരിക്കുന്നത് എന്നതാണ് യാതാർത്ഥ്യം.

أنَّ امرأةً يَهوديَّةً أتت رسولَ اللَّهِ صلَّى اللَّهُ عليْهِ وسلَّمَ بشاةٍ مسمومةٍ فأَكلَ منْها فجيءَ بِها إلى رسولِ اللَّهِ صلَّى اللَّهُ عليْهِ وسلَّمَ فسألَها عن ذلِكَ ؟ فقالت : أردتُ لأقتلَكَ قالَ : ما كانَ اللَّهُ ليسلِّطَكِ على ذلكَ أو قالَ عليَّ فقالوا ألا نقتلُها قالَ لا قالَ أنس فما زلتُ أعرِفُها في لَهواتِ رسولِ اللَّهِ صلَّى اللَّهُ عليْهِ وسلَّمَ

ഒരു ജൂത സ്ത്രീ അല്ലാഹുവിന്റെ ദൂതന് വിഷം തേച്ച ആട് (സമ്മാനമായി) നൽകുകയുണ്ടായി. അദ്ദേഹം അതിൽ നിന്ന് ഭക്ഷിച്ചു. പിന്നീട് (ജൂത സ്ത്രീ വിഷം തേച്ചതിനെ സംബന്ധിച്ച് അല്ലാഹു അദ്ദേഹത്തെ അറിയച്ചപ്പോൾ) ആ സ്ത്രീയെ അല്ലാഹുവിന്റെ ദൂതന്റെ അടുക്കൽ കൊണ്ട് വരപ്പെട്ടു. അദ്ദേഹം അതിനെ പറ്റി സ്ത്രീയോട് ചോദിച്ചപ്പോൾ അവർ പറഞ്ഞു: താങ്കളെ കൊല്ലാനാണ് ഞാൻ ഉദ്ദേശിച്ചത്. പ്രവാചകൻ (സ) പറഞ്ഞു: അല്ലാഹു അതിനുള്ള പരമാധികാരം നിനക്ക് നൽകില്ല. പ്രവാചകാനുചരന്മാർ ചോദിച്ചു: ഞങ്ങൾ അവൾക്ക് വധശിക്ഷ നൽകട്ടെ ? പ്രവാചകൻ (സ) പറഞ്ഞു: അരുത്. അനസ് പറഞ്ഞു: ആ വിഷത്തിന്റെ അടയാളം പ്രവാചകന്റെ തൊണ്ട ഭാഗങ്ങളിൽ ഞാൻ മനസ്സിലാക്കി കൊണ്ടിരുന്നു. (സ്വഹീഹുൽ ബുഖാരി: 2617, സ്വഹീഹു മുസ്‌ലിം: 2190)

പ്രവാചകന്റെ കാരുണ്യവും അന്നത്തെ ജൂതരുടെ ശത്രുതയും ഈ സംഭവത്തിൽ നിന്നും സുതരാം വ്യക്തമാണ്. തന്നെ വധിക്കാൻ ശ്രമിച്ച സ്ത്രീക്ക് വധശിക്ഷ നൽകട്ടെ എന്ന് ചോദിച്ച അനുചരന്മാരോട് അരുത് എന്നു പറഞ്ഞ ഈ മഹാ മനീഷിയുടെ ഹൃദയ വിശാലതയെയാണ് ഹദീസ് തെളിയിക്കുന്നത്. മത വർഗീയതയില്ലാതെ ജൂത സ്ത്രീ ഒരുക്കിയ വിരുന്നിൽ പങ്കെടുത്ത സഹിഷ്ണുതയെയാണ് ഹദീസ് പരിചയപ്പെടുത്തുന്നത്. മരണം വരെ ജീവിതത്തിന്റെ ഓരോ സന്തിയിലും ജൂത വിഷത്തിന്റെ സ്വാധീനം അനുഭവിച്ചിട്ടും ജൂതന്മാരോട് പക വെച്ചു പുലർത്താതെ… വിജയിച്ചടക്കിയ ഖൈബർ ഭൂമി ജൂതന്മാരുടെ അപേക്ഷ പ്രകാരം അവർക്കു തന്നെ തിരിച്ചു കൊടുത്ത ദയാപരനായ പ്രവാചകന്റെ (സ) ചിത്രമാണ് ഹദീസിൽ നാം ദർശിക്കുന്നത്. വിഷം നൽകിയ സ്ത്രീയുടെ സമൂഹം പ്രവാചകന്റെ മരണം വരെ ഖൈബറിൽ ജീവിച്ചു !!

സ്വഹീഹുൽ ബുഖാരിയിലെ ഹദീസ് പ്രവാചകന്റെ ക്രൂരതയെ തെളിയിക്കുന്നു എന്ന വാദം വിദ്വേഷ പ്രചാരണത്തിന്റെ മിഷണറി സ്റ്റൈൽ മാത്രമാണ്. അഥവാ, പരസ്പരം ബന്ധമില്ലാത്ത കാര്യങ്ങളെ ദുർവ്യാഖ്യാനിച്ച് ബന്ധം മെനഞ്ഞെടുക്കുന്ന മിഷനറി കുബുദ്ധിയുടെ സൃഷ്ടിപ്പാണ് ഈ ദുഷ്പ്രചാരണം. ഇത്തരം ഇല്ലാത്ത ബന്ധങ്ങൾ ദുർവ്യാഖ്യാനിച്ച് ഉണ്ടാക്കാനായിരുന്നെങ്കിൽ അതിന് ഏറ്റവും എളുപ്പം വഴങ്ങുന്ന ഗ്രന്ഥം ബൈബിളാണെന്നതിൽ ആർക്കാണ് സംശയം? യേശുവെ കുരിശിൽ തറച്ചത് യേശുവിന്റെ ക്രൂരമായ ദ്രോഹങ്ങൾ സഹിക്കാൻ കഴിയാത്തത് കൊണ്ടാണ് ജൂതന്മാർ കൊന്നത് എന്ന് വാദിച്ചാൽ എന്തു മറുപടിയാണ് ഈ മിഷണറി ദുർവ്യാഖ്യാനക്കാർക്ക് മറുപടി പറയാനുണ്ടാവുക.?!

പരീശകളെ “കപടവിശ്വാസികളിൽ ഏറ്റവും മോശം” (ബൈബിൾ: മത്തായി : 23:4-36) ജനതയെന്നും ജൂതന്മാരെ “പിശാചിന്റെ മക്കൾ” (ബൈബിൾ: യോഹന്നാന്‍: 8:44) എന്നും അവമതിക്കുകയും ദേവാലയത്തിലെ കച്ചവടക്കാരേയും അവരുടെ ആടുകളേയും കന്നുകാലികളേയും ചാട്ടവാറുകൊണ്ട് അടിച്ച് പുറത്താക്കുകയും നാണയങ്ങൾ തട്ടിതെറിപ്പിക്കുകയും ചെയ്ത (ബൈബിൾ: യോഹന്നാന്‍: 2:15) യേശുവിന്റെ ദ്രോഹങ്ങൾക്കുള്ള മറുപടിയായിരുന്നു ക്രൂശീകരണം എന്നാണ് ജൂതന്മാർക്ക് പറയാനുള്ളത്. ഈ വാദത്തിന് മിഷണറിമാർക്ക് എന്താണോ മറുപടി പറയാനുള്ളത് അതു തന്നെയാണ് മുസ്‌ലിംകൾക്ക് ഖൈബറിൽ പ്രവാചകനെ വിഷം കൊടുത്തു വധിക്കാൻ ശ്രമിച്ച ജൂതന്മാരുടെ കാര്യത്തിലും പറയാനുള്ളത്. (ഇസ്‌ലാം പഠിപ്പിച്ച നീതി ബോധവും സത്യസന്ധതയും ശിരസാവഹിക്കുന്ന മുസ്‌ലിംകളെ സംബന്ധിച്ചിടത്തോളം ബൈബിളിൽ യേശുക്രിസ്തു ചെയ്തെന്നും പഠിപ്പിച്ചെന്നും പറയുന്ന ശുദ്ധ നുണകളും അദ്ദേഹം ക്രൂശീകരിക്കപ്പെട്ടുവെന്ന പെരും നുണയും വിശ്വാസയോഗ്യമല്ല എന്നത് സാന്ദർഭികമായി ഉണർത്തട്ടെ.)

പ്രവാചകന്മാരെ കൊല്ലാനുള്ള ജൂത പ്രവണതയെ സംബന്ധിച്ച് കുർആൻ പലയിടത്തും സംസാരിച്ചിട്ടുണ്ട് (കുർആൻ:4:155, 3:181). ഇത് ആ ജനതയുടെ സ്വഭാവ വൈകൃതത്തിന്റേയും വഞ്ചനാത്മകതയുടേയും തെളിവാണ്. (ഇതിന് അപവാദമായ ഒരു വിഭാഗം അവരിലുണ്ട് എന്നത് വിസ്മരിക്കുന്നില്ല).

മുഹമ്മദ് നബിയുടെ(സ) ക്രൂരതയാണ് അവരെ അതിന് പ്രേരിപ്പിച്ചതെങ്കിൽ ബൈബിൾ തന്നെ കഥ പറഞ്ഞു തരുന്ന പ്രകാരം പ്രവാചകന്മാരായ ആബേൽ, സക്കരിയ്യ, യോസേഫ്, യേശു എന്നിവരെയുമൊക്കെ ജൂതന്മാർ വധിക്കാൻ ശ്രമിച്ചതിലുള്ള പ്രേരണ എന്തായിരുന്നു, അച്ചന്മാരേ ?! ‘പ്രവാചകന്മാരെ കൊല്ലുന്ന ധിക്കാരികളെന്ന്’ ജൂതന്മാരെ സംബന്ധിച്ച് യേശു ക്രിസ്തു തന്നെ പഠിപ്പിച്ചതിന് എന്ത് മറുപടിയുണ്ട് ?!! (ബൈബിൾ: മത്തായി 23:23-31, ലൂക്ക 11:47)

ഖൈബറിലെ ജൂത വിഷം ബാധിച്ചാണ് മുഹമ്മദ് നബി (സ) മരണപ്പെട്ടത് എന്ന വാദത്തിന് തെളിവായ ദുർവ്യാഖ്യാനിക്കപ്പെടുന്ന മറ്റൊരു ഹദീസാണ് ഇത്:

പ്രവാചകൻ (സ) തന്റെ മരണ കാരണമായ രോഗത്തിന്റെ അവസരത്തിൽ പറയുമായിരുന്നു: ആഇശാ, ഖൈബറിൽ ഞാൻ കഴിച്ച ഭക്ഷണത്തിന്റെ വേദന എനിക്ക് ഇപ്പോൾ അനുഭവപ്പെടുന്നു. ഈ ആവസരത്തിൽ ആ വിഷത്തിൽ നിന്ന് എന്റെ രക്തധമനിയെ മുറിക്കുന്ന വേദന എനിക്ക് അനുഭവപ്പെടുന്നു. (സ്വഹീഹുൽ ബുഖാരി: 4428)

മുമ്പ് വിശദീകരിച്ചത് പോലെ ഖൈബറിൽ ജൂത സ്ത്രീ വിഷം നൽകിയതിന് ശേഷം ചുരുങ്ങിയത് നാല് വർഷം പ്രവാചകൻ (സ) ജീവിച്ചിട്ടുണ്ട്. ജൂത സ്ത്രീ നൽകിയ വിഷത്താൽ മരിക്കുന്നതിൽ നിന്ന് അല്ലാഹു പ്രവാചകന് സംരക്ഷണമേകി.

“ദൈവദൂതരേ, നിന്റെ നാഥനില്‍നിന്ന് നിനക്ക് ഇറക്കിക്കിട്ടിയത് ജനങ്ങള്‍ക്കെത്തിച്ചുകൊടുക്കുക… ജനങ്ങളില്‍നിന്ന് അല്ലാഹു നിന്നെ സംരക്ഷിക്കുന്നതാണ്.” (കുർആൻ: 5:67) എന്ന അല്ലാഹുവിന്റെ വാഗ്ദാനമാണ് ഇതിലൂടെ പുലർന്നത്. മുഹമ്മദ് നബി (സ) അല്ലാഹുവിന്റെ ദൂതനാണെന്നാണ് ഈ മുഅ്ജിസത്ത് (അമാനുഷിക ദൃഷ്ടാന്തം) തെളിയിക്കുന്നത്.

“അവർ പറഞ്ഞു: താങ്കളെ കൊല്ലാനാണ് ഞാൻ ഉദ്ദേശിച്ചത്. പ്രവാചകൻ (സ) പറഞ്ഞു: അല്ലാഹു അതിനുള്ള പരമാധികാരം നിനക്ക് നൽകില്ല.” എന്ന ഹദീസിന്റെ ഭാഗത്ത് നിന്ന് മനസ്സിലാക്കാവുന്നത് ജനങ്ങളുടെ ഉപദ്രവത്തിൽ നിന്ന് പ്രവാചന് നൽകപ്പെട്ടിട്ടുള്ള ദിവ്യ സംരക്ഷണമാണ്.

മരണ കാരണമായ വിഷത്തിൽ നിന്ന് സംരക്ഷണം നൽകപ്പെട്ടത് പ്രവാചകന്റെ മുഅ്ജിസത്ത് ആകുന്നു. (ശർഹു മുസ്‌ലിം: 14: 34)

ജൂത സ്ത്രീ നൽകിയ വിഷത്താൽ മരിക്കുന്നതിൽ നിന്ന് അല്ലാഹു പ്രവാചകന് സംരക്ഷണമേകിയെങ്കിലും അതിന്റെ സ്വാധീനത്താലുള്ള ചില ബുദ്ധിമുട്ടുകൾ ചില സന്ദർഭങ്ങളിൽ പ്രവാചകന് അനുഭവപ്പെട്ടിരുന്നു. മരണ സമയത്ത് ഇത് ശക്തമായി അനുഭവപ്പെട്ടു എന്നു മാത്രമാണ് ഈ ഹദീസ് സൂചിപ്പിക്കുന്നത്. അതല്ലാതെ വിഷബാധയേറ്റ് മരിച്ചു എന്നല്ല. മുമ്പ് സൂചിപ്പിച്ചത് പോലെ, ജീവിതത്തിൽ ഏതോ ദശയിൽ ഒരു അപകടത്തിൽ പരിക്കേറ്റ വ്യക്തിക്ക് മരണ സമയത്ത് പരിക്കുണ്ടായിരുന്ന ഭാഗത്ത് വേദന അനുഭവപ്പെട്ടാൽ അപകടത്തിൽ മരിച്ചു എന്ന് വ്യാഖ്യാനിക്കാമോ?

ജൂതന്മാരെ തങ്ങളുടെ നാടായ മദീനയിൽ നിന്ന് പുറത്താക്കിയത് മുഹമ്മദ് നബി (സ) ചെയ്ത ക്രൂരതയല്ലെ എന്നതാണ് അടുത്ത വിമർശനം. ഇന്ന്, ഫലസ്തീനിലെ നാലു വയസ്സുകാരൻ തങ്ങളുടെ ഒരു പട്ടാളക്കാരനെ നോക്കി ഗോഷ്ടി കാണിച്ചു എന്ന ‘ന്യായീകരണവും’ പറഞ്ഞ് അധിനിവേശ രാഷ്ട്രങ്ങളിൽ നിന്ന് ഇരന്നുവാങ്ങിയ മിസൈലുകൾ എൽ.പി. സ്കൂൾ കുട്ടികളുടെ തലയിലേക്ക് വാരിയെറിയുന്ന ജൂത ക്രൂതകളുടെ സമകാലിക ചിത്രം ലോകത്തിന് മുമ്പിലുണ്ട്. അതുകൊണ്ട് തന്നെ, ജൂതന്മാരെ ഹിജാസിൽ നിന്ന് നാടുകടത്തിയതിന്റെ കാരണം മനസ്സിലാക്കി കൊടുക്കാൻ അധികം കഷ്ടപ്പെടേണ്ടതൊന്നുമില്ല.

ജൂതന്മാരെ മദീനയിൽ നിന്ന് പുറത്താക്കാനും ഖൈബർ യുദ്ധം നടത്താനുമുള്ള ചരിത്ര കാരണങ്ങൾ ഹ്രസ്വമായി ഇവിടെ വിവരിക്കാം:

പ്രവാചകൻ (സ) മദീനയിൽ എത്തിയപ്പോൾ മക്കയിൽ നിന്ന് പലായനം ചെയ്തുവന്നവർക്കും (മുഹാജിറുകൾ) മദീനയിലെ മുസ്‌ലിംകൾക്കും (അൻസ്വാറുകൾ) ഇടയിലും ഒരു കരാറുണ്ടാക്കി. മദീനയിലുണ്ടായിരുന്ന ജൂത ഗോത്രങ്ങളെയും ആ കരാറിൽ ഉൾപ്പെടുത്തി. (സീറത്തു ഇബ്നു ഹിശാം: 2/ 147, അൽ അംവാൽ: 1/307, അൻ സാബുൽ അശ്റാഫ്: 1/286, സീറത്തു ഇബ്നു ഇസ്ഹാഖ്:)

ജൂതന്മാർക്ക് അവരുടെ മതവും മുസ്‌ലിംകൾക്ക് അവരുടെ മതവുമനുസരിച്ച് ജീവിക്കാം; അക്രമം പ്രവർത്തിച്ചവരും കുറ്റവാളികളും ഒഴികെ. ഈ കരാറുകാർക്കെതിരെ യുദ്ധം ചെയ്യുന്നവരെ പ്രതിരോധിക്കാൻ കരാറിലേർപ്പെട്ടവർ പരസ്പരം സഹായിക്കണം. കരാറിലുള്ള ഇരു കക്ഷികൾക്കുമെതിരെ (മുസ്‌ലിംകളും ജൂതന്മാരും) പുറത്തു നിന്നും ആരെങ്കിലും യുദ്ധം ചെയ്യുകയാണെങ്കിൽ ശത്രു പക്ഷത്തെ സഹായിക്കരുത്, പരസ്പരം സഹായിക്കണം. പരസ്പരം ഗുണകാംക്ഷയും നന്മയും വെച്ചുപുലർത്തണം. ചരിത്രകാരന്മാരായ ഇബ്നു ഇസ്ഹാഖ് (സീറത്തു ഇബ്നു ഇസ്ഹാഖ്), ഇബ്നു ഹിശാം (സീറത്തു ഇബ്നു ഹിശാം: 2/ 147), വാഖിദി (മഗാസി: 1/176), തബ്‌രി (താരീഖു തബ്‌രി: 2/479), ഇബ്നുൽ അസീർ (അൽ കാമിൽ: 2/96), കാസിം ഇബ്നു സലാം (അൽ അംവാൽ: 1/446), ബലാദുരി (അൻസാബുൽ അശ്റാഫ്: 1/286) തുടങ്ങിയവർ ഈ സമാധാന കരാറിനെ സംബന്ധിച്ച് വിശദീകരിക്കുന്നുണ്ട്.

‘മദീനാ പ്രമാണം’ (وثيقة المدينة) എന്ന പേരിൽ ചരിത്ര പ്രസിദ്ധമായ ഈ കരാർ ഇസ്‌ലാമിക രാഷ്ട്രത്തിന്റെ ആദ്യ ഭരണഘടനയായി പരിചയപ്പെടുത്തപ്പെടുന്നു.

ഇബ്നുൽ ഖയ്യിം (റ) പറയുന്നു: അല്ലാഹുവിന്റെ ദൂതൻ മദീനയിലുണ്ടായിരുന്ന ജൂതന്മാരുമായി സമാധാന സന്ധിയുണ്ടാക്കി. ഇരു കൂട്ടർക്കുമിടയിൽ ഒരു കരാർ പത്രം എഴുതുകയും ചെയ്തു. ജൂതന്മാരുടെ കാരണവരും വേദശാസ്ത്ര പണ്ഡിതനുമായ അബ്ദുല്ല ഇബ്നുസ്സലാം ഈ കരാറിന് മുൻകൈയെടുത്തു. അദ്ദേഹം പിന്നീട് ഇസ്‌ലാം സ്വീകരിച്ചു, സത്യനിഷേധമല്ലാതെ മറ്റു ജുതന്മാരെ ഇസ്‌ലാം സ്വീകരിക്കുന്നതിൽ നിന്ന് തടഞ്ഞില്ല. അവർ മൂന്ന് ഗോത്രങ്ങളായിരുന്നു: ബനൂ ഖൈനുകാഅ്, ബനൂ നളീർ, ബനൂ ഖുറൈള. മൂന്നു കൂട്ടരും മുസ്‌ലിംകൾക്കെതിരെ കലാപമുണ്ടാക്കി, ബനൂ ഖൈനുകാഅ് ഗോത്രത്തോട് പ്രവാചകൻ (സ) അനുകമ്പ കാണിച്ചു, ബനൂ നളീറുകാരെ മദീനയിൽ നിന്ന് ഒഴിപ്പിച്ചു, ബനൂ ഖുറൈളക്കാരോട് ആദ്യം കാരുണ്യം കാണിച്ചെങ്കിലും അവർ യുദ്ധം ചെയ്തപ്പോൾ അവരിലെ പുരുഷന്മാരെ വധിച്ചു. സൂറത്തുൽ ഹശ്ർ അവതരിപ്പിക്കപ്പെട്ടത് ബനൂ നളീറുകാരുടെ വിഷയത്തിലും സൂറത്തുൽ അഹ്സാബ് അവതരിപ്പിക്കപ്പെട്ടത് ബനൂ ഖുറൈളക്കാരുടെ വിഷയത്തിലുമായിരുന്നു. (സാദുൽ മആദ്: 3/58,59)

ജർമ്മൻ ബൈബിൾ പണ്ഡിതനും രേഖാപരികൽപ്പനയുടെ ഉപജ്ഞാതാക്കളിൽ ഒരാളും (documentary hypothesis) ഓറിയന്റലിസ്റ്റുമായ ജൂലിയസ് വെൽ‌ഹൗസെൻ തന്റെ ‘Arab kingdom and it’s fall’ എന്ന ഗ്രന്ഥത്തിൽ (പേജ്: 11) പറയുന്നു:

“മക്കയിൽ നിന്നും മദീനയിലേക്ക് പാലായനം ചെയ്ത ശേഷമുള്ള ആദ്യ ഘട്ടത്തിൽ, ബദർ യുദ്ധത്തിന് മുമ്പായി, മുഹമ്മദ് ഉണ്ടാക്കിയ കരാറിന്റെ രേഖ, കാലം നമുക്കായി സൂക്ഷിക്കുകയുണ്ടായി. മദീനയിലെ ആദ്യ ഘട്ടത്തിലെ പൊതു ജീവിതത്തേയും രാഷ്ട്രീയത്തേയും നിയന്ത്രിച്ചിരുന്നതും സമൂഹത്തിൽ പ്രയോഗവൽകരിച്ചിരുന്നതുമായ നിയമത്തിലെ ചില പ്രധാന പോയിന്റുകൾ പ്രസ്തുത രേഖയിൽ വ്യക്തമാക്കപ്പെടുന്നുണ്ട്… മദീന എങ്ങനെ അഖണ്ഡതയുള്ള ഒരൊറ്റ സമൂഹമായി മാറി എന്ന് അതിൽ നിന്നും മനസ്സിലാക്കാം.” (അവലംബം: ഗ്രന്ഥത്തിന്റെ അറബി പരിഭാഷ)

ഓറിയന്റലിസ്റ്റായ ഡോ. ഇസ്രായേൽ ബെൻസീവ് (History of jews in Arabia: Pre-Islam and Early Islam: 114,115) തെളിവുകൾ നിരത്തി ഈ കരാർ പത്രത്തെ ചരിത്ര വസ്തുതയാണെന്ന് സമർത്ഥിക്കുന്നു. ഇറ്റാലിയൻ ഓറിയന്റലിസ്റ്റായ ലിയോൺ കൈതാനിയും ഈ സമാധാന സന്ധിയുടെ സത്യതയെ അംഗീകരിക്കുന്നുണ്ട്.

മദീനയുടെ വലിയൊരു ഭാഗം യുദ്ധവും കൊലപാതകവും നിഷിദ്ധമാക്കപ്പെട്ട പരിശുദ്ധ ഭൂമിയായി പ്രവാചകൻ (സ) പ്രഖ്യാപിച്ചു.

ജാബിർ (റ) നിവേദനം അല്ലാഹുവിന്റെ ദൂതൻ (സ) പറഞ്ഞു: തീർച്ചയായും ഇബ്രാഹീം മക്കയെ പവിത്രമായി പ്രഖ്യാപിച്ചു; നിശ്ചയം മദീനയെ -രണ്ട് പർവതങ്ങൾക്കിടയിലുള്ള പ്രദേശത്തെ – പവിത്രമായി ഞാനിതാ പ്രഖ്യാപിക്കുന്നു. അവിടെയുള്ള ഒരു വൃക്ഷവും (അനാവശ്യമായി) മുറിക്കപ്പെടരുത്, ഒരു ഉരുവും വേട്ടയാടപ്പെടരുത്.” (സ്വഹീഹുൽ ബുഖാരി: 2889, സ്വഹീഹു മുസ്‌ലിം: 3383, 459, 1363)

ജൂതന്മാരോട് പ്രവാചകൻ (സ) ദീക്ഷിച്ച കാരുണ്യവും നീതിയും, തിരിച്ച് യഹൂദർ ചെയ്തിരുന്ന ദ്രോഹങ്ങളും ശത്രുതയും ചരിത്രത്തിൽ നിന്നും ഹദീസുകളിൽ നിന്നും വായിച്ചെടുക്കാവുന്നതാണ്:

ഖൈബറുകാർ അബ്ദുല്ലാഹിബ്നു സഹ്‌ലിനെ (റ) വധിച്ചപ്പോൾ വ്യക്തമായ തെളിവില്ലാത്തതിനാൽ കൊലയാളികളായ ജൂതരെ പ്രവാചകൻ വെറുതെ വിട്ടു. (സ്വഹീഹുൽ ബുഖാരി: 6769, സ്വഹീഹു മുസ്‌ലിം: 1669)

മുസ്‌ലിമായ അശ്അസ് ഇബ്നു ഖൈസും ഒരു ജൂതനും തമ്മിൽ യമനിലെ ഒരു ഭൂമിയുടെ കാര്യത്തിൽ തർക്കമുണ്ടായി പ്രവാചക സന്നിധിയിൽ വിധിക്കായി വന്നപ്പോൾ അശ്അസിന് തെളിവ് ഹാജരാക്കാൻ കഴിയാഞ്ഞതിനാൽ ഭൂമി ജൂതന്റേതായി പ്രവാചകൻ (സ) വിധിച്ചു. (സ്വഹീഹുൽ ബുഖാരി: 2525, സ്വഹീഹു മുസ്‌ലിം: 138)

ജൂതന്മാർക്ക് അവർക്കിടയിലെ വ്യക്തിപരവും സാമൂഹികവും മതപരവുമായ കാര്യങ്ങളിൽ അവരുടെ മതവും വേദവും വെച്ച് തന്നെ വിധി പറയാനുള്ള സ്വാതന്ത്ര്യം മദീനയിൽ നടപ്പാക്കി.

“…അവര്‍ നിന്‍റെ അടുത്ത് വരുകയാണെങ്കില്‍ അവര്‍ക്കിടയില്‍ നീ തീര്‍പ്പുകല്‍പിക്കുകയോ, അവരെ അവഗണിച്ച് കളയുകയോ ചെയ്യുക. നീ അവരെ അവഗണിച്ച് കളയുന്ന പക്ഷം അവര്‍ നിനക്ക് ഒരു ദോഷവും വരുത്തുകയില്ല. എന്നാല്‍ നീ തീര്‍പ്പുകല്‍പിക്കുകയാണെങ്കില്‍ അവര്‍ക്കിടയില്‍ നീതിപൂര്‍വ്വം തീര്‍പ്പുകല്‍പിക്കുക. നീതിപാലിക്കുന്നവരെ തീര്‍ച്ചയായും അല്ലാഹു സ്നേഹിക്കുന്നു.” (ഖുർആൻ: 5:42)

സ്നേഹ സംവാദങ്ങളുടേയും മത സഹവർത്തിത്വത്തിന്റേയും മാർഗം ഇസ്‌ലാം വേദക്കാർക്ക് മുമ്പിൽ മലർക്കെ തുറന്നിട്ടു: “വേദക്കാരോട് ഏറ്റവും നല്ല രീതിയിലല്ലാതെ നിങ്ങള്‍ സംവാദം നടത്തരുത്‌- അവരില്‍ നിന്ന് അക്രമം പ്രവര്‍ത്തിച്ചവരോടൊഴികെ. നിങ്ങള്‍ (അവരോട്‌) പറയുക: ഞങ്ങള്‍ക്ക് അവതരിപ്പിക്കപ്പെട്ടതിലും നിങ്ങള്‍ക്ക് അവതരിപ്പിക്കപ്പെട്ടതിലും ഞങ്ങള്‍ വിശ്വസിച്ചിരിക്കുന്നു. ഞങ്ങളുടെ ദൈവവും നിങ്ങളുടെ ദൈവവും ഒരുവനാകുന്നു. ഞങ്ങള്‍ അവന് കീഴ്പെട്ടവരുമാകുന്നു.” (ഖുർആൻ: 29:46)

അവർക്ക് ധനം സമ്പാദിക്കാനും ഉടമപ്പെടുത്താനുമുള്ള പൂർണമായ സ്വാതന്ത്ര്യം നൽകി. ആഇശ (റ) പറഞ്ഞു: അല്ലാഹുവിന്റെ ദൂതൻ (സ) തന്റെ പടയങ്കി ജൂതന് പണയം വെച്ച് ധാന്യം കടം വാങ്ങുകയുണ്ടായി (സ്വഹീഹുൽ ബുഖാരി:1990, സ്വഹീഹു മുസ്‌ലിം: 1603)

കൊലയും കൊള്ളയുമായിരുന്നില്ല മദീനയിലെ ഇസ്‌ലാമിക സാമൂഹിക വ്യവസ്ഥ. മദീനയിലെ ഭരണാധികാരിയായിട്ടും ജൂതന്മാരിൽ നിന്ന് – അവർ യുദ്ധം ചെയ്യും വരെ – സമ്പത്ത് തട്ടിയെടുക്കുകയോ പിടിച്ച് പറിക്കുകയോ ചെയ്തിട്ടില്ല.

ജൂതന്മാരുടെ ഗൂഢാലോചനയുടെ ഭാഗമായി ലബീദിബ്നു അഅ്സം എന്ന ജൂതൻ പ്രവാചകനെ വധിക്കാനായി അദ്ദേഹത്തിന് (സ) മാരണം ചെയ്ത സംഭവവും ഹദീസുകളിൽ കാണാം. (സ്വഹീഹുൽ ബുഖാരി: 3268, സ്വഹീഹു മുസ്‌ലിം :2189)

പ്രവാചകന് (സ) ജൂതന്മാർ മാരണം ചെയ്തെന്ന് മുസ്‌ലിംകൾ അറിഞ്ഞാൽ അത് ജൂതന്മാർക്കും മുസ്‌ലിംകൾക്കുമിടയിൽ ശത്രുതക്ക് കാരണമായേക്കാം എന്നതിനാൽ ജനങ്ങൾ അധികം കാണാതെ അതു നശിപ്പിച്ചു കളയാൻ പ്രവാചകൻ (സ) നിർദേശിച്ചു. (സ്വഹീഹുൽ ബുഖാരി: 3268)

ആ ജൂതനോട് അതിനെ പറ്റി പ്രവാചകൻ (സ) ഒരിക്കൽ പോലും സ്മരിക്കുകയോ എന്തെങ്കിലും പരിഭവം മുഖത്ത് കാട്ടുകയോ ചെയ്തില്ല. (സുനനു നസാഈ: 4080)

എന്നാൽ ജൂത സമൂഹം മുസ്‌ലിംകളുമായി യാതൊരുവിധ സഹവർത്തിത്വത്തിനും തയ്യാറായിരുന്നില്ല. ബനൂ നളീർ, ഖുറൈള എന്നിവർ കരാർ ലംഘിക്കുകയാണുണ്ടായത്.

“അവര്‍ (യഹൂദര്‍) ഏതൊരു കരാര്‍ ചെയ്തു കഴിയുമ്പോഴും അവരില്‍ ഒരു വിഭാഗം അത് വലിച്ചെറിയുകയാണോ? തന്നെയുമല്ല, അവരില്‍ അധികപേര്‍ക്കും വിശ്വാസം തന്നെയില്ല. (ഖുർആൻ 2: 100)

മക്കക്കാർ മുസ്‌ലിംകൾക്കെതിരെ യുദ്ധം ചെയ്തപ്പോൾ ഖുറൈളക്കാർ സഹായിക്കാതെ വിട്ടു നിന്നു എന്നതു മാത്രമായിരുന്നു അവരുടെ കരാർ ലംഘനം എന്നതിനാൽ മദീനയിൽ തന്നെ താമസിക്കാൻ പ്രവാചകൻ (സ) അനുവദിച്ചു. അവരോട് കരുണ കാണിക്കുകയും നാടുകടത്താതിരിക്കുകയും ചെയ്തു. എന്നാൽ ബനൂ നളീറുകാർ കരാർ ലംഘിച്ച് മുസ്‌ലിംകളെ സഹായിക്കാതെ വിട്ടു നിൽക്കുക മാത്രമല്ല ചെയ്തത്. മറിച്ച് മക്കയിലേക്ക് ആളെ അയച്ച് യുദ്ധത്തിനായി അവരെ പ്രോത്സാഹിപ്പിക്കുകയും മുസ്‌ലിം സൈന്യത്തിന്റെ ദുർബലതകൾ അറിയിച്ചു കൊടുക്കുകയും ചെയ്തു. (മഗാസി: മൂസാ ഇബ്നു ഉഖ്ബ: 210)

ബദർ യുദ്ധത്തിന് ശേഷം മദീനയിലേക്ക് യാത്ര ചെയ്ത ബനൂ നളീറുകാരനും കവിയുമായിരുന്ന കഅ്ബിബ്നു അശ്റഫ് കുറൈശികളിൽ നിന്ന് യുദ്ധത്തിൽ കൊല്ലപെട്ടവർക്കായി വിലാപകാവ്യം രചിക്കുകയും യുദ്ധത്തിനായി അവരെ പ്രചോദിപ്പിക്കുകയും ചെയ്തു. കഅ്ബയുടെ വിരിയിൽ പിടിച്ച് മുസ്‌ലിംകളോട് യുദ്ധം ചെയ്യുമെന്ന് ഖുറൈശികളെ കൊണ്ട് കരാർ ചെയ്യിപ്പിച്ചു. (ഫത്ഹുൽ ബാരി: 7/337) മദീനയിൽ വന്ന ശേഷം പ്രവാചകനെ ആക്ഷേപിച്ചു കൊണ്ട് കവിത പാടി. പ്രകോപനം സൃഷ്ടിക്കാനായി പ്രവാചകാനുചരന്മാരുടെ ഭാര്യമാരെ അസഭ്യവാക്കുകൾ കൊണ്ട് വർണിച്ച് ഗസലുകൾ പാടി. ഈ സംഭവം ഒക്‌ലഹോമ സർവകലാശാലയിലെ ജൂഡായിക് ചരിത്രത്തിന്റെ എമെറിറ്റസ് ചെയറും ഒറിയന്റലിസ്റ്റുമായ നോർമൻ ആർതർ സ്റ്റിൽമാൻ തന്റെ ഗ്രന്ഥത്തിൽ എഴുതി ചേർത്തിട്ടുണ്ട്. (The Jews of Arab Lands: A History and Source Book: Page: 124, The Jewish publication society of America)

ദിയ്യത്തുമായി ബന്ധപ്പെട്ട് സഹായത്തിനായി ബനൂ നളീറുകാരുടെ അടുത്ത് ഒരുപറ്റം അനുചരന്മാരോടൊപ്പം പ്രവാചകൻ (സ) ചെന്നപ്പോൾ അദ്ദേഹത്തെ ബനൂ നളീറുകാർ ഒരിടത്ത് ഇരുത്തി. ഹുയൈയ് ഇബ്നു അഖ്തബ് ജൂതന്മാരോട് പറഞ്ഞു: “ജൂത സമൂഹമേ, മുഹമ്മദ് ഇതാ ഒരു ചെറിയ അനുയായി വൃന്ദവുമായി വന്നിരിക്കുന്നു. അവർ പത്തുപേർ പോലുമില്ല. അവർ ഇരിക്കുന്ന ഭവനത്തിന് മുകളിൽ നിന്ന് കല്ല് താഴേക്കിട്ട് മുഹമ്മദിനെ കൊല്ലുക. മുഹമ്മദിനെ കൊല്ലാൻ ഇതിലും നല്ല അവസരമില്ല.” അംറിബ്നു ജിഹശ് ഈ ദൗത്യമേറ്റെടുത്തു, ഭവനത്തിന് മുകളിൽ കയറി കല്ലെടുത്തു. പക്ഷെ ജൂതന്മാരുടെ ഈ കുതന്ത്രം അല്ലാഹു ദിവ്യഉൽബോധനം വഴി പ്രവാചകന് അറിയിച്ചു കൊടുത്തു. അദ്ദേഹം അവിടെ നിന്ന് പെട്ടെന്ന് എഴുന്നേറ്റു പോയി. (The Jews of Arab Lands: A History and Source Book: Page: 129, 130.The Jewish publication society of America )

ഇബ്നു ഉമർ (റ) പറഞ്ഞു: ബനൂ നളീർ, ഖുറൈള എന്നീ ജൂത ഗോത്രങ്ങൾ അല്ലാഹുവിന്റെ ദൂതനോട് യുദ്ധം ചെയ്തു. അപ്പോൾ അല്ലാഹുവിന്റെ ദൂതൻ (സ) അവരെ (മദീനയിൽ നിന്നും ബനൂ നളീറുകാരെ) പുറത്താക്കി. ഖുറൈളക്കാരെ (അവർ യുദ്ധത്തിൽ നേരിട്ട് പങ്കാളികളായില്ല എന്നതിനാൽ പ്രവാചകൻ (സ) മദീനയിൽ) താമസിപ്പിച്ചു; അതിന് ശേഷം ഖുറൈളക്കാരും യുദ്ധം ചെയ്യുന്നത് വരെ. (സ്വഹീഹുൽ ബുഖാരി: 4028, സ്വഹീഹു മുസ്‌ലിം: 1766)

‘നിങ്ങൾക്ക് കോട്ടയും സുരക്ഷയുമുണ്ടായിരിക്കെ നിങ്ങൾ എന്ത് കൊണ്ട് മുഹമ്മദിനെതിരെ സംഘടിക്കുന്നില്ല ?’ എന്ന് ആരാഞ്ഞു കൊണ്ട് ബദർ യുദ്ധാനന്തരം മക്കയിലെ ഖുറൈശികൾ മദീനയിലെ ജൂതന്മാർക്ക് കത്തെഴുതി. അങ്ങനെ ബനൂ നളീറുകാർ പ്രവാചകനെ ചതിക്കാൻ ഗൂഢാലോചന നടത്തി. പ്രവാചകനോട് അദ്ദേഹത്തിന്റെ മൂന്ന് അനുയായികളെ തങ്ങളിലേക്ക് അയക്കാനും ഞങ്ങളിലെ മൂന്ന് പുരോഹിതന്മാരുമായി അവർ സംസാരിച്ചതിന് ശേഷം അവർ ഇസ്‌ലാമിൽ വിശ്വസിക്കുകയാണെങ്കിൽ ഞങ്ങൾ മുഴുവനും ഇസ്‌ലാം സ്വീകരിക്കാം എന്നും പറഞ്ഞു. ഇത് കേട്ട് പ്രവാചകൻ (സ) അപ്രകാരം ദൂതന്മാരെ അയച്ചു. ബനൂ നളീറുകാർ സായുധരായി ഈ അനുചരന്മാരെ വധിക്കാനായി ഒരുങ്ങി നിന്നു. ബനൂ നളീറുകാരിൽ പെട്ട ഒരു സ്ത്രീ അൻസ്വാരികളിൽ പെട്ട തന്റെ സഹോദരന് ബനൂ നളീറുകാരുടെ ചതിയെ പറ്റി രഹസ്യമായി വിവരമറിയിച്ചു. ഇതറിഞ്ഞ പ്രവാചകൻ (സ) അനുചരന്മാരെ തിരിച്ചു വിളിച്ചു. ഈ വഞ്ചനക്കും കരാർ ലംഘനത്തിനും പകരമായി മുസ്‌ലിംകൾ ബനൂ നളീറുകാരുടെ കോട്ട ഉപരോധിച്ചു. ഉപരോധം ശക്തമായപ്പോൾ മദീനയിൽ നിന്നും നാടുകടത്തപെടുക എന്ന നിബന്ധനയോടെ അവർ കീഴടങ്ങി. ഈ വഞ്ചനകൾക്ക് ശേഷവും മദീനയിൽ താമസിക്കാൻ കഴിയില്ലെന്നും പത്ത് ദിവസം കൊണ്ട് ഒഴിഞ്ഞു പോകണമെന്നും മുഹമ്മദ് ഇബ്നു മസ്‌ലമയെ ദൂതനായി അയച്ച് ബനൂ നളീറുകാരെ പ്രവാചകൻ (സ) വിവരമറിയിച്ചു. സംഭവം സ്വഹീഹായ സനദു സഹിതം ഇമാം ഇബ്നു ഹജർ അൽ അസ്കലാനി തന്റെ ‘ഫത്ഹുൽ ബാരി’യിൽ (7/331, 332) ഉദ്ധരിക്കുന്നുണ്ട്.

കുറച്ച് കാലങ്ങൾക്ക് ശേഷം മക്കക്കാർ മുസ്‌ലിംകൾക്കെതിരെ ഖന്ദക്ക് യുദ്ധത്തിന് നേതൃത്വം നൽകിയപ്പോൾ ഖുറൈളക്കാർ അവരോടൊപ്പം സഖ്യം ചേരുകയും മദീനയിൽ ഒറ്റപ്പെട്ട സ്ത്രീകളെ ആക്രമിക്കുകയും ചെയ്തപ്പോൾ മാത്രമാണ് അവരേയും മദീനയിൽ നിന്ന് പ്രവാചകൻ (സ) പുറത്താക്കിയത്. (സീറത്തു ഇബ്നു ഹിശാം: 3/ 231- 233)

“അവരില്‍ ഒരു വിഭാഗവുമായി നീ കരാറില്‍ ഏര്‍പെടുകയുണ്ടായി. എന്നിട്ട് ഓരോ തവണയും തങ്ങളുടെ കരാര്‍ അവര്‍ ലംഘിച്ചുകൊണ്ടിരുന്നു. അവര്‍ (അല്ലാഹുവെ) സൂക്ഷിക്കുന്നുമില്ല.” (ഖുർആൻ 8: 56) ഈ വചനവും അവതരിപ്പിക്കപ്പെട്ടതും ബനൂ ഖൈനുകാഉകാരുടെ കാര്യത്തിലാണെന്ന് ഇമാം മുജാഹിദ് വ്യക്തമാക്കുന്നു. (അദ്ദുർറുൽ മൻസൂർ: സുയൂത്വി: 3/191, മഗാസി: വാഖിദി: 1/135)

ഖന്ദക്ക് യുദ്ധത്തോടെ മുസ്‌ലിംകളുമായുള്ള സന്ധി റദ്ദു ചെയ്തുവെന്ന് പരസ്യമായി പ്രഖ്യാപിച്ച ബനൂ ഖുറൈളക്കാരോട് കരാർ പുനസ്ഥാപിക്കാൻ ആവശ്യപ്പെട്ട് പ്രവാചകൻ നിയോഗിച്ച സഅ്ദിബ്നു ഉബാദ സഅ്ദിബ്നു മുആദ് (റ) എന്നിവരെ ബനൂ ഖുറൈളക്കാർ ശകാരിക്കുകയും തിരസ്കരിക്കുകയും ചെയ്തു. (സീറത്തു ഇബ്നു ഹിശാം: 2/222)

മുസ്‌ലിംകൾ ഖന്ദക്ക് യുദ്ധത്തിൽ മുഴുകിയിരിക്കെ മദീനയിലെ സ്ത്രീകളെ ആക്രമിക്കാൻ ബനൂ ഖുറൈളക്കാർ പദ്ധതിയിട്ടു. മുസ്‌ലിംകൾ സ്ത്രീകളേയും കുട്ടികളേയും താമസിപ്പിച്ചിരിക്കുന്ന കോട്ടയെ ഖുറൈളക്കാരിലെ ദൂതൻ വലം വെച്ചു. അയാൾ ഉള്ളിലേക്ക് എത്തി നോക്കിയപ്പോൾ സ്വഫിയ്യ ബിൻത്ത് അബ്ദുൽ മുത്വലിബ് (റ) കുന്തം കൊണ്ട് അയാളെ കുത്തിയിട്ടു. മുസ്‌ലിംകൾ കോട്ടയിലും സൈന്യത്തെ വിന്യസിച്ചിട്ടുണ്ടെന്ന് ബനൂ ഖുറൈളക്കാർ തെറ്റിദ്ധരിക്കുകയും ചെയ്തു. (അൽ ഇസ്വാബ: 2/56, 8/214, മുസ്തദ്റഖു ഹാകിം:6945, മജ്മഉ സവാഇദ്: 6/115)

ബനൂ ഖൈനുകാഅ് ഗോത്രത്തിൽ പെട്ടവർ ഒരു മുസ്‌ലിം സ്ത്രീയുടെ വസ്ത്രം ബലം പ്രയോഗിച്ച് അഴിക്കുകയും അവളുടെ നഗ്നത കണ്ട് കൂട്ടത്തോടെ ചിരിക്കുകയും ചെയ്തു. ഉടനെ ഒരു മുസ്‌ലിം, ആ സ്ത്രീയുടെ വസ്ത്രം വലിച്ചൂരിയ വ്യക്തിയെ വധിച്ചു. ബനൂ ഖൈനുകാഅ് സംഘം അദ്ദേഹത്തെ വധിച്ചു. ഈ സംഭവവുമായി ബന്ധപ്പെട്ട കലാപത്തെ തുടർന്നാണ് ബനൂ ഖൈനുകാഉകാർ നാടുകടത്തപെട്ടത്. (സീറത്തു ഇബ്നു ഹിശാം: 2/642, അൽ മഗാസി: വാകിദി: 1/176, 177)

ഈ കലാപകാരികളും യുദ്ധ കൊതിയരുമായ സമൂഹത്തെയാണോ മുഹമ്മദ് നബിയുടെ ക്രൂരതകളുടെ ഇരകളായി വിമർശകർ ദുർവ്യാഖ്യാനിക്കുന്നത് ?! ഇത്തരം കുൽസിത പ്രവർത്തനങ്ങളുടേയും രാജ്യദ്രോഹത്തിന്റെയും പേരിലാണ് മദീനയിൽ നിന്നും ജൂതന്മാരെ പ്രവാചകൻ (സ) പുറത്താക്കിയത്. എന്നിട്ടും ഖൈബറിൽ താമസമാക്കിയ ജൂത സമൂഹം മുസ്‌ലിം സമൂഹത്തേയും രാഷ്ട്രത്തേയും വെറുതെ വിടാൻ തയ്യാറായിരുന്നില്ല ! മദീനയിൽ നിന്നും നാടുകടത്തപ്പെട്ട ബനൂ നളീറുകാർക്ക് ഖൈബറിലെ ജൂതർ അഭയം നൽകി, അവർ അറബികളിലെ പല ഗോത്രങ്ങളേയും മുസ്‌ലിംകൾക്കെതിരെ യുദ്ധത്തിനായി പ്രേരിപ്പിക്കുവാനും ആരംഭിച്ചു. ബനൂ നളീറുകാരടങ്ങുന്ന ഖൈബറുകാർ ഗോത്രങ്ങളെ സംഘടിപ്പിക്കുകയും ഖുറൈശികളെ പ്രലോഭിപ്പിക്കുകയും ചെയ്തതിലൂടെയാണ് ഖുറൈശികൾ മുസ്‌ലിംകൾക്കെതിരെ ഖന്ദഖ് യുദ്ധം നടത്തുന്നത്. (ഉംദത്തുൽ കാരി: ഇമാം അൽ ഐനി: 17/176, സീറത്തു ഇബ്നു ഹിശാം: 2/441, സിയറു അഅ്ലാമിന്നു ബലാഅ്: 1/457, അൽ ബിദായ വന്നിഹായ: 8/50)

ഹിജ്റ ഏഴാം വർഷം നടന്ന ഖൈബർ യുദ്ധത്തിനൊടുവിൽ ജൂതർ പരാജയപ്പെട്ടു. ജൂതന്മാരെ ഒന്നടങ്കം ഖൈബറിൽ നിന്നും നാടുകടത്താൻ മുസ്‌ലിംകൾ തീരുമാനിച്ചെങ്കിലും ജൂതർ പ്രവാചകനോട് സന്ധിക്കായി കേണു. ഖൈബർ യുദ്ധാനന്തരവും ജൂതന്മാരോട് അനുനയത്തോടെയും കരുണയോടെയും പെരുമാറാൻ പ്രവാചകൻ (സ) മടിച്ചില്ല.

ഇബ്നു ഉമർ (റ) പറഞ്ഞു: അല്ലാഹുവിന്റെ റസൂൽ (സ) ഖൈബർ ദേശം യഹൂദന്മാർക്ക് തന്നെ നൽകി, അവർ അതിൽ പ്രവർത്തിക്കുകയും കൃഷിചെയ്യുകയും അതിന്റെ വിളവിന്റെ പകുതി മുസ്‌ലിംകൾക്ക് നൽകുകയും ചെയ്യുക എന്ന നിബന്ധനയോടെ. (സ്വഹീഹുൽ ബുഖാരി: 2165, സ്വഹീഹു മുസ്‌ലിം: 1551)

ഖൈബറിലെ ജൂതരോട് ഇത്തരമൊരു അനുനയ സമീപനം സ്വീകരിച്ചിട്ടും അവർ തിരിച്ച് വെച്ചുപുലർത്തിയ നയം ശത്രുത തന്നെയായിരുന്നു. ഈ ചരിത്ര ദശയിലാണ് ഖൈബറിലെ ജൂതന്മാരിൽ പെട്ട ഒരു സ്ത്രീ പ്രവാചകനേയും(സ) അനുയായികളേയും വിരുന്നു വിളിക്കുകയും വിഷം തേച്ച ആട്ടിൻ മാംസം വിളമ്പുകയും ചെയ്തത്. വിഷം ചേർത്തിട്ടുണ്ടെന്ന് അറിഞ്ഞപ്പോൾ ആ സ്ത്രീയെ വധിക്കട്ടെയെന്ന് അനുചരന്മാർ ചോദിച്ചുവെങ്കിലും പ്രവാചകൻ വേണ്ടെന്ന് പറഞ്ഞു. (സ്വഹീഹുൽ ബുഖാരി: 3169)

ഒരിക്കലും സമാധാന സന്ധികൾ നിലനിർത്താനും യുദ്ധ കലാപ നിരോധന കരാറുകൾ പാലിക്കാനും തയ്യാറല്ലാത്ത സമൂഹങ്ങൾ മദീനയിൽ നിലനിൽക്കുമ്പോൾ മക്കയിലേയും മദീനയിലേയും പുണ്യ ഭൂമി, യുദ്ധം നിഷിദ്ധമാക്കപ്പെട്ട ഹറമുകൾ ചോരയാൽ ചുവന്നുകൊണ്ട് തന്നെ നിലനിൽക്കും. അരക്ഷിതത്വവും അഹിംസയും തീരാ കഥകളാകുമെന്ന ബോദ്ധ്യത്തിൽ നിന്നാണ് ജൂതന്മാരെ ഹിജാസിൽ നിന്നും പുറത്താക്കാൻ പ്രവാചകൻ (സ) നിർബന്ധിതനാകുന്നത്. ജൂത ക്രൈസ്തവരടങ്ങുന്ന അമുസ്‌ലിംകൾ മക്കാ മദീന പള്ളികൾ ഒഴികെയുള്ള, ഹിജാസിലെ മറ്റു സ്ഥലങ്ങളിൽ താമസിക്കുന്നത് നിഷിദ്ധമായി ഇസ്‌ലാം കാണുന്നില്ല. അതുകൊണ്ടാണ് പ്രവാചകൻ (സ) തന്നെയും മരണം വരെ അവരെ പുറത്താക്കാതിരുന്നത്. പുറത്താക്കണമെന്ന് പറഞ്ഞതാകട്ടെ ജീവിതത്തിന്റെ അവസാന ശ്വാസം വരെ ജൂതന്മാരും ഖുറൈശികളും ഹിജാസിൽ സമാധാനാന്തരീക്ഷം പുലരുവാൻ സമ്മതിക്കില്ല എന്ന് മനസ്സിലാക്കിയപ്പോഴാണ്.

പ്രവാചകനായി നിയോഗിതനായപ്പോഴോ, മക്കയിൽ നിന്നും മദീനയിലേക്ക് വന്ന ഉടനെയോ, മദീനയുടെ ഭരണാധികാരി ആയ ഉടനെയോ ‘ജൂതന്മാരെ മദീനയിൽ നിന്ന് പുറത്താക്കണമെന്ന്’ പ്രവാചകൻ പറഞ്ഞിരുന്നെങ്കിൽ അദ്ദേഹത്തെ വർഗീയ വാദിയെന്നും ക്രൂരനെന്നും വിളിക്കുന്നതിന് ന്യായം കണ്ടെത്താമായിരുന്നു.

നബി(സ്വ)യും ‘ശവരതി’യാരോപണവും

വിമര്‍ശനം:

നബിജീവിതം വിശുദ്ധമായിരുന്നു എന്ന ധാരണ തെറ്റാണെന്ന് വ്യക്തമാക്കുന്ന സംഭവമാണ് ഫാത്തിമ ബിന്‍ത് അസദ് എന്ന സ്ത്രീയുടെ മൃതദേഹത്തെ ഖബ്റില്‍ (ശവക്കുഴി) വെച്ച് ഭോഗിച്ച പ്രവാചക നടപടി. ശവരതിപോലും നടത്താന്‍ മടിയില്ലായിരുന്ന ഒരാളെയാണ് മുസ്‌ലിംകള്‍ മാര്‍ഗദര്‍ശകനായി ഉള്‍ക്കൊള്ളുന്നത് എന്നത് എത്രമാത്രം ലജ്ജാവഹമാണ്. രക്തബന്ധമോ കുടുംബബന്ധമോ പോലും പരിഗണിക്കാതെ ഏത് സ്ത്രീയെയും ഭോഗിക്കുവാന്‍ നബിക്ക് അല്ലാഹു അനുവാദം നല്‍കുന്നതിനു മുമ്പായിരുന്നു (ക്വുര്‍ആന്‍ 33:50) ഈ ശവരതി നടന്നതെന്നു കാണാം. സ്ത്രീകളെ കേവലം ലൈംഗിക ഉപഭോഗ വസ്തു മാത്രമായി കണ്ടിരുന്ന ഒരാള്‍ എങ്ങിനെയാണ് ദൈവദൂതനും മാര്‍ഗദര്‍ശകനുമാവുക!?.

മറുപടി:

പ്രശസ്ത മനഃശാസ്ത്രജ്ഞന്‍ സ്റ്റീവന്‍ സ്റ്റോസ്നി (Steven Stonsy phD) ഓസ്‌കാര്‍ വൈല്‍ഡിന്റെ വാചകങ്ങള്‍ ഉദ്ദരിച്ചുകൊണ്ട് ഇപ്രകാരം പറയുകയുണ്ടായി ”ഓസ്‌കാര്‍ വൈല്‍ഡ് ഒരിക്കല്‍ പറഞ്ഞതുപോലെ ‘ആത്മകഥയുടെ വിശ്വസനീയമായ ഒരേയൊരു രൂപമാണ് നിരൂപണം’. ഒരു നിരൂപണം നിരൂപകന്‍ വിമര്‍ശിക്കുന്ന ആളുകളേക്കാള്‍ – അത് പുരുഷനാകട്ടെ സ്ത്രീയാകട്ടെ – കൂടുതല്‍ നിരൂപകന്റെ മനഃശാസ്ത്രത്തെയാണ് നമ്മോട് പറഞ്ഞു തരുന്നത്. ഒരാളുടെ വിമര്‍ശനങ്ങള്‍ കേട്ടാല്‍ മാത്രം വിദഗ്ധരായ പ്രൊഫഷണലുകള്‍ക്ക് വിമര്‍ശകന്റെ രോഗനിര്‍ണയ സിദ്ധാന്തം രൂപപ്പെടുത്താന്‍ കഴിയും” (www.spychologytoday.com)

സ്റ്റീവന്‍ സ്റ്റോസ്നി പറഞ്ഞതുപോലെ, പ്രവാചകനെതിരെയുള്ള ഈ ആരോപണം വാസ്തവത്തില്‍ അടിവരയിടുന്നത് ആരോപകരുടെ മനഃശാസ്ത്രത്തെയാണ്. എത്രമാത്രം നികൃഷ്ഠവും നിന്ദ്യവുമായ മനോവൈകൃതങ്ങള്‍ക്ക് അടിപ്പെട്ടവരാണ് നബിവിമര്‍ശകരെന്ന സത്യം അര്‍ഥശങ്കക്കിടയില്ലാത്ത വിധം ബോധ്യപ്പെടുത്തുന്നതാണ് ഈ വിമര്‍ശനം. തങ്ങളുടെ ഹൃദയങ്ങളില്‍ കട്ടപിടിച്ചു കിടക്കുന്ന വൈകൃതങ്ങളുടെ ചഷകങ്ങളിലൂടെ പ്രവാചകജീവിതത്തെ തലതിരിഞ്ഞു നോക്കികാണുന്ന ഇത്തരം നബിനിന്ദകരില്‍ നിന്നും വസ്തു നിഷ്ഠമായ നിരൂപണങ്ങള്‍ പ്രതീക്ഷിക്കുന്നതാണ് ഏറ്റവും വലിയ തെറ്റ്. തികഞ്ഞ അരാജകവാദികളും ജീവിത മൂല്യങ്ങളോട് കടുത്ത വിരോധം വെച്ചുപുലര്‍ത്തുകയും ചെയ്യുന്ന നവനാസ്തിക ഞരമ്പുരോഗികളാണ് ഈ കഠിനമായ നബിനിന്ദ സമൂഹമാധ്യമങ്ങളിലൂടെ പ്രചരിപ്പിച്ചുകൊണ്ടിരിക്കന്നതിന് മുഖ്യ കാര്‍മികത്വം വഹിച്ചുകൊണ്ടിരിക്കുന്നത്. തങ്ങളുടെ മനോവൈകൃതങ്ങള്‍ക്കനുസരിച്ച് അവര്‍ നബിജീവിതത്തെ ദുര്‍വ്യാഖ്യാനിച്ചും ദുഷിപ്പിച്ചും അവതരിപ്പിക്കുന്നു എന്നത് ആശ്ചര്യജനകമായ കാര്യമൊന്നുമല്ല. കാരണം അവര്‍ വിഭാവനം ചെയ്യുന്ന ഭോഗതൃഷ്ണാമയമായ കുത്തഴിഞ്ഞ ജീവിതാസ്വാധനങ്ങള്‍ക്കെതിരെ, ജീവിത വിശുദ്ധിയുടെ വഴിയടയാളങ്ങള്‍ ചൂണ്ടികാണിക്കുന്ന ഏറ്റവും കരുത്തുറ്റ വ്യക്തിത്വം മുഹമ്മദ് നബി(സ്വ)യാണ്. അതുകൊണ്ടു തന്നെ നബിജീവിതത്തെ അപകീര്‍ത്തിപ്പെടുത്താനും നുണകളിലൂടെ ആ വിശുദ്ധ ജീവിതത്തെ തമസ്‌കരിക്കാനും അവര്‍ അത്യാര്‍ത്തരാണ്; എന്നും എപ്പോഴും. മാനവികതക്ക് ജീവിത മൂല്യങ്ങള്‍ പകര്‍ന്നു നല്‍കിയ മഹാ മാനുഷികളുടെ ചരിത്രവും ജീവിതവും ദുര്‍വ്യാഖ്യാനിച്ച്, അവരെയും തങ്ങളെ പോലെ അരാജകത്വാസ്വാധകരും മൂല്യനിരാസ പ്രണയികളുമാക്കി ചിത്രീകരിക്കുക വഴി ജീവിത വിശുദ്ധിയുടെ പാഥേയത്തെ പ്രകാശ പൂരിതമാക്കുന്ന വഴിവിളക്കുകളെ കെടുത്തികളയാനാണ് അവര്‍ വെമ്പല്‍ കൊള്ളുന്നത്. നല്ലവരെയും നന്മയിലേക്ക് വിളിച്ചവരെയും ഇരുട്ടുകൊണ്ട് മറച്ചുപിടിക്കുക എന്നത് നവനാസ്തിക ഞരമ്പുരോഗികളുടെ പ്രഥമ അജണ്ടകളില്‍ പെട്ടതാണെന്ന് അവരെ അറിയുന്നവര്‍ക്കെല്ലാമറിയാവുന്ന കാര്യമാണ്. അതുകൊണ്ടു തന്നെ മുഹമ്മദ് നബി (സ്വ) ഒരു ഖബ്‌റിന്നരികെ ഇരിക്കുന്ന സന്ദര്‍ഭം ഹദീഥ് ഗ്രന്ഥങ്ങളില്‍ കാണുമ്പോഴേക്കും അതില്‍ നിന്നും ശവരതി വിഭാവനം ചെയ്യാനും, ക്രിസ്തുവിനരികെ മഗ്‌ദല മറിയം നില്‍ക്കുന്ന ചിത്രം കാണുമ്പോഴേക്കും അതില്‍ നിന്നും രഹസ്യവേഴ്ച്ചയുടെ ഇല്ലാ കഥകള്‍ മെനഞ്ഞെടുക്കാനും അവര്‍ നിതാന്ത പരിശ്രമത്തിലായിരിക്കും. കാരണം അവരുടെ മനോവൈകൃതം അത്രമേല്‍ കഠിനമാണ്; നിന്ദിതവും.

പക്ഷെ, അത്ഭുതകരമായ കാര്യം, മുഹമ്മദ് നബി(സ്വ)ക്കെതിരെയുള്ള ഹീനമായ ഈ നുണകഥ നവനാസ്തികരടക്കമുള്ള ഇസ്‌ലാംവിരോധികള്‍ക്കെല്ലാം വേവിച്ച് വിളമ്പിയത് ചില മിഷനറി അടുക്കളയിലാണെന്നതാണ്. ഒരു ശരാശരി ധാര്‍മികത പോലും പല മിഷനറി ഗ്രൂപ്പുകളില്‍ നിന്നും പ്രതീക്ഷിക്കാന്‍ സാധ്യമല്ലാതായിരിക്കുന്നു. മൂടുപടം മാറ്റിയത് മിഷനറിമാരുടെ ഇസ്‌ലാംവിരോധം മാത്രമല്ല, അവരുടെ മനോവൈകൃതങ്ങള്‍ കൂടിയാണ്. നിരൂപണങ്ങള്‍ നിരൂപകന്റെ മനഃശാസ്ത്രത്തെ കൂടി അടയാളപ്പെടുത്തുന്നുണ്ട് എന്ന മനഃശാസ്ത്ര നിരീക്ഷണം അതാണല്ലോ നമ്മെ പഠിപ്പിക്കുന്നത്. അടുത്ത കാലത്തായി ചില മിഷനറിമാരുടെയും സുവിശേഷ വേലക്കാരുടെയും ഇസ്‌ലാംവിമര്‍ശനങ്ങള്‍ സഭ്യതയുടെ സകല സീമകളും ഉല്ലെങ്കിക്കുന്നതാണ്. മനോവൈകൃതങ്ങളുടെ തള്ളിച്ചക്കൊപ്പം ഇസ്‌ലാംവിരോധവും കെട്ടുപിണയുമ്പോള്‍, ചില മിഷനറി-സുവിശേഷ വേലക്കാര്‍ സമൂഹ മാധ്യമങ്ങളില്‍ മേല്‍വിലാസമില്ലാത്ത ഇത്തരം നബിനിന്ദകള്‍ വിസര്‍ജിക്കുന്നത് ഒരു സ്ഥിരം ഏര്‍പ്പാടാക്കിയിട്ടുണ്ട്; എല്ലാവരുമല്ലെങ്കിലും ചിലര്‍ അല്ല പലരും. മഠങ്ങളിലും കോണ്‍വെന്റുകളിലും വര്‍ദ്ധിച്ചു വരുന്ന കന്യാസ്ത്രീ ‘അത്മഹത്യ’കള്‍ക്കു പിന്നില്‍ ഇത്തരം വൈകൃതംപേറികളുടെ തള്ളിച്ചകളാണോ എന്നാരെങ്കിലും സംശയിച്ചാല്‍, അത് തീര്‍ത്തും ഒരു ക്രൈസ്തവ വിരുദ്ധ നിലപാടുമാത്രമായി വിലയിരുത്താന്‍ എപ്പോഴും കഴിയില്ലല്ലോ!?.

ഇനി നമുക്കു പരിശോദിക്കുവാനുള്ളത് പ്രവാചകനെതിരെയുള്ള ‘ശവരതി’യാരോപണത്തിന്റെ നിജസ്ഥിതിയാണ്. ‘ഫാത്തിമ ബിന്‍ത് അസദ് എന്ന സ്ത്രീയുടെ മൃതദേഹത്തെ പ്രവാചകന്‍ ഖബ്റില്‍ വെച്ച് ഭോഗിച്ചു’ എന്നതാണ് ആരോപണം. എത്രമാത്രം കടുത്ത ദുരാരോപണം. കല്ലുവെച്ച നുണയല്ലാതെ മറ്റൊന്നുമല്ലിത്. പ്രസ്തുത ആരോപണത്തിന്റെ നിജസ്ഥിതി പരിശോദിക്കും മുമ്പ് ആരാണ് ഈ ‘ഫാത്തിമ ബിന്‍ത് അസദ്’ എന്ന് നാം അറിയേണ്ടിയിരിക്കുന്നു. നബി(സ്വ)യുടെ പിതൃവ്യന്‍ അബൂത്വാലിബിന്റെ ഭാര്യ. അഥവാ പ്രവാചകന്റെ പിതൃവ്യ. നബി(സ്വ)യുടെ മകള്‍ ഫാത്തിമ(റ)യുടെ ഭര്‍ത്താവ് അലി(റ)യുടെ മാതാവ്. ചെറുപ്രായത്തില്‍ തന്നെ മതാ-പിതാക്കള്‍ നഷ്ടപ്പെട്ട നബി(സ്വ)യെ എടുത്തു വളര്‍ത്തിയ, ഊട്ടിയ, ഉറക്കിയ പ്രിയ പോറ്റുമ്മ. പ്രവാചകന്‍ (സ്വ) അവരെ ‘ഉമ്മ’ എന്നാണ് തന്റെ മരണം വരെ സംബോധന ചെയ്തത്. അബൂത്വാലിബിനു ശേഷം നബി(സ്വ)യോട് ഏറ്റവുമധികം രക്തബന്ധപരിഗണന കാണിച്ച വ്യക്തി. ‘എന്റെ പെറ്റുമ്മക്കു ശേഷമുള്ള എന്റെ ഉമ്മ’ എന്നാണ്, ശവരതിയാരോപണത്തിനായി ഇസ്‌ലാംവിരോധികള്‍ ദുര്‍വ്യാഖ്യാനിക്കുന്ന നിവേദനത്തില്‍ പോലും പ്രവാചകന്‍ അവരെ സംബോധന ചെയ്തത്. പ്രവാചകനുമേല്‍ ശവരതിയാരോപിക്കുവാനായി ഇസ്‌ലാംവിദ്വേഷികള്‍ വളച്ചൊടിച്ച നിവേദനത്തില്‍ തന്നെ, നബി (സ്വ) അവരെ പറ്റി ഇങ്ങനെ പറഞ്ഞതായി രേഖപ്പെടുത്തപെട്ടിരിക്കുന്നു: ”എന്റെ ഉമ്മാ, നിങ്ങള്‍ക്ക് അല്ലാഹു കാരുണ്യം നല്‍കട്ടെ. നിങ്ങള്‍ എന്റെ ഉമ്മക്കു ശേഷം എന്റെ ഉമ്മയായിരുന്നു. നിങ്ങള്‍ വിശപ്പു സഹിക്കുകയും എന്നെ ഭക്ഷണം ഊട്ടുകയും ചെയ്തിരുന്നു. നിങ്ങള്‍ക്ക് വസ്ത്രമില്ലാതിരുന്നിട്ടും എനിക്കു വസ്ത്രം നല്‍കിയിരുന്നു. വിശിഷ്ടമായ ഭക്ഷണങ്ങള്‍ നിങ്ങളെന്നെ ഊട്ടുകയും നിങ്ങള്‍ക്കത് സ്വയം നിഷേധിക്കുകയും ചെയ്തിരുന്നു. അതു മൂലം നിങ്ങള്‍ ഉദ്ദേശിച്ചിരുന്നത് അല്ലാഹുവിന്റെ പ്രീതിയും പരലോക മോക്ഷവുമായിരുന്നു.” മാത്രമല്ല ”അബൂത്വാലിബിന്റെ വിയോഗാനന്തരം ഏറ്റവും മെച്ചപ്പെട്ട രീതിയില്‍ എന്നെ സംരക്ഷിച്ച സ്ത്രീയാണവര്‍” എന്നുകൂടി നബി (സ്വ) അവരെ പറ്റി അനുസ്മരിച്ചതും പ്രസ്തുത നിവേദനങ്ങളില്‍ തന്നെ കാണാവുന്നതാണ്. ഇത്രയെല്ലാം കണ്ടിട്ടും, പ്രവാചകന് അവരോടുണ്ടായിരുന്ന അളവറ്റ സ്നേഹവും വാത്സല്യവും കാരുണ്യവും ആദരവുമെല്ലാം തിരിച്ചറിഞ്ഞിട്ടും, പ്രസ്തുത നിവേദനങ്ങളില്‍ നിന്നും അതെല്ലാം അടര്‍ത്തിമാറ്റി ആ നിവേദനങ്ങളെ തന്നെയെടുത്ത് ക്രൂരമായി ദുര്‍വ്യാഖ്യാനിച്ച് അതില്‍ നിന്നും ശവരതി കണ്ടെടുത്തവരുടെ അന്തരംഗം എത്രമേല്‍ വൃത്തിഹീനമായിരിക്കും. ഏതു കുമ്പസാര കൂട്ടില്‍ കൊണ്ടുപോയി കഴുകികളയും ഈ നികൃഷ്ഠത!.

വൈകൃതാനുരാഗികള്‍ പ്രവാചകനെതിരെ ഉന്നയിച്ച ശവരതിയാരോപണത്തിന്റെ യാഥാര്‍ഥ്യമറിയാന്‍ പ്രസ്തുത നിവേദനങ്ങളും അവയുടെ സ്വീകാര്യതയും നമുക്ക് പരിശോദനാവിധേയമാക്കാം. ”അനസ് (റ) പറയുന്നു: അലി(റ)യുടെ മാതാവായ ഫാത്തിമ ബിന്‍ത് അസദ് ബ്നു ഹാശിം മരണപ്പെട്ടപ്പോള്‍ പ്രവാചകന്‍ അവരുടെ അടുത്തേക്ക് പ്രവേശിച്ചു. അവരുടെ (മൃതദേഹത്തിന്റെ) തല ഭാഗത്ത് അദ്ദേഹം ഇരുന്നു. എന്നിട്ടവിടുന്ന് പറഞ്ഞു: എന്റെ ഉമ്മാ, നിങ്ങള്‍ക്ക് അല്ലാഹു കാരുണ്യം നല്‍കട്ടെ. നിങ്ങള്‍ എന്റെ ഉമ്മക്കു ശേഷം എന്റെ ഉമ്മയായിരുന്നു. നിങ്ങള്‍ വിശപ്പു സഹിക്കുകയും എന്നെ ഭക്ഷണം ഊട്ടുകയും ചെയ്തിരുന്നു. നിങ്ങള്‍ക്ക് വസ്ത്രമില്ലാതിരുന്നിട്ടും എനിക്കു വസ്ത്രം നല്‍കിയിരുന്നു. വിശിഷ്ടമായ ഭക്ഷണങ്ങള്‍ നിങ്ങളെന്നെ ഊട്ടുകയും നിങ്ങള്‍ക്കത് സ്വയം നിഷേധിക്കുകയും ചെയ്തിരുന്നു. അതു മൂലം നിങ്ങള്‍ ഉദ്ദേശിച്ചിരുന്നത് അല്ലാഹുവിന്റെ പ്രീതിയും പരലോക മോക്ഷവുമായിരുന്നു. ശേഷം അവരുടെ മയ്യിത്ത് കുളിപ്പിക്കുവാന്‍ പ്രവാചകന്‍ കല്‍പ്പിച്ചു. മൂന്നുവട്ടം കഴുകുന്ന രീതിയിലാകണമെന്ന് നിര്‍ദ്ദേശിച്ചു. കര്‍പൂരം കലര്‍ത്തിയ വെള്ളമെത്തിയപ്പോള്‍ പ്രവാചകന്‍ തന്റെ കൈ കൊണ്ട് അതവരുടെ മേല്‍ ഒഴിച്ചു. ശേഷം പ്രവാചകന്‍ തന്റെ മേല്‍കുപ്പായം ഊരി. എന്നിട്ട് അവരുടെ വസ്ത്രത്തിന്റെ മുകളില്‍ പ്രവാചകന്‍ തന്റെ വസ്ത്രം കഫന്‍ ചെയ്തു. എന്നിട്ട് നബി (സ്വ) ഉസാമത്ത് ബ്നു സൈദ്, അബൂഅയ്യൂബുല്‍ അന്‍സാരി, ഉമ്മറിബ്നുല്‍ ഖത്താബ് എന്ന മൂന്നുപേരെ വിളിച്ചു. അവരുടെ കൂടെ കറുത്ത ഒരു ബാലനുമുണ്ടായിരുന്നു. എന്നിട്ടവരോട് കുഴി കുഴിക്കാന്‍ പറഞ്ഞു. അങ്ങനെ അവരുടെ ഖബ്ര്‍ അവര്‍ കുഴിച്ചു. അങ്ങനെ ലഹ്ദ് എത്തിയപ്പോള്‍ പ്രവാചകന്‍ തന്റെ കൈ കൊണ്ട് ലഹ്ദ് കുഴിച്ചു. എന്നിട്ടാ ലഹ്ദിന്റെ മണ്ണ് പ്രവാചകന്‍ തന്നെ തന്റെ കൈ കൊണ്ട് പുറത്തെടുത്തു. ശേഷം അതില്‍ നിന്നും വിരമിച്ചപ്പോള്‍ പ്രവാചകന്‍ ആ ലഹ്ദിന്റെ ഉള്ളിലേക്ക് പ്രവേശിച്ചു. എന്നിട്ടവിടെ കിടന്നു. എന്നിട്ട് പ്രവാചകന്‍ പ്രാർത്ഥിച്ചു: ജീവിപ്പിക്കുകയും മരിപ്പിക്കുകയും ചെയ്യുന്ന, ഒരിക്കലും മരിക്കാത്ത എന്നെന്നും ജീവിച്ചിരിക്കുന്നവനായ അല്ലാഹുവേ, എന്റെ ഉമ്മയായ ഫാത്തിമ ബിന്‍ത് അസദിന് നീ പൊറുത്തു കൊടുക്കേണമേ, അവര്‍ക്കനുകൂലമായ പ്രമാണങ്ങള്‍ നീ അവര്‍ക്ക് നല്‍കേണമേ, അവരുടെ പ്രവേശന സ്ഥാനം നീ വിശാലമാക്കേണമേ, നിന്റെ നബിയുടെയും എനിക്കു മുമ്പുള്ള നിന്റെ മറ്റു പ്രവാചകന്മാരുടെയും അവകാശം കൊണ്ട് ഞാന്‍ ചോദിക്കുന്നു. തീര്‍ച്ചയായും നീ കാരുണ്യവാന്മാരില്‍ അങ്ങേയറ്റം കാരുണ്യവാനാണ്. ശേഷം പ്രവാചകന്‍ നാല് തവണ തക്ബീര്‍ ചൊല്ലി (മയ്യിത്ത് നമസ്‌കരിച്ചു) ശേഷം അവരെ (ഫാത്തിമ ബിന്‍ത് അസദിന്റെ മൃതദേഹം) നബി(സ്വ)യും അബ്ബാസും അബൂബഖറും ചേര്‍ന്ന് (റ)ഖബ്‌റിലേക്ക് പ്രവേശിപ്പിച്ചു.” (ത്വബ്റാനി, അല്‍ കബീര്‍: 24/351, അല്‍ ഹില്‍യ: അബൂ നുഐം: 3/121).

ഇതാണ് പ്രവാചകനു മേല്‍ ശവരതിയാരോപിക്കുവാന്‍ നബിവൈരികള്‍ (ഹദീഥ് പൂര്‍ണരൂപത്തില്‍ കൊടുക്കാതെ ഹദീഥ് നമ്പര്‍ മാത്രം നല്‍കികൊണ്ട്) ഉദ്ദരിക്കുന്ന ‘തെളിവ്’. ഹദീഥിന്റെ സ്വീകാര്യത പരിശോദിക്കും മുമ്പ് അതില്‍ പറഞ്ഞ കാര്യങ്ങള്‍ നമുക്കു വിശകലനം ചെയ്യാം.

1, മരണപ്പെട്ടുപോയ തന്റെ പോറ്റുമ്മയോട് ഒരു മകനെന്ന നിലയില്‍ പ്രവാചകനുണ്ടായിരുന്ന അളവറ്റ ആദരവും സ്നേഹവും വാത്സല്യവും കാരുണ്യവും ആര്‍ദ്രതയുമെല്ലാം ഹദീഥില്‍ നിന്നും കാഴ്ചയുള്ള കണ്ണുകള്‍ക്കെല്ലാം വ്യക്തമായും കാണാവുന്നതാണ്. അതെല്ലാം കണ്ടിട്ടും, ആ മകനും മാതാവിന്നുമിടയിലെ വേര്‍പാടിന്റെ ദുഃഖം തളംകെട്ടിയ ചരിത്ര നിമിഷങ്ങളില്‍ നിന്നുപോലും അശ്ലീലതകള്‍ ചികഞ്ഞെടുക്കാന്‍ ലൈംഗിക വൈകൃതങ്ങള്‍കൊണ്ട് ഹൃദയം നുരുമ്പിച്ചവര്‍ക്കല്ലാതെ സാധ്യമല്ല.

2, പ്രവാചകന്‍ തന്റെ മേല്‍കുപ്പായം ഊരി ഫാത്തിമ ബിന്‍ത് അസദിന്റെ മൃതദേഹത്തെ അണിയിക്കുന്നത് അവരെ ഖബ്റില്‍ വെക്കുന്നതിനും എത്രയോ മുമ്പാണെന്ന് ഹദീഥില്‍ നിന്നും സുവ്യക്തമാണ്. അതും അവരെ മൂടിയ വസ്ത്രത്തിനു മീതെയായിരുന്നു എന്നതും ഹദീഥില്‍ കാണാം. ഖബ്‌റിലിറങ്ങി വസ്ത്രമുരിഞ്ഞ് ശവരതിയിലേര്‍പ്പെട്ടു എന്ന കല്ലുവെച്ച കളവ് ഞരമ്പുരോഗികളുടെ ഭാവനാ വിന്ന്യാസങ്ങള്‍ മാത്രമാണ്.

3, പ്രവാചകന്‍ കിടന്നത് ഖബ്‌റിലെ ലഹ്ദിലാണ്. അവിടുന്ന് തന്നെയാണ് ലഹ്ദ് കുഴിച്ചതും. എന്താണ് ലഹ്ദ്? ഖബ്റിന്റെ പാര്‍ശ്വം ഖിബ്‌ലയുടെ ഭാഗത്തേക്ക് തുരന്നുണ്ടാക്കുന്ന ഉള്‍ഖബ്‌റിനാണ് ലഹ്ദ് എന്നു പറയുക (ഫിഖ്ഹുസ്സുന്നഃ, ഭാഗം 4, പേജ്: 398). അഥവാ ലഹ്ദ് മയ്യിത്തിനെ മാത്രം ഉള്‍കൊള്ളാന്‍ പര്യാപ്തമായ ഖബ്‌റിന്റെ ഏറ്റവും താഴെ പാര്‍ശ്വം ഖിബ്‌ലയുടെ ഭാഗത്തേക്ക് തുരന്നുണ്ടാക്കുന്ന ഇടുങ്ങിയ ഉള്‍ഭാഗമാണ്. അവിടെ മയ്യിത്ത് വെക്കും മുമ്പാണ് പ്രവാചകന്‍ കിടന്നത്. ഖബ്‌റിന്റെ ആത്മീയവും ഭൗതികവുമായ ഇടുക്കത്തില്‍ നിന്നും തന്റെ പോറ്റുമ്മക്ക് വിശാലത ലഭിക്കാനായാണ് നബി (സ്വ) അങ്ങനെ ചെയ്തത്. മയ്യിത്തിനെ മാത്രം ഉള്‍കൊള്ളാന്‍ പര്യാപ്തമായ ഇടുങ്ങിയ ഉള്‍ഭാഗമായ ലഹ്ദില്‍ മയ്യിത്തിനൊപ്പം മറ്റൊരാള്‍ക്ക് കൂടി കിടക്കുവാന്‍ സാധ്യമല്ലെന്ന് ലഹ്ദ് കണ്ടിട്ടുള്ളവര്‍ക്കെല്ലാം അറിയാം. അത് കൃത്യമായി അറിയാവുന്നതു കൊണ്ടാണ് ശവരതിയാരോപകര്‍ നബി ലഹ്ദില്‍ കിടന്നു എന്നു പറയാതെ, ഖബ്‌റില്‍ കിടന്നു എന്നു പറഞ്ഞുകൊണ്ടിരിക്കുന്നത്. കാരണം ഹദീഥുകളില്‍ നബി (സ്വ) കിടന്നത് ലഹ്ദിലാണെന്ന് വ്യക്തമാക്കിയിട്ടും അതു മൂടിവെക്കുന്നത് ലഹ്ദ് എന്താണെന്നറിയുന്നവര്‍ക്കിടയില്‍ ശവരതിയാരോപണത്തിന്റെ കാറ്റുപോകുമെന്ന് കൃത്യമായ ധാരണയുള്ളതുകൊണ്ടു മാത്രമാണ്.

4, ലഹ്ദില്‍ കിടന്നതിന് ശേഷം അതില്‍ നിന്നും പുറത്തുവന്നതിനു ശേഷമാണ് നബി (സ്വ) ഫാത്തിമ ബിന്‍ത് അസദിന്റെ മയ്യിത്ത് നമസ്‌കരിക്കുന്നത്. അതിനും ശേഷമാണ് അവരുടെ മയ്യിത്ത്, അബ്ബാസിനോടും അബൂബഖറിനോടും ചേര്‍ന്ന് നബി (സ്വ) ഖബ്റിലെ ലഹ്ദിലേക്ക് ഇറക്കി വെക്കുന്നത്. അപ്പോള്‍ ഖബ്‌റില്‍ വെച്ച് പ്രവാചകന്‍ ശവരതി നടത്തിയെന്നത് കല്ലുവെച്ച കള്ളമാണെന്നര്‍ഥം.

5. എന്തിനാണ് പ്രവാചകന്‍ (സ) ലഹ്ദില്‍ അവരുടെ സ്ഥാനത്ത് കിടന്നത് എന്നതും സുവ്യക്തമാണ്.

‘അദ്ദേഹം കബ്‌റിന്റെ (ലഹ്ദില്‍) അതിരുകള്‍ തിക്കി. അത് വിശാലമാക്കുന്നതു പോലെ…’ (അല്‍ മുസ്തദ്‌റക്: ഹാകിം: 3/108?)

ഉമര്‍ ഇബ്‌നു ശബ്ബ: പറഞ്ഞു: ‘അദ്ദേഹം ലഹ്ദില്‍ കിടന്നു; അത് (അതിരുകള്‍ ഒതുക്കി) വിശാലമാക്കാനെന്ന പോലെ.’ (താരീഖുല്‍ മദീന: 1: 124)

ലഹ്ദില്‍ കിടന്നാല്‍ മാത്രമെ ലഹ്ദിന്റെ ഇടുക്കവും കുടുസ്സതയും ഒരു മയ്യിത്തിന് (മൃതശരീരം) എത്രയുണ്ടെന്ന് കൃത്യമായി മനസ്സിലാക്കാന്‍ സാധിക്കു. അതുകൊണ്ട് ഫാതിമയുടെ മയ്യിത്ത് ലഹ്ദില്‍ വെക്കുന്നതിന് മുമ്പ് പ്രവാചകന്‍ ആ സ്ഥാനത്ത് കിടന്നു നോക്കി. എന്നിട്ട് ലഹ്ദില്‍ ഇടുങ്ങി നില്‍ക്കുന്ന അതിരുകള്‍ തിക്കി ഒതുക്കി. ലഹ്ദില്‍ വെക്കാന്‍ പോകുന്ന മയ്യിത്തിന് വേണ്ടി അത് വിശാലമാക്കി. ഇപ്രകാരം ഭൗതീകമായി, പ്രതീകാത്മായി ലഹ്ദ് വിശാലമാക്കുന്നതിലൂടെ കബ്‌റിലെ ആത്മീയ വാസത്തിലും ഫാതിമക്ക് ഈ വിശാലത അനുഭവപ്പെടാന്‍ വേണ്ടി ആ കര്‍മ്മത്തിലൂടെ അദ്ദേഹം പ്രാര്‍ത്ഥിച്ചു. കാരണം എല്ലാ മനുഷ്യരും കബ്‌റില്‍ ആത്മീയമായി ‘ഇടുക്കം’ അനുഭവിക്കുമെന്ന് പ്രവാചകന്‍ (സ) തന്നെ പഠിപ്പിച്ചിട്ടുണ്ട്. (മുസ്‌നദു അഹ്‌മദ്: 24707, ശുഅ്ബുല്‍ ഈമാന്‍: 396)

‘അവളുടെ ഖബ്റില്‍ അവരോടൊപ്പം ഞാന്‍ കിടന്നത് അവള്‍ക്ക് ഖബ്‌റിന്റെ ഇടുക്കം ലളിതമാകാന്‍ വേണ്ടിയാണ്’ എന്ന് പറഞ്ഞതിന്റെ വിവക്ഷ ഇതാണ്. ലൈംഗിതയുമായി അയല്‍ബന്ധം പോലുമില്ലാത്ത, മാതാവിന്റെ പരലോക ജീവിതത്തെ സംബന്ധിച്ച് ഒരു മകന്റെ തീര്‍ത്തും നിഷ്‌കളങ്കമായ വ്യാകുലത മാത്രമാണ് ഈ ലഹ്ദിലെ കിടത്തം തെളിയിക്കുന്നത്.

6, സ്വാഭാവികമായും പകല്‍ വെളിച്ചത്ത്, പൊതുജന മധ്യത്തില്‍, മയ്യിത്തിന്റെ ബന്ധുക്കളുടെ സാന്നിധ്യത്തില്‍ വെച്ചു നടന്ന മൃതദേഹ സംസ്‌കരണ ചടങ്ങായിരുന്നു അതെന്ന മിനിമം ബോധമില്ലാതെയല്ല ഈ ലൈംഗിക വൈകൃതാനുരാഗികള്‍ ഇത്തരം കള്ള പ്രചരണം അഴിച്ചു വിട്ടുകൊണ്ടിരിക്കുന്നത്. അടങ്ങാത്ത നബിവൈര്യവും ഹൃദയത്തില്‍ കട്ടപിടിച്ചു കിടക്കുന്ന ലൈംഗിക വൈകൃതാനുരാഗവും കെട്ടുപിണഞ്ഞ മനോരോഗത്തെ ചികിത്സിക്കുവാനുള്ള കരുത്ത് അവരുടെയൊന്നും ശാസ്ത്രാവബോധത്തിനോ സുവിശേഷ ദര്‍ശനങ്ങള്‍ക്കോ സനാധന മൂല്യങ്ങള്‍ക്കോ അശേഷമില്ലാത്തതുകൊണ്ടാണ്.

ആരോപണ വിധേയമായ ഹദീഥിന്റെ ന്യൂനതകള്‍:

നബിവൈരികള്‍ പ്രവാചകനെതിരെ ഉന്നയിക്കുന്ന ശവരതിയാരോപണം കേവലം അവരുടെ മനോവൈകൃതങ്ങളുടെ ഭാവനാ വിന്ന്യാസങ്ങള്‍ മാത്രമാണെന്നു നാം മനസ്സിലാക്കി. ആരോപണം ആരോപിക്കപ്പെട്ട ഹദീഥുകളില്‍ ഭൂതകണ്ണാടി വെച്ചു തിരഞ്ഞാല്‍ പോലും കാണാനാവില്ലെന്ന വസ്തുത നിലനില്‍ക്കെ തന്നെ പ്രസ്തുത ഹദീഥുകളുടെ സ്വീകാര്യതയെ കൂടി നമുക്കു പഠനവിധേയമാക്കാം. ഹദീഥ്, സ്വീകാര്യതയുടെ മാനദണ്ഡങ്ങള്‍ പൂര്‍ത്തീകരിക്കപ്പെട്ടതാണോ അല്ലയോ എന്ന് പരിശോദിക്കുക ഇസ്‌ലാമിക വൈജ്ഞാനിക രംഗത്ത് വളരെ പ്രധാനപ്പെട്ട കാര്യമാണല്ലോ? കാരണം സ്വീകാര്യയോഗ്യമായ ഹദീഥുകള്‍ക്ക് മാത്രമാണല്ലോ പ്രമാണപരതയുണ്ടാവുക. അല്ലാത്തവ മുസ്‌ലിംകള്‍ പ്രമാണമായി സ്വീകരിക്കാന്‍ പാടുള്ളതല്ലല്ലോ. അതുകൊണ്ടു തന്നെ ഉപര്യുക്ത ഹദീഥുകളുടെ സ്വീകാര്യത കൂടി നമുക്കന്വേഷണവിധേയമാക്കാം.

അഞ്ചു പരമ്പരകളിലൂടെയാണ് ഈ സംഭവം ഉദ്ധരിക്കപ്പെട്ടിരിക്കുന്നത്.

1. (ത്വബ്റാനി: അല്‍കബീര്‍: 24/351, അല്‍ ഹില്‍യ: അബൂ നുഐം: 3/121)

നിവേദക പരമ്പര: അഹ്‌മദിബ്നു ഹമ്മാദ് അസ്സഗ്ബയില്‍ നിന്ന് – റൗഹിബ്നു സ്വലാഹ് നമ്മോട് പറഞ്ഞു – സുഫ്‌യാനു സൗരി നമ്മോട് പറഞ്ഞു – ആസ്വിം അല്‍ അഹ്‌വലില്‍ നിന്ന് – അനസില്‍ നിന്ന്….

പരമ്പരയിലെ റൗഹിബ്നു സ്വലാഹ് ദുര്‍ബലനാണെന്ന് ഹദീഥ് പണ്ഡിതനായ ഇബ്നുഅദിയ്യ് വ്യക്തമാക്കിയിട്ടുണ്ട്. വളരെ ദുര്‍ബലമായ ഹദീഥുകള്‍ ഉദ്ധരിക്കാറുണ്ടെന്ന് അഭിപ്രായപ്പെടുകയും ചെയ്തു. ഇബ്നു മാകൂലാ പറയുന്നു: റൗഹിബ്നു സ്വലാഹിനെ ഹദീഥ് പണ്ഡിതന്മാര്‍ ദുര്‍ബലനായാണ് കാണുന്നത്. ഇബ്നു യൂനുസ് പറയുന്നു: വിശ്വസ്ഥരായ നിവേദകര്‍ക്കെതിരായി വളരെ ദുര്‍ബലമായ ഹദീഥുകള്‍ അയാള്‍ ഉദ്ധരിക്കാറുണ്ട്. ദാറകുത്നി പറഞ്ഞു: ഹദീഥിന്റെ വിഷയത്തില്‍ ദുര്‍ബലന്‍. (അസ്സികാത്ത്: ഇബ്നുഹിബ്ബാന്‍: 8/ 244, അല്‍ കാമില്‍: 3/1006, മീസാന്‍: 2/58, അല്ലിസാന്‍: 2/465466)

കൂടാതെ ദുര്‍ബലനായ റൗഹിബ്നു സ്വലാഹ് മാത്രമാണ് സുഫ്‌യാനു സൗരിയില്‍ നിന്നും ഈ കഥ ഉദ്ധരിക്കുന്നത് എന്നതും റൗഹിബ്നു സ്വലാഹ് ഈജിപ്റ്റുകാരനും സുഫ്യാനു സൗരി കൂഫക്കാരനുമായതിനാല്‍ റൗഹിബ്നു സ്വലാഹ്, സുഫ്‌യാനു സൗരിയില്‍ നിന്ന് ഇങ്ങനെയൊരു കഥ കേള്‍ക്കാന്‍ സാധ്യതയില്ല എന്നതും നിവേദക പരമ്പരയുടെ മറ്റു ന്യൂനതകളായി ഹദീഥ് പണ്ഡിതര്‍ സൂചിപ്പിക്കുന്നുണ്ട്. (മുകദ്ദിമ സ്വഹീഹു മുസ്‌ലിം: 1/7, അല്‍ അവ്‌സത്: ത്വബ്റാനി: 1/153, അല്‍ ഹില്‍യ: 3/121, സില്‍സിലത്തു ദഈഫ: 1/32, അസ്സികാത്ത്: 8/244)

2. (മജ്‌മഉ സവാഇദ്:9/257, അവ്സത്ത്: ത്വബ്റാനി)

നിവേദക പരമ്പര: സഅ്ദാന്‍ ഇബ്നുല്‍ വലീദില്‍ നിന്ന് – അത്വാഅ് ഇബ്നു അബീ റബാഹില്‍ നിന്ന് – ഇബ്നു അബ്ബാസ് പറഞ്ഞു….

നിവേദക പരമ്പരയിലെ സഅ്ദാന്‍ ഇബ്നുല്‍ വലീദ് ‘മജ്ഹൂല്‍’ (വ്യക്തിത്വമോ വിശ്വസ്ഥതയോ അറിയപ്പെടാത്ത വ്യക്തി) ആണ്. (മജ്‌മഉ സവാഇദ്: 9/257)

3. (താരീഖുല്‍ മദീന: ഇബ്നു ശബ്ബ: 1/124)

നിവേദക പരമ്പര: കാസിം ഇബ്നു മുഹമ്മദുല്‍ ഹാശിമി പറഞ്ഞു- അയാള്‍ തന്റെ പിതാമഹനില്‍ നിന്ന് – അയാള്‍ ജാബിറില്‍ നിന്ന്….

പരമ്പര വളരെ ദുര്‍ബലമാണ്. കാരണം കാസിം ഇബ്നു മുഹമ്മദുല്‍ ഹാശിമി ഹദീഥ് നിവേദനത്തില്‍ പരിഗണനീയനേയല്ല എന്ന് സര്‍വ്വ ഹദീഥ് പണ്ഡിതരും വ്യക്തമാക്കിയിട്ടുണ്ട്. അബൂ ഹാതിം പറഞ്ഞു: കാസിം ഇബ്നു മുഹമ്മദുല്‍ ഹാശിമി, ‘മത്റൂക്’ (കളവ് പറയുന്നവനായി ആരോപിതന്‍) ആകുന്നു.

ഇമാം അഹ്‌മദ് പറഞ്ഞു: അയാള്‍ ഹദീഥിന്റെ വിഷയത്തില്‍ ഒന്നുമല്ല. അബൂ സര്‍അ പറഞ്ഞു: വിശ്വസ്ഥരായ നിവേദകര്‍ക്കെതിരായി വളരെ ദുര്‍ബലമായ ഹദീഥുകള്‍ അയാള്‍ ഉദ്ധരിക്കാറുണ്ട്. (മീസാനുല്‍ ഇഅ്തിദാല്‍: 3/379)

4. (താരീഖുല്‍ മദീന: ഇബ്നു ശബ്ബ: 1/123)

നിവേദക പരമ്പര: അബ്ദുല്‍ അസീസ് ഇബ്നു മുഹമ്മദ് അദ്ദുറാവര്‍ദി- അയാള്‍ അബ്ദുല്ലാഹിബ്നു ജഅ്ഫറില്‍ നിന്ന് – അയാള്‍ അംറിബ്നു ദീനാറില്‍ നിന്ന് – അദ്ദേഹം മുഹമ്മദിബ്നു അലിയില്‍ നിന്ന്….

പരമ്പര ദുര്‍ബലം: അബ്ദുല്‍ അസീസ് ഇബ്നു മുഹമ്മദ് അദ്ദുറാവര്‍ദി ദുര്‍ബലനാണ്. ഹൃദ്യസ്ഥ ശേഷി കുറവായതിനാല്‍ ധാരാളം അബദ്ധങ്ങള്‍ ഉദ്ധരിക്കാറുണ്ടെന്ന് ഹദീഥ് പണ്ഡിതര്‍ വ്യക്തമാക്കിയിട്ടുണ്ട്. (മീസാനുല്‍ ഇഅ്തിദാല്‍: 2/633634)

മാത്രമല്ല നിവേദക പരമ്പര ‘മുര്‍സല്‍’ ആകുന്നു അഥവാ പ്രവാചകനിലേക്കെത്താതെ കണ്ണി മുറിഞ്ഞതാകുന്നു. പ്രവാചക ശിഷ്യനല്ലാത്ത മുഹമ്മദുല്‍ ഹനഫിയ്യയാണ് കഥ പറയുന്നത്.

5. മുഹമ്മദിബ്നു ഉമറുബ്നു അലിയില്‍ നിന്ന് പരമ്പര മുറിഞ്ഞതാണ് മറ്റൊരു നിവേദനം. (ഉസ്ദുല്‍ ഗായ: 6/217)

നിവേദക പരമ്പര: ഇബ്നുല്‍ അസീറില്‍ നിന്ന്- അബ്ദുല്ലാഹിബ്നു മുഹമ്മദിബ്നു ഉമറുബ്നു അലി പിതാവില്‍ നിന്നും ഉദ്ധരിക്കുന്നു….

പരമ്പരയിലെ അബ്ദുല്ലാഹിബ്നു മുഹമ്മദിബ്നു ഉമറുബ്നു അലി തന്റെ പിതാമഹനായ അലിയില്‍ നിന്നും ഉദ്ധരിക്കുന്ന നിവേദനങ്ങളെല്ലാം പരമ്പര മുറിഞ്ഞവയാണെന്ന് ഹദീഥ് പണ്ഡിതര്‍ വ്യക്തമാക്കിയിട്ടുണ്ട്. (അത്തക്‌രീബ്: ഇബ്നു ഹജര്‍: 6170). പിന്നെ എങ്ങനെ പ്രവാചകനില്‍ നിന്ന് അദ്ദേഹം നിവേദനം ചെയ്യും.?!

ഹദീഥുകളില്‍ നിന്നും വിമര്‍ശകന്മാര്‍ മുഹമ്മദിന്റെ മുഖമൂടി ഊരിയെടുക്കുമ്പോള്‍, അതിനു മുമ്പില്‍ കുടുങ്ങുന്ന വേളയില്‍ മുസ്‌ലിംകള്‍ പുറത്തെടുക്കുന്ന അവസാനത്തെ അടവാണ് ‘ഹദീഥ് ദുര്‍ബലമാക്കുന്നു’ എന്ന പ്രഖ്യാപനമെന്ന്, ഹദീഥ് നിദാനശാസ്ത്രത്തിന്റെ ബാല പാഠം പോലും കേട്ടിട്ടില്ലാത്ത ചില നാസ്തിക-മിഷനറി ഇസ്‌ലാം വിമര്‍ശകര്‍ തട്ടിവിടാറുണ്ട്. അവരോടു പറയട്ടെ, മുസ്‌ലിംകള്‍ ഹദീഥുകളെ കാണുന്നത് ദൈവിക വെളിപാടുകളായാണ്. അതുകൊണ്ടു തന്നെ അവര്‍ക്ക് അതിന്റെ പ്രാമാണികത കണ്ടെത്താന്‍ കൃത്യമായ മാനദണ്ഡങ്ങളുണ്ട്. അത് പൗരാണിക കാലം തൊട്ടേ അവര്‍ സ്വീകരിച്ചിട്ടുള്ള നടപടിക്രമമാണ്. പൗരാണികരും ആധുനികരുമായ ലോക മുസ്‌ലിം പണ്ഡിതന്മാരെല്ലാം സര്‍വാംഗീകൃതമായി അംഗീകരിച്ചിരിക്കുന്ന ആ നിയമം നമുക്കും മനസ്സിലാക്കാം. ഒരു ഹദീഥ് തള്ളുന്നതും കൊള്ളുന്നതും എന്തിന്റെയടിസ്ഥാനത്തിലാണെന്ന് അപ്പോള്‍ ബോധ്യമാകും.

1, നിവേദക പരമ്പരയിലെ സര്‍വ്വ നിവേദകര്‍ക്കും അദാലത്ത് ഉണ്ടാവുക. അഥവാ നിവേദക പരമ്പരയിലെ സര്‍വ്വ നിവേദകരും വിശ്വസ്ഥരും നീതിമാന്മാരും ഭക്തരുമാവുക.

2, നിവേദക പരമ്പരയിലെ സര്‍വ്വ നിവേദകരുടേയും ഓര്‍മ്മശക്തി സമ്പൂര്‍ണമായിരിക്കുക.

3, നിവേദക പരമ്പര കണ്ണി ചേര്‍ന്നതാവുക; മുറിഞ്ഞതാവാതിരിക്കുക. അഥവാ പരമ്പരയിലെ നിവേദകര്‍ ഏത് നിവേദകരില്‍ നിന്നാണോ ഒരു ഹദീഥ് ഉദ്ധരിക്കുന്നത്, അവര്‍ പരസ്പരം കണ്ടുമുട്ടുകയും നേരിട്ട് ആ ഹദീഥ് കേള്‍ക്കുകയും ചെയ്തിട്ടുണ്ടെന്ന് ഉറപ്പുണ്ടാവുക.

4, നിവേദനത്തിന്റെ പരമ്പരയും (സനദ്), ഉള്ളടക്കവും (മത്നും) കൂടുതല്‍ വിശ്വസ്ഥരും ശ്രേഷ്ടരുമായ നിവേദകര്‍ക്കോ നിവേദനങ്ങള്‍ക്കോ എതിരാവാതിരിക്കുക.

5, നിവേദനം ഇല്ലത്ത് അഥവാ സൂക്ഷ്മമായ ന്യൂനതകളില്‍ നിന്ന് മുക്തമാകണം.

ഈ നിബന്ധനകള്‍ ഒത്ത ഹദീഥുകളെയാണ് ‘സ്വഹീഹ്’ എന്ന് മുസ്‌ലിംകള്‍ വിളിക്കുന്നത്. മുകളില്‍ പ്രസ്ഥാവിച്ച അഞ്ച് നിബന്ധനകളില്‍ ഒന്നില്‍ ന്യൂനതകള്‍ ഉള്ളതായ ഹദീഥുകള്‍ ള്വഈഫ് (ദുര്‍ബലം) ആയ ഹദീഥുകളാകുന്നു. അവ ഇസ്‌ലാമില്‍ പ്രമാണമല്ല. പൗരാണികരും ആധുനികരുമായ ലോക മുസ്‌ലിം പണ്ഡിതന്മാരെല്ലാം അര്‍ത്ഥശങ്കക്കിടയില്ലാത്ത വിധം വ്യക്തമാക്കിയ സര്‍വാംഗീകൃതമായ നിയമമാണിത്. (അര്‍രിസാല: ശാഫിഈ: 370-371, നുസ്ഹത്തുന്നളര്‍: 52, നുഖ്ബത്തുല്‍ ഫികര്‍: ഇബ്നു ഹജര്‍: 30, ഉലൂമുല്‍ ഹദീസ്: 30, മുഖദ്ദിമത്തു ഇബ്നു സ്വലാഹ്: 8, അല്‍ മൂകിദ: ദഹബി: 24, തദ്രീബുര്‍ റാവി: സുയൂത്വി: 1/6875, 155, അല്‍ഫിയ്യ: 19, മന്‍ദൂമത്തുല്‍ ബൈകൂനി: 30, ഹാശിയത്തുല്‍ അജ്ഹുരി: 6, ഇഖ്തിസ്വാറു ഉലൂമുല്‍ ഹദീസ്: ഇബ്നു കസീര്‍: 22, അല്‍ മുക്നിഅ്: ഇബ്നു മുലകിന്‍: 1/42, അല്‍ ജാമിഅ്: ഖത്തീബുല്‍ ബഗ്‌ദാദി: 2/295, അല്‍ ഇക്തിറാഹ്: ഇബ്നു ദകീകുല്‍ ഈദ്: 215 – 216, മുകദ്ദിമത്തു മുസ്‌ലിം: ഇമാം നവവി , അത്തക്‌രീബ്: 105,മജ്‌മഉല്‍ ഫതാവാ ഇബ്നു തീമിയ്യ: 1/250, മആലിമുസ്സുനന്‍: ഖത്താബി: 1/10, ശര്‍ഹുല്‍ അല്‍ഫിയ: ഇറാകി: 1/111, സ്വഹീഹു തര്‍ഗീബു വതര്‍ഹീബ്: 1/4767)

ശവരതിയാരോപണത്തിന്റെ അവസാന പുകയും ഇവിടെ കെട്ടടങ്ങുകയാണ്. വിശുദ്ധിയുടെ വിസ്മയ പ്രകാശമായ നബിജീവിതത്തെ അപകീര്‍ത്തിപ്പെടുത്താനുള്ള മിഷനറി-നവനാസ്തിക വൈകൃതാനുരക്തരുടെ എല്ലാ പരിശ്രമങ്ങളും പാഴ്‌വേലയായി പരിസമാപ്തി കുറിക്കപ്പെടുകയാണ് സ്ഥിരം പതിവ്. പ്രവാചകനെതിരെയുള്ള ശവരതിയാരോപണത്തിന്റെ ഊര്‍ദ്ധ്വശ്വാസവും ഇവിടെ നിലച്ചിരിക്കുന്നു. എങ്കിലും, എങ്ങനെയാണ് ഹദീഥുകളില്‍ നിന്നും – അവ പ്രബലമോ ദുര്‍ബലമോ എന്നു പോലും പരിഗണിക്കാതെ – പ്രവാചക വിരോധികള്‍ ദുരാരോപണങ്ങളും കള്ള കഥകളും മെനഞ്ഞെടുക്കുന്നതെന്ന് പഠിക്കുവാനുള്ള ഒരു സന്ദര്‍ഭമായി ഈ വിഷയസംബന്ധിയായ ചര്‍ച്ചയെ നാം ഉപയോഗപ്പെടുത്തേണ്ടിയിരിക്കുന്നു. ഈ ദുരാരോപണത്തിന്റെ, അഥവാ ശവരതിയാരോപണത്തിന്റെ നിര്‍മിതിയുടെ വഴിയിടങ്ങളിലൂടെ സഞ്ചരിച്ചാല്‍ മാത്രം മതി നമുക്കതു നിഷ്പ്രയാസം കണ്ടെത്താനാകും. പ്രവാചകനു മേല്‍ നികൃഷ്ഠമായ ഒരാരോപണം നിര്‍മിക്കുന്നതിനായി നബിവൈരികള്‍ ഹദീഥ് ഗ്രന്ഥങ്ങള്‍ തിരയുന്നു. നിര്‍മാതാക്കള്‍ ലൈംഗിക വൈകൃതങ്ങള്‍ക്ക് അടിപ്പെട്ടവരായതുകൊണ്ടു തന്നെ പ്രവാചകനെതിരെ ലൈംഗിക വൈകൃതം ആരോപിക്കുന്നതിലായിരിക്കും അവരുടെ ഉത്സാഹം മുഴുവനും. ഹദീഥ് ഗ്രന്ഥങ്ങള്‍ തിരഞ്ഞപ്പോള്‍ അവര്‍ക്കു കിട്ടിയത് ഫാത്തിമ ബിന്‍ത് അസദിന്റെ മൃതദേഹ സംസ്‌കരണത്തിന്റെ ഹദീഥുകള്‍. ഹദീഥുകള്‍ പരിശോദിച്ചപ്പോള്‍ എല്ലാം ദുര്‍ബലമായ നിവേദനങ്ങളാണെന്ന് കാണുന്നു. അതുകൊണ്ടു തന്നെ ഹദീഥുകളുടെ സമാഹാരങ്ങളും അവയിലെ ക്രമനമ്പറുകളും തെറ്റായി ചേര്‍ത്തുകൊണ്ട് തങ്ങളുടെ ദുര്‍വ്യാഖ്യാന അച്ചടിശാലകളില്‍ അച്ചുനിരത്തുന്നു. ശരിയായ നിലക്ക് അവ ഉദ്ദരിച്ചാല്‍ അവയുടെ ദുര്‍ബലതയും നിജസ്ഥിതിയും കൈയോടെ പെടുന്നനെ പിടിക്കപ്പെടുമെന്ന് അവര്‍ക്കറിയാം. സമാഹാരങ്ങളും ക്രമനമ്പറുകളും തെറ്റായി ചേര്‍ത്തവതരിപ്പിക്കുമ്പോള്‍ കൈക്രിയകള്‍ കണ്ടെത്താന്‍ ചെറിയ കാലതാമസം വരും. ആ ഹൃസ്വകാലയളവില്‍ മറുപടി പറയപ്പെടാതെ നില്‍ക്കുന്ന തങ്ങളുടെ കള്ള കഥകളെ സമൂഹ മധ്യത്തില്‍ വെല്ലുവിളികളോടെ പ്രചരിപ്പിച്ചുകൊണ്ടിരിക്കുക. നബിവിരോധം അത്രയെങ്കിലും സജീവമാക്കുക.

പ്രവാചകനെതിരെ ശവരതിയാരോപണമാണ് അവര്‍ ലക്ഷ്യം വെച്ചതെന്നതുകൊണ്ട് തന്നെ അതിനൊപ്പിച്ചു ദുര്‍വ്യാഖ്യാനം ചമക്കാനൊക്കുന്ന ചരിത്ര രംഗങ്ങള്‍ നിവേദനങ്ങളില്‍ ആദ്യം പരതുന്നു. അവിടെ നോക്കുമ്പോള്‍ ആകെ കൂടി കാണാന്‍ കഴിയുന്നത്, ഫാത്തിമ ബിന്‍ത് അസദ് മരണപ്പെടുന്നു. നബി (സ്വ) അവര്‍ക്കുവേണ്ടി പ്രാർത്ഥിക്കുന്നു. അവര്‍ നബിക്കു ചെയ്ത സേവനങ്ങള്‍ അനുസ്മരിക്കുന്നു. ശേഷം അവരെ കുളിപ്പിക്കാന്‍ നിര്‍ദ്ദേശിക്കുന്നു. അനന്തരം അവരെ പൊതിഞ്ഞ വസ്ത്രത്തിനുമേല്‍ നബിയുടെ മേല്‍കുപ്പായമണിയിക്കുന്നു. അതിനു ശേഷം അവര്‍ക്കു വേണ്ടി ഖബ്ര്‍ കുഴിക്കാനാവശ്യപ്പെടുന്നു. ഖബ്‌റിന്റെ ലഹദ് എത്താറായപ്പോള്‍ അത് നബിതന്നെ കുഴിക്കുന്നു. അനന്തരം അവിടുന്ന് ഒരാള്‍ക്ക് മാത്രം കിടക്കാന്‍ കഴിയുന്ന ലഹ്ദില്‍ കിടക്കുന്നു. പ്രാർത്ഥിക്കുന്നു. അതിനു ശേഷം തക്ബീര്‍ ചൊല്ലി മയ്യിത്ത് നമസ്‌കരിക്കുന്നു. അതിനും ശേഷം മറ്റുള്ളവരുടെ സഹായത്തോടെ മയ്യിത്ത് ഖബ്റിലെ ലഹദില്‍ വെക്കുന്നു. ജനങ്ങളും മയ്യിത്തിന്റെ ഉറ്റ ബന്ധുക്കളും പ്രസ്തുത ചടങ്ങുകള്‍ക്ക് സാക്ഷികളാകുന്നു. പക്ഷെ, കഥ ഇതല്ല അവര്‍ക്കുവേണ്ടത്. മയ്യിത്ത് നഗ്‌നമായി ഖബ്റില്‍ വെക്കുന്നു. അല്ലെങ്കില്‍ ഖബ്‌റില്‍ വെച്ച മയ്യിത്തില്‍ നിന്നും വസ്ത്രമുരിയുന്നു. എന്നിട്ട് തന്റെ വസ്ത്രമഴിക്കുന്നു. ശവഭോഗം ചെയ്യുന്നു. ശേഷം തന്റെ വസ്ത്രം മയ്യിത്തിനെ അണിയിക്കുന്നു. എന്നിട്ട് പ്രവാചകന്‍ പറയുന്നു: ‘ഞാന്‍ എന്റെ കുപ്പായം അവളെ അണിയിച്ചു. ഖബ്റിലെ വേദന അകറ്റുന്നതിന്’ ഇതാണ് അവരുടെ വൈകൃത ഭാവനകള്‍ മെനഞ്ഞെടുത്ത ചിത്രം. പക്ഷെ നബി തന്റെ വസ്ത്രം മയ്യിത്തിനെ അണിയിപ്പിച്ചത് മയ്യിത്ത് കുളിപ്പിച്ച ശേഷം, അതിനെ പൊതിഞ്ഞ പുടവക്കുമേല്‍. അവിടുന്ന് ലഹ്ദില്‍ കിടന്നത് മയ്യിത്ത് വെക്കും മുമ്പ്. അതിനു ശേഷം മയ്യിത്ത് നമസ്‌കരിക്കുന്നു. അതിനും ശേഷമാണ് മയ്യിത്ത് ഖബ്‌റിലെ ലഹദില്‍ വെക്കുന്നത്. കഥ ഒരു നിലക്കും ഭാവനകള്‍ക്കൊത്ത് ശരിയായി വരുന്നില്ല. അപ്പോള്‍ പിന്നെ തങ്ങളുടെ ഭാവനകള്‍ക്കൊത്ത് കഥ തിരുത്തിയെഴുതാന്‍ പറ്റിയ വല്ല വാചകങ്ങളും നിവേദനങ്ങളില്‍ നിന്നും (നിവേദനങ്ങളെല്ലാം ദുര്‍ബലമാണെങ്കിലും) അടര്‍ത്തിയെടുക്കാനുണ്ടോ എന്നതാണ് അടുത്ത റിസെര്‍ച്ച്. ഇതാ കിട്ടിപ്പോയി ത്വബ്റാനിയുടെ അല്‍ മുഅ്ജമുല്‍ ഔസത്ത്. അവിടെ 6935 മത്തെ നിവേദനത്തില്‍ നിന്നും ഒരു വാചകം മാത്രം അടര്‍ത്തിയെടുക്കാം. ‘വള്വ്ത്വജഅത്തു മഅഹാ ഫീ ഖബ്രിഹാ’ (അവരുടെ ഖബ്റില്‍ അവളോടൊപ്പം ഞാന്‍ കിടന്നത്…). അത്രമാത്രം അങ്ങ് അടര്‍ത്തിയെടുത്ത് തങ്ങളുടെ ഭാവനകള്‍ക്കൊത്ത് കഥ തിരുത്തിയെഴുതുന്നു. ശേഷം ശവരതിയാരോപണം ദുര്‍വ്യാഖ്യാന അച്ചടിശാലകളില്‍ മഷിയുണക്കിയെടുക്കുന്നു. ‘വള്വ്ത്വജഅത്തു മഅഹാ ഫീ ഖബ്രിഹാ’ (അവളുടെ ഖബ്റില്‍ അവളോടൊപ്പം ഞാന്‍ കിടന്നത്…) എന്നതുകൊണ്ട് എന്താണ് ഉദ്ദേശിക്കപ്പെട്ടത്?. അതൊരു പ്രയോഗം മാത്രമാണ്. ഇംഗ്ലീഷില്‍ “Think in her shoes’ എന്നു പറയുന്നതു പോലെ. അതിനാരെങ്കിലും ‘അവളുടെ ഷൂസില്‍ കയറി ചിന്തിക്കുക’ എന്നര്‍ഥം മനസ്സിലാക്കുമോ!. ‘അവളുടെ സ്ഥാനത്തു നിന്നു ചിന്തിക്കുക’ എന്നേ അതിനുദ്ധേശമുള്ളൂ. അതുപോലെ ‘വള്വ്ത്വജഅത്തു മഅഹാ ഫീ ഖബ്രിഹാ’ (അവളുടെ ഖബ്‌റില്‍ അവളോടൊപ്പം ഞാന്‍ കിടന്നു) എന്നതുകൊണ്ടുദ്ദേശിക്കപ്പെടുന്നത് ‘ഫീ മൗള്വിഇഹാ’ അഥവാ ‘അവളുടെ സ്ഥാനത്ത് ഞാന്‍ കിടന്നു’ എന്നു മാത്രമാണ്. മാത്രമല്ല മറ്റു പല നിവേദനങ്ങളിലും വന്നിട്ടുള്ള പദം ‘വള്വ്ത്വജഅത്തു ഫീ ഖബ്രിഹാ’ (അവളുടെ ഖബ്‌റില്‍ ഞാന്‍ കിടന്നു) എന്നാണ്. (കന്‍സുല്‍ ഉമ്മാല്‍)

ഇനി, ‘വള്വ്ത്വജഅത്തു മഅഹാ ഫീ ഖബ്രിഹാ’ (അവളുടെ ഖബ്റില്‍ അവളോടൊപ്പം ഞാന്‍ കിടന്നത്…) എന്നു പറയപ്പെട്ട ത്വബ്റാനിയുടെ അല്‍ മുഅ്ജമുല്‍ ഔസത്തിലും സംഭവ വിവരണത്തിന് മേല്‍ പ്രസ്താവിക്കപ്പെട്ടതില്‍ നിന്നും യാതൊരു വ്യത്യാസവും കാണാനാകില്ല. ഫാത്തിമ ബിന്‍ത് അസദ് മരണപ്പെടുന്നു. നബി (സ്വ) അവര്‍ക്കുവേണ്ടി പ്രാർത്ഥിക്കുന്നു. അവര്‍ നബിക്കു ചെയ്ത സേവനങ്ങള്‍ അനുസ്മരിക്കുന്നു. ശേഷം അവരെ കുളിപ്പിക്കാന്‍ നിര്‍ദ്ദേശിക്കുന്നു. അനന്തരം അവരെ പൊതിഞ്ഞ വസ്ത്രത്തിനുമേല്‍ നബിയുടെ മേല്‍കുപ്പായമണിയിക്കുന്നു. അതിനു ശേഷം ഖബ്ര്‍ കഴിക്കുന്നു. പിന്നെ ലഹ്ദ് നബി കുഴിക്കുന്നു. അതില്‍ കിടക്കുന്നു. പ്രാർത്ഥിക്കുന്നു. മയ്യിത്ത് നമസ്‌കരിക്കുന്നു. അതിനും ശേഷം മയ്യിത്ത് ഖബ്‌റിലെ ലഹ്ദില്‍ വെക്കുന്നു. സാധാരണഗതിയില്‍ നിന്നും വ്യത്യസ്തമായി മയ്യിത്തിന് ഇത്ര പരിചരണവും പ്രാര്‍ഥനയും നല്‍കിയതിന്റെ ഉദ്ദേശമെന്താണെന്ന സ്വഹാബാക്കളുടെ ചോദ്യത്തിന് മറുപടിയായി നബി(സ്വ) പറഞ്ഞു: ”ഞാനെന്റെ മേല്‍വസ്ത്രം അവളെ ധരിപ്പിച്ചു; സ്വര്‍ഗത്തിലെ പുടവ ധരിക്കപ്പെടാനുള്ള സൗഭാഗ്യം അവള്‍ക്കു ലഭിക്കാനായി. അവളുടെ ഖബ്‌റില്‍ അവളോടൊപ്പം ഞാന്‍ കിടന്നു; അവള്‍ക്ക് ഖബ്‌റിന്റെ ഇടുക്കം ലളിതമാകാന്‍ വേണ്ടി. കാരണം, അബൂത്വാലിനു ശേഷം എന്നോട് ഏറ്റവും നന്നായി പെരുമാറുന്ന ആളായിരുന്നു അവര്‍”. ത്വബ്റാനിയുടെ അല്‍ മുഅ്ജമുല്‍ ഔസത്ത് 6935 മത്തെ നിവേദനത്തില്‍ നിന്നും ഇതിനപ്പിറം ഒന്നും തരപ്പെടില്ല. ഈ നിവേദനത്തില്‍ ‘അവളുടെ ഖബ്‌റില്‍ അവളോടൊപ്പം ഞാന്‍ കിടന്നത് അവള്‍ക്ക് ഖബ്‌റിന്റെ ഇടുക്കം ലളിതമാകാന്‍ വേണ്ടി’ എന്നു പറഞ്ഞത് – മയ്യിത്ത് ഖബ്റിലെ ലഹദില്‍ വെക്കുന്നതിനും മയ്യിത്ത് നമസ്‌കരിക്കുന്നതിനും മുമ്പ് – നബി ലഹ്ദ് കുഴിച്ച് അതില്‍ കിടന്നു പ്രാർത്ഥിച്ച സംഭവത്തെ പറ്റി തന്നെയാണെന്ന് ആര്‍ക്കും ഒറ്റ വായനയില്‍ തന്നെ ബോധ്യമാകുന്ന കാര്യമാണ്. അപ്പോള്‍ ‘അവളുടെ ഖബ്റില്‍ അവളോടൊപ്പം ഞാന്‍ കിടന്നു’ എന്നു പറഞ്ഞതിന്റെ ഉദ്ദേശം ‘അവളുടെ സ്ഥാനത്ത് ഞാന്‍ കിടന്നു’ എന്നു മാത്രമാണെന്നു വ്യക്തം.

അല്ലാതെ ആഭാസന്മാരുടെ അശ്ലീല ഭാവനകള്‍ക്കൊത്ത് പ്രവാചക ജീവിതത്തെ വളച്ചൊടിക്കാനൊക്കുന്ന ഒന്നും ആ വിശുദ്ധ ജീവിതത്തില്‍ നിന്നും കിട്ടാന്‍ പോകുന്നില്ല. പക്ഷെ, കൂട്ടത്തില്‍ ഒന്നുകൂടി ഓര്‍മപ്പെടുത്തട്ടെ സംഭവം ദുര്‍ബലമായ നിവേദനമാണ്; തെളിവിനു കൊള്ളില്ല.

വിശുദ്ധിയുടെ നിതാന്ത പരിമളമയമായ ആ ജീവിതം ചരിത്രത്തിന്റെ സ്വര്‍ണ ലിപികളാല്‍ ആലേഖനം ചെയ്യപ്പെട്ടതാണ്. അവിടെ എത്ര ഉരച്ചു നോക്കിയാലും ശവരതി പോയിട്ട് ഒരു പരസ്ത്രീ കരസ്പര്‍ശനത്തിന്റെ ചെറുതരികള്‍ പോലും കാണ്ടുകിട്ടില്ല. നൂറ്റാണ്ടുകളായി, എത്രയോ വമ്പന്‍ വിമര്‍ശകന്മാര്‍ ഉരക്കാന്‍ തുടങ്ങിയിട്ട്. ഉരക്കല്ലുകള്‍ തേഞ്ഞുതീര്‍ന്നിട്ടും അവര്‍ക്കൊന്നും മരുന്നിനുപോലും ഇന്നോളം ഒന്നും തരപ്പെട്ടിട്ടില്ല. പിന്നെയല്ലേ ഈ മനോരോഗികള്‍ ഉരച്ചിട്ട്!.

”ആഇശ (റ) പറയുന്നു: പ്രവാചകന്‍ ഒരിക്കലും ഒരു (അന്യ) സ്ത്രീയുടെയും കൈപ്പടം തൊട്ടിട്ടില്ല. അവരോട് ബൈഅത്ത് (പ്രതിജ്ഞ) എടുക്കാന്‍ പോകുന്ന സമയത്ത് പോലും അവരോട് പറയാറുണ്ട് നിങ്ങളോട് ഞാന്‍ എന്റെ സംസാരത്തിലൂടെ ബൈഅത്ത് ചെയ്തിരിക്കുന്നു.” (മുസ്‌ലിം: 1866)

”ഉമൈമഃ ബിന്‍ത് റക്വീക്വ (റ) പറഞ്ഞു: അവര്‍ നബി(സ്വ)ക്ക് കൈകൊടുത്ത സമയത്ത് നബി (സ്വ) അവരോട് പറഞ്ഞു: തീര്‍ച്ചയായും ഞാന്‍ (അന്യ) സ്ത്രീകള്‍ക്ക് കൈകൊടുക്കയില്ല.” (നസാഈ: 4181, ഇബ്‌നു മാജ: 2874)

”മഅ്ക്വലുബ്നു യസാര്‍ നിവേദനം: നബി (സ്വ) പറഞ്ഞു: തനിക്ക് അനുവദനീയമല്ലാത്ത ഒരു സ്ത്രീയെ സ്പര്‍ശിക്കുന്നതിനെക്കാള്‍ അവന് ഉത്തമമായത് ഇരുമ്പാണി തലയില്‍ തറക്കുന്നതാണ്.” (ത്വബ്‌റാനി)

ഇതാണ് മുഹമ്മദ് നബി (സ്വ) വിശുദ്ധ ജീവിതത്തിന്റെ നിതാന്ത വിസ്മയം. ലൈംഗിക വൈകൃതം ഹൃദയങ്ങളില്‍ കട്ടപിടിച്ചു കിടക്കുന്നവര്‍ക്ക് ആ ജീവിതവിശുദ്ധി ഉള്‍കൊള്ളാനാകില്ല; സമ്മതിച്ചു തരാനാകില്ല. ഈ രോഗാധുരമായ മനസ്സുകളില്‍ നിന്നു മാത്രമാണ് ഇത്തരം നീചവും നികൃഷ്ടവുമായ ദുരാരോപണങ്ങള്‍ ജന്മമെടുക്കുന്നത്. അവര്‍ പ്രവാചകനെ തെറി വിളിച്ചുകൊണ്ടേയിരിക്കും. അപകീര്‍ത്തിപ്പെടുത്തികൊണ്ടേയിരിക്കും. കാരണം പരിശുദ്ധരെയും പരിശുദ്ധിയിലേക്ക് മാനവികതയെ വിളിക്കുന്നവരെയും അവര്‍ക്ക് സഹിക്കാനാകില്ല; ഉള്‍കൊള്ളാനും. ഇഛാവൈകൃതങ്ങളുടെ ലോകം പടുത്തുയര്‍ത്താന്‍ അവര്‍ക്കു മുമ്പിലുള്ള ഏറ്റവും വലിയ തടസ്സം നബിയും അവിടുത്തെ ജീവിതവും സന്ദേശവുമാണ്. അതാണ് അവരില്‍ പലരുടെയും നബിവിരോധത്തിന്റെ മനഃശാസ്ത്രം.

അടുത്ത വിമര്‍ശനവും കൂടി നമുക്കൊന്ന് വിശകലനം ചെയ്യാം.

‘രക്തബന്ധമോ കുടുംബബന്ധമോ പോലും പരിഗണിക്കാതെ ഏത് സ്ത്രീയെയും ഭോഗിക്കുവാന്‍ നബിക്ക് അല്ലാഹു തന്നെ അനുവാദം നല്‍കുന്നതിനു മുമ്പായിരുന്നു (ക്വുര്‍ആന്‍ 33: 50) ഈ ശവരതി നടന്നതെന്നു കാണാം’. എന്താണ് ക്വുര്‍ആന്‍ 33: 50 ല്‍ പറയുന്നത്. ആദ്യം നമുക്ക് സൂക്തം ഒന്നു പരിശോദിക്കാം.

”നബിയേ, നീ വിവാഹമൂല്യം കൊടുത്തിട്ടുള്ളവരായ നിന്റെ ഭാര്യമാരെ നിനക്ക് നാം അനുവദിച്ചു തന്നിരിക്കുന്നു. അല്ലാഹു നിനക്ക് (യുദ്ധത്തില്‍) അധീനപ്പെടുത്തിത്തന്ന കൂട്ടത്തില്‍ നിന്റെ വലതുകൈ ഉടമപ്പെടുത്തിയ (അടിമ) സ്ത്രീകളെയും നിന്നോടൊപ്പം സ്വദേശം വിട്ടുപോന്നവരായ നിന്റെ പിതൃവ്യന്റെ പുത്രിമാര്‍, നിന്റെ പിതൃസഹോദരിമാരുടെ പുത്രിമാര്‍, നിന്റെ അമ്മാവന്റെ പുത്രിമാര്‍, നിന്റെ മാതൃസഹോദരിമാരുടെ പുത്രിമാര്‍ എന്നിവരെയും (വിവാഹം ചെയ്യാന്‍ അനുവദിച്ചിരിക്കുന്നു.) സത്യവിശ്വാസിനിയായ ഒരു സ്ത്രീ സ്വദേഹം നബിക്ക് ദാനം ചെയ്യുന്ന പക്ഷം നബി അവളെ വിവാഹം കഴിക്കാന്‍ ഉദ്ദേശിക്കുന്നെങ്കില്‍ അതും (അനുവദിച്ചിരിക്കുന്നു.) ഇത് സത്യവിശ്വാസികളെ കൂടാതെ നിനക്ക് മാത്രമുള്ളതാകുന്നു. അവരുടെ ഭാര്യമാരുടെയും അവരുടെ വലതുകൈകള്‍ ഉടമപ്പെടുത്തിയവരുടേയും കാര്യത്തില്‍ നാം നിയമമായി നിശ്ചയിച്ചിട്ടുള്ളത് നമുക്കറിയാം. നിനക്ക് യാതൊരു വിഷമവും ഉണ്ടാവാതിരിക്കാന്‍ വേണ്ടിയത്രെ ഇത്. അല്ലാഹു ഏറെ പൊറുക്കുന്നവനും കരുണാനിധിയുമാകുന്നു”. (കുര്‍ആന്‍: 33: 50)

വിമര്‍ശകന്മാര്‍ ജല്‍പിക്കുന്നതു പോലെ പ്രവാചകന് ലൈംഗിക ബന്ധത്തിലേര്‍പെടാന്‍ അനുവദിക്കപ്പെട്ട സ്ത്രീജന ലിസ്റ്റ് അല്ലിത്. മറിച്ച് പ്രവാചകന് വിവാഹ ബന്ധത്തിലേര്‍പെടാന്‍ അനുവദിക്കപ്പെട്ട സ്ത്രീകള്‍ ആരൊക്കെ? വിവാഹം ചെയ്യാവുന്ന സ്ത്രീകള്‍ ആരൊക്കെ? പ്രവാചകന് മാത്രമായി അനുവദിക്കപ്പെട്ട വിവാഹ രീതി ഏത്? ഇതൊക്കെയാണ് സൂക്തം കൈകാര്യം ചെയ്യുന്ന വിഷയം. അതിനെയെല്ലാം ഒന്നിച്ച് ‘ലൈംഗികാനുവാദ’മെന്ന പായയില്‍ ചുരുട്ടികെട്ടിയതാണ് വിമര്‍ശകര്‍.

ആദ്യം പറഞ്ഞ അല്ലെങ്കില്‍ ലൈംഗിക ബന്ധത്തിലേര്‍പെടാന്‍ അനുവദിക്കപ്പെട്ട സ്ത്രീകള്‍ ആരെല്ലാമാണ്. ഭാര്യമാരും സ്വന്തം അടിമസ്ത്രീകളും. അടിമസ്ത്രീകളുമായി ലൈംഗിക ബന്ധം അനുവദിക്കുക മാനവിക വിരുദ്ധമല്ലേ? അടിമത്ത സമ്പ്രദായം നിലനിന്നിരുന്ന സാമൂഹ്യ സാഹചര്യങ്ങളെ പറ്റി യാതൊരു ധാരണയുമില്ലാത്തതുകൊണ്ട് മാത്രമാണ് ഈ സംശയം ഉണ്ടാകുന്നത്. അത് നിലനിന്നിരുന്ന ഭൂമികയില്‍ നിന്നും ആ സമ്പ്രദായത്തെ അടര്‍ത്തിയെടുത്ത്, ഇന്നത്തെ ലോകക്രമത്തില്‍ കൊണ്ടുവെച്ച് അതിനെ വിലയിരുത്തുമ്പോള്‍ അത് മാനവിക വിരുദ്ധമായി തോന്നാം. പക്ഷെ അതിന്റെ ഭൂമികയില്‍ നിന്നുകൊണ്ട് ഇസ്‌ലാം അതിനെ സമീപിച്ച രീതിയെ പറ്റി വിലയിരുത്തുമ്പോഴാണ് പ്രസ്തുത മേഖലയില്‍ ഇസ്‌ലാം സ്വീകരിച്ച മാനവിക നിലപാട് നമുക്ക് മനസ്സിലാവുകയുള്ളൂ. ഒന്നാമതായി നാം മനസ്സിലാക്കേണ്ടത് അത് ഇസ്‌ലാം കൊണ്ടുവന്ന സമ്പ്രദായമല്ല. പ്രവാചകന് എത്രയോ നൂറ്റാണ്ടുകള്‍ക്കു മുമ്പുതന്നെ അത് സമൂഹത്തില്‍ നിലനിന്നിരുന്നു എന്നതാണ്. അതുകൊണ്ടാണ് ബൈബിളില്‍ പോലും അടിമത്ത സമ്പ്രദായവുമായി ബന്ധപ്പെട്ട ധാരാളം പരാമര്‍ശങ്ങള്‍ കാണാന്‍ കഴിയുന്നത്. (ലേവ്യ, 25: 44-46) ബൈബിളിലെ പല പ്രവാചകന്മാരും അടിമസ്ത്രീകളുള്ളവരോ, യുദ്ധങ്ങളില്‍ ബന്ധികളാക്കപ്പെട്ട സ്ത്രീകളെ അടിമകളായി സ്വീകരിച്ചവരോ അയിരുന്നെന്ന് കാണാനാകും. അബ്രഹാമും ദാവീദും മോശെയുമെല്ലാം ഉദാഹരണം. കേവലം ഒരു രാഷ്ട്രത്തിന്റെ ഭൂമിശാസ്ത്രപരമായ നാലതിരുകള്‍ക്കുള്ളില്‍ അപൂര്‍വമായി കാണപ്പെട്ടിരുന്ന ഒറ്റപ്പെട്ട ഒരു സാമൂഹിക പ്രതിഭാസമായിരുന്നില്ല അടിമത്ത സമ്പ്രദായം. ആ കാലഘട്ടത്തില്‍ ലോകത്തിന്റെ തന്നെ സാമൂഹ്യ സാമ്പത്തിക രാഷ്ട്രീയ മേഖലകളില്‍ ആഴത്തില്‍ വേരൂന്നിയ ഒരു സാമൂഹിക പ്രതിഭാസമായിരുന്നു അത്. ഇന്നത്തെ ഭാഷയില്‍ പറഞ്ഞാല്‍, യുദ്ധരംഗങ്ങളില്‍ ബന്ധികളാക്കപ്പെടുന്നവര്‍ മോചിപ്പിക്കപ്പെടുന്നില്ലെങ്കില്‍ അടിമകളാക്കപ്പെടുകയെന്നത് അന്നത്തെ അന്താരാഷ്ട്ര നിയമമായിരുന്നു എന്നുപോലും വേണമെങ്കില്‍ പറയാവുന്നതാണ്. അത് അന്നത്തെ അന്താരാഷ്ട്ര നൈതികതക്കെതിരായിരുന്നില്ല. ആ കാലഘട്ടത്തിലെ ഒരു രാഷ്ട്രത്തിനോ സാമ്രാജ്യത്വത്തിനോ പോലും, ഒരൊറ്റ സുപ്രഭാതത്തില്‍ തുടച്ചു നീക്കാന്‍ കഴിയുന്ന ഒന്നായിരുന്നില്ല അതെന്ന് പൗരാണിക ചരിത്രത്തെ പറ്റി ശരാശരി അറിവെങ്കിലുമുള്ള എല്ലാവര്‍ക്കുമറിയാവുന്ന കാര്യമാണ്. ആ യാഥാര്‍ത്ഥത്തെ ഉള്‍കൊള്ളാതിരിക്കാന്‍ ഇസ്‌ലാമികസാമ്രാജ്യത്വത്തിനെന്നല്ല ഒരു സാമ്രാജ്യത്വത്തിനും സാധ്യമല്ല. അങ്ങനെ വന്നാല്‍ അത് ആ സാമ്രാജ്യത്വത്തിന്റെ നാശത്തിലായിരിക്കുമവസാനിക്കുക. കാരണം അത്രമേല്‍ അത് അന്നത്തെ സാമൂഹ്യ സാമ്പത്തിക രാഷ്ട്രീയ മേഖലകളില്‍ ശക്തമായ സ്വാധീനം ചെലുത്തുന്ന ഒരു ഘടകമായിരുന്നു. പക്ഷെ മാനവികതയുടെ മതമായ ഇസ്‌ലാം ആ പരിധിക്കുള്ളില്‍ നിന്നു കൊണ്ട് തന്നെ കഴിയുന്നത്ര മാനവികമായ പരിഷ്‌കരണങ്ങള്‍ പ്രസ്തുത രംഗത്ത് നടപ്പാക്കുകയാണുണ്ടായത്. ആ പരിഷ്‌കരണങ്ങളാകട്ടെ അടിമകള്‍ക്ക് സ്വതന്ത്ര തുല്ല്യമായ ആശ്വാസമാണ് പ്രദാനം ചെയ്തത്. ഇസ്‌ലാമികസാമ്രാജ്യം അടിമകള്‍ക്ക് ചരിത്രത്തില്‍ മറ്റൊരിടത്തുമില്ലാതിരുന്ന അസ്തിത്വവും വ്യക്തിത്വവും നല്‍കിയാദരിച്ചു. അവര്‍ക്ക് ചോദിക്കാനും പറയാനും ആളുണ്ടായി. പിഴുതെടുക്കാന്‍ സമൂഹത്തിനു സാധ്യമല്ലാത്ത വിധം വേരും വിലാസവുമുണ്ടായവര്‍ക്ക്. ജീവിതത്തില്‍ തിരഞ്ഞെടുപ്പുകള്‍ക്ക് അവകാശമുണ്ടായവര്‍ക്ക്; അത് സ്വതന്ത്രനത്ര വന്നില്ലെങ്കില്‍ പോലും. കാരണം അവിടെ പരിധി നിശ്ചയിച്ചത് ഇസ്‌ലാമല്ല, സാമൂഹികാന്തരീക്ഷമായിരുന്നു. അവര്‍ക്ക് ഉടുക്കാനും ഉണ്ണാനും ഉറങ്ങാനും അവകാശമുണ്ടായി. ആകാശത്തിനു കീഴെ ഭൂമിക്കു മുകളില്‍ അവര്‍ക്കാദ്യമായി അന്തസ്സുണ്ടായി. ഈ ചരിത്ര പശ്ചാതലത്തില്‍ നിന്നു കൊണ്ട് വേണം അടിമസ്ത്രീയുമായുള്ള ലൈംഗിക ബന്ധത്തിന് ഉടമക്ക് അവകാശം നല്‍കുന്ന ഇസ്‌ലാമിക നിയമത്തെ നോക്കി കാണാന്‍.

പ്രവാചക കാലഘട്ടത്തിനു മുമ്പ് തന്നെ, അടിമ സ്ത്രീ എന്നത് ഉടമയുടെ സ്വകാര്യ വരുമാന സ്രോതസ്സുകൂടിയായിരുന്നു. അവളെ ഉടമ ലൈംഗികമായി ഭോഗിക്കുന്നതിനു പുറമെ ഗാര്‍ഹികവും സാമൂഹികവുമായ പല മേഖലകളിലും വരുമാന സ്രോതസ്സായി ഉപയോഗപ്പെടുത്തുകയും ചെയ്യും. അതില്‍ ഏറ്റവും അധികം അവളെ ഉപയോഗിച്ച് വരുമാനമുണ്ടാക്കിയിരുന്നത് വേശ്യാവൃത്തിയിലൂടെയാണ്. അതിലൂടെ പണമുണ്ടാക്കി ഉടമക്കെത്തിക്കണം. അവളുടെ ഉപജീവനമോ പരിചരണമോ അയാളുടെ ബാധ്യതയായിരുന്നില്ല. അതെല്ലാം അവള്‍ കണ്ടെത്തണം. ഉടമക്കു പുറമെ വേശ്യാവൃത്തിയിലൂടെയും അല്ലാതെ ഉടമയുടെ അഥിതി സല്‍ക്കാരത്തിന്റെ ഭാഗമായും അവള്‍ക്ക് ഒരുപാട് പുരുഷന്മാരോടൊപ്പം തന്റെ ശരീരം പങ്കുവെക്കേണ്ടി വരുമായിരുന്നു. ആര്‍ക്കും എപ്പോള്‍ വേണമെങ്കിലും ഭോഗിക്കാവുന്ന ഒരു വില്‍പന ചരക്കായിരുന്നു അക്ഷരാര്‍ഥത്തില്‍ ഇസ്‌ലാംമിനു മുമ്പുള്ള അടിമ സ്ത്രീ. അനവധി പുരുഷന്മാരുമായി ശരീരം പങ്കുവെക്കപ്പെടുന്നതു കൊണ്ടു തന്നെ അവള്‍ ഗര്‍ഭിണിയാവുകയോ പ്രസവിക്കുകയോ ചെയ്താല്‍ ആ ഉത്തരവാദിത്തം ഏറ്റെടുക്കാന്‍ ആരുമുണ്ടാവില്ല. ഇനി കുഞ്ഞ് ആരുടേതാണ് എന്നു തിരിഞ്ഞാല്‍ പോലും ഏറ്റെടുക്കാന്‍ ആളുണ്ടാവില്ല. പിതൃത്വം ഒരാളിലേക്കും ചേര്‍ത്തിപറയാന്‍ അവള്‍ക്കവകാശമില്ല. കുഞ്ഞിനെ അവള്‍ തനിച്ചു വളര്‍ത്തണം. ആ കുഞ്ഞ് വലുതായാല്‍ അന്നത്തെ സാമൂഹിക രീതിയനുസരിച്ച് അതും അടിമയായി മാറും. പെണ്‍ കുഞ്ഞാണെങ്കില്‍ മാതാവിന്റെ ജീവിത വഴിയിടങ്ങളിലൂടെ തന്നെ അവളും സഞ്ചരിക്കേണ്ടി വരും; പലപ്പോഴും. വേരില്ല വിലാസമില്ല സ്വത്തില്ല സന്താനങ്ങള്‍ക്ക് തന്തയില്ല. ഇനി അവള്‍ ഭര്‍തൃമതിയാണെങ്കിലോ കാര്യമായ മാറ്റമൊന്നുമില്ല കുഞ്ഞിന്റെ പേരിനൊരു വാലുണ്ടായി എന്നു മാത്രം. ആരുടേതാണെങ്കിലും പിതൃകോളം ഭര്‍ത്താവിന്റെ പേരുകൊണ്ട് പൂരിപ്പിക്കുമെന്നര്‍ഥം.

അടിമ സ്ത്രീയുടെ ഈ ഖേദഘതിക്ക് ഇസ്‌ലാം അറുതി വരുത്തി. ഇസ്‌ലാം പറഞ്ഞു ആര്‍ക്കെങ്കിലും അടിമ സ്ത്രീയുണ്ടായാല്‍ ഒന്നുകില്‍ അവളെ മോചിപ്പിച്ച് നല്ല നിലയില്‍ വിവാഹം ചെയ്തുകൊടുക്കുക. അല്ലെങ്കില്‍ അവള്‍ക്കുമേലുള്ള അധികാരം (വിലായത്ത്) നിലനിര്‍ത്തികൊണ്ട്, പ്രായശ്ചിത്തമായോ പുണ്യകര്‍മമായോ അവളെ മോചിപ്പിക്കക. അതുമല്ലെങ്കില്‍ അവളുടെ ഉപജീവന-പരിചരണ ഉത്തരവാദിത്തമേറ്റെടുത്ത് ഉടമ മാത്രം അവളെ പ്രാപിക്കുക. അതു വഴി അവള്‍ക്ക് കുഞ്ഞുണ്ടായാല്‍ അതിന്റെ ഉത്തരവാദിത്തവും ഉടമക്കായിരിക്കും. അവന്റെ സ്വത്തില്‍ ആ കുഞ്ഞിന് അവകാശമുണ്ടായിരിക്കും. കുഞ്ഞിന്റെ സ്വത്തില്‍ സ്വാഭാവികമായും അമ്മക്കും ഒരവകാശമുണ്ടായിരിക്കും. വേശ്യാവൃത്തിയിലൂടെ അവളെ ഉപയോഗിച്ച് വരുമാനമുണ്ടാക്കുന്ന ഏര്‍പ്പാട് അവസാനിപ്പിക്കുകയും ഉടമയിലൂടെ മാത്രമായി അവള്‍ക്ക് ലൈംഗികവും ഗാര്‍ഹികവുമായ ഒരു ജീവിതം തുറന്നു കൊടുക്കുകയും ചെയ്തു ഇസ്‌ലാം. അവള്‍ക്കും അവളുടെ പരമ്പരകള്‍ക്കും വേരും വിലാസവും സ്വത്തും കുടുംബവും അവകാശവും സംരക്ഷണവും; അല്ല ജീവിതവും കൊടുത്തു ഇസ്‌ലാം. എത്ര മാനവികമായ ഇടപെടല്‍. ഒരേ സമയം ഇസ്‌ലാം, അവളെ ഉപഭോഗ വസ്തുവല്ലാതാക്കുകയും എന്നാല്‍ ഉടമയിലൂടെ അവളുടെ ലൈംഗിക മോഹങ്ങള്‍ക്ക് ഇടംകൊടുക്കുകയും ചെയ്തു. അതാണ് ഇസ്‌ലാമിലെ വലതുകൈ ഉടമപ്പെടുത്തിയ സ്ത്രീകള്‍ അഥവാ അടിമ സ്ത്രീകള്‍. ”നബിയേ, നീ വിവാഹമൂല്യം കൊടുത്തിട്ടുള്ളവരായ നിന്റെ ഭാര്യമാരെ നിനക്ക് നാം അനുവദിച്ചു തന്നിരിക്കുന്നു. അല്ലാഹു നിനക്ക് (യുദ്ധത്തില്‍) അധീനപ്പെടുത്തിത്തന്ന കൂട്ടത്തില്‍ നിന്റെ വലതുകൈ ഉടമപ്പെടുത്തിയ (അടിമ) സ്ത്രീകളെയും…” എന്നു പറഞ്ഞ സൂക്തത്തില്‍ മാനവിക വിരുദ്ധയില്ലെന്ന് ഇപ്പോള്‍ മനസ്സിലായല്ലോ?.

ഇനി രണ്ടാമതായി സൂക്തം എണ്ണി പറയുന്നത് ലൈംഗിക ബന്ധത്തിലേര്‍പ്പെടാന്‍ അനുവധിക്കപ്പെട്ട സ്ത്രീകളെ പറ്റിയല്ല. മറിച്ച് നബിക്ക് വിവാഹം ചെയ്യാന്‍ അനുവാദമുള്ള കുടുംബ ബന്ധുക്കളില്‍ ചിലരെ പറ്റിയാണെന്ന് ഇബ്‌നു അബ്ബാസി(റ)ല്‍ നിന്നും ഇമാം ത്വബ്‌രി ഉദ്ദരിക്കുന്നുണ്ട് (തഫ്‌സീറു ത്വബ്‌രി: 33:50 ന്റെ വ്യാഖ്യാനം). അതാരൊക്കെയാണ്. ”…നിന്നോടൊപ്പം സ്വദേശം വിട്ടുപോന്നവരായ നിന്റെ പിതൃവ്യന്റെ പുത്രിമാര്‍, നിന്റെ പിതൃസഹോദരിമാരുടെ പുത്രിമാര്‍, നിന്റെ അമ്മാവന്റെ പുത്രിമാര്‍, നിന്റെ മാതൃസഹോദരിമാരുടെ പുത്രിമാര്‍ എന്നിവരെയും (വിവാഹം ചെയ്യാന്‍ അനുവദിച്ചിരിക്കുന്നു.)…”. കുടുംബ ബന്ധുക്കളില്‍ നിന്നും നബി(സ്വ)ക്ക് വിവാഹം ചെയ്യാന്‍ പാടില്ലാത്തവരും പാടുള്ളവരും ആരൊക്കെയാണെന്ന് പഠിപ്പിക്കപ്പെടേണ്ടത് വളരെ പ്രധാനപ്പെട്ട കാര്യമാണ്. ഇത്തരം കുടുംബ ബന്ധത്തിലുള്ള സ്ത്രീകളെ വിവാഹം ചെയ്യാന്‍ എല്ലാ മുസ്‌ലിം പുരുഷന്മാര്‍ക്കും അനുവാദമുണ്ടെങ്കിലും പ്രവാചകന് പക്ഷെ ഹിജ്‌റക്കുശേഷം, അദ്ധേഹത്തോടൊപ്പം ഹിജ്‌റ ചെയ്ത കുടുംബ ബന്ധുക്കളെ മാത്രമേ -ബന്ധുക്കളില്‍ നിന്നും- വിവാഹം ചെയ്യാന്‍ പാടുള്ളൂ എന്ന് ഈ സൂക്തത്തിലുടെ പ്രത്യേകം നിയമമാക്കിയിരിക്കുന്നു എന്നു കുറിക്കുന്ന പല നിവേദനങ്ങളും തഫ്‌സീറുകളില്‍ കാണാം. (തഫ്‌സീറു ത്വബ്‌രി: 33:50 ന്റെ വ്യാഖ്യാനം നോക്കുക).

അപ്പോള്‍ ഹിജ്‌റക്കു ശേഷം കുടുംബ ബന്ധുക്കളില്‍ നിന്നും നബിക്ക് വിവാഹം ചെയ്യാവുന്നവരായി സൂക്തം എണ്ണി പറഞ്ഞത് ആരെയൊക്കെയാണ്. ‘നിന്നോടൊപ്പം സ്വദേശം വിട്ടുപോന്നവരായ’ അമ്മാവന്റെയും അമ്മായിയുടെയും പിതൃവ്യന്റെയും മാതൃസഹോദരിമാരുടെയും പുത്രിമാര്‍. ‘പുത്രിമാര്‍’ എന്നത് തെറ്റാതെ പ്രത്യേകം പഠിച്ചുവെക്കണം. കാരണം ചില മിഷനറി അച്ചടിശാലകളില്‍ പലപ്പോഴും ‘പുത്രിമാര്‍’ എന്നത്, അച്ച് നഷ്ടപ്പെട്ടിട്ടാണോ അതല്ല അച്ചടിപ്പിശാച് പിടികൂടിയിട്ടാണോ എന്നറിയില്ല; വിട്ട് പോകാറുണ്ട് പലപ്പേഴും. അങ്ങനെ വിട്ടുപോയവ മാത്രം പ്രസിദ്ധീകരിക്കാന്‍, ഏറ്റവും ഉളുപ്പ് നഷ്ടപ്പെട്ടവരെ തേടിയുള്ള മിഷനറി അച്ചടിശാലകളുടെ യാത്രകള്‍ നവനാസ്തികര്‍ക്കും എക്‌സ് മുസ്‌ലിംകള്‍ക്കും മുമ്പിലാണ് മിക്കവാറും ചെന്നവസാനിക്കാറുള്ളത്. ‘പുത്രിമാര്‍’ വിട്ടുപോയാല്‍ പിന്നെ ഭാക്കിയാവുന്നതാരൊക്കെയാണ്. അമ്മായിയും മാതൃസഹോദരിയും അമ്മാവനും പിതൃവ്യനും മാത്രമാകും. ‘ലേഡീസ് ഫസ്റ്റ്’ എന്നതാണല്ലോ പരിഷ്‌കാരികളുടെ ഒരു നിലപാട്. അപ്പോള്‍ അമ്മാവനും പിതൃവ്യനും മാറ്റി നിര്‍ത്തപ്പെടും. പിന്നെ പറയാനുണ്ടോ ആവശ്യത്തിനുള്ള വകയായല്ലോ. അമ്മായിയെയും മാതൃസഹോദരിയെയും കാമിക്കാനും കല്ല്യാണം കഴിക്കാനും മുഹമ്മദിനെ ക്വുര്‍ആന്‍ അനുവദിക്കുന്നു. കഷ്ടം തന്നെ മുസ്‌ലിംകളുടെ ധാര്‍മിക ബോധം. പ്രചരണ വാഹനത്തിന്റെ സ്റ്റിയറിംങ് പിന്നെ നവനാസ്തിക മനോരോഗികളുടെ കൈയ്യിലായിരിക്കും.

മൂന്നാമതായി സൂക്തം പരാമര്‍ശിക്കുന്ന വിഷയമെന്താണ്. ”…സത്യവിശ്വാസിനിയായ ഒരു സ്ത്രീ സ്വദേഹം നബിക്ക് ദാനം ചെയ്യുന്ന പക്ഷം നബി അവളെ വിവാഹം കഴിക്കാന്‍ ഉദ്ദേശിക്കുന്നെങ്കില്‍ അതും (അനുവദിച്ചിരിക്കുന്നു.) ഇത് സത്യവിശ്വാസികളെ കൂടാതെ നിനക്ക് മാത്രമുള്ളതാകുന്നു…”. എന്താണ് സംഭവം. ഒരു സ്ത്രീ സ്വദേഹം നബിക്ക് ദാനം ചെയ്യുന്ന പക്ഷം അവളെ നബിക്കു വിവാഹം ചെയ്യാം. അതു പ്രവാചകനു മാത്രം അനുവദിക്കപ്പെട്ട നിയമമാണ്. മുസ്‌ലിംകളില്‍ മറ്റാര്‍ക്കും ഈ അനുമതി ബാധകമല്ല. ‘സ്വദേഹം ദാനം ചെയ്യുക’ എന്നു പറഞ്ഞാല്‍ എന്താണ്. ശരീര ദാനമാണോ അവിടെ ഉദ്ധേശിക്കുന്നത്. അല്ലേ അല്ല. മറിച്ച് അതുകൊണ്ടര്‍ത്ഥമാക്കുന്നതെന്താണെന്ന് പണ്ഡിതന്മാര്‍ വിശദീകരിച്ചിട്ടുണ്ട്.

”ഞാന്‍ എന്നെ താങ്കള്‍ക്കു മുമ്പില്‍ വിവാഹത്തിനായി ഇഷ്ടദാനം ചെയ്യുന്നു’ എന്ന വാചകത്തില്‍ ഒരു മുള്വാഫ് (Possession) (ഭാഷാ പരമായ ഭംഗിക്കായി) വിട്ടുകളഞ്ഞതാണ്. യഥാർത്ഥത്തില്‍ വാചകത്തിന്റെ വിവക്ഷ ഇപ്രകാരമാണ്: ‘ഞാന്‍ എന്നെ അഥവാ എന്റെ വിവാഹ കാര്യത്തെ താങ്കള്‍ക്കു മുമ്പില്‍ ഇഷ്ടദാനം ചെയ്യുന്നു’. കാരണം ഒരു സ്വതന്ത്ര്യ സ്ത്രീ ഉടമപ്പെടുത്തപ്പെടുകയോ ദാനം ചെയ്യപ്പെടുകയോ ഇല്ലല്ലോ. മഹര്‍ ഇല്ലാതെ തന്നെ താങ്കളെ വിവാഹം ചെയ്യാന്‍ ഞാന്‍ തയ്യാറാണ് എന്നാണ് ആ സ്ത്രീ പറഞ്ഞതിന്റെ വിവക്ഷ.” (ഫത്ഹുല്‍ ബാരി: 9/112, ഫത്ഹുല്‍ മുന്‍ഇം: 5/540). എന്നാല്‍ അനുവാദമുണ്ടായിരുന്നിട്ടും പ്രവാചകന്‍ അത്തരത്തിലുള്ള വിവാഹം ഒരിക്കലും ചെയ്തിട്ടില്ലെന്ന് ഇബ്‌നു അബ്ബാസി(റ)ല്‍ നിന്ന് ഇമാം കുര്‍തുബിയും (തഫ്‌സീറുല്‍ കുര്‍തുബി, 33:50 ന്റെ വ്യാഖ്യാനം) ഇബ്‌നു ഹജറും (ഫത്ഹുല്‍ ബാരി, 8:526) ഉദ്ദരിച്ചിട്ടുണ്ട്.

അപ്പോള്‍ ഇത്രയുള്ളൂ കാര്യം. ‘സ്വദേഹം ദാനം ചെയ്യുക’ എന്നു പറഞ്ഞാല്‍ മഹര്‍ ഇല്ലാതെ തന്നെ താങ്കളെ വിവാഹം ചെയ്യാന്‍ ഞാന്‍ തയ്യാറാണ് എന്നുമാത്രമേ അതിനര്‍ത്ഥമുള്ളൂ. അത് സ്ത്രീയുടെ തീരുമാനാധികാരപരിധിയില്‍ വരുന്ന സംഗതിയാണ്. ധാര്‍മികതാലംഘനത്തിന്റെ ഒരു പ്രശ്‌നവും അവിടെ ഉരുത്തിരിയുന്നില്ല. അറബി ഭാഷയോ ഭാഷാ പ്രയോഗങ്ങളോ തിരിയാത്തവര്‍ ക്വുര്‍ആനും ഹദീഥും നിരൂപണം ചെയ്യാനൊരുമ്പെട്ടാല്‍ ഇതല്ല ഇതിനപ്പുറവും പറയും. വിവരക്കേട് ഇസ്‌ലാംവിമര്‍ശകര്‍ക്ക് ഒരലങ്കാരമാണെന്നു തോന്നുന്നു. കഷ്ടം

വിമർശനം:

(സോഷ്യൽ മീഡിയകളിൽ വ്യാപകമായി പ്രചരിക്കുന്ന വിമർശനം വിമർശകരുടെ തെറി വിളികൾ മാത്രം ഒഴിവാക്കി ഇവിടെ എടുത്തു ചേർക്കുന്നു:)

“മുഹമ്മദ് ഒരു സ്വവർഗഭോഗി കൂടി ആയിരുന്നു. അനസ് എന്ന മുഹമ്മദിന്റെ കൂട്ടാളിയുമായി മുഹമ്മദ് കാമകേളിയിൽ ഏർപ്പെട്ടു എന്ന് അനസ് തന്നെ വിവരിക്കുന്നു. Bukhari 4:52:143

മുഹമ്മദ് ആൺകുട്ടികളുടെ നാവും ചുണ്ടും നക്കിയിരുന്നതായി വിവരിക്കുന്നു. Musnad Ahmad 16245. (Pedophilic prophet)”

മറുപടി:

ഒരു തെറി പ്രവാചകന്റെ(സ) മേൽ സ്വന്തമായി കെട്ടിയുണ്ടാക്കുകയും അതിന് തെളിവുണ്ടെന്ന് വരുത്തി തീർക്കാൻ ചില ഗ്രന്ഥങ്ങളും ‘നമ്പറു’കളും കുറിക്കുകയും ചെയ്യുക എന്നത് ഇസ്‌ലാം വിമർശകരുടെ സ്ഥിരം ഒരു ‘നമ്പറാ’ണ്. പറഞ്ഞ തെറിക്ക് ഉപോൽബലകമായ യാതൊന്നും പ്രസ്‌തുത നമ്പറുകളിലുള്ള ഹദീസുകളിൽ ഉണ്ടാകില്ലെന്നതാണ് യാഥാർത്ഥ്യം. ഇവിടെയും അതു തന്നെയാണ് നടന്നിരിക്കുന്നത്. ലൈംഗികതയുമായി യാതൊരു ബന്ധവുമില്ലാത്ത രണ്ട് ഹദീസുകളെ സ്വവർഗാനുരാഗവും ബാലാനുരാഗവുമായി വളച്ചൊടിക്കാനുള്ള കുൽസിത ശ്രമം. അതിലെ കള്ളത്തരം തിരിച്ചറിയണമെങ്കിൽ വിമർശകർ ‘നമ്പറിറക്കിയ’ രണ്ട് ഹദീസുകളും ആദ്യം ഒന്ന് നേരിട്ട് വായിക്കാം.

1. ﻋﻦ ﺃﻧﺲ ﺑﻦ ﻣﺎﻟﻚ ﺭﺿﻲ اﻟﻠﻪ ﻋﻨﻪ: ﺃﻥ اﻟﻨﺒﻲ ﺻﻠﻰ اﻟﻠﻪ ﻋﻠﻴﻪ ﻭﺳﻠﻢ، ﻗﺎﻝ ﻷﺑﻲ ﻃﻠﺤﺔ: «اﻟﺘﻤﺲ ﻏﻼﻣﺎ ﻣﻦ ﻏﻠﻤﺎﻧﻜﻢ ﻳﺨﺪﻣﻨﻲ ﺣﺘﻰ ﺃﺧﺮﺝ ﺇﻟﻰ ﺧﻴﺒﺮ» ﻓﺨﺮﺝ ﺑﻲ ﺃﺑﻮ ﻃﻠﺤﺔ ﻣﺮﺩﻓﻲ، ﻭﺃﻧﺎ ﻏﻼﻡ ﺭاﻫﻘﺖ اﻟﺤﻠﻢ، ﻓﻜﻨﺖ ﺃﺧﺪﻡ ﺭﺳﻮﻝ اﻟﻠﻪ ﺻﻠﻰ اﻟﻠﻪ ﻋﻠﻴﻪ ﻭﺳﻠﻢ، ﺇﺫا ﻧﺰﻝ، ﻓﻜﻨﺖ ﺃﺳﻤﻌﻪ ﻛﺜﻴﺮا ﻳﻘﻮﻝ: اﻟﻠﻬﻢ ﺇﻧﻲ ﺃﻋﻮﺫ ﺑﻚ ﻣﻦ اﻟﻬﻢ ﻭاﻟﺤﺰﻥ، ﻭاﻟﻌﺠﺰ ﻭاﻟﻜﺴﻞ، ﻭاﻟﺒﺨﻞ ﻭاﻟﺠﺒﻦ، ﻭﺿﻠﻊ اﻟﺪﻳﻦ، ﻭﻏﻠﺒﺔ اﻟﺮﺟﺎﻝ

അനസ് ബിൻ മാലികിൽ (റ) നിന്ന്: പ്രവാചകൻ (സ) (അനസിന്റെ പിതാവായ) അബൂ ത്വൽഹയോട് പറഞ്ഞു: ഞാൻ ഖൈബർ (യുദ്ധത്തിന്) പുറപ്പെടുമ്പോൾ നിങ്ങളുടെ മക്കളിൽ ഒരാളെ എനിക്ക് സഹായങ്ങൾ ചെയ്തു തരാനായി കൂടെ കൂട്ടുക. അപ്പോൾ അബൂ ത്വൽഹ എന്നെ അദ്ദേഹത്തിന്റെ പിന്നിലിരുത്തി (കുതിരപ്പുറത്ത്) പുറപ്പെട്ടു. ഞാനന്ന് പ്രായപൂർത്തി എത്താറായ പയ്യനായിരുന്നു. (പടയോട്ടത്തിനിടയിൽ) ഒരു സ്ഥലത്ത് വിശ്രമിക്കാൻ ഇറങ്ങിയാൽ ഞാൻ അല്ലാഹുവിന്റെ ദൂതന്(സ) സഹായങ്ങൾ ചെയ്തു കൊടുക്കുമായിരുന്നു. അദ്ദേഹം (സ) ധാരാളമായി ഇപ്രകാരം പ്രാർത്ഥിക്കുന്നത് ഞാൻ കേൾക്കുമായിരുന്നു: “അല്ലാഹുവേ! ആകുലത, ദുഃഖം, ദുര്‍ബലത, മടി, പിശുക്ക്, ഭീരുത്വം, കടഭാരം, ആളുകള്‍ എന്നെ കീഴ്‌പ്പെടുത്തല്‍ എന്നിവയില്‍ നിന്നെല്ലാം ഞാന്‍ നിന്നോട് രക്ഷതേടുന്നു.” (സ്വഹീഹുൽ ബുഖാരി: 2893)

പ്രവാചകന്റെ (സ) മാതൃസഹോദരിയാണ് അനസിന്റെ (റ) മാതാവ് ഉമ്മു സുലൈം. അഥവാ അനസിന്റെ കൊച്ചാപ്പയാണ് പ്രവാചകൻ (സ) എന്ന് സാന്ദർഭികമായി സൂചിപ്പിക്കട്ടെ. സാധാരണക്കാർ ഈ ഹദീസ് വായിക്കുമ്പോൾ അതിൽ ലൈംഗികതയുടെ ലാഞ്ചന പോലും കണ്ടെന്നു വരില്ല. (അതുകൊണ്ടാണ് വിമർശകർ ഹദീസിന്റെ നമ്പർ മാത്രം കൊടുക്കുന്നത്) പക്ഷെ മനസ്സിൽ രോഗമുള്ളവർ ഇത് വായിച്ചപ്പോൾ സ്വവർഗരതിയും ബാലാനുരാഗവുമൊക്കെ കണ്ടെത്തി. ഉപ്പയോടൊപ്പം യാത്ര ചെയ്ത് യാത്രക്കിടയിൽ കൊച്ചാപ്പക്ക് സഹായങ്ങൾ ചെയ്ത് കൊടുക്കുന്നതിൽ ലൈംഗികത ചികഞ്ഞെടുക്കാൻ പോൺ സിനിമാ സംവിധായകന്മാരുടെ വൈദഗ്‌ദ്ധ്യം തന്നെ വേണം. ഈ പരിചയ സമ്പന്നതയുടെ വെളിച്ചത്തിൽ വിമർശകർ രണ്ട് ലൈംഗിക സൂചനകൾ മോർഫ് ചെയ്തെടുത്തു.

ഹദീസിൽ “(പടയോട്ടത്തിനിടയിൽ) ഒരു സ്ഥലത്ത് വിശ്രമിക്കാൻ ഇറങ്ങിയാൽ ഞാൻ അല്ലാഹുവിന്റെ ദൂതന് (സ) സഹായങ്ങൾ ചെയ്തു കൊടുക്കുമായിരുന്നു.” എന്ന വരിയാണ് ഒന്ന്. വിശ്രമിക്കാനിറങ്ങുമ്പോൾ ഒരാൾക്ക് സഹായങ്ങൾ ചെയ്യുക എന്ന് പറഞ്ഞാൽ അത് “കാമകേളിയിൽ ഏർപ്പെട്ടു” എന്നതിനുള്ള തെളിവാണ് പോലും!!

സ്ത്രീ, ബാലൻ, ശവം, കിടക്ക, ഉറക്കം, വിശ്രമം തുടങ്ങിയ പദങ്ങൾ ഒരു വാചകത്തിൽ വരുമ്പോഴേക്കും ലൈംഗിക ഉത്ഥാപനം ഉണ്ടാകുന്ന ഞരമ്പുരോഗികൾക്കല്ലാതെ ഈ ഹദീസിൽ എന്ത് ലൈംഗീകതയാണ് നിരീക്ഷിക്കാൻ കഴിയുക ?! ലൈംഗികതയുടെ ലാഞ്ചനയുള്ള ഒരു വരിയെങ്കിലും ഹദീസിൽ കാണിച്ചു തരാൻ ഈ ലൈംഗിക രോഗികൾക്ക് കഴിയുമോ ?!

പ്രവാചകന് ഭക്ഷണമെത്തിക്കുക, ആളുകളെ തന്റെ അടുത്തേക്ക് വിരുന്നിനായോ മറ്റൊ ക്ഷണിക്കുക (സ്വഹീഹു മുസ്‌ലിം:3580) ആവശ്യ സാധനങ്ങൾ വാങ്ങിപ്പിക്കുക (സ്വഹീഹു മുസ്‌ലിം: 2310) സന്ദേശങ്ങൾ അറിയിക്കാനുള്ളവരെ അറിയിക്കാനായി നിയോഗിക്കുക (സ്വഹീഹു ബുഖാരി: 5170) എന്നതൊക്കെയാണ് താൻ ചെയ്തിരുന്ന സഹായങ്ങൾ എന്നും പ്രവാചകൻ അശ്ലീലം പറയുന്നവനൊ പ്രവർത്തിക്കുന്നവനോ ആയിരുന്നില്ല എന്നും (സ്വഹീഹു ബുഖാരി: 6031) അനസ് (റ) മറ്റു ഹദീസുകളിൽ വിവരിക്കുന്നുണ്ടെങ്കിലും അതൊന്നും ഈ വെറുപ്പിന്റെ അപ്പോസ്തലന്മാർക്ക് ബാധകമല്ല.

ഹദീസിനെ ലൈംഗീകവൽക്കരിക്കാൻ വിമർശകർ പയറ്റുന്ന മറ്റൊരു ദുർവ്യാഖ്യാനം ഇപ്രകാരമാണ്:

ഹദീസിൽ പ്രവാചകൻ (സ) ധാരാളമായി പ്രാർത്ഥിക്കുമായിരുന്നു എന്ന് അനസ് വിവരിക്കുന്ന പ്രാർത്ഥനയിൽ “ആളുകള്‍ എന്നെ കീഴ്‌പ്പെടുത്തല്‍ എന്നിവയില്‍ നിന്നെല്ലാം ഞാന്‍ നിന്നോട് രക്ഷതേടുന്നു” എന്ന വാചകമുണ്ട്. ഇതിന് overcome by men എന്നാണ് ഇംഗ്ലീഷ് ഭാഷയിൽ നൽകിയിരിക്കുന്ന പരിഭാഷ. ഇത് അനസിനെ മുഹമ്മദ് നബി ലൈംഗീകമായി പീഡിപ്പിച്ചതിനെ തെളിയിക്കുന്നു എന്നാണ് (ദുർ)വ്യാഖ്യാനം.!!! അങ്ങനെയാണെങ്കിൽ ഈ പ്രാർത്ഥന പ്രാർത്ഥിക്കേണ്ടിയിരുന്നത് അനസ് (റ) അല്ലെ ? പ്രവാചകൻ അല്ലല്ലൊ എന്ന് ചിന്തിക്കാനുള്ള സാമാന്യ ബുദ്ധി വിമർശകർക്ക് – വർഗീയതയുടെ അതിപ്രസരത്താൽ – നഷ്ടപ്പെട്ടിരിക്കുന്നു.

പ്രാർത്ഥനയിലെ (ﻏﻠﺒﺔ اﻟﺮﺟﺎﻝ) ‘ആളുകള്‍ എന്നെ കീഴ്‌പ്പെടുത്തല്‍’ എന്നതിൽ ലൈംഗികതയൊന്നും ഇല്ല.

ശൈഖ് ശംസുൽ ഹക് അളീമാബാദി പറഞ്ഞു: ‘ശക്തിയും സ്വാധീനവുമുള്ളവർ ദുർബലരെ അടിച്ചമർത്തുന്നതാണ് ഗലബത്തുർ രിജാൽ. രിജാൽ എന്നത് കൊണ്ട് ഇവിടെ ഉദ്ദേശം അക്രമികളും കടം തന്നത് ചൂഷണം ചെയ്ത് സാമ്പത്തികമായി ഉപദ്രവിക്കുന്നവരുമാണ്. അതിൽ നിന്നും പ്രവാചകൻ ശരണം തേടാൻ കാരണം അത് മാനസികമായി ഒരാളെ തളർത്തിക്കളയും എന്നതിനാലാണ്.’ (ഔനുൽ മഅ്ബൂദ്: 4/281)

ആകുലത, ദുഃഖം, ദുര്‍ബലത, മടി, പിശുക്ക്, ഭീരുത്വം, കടഭാരം എന്നിവയിൽ നിന്ന് അല്ലാഹുവോട് രക്ഷ തേടുന്നതിനൊപ്പം ‘മറ്റുള്ളവർ തലയിൽ കയറി നിരങ്ങുന്നതിൽ നിന്നും’ പ്രവാചകൻ (സ) രക്ഷ തേടി. ഇതിലെവിടെയാണ് “കാമകേളിയിൽ ഏർപ്പെടൽ” ??? കടം കയറി പള്ളിയിൽ ദുഖിതനായി ഇരിക്കുന്ന അബൂഉമാമ എന്ന അനുചരനും ഇതേ പ്രാർത്ഥന പ്രവാചകൻ (സ) പഠിപ്പിച്ചു കൊടുത്തതായി ഹദീസ് ഉദ്ധരിക്കപ്പെട്ടിരിക്കുന്നു. (അബൂദാവൂദ്: 1555)

2. رأيْتُ رسولَ اللهِ صلَّى اللهُ عليه وسلَّمَ يَمُصُّ لِسانَه -أو قال: شَفَتَه، يعني: الحسَنَ بنَ عليٍّ صلواتُ اللهِ عليه-، وإنَّه لن يُعذَّبَ لِسانٌ أو شَفَتانِ مَصَّهُما رسولُ اللهِ صلَّى اللهُ عليه وسلَّمَ.

പ്രവാചകന് തന്റെ പേരകുഞ്ഞായിരുന്ന ഹസനോട് ഉണ്ടായിരുന്ന അതിയായ സ്നേഹത്തെ സംബന്ധിച്ച് സ്മരിക്കവെ മുആവിയ (റ) പറയുന്നു: അല്ലാഹുവിന്റെ ദൂതൻ ഹസൻ ഇബ്നു അലിയുടെ ചുണ്ട് അതൊ നാവ് (ശരിക്കും ഓർമ്മയില്ല) ചപ്പുന്നത് ഞാൻ കണ്ടിട്ടുണ്ട്…” (മുസ്നദു അഹ്മദ്: 16848)

“മുഹമ്മദ് ആൺകുട്ടികളുടെ നാവും ചുണ്ടും നക്കിയിരുന്നു” എന്ന് പറഞ്ഞ് ഹദീസിന്റെ നമ്പർ കൊടുത്തവർ ആ ‘ആൺകുട്ടികൾ’ എന്ന ബഹുവചനമല്ല ഹദീസിൽ ഉള്ളത് എന്നും തന്റെ പേരകുഞ്ഞാണെന്നും വ്യക്തമാകാതിരിക്കാനാണ് ‘നമ്പർ’ ഇറക്കിയത്. യഥാർത്ഥത്തിൽ, ഒരു സ്നേഹ നിധിയായ മുത്തച്ഛൻ മുലകുടി പ്രായമുള്ള തന്റെ പേരകുഞ്ഞിനെ ലാളിക്കുന്നതിനിടയിൽ ഗാഢമായി ചുംബിച്ചതാണ് ഹദീസിലെ രംഗം. അതും എല്ലാവരും കാൺകെ. ഇതിനെയാണ് വിമർശകർ സ്വവർഗ്ഗഭോഗമായും കുട്ടികളോടുള്ള ലൈംഗിക ആസക്തിയായും (pedophilia) ചിത്രീകരിക്കുന്നത് ! ഒരു പിതാവ് തന്റെ പിഞ്ചുകുഞ്ഞിനെ സ്നേഹത്താൽ ചുംബിച്ചെന്നും ചപ്പിയെന്നും വരാം. ആ പുത്രവാത്സല്യവും കുട്ടികളോടുള്ള ലൈംഗിക ആസക്തിയും തമ്മിലുള്ള പരസ്പര വ്യത്യാസം തിരിച്ചറിയാനുള്ള കഴിവ് ബന്ധങ്ങളുടെ പവിത്രത തിരിച്ചറിയുന്നവർക്ക് പറഞ്ഞിട്ടുള്ളതാണ്; അമ്മമാരെയും പെങ്ങമ്മാരെയും ഉൾപ്പെടെ ആരെയും ഭോഗിക്കാം എന്നും അഗമ്യഗമനം (incest) ലൈംഗിക സ്വാതന്ത്ര്യത്തിന്റെ ഭാഗമാണെന്നും സ്റ്റേജ് ഒരുക്കി പ്രസംഗിക്കുകയും യൂറ്റൂബിൽ പരസ്യമായി പ്രചരിപ്പിക്കുകയും ചെയ്യുന്നവർക്ക് ലഭിക്കുന്നതല്ല. (https://youtu.be/nkiBbtydlwA)

ഇനി, പ്രവാചകൻ (സ) മുലകുടി പ്രായമുള്ള പേരകുഞ്ഞിന്റെ നാവ് ചപ്പിയ സാഹചര്യം ചില നിവേദനങ്ങളിൽ വന്നതു കൂടി ചേർത്തു വായിക്കുമ്പോഴാണ് സംഭവത്തിന്റെ യഥാർത്ഥ ചിത്രം നമുക്ക് ലഭിക്കുക. സംഭവത്തിന്റെ ചുരുക്കമിതാണ്:

ഒരിക്കൽ ഒരു യാത്രാ വേളയിൽ മുല കുടി പ്രായക്കാരായ, പ്രവാചകന്റെ (സ) പേര കിടാങ്ങളായ ഹസൻ, ഹുസൈൻ എന്നിവർ (റ) ശക്തമായി കരയാൻ തുടങ്ങി. അവരോടൊപ്പം അവരുടെ മാതാവുമുണ്ട്. “എന്റെ കുഞ്ഞുമക്കൾ എന്തേ കരയുന്നത് ?” എന്ന് പ്രവാചകൻ (സ) ചോദിച്ചു. ഫാതിമ പറഞ്ഞു: ദാഹം കൊണ്ട്. പ്രവാചകൻ (സ) വെള്ളമന്വേഷിച്ചു. അന്ന് വരൾച്ചയുടെ സമയമാണ്. അദ്ദേഹം ജനങ്ങളോട് വിളിച്ച് ചോദിച്ചു: “നിങ്ങളുടെ ആരുടെയെങ്കിലും അടുക്കൽ വെള്ളമുണ്ടോ ?” ആരുടെ കയ്യിലുമുണ്ടായില്ല. കുഞ്ഞുങ്ങൾ ശക്തമായി കരഞ്ഞു കൊണ്ടിരുന്നു. അപ്പോൾ പ്രവാചകൻ (സ) പറഞ്ഞു: “അവരിൽ ഒരാളെ ഇങ്ങു തരൂ” കുഞ്ഞുങ്ങളിലൊരാളെ പ്രവാചകൻ (സ) കയ്യിലെടുത്തു, മടിയിൽ കിടത്തി, നാവ് നീട്ടി. കുഞ്ഞ് (ഹസൻ (റ)) അദ്ദേഹത്തിന്റെ നാവ് ചപ്പാൻ തുടങ്ങി. കരച്ചിൽ നിർത്തി ശാന്തമായി ഉറങ്ങി…. (അൽ മുഅ്ജമുൽ കബീർ: ത്വബ്റാനി: 2656)

3. ഒരു മനുഷ്യന് പ്രവർത്തിക്കാൻ കഴിയുന്നതിൽ വെച്ച് ഏറ്റവും വലിയ പാതകമായി സ്വവർഗരതിയെ പഠിപ്പിച്ച വ്യക്തിയാണ് പ്രവാചകൻ (സ). സ്വവർഗരതിയിൽ കൂട്ടായി നിർലജ്ജം അഭിരമിച്ച ലൂത്ത് നബിയുടെ ജനതയെ അല്ലാഹു ശിക്ഷിച്ച ചരിത്രം പ്രവാചകൻ (സ) ഈ ലോകത്തെ പഠിപ്പിച്ചു:

“അങ്ങനെ സൂര്യോദയത്തോടെ ആ ഘോരശബ്ദം അവരെ പിടികൂടി. അങ്ങനെ ആ രാജ്യത്തെ നാം തലകീഴായി മറിക്കുകയും, ചുട്ടുപഴുത്ത ഇഷ്ടികക്കല്ലുകള്‍ അവരുടെ മേല്‍ നാം വര്‍ഷിക്കുകയും ചെയ്തു.” (കുർആൻ 15: 73,74)

പ്രവാചകൻ (സ) പറഞ്ഞു: ആരെങ്കിലും ലൂത്ത് നബിയുടെ ജനതയുടെ പ്രവർത്തനം (സ്വവർഗരതി) പ്രവർത്തിച്ചാൽ ആ പ്രവർത്തനത്തിൽ ഏർപ്പെട്ടവന്റെയും അതിന് വിധേയനായി നിന്നു കൊടുത്തവന്റെയും മേൽ വധശിക്ഷ നടപ്പാക്കണം. (തുർമുദി: 1456, അബൂദാവൂദ്: 4462, ഇബ്നുമാജ: 2561)

പ്രവാചകൻ (സ) പറഞ്ഞു: ലൂത്ത് നബിയുടെ ജനതയുടെ പ്രവർത്തനം (സ്വവർഗരതി) പ്രവർത്തിച്ചവനെ അല്ലാഹു ശപിച്ചിരിക്കുന്നു. പ്രവാചകൻ (സ) മൂന്ന് തവണ ഇത് ആവർത്തിച്ചു പറഞ്ഞു. (മുസ്നദു അഹ്മദ്: 2915)

വിമർശനം:

“അജ്ഞാത കാലഘട്ടത്തിലെ പോലെ ആരെങ്കിലും തന്റെ ഗോത്ര വംശത്തെക്കുറിച്ച് വീമ്പിളക്കി അതിലേക്ക് ക്ഷണിക്കുന്നത് നിങ്ങൾ കേൾക്കുന്നുവെങ്കിൽ, പിതാവിന്റെ പുരുഷ അംഗത്തെ കടിക്കാൻ പറയുക, എന്ന് മുഹമ്മദ് നബി പറഞ്ഞു.

മറുപടി:

1. ഹദീസിന്റെ നിവേദക പരമ്പരകളെല്ലാം ദുർബലമാണ്. സ്വീകാര്യയോഗ്യവും സ്ഥിരപ്പെട്ടവയുമായ ഹദീസുകൾക്ക് എതിരുമാണ്.

കാരണങ്ങൾ:

ഹദീസ് സ്വഹീഹ് (സ്വീകാര്യതയുടെ നിബന്ധനകൾ പൂർത്തീകരിക്കപ്പെട്ടത്) അല്ല; ദുർബലമാണ്. ഹദീസിന്റെ ദുർബലതയെ സംബന്ധിച്ച ചർച്ചകളിലേക്ക് കടക്കും മുമ്പ് ഹദീസിന്റെ വിവർത്തനത്തിൽ സംഭവിച്ചു പോവുന്ന ആശയ ചോർച്ചയെ സാന്ദർഭികമായി ഒന്ന് വിലയിരുത്തേണ്ടതുണ്ട്.

مَنْ تَعَزَّى بِعَزَاءِ الْجَاهِلِيَّةِ، فَأَعِضُّوهُ، وَلَا تَكْنُوا

പിതാവിന്റെ പുരുഷ അംഗത്തെ കടിക്കാനൊന്നും ഹദീസിലില്ല. അറബി ഭാഷാ പ്രയോഗങ്ങളെ സംബന്ധിച്ച സൂക്ഷമമായ അറിവിന്റെ അഭാവമാണ് ഇത്തരമൊരു വിവർത്തനാർത്ഥത്തിലേക്ക് നയിക്കുന്നത്. ഹദീസിലെ ‘അൽ അദ്ദു’ (الْعَضُّ) എന്ന പദത്തിന് പല്ല് കൊണ്ട് കടിച്ചു പിടിക്കുക എന്ന ഒരു അർത്ഥമുണ്ടെങ്കിലും വാചകത്തിനനുസരിച്ച് അതിന്റെ അർത്ഥം വ്യത്യാസപെടുന്നുവെന്നത് അറബി ഭാഷാ പണ്ഡിതർ വ്യക്തമാക്കിയിട്ടുണ്ട്. ഒരു വസ്തുവെയോ, ആശയത്തെയോ, വ്യക്തിയെയോ വിട്ടുവീഴ്ച്ചയില്ലാതെ മുറുകെ പിടിക്കുക, വേർപിരിയാതെ കൂടെ കൂടുക, നിർബാധം ചേർന്നിരിക്കുക എന്നൊക്കെയാണ് ‘അദീദ്’ (العَضِيض) എന്ന പദത്തിന്റെ അടിസ്ഥാനപരമായ അർത്ഥം എന്ന് അൽ മുഗ്‌രിബ് ഫീ തർതീബിൽ മുഅ്രിബ്: മുത്വരിസി, അൽ മിസ്ബാഹുൽ മുനീർ: ഫയ്യൂമി, മുഅ്ജമുല്ലുഗ അൽ അറബിയ്യ: അഹ്മദ് മുഖ്താർ, അസാസുൽ ബലാഗ: സമഖ്ശരി, താജുല്ലുഗ വസിഹാഹുൽ അറബിയ്യ: ജൗഹരി, ലിസാനുൽ അറബ്: ഇബ്നു മൻദൂർ തുടങ്ങിയ അറബി ഭാഷാ നിഘണ്ടുക്കൾ പ്രസ്ഥാവിക്കുന്നുണ്ട്.

മറ്റു ഹദീസുകളിൽ ‘അൽ അദ്ദു’ (الْعَضُّ), ‘അദീദ്’ (العَضِيض) എന്നീ പദങ്ങൾ പ്രയോഗിച്ചേടത്തും ‘പല്ല് കൊണ്ട് കടിക്കുക’ എന്ന് അക്ഷരാർത്ഥത്തിൽ ഉദ്ദേശിക്കപ്പെടുന്നില്ല എന്ന് വ്യക്തമാവുന്നു. ഉദാഹരണങ്ങൾ കാണുക:

فَعَلَيْكُمْ بِسُنَّتِي وَسُنَّةِ الْخُلَفَاءِ الرَّاشِدِينَ الْمَهْدِيينَ، عَضُّوا عَلَيْهَا بِالنَّوَاجِذِ.

“നിങ്ങൾ എന്റെ ചര്യയും സച്ചരിതരായ ഖലീഫമാരുടെ ചര്യയും സ്വീകരിക്കുക. ‘അവയെ അണപ്പല്ല് കൊണ്ട് കടിച്ചു പിടിക്കുകയും ചെയ്യുക” എന്ന് പ്രവാചകൻ (സ) പറഞ്ഞതായി ഒരു ഹദീസിൽ കാണാം. (അബൂദാവൂദ്: 4607, തുർമുദി: 266)

ഹദീസിലെ عَضُّوا عَلَيْهَا بِالنَّوَاجِذِ എന്ന വാചകത്തിന്റെ പദാനുപദ വിവർത്തനമാണ് ‘അവയെ അണപ്പല്ല് കൊണ്ട് കടിച്ചു പിടിക്കുകയും ചെയ്യുക’ എന്ന് നൽകിയിരിക്കുന്നത്. എന്നാൽ പ്രവാചകന്റെയും(സ) ഖലീഫമാരുടെയും ജീവിതചര്യ അണപ്പല്ല് കൊണ്ട് കടിച്ചു പിടിക്കുക എന്നത് അക്ഷരാർത്ഥിൽ സംഭവ്യമല്ലല്ലൊ. ഇവിടെ ഉദ്ദേശ്യം അവരുടെ ജീവിതചര്യയെ ആദർശമായി മുറുകെ പിടിക്കുക, വേർപിരിയാതെ കൂടെ കൂട്ടുക എന്നൊക്കെയാണ്.

മറ്റൊരു ഹദീസിൽ പ്രവാചകൻ (സ) പറഞ്ഞതായി കാണാം:

ولَو أنْ تَعَضَّ بأصلِ شجَرَةٍ حتى يُدْرِكَكَ الموتُ وأنتَ كذلِكَ

“… മുസ്‌ലിംകളുടെ നേതാവിനേയും സംഘ ശക്തിയെയും നീ മുറുകെ പിടിക്കുക… മുസ്‌ലിംകൾക്ക് ഒരു നേതാവും സംഘ ശക്തിയും ഇല്ലെങ്കിൽ നീ എല്ലാ (ഒറ്റപ്പെട്ട അരക്ഷിത) കക്ഷികളേയും വേർപ്പിരിയുക. ‘ഒരു മരത്തിന്റെ മുരട് കടിച്ച് പിടിച്ച് കൊണ്ടെങ്കിലും’ (തനിച്ച് കഴിയുക). നിനക്ക് മരണം സംഭവിക്കുന്നത് വരെ അപ്രകാരം നിലകൊള്ളുക.” (സ്വഹീഹുൽ ബുഖാരി: 3606) ഹദീസിലെ ولَو أنْ تَعَضَّ بأصلِ شجَرَةٍ എന്ന വാചകത്തിന്റെ പദാനുപദ വിവർത്തനമാണ് ‘ഒരു മരത്തിന്റെ മുരട് കടിച്ച് പിടിച്ച് കൊണ്ടെങ്കിലും’ എന്ന് നൽകിയിരിക്കുന്നത്. ഇവിടെ മരത്തിന്റെ മുരട് കടിക്കുക എന്ന പദാനുപദ വിവർത്തനമല്ലല്ലൊ ഉദ്ദേശ്യം. ഉദ്ദേശ്യം മരത്തെ മുറുകെ പിടിക്കുക, വേർപിരിയാതെ കൂടെ കൂടുക എന്നൊക്കെയാണ്. അഥവാ ഒറ്റപ്പെട്ട അരക്ഷിത കക്ഷികൾ വിഹരിക്കുന്ന നാഗരികമായ പ്രദേശങ്ങളെ വിട്ടെറിഞ്ഞ് വൃക്ഷവന്യമായ ഗ്രാമങ്ങളിൽ ഗൃഹജമായ ജീവിതം നയിക്കുക എന്നാണ് ഹദീസിന്റെ താൽപര്യം.

ഇനി, ഈ നിഘണ്ടുക്കളുടെയൊന്നും സഹായമില്ലാതെ, അൽപം ‘ചന്ത പരിജ്ഞാന’മുണ്ടെങ്കിൽ തന്നെ ഹദീസിൽ ‘അൽ അദ്ദു’ (الْعَضُّ) എന്ന പദം കൊണ്ട്, ‘പല്ല് കൊണ്ട് കടിച്ചു പിടിക്കുക’ എന്ന അർത്ഥമല്ല ഉദ്ദേശിക്കപ്പെട്ടിരിക്കുന്നത് എന്ന് മനസ്സിലാക്കാം. കാരണം, ‘ലിംഗം കടിക്കുക’ എന്ന ഒരു തെറിയുണ്ടോ ? ലിംഗം ഈമ്പുക എന്നതാണ് തെറിയെന്ന് അൽപമെങ്കിലും തെറികൾ കേട്ടിട്ടുള്ളവർക്ക് അറിയാം. അപ്പോൾ ഹദീസിൽ വംശവെറിയനെ, ‘ലിംഗം ഈമ്പുക’ എന്ന ചന്ത തെറി വിളിക്കാനല്ല ആഹ്വാനം ചെയ്യുന്നത് എന്ന് വ്യക്തം.

പിന്നെ എന്താണ് ഹദീസിന്റെ വിവക്ഷ?

വംശ വെറിയെ (racism) ഇന്ന് ലോകത്ത് നിലനിൽക്കുന്ന ഏറ്റവും വലിയ പാതകങ്ങളിലും അപകടങ്ങളിലുമൊന്നായി ഇസ്‌ലാം കാണുന്നു. വർണ വർഗ വെറിയാണ് ലോകത്തെ രക്ത ചുഴിയിൽ ആഴ്ത്തിയ ലോക മഹാ യുദ്ധങ്ങളിലേക്ക് മനുഷ്യരെ നയിച്ച ചാലക ശക്തി. അടിമകളാക്കപ്പെട്ട കോടിക്കണക്കിന് മനുഷ്യരുടെ ചോരക്കൊണ്ട് എഴുതപ്പെട്ട സവർണ അധിനിവേശ ചരിത്ര കഥകൾ എല്ലാ ദേശക്കാർക്കും നാട്ടുകാർക്കും പറയാനുണ്ട്. എന്താണ് യഥാർത്ഥത്തിൽ വംശ വിരോധത്തിന്റെ അടിത്തറ? പിതാക്കന്മാരിലൂടെ ഒരു വംശത്തിനോ വർഗത്തിനോ മറ്റുള്ളവർക്ക് മേൽ പരിശുദ്ധിയും ശ്രേഷ്ഠതതയും സിദ്ധിക്കുന്നു എന്ന സങ്കൽപമാണല്ലൊ ഈ വിവേചന ആശയത്തിന് നിദാനം. ഒരു പ്രത്യേക വർണത്തിലൊ, ഗോത്രത്തിലോ, വംശത്തിലൊ ഉള്ള പിതാക്കന്മാരുടെ പുത്രന്മാരായി ജനിക്കാൻ കഴിഞ്ഞാൽ താൻ ശ്രേഷ്‌ഠനായി എന്നർത്ഥം. ഈ ശ്രേഷ്ഠത പരിശ്രമങ്ങളിലൂടെയൊ സേവനങ്ങളിലൂടെയൊ നേടിയെടുക്കാൻ സാധിക്കില്ല. പച്ചയായി പറഞ്ഞാൽ ഈ ശ്രേഷ്ഠതക്ക് നിദാനം ഏത് ലിംഗത്തിൽ നിന്നുള്ള ബീജത്തിൽ നിന്ന് പിറക്കുന്നു എന്നതാണ്. അഥവാ വംശവെറിയുടെ ആധാരം ലിംഗ ശ്രേഷ്ഠതയാണ്. ഇത്തരമൊരു തരം താഴ്ന്ന ആശയത്തിലേക്ക് ക്ഷണിക്കുന്നവനോട് “നീ നിന്റെ പിതാവിന്റെ ലിംഗത്തെ അവലംബിച്ചും ആദർശമായി ആചരിച്ചും ജീവിച്ചു കൊള്ളുക” എന്ന് പറയാനാണ് ഹദീസിൽ ഉള്ളടങ്ങിയ കൽപ്പന. ഇതിലൂടെ വംശ വെറിയെ (racism) പച്ചക്ക് തൊലിയുരിച്ചു കാണിക്കുകയാണ് ചെയ്യുന്നത്. വംശീയ വിരോധി യഥാർത്ഥത്തിൽ തങ്ങളുടെ പിതാക്കന്മാരുടെ ഏറ്റവും താഴ്ന്ന ഒരു അവയവമായ ലിംഗത്തെ ആദരിച്ചു പൂജിക്കുന്ന തനി വിഡ്ഢിയാണ്, വംശീയമായ യാഥാസ്ഥിതികത്വത്തിന്റെ ആരാധനാ മൂർത്തി താന്താങ്ങളുടെ പിതാക്കന്മാരുടെ ലിംഗങ്ങളാണ്. ആ ലിംഗങ്ങളുടെ മഹത്വവും പരിപാവനത്വവും ശ്രേഷ്ടതയുമൊക്കെയാണ് ഒരോ റേസിസ്റ്റും നാസിസ്റ്റും ഉൽഘോഷിക്കുന്നതും പ്രകീർത്തിക്കുന്നതും. അതിനപ്പുറം ഒരു ആശയ പൊലിമയും അവനില്ല എന്ന് പച്ചക്ക് പറഞ്ഞ് നാണം കെടുത്തലാണ് ഒരു റേസിസ്റ്റിന് നൽകാനുള്ള ശിക്ഷ.

ഈ അർത്ഥത്തിലാണ് ഹദീസ് നിലകൊള്ളുന്നത്. അല്ലാതെ വിമർശകർ ചിത്രീകരിക്കും പോലെ, അത് തർക്കമുണ്ടായാൽ പയറ്റാനായി ഉപയോഗിക്കേണ്ട ഒരു ചന്ത തെറിയല്ല. ഹദീസിനെ മുൻപത്തെ ഖണ്ഡികയിൽ വിശദീകരിച്ച ആശയത്തിൽ -വിമർശകർ ചിത്രീകരിക്കുന്ന രൂപത്തിലല്ല- സ്വീകരിച്ച ചില മുസലിം പണ്ഡിതന്മാരുണ്ട്. അവരെ മുൻ നിർത്തി ഈ ഹദീസ് സ്വീകാര്യയോഗ്യമായി അവതരിപ്പിക്കുകയാണ് വിമർശകർ സാധാരണ ചെയ്യാറുള്ളത്. പക്ഷെ ഹദീസിന്റെ നിവേദകപരമ്പരകളെല്ലാം ദുർബലമായതു കൊണ്ട് തന്നെ ഹദീസ് മുസ്‌ലിംകളുടെ അടുക്കൽ സ്വീകാര്യയോഗ്യമല്ല.

2. ഹദീസിന്റെ നിവേദകപരമ്പരകളെല്ലാം ദുർബലമാണ്.

നിവേദക പരമ്പര: ഒന്ന്:

ഔഫിബ്നു അബീ ജമീല അൽഅഅ്റാബി – ഹസനിൽ നിന്ന് – ഉത്ബി ഇബ്നു ദംറയിൽ നിന്ന് – അദ്ദേഹം പറഞ്ഞു:…

പരമ്പരയിലെ ഹസൻ, ഹസനുൽ ബസ്വരിയാണ്. അദ്ദേഹം പ്രസിദ്ധ ഹദീസ് പണ്ഡിതനാണെങ്കിലും ‘മുദല്ലിസാ’ണ്. (ത്വബകാത്തുൽ മുദല്ലിസീൻ: ഇബ്നു ഹജർ: 29, അത്തബ് യീൻ ഫീ അസ്മാഇൽ മുദല്ലിസീൻ: അബുൽ വഫാ,അത്തബ് യീൻ ഫീ അസ്മാഇൽ മുദല്ലിസീൻ: ഇബ്നുൽ അജ്മി: 20, അസ്സികാത്ത്: ഇബ്നു ഹിബ്ബാൻ)

ഹദീസ് പണ്ഡിതന്മാർ, ‘മുദല്ലിസെ’ന്ന സാങ്കേതിക പദമുപയോഗിച്ച് വിളിക്കുന്ന നിവേദകർ, തങ്ങൾ ആരിൽ നിന്നാണോ ഹദീസ് ഉദ്ധരിക്കുന്നത് അവരിൽ നിന്ന് ഇടയാളന്മാരില്ലാതെ നേരിട്ട് കേട്ടുവെന്ന് സൂചിപ്പിക്കുന്ന ‘എന്നോട് പറഞ്ഞു’ എന്ന വാചകം ഉപയോഗിക്കേണ്ടതുണ്ട്. ചർച്ച ചെയ്യപ്പെടുന്ന ഹദീസിൽ അപ്രകാരം ഒരു വാചകം ഹസനുൽ ബസ്വരി ഉപയോഗിച്ചിട്ടില്ല. “ഹസനിൽ നിന്ന് – ഉത്ബി ഇബ്നു ദംറയിൽ നിന്ന് ” എന്നാണ് വന്നിരിക്കുന്നത്. അതിനാൽ ഹദീസ്, ദുർബല ഹദീസുകളുടെ ഇനങ്ങളിലൊന്നായ ‘മുദല്ലസി’ൽ ഉൾപെടുന്നു.

കൂടാതെ, പരമ്പരയിലെ ഉത്ബി ഇബ്നു ദംറ അറിയപ്പെടാത്ത വ്യക്തിയാണെന്ന് ഇബ്നുൽ മദീനി പറയുന്നു. (തഹ്ദീബുതഹ്ദീബ്: 7:96)

നിവേദക പരമ്പര: രണ്ട്:

അബ്ദുല്ല പറഞ്ഞു – നമ്മോട് മുഹമ്മദ് ഇബ്നു അംറ് ഇബ്നുൽ അബ്ബാസ് അൽ ബാഹുലി അറിയിച്ചു – നമ്മോട് സുഫ്‌യാൻ പറഞ്ഞു – അദ്ദേഹം ആസ്വിമിൽ നിന്ന് – അദ്ദേഹം അബൂ ഉസ്മാനിൽ നിന്ന് – അദ്ദേഹം ഉബയ്യിൽ നിന്ന്….

പരമ്പരയിലെ മുഹമ്മദ് ഇബ്നു അംറ് ഇബ്നുൽ അബ്ബാസ് അൽ ബാഹുലിയെ വിശ്വസ്‌തനായി ആരും പ്രസ്താവിച്ചിട്ടില്ല. റാഫിദി- സിന്ദീക് എന്നിങ്ങനെ, ഹദീസ് പണ്ഡിതർ വ്യാജന്മാരെന്നും പിഴച്ച കക്ഷിക്കാരെന്നും വ്യക്തമാക്കിയ ചില തൽപര കക്ഷികൾ മാത്രമാണ് ഈ നിവേദകനെ വിശ്വസ്‌തനെന്ന് വിശേഷിപ്പിച്ച്. ഇത് ഈ നിവേദകനും തൽപരകക്ഷിക്കാരായ കള്ള കഥ നിർമാതാക്കളും തമ്മിലുള്ള അന്തർധാരയെ വെളിപ്പെടുത്തുന്നു. അതുകൊണ്ട് തന്നെ നിവേദനം ദുർബലമാണ്.

സമാനമായ ഹദീസ് ത്വബ്റാനി മറ്റൊരു പരമ്പരയിലൂടെ ഉദ്ധരിക്കുന്നുണ്ടെങ്കിലും അത് അങ്ങേയറ്റം ദുർബലമാണ്.

قال الطبراني : حدثنا أحمد بن مسعود ثنا عمرو بن أبي سلمة عن سعيد بن بشير عن قتادة عن الحسن عن مكحول عن عجرد بن مدراع التميمي

പരമ്പരയിലെ അംറിബ്നു അബീ സലമ നിവേദനത്തിൽ ധാരാളം തെറ്റുകൾ സംഭവിക്കുന്ന വ്യക്തിയാണ്. (തക്‌രീബു തഹ്ദീബ്: 1/736) പരമ്പരയിലെ മറ്റൊരു നിവേദകനായ സഈദിബ്നുൽ ബശീർ ദുർബലനാണ്. (അത്തഹ്ദീബ്) വേറെയും അനവധി ന്യൂനതകൾ പരമ്പരക്കുണ്ട്. ഇനി ഈ ദുർബലമായ പരമ്പരകളെല്ലാം ചേർത്തു വെച്ചാൽ അവ പരസ്പരം ശക്തിപകരുന്നു എന്ന് വാദിക്കാനും വകുപ്പില്ല. കാരണം വളരെയധികം വിശ്വസ്‌തരായ നിവേദകർ ഉദ്ധരിച്ച, സ്ഥിരപ്പെട്ട ഒട്ടനവധി ഹദീസുകൾക്ക് വിരുദ്ധമാണ് ഈ ദുർബല നിവേദനങ്ങൾ.

3. ഹദീസ് വളരെയധികം വിശ്വസ്‌തരായ നിവേദകർ ഉദ്ധരിച്ച, സ്ഥിരപ്പെട്ട ഒട്ടനവധി ഹദീസുകൾക്ക് വിരുദ്ധമാണ്.

പ്രസ്തുത ഹദീസുകളിൽ ചിലത്:

* അല്ലാഹുവിന്റെ തിരുദൂതൻ (സ) പറഞ്ഞു: “ഒരു വ്യക്തി തന്റെ പിതാവിനെ ശപിക്കുക എന്നത് വൻപാപങ്ങളിൽ ഒന്നാണ്.” അദ്ദേഹത്തോട് ചോദിക്കപ്പെട്ടു: അല്ലാഹുവിന്റെ തിരുദൂതരേ, എങ്ങനെയാണ് ഒരാൾ തന്റെ പിതാവിനെ ശപിക്കുക? അപ്പോൾ അദ്ദേഹം പറഞ്ഞു: “ഒരാൾ മറ്റൊരാളുടെ പിതാവിനെ ശകാരിക്കുന്നു. അപ്പോൾ മറ്റയാൾ തിരിച്ചും അയാളുടെ പിതാവിനേയും മാതാവിനേയും ശകാരിക്കുന്നു.” (സ്വഹീഹുൽ ബുഖാരി: 5636, സ്വഹീഹു മുസ്‌ലിം: 155)

* ആഇശ (റ) പറഞ്ഞു: ഒരു സ്ത്രീ പ്രവാചകനോട് ആർത്തവകാരിയായതിന് ശേഷമുള്ള കുളിയെ സംബന്ധിച്ച് ചോദിച്ചു. അവരോട് കുളിക്കാൻ അദ്ദേഹം കൽപിച്ചു, ഇപ്രകാരം പറയുകയും ചെയ്തു: കസ്‌തൂരി മുക്കിയ ഒരു പഞ്ഞിയെടുകയും അത് കൊണ്ട് ശുദ്ധിയാക്കുകയും ചെയ്യുക. സ്ത്രീ ചോദിച്ചു: എങ്ങനെയാണ് ഞാൻ ശുദ്ധിയാക്കേണ്ടത്? പ്രവാചകൻ (സ) പറഞ്ഞു: അത് കൊണ്ട് ശുദ്ധിയാക്കുകയും ചെയ്യുക. സ്ത്രീ ചോദിച്ചു: എങ്ങനെ? പ്രവാചകൻ (സ) പറഞ്ഞു: അല്ലാഹു പരിശുദ്ധൻ, ശുദ്ധിയാക്കുക. എന്നിട്ട് എന്നെ (ആഇശ) തന്റെ അടുത്തേക്ക് ക്ഷണിച്ച് (ആ സ്ത്രീക്ക് വ്യക്തമായി പറഞ്ഞു കൊടുക്കാൻ ആവശ്യപ്പെട്ടു). അപ്പോൾ ഞാൻ അവർക്ക് പറഞ്ഞു കൊടുത്തു: ആർത്തവ രക്തത്തിന്റെ പാട് പിന്തുടർന്ന് കഴുകി കളയുക… (സ്വഹീഹുൽ ബുഖാരി: 308, സ്വഹീഹു മുസ്‌ലിം: 332)

ചോദ്യകർത്താവിനോട് മുഖത്ത് നോക്കി ആർത്തവ രക്തത്തെ സംബന്ധിച്ചും സ്ത്രീകളുടെ അവയവങ്ങളെ സംബന്ധിച്ചും സംസാരിക്കാൻ ലജ്ജിച്ചത് കൊണ്ടാണ് “എങ്ങനെയാണ് ഞാൻ ശുദ്ധിയാക്കേണ്ടത്?” എന്ന് ചോദ്യകർത്താവ് ആവർത്തിച്ച് ചോദിച്ചിട്ടും പ്രവാചകൻ (സ) ഒഴിഞ്ഞു മാറിയതും അത് വിശദീകരിച്ചു കൊടുക്കാൻ ഭാര്യയോട് അപേക്ഷിച്ചതും. അതേ പ്രവാചകൻ (സ) ശിഷ്യന്മാരോട് പിതാക്കളുടെ അവയവം എടുത്തു പറഞ്ഞ് ശകാരിക്കാൻ ഉപദേശിക്കുകയെന്നത് മനശാസ്ത്രപരമായും സ്വാഭാവികമായും അസംഭവ്യമാണ്.

വിമർശനം:

മുഹമ്മദ് നബി, ശിഷ്യൻ സൈദിന്റെ ഭാര്യ സൈനബിനെ മോഹിക്കുകയും അദ്ദേഹത്തെ കൊണ്ട് ഭാര്യയെ നിർബന്ധപൂർവ്വം വിവാഹമോചനം ചെയ്യിപ്പിക്കുകയും ചെയ്തു. “അല്ലാഹു വെളിപ്പെടുത്താന്‍ പോകുന്ന ഒരു കാര്യം നിന്‍റെ മനസ്സില്‍ നീ മറച്ചു വെക്കുകയും ജനങ്ങളെ നീ പേടിക്കുകയും ചെയ്തിരുന്നു…” (കുർആൻ: 33:37) എന്ന് കുർആൻ സൂചിപ്പിച്ചത് മുഹമ്മദ് നബി സൈദിന്റെ ഭാര്യയെ വിവാഹം കഴിച്ചാല്‍ കൊള്ളാമെന്ന് ആഗ്രഹിച്ചതിനെ സംബന്ധിച്ചാണ്.

മറുപടി:

ചരിത്ര നിവേദനങ്ങൾ വളരെ ദുർബലവും സ്വീകാര്യയോഗ്യവും സ്ഥിരപ്പെട്ടവയുമായ ഹദീസുകൾക്ക് എതിരുമാണ്.

കാരണങ്ങൾ:

1. പ്രവാചകൻ (സ) സൈദിന്റെ(റ) ഭാര്യയെ മോഹിച്ചുവെന്നോ അദ്ദേഹത്തെ കൊണ്ട് ഭാര്യയെ നിർബന്ധപൂർവ്വം വിവാഹമോചനം ചെയ്യിപ്പിച്ചുവെന്നോ നേരിയ സൂചനയെങ്കിലും ഉൾകൊള്ളുന്ന നിവേദനങ്ങളുടെ നിവേദക പരമ്പരകൾ വളരെ ദുർബലങ്ങളാണ്: (അവലംബം: http://www.ebnmaryam.com/Zaynab.htm)

നിവേദക പരമ്പര: ഒന്ന്

ذكرها ابن سعد في طبقاته (8/101) و من طريقه ساقها ابن جرير في تاريخه (3/161) : قال : أخبرنا محمد بن عمر ، قال : حدثني عبد الله بن عامر الأسلمي ، عن محمد بن يحيى بن حبان ، قال :

ത്വബകാത്തു ഇബ്‌നു സഅ്ദ് (8/101): നമ്മോട് മുഹമ്മദിബ്‌നു ഉമർ പറഞ്ഞു – എന്നോട് അബ്ദുല്ലാഹിബ്‌നു ആമിറുൽ അസ്‌ലമി പറഞ്ഞു – മുഹമ്മദിബ്‌നു യഹ്‌യ ഇബ്‌നുഹിബ്ബാൻ പറഞ്ഞു:…

ഈ നിവേദക പരമ്പരയിൽ മൂന്ന് ന്യൂനതകൾ ഉണ്ട്:

ഒന്നാമത്തെ ന്യൂനത: നിവേദനം മുർസൽ (المرسل ) അഥവാ പരമ്പര കണ്ണിമുറിഞ്ഞതാണ്. പ്രവാചക കാലഘട്ടത്തിലെ ഈ സംഭവം പ്രവാചക കാലഘട്ടത്തിന് ശേഷം, ഒരു തലമുറക്ക് ശേഷം ജനിച്ച മുഹമ്മദിബ്‌നു യഹ്‌യ ഇബ്‌നുഹിബ്ബാൻ എന്ന താബിഈ (ജനനം: ഹിജ്റാബ്ദം: 47) യാണ് പ്രസ്ഥാവിക്കുന്നത്. അദ്ദേഹം ഈ സംഭവത്തിന് സാക്ഷിയല്ല. (തഹ്ദീബ്:9/507-508)

രണ്ടാമത്തെ ന്യൂനത: നിവേദക പരമ്പരയിലെ അബ്ദുല്ലാഹിബ്‌നു ആമിറുൽ അസ്‌ലമി ദുർബലനാണ് എന്നതിൽ ഹദീസ് പണ്ഡിതന്മാർ യോജിച്ചിരിക്കുന്നു. ഇമാം അബൂ ഹാതിം പറഞ്ഞു: വിശ്വസ്തരായ നിവേദകർക്കെതിരെ വളരെ ദുർബലമായ നിവേദനങ്ങൾ ഉദ്ധരിക്കുന്ന വ്യക്തിയാണ് അബ്ദുല്ലാഹിബ്‌നു ആമിറുൽ അസ്‌ലമി. (തഹ്ദീബ്: 5/275, മീസാനുൽ ഇഅ്തിദാൽ: 2/448 )

മൂന്നാമത്തെ ന്യൂനത: നിവേദക പരമ്പരയിലെ മുഹമ്മദിബ്‌നു ഉമർ വാക്കിദുൽ അസ്‌ലമി (സത്യസന്ധതയിൽ) ദുർബലനാണ്. അഹ്മദിബ്‌നു ഹമ്പൽ പറഞ്ഞു: മുഹമ്മദിബ്‌നു ഉമർ വാക്കിദുൽ അസ്‌ലമി നുണയനാണ്; അയാൾ ഹദീസുകളിൽ കോട്ടിമാട്ടുമായിരുന്നു. യഹ്‌യ പറഞ്ഞു: അയാൾ വിശ്വസ്തനല്ല. അയാളുടെ ഹദീസുകൾ എഴുതിവെക്കാൻ കൊള്ളാത്തത്രയും അവിശ്വസനീയമാണ്. ഇമാം ബുഖാരി, റാസി, നസാഈ എന്നിവർ പറഞ്ഞു: അയാൾ കളവു കൊണ്ട് ആരോപിതനാണ്. റാസി, നസാഈ എന്നിവർ പറഞ്ഞു: അയാൾ വ്യാജ ഹദീസുകൾ ഉണ്ടാക്കുന്ന വ്യക്തിയായിരുന്നു. ഇമാം ദാറക്കുത്നി പറഞ്ഞു: അയാളിൽ ദൗർബല്യമുണ്ട്. ഇസ്ഹാകിബ്‌നു റാഹൂയ പറഞ്ഞു: അയാൾ നുണയനാണ്. (അദ്ദുഅഫാഉ വൽ മത്റൂകീൻ: ഇബ്‌നുൽ ജൗസി: 3 / 87, അദ്ദുഅഫാഉ സ്സ്വഗീർ: ബുഖാരി: 334, അൽ ജർഹുവതഅദീൽ: അബൂഹാതിം: 8/21, അൽ കാമിൽ ഇബ്‌നു അദിയ്യ്: 7/ 481)

നിവേദക പരമ്പര: രണ്ട്:

ذكرها ابن جرير في تفسيره (22/13 ) قال : حدثني يونس ، قال : أخبرنا ابن وهب ، قال : قال ابن زيد : كان النبي صلى الله عليه وسلم

ത്വബ്‌രി തന്റെ തഫ്സീറിൽ (22/13) പറഞ്ഞു: എന്നോട് യൂനസ് പറഞ്ഞു – നമ്മോട് ഇബ്‌നു വഹബ് അറിയിച്ചു – ഇബ്‌നു സൈദ് പറഞ്ഞു:….

ഈ നിവേദക പരമ്പരയിൽ രണ്ട് ന്യൂനതകൾ ഉണ്ട്:

ഒന്നാമത്തെ ന്യൂനത: നിവേദനം മുഅ്ദൽ (المعضل) അഥവാ പരമ്പരയിലെ രണ്ട് കണ്ണികൾ മുറിഞ്ഞതാണ്. പ്രവാചക കാലഘട്ടത്തിലെ ഈ സംഭവം പ്രവാചക കാലഘട്ടത്തിന് രണ്ട് തലമുറക്ക് ശേഷം ജനിച്ച ഇബ്‌നു സൈദ് എന്ന അബ്ദുർറഹ്മാൻ ഇബ്‌നു സൈദ് ഇബ്‌നു അസ്‌ലം ആണ് ഉദ്ധരിക്കുന്നത്. അദ്ദേഹം ഈ സംഭവത്തിന് സാക്ഷിയല്ല. പ്രവാചകാനുചരനോ (സ്വഹാബി), പ്രവാചകാനുചരരുടെ അനുചരനോ (താബിഈ) അല്ല. അതുകൊണ്ട് തന്നെ പരമ്പരയിൽ രണ്ടോ രണ്ടിലധികമോ നിവേദകർ വിട്ടു പോയിട്ടുണ്ട്.

രണ്ടാമത്തെ ന്യൂനത: അബ്ദുർറഹ്മാൻ ഇബ്‌നു സൈദ് ഇബ്‌നു അസ്‌ലം എന്ന ഈ നിവേദകൻ തന്നെ അങ്ങേയറ്റം ദുർബലനാണ് എന്ന കാര്യത്തിൽ ഹദീസ് പണ്ഡിതർ യോജിച്ചിരിക്കുന്നു. ഇമാം ബുഖാരി, അബൂ ഹാതിം, അലിയ്യിബ്‌നുൽ മദീനി എന്നിവർ പറയുന്നത് ഇയാൾ അങ്ങേയറ്റം ദുർബലനാണ് എന്നാണ്. ചിലരെല്ലാം ഇയാൾ മത്റൂക് (متروك) അഥവാ കളവ് കൊണ്ട് ആരോപിതനാണെന്നും അഭിപ്രായപ്പെടുന്നു. (തഹ്ദീബ്: 6/178)

നിവേദക പരമ്പര: മൂന്ന്:

ذكرها أحمد في مسنده (3/149-150) ، قال : حدثنا مؤمل بن إسماعيل قال : حدثنا حماد بن زيد ، قال : حدثنا ثابت عن أنس قال :…

അഹ്മദ് തന്റെ മുസ്നദിൽ (3/149-150) പറഞ്ഞു: നമ്മോട് മുഅമ്മിൽ ഇബ്‌നു ഇസ്മാഈൽ പറഞ്ഞു – നമ്മോട് ഹമ്മാദി ബ്‌നു സൈദ് പറഞ്ഞു – നമ്മോട് സാബിത് പറഞ്ഞു – അദ്ദേഹം അനസിൽ നിന്ന്:….

നിവേദക പരമ്പരയിലെ മുഅമ്മിൽ ഇബ്‌നു ഇസ്മാഈലിനെ ഹദീസ് പണ്ഡിതന്മാരിൽ അധികപേരും വളരെ ദുർബലനായ നിവേദകനായാണ് കാണുന്നത്. അദ്ദേഹം തന്റെ ഗുരുവര്യമാരിൽ നിന്ന് മുൻകറായ ഹദീസുകൾ (വിശ്വസ്‌തരായ നിവേദകരുടെ നിവേദനങ്ങൾക്ക് ഘടക വിരുദ്ധമായ അങ്ങേയറ്റം ദുർബലമായ നിവേദനങ്ങൾ) ഉദ്ധരിക്കുമായിരുന്നു. മുഹമ്മദിബ്‌നു നസ്വർ അൽ മറൂസി പറഞ്ഞു: മുഅമ്മിൽ ഒരു ഹദീസ് ഒറ്റക്ക് നിവേദനം ചെയ്താൽ അത് സ്വീകരിക്കുന്നതിന് മുമ്പ് പരിശോധനക്ക് വിധേയമാക്കണം. കാരണം അയാൾ നിവേദനത്തിൽ ഒട്ടനവധി പിഴവുകൾ വരുത്തുന്ന, മനപാഠ ശേഷി തീർത്തും മോശപ്പെട്ട വ്യക്തിയാണ്. (തഹ്ദീബ്: 10/381)

നിവേദക പരമ്പര: നാല്:

رواها ابن جرير في تفسيره (22/13) ، قال : حدثنا بشر ، قال : حدثنا يزيد ، قال : حدثنا سعيد عن قتادة

ഇബ്‌നു ജരീർ തന്റെ തഫ്സീറിൽ (22/13) പറഞ്ഞു: നമ്മോട് ബിശ്ർ പറഞ്ഞു- നമ്മോട് യസീദ് പറഞ്ഞു – നമ്മോട് സഈദ് പറഞ്ഞു – കത്താദയിൽ നിന്ന്:….

കത്താദ പ്രവാചകാനുചരനല്ല എന്ന് മാത്രമല്ല സംഭവത്തിന് സാക്ഷിയായ ആരിൽ നിന്നുമല്ല ഉദ്ധരിക്കുന്നത്. അതുകൊണ്ട് തന്നെ പരമ്പര മുറിഞ്ഞതാണ്. മാത്രമല്ല കത്താദ ആരിൽ നിന്നാണോ ഒരു സംഭവം ഉദ്ധരിക്കുന്നത് അവരിൽ നിന്ന് നേരിട്ട് കേട്ടു എന്ന് സൂചിപ്പിക്കുന്ന പ്രയോഗം നിവേദനത്തിൽ ഉപയോഗിച്ചില്ലെങ്കിൽ കത്താദയുടെ നിവേദനം സ്വീകരിക്കപ്പെടില്ല എന്ന് പണ്ഡിതന്മാർ വ്യക്തമാക്കിയിട്ടുണ്ട്. (ജാമിഉ തഹ്സ്വീൽ ഫീ അഹ്കാമിൽ മറാസീൽ: 101)

കുർതുബി തന്റെ തഫ്സീറിൽ (14/190) മുകാതിൽ എന്ന നിവേദകനിൽ നിന്ന് ഉദ്ധരിക്കുന്നതാണ് മറ്റൊരു നിവേദനം. മുകാതിലിന്റെ മുഴുവൻ നാമം മുകാതിലിബ്‌നു സുലൈമാൻ എന്നാണ്. ഇയാൾ നുണയനും വ്യാജ ഹദീസ് നിർമാതാവുമാണെന്ന് ഹദീസ് പണ്ഡിതർ വ്യക്തമാക്കിയിട്ടുണ്ട്. (തഹ്ദീബ്: 10/279-285) മറ്റു ചില പണ്ഡിതർ തങ്ങളുടെ സീറകളിൽ സമാനമായ നിവേദനങ്ങൾ സൂചിപ്പിക്കുന്നുണ്ടെങ്കിലും അവക്കൊന്നും തന്നെ നിവേദക പരമ്പര പോലും നൽകാത്തതിനാൽ അവ അസ്വീകാര്യമാണ്.

2. പ്രവാചകൻ (സ) സൈദിനെ കൊണ്ട് ഭാര്യയെ നിർബന്ധപൂർവ്വം വിവാഹമോചനം ചെയ്യിപ്പിച്ചു എന്ന് സൂചിപ്പിക്കുന്ന ചരിത്ര നിവേദനങ്ങളെല്ലാം വ്യാജങ്ങളും തനി കളവുകളുമാണെന്നും മുസ്‌ലിം പണ്ഡിതന്മാർ തന്നെ – ഇസ്‌ലാമിക വിമർശകർ ജന്മമെടുക്കുന്നതിന് നൂറ്റാണ്ടുകൾക്ക് മുമ്പ് – വ്യക്തമാക്കിയിട്ടുണ്ട്.

ഇബ്‌നുൽ അറബി പറഞ്ഞു: ഈ നിവേദനങ്ങളുടെയെല്ലാം പരമ്പര അടിസ്ഥാനരഹിതങ്ങളാണ്. (അഹ്കാമുൽ കുർആൻ: 3/1543)

ഇമാം കുർതുബി പറഞ്ഞു: …. ഇത്തരം നിവേദനങ്ങളുടെ ഉൽഭവം പ്രവാചകന്റെ പാപസുരക്ഷിതത്വത്തെ സംബന്ധിച്ച് യാതൊരു അറിവുമില്ലാത്ത വിഡ്ഢികളിൽ നിന്നും പ്രവാചകന്റെ പാവനത്വം കുറച്ചു കാണുന്നവരിൽ നിന്നും മാത്രമാണ്. (അൽ ജാമിഉ ലി അഹ്കാമിൽ കുർആൻ: 14/191)

ഇമാം ഇബ്‌നുകസീർ, ഇബ്‌നു ഹജർ, ശൈഖ് ശംക്വീത്വി തുടങ്ങി ഒട്ടനവധി ഹദീസ് – ചരിത്ര പണ്ഡിതർ വേറെയും ഇത്തരം നിവേദനങ്ങളെല്ലാം അടിസ്ഥാനരഹിതമാണെന്ന അഭിപ്രായക്കാരാണ്. (തഫ്സീറുൽ കുർആനിൽ അളീം: 3/491, ഫത്ഹുൽ ബാരി: 8/524, അദ്‌വാഉൽ ബയാൻ: 6/639)

മഹാ ഭൂരിപക്ഷം വരുന്ന ഹദീസ് – ചരിത്ര പണ്ഡിതന്മാർ പ്രമാണബദ്ധമായി വ്യക്തമാക്കിയതിന് വിപരീതമായി, സൈദ് – സൈനബ് ദാമ്പത്യ കഥയുമായി ബന്ധപ്പെട്ട വ്യാജ നിവേദനങ്ങളുടെ അടിസ്ഥാനത്തിൽ പ്രവാചകന്റെ(സ) കണ്ണ് പതിഞ്ഞ സ്ത്രീകളെ ഭർത്താക്കന്മാർ വിവാഹം മോചനം ചെയ്യണമെന്നും അത്തരം സ്ത്രീകളെ വിവാഹം ചെയ്യൽ പ്രവാചകന് (സ) അനുവദനീയമാണ് എന്നുമൊക്കെ ചില മുസ്‌ലിം പണ്ഡിതന്മാരുടെ ഒറ്റപ്പെട്ട അഭിപ്രായങ്ങൾ ഗ്രന്ഥങ്ങളിൽ (തഫ്സീറുൽ കുർതുബി) എഴുതപ്പെട്ടിരിക്കുന്നു എന്നത് ഇസ്‌ലാം മതത്തിനോ, മുസ്‌ലിംകൾക്കോ ബാധകമല്ല. അവയൊന്നും ഇസ്‌ലാം മതത്തിൽ പ്രമാണമല്ല. ഇസ്‌ലാമിലെ പ്രമാണം കുർആനും സ്വഹീഹായ ഹദീസുകളുമാണ്. ഈ തെളിവുകളുടെ പിൻബലമില്ലാത്ത അത്തരം കേവലാഭിപ്രായങ്ങൾ, പ്രമാണ വിരുദ്ധമായ നിലപാടുകൾ യാതൊരു പരിഗണനയും കൂടാതെ തള്ളിക്കളയണമെന്ന് മുസ്‌ലിം പണ്ഡിതന്മാർ തന്നെ വ്യക്തമാക്കിയിട്ടുണ്ട്.

3. ഈ വ്യാജ നിവേദനങ്ങളുടെ നിവേദകർ ദുർബലരും അവിശ്വസ്ഥരുമാണ് എന്നതിന് പുറമെ ഇവയുടെ ഉള്ളടക്കവും (മത്ന് المتن) ധാരാളം വൈരുധ്യങ്ങൾ നിറഞ്ഞവയാണ്:

ചില നിവേദനങ്ങളിൽ സൈദ് നാട്ടിലില്ലാത്തപ്പോൾ പ്രവാചകൻ (സ) അദ്ദേഹത്തിന്റെ വീട്ടിൽ ചെന്നുവെന്നും അപ്പോൾ സൈനബിനെ കണ്ടുമുട്ടി എന്നും ചില നിവേദനങ്ങളിൽ സൈദ് രോഗിയായിരിക്കെ വീട്ടിൽ സന്ദർശിച്ചുവെന്നും അപ്പോഴാണ് സൈനബിനെ കണ്ടുമുട്ടിയത് എന്നും കാണാം! ഇത് രണ്ടും എങ്ങനെ യോജിക്കും?! സൈനബിനെ പ്രവാചകൻ (സ) കണ്ടത് എങ്ങനെയാണെന്നതിലും ഈ വ്യാജ നിവേദനങ്ങൾ പരസ്പരം വൈരുധ്യത്തിലാണ്. ജനലിലൂടെ സൈനബിനെ കണ്ടു എന്ന് ചില നിവേദനങ്ങൾ… മുടി കെട്ട് കൊണ്ടുള്ള മറ കാറ്റിൽ പാറിയപ്പോൾ കണ്ടു എന്ന് മറ്റു ചിലതിലും പറയുന്നു.

പ്രവാചകന് (സ) പന്ത്രണ്ട് വയസ്സുള്ളപ്പോൾ സൈനബ് (റ) ജനിച്ചിട്ടുണ്ട്. പ്രവാചകന്റെ (സ) പിതൃവ്യപുത്രിയാണ് സൈനബ് (റ). സൈനബ് (റ) വളർന്നതും യുവതിയായതും പ്രവാചകന്റെ (സ) കൺമുമ്പിൽ തന്നെയാണ്. അവരെ സൈദിന് (റ) വിവാഹം കഴിപ്പിച്ചു നൽകിയതും പ്രവാചകൻ (സ) തന്നെ. പലവുരു പ്രവാചകൻ (സ) സൈനബിനെ (റ) കണ്ടിട്ടുണ്ട് എന്ന വസ്തുത ഈ വ്യാജ നിവേദനങ്ങളെയെല്ലാം ഖണ്ഡിക്കുന്നു. സൈനബ് (റ) മക്കയിൽ വെച്ച് ഇസ്‌ലാം സ്വീകരിച്ചു. പ്രവാചകനോടൊപ്പം മദീനയിലേക്ക് പലായനം ചെയ്തു. പ്രവാചകന് (സ) സൈനബിനെ (റ) വിവാഹം ചെയ്യാൻ ആഗ്രഹമുണ്ടായിരുന്നെങ്കിൽ സൈദിന് വിവാഹം ചെയ്യിപ്പിക്കുന്നതിന് മുമ്പ് ഒരു സൂചനയെങ്കിലും നൽകിയിരുന്നെങ്കിൽ സൈനബ് (റ) അതിന് തയ്യാറാകുമായിരുന്നു. എന്ന് മാത്രമല്ല സൈദുമായി വിവാഹം നടക്കുന്നതിന് മുമ്പ് വിവാഹമൂല്യമൊന്നുമില്ലാതെ, തന്നെ വിവാഹം ചെയ്യാൻ തയ്യാറാണോ എന്ന് പ്രവാചകനോട് (സ) അങ്ങോട്ട്, സൈനബ് (റ) ചോദിച്ചതായും ചില ചരിത്ര നിവേദനങ്ങളിൽ കാണാം. (അഹ്കാമുൽ കുർആൻ: 3/1543) എന്നിരിക്കെ സൈദുമായുള്ള വിവാഹ ശേഷം സൈനബിനെ (റ) വിവാഹം കഴിക്കാൻ പ്രവാചകൻ (സ) നടത്തിയ ‘പരാക്രമ കഥകൾ’ ചിരിച്ചു തള്ളേണ്ട ഫലിതങ്ങൾ മാത്രമാണ്.

4. സ്വീകാര്യയോഗ്യമായ ഹദീസുകളും (الحديث الصحيح) സ്ഥിരപ്പെട്ട ചരിത്ര നിവേദനങ്ങളും തെളിയിക്കുന്നത് ദുർബലമായ ഈ കഥക്ക് നേർവിപരീതമാണ്:

പ്രവാചകൻ (സ) സൈദിന് (റ) സൈനബിനെ (റ) വിവാഹം കഴിപ്പിച്ചു കൊടുത്തു. എന്നാൽ ഇരുവർക്കും പരസ്പരം സ്നേഹ സമ്പന്നമായ ഒരു ദാമ്പത്യ ജീവിതത്തിൽ നിരാശ സംജാതമായി. അവർ പരസ്പരം അകന്നു. സൈദിന് (റ) ഭാര്യയുമായി യോജിച്ചു പോകാനാകില്ലെന്നു കണ്ടപ്പോള്‍ അവരെ വിവാഹമോചനം നടത്തുന്നതിനെപ്പറ്റി പ്രവാചകനോട് (സ) പരാതിപ്പെട്ടപ്പോഴും ആ ബന്ധം തുടരാനാണ് പ്രവാചകൻ (സ) അദ്ദേഹത്തോട് കൽപ്പിച്ചത് എന്നത് സ്വീകാര്യയോഗ്യമായ ഹദീസിലൂടെ സ്ഥിരപ്പെട്ട ഒരു വസ്തുതയാണ്.

جَاءَ زَيْدُ بْنُ حَارِثَةَ يَشْكُو ، فَجَعَلَ النَّبِيُّ صَلَّى اللَّهُ عَلَيْهِ وَسَلَّمَ يَقُولُ : ( اتَّقِ اللَّهَ وَأَمْسِكْ عَلَيْكَ زَوْجَكَ )

സൈദുബ്‌നു ഹാരിസ (റ) സങ്കടപ്പെട്ടുകൊണ്ടു പ്രവാചകന്റെ (സ) അടുക്കല്‍ വന്നു. അപ്പോള്‍ പ്രവാചകൻ (സ) ഇപ്രകാരം അദ്ദേഹത്തോട് ഉപദേശിച്ചു കൊണ്ടിരുന്നു: “നീ അല്ലാഹുവിനെ സൂക്ഷിക്കുക: നിന്‍റെ ഭാര്യയെ കൂടെ താമസിപ്പിക്കുകയും ചെയ്യുക (വിവാഹ മോചനം ചെയ്യരുത്)”. (സ്വഹീഹുൽ ബുഖാരി: 6984, ജാമിഉൽ ബയാൻ: ത്വബ്‌രി: 22/11, തഫ്‌സീറുൽ കുർആനിൽ അളീം: ഇബ്‌നുകസീർ: 3/489)

പ്രവാചകൻ (സ) സൈദിനെ(റ) കൊണ്ട് ഭാര്യയെ വിവാഹമോചനം ചെയ്യിപ്പിച്ചു എന്ന കളള കഥക്ക് നേർ വിപരീതമായി സൈനബുമായി ദാമ്പത്യ ജീവിതം തുടരാൻ പരമാവധി പരിശ്രമിക്കാനാണ് പ്രവാചകൻ (സ) ഉപദേശിച്ചത് എന്ന് വ്യക്തമായും ഹദീസ് തെളിയിക്കുന്നു. അങ്ങനെ ദാമ്പത്യം തീരെ തുടർന്നു പോകാൻ സാധിക്കാത്തതിനാൽ ദമ്പതികൾ പരസ്പരം വേർപിരിഞ്ഞുവെന്നും ശേഷം സൈനബ് (റ) തന്റെ ഇദ്ദാ കാലഘട്ടം കഴിഞ്ഞപ്പോൾ പ്രവാചകനെ വിവാഹം ചെയ്യുകയും അതിൽ അതിയായി സന്തോഷിക്കുകയും ചെയ്തുവെന്ന് സ്വീകാര്യയോഗ്യമായ ഹദീസുകളിലും സ്ഥിരപ്പെട്ട ചരിത്ര നിവേദനങ്ങളിലും നാം വായിക്കുന്നു. സൈനബ് (റ), പ്രവാചകനുമായുള്ള വിവാഹത്തോടുകൂടി സത്യവിശ്വാസികളുടെ മാതാക്കളില്‍ ഒരാളായിത്തീരുകയും അഭിമാനപൂര്‍വ്വം പ്രവാചകന്റെ(സ) ഭാര്യമാരോടു ഇങ്ങിനെ പറയുകയും ചെയ്യുമായിരുന്നുവെന്ന് ഹദീസില്‍ വന്നിരിക്കുന്നു: “നിങ്ങളെ നിങ്ങളുടെ വീട്ടുകാര്‍ വിവാഹം ചെയ്യിച്ചു കൊടുത്തതാണ്; എന്നെ ഏഴു ആകാശങ്ങള്‍ക്കു മീതെ നിന്നു അല്ലാഹു വിവാഹം ചെയ്യിച്ചു കൊടുത്തതാണ്.” (സ്വഹീഹുൽ ബുഖാരി: 6984).

ഇതെല്ലാം താഴെ പറയുന്ന കാര്യങ്ങൾ തെളിയിക്കുന്നു:

* പ്രവാചകൻ (സ) സൈദിനോട് (റ) തന്റെ ഭാര്യയെ വിവാഹ മോചനം ചെയ്യാൻ കൽപ്പിച്ചിട്ടില്ല. നേർ വിപരീതമാണ് പ്രവാചകൻ (സ) അദ്ദേഹത്തോട് ഉപദേശിച്ചത്.

* സൈദിന്(റ) സൈനബിനോടൊപ്പവും(റ) സൈനബിന്(റ) സൈദിനോടൊപ്പവുമുള്ള(റ) ദാമ്പത്യ ജീവിതം തൃപ്തികരമായിരുന്നില്ല. അവർ തമ്മിൽ നടന്ന വിവാഹ മോചനം അവർ സ്വയം എടുത്ത തീരുമാനത്തിന്റെ അടിസ്ഥാനത്തിലായിരുന്നു. പ്രവാചകന്(സ) അതിൽ യാതൊരു പങ്കുമില്ല.

* മാസങ്ങൾക്ക് ശേഷം പ്രവാചകനുമായി(സ) നടന്ന വിവാഹത്തിൽ സൈനബ്(റ) അങ്ങേയറ്റം സന്തുഷ്ടയും അഭിമാനിയുമായിരുന്നു എന്നത് സൈദ് – സൈനബ് (റ) ദമ്പതികളുടെ വിവാഹ മോചനം അവർ സ്വയം എടുത്ത തീരുമാനത്തിന്റെ അടിസ്ഥാനത്തിലായിരുന്നു എന്ന് തെളിയിക്കുന്നു.

5. “അല്ലാഹു വെളിപ്പെടുത്താന്‍ പോകുന്ന ഒരു കാര്യം നിന്‍റെ മനസ്സില്‍ നീ മറച്ചു വെക്കുകയും ജനങ്ങളെ നീ പേടിക്കുകയും ചെയ്തിരുന്നു. എന്നാല്‍ നീ പേടിക്കുവാന്‍ ഏറ്റവും അര്‍ഹതയുള്ളവന്‍ അല്ലാഹുവാകുന്നു.” (കുർആൻ: 33:37) എന്ന് കുർആൻ സൂചിപ്പിച്ചത് പ്രവാചകൻ (സ) സൈദിന്റെ(റ) ഭാര്യയെ മോഹിക്കുകയും അദ്ദേഹത്തെ കൊണ്ട് ഭാര്യയെ വിവാഹമോചനം ചെയ്യിപ്പിക്കുകയും ചെയ്തതിനെ സംബന്ധിച്ചാണ് എന്നതും ശുദ്ധ അസംബന്ധമാണ്. കാരണങ്ങൾ:

* അതായിരുന്നു മുഹമ്മദ് നബി (സ) തന്റെ മനസ്സിൽ മറച്ചുവെച്ച രഹസ്യമെങ്കിൽ അദ്ദേഹം തന്നെ രചിച്ചുവെന്ന് വിമർശകർ വാദിക്കുന്ന ‘ഖുറാനിൽ’ ആ കള്ളത്തരം സൂചിപ്പിക്കുന്ന ഒരു വചനം അദ്ദേഹം തന്നെ മാലോകർക്ക് ഓതി കൊടുക്കുമായിരുന്നോ ?

* സ്വീകാര്യയോഗ്യമായ ഒരു ഹദീസും ഇത്തരമൊരു വാദത്തിന് പിൻബലമേകുന്നില്ല.

* സ്വീകാര്യയോഗ്യമായ നിവേദനങ്ങൾ, എന്താണ് മുഹമ്മദ് നബി (സ) തന്റെ ‘മനസ്സിൽ മറച്ചുവെച്ച ഒരു കാര്യം’ എന്ന് വ്യക്തമാക്കിയിട്ടുണ്ട്. ആ രഹസ്യമെന്താണ് എന്ന് കാണുക: “സൈദ്‌ (റ) തന്‍റെ ഭാര്യയുമായി യോജിച്ചു പോകുകയില്ലെന്നു കണ്ടപ്പോള്‍ അവരെ വിവാഹമോചനം (طلاق) നടത്തുന്നതിനെപ്പറ്റി തിരുമേനി(സ)യോട് ആലോചിച്ചു. ഇരുഭാഗത്തെയും ഗുണകാംക്ഷിയാണല്ലോ നബി(സ). അപ്പോഴായിരുന്നു അദ്ദേഹത്തോടു നബി(സ) ഭാര്യയെ വെച്ചുകൊള്ളുവാനും, അല്ലാഹുവിനെ സൂക്ഷിക്കുവാനും പറഞ്ഞത്. ഈ വിവാഹം അധികം നീണ്ടുപോകുകയില്ലെന്നും, സൈദ്‌ (റ) ഭാര്യയെ വേര്‍പ്പെടുത്തുക തന്നെ ചെയ്യുമെന്നും, അനന്തരം സൈനബ(റ)യെ തിരുമേനി(സ) വിവാഹം ചെയ്‌വാനിരിക്കുന്നുവെന്നും തിരുമേനി(സ)ക്കു (ദിവ്യ ബോധനത്തിലൂടെ) അറിവു ലഭിച്ചിട്ടുണ്ടായിരുന്നു. പക്ഷേ, നടക്കുവാനിരിക്കുന്ന ആ വിഷയത്തെക്കുറിച്ച് സൈദി(റ)നോടു തിരുമേനി(സ) ഒന്നും പ്രസ്താവിച്ചില്ല. ഭാര്യയെ വിവാഹമോചനം നടത്താതിരിക്കുവാന്‍ സാധാരണമട്ടില്‍ ഉപദേശിക്കുകയാണ് ചെയ്തത്. ജനസംസാരത്തിനു ഇടയാകരുതെന്നും, അല്ലാഹു ഉദ്ദേശിച്ച കാര്യം അവന്‍ ഉദ്ദേശിക്കുമ്പോള്‍ അവന്‍ നടപ്പിലാക്കിക്കൊള്ളുമെന്നും തിരുമേനി(സ) കരുതി. ഇതിനെ സൂചിപ്പിച്ചുകൊണ്ടാണ് ‘അല്ലാഹു വെളിവാക്കുവാന്‍ പോകുന്ന കാര്യം നീ നിന്‍റെ മനസ്സില്‍ മറച്ചുവെക്കുകയും ചെയ്തിരിക്കുന്നു’, (وَتُخْفِي فِي نَفْسِكَ مَا اللَّـهُ مُبْدِيهِ) എന്നു പറഞ്ഞത്. (മറിച്ചു ചിലര്‍ പ്രസ്താവിക്കുന്നതുപോലെ, അവരെ വിവാഹംകഴിച്ചാല്‍ കൊള്ളാമെന്ന ആഗ്രഹമല്ല എന്നു താല്‍പര്യം.) യഥാര്‍ത്ഥവും സത്യവും തുറന്നു പറയുന്നതില്‍ ആരെയും ശങ്കിക്കേണ്ടതില്ല, അതില്‍ ജനസംസാരം ഭയപ്പെടേണ്ടതുമില്ല, അതു തുറന്നു പറയാതിരിക്കുന്നതില്‍ അല്ലാഹുവിനെ ഭയപ്പെടുകയാണ് വേണ്ടത് എന്ന് (അല്ലാഹു) നബി(സ)യെ ഓര്‍മ്മിപ്പിക്കുകയും ചെയ്യുന്നു.” (തഫ്സീറുൽ അമാനി: 33:37 ന്റെ വ്യാഖ്യാനം)

അഥവാ പ്രവാചകനായത് കൊണ്ട് തന്നെ – സൈദ് സൈനബിനെ വിവാഹമോചനം ചെയ്യുമെന്നും കാലങ്ങൾക്ക് ശേഷം താൻ സൈനബിനെ വിവാഹം ചെയ്യുമെന്നുമുൾപ്പെടെ – ഭാവിയിൽ നടക്കാനിരിക്കുന്ന കാര്യങ്ങൾ അദ്ദേഹത്തിന് അല്ലാഹു നൽകിയ ദിവ്യബോധനത്തിലൂടെ അറിയാമായിരുന്നു. എന്നിട്ടും അവ മനസ്സിൽ രഹസ്യമായി മറച്ചുവെച്ച് സൈനബുമായി ദാമ്പത്യ ജീവിതം തുടരാൻ പരമാവധി പരിശ്രമിക്കാനാണ് പ്രവാചകൻ (സ) സൈദിനെ ഉപദേശിച്ചത്. ഇതാകട്ടെ പ്രവാചകന്റെ(സ) മാന്യതയേയും ലൈംഗിക ശുദ്ധിയേയുമാണ് തെളിയിക്കുന്നത്. അദ്ദേഹം സ്ത്രീലമ്പടനായിരുന്നു എന്ന വിമർശകരുടെ വാദത്തെ ഈ സംഭവം തച്ചുടക്കുന്നു.

രാത്രി, പാത്രത്തിൽ ഒഴിച്ചിരുന്ന പ്രവാചകന്റെ(സ) മൂത്രം ഉമ്മു അയ്‌മൻ (റ) കുടിച്ചു എന്ന കഥ. (മുസ്തദ്റക് ഹാകിം: 70/ 4)

ഹദീസ് വളരെ ദുർബലമാണ്.

കാരണങ്ങൾ:

1, നിവേദകനായ നുബൈഹ് അൽ അനസിയുടേയും ഉമ്മു അയ്‌മന്റേയും ഇടയിൽ നിവേദക പരമ്പര മുറിഞ്ഞതാണെന്ന് ഇമാം ഇബ്‌നു ഹജർ വ്യക്തമാക്കുന്നു. (അത്തൽഖീസുൽ ഹബീർ ഫീ തഖ്‌രീജി അഹാദീസിർ റാഫിഈ: 1/171)

2. നിവേദക പരമ്പരയിലെ (സനദ്) ‘അബൂമാലിക് അന്നഖഇ’ യുടെ മുഴുവൻ നാമം അബ്ദുൽ മലിക് ഇബ്‌നു ഹുസൈൻ എന്നാണ്. ഇയാൾ ദുർബലനാണെന്ന കാര്യത്തിൽ ഹദീസ് പണ്ഡിതന്മാരെല്ലാം ഏകാഭിപ്രായക്കാരാണ്.

ഇമാം നസാഇ പറഞ്ഞു: അയാൾ കളവ് പറയുന്നവനായി ആരോപിതനാണ്. ഇമാം അബൂ ഹാതിം പറഞ്ഞു: അയാളുടെ ഹദീസുകൾ ദുർബലമാണ്. ഇമാം അംറിബ്‌നു അലി പറഞ്ഞു: ദുർബലൻ, വിശ്വസ്ഥരായ നിവേദകർക്കെതിരെ ദുർബലമായ ഹദീസ് ഉദ്ധരിക്കുന്ന വ്യക്തി. ഇമാം നസാഇയുടെ ‘ദുഅഫാഉ വൽ മത്റൂകീൻ’, ഇബ്‌നു അബീഹാതിമിന്റെ ‘ജർഹുവ തഅ്ദീൽ’ എന്നീ ഗ്രന്ഥങ്ങളിൽ ഇത് വ്യക്തമാക്കപ്പെട്ടിട്ടുണ്ട്.

കൂടാതെ ഇമാം ദാറകുത്നിയും (അൽ ഇലല്: 15 / 415) ഇമാം ഇബ്‌നു ഹജറുൽ അസ്‌കലാനിയും (അത്തൽഖീസുൽ ഹബീർ ഫീ തഖ്‌രീജി അഹാദീസിർ റാഫിഈ: 1/171) ഈ ഹദീസ് മുകളിൽ സൂചിപ്പിച്ച കാരണങ്ങളാൾ ദുർബലമാണെന്ന് വ്യക്തമാക്കിയിട്ടുണ്ട്.

3. പ്രവാചകൻ (സ) മലമൂത്ര വിസർജനത്തിന് ശേഷം കണിശതയോടെ അവയവങ്ങൾ വൃത്തിയാക്കിയിരുന്നതായി സ്വഹീഹായ ഒട്ടനവധി ഹദീസുകളിൽ സ്ഥിരപ്പെട്ട കാര്യമാണ്. പ്രവാചകന്റെ മലമൂത്ര വിസർജ്യങ്ങൾ ശുദ്ധമായിരുന്നെങ്കിൽ അദ്ദേഹം അപ്രകാരം ചെയ്യുമായിരുന്നില്ല. ഇത്തരം കാര്യങ്ങളിൽ പ്രവാചകൻ (സ) മറ്റുള്ള മനുഷ്യന്മാരെ പോലെ തന്നെയാണ്. “(നബിയേ,) പറയുക: ഞാന്‍ നിങ്ങളെപ്പോലെയുള്ള ഒരു മനുഷ്യന്‍ മാത്രമാകുന്നു.” (ഖുർആൻ: 18: 110)

വിമർശനം:

മുഹമ്മദ് ശവക്കുഴിയിൽ വെച്ച് ശവരതിക്ക് അഹ്വാനം നൽകുന്നു. Al bukhari 23-426,374.

മറുപടി:

കുറേ നുണകൾ പറയുക, എന്നിട്ട് കുറച്ച് ‘നമ്പറും’ എഴുതുക എന്ന വെറുപ്പിന്റെ അപ്പോസ്‌തലന്മാരുടെ സ്ഥിരം ‘നമ്പർ’ തന്നെയാണ് ഇവിടെയും പയറ്റിയിരിക്കുന്നത്. ആദ്യം ഹദീസ് നേരിട്ട് വായിക്കാം:

حَدَّثَنَا عَبْدُ اللَّهِ بْنُ مُحَمَّدٍ، حَدَّثَنَا أَبُو عَامِرٍ، حَدَّثَنَا فُلَيْحُ بْنُ سُلَيْمَانَ، عَنْ هِلاَلِ بْنِ عَلِيٍّ، عَنْ أَنَسِ بْنِ مَالِكٍ ـ رضى الله عنه ـ قَالَ شَهِدْنَا بِنْتًا لِرَسُولِ اللَّهِ صلى الله عليه وسلم قَالَ وَرَسُولُ اللَّهِ صلى الله عليه وسلم جَالِسٌ عَلَى الْقَبْرِ ـ قَالَ فَرَأَيْتُ عَيْنَيْهِ تَدْمَعَانِ قَالَ ـ فَقَالَ ‏”‏ هَلْ مِنْكُمْ رَجُلٌ لَمْ يُقَارِفِ اللَّيْلَةَ ‏”‏‏.‏ فَقَالَ أَبُو طَلْحَةَ أَنَا‏.‏ قَالَ ‏”‏ فَانْزِلْ ‏”‏‏.‏ قَالَ فَنَزَلَ فِي قَبْرِهَا‏.‏

അനസ് (റ) നിവേദനം: പ്രവാചകന്റെ(സ) ഒരു പുത്രിയുടെ മരണാനന്തര ചടങ്ങിന് ഞങ്ങൾ സാക്ഷികളായി. പ്രവാചകൻ (സ) ‘ഖബ്റി’ന്നരികിൽ (ശവക്കുഴി) ഇരിക്കുകയാണ്. അനസ് (റ) പറയുന്നു: അദ്ദേഹത്തിന്റെ ഇരുകണ്ണുകളിൽ നിന്നും കണ്ണുനീരൊഴുകുന്നത് ഞാൻ കണ്ടു. പ്രവാചകൻ (സ) ചോദിച്ചു: നിങ്ങളുടെ കൂട്ടത്തിൽ ഇന്നലെ രാത്രി സ്വഭാര്യയുമായി സംസർഗത്തിൽ ഏർപ്പെടാത്തവർ അല്ലെങ്കിൽ വഴക്കിൽ ഏർപ്പെടാത്തവർ ആരെങ്കിലുമുണ്ടോ? അബൂത്വൽഹ (റ) പറഞ്ഞു: ഞാൻ ഉണ്ട്. പ്രവാചകൻ (സ) പറഞ്ഞു: എങ്കിൽ താങ്കൾ ഖബറിൽ ഇറങ്ങുക. അങ്ങനെ അദ്ദേഹം അവരുടെ ഖബറിൽ ഇറങ്ങി. (ബുഖാരി. 2. 23. 374)

ഇതിൽ ശവരതി ഒളിഞ്ഞിരിക്കുന്നത് എവിടെയാണെന്ന് ഹദീസ് വായിച്ചിട്ട് മനസ്സിലാവാത്ത നിഷ്കളങ്കരുണ്ടെങ്കിൽ അവർക്കായി വിമർശകരുടെ ദുർവ്യാഖ്യാന കസർത്ത് വ്യക്തമാക്കാം. “നിങ്ങളുടെ കൂട്ടത്തിൽ ഇന്നലെ രാത്രി സ്വഭാര്യയുമായി സംസർഗത്തിലേർപ്പെടാത്തവർ ആരെങ്കിലുമുണ്ടോ?” എന്ന് പ്രവാചകൻ (സ) ചോദിച്ചതിന്റെ അർത്ഥം “ഭാര്യയുമായി സംസർഗത്തിലേർപ്പെടാത്തവർ ഖബറിലിറങ്ങി എന്റെ മകളുമായി ശവരതിയിൽ ഏർപ്പെട്ടു കൊള്ളു” എന്ന് സൂചിപ്പിക്കുകയാണ്. ഇതാണ് പുതു തലമുറയിലെ ഇസ്‌ലാം വിമർശകരായ ഞരമ്പ് രോഗികൾ ഹദീസിന് നൽകുന്ന വ്യാഖ്യാനം !!! മകളുടെ വിയോഗത്തിൽ അടക്കാൻ കഴിയാത്ത വ്യഥയാൽ കരഞ്ഞ് കണ്ണുനീരു വാർക്കുന്ന വൃദ്ധനായ ഒരു ഉപ്പ മകളുമായി ശവരതിയിൽ ഏർപ്പെടാൻ കൽപ്പിച്ചുവെന്ന് പറയാൻ മാത്രം കരിങ്കല്ലുകളാണല്ലൊ ഇക്കൂട്ടരുടെ മനസ്സുകളുടെ സ്ഥാനത്ത്…!!!

“പിന്നീട് അതിന് ശേഷവും നിങ്ങളുടെ മനസ്സുകള്‍ കടുത്തുപോയി. അവ പാറപോലെയോ അതിനെക്കാള്‍ കടുത്തതോ ആയി ഭവിച്ചു. പാറകളില്‍ ചിലതില്‍ നിന്ന് നദികള്‍ പൊട്ടി ഒഴുകാറുണ്ട്‌. ചിലത് പിളര്‍ന്ന് വെള്ളം പുറത്ത് വരുന്നു. ചിലത് ദൈവഭയത്താല്‍ താഴോട്ട് ഉരുണ്ടു വീഴുകയും ചെയ്യുന്നു. നിങ്ങള്‍ പ്രവര്‍ത്തിക്കുന്ന യാതൊന്നിനെപറ്റിയും അല്ലാഹു ഒട്ടും അശ്രദ്ധനല്ല.” (കുർആൻ: 2:74)

ഇത്തരം രോഗാതുര മനസ്സുള്ളവരാണല്ലൊ നമുക്ക് ചുറ്റും മാന്യന്മാരെ പോലെ ജീവിക്കുന്നത് എന്നോർക്കുമ്പോൾ ദുഖവും അതിലേറെ ഭയവുമാണ് അനുഭവപ്പെടുന്നത്. നമ്മുടെ ഉമ്മ പെങ്ങമ്മാർ ശ്മശാനത്തിലും സുരക്ഷിതരാണോ എന്ന് ചിന്തിച്ച് പോവുന്നു. അല്ലാഹുവിൽ ശരണം.

Vol. 2, Book 23, Hadith 426 എന്ന് നമ്പറിട്ട് കൊടുത്ത ഹദീസിൽ തന്നെ അബൂത്വൽഹ (റ) ഖബറിൽ ഇറങ്ങിയിട്ട് എന്താണ് ചെയ്തത് എന്നും വ്യക്തമായി പറയുന്നുണ്ട് എങ്കിലും അത് ഈ പോൺ ആർട്ടിസ്റ്റുകൾ മറച്ചുവെക്കുകയാണ് പതിവ്.

فَنَزَلَ فِي قَبْرِهَا فَقَبَرَهَا‏ അങ്ങനെ അബൂത്വൽഹ (റ) ഖബറിൽ ഇറങ്ങുകയും അവരെ മറമാടുകയും ചെയ്തു. (സ്വഹീഹുൽ ബുഖാരി: 1342, അഹ്‌മദ്: 12275)

“നിങ്ങളുടെ കൂട്ടത്തിൽ ഇന്നലെ രാത്രി സ്വഭാര്യയുമായി സംസർഗത്തിലേർപ്പെടാത്തവർ ആരെങ്കിലുമുണ്ടോ?” അല്ലെങ്കിൽ “ഇന്നലെ രാത്രി സ്വഭാര്യയുമായി സംസർഗത്തിലേർപ്പെടാത്തവർ മാത്രം ഖബറിൽ ഇറങ്ങുക” എന്ന് പ്രവാചകൻ (സ) പറയാൻ കാരണം ഇസ്‌ലാമിക നിയമ പ്രകാരം തലേന്ന് ഭാര്യമാരുമായി സംസർഗത്തിലേർപ്പെട്ടവരൊ, മനസ്സ് ഏകാഗ്രമല്ലാത്തവരൊ, കലുഷമായ മനസ്സുള്ളവരോ മൃതദേഹം മറമാടുന്ന കർമ്മം ചെയ്യാൻ പാടില്ല എന്നത് കൊണ്ടാണ്. (പ്രവാചകൻ നേരിട്ട് ഇറങ്ങാതിരുന്നത് പുത്രവിയോഗ ദുഖത്താലുമാണ്). ഏകാഗ്രമല്ലാത്ത മനസ്സുമായി ചെയ്യേണ്ട ഒന്നല്ല മൃതശരീര സംസ്ക്കരണം. മരണചിന്തയിലും ഭക്തി സാന്ദ്രതയിലും പ്രാർത്ഥനാ മനസ്സോടെയും നിർവഹിക്കേണ്ട കർമ്മമാണത്.

ശൈഖ് സ്വാലിഹ് അൽ മുനജ്ജിദ് പറയുന്നു: “പുലർച്ചയാണ് മൃതശരീരം മറമാടുന്നത്. അന്ന് രാത്രി ഭാര്യമാരുമായി സംസർഗത്തിൽ ഏർപ്പെട്ട ഒരു വ്യക്തിയെ സംബന്ധിച്ചിടത്തോളം അയാളുടെ മനസ്സിൽ ആ ചിന്തകളുടെ ശേഷിപ്പുകളുണ്ടാവും. എന്നാൽ മൃതദേഹം മറവു ചെയ്യുക എന്നത് ഭക്തിസാന്ദ്രവും ഭയവും മരണചിന്തയുമെല്ലാം പ്രബലമായി നിൽക്കേണ്ട സന്ദർഭമാണല്ലൊ. അപ്പോൾ തലേന്ന് രാത്രിയിലെ ഓർമകൾ ഉള്ള ഒരാൾ മൃതദേഹം മറവു ചെയ്യുമ്പോൾ ശ്രദ്ധിക്കേണ്ട ബാധ്യതകളിലും കർമ്മങ്ങളിലും ചിലത് മറന്നു പോകാൻ സാധ്യതയുണ്ട് എന്നതിനാലാണ് തലേന്ന് ഭാര്യമാരുമായി സംസർഗത്തിൽ ഏർപ്പെടാത്തവർ മൃതദേഹം മറമാടാൻ നിർദ്ദേശിച്ചത്.” (ഇസ്‌ലാം സുആൽ വൽജവാബ്: 246638 )

ഇബ്നുൽ ജൗസി പറഞ്ഞു: “ഒരാൾ തൊട്ടടുത്ത് ചെയ്ത ഒരു കാര്യം ഓർക്കാൻ സാധ്യത കൂടുതലാണ്. ഇതില്ലാതിരിക്കാനാണ് തൊട്ടു തലേന്ന് ഭാര്യമാരുമായി സംസർഗത്തിൽ ഏർപ്പെടാത്തവർ മൃതദേഹം മറമാടാൻ നിർദ്ദേശിച്ചത്.” (കശ്ഫുൽ മുശ്കിൽ: 3: 296)

قال الحافظ ابن حجر رحمه الله وَعَلَّلَ ذَلِكَ بَعْضهمْ بِأَنَّهُ حِينَئِذٍ يَأْمَن مِنْ أَنْ يُذَكِّرهُ الشَّيْطَان بِمَا كَانَ مِنْهُ تِلْكَ اللَّيْلَة

ഇബ്നു ഹജർ പറഞ്ഞു: “പിശാച് തലേ ദിവസത്തെ ഓർമകളിൽ വല്ലതും മനസ്സിൽ ഇട്ടു കൊടുക്കുന്നതിൽ നിന്ന് സുരക്ഷിതനായിരിക്കാനാണ് തലേന്ന് ഭാര്യമാരുമായി സംസർഗത്തിൽ ഏർപ്പെടാത്തവർ മൃതദേഹം മറമാടാൻ നിർദ്ദേശിച്ചത്.” (ഫത്ഹുൽ ബാരി: 3:159)

خَشِيَ صَلَّى اللَّهُ عَلَيْهِ وَسَلَّمَ إِنْ نَزَلَ أَنْ يَتَذَكَّرَ شَيْئًا، فَيَذْهَلَ مِنَ الْإِتْيَانِ بِكَمَالِ الْمَنْدُوبَاتِ الَّتِي تُفْعَلُ بِالْمَيِّتِ فِي الْقَبْرِ അലി അൽകാരി പറഞ്ഞു: “തലേന്ന് രാത്രിയിലെ ഓർമകൾ മയ്യത്ത് മറമാടുന്ന വേളയിൽ ഒരാൾക്കുണ്ടാകുന്നതിനെ പ്രവാചകൻ (സ) ഭയപ്പെട്ടു. അത്തരം ഓർമകൾ മനസ്സിൽ നില നിന്നാൽ മൃതദേഹം മറവു ചെയ്യുമ്പോൾ ശ്രദ്ധിക്കേണ്ട മര്യാദകളൊ ഐച്ഛിക പുണ്യകർമങ്ങളൊ മറന്നു പോകാൻ സാധ്യതയുണ്ടല്ലൊ.” (മിർകാത്തുൽ മഫാതീഹ്: 3:1227)

ആധുനിക വിമർശകരുടെ ശവരതി ആരോപണങ്ങളൊന്നും ദുർവ്യാഖ്യാന ഫാക്റ്ററികളുടെ മൂശയിലിട്ട് വാർത്തെടുക്കപ്പെടുന്നതിനും നൂറ്റാണ്ടുകൾക്ക് മുമ്പ് ജീവിച്ച് മരിച്ചു പോയ പൗരാണികരായ ഒട്ടനവധി ഹദീസ് പണ്ഡിതന്മാർ ഈ യാഥാർഥ്യം വ്യക്തമാക്കിയിട്ടുണ്ട്. (തുഹ്ഫത്തുൽ മുഹ്താജ്: 3: 170, നൈലുൽ ഔതാർ: 4: 106, മിർആത്തുൽ മഫാതീഹ്: 5: 451)

2. هَلْ فِيكُمْ مِنْ أَحَدٍ لَمْ يُقَارِفِ اللَّيْلَةَ എന്ന പ്രവാചകന്റെ ചോദ്യത്തിന് “ഇന്നലെ രാത്രി ഭാര്യമാരുമായി വഴക്കു കൂടാത്തവർ ആരുണ്ട്?” എന്നാണ് ഭാഷാർത്ഥം. ‘യുകാരിഫു’ (يُقَارِفِ) എന്ന പദത്തിന് ‘ഭാര്യമാരുമായി സംസർഗത്തിൽ ഏർപ്പെടുക’ എന്നല്ല അർത്ഥം ‘ഭാര്യമാരുമായി വഴക്കിടുക’, ‘തിന്മ വല്ലതും പ്രവർത്തിക്കുക’ എന്നൊക്കെയാണ് അർത്ഥം എന്ന് അറബി ഭാഷാ പണ്ഡിതരായ ഒരുപാട് മുഹദ്ദിസുകൾ പറഞ്ഞിട്ടുണ്ട്. (ശർഹു സുന്ന: ബഗ്‌വി: 1513, ഉംദത്തുൽ കാരി: 8:76, ഹുസ്നുൽ ഉസ്വ: 1:471, മിർആത്തുൽ മഫാതീഹ്: 5: 450, ശർഹുൽ കസ്തല്ലാനി, മുസ്നദു അഹ്മദ്: 13383, അൽ മജ്മൂഅ് ശർഹുൽ മുഹദ്ദബ്: 5:289)

തലേ ദിവസം ഭാര്യമാരുമായി വഴക്കിടുക, അല്ലെങ്കിൽ വല്ല പാപകൃത്യങ്ങളും ചെയ്യുക… ഇതിൽ രണ്ടിൽ ഏതായാലും മനസ്സ് ശാന്തവും നിശ്ചലവുമായിരിക്കില്ല. മനസ്സ് പ്രക്ഷുബ്‌ധവും വികാര തീവ്രവുമായിരിക്കും. അത്തരമൊരു മനസ്സുമായി മൃതദേഹം മറമാടൽ ക്രിയയിൽ ഏർപ്പെട്ടാൽ ചെയ്യേണ്ട പ്രസ്‌തുത കർമ്മത്തിൽ പല പാളിച്ചകൾ സംഭവിച്ചേക്കാം. ഭക്തിയോടെയും മരണചിന്തയോടെയും അത് നിർവഹിക്കാൻ കഴിയാതെ വരാം. അതുകൊണ്ടാണ് തലേ ദിവസം രാത്രി ഭാര്യമാരുമായി വഴക്കിടാത്തവർ അല്ലെങ്കിൽ ഇന്നലെ രാത്രി പാപമൊന്നും ചെയ്തിട്ടില്ലാത്തവർ മൃതദേഹം മറമാടാൻ നിർദേശിച്ചത്.

ﻭَﺣُﻜِﻲَ ﻋَﻦِ اﻟﻄَّﺤَﺎﻭِﻱِّ ﺃَﻧَّﻪُ ﻗَﺎﻝَ ﻟَﻢْ ﻳُﻘَﺎﺭِﻑْ ﺗَﺼْﺤِﻴﻒٌ ﻭَاﻟﺼَّﻮَاﺏُ ﻟَﻢْ ﻳُﻘَﺎﻭِﻝْ ﺃَﻱْ ﻟَﻢْ ﻳُﻨَﺎﺯِﻉْ ﻏَﻴْﺮَﻩُ اﻟْﻜَﻼَﻡَ

ഇമാം ത്വഹാവി പറഞ്ഞു: യുകാരിഫു (يُقَارِفِ) എന്ന പദത്തിന് ‘വഴക്കിടുക, തർക്കിക്കുക’ എന്നൊക്കെയാണ് അർത്ഥം. ഇതാണ് ഹദീസിന്റെ ശരിയായ അർത്ഥം. അതായത് മറ്റുള്ളവരുമായി വഴക്കിനും വക്കാണത്തിനുമൊന്നും പോയിട്ടില്ലാത്തവർ വേണം മൃതദേഹം മറമാടാൻ. (ഫത്ഹുൽ ബാരി: 3: 158)

അപ്പോൾ ഭാര്യമാരുമായി വഴക്കിട്ട മനസ്സുമായി മൃതദേഹ സംസ്കരണത്തിൽ ഏർപ്പെട്ടാലും ഭക്തിയോടെയും മരണചിന്തയോടെയും ചെയ്യേണ്ട പ്രസ്തുത കർമ്മത്തിൽ – തിടുക്കം കൂട്ടുക, അശ്രദ്ധ മൂലം പല മര്യാദകളും മറന്നു പോവുക തുടങ്ങിയ – പാളിച്ചകൾ സംഭവിച്ചേക്കാം. അതുകൊണ്ടാണ് ഭാര്യമാരുമായി വഴക്കിടാത്തവർ മൃതദേഹം മറമാടാൻ പ്രവാചകൻ (സ) നിർദേശിച്ചത് എന്ന് ഒരു കൂട്ടം പണ്ഡിതർ അഭിപ്രായപ്പെടുന്നു.

കർമാനി പറയുന്നു: “മനസ്സ് ശാന്തവും നിശ്ചലവുമായിരിക്കുന്നവരാവണം മൃതദേഹം മറമാടാൻ എന്നത് കൊണ്ട് ആണ് പ്രവാചകൻ (സ) അപ്രകാരം നിർദേശിച്ചത്. (ഉംദത്തുൽ കാരി: 8:76)

എന്ന് വച്ചാൽ ശവരതി ചിന്ത പോയിട്ട്, ഇഹലോക സംബന്ധമായ ചിന്തകളുടെ ഒരു കണിക പോലും മനസ്സിൽ അവശേഷിക്കാത്ത, പാപചിന്തകളിൽ മുക്തമായ ഏകാഗ്ര മനസ്സിനുടമകളാണ് ഖബർ സംസ്ക്കരണ പ്രവർത്തനങ്ങളിൽ ഏർപ്പെടേണ്ടത് എന്നായിരുന്നു ആ മഹാനുഭാവൻ കരഞ്ഞു കൊണ്ട് കൽപ്പിച്ച കൽപനയുടെ ഉദ്ദേശ്യം ! അത് നേർവിപരീത ദിശയുടെ അഗ്രത്തിലെത്തിച്ചു ഈ വെറുപ്പിന്റെ പ്രചാരകർ !!!

ശുദ്ധ അസംബന്ധം. ഏതോ മാനസിക രോഗികളുടെ തെറ്റിദ്ധരിപ്പിക്കൽ മാത്രം. മുഹമ്മദ് നബിയുടെ(സ്വ) ജീവിത വിശുദ്ധിയെ കുറിച്ച് അറിയുന്ന ഒരാളും ആരോപിക്കാത്ത നീച ആരോപണം. പ്രമാണങ്ങളിലെ പരാമർശങ്ങൾ ഹീനമായ താല്പര്യങ്ങൾക്ക് വളച്ചൊടിച്ചുകൊണ്ടുള്ള തരംതാണ പ്രോപഗണ്ട.

നീതിയും സത്യസന്ധതയും ഇല്ലാത്ത ഇസ്‌ലാം വിരോധികൾ തെറ്റിദ്ധരിപ്പിക്കുന്ന പ്രസ്തുത സംഭവത്തിന്റെ യാഥാർഥ്യം ഇങ്ങനെയാണ്: ബാല്യത്തിൽ ഉമ്മ മരണപ്പെട്ട മുഹമ്മദ് നബിയുടെ(സ്വ) വളർത്തുമ്മയാണ് ഫാത്തിമ ബിൻത് അസദ്. സ്വന്തം അമ്മയേക്കാൾ ഏറെക്കാലം നബിയെ പരിചരിച്ച നബിയുടെ സ്നേഹവതിയായ മാതാവ്. നബിയുടെ(സ്വ) വത്സല പിതൃവ്യൻ അബൂ ത്വാലിബിന്റെ ഭാര്യ. നബിയുടെ(സ്വ) വളർത്തുമകനും സന്തത സഹചാരിയും പിന്നീട് മരുമകനുമായ അലിയുടെ ഉമ്മ. നബിയുടെ(സ്വ) ധീര അനുയായികളായിരുന്ന ജഅഫറിന്റെയും അഖീലിന്റെയും ജുമാന(1)യുടെയും ഉമ്മ. ഭർത്താവ് ഇസ്‌ലാമിലേക്ക് വരാൻ മടിച്ചപ്പോഴും, നബിയുടെ അനുയായി ആയി പ്രവർത്തിച്ച ധീര വനിത. ഭർത്താവിന്റെ വിയോഗത്തിന് ശേഷം മക്കളോടൊപ്പം ഇസ്‌ലാമിനെയും നബിയെയും പ്രതിരോധിക്കാൻ മുന്നണിയിൽ നിരന്ന ആദർശശാലിനി. ഒരു ഹാശിമിക്ക് പിറന്ന ചരിത്രത്തിലെ ആദ്യത്തെ ഹാശിമി വനിതയെന്ന കുടുംബ അന്തസ്സിന്റെ ഉടമയും, ഒരു ഖലീഫയുടെ മാതാവായ ആദ്യത്തെ ഹാശിമി വനിതയെന്ന വ്യക്തിഗത നേട്ടത്തിന്റെ ഉടമയുമായ മഹതിയെകുറിച്ച് പ്രമുഖ ചരിത്രകാരൻമാരായ ഇബ്‌നു സഅദ് ‘ത്വബഖാത്തുൽ കുബ്റാ’യിലും, ഇബ്‌നുൽ അസീർ ‘ഉസുദുൽ ഗ്വാബ’യിലും (7168), ഇബ്‌നു അബ്ദിൽ ബർറ് ‘അൽ ഇസ്തീആബി’ലും, ഹാഫിള് ഇബ്‌നു ഹജർ അൽ അസ്ഖലാനി ‘അൽ ഇസ്വാബ’യിലും പരിചയപ്പെടുത്തുന്നു. പ്രമുഖരായ മൂന്നു മുഹദ്ദിസുകൾ മഹതിയിലൂടെ കടന്നുവന്ന ധാരാളം ഹദീസുകൾ ഉദ്ധരിച്ചിട്ടുണ്ട്.

തന്റെ മകൻ അലിയെ(റ) വീട്ടിലാക്കി നബി (സ്വ) മദീനയിലേക്ക് പലായനം ചെയ്തപ്പോൾ, മറ്റു വിശ്വാസികളോടൊപ്പം മദീനയിലേക്ക് ഹിജ്‌റ ചെയ്ത മുഹാജിറ. അലി വൈകാതെ മദീനയിലെത്തി. ജഅഫർ മറ്റു വിശ്വാസികളോടൊപ്പം ആഫ്രിക്കയിലെ എത്യോപ്പ്യയിലേക്ക് ഹിജ്‌റ പോയതാണ്). മദീനയിൽ കുറച്ചുകാലം ജീവിച്ച മഹതിയുടെ വിയോഗം മദീനയിൽ തന്നെയായിരുന്നുവെന്ന് ഇബ്‌നുൽ അസീർ, ഇബ്‌നു അബ്ദിൽ ബർറ്, ഇബ്‌നു അസാകിർ (താരീഖ് ദിമിശ്ഖ്), ദഹബി (താരീഖുൽ ഇസ്‌ലാം) മുതലായ എല്ലാ ജീവചരിത്രകാരന്മാരും രേഖപ്പെടുത്തുന്നു. ഹിജ്‌റ രണ്ടാം വർഷം ബദ്ർ പോരാട്ടത്തിന് ശേഷം, മകൻ അലി നബിയുടെ പുത്രി ഫാഥ്വിമയെ വിവാഹം ചെയ്ത ശേഷക്കാലവും മഹതി ജീവിച്ചിരുന്നിട്ടുണ്ട്. ഫാഥ്വിമയുടെ വീട്ടുജോലിഭാരങ്ങൾ സംബന്ധമായി അലി ഉമ്മയോട് സങ്കടപ്പെടുന്ന രംഗം ചരിത്രത്തിൽ രേഖപ്പെട്ടിട്ടുണ്ട്. അലിയുടെയും ഫാഥ്വിമയുടെയും കൂടെ അവരുടെ വീട്ടിൽ മഹതിയും ഉണ്ടായിരുന്നുവെന്നാണ് വിവിധ പരാമർശങ്ങൾ വ്യക്തമാക്കുന്നത്. ഹിജ്‌റ മൂന്നാം വർഷം ശവ്വാലിൽ നടന്ന ഉഹ്ദ് പോരാട്ടത്തിലായിരുന്നുവല്ലോ, നബിയുടെ പിതൃവ്യൻ ഹംസ വധിക്കപ്പെട്ടത്. ഹംസയുടെ മകൾ ഫാഥ്വിമയുടെ രക്ഷാധികാരം നബിയിലെത്തി. നബി (സ്വ) അവളെ സലമത്ത് ബ്‌നു അബീ സലമത്തിന് പിന്നീട് വിവാഹം ചെയ്തു കൊടുത്തു. നബിയ്ക്ക്(സ്വ) ഈയ്യിടെ കുറച്ചു കസവുപട്ടുതുണി സമ്മാനമായി ലഭിച്ചിരുന്നു. അത് ‘ഫാഥ്വിമമാർക്ക് മുഖമക്കനയാക്കി നൽകു’വാൻ നബി (സ്വ) നിർദ്ദേശിച്ചു. അതുപ്രകാരം, ആ തുണി ഉപയോഗിച്ച് നബിയുടെ പുത്രി ഫാഥ്വിമ, ചർച്ച ചെയ്യപ്പെടുന്ന അലിയുടെ ഉമ്മ ഫാഥ്വിമ, ഹംസയുടെ മകൾ ഫാഥ്വിമ, അലിയുടെ സഹോദരൻ അഖീലിന്റെ പത്നി ഫാഥ്വിമ എന്നിവർക്ക് മക്കനയുണ്ടാക്കി നൽകി. ഇക്കാര്യം പ്രമുഖ ഹദീസ് സമാഹാരമായ ഇബ്‌നു മാജയിൽ (ഹദീസ് നമ്പർ 3596) രേഖപ്പെടുത്തിയതിനു പുറമെ, ഹാഫിള് അസ്ഖലാനിയെ പോലുള്ള ചരിത്രകാരന്മാരും സ്ഥിരീകരിച്ചിട്ടുണ്ട്. ഹിജ്‌റ മൂന്നാം വർഷാവസാനം വരെയെങ്കിലും ഫാഥ്വിമ ബിൻത് അസദ് മദീനയിൽ ജീവിച്ചിരുന്നിട്ടുണ്ടെന്ന്‌ ഈ സംഭവം തെളിയിക്കുന്നു. പ്രവാചകൻ (സ്വ) മഹതിയുടെ ഭവനം സന്ദർശിക്കാറുണ്ടായിരുന്നുവെന്നും അവിടെ ‘ഉച്ചയുറക്കം’ (ഖൈലൂലത്ത്) നടത്താറുണ്ടായിരുന്നുവെന്നും ഇബ്‌നു സഅദ് രേഖപ്പെടുത്തുന്നു. وَكَانَ رَسُولُ اللَّهِ – صَلَّى اللَّهُ عَلَيْهِ وَسَلَّمَ – يزورها ويقيل في بيتها.

നബിയ്ക്ക് പ്രിയപ്പെട്ട മാതാവായിരുന്നു ഫാഥ്വിമ ബിൻത് അസദ്. അബൂ ത്വാലിബിന് ശേഷം മഹതിയെക്കാൾ തന്നോട് രക്തബന്ധപരിഗണന കാണിച്ച മറ്റാരേയും ഞാൻ കണ്ടിട്ടില്ല’ (لم نلق بعد أبي طالب أبرّ بي منها) എന്ന് നബി (സ്വ) പറയുകയുണ്ടായി. മഹതി മരണപ്പെട്ടപ്പോൾ, വീട്ടിലെത്തി, മഹതിയുടെ തലയുടെ ഭാഗത്തിരുന്ന് ഇങ്ങനെ അനുശോചിച്ചു: “എന്റെ പ്രിയ മാതാവേ, അങ്ങയെ അല്ലാഹു അനുഗ്രഹിക്കട്ടെ. എന്റെ പെറ്റുമ്മയുടെ വിയോഗാനന്തരം അങ്ങായിരുന്നു എന്റെ ഉമ്മ. വിശപ്പ് സഹിച്ചും ഉള്ളതെടുത്ത് അങ്ങ് എന്റെ വയറു നിറയെ തീറ്റി. ഉടുക്കാൻ ആവശ്യമുണ്ടായിട്ടും കിട്ടുന്ന വസ്ത്രങ്ങൾ അങ്ങെന്നെ അണിയിച്ചു. സ്വന്തം ഇഷ്ടങ്ങൾ വെടിഞ്ഞും അങ്ങെന്നെ രുചിയുള്ളത് ഭക്ഷിപ്പിച്ചു. അല്ലാഹുവിന്റെ പ്രീതിയും പരലോക മോക്ഷവും മാത്രമായിരുന്നു, ഇതുകൊണ്ടെല്ലാം അങ്ങ് ലക്ഷ്യം വെച്ചത്”. رَحِمَكِ اللَّهُ يَا أُمِّي، كُنْتِ أُمِّي بَعْدَ أُمِّي، تَجُوعِينَ وَتُشْبِعِينِي، وَتَعْرَيْنَ وَتَكْسِينِي، وَتَمْنَعِينَ نَفْسَكِ طَيِّبًا وَتُطْعِمِينِي، تُرِيدِينَ بِذَلِكَ وَجْهَ اللَّهِ وَالدَّارَ الْآخِرَةَ “. പിന്നെ, മഹതിയെ മൂന്നുവട്ടം ദേഹമാസകലം വെള്ളമൊഴിച്ചു കുളിപ്പിക്കാൻ നിർദ്ദേശിച്ചു. കർപ്പൂരം ഒഴിച്ചിട്ടുള്ള വെള്ളം മൃത ശരീരത്തിൽ ഒഴിക്കാൻ സമയമായപ്പോൾ, നബി (സ്വ) തന്നെ അതുവാങ്ങി, മഹതിയുടെ ശരീരത്തിൽ ഒഴിച്ചു. കുളിപ്പിച്ച് കഴിഞ്ഞപ്പോൾ, നബി അണിഞ്ഞിരുന്ന നീളക്കുപ്പായം (ഖമീസ്) അഴിച്ചു മഹതിയെ ഉടുപ്പിച്ചു. അതിനുമുകളിൽ രോമത്തിന്റെ കറുത്ത തുണികൊണ്ട് പൊതിഞ്ഞു. ശേഷം, ഉസാമത്ത് ബ്‌നു സൈദ്, അബൂ അയ്യൂബിൽ അൻസ്വാരി, ഉമർ ബ്‌നുൽ ഖത്വാബ്, ഒരു കറുത്ത സേവകൻ എന്നിവരെ വിളിച്ചു ഖബർ കുഴിക്കാൻ ആവശ്യപ്പെട്ടു. അവർ കുഴിക്കാൻ തുടങ്ങി. കുഴിയായപ്പോൾ, നബി (സ്വ) തന്നെ കുഴിയിലിറങ്ങി, തന്റെ കൈകൾ കൊണ്ട് കുഴിയുടെ ഒരുവശത്തുനിന്നും(2) മണ്ണ് മാന്തിയെടുക്കാൻ തുടങ്ങി. ആവശ്യത്തിന് മണ്ണെടുത്ത ശേഷം, നബി (സ്വ) അതിൽ ചെരിഞ്ഞു കിടന്നു (മയ്യിത്ത് വെക്കാറുള്ള പോലെ). എന്നിട്ടിങ്ങനെ പ്രാർത്ഥിച്ചു: “ജീവിച്ചിരിക്കുന്നവനും മരിക്കാത്തവനുമായ അല്ലാഹുവേ, എന്റെ പ്രിയമാതാവ് ഫാഥ്വിമ ബിൻത് അസദിന് നീ പൊറുത്തുകൊടുക്കണേ, അവർക്ക് (ഖബറിലെ ചോദ്യസമയത്ത്) സത്യപ്രമാണം നൽകി അനുഗ്രഹിക്കണേ. നിന്റെ ഈ പ്രവാചകന്റെയും എനിക്കുമുമ്പുള്ള പ്രവാചകന്മാരുടെയും മഹത്വത്തെ പരിഗണിച്ച് അവരുടെ ഖബർ നീ വിശാലമാക്കണേ, നിശ്ചയം നീ മഹാ കാരുണ്യവാനല്ലോ’. اللَّهُ الَّذِي يُحْيِي وَيُمِيتُ، وَهُوَ حَيٌّ لَا يَمُوتُ، اغْفِرْ لِأُمِّي فَاطِمَةَ بِنْتِ أَسَدٍ، وَلَقِّنْهَا حُجَّتَهَا، وَوَسِّعْ عَلَيْهَا مُدْخَلَهَا بِحَقِّ نَبِيِّكَ وَالْأَنْبِيَاءِ الَّذِينَ مِنْ قَبْلِي ; فَإِنَّكَ أَرْحَمُ الرَّاحِمِينَ” ശേഷം, നാല് തക്ബീറുകൾ ചൊല്ലി മയ്യിത്ത് നിസ്കരിച്ചു. നബിയും അബൂബക്കർ സ്വിദ്ധീഖും നബിയുടെ മൂത്ത പിതൃവ്യൻ അബ്ബാസും ചേർന്ന് മഹതിയെ ലഹ്‌ദിലേക്ക് ചെരിച്ചുകടത്തിവെച്ചു. ഹദീസ് സമാഹർത്താക്കളായ ത്വബ്റാനി തന്റെ കബീറിലും ഔസത്തിലും ഈ സംഭവം ഇതുപോലെ ഉദ്ധരിച്ചിരിക്കുന്നു(3). പതിവില്ലാത്ത ഒരു പ്രവൃത്തി നബിയിൽ നിന്നും കാണാനിടയായ അനുയായികൾ, മഹതിയെ അടക്കം ചെയ്ത ശേഷം നബിയോട് അന്വേഷിച്ചു: “അല്ലാഹുവിന്റെ ദൂതരേ, മറ്റാരുടെയും ഖബറടക്ക സമയത്ത് ചെയ്യാത്ത ചില കാര്യങ്ങൾ അങ്ങ് ഇവിടെ ചെയ്തല്ലോ?!”. അപ്പോൾ നബി പ്രതിവചിച്ചു: “എന്റെ ഖമീസ് ഞാൻ അവർക്ക് അണിഞ്ഞുകൊടുത്തത്, അവർക്ക് സ്വർഗ്ഗത്തിലെ പുടവകൾ അണിയക്കപ്പെടുവാനായിട്ടാകുന്നു. അവർ കിടക്കുന്ന ഖബ്‌റിൽ ഞാൻ കയറിക്കിടന്നത് അവർക്ക് ഖബറിന്റെ ഇടുക്കം ലഘൂകരിക്കപ്പെടാൻ വേണ്ടിയാകുന്നു. കാരണം, അബൂ ത്വാലിബ് മരണപ്പെട്ട ശേഷം ഏറ്റവും മെച്ചപ്പെട്ട രീതിയിൽ എന്നെ സംരക്ഷിച്ച സ്ത്രീയാണവർ.” (ത്വബ്റാനി)

أَلْبَسْتُهَا قَمِيصِي ; لِتَلْبَسَ مِنْ ثِيَابِ الْجَنَّةِ، وَاضْطَّجَعْتُ مَعَهَا فِي قَبْرِهَا ; خُفِّفَ عَنْهَا مِنْ ضَغْطَةِ الْقَبْرِ؛ إِنَّهَا كَانَتْ مِنْ أَحْسَنِ خَلْقِ اللَّهِ إِلَيَّ صَنِيعًا بَعْدَ أَبِي طَالِبٍ. അനേകം ചരിത്രകാരന്മാരും ഈ സംഭവം ഉദ്ധരിച്ചിട്ടുണ്ട്. തന്റെ വളർത്തുമ്മയായ അമ്മായിയോട് നബി (സ്വ) കാണിച്ച മാതൃകാപരമായ സ്നേഹകടപ്പാടുകൾ എത്ര മഹോന്നതമായിരുന്നു എന്ന് ഈ സംഭവം കാണിക്കുന്നു.

എന്നാൽ, ഇസ്‌ലാമിനെയും നബിയെയും എങ്ങനെയെങ്കിലും അവഹേളിക്കാൻ ആഗ്രഹിക്കുന്ന മലിനമനസ്‌കർ ഈ സംഭവത്തെ ശവഭോഗത്തിലേക്ക് തിരിച്ചുവിട്ടത് കാണുമ്പോൾ, വിരോധം മനുഷ്യരെ എത്ര അധമരാക്കിത്തീർക്കുന്നു എന്ന് മനസ്സിലാകും. ഒരു സംഭവം ഹദീസുകളിൽ രണ്ടുവിധത്തിൽ ഉദ്ധരിക്കാറുണ്ട്. സാമാന്യം വിശദമായും, വളരെ ചുരുക്കിയും. രണ്ടും രണ്ടു സന്ദർഭങ്ങളിൽ പ്രസക്തവുമാണ്. എന്നാൽ, ചുരുക്ക വിവരണത്തിൽ പറയുന്ന കാര്യങ്ങളെ തെറ്റിദ്ധരിപ്പിച്ചു, യഥാർത്ഥ വശം മറച്ചുവെക്കുകയായിരുന്നു ആരോപകർ. ഈ സംഭവത്തെ നരാധമർ അവതരിപ്പിച്ച രീതി ഇങ്ങനെയായിരുന്നു: മദീനയിൽ ഒരു സ്ത്രീ മരണപ്പെടുന്നു. അവരുടെ കുഴിമാടത്തിൽ പ്രവാചകൻ ഇറങ്ങുന്നു. തന്റെ വസ്ത്രം അഴിച്ചു ആ ശവത്തോടൊപ്പം കിടക്കുന്നു?! നോക്കൂ, ആ സ്ത്രീ നബിയുടെ അമ്മായിയും മാതാവുമാണെന്ന കാര്യം മറച്ചുവെച്ചു. മയ്യിത്തിനെ പുടവ അണിയിക്കുന്ന സമയത്ത് അവിടെവെച്ചാണ് തന്റെ ഖമീസ് അഴിച്ചു അതിൽ ഉമ്മയെ പൊതിഞ്ഞതെന്ന കാര്യം ഒളിച്ചുവെച്ചു. മദീനയിൽ തന്റെ അനുയായികൾ ചേർന്ന് ഖബർ കുഴിക്കുന്നതിനിടയിൽ, അവർക്ക് മുന്നിൽ വെച്ചായിരുന്നു നബി, മയ്യിത്ത് കിടത്തേണ്ട മടയിൽ ചെരിഞ്ഞുകിടന്നതെന്നും അതിനു ശേഷമായിരുന്നു മയ്യിത്ത് മറ്റുള്ളവരുടെ സഹായത്തോടെ അതേസ്ഥാനത്തേക്ക് കടത്തിവെക്കുന്നതെന്നുമുള്ള സംഗതി പുറത്തുപറഞ്ഞില്ല. മയ്യിത്ത് കിടത്തുന്ന ഇത്തരം ഖബറിലെ മടയിൽ രണ്ടാമതൊരാൾക്ക് കിടക്കാൻ കഴിയില്ലെന്ന നാട്ടനുഭവം പൂഴ്ത്തിവെച്ചു. എന്തിനാണിതെല്ലാം?! മുഹമ്മദ് ശവത്തെ ഭോഗിച്ചുവെന്നു വരുത്താൻ!! അസൂയയും വിദ്വേഷവും മനുഷ്യരെ കൊണ്ടെത്തിക്കുന്ന അനാവസ്ഥ എത്ര ഭീകരമാണ്!!

സംഭവം വളച്ചൊടിക്കാൻ ദുഷ്ടമനസ്കർ നിവേദനത്തിൽ വന്ന وَاضْطَّجَعْتُ مَعَهَا فِي قَبْرِهَا എന്ന പദത്തിന്, സംഭവത്തിന്റെ സന്ദർഭവും സാഹചര്യവും മറച്ചുവെച്ചുകൊണ്ട്, തെറ്റായി അർത്ഥം കല്പിക്കുകയായിരുന്നു. ‘ഇള്തജഅ’ =’ഭോഗിച്ചു’ എന്ന അർത്ഥത്തിൽ അവർ ചാടിപ്പിടിക്കുകയായിരുന്നു. പദപ്രയോഗത്തെ തെറ്റിദ്ധരിപ്പിച്ചുകൊണ്ട്, താന്തോന്നികൾക്ക് ശവഭോഗത്തെ അനുക്കൂലമാക്കിയെടുക്കാനും സംസ്കാരസമ്പന്നർക്ക് മുന്നിൽ പ്രവാചകനെ അവഹേളിക്കാനും നടത്തുന്ന സാഹസം പക്ഷേ, പഠിതാക്കൾക്ക് നബിയിലെ മനുഷ്യനെ മനസ്സിലാക്കാൻ കൂടുതൽ അവസരമുണ്ടാക്കി എന്നതാണ് ഗുണഫലം.

കുറിപ്പുകൾ

(1) ഹിജ്‌റ ഏഴാം വർഷം മുഹറമിലെ ഖൈബർ പോരാട്ട ശേഷം, മഹതിയുടെ മകൾ ജുമാനയ്ക്ക് നബി (സ്വ) മുപ്പത് വസ്ഖ് (ഏതാണ്ട് നൂറ്റി അമ്പത് കിലോ ഭാരം വരുന്ന ഒട്ടകച്ചുമടാണ്‌ വസ്ഖ് (30 * 150 = 4500 kg) വസ്തുക്കൾ സമ്മാനമായി കൊടുത്തയച്ച സംഭവം ചരിത്രത്തിലുണ്ട്. (അർറൗളത്തുൽ ഫൈഹാ/ യാസീനിൽ ഉമരി/മ. ഹി. 1232 ) (2)രണ്ടുതരം ഖബ്ർ ഉണ്ട്. നേരെ താഴേക്ക് കുഴിക്കുന്ന സാധാരണ രൂപം. താഴേക്ക് കുഴിച്ച ശേഷം, ഒരാളെ ചെരിച്ച് കിടത്താവുന്നവിധം ഖിബ്‌ലയുടെ ഭാഗത്തുനിന്നും മണ്ണെടുക്കുന്ന രണ്ടാം രൂപം. മുകളിൽ നിന്നും നോക്കിയാൽ മയ്യിത്ത് ഒറ്റനോട്ടത്തിൽ കാണില്ല. മുകൾ ഭാഗം അടയ്ക്കുമ്പോൾ മയ്യിത്തിനു മേൽ മണ്ണോ മറ്റോ വീഴാതെ സംരക്ഷിക്കുകയാണ് ഒരു ലക്ഷ്യം. ഉറച്ച മണ്ണുള്ളിടത്താണ് ഇത് പ്രായോഗികമാവുക. ലഹ്ദ് എന്നാണിതിന് പേര്. (3) ഹദീസ് നിവേദകരിൽ റൗഹ്ബ്നു സ്വലാഹ് എന്ന വ്യക്തി ദുർബ്ബലനാണെന്ന അഭിപ്രായം നൂറുദ്ധീൻ അൽഹൈസമി (ഹി 807) പ്രകടിപ്പിക്കുന്നുവെങ്കിലും, ആദ്യകാല നിരൂപകനായ ഇബ്നു ഹിബ്ബാൻ, ഹാകിം എന്നിവർ അദ്ദേഹം വിശ്വസ്തനാണെന്ന് സമ്മതിച്ചകാര്യം ശ്രദ്ധേയമാണ്. ഒരു മതവിധി പറയാനുള്ള ബലം ഇല്ലെന്നു വന്നാലും, ഒരു സംഭവം സ്ഥിരീകരിക്കാൻ ഈ നിവേദനം ധാരാളം മതി.

വിമർശനം:

മുഹമ്മദ് നബിയുടെ കുടുംബ പരമ്പര കിന്ദക്കാരിലേക്കാണ് എത്തുന്നത് എന്ന് കിന്ദക്കാർ വാദിച്ചത് മുഹമ്മദ് നബിയുടെ പിതൃത്വത്തെ ചോദ്യം ചെയ്യുന്ന തെളിവല്ലെ ?

മറുപടി:

ഹദീസ് ദുർബലമാണെന്നതിനു പുറമെ ഹദീസിന്റെ ആശയത്തെ വിമർശകർ ദുർവ്യാഖ്യാനിക്കുക കൂടി ചെയ്തിട്ടുണ്ട്.

കാരണങ്ങൾ:

1. ഹദീസിന്റെ നിവേദക പരമ്പര ഇപ്രകാരമാണ്:

നമ്മോട് അബുൽ ഹസൻ അലിയ്യിബ്‌നു അഹ്‌മദ് ഇബ്‌നു ഉമറിബ്‌നു ഹഫ്സ് നമ്മെ അറിയിച്ചു – അബൂ ഈസാ ബക്കാറിബ്‌നു അഹ്‌മദിബ്‌നു ബക്കാർ നമ്മോട് പറഞ്ഞു – അബൂ ജഅ്ഫറിബ്‌നു മൂസാ ഇബ്‌നു സഅ്ദ് നമ്മോട് പറഞ്ഞു – അബൂജഅ്ഫർ മുഹമ്മദിബ്‌നു അബ്ബാൻ അൽ കലാനസി നമ്മോട് പറഞ്ഞു – നമ്മോട് അബൂ മുഹമ്മദ് അബ്ദുല്ലാഹിബ്‌നു മുഹമ്മദിബ്‌നു റബീഅ അൽകുദാമി പറഞ്ഞു – മാലികിബ്‌നു അനസിൽ നിന്ന് – അദ്ദേഹം സുഹ്‌രിയിൽ നിന്ന് – അദ്ദേഹം അനസിബ്‌നു മാലികിൽ നിന്നും – അദ്ദേഹം അബൂ ബക്കറിബ്‌നു അബ്ദുർറഹ്‌മാനിൽ നിന്നും, അദ്ദേഹം പറഞ്ഞു:…

നിവേദക പരമ്പരയിലെ ‘അബൂ മുഹമ്മദ് അബ്ദുല്ലാഹിബ്‌നു മുഹമ്മദിബ്‌നു റബീഅ അൽകുദാമി’ അങ്ങേയറ്റം ദുർബലനാണ്.

ഇബ്‌നുകസീർ പറഞ്ഞു: കുദാമി തനിച്ച് നിവേദനം ചെയ്യുന്നതാണ് ഈ ഹദീസ്. അയാൾ ദുർബലനാണ്. ഇമാം ദഹബി പറഞ്ഞു: അയാൾ ദുർബലനാണ്. അയാൾ ഇമാം മാലികിൽ നിന്ന് വ്യാജ നിവേദനങ്ങൾ ഉദ്ധരിക്കുമായിരുന്നു. (മീസാനുൽ ഇഅ്തിദാൽ) ഹാകിം പറഞ്ഞു: ഇമാം മാലികിൽ നിന്ന് അയാൾ കള്ള ഹദീസുകൾ ഉദ്ധരിക്കുമായിരുന്നു. കുടുംബ പരമ്പരകളെ സംബന്ധിച്ച ഗ്രന്ഥമായ ‘അൽ അൻസാബ്’ ൽ ഇമാം സംആനി കുദാമിയെ സംബന്ധിച്ച് ഇപ്രകാരമെഴുതി: അദ്ദേഹം ചരിത്ര നിവേദനങ്ങളിൽ കൃത്രിമങ്ങൾ കാണിക്കുമായിരുന്നു എന്നതുകൊണ്ട് തന്നെ അയാളുടെ നിവേദനങ്ങൾ പ്രമാണമായി സ്വീകരിക്കപ്പെടില്ല. (ലിസാനുൽ മീസാൻ: ഇബ്‌നു ഹജർ: 3/335, അൽ മജ്‌റൂഹീൻ: 2/39)

കൂടാതെ, ഇബ്‌നുഹിബ്ബാൻ, ഇബ്‌നു അദിയ്യ്, ദാറകുത്നി എന്നിവരും അയാൾ ദുർബലനാണെന്ന് കട്ടായം പറയുന്നു.

ഇബ്‌നു സഅ്ദ് – മഅ്നിബ്‌നു ഈസായിൽ നിന്നും – അദ്ദേഹം ഇബ്‌നു അബി ദിഅ്ബിൽ നിന്നും – അദ്ദേഹം പിതാവിൽ നിന്നും ഉദ്ധരിക്കുന്ന ഒരു നിവേദനം ഇതേ സംഭവത്തെ സംബന്ധിച്ച് ഉദ്ധരിക്കപ്പെട്ടിട്ടുണ്ടെങ്കിലും (ത്വബകാത്തു ഇബ്‌നു സഅ്ദ്: 1:23) ആ നിവേദനവും ദുർബലമാണ്; നിവേദക പരമ്പര പ്രവാചകനിലേക്ക് എത്തുന്നത് (മുസ്നദ് المسند) അല്ല. വേറേയും നിവേദക പരമ്പരകളിലൂടെ ഇതേ സംഭവം ഉദ്ധരിക്കപ്പെട്ടിട്ടുണ്ടെങ്കിലും എല്ലാം അങ്ങേയറ്റം ദുർബലങ്ങളാണ്.

2. ഹദീസ് സ്വീകാര്യയോഗ്യമാണെന്ന് വാദത്തിന് അംഗീകരിച്ചാൽ തന്നെ പ്രവാചകന്റെ(സ) പിതൃത്വവുമായി യാതൊരു ബന്ധവുമില്ലാത്ത ഒരു നിവേദനത്തെ, പ്രവാചകൻ (സ) പിതൃശൂന്യനാണ് എന്ന് വരുത്തി തീർക്കാനായി നിവേദനത്തിന്റെ ആശയത്തെ വിമർശകർ അങ്ങേയറ്റം പണിപ്പെട്ട് ദുർവ്യാഖ്യാനിച്ചിരിക്കുകയാണ്.

നിവേദനത്തിന്റെ ആശയ സംഗ്രഹം:

ആകിലുൽ മുറാർ കിന്ദക്കാരുടെ രാജാവും നേതാവുമായിരുന്നു. ഇസ്‌ലാമിക കാലഘട്ടത്തിന് മുമ്പ്, ജാഹിലി കാലഘട്ടത്തിൽ കച്ചവടക്കാരായ അറബികൾക്ക് -വിശിഷ്യ കുറൈശികൾക്ക് – പല നാടുകളിലും കച്ചവടാർത്ഥം സഞ്ചരിക്കുകയും താമസിക്കുകയും ചെയ്യേണ്ടിവരുമായിരുന്നു. അബ്ബാസിബ്‌നു അബ്ദുൽ മുത്വലിബ്, റബീഅത്തിബ്‌നു ഹാരിസ്, അബൂസുഫ്‌യാൻ തുടങ്ങിയ കുറൈശി നേതാക്കൾ കച്ചവടാർത്ഥം അന്യ നാടുകളിൽ ചെന്ന് താമസിക്കുമ്പോൾ, ആ നാട്ടുകാർ “നിങ്ങൾ ആരാണ്” എന്ന് ചോദിച്ചാൽ ” ഞങ്ങൾ ആകിലുൽ മുറാർ സന്തതികളാണ്” എന്ന് പ്രതികരിക്കുമായിരുന്നു. അന്യനാട്ടിൽ സ്ഥാനവും സുരക്ഷയും ലഭിക്കാൻ വേണ്ടിയും കിന്ദക്കാരെ കൊണ്ട് അഭിമാനിച്ചു കൊണ്ടുമായിരുന്നു അവർ ഇപ്രകാരം പറഞ്ഞിരുന്നത്. അല്ലാതെ കുടുംബ ബന്ധം ഉള്ളതു കൊണ്ടായിരുന്നില്ല. പക്ഷെ കുറൈശി നേതാക്കളുടെ ഈ സംസാരം കിന്ദക്കാരുടെ അടുത്തെത്തിയപ്പോൾ കുറൈശികൾക്ക് തങ്ങളുമായി കുടുംബ ബന്ധമുണ്ടെന്ന് കിന്ദക്കാർ തെറ്റിദ്ധരിച്ചു. കുറൈശികൾക്ക് തങ്ങളുമായി കുടുംബ ബന്ധമുണ്ടെന്നതിൽ അവർ അഭിമാനം കൊണ്ടു. പിന്നീട് കുറൈശികളിൽ നിന്നും പ്രവാചകൻ (സ) നിയോഗിതനായപ്പോൾ കിന്ദക്കാരിൽ ഈ തെറ്റിദ്ധാരണ വെച്ചുപുലർത്തുന്ന ചിലർ കൂടുതൽ അഭിമാനിച്ചു. അവർ പ്രവാചകനോട്(സ) പറഞ്ഞു: “ഞങ്ങൾ ആകിലുൽ മുറാർ സന്തതികളാണ്. നിങ്ങളും ആകിലുൽ മുറാർ സന്തതികളല്ലെ ?” അപ്പോൾ പ്രവാചകൻ (സ) അവരുടെ തെറ്റിദ്ധാരണ തിരുത്തി. അബ്ബാസിബ്‌നു അബ്ദുൽ മുത്വലിബ്, റബീഅത്തിബ്‌നു ഹാരിസ്, അബൂസുഫ്‌യാൻ തുടങ്ങിയ കുറൈശി നേതാക്കൾ കച്ചവടാർത്ഥം അന്യ നാടുകളിൽ ചെന്ന് താമസിക്കുമ്പോൾ,”ഞങ്ങൾ ആകിലുൽ മുറാർ സന്തതികളാണ്” എന്ന് പറഞ്ഞിരുന്നത്, അന്യനാട്ടിൽ സ്ഥാനവും സുരക്ഷയും ലഭിക്കാൻ വേണ്ടിയും കിന്ദക്കാരെ കൊണ്ട് അഭിമാനം പ്രകടിപ്പിച്ചു കൊണ്ടുമായിരുന്നു. അല്ലാതെ കുടുംബ ബന്ധം ഉള്ളതു കൊണ്ടായിരുന്നില്ല. കുടുംബബന്ധത്തിലാണെങ്കിൽ “ഞങ്ങൾ നദ്ർ ഇബ്‌നു കിനാനയുടെ മക്കളാണ്” എന്ന് പ്രവാചകൻ (സ) അവരെ പഠിപ്പിക്കുകയും ചെയ്തു. (ത്വബകാത്തുൽ കുബ്റാ: ഇബ്‌നു സഅ്ദ്: 1/708, അൽ ബിദായ വന്നിഹായ: 5/85, സാദുൽ മആദ്: 3/676)

ഇവിടെ കിന്ദക്കാർ സംസാരിക്കുന്നത് പ്രവാചകന്റെ(സ) പിതൃത്വത്തെ സംബന്ധിച്ചേയല്ല എന്ന് നിവേദനം ഒരു വട്ടം വായിച്ചാൽ തന്നെ വ്യക്തം. മറിച്ച് കുറൈശി ഗോത്രത്തെ സംബന്ധിച്ചാണ്. മുസ്‌ലിംകളും അമുസ്‌ലിംകളുമടങ്ങുന്ന കുറൈശി ഗോത്രത്തെ സംബന്ധിച്ച്. പ്രവാചകൻ അവരോട് പറഞ്ഞ മറുപടിയും ശ്രദ്ധിക്കുക: نحن بنو النضر بن كنانة

“ഞങ്ങൾ നദ്ർ ഇബ്‌നു കിനാനയുടെ മക്കളാണ്”. നദ്ർ ഇബ്‌നു കിനാനയുടെ മറ്റൊരു പേരാണ് കുറൈശ്. (ലിസാനുൽ അറബ്: 6: 335, അൽ ബിദായ വന്നിഹായ: 3: 219, താരീഖുൽ അറബിൽ കദീം: 1: 172) അപ്പോൾ ഞങ്ങൾ കുറൈശികൾ നദ്ർ ഇബ്‌നു കിനാനയുടെ സന്തതികളാണ്, ആകിലുൽ മുറാറിന്റേതല്ല എന്നാണ് പ്രവാചകന്റെ മറുപടി. സ്വന്തം പിതൃത്വത്തെയാണ് പ്രവാചകൻ ഈ നിവേദനത്തിൽ പ്രതിരോധിക്കുന്നത് എങ്കിൽ “ഞാൻ അബ്ദുല്ലയുടെ മകനാണ്” എന്നല്ലെ പറയേണ്ടിയിരുന്നത്.?!

മാത്രമല്ല കിന്ദക്കാരിൽ ചിലർക്ക് മാത്രമായിരുന്നു ഈ തെറ്റിദ്ധാരണ ഉണ്ടായിരുന്നത്. അവരിലെ ഭൂരിഭാഗത്തിനും കുറൈശികൾ വിശിഷ്യാ ഹാശിം കുടുംബം കിന്ദക്കാരിൽ പെട്ടവരല്ല എന്ന് നന്നായി അറിയാമായിരുന്നു. അതുകൊണ്ടാണ് പ്രവാചകൻ (സ) കിന്ദക്കാരോട് ഏക ദൈവ വിശ്വാസം പ്രബോധനം ചെയ്തപ്പോൾ അവർ ഇപ്രകാരം പ്രതികരിച്ചത്: “താങ്കൾ താങ്കളുടെ ഗോത്രത്തിലേക്ക് മടങ്ങി പോവുക. താങ്കളെ ഞങ്ങൾക്ക് യാതൊരു ആവശ്യവുമില്ല…” (ഇംതാഉൽ അസ്മാഅ്: മക്‌രീസി: 8:313)

3. പ്രവാചകൻ (സ) മക്കയിൽ ഇസ്‌ലാമിക പ്രബോധനം ആരംഭിച്ചപ്പോൾ കലി പൂണ്ട കുറൈശികൾ പ്രവാചകന്റെ പിതൃവ്യൻ (പിതാവ് അബ്ദുല്ലയുടെ സഹോദരൻ) അബൂ ത്വാലിബിന്റെ അടുത്തു വന്ന് പരാതിപ്പെടുകയുണ്ടായി. അവർ അവരുടെ സംഭാഷണം തുടങ്ങുന്നത് ഇപ്രകാരമാണ്: “താങ്കളുടെ സഹോദര പുത്രൻ നമ്മളുടെ ദൈവങ്ങളെ അവമതിക്കുന്നു, ഇന്നയിന്നതെല്ലാം ചെയ്യുന്നു, ഇന്നതിന്നതെല്ലാം പറയുന്നു. താങ്കൾ അവനെ തടയണം…” അപ്പോൾ അബൂ ത്വാലിബ് പ്രവാചകന്റെ അടുത്ത് ചെന്ന് ചോദിച്ചു: “എന്റെ സഹോദര പുത്രാ, താങ്കളുടെ സമുദായം താങ്കളെ സംബന്ധിച്ച് പരാതി പറയുന്നുവല്ലോ ?….” (തഫ്സീറുത്വബ്‌രി: 23/ 149)

കുറൈശി നേതാവായിരുന്ന അബ്ദുൽ മുത്വലിബ് സൈഫിബ്‌നു ദിയസിനോട് നടത്തിയ സംഭാഷണത്തിൽ ഇപ്രകാരം കാണാം: “അല്ലയോ രാജാവെ, എനിക്ക് ഒരു പുത്രനുണ്ടായിരുന്നു (അബ്ദുല്ല). അവൻ എനിക്ക് വളരെ പ്രിയപ്പെട്ടവനായിരുന്നു. അവനോടെനിക്ക് വലിയ വാത്സ്യല്യവുമായിരുന്നു. എന്റെ സമൂഹത്തിലെ തന്നെ വിശിഷ്ട വനിതകളിൽ ഒരുവളായ ആമിന ബിൻത്ത് വഹബ് ഇബ്‌നു അബ്ദു മനാഫിബ്‌നു സഹ്‌റയെ ഞാൻ അവന് വിവാഹം കഴിപ്പിച്ചു നൽകി. അങ്ങനെ അവർക്കൊരു കുഞ്ഞുണ്ടായി. അവന് ഞാൻ മുഹമ്മദ് എന്ന് പേരിട്ടു. അവന്റെ പിതാവും മാതാവും മരണപ്പെടുകയുണ്ടായി. അങ്ങനെ അവന്റെ ചുമതല ഞാനും അവന്റെ പിതൃവ്യനും ഏറ്റെടുത്തു…” (ദലാഇലുന്നുബുവ്വ: 2:13, താരീഖു മദീനത്തി ദിമശ്ക്ക്: 3/449)

കുറൈശി പ്രമുഖനായിരുന്ന ത്വൽഹ (റ) ഒരിക്കൽ കച്ചവടത്തിനായി യാത്രയിലായിരിക്കെ ഒരു സന്യാസിവര്യൻ ‘ഈ മാസം പ്രവാചകന്മാരിൽ അന്തിമൻ പുറപ്പെടുന്ന മാസമാണെന്ന്’ സൂചിപ്പിക്കുകയുണ്ടായി. ത്വൽഹ (റ) പറയുന്നു “… അങ്ങനെ ഞാൻ ഉടൻ യാത്ര പുറപ്പെട്ടു. മക്കയിലെത്തി, മക്കക്കാരോട് എന്റെ അസാന്നിധ്യത്തിൽ വല്ലതും നടന്നോ എന്ന് ആരാഞ്ഞു. അവർ പറഞ്ഞു: “അബ്ദുല്ലയുടെ പുത്രൻ മുഹമ്മദ് പ്രവാചകത്വം വാദിക്കുകയുണ്ടായി. അബൂ കുഹാഫയുടെ മകൻ അദ്ദേഹത്തെ വിശ്വാസത്തിൽ പിന്തുടരുകയും ചെയ്തു…” (തഹ്ദീബുൽ കമാൽ: 5:1-11)

ബുഹൈരിയുടെ കഥയിൽ, പ്രവാചകത്വ അടയാളങ്ങൾ ബുഹൈരി കണ്ടെത്തിയ യുവാവിനെ സംബന്ധിച്ച് അദ്ദേഹം കുറൈശികളോട് ചോദിച്ചപ്പോൾ കുറൈശികൾ ഇപ്രകാരം പറഞ്ഞു: “അല്ലാഹുവാണെ, ഈ യുവാവ് ഞങ്ങളിലെ ഉന്നത കുടുംബത്തിൽ ജനിച്ച വ്യക്തിയാണ്.” അബീത്വാലിബിനെ ചൂണ്ടി കൊണ്ട് അവർ പറഞ്ഞു: “ഇത് ഈ വ്യക്തിയുടെ സഹോദര പുത്രനാണ്.” (സീറത്തു ഇബ്‌നു ഹിശാം: 1:117)

‘അബ്ദുല്ലയുടെ പുത്രൻ’ എന്ന് പ്രവാചകനെ കുറൈശികൾ പല സന്ദർഭങ്ങളിലും വിളിച്ചിരുന്നതായുള്ള ചരിത്ര ഗ്രന്ഥങ്ങളിലെ വാചകങ്ങൾ എണ്ണമറ്റതാണ്. (അസ്സീറത്തുൽ ഹലബിയ:1:194, സീറത്തു ഇബ്‌നു ഹിശാം: 1:117, അൽ ബിദായ വന്നിഹായ)

പ്രവാചകൻ (സ) ജനിച്ചു വളർന്ന സമൂഹം മുഴുവൻ -അദ്ദേഹത്തോട് ആദർശപരമായി ശത്രുക്കളായിരിക്കെ തന്നെ – പലയാവർത്തി അദ്ദേഹം അബ്ദുല്ലയുടെ പുത്രനും കുലീനനുമാണെന്ന് പ്രഖ്യാപിക്കുന്നു. എന്നിട്ടും അദ്ദേഹത്തെ പിതൃശൂന്യനായി ദുർവ്യാഖ്യാനിക്കുന്നതിലെ കുബുദ്ധി വിചിത്രം തന്നെ !!!

പുറമ്പോക്കില്‍ നില്‍ക്കുന്ന ഒരു ഈന്തപ്പന മരത്തോട് ഉപമിച്ചു കൊണ്ട് മുഹമ്മദ് നബിയുടെ പിതൃത്വവും കുടുംബ പരമ്പരയും കുറൈശികൾ അധിക്ഷേപിക്കുകയും നിഷേധിക്കുകയും ചെയ്തിരുന്നലോ ?

മറുപടി:

ഹദീസ് ദുർബലമാണെന്നതിനു പുറമെ ഹദീസിന്റെ ആശയത്തെ വിമർശകർ ദുർവ്യാഖ്യാനിക്കുക കൂടി ചെയ്തിട്ടുണ്ട്.

കാരണങ്ങൾ:

വിമർശകർ തങ്ങളുടെ വാദത്തിനായി തെളിവുപിടിച്ച നിവേദനം (തുർമുദി: 5/584: ഹദീസ് നമ്പർ: 3607) വളരെ ദുർബലമാണ് (ദഈഫ് ضعيف). നിവേദനത്തിന്റെ നിവേദക പരമ്പര (സനദ് السند) ഇപ്രകാരമാണ്:

حدثنا يوسف بن موسى البغدادي حدثنا عبيد الله بن موسى عن إسماعيل بن أبي خالد عن يزيد بن أبي زياد عن عبد الله بن الحارث عن العباس بن عبد المطلب قال

യൂസുഫുബ്‌നു മുസാ അൽ ബഗ്ദാദി നമ്മോട് പറഞ്ഞു – ഉബൈദുല്ലാഹിബ്‌നു മൂസാ നമ്മോട് പറഞ്ഞു – ഇസ്മാഈൽ ഇബ്‌നു അബീ ഖാലിദിൽ നിന്ന് – യസീദ് ഇബ്‌നു അബീസിയാദിൽ നിന്ന് – അദ്ദേഹം അബ്ദുല്ലാഹിബ്‌നുൽ ഹാരിസിൽ നിന്ന് – അദ്ദേഹം അബ്ബാസിബ്‌നു അബ്ദുൽ മുത്വലിബിൽ നിന്ന്, അദ്ദേഹം പറഞ്ഞു:…

നിവേദക പരമ്പരയിലെ യസീദ് ഇബ്‌നു അബീസിയാദ് വളരെ ദുർബലനാണെന്ന് ഹദീസ് നിദാനശാസ്ത്ര പണ്ഡിതർ വ്യക്തമാക്കിയിട്ടുണ്ട്.

ബൈഹകി പറഞ്ഞു: വിശ്വാസ്യതയുടെ കാര്യത്തിൽ ഒട്ടും ശക്തനല്ല. ഔസാഈ പറഞ്ഞു: യസീദ് ഹദീസിന്റെ കാര്യത്തിൽ ദുർബലനാണ്, അയാളുടെ ഹദീസ് പ്രവാചക ചര്യക്ക് വിരുദ്ധമാണ്. ഇബ്‌നു ഹിബ്ബാൻ പറഞ്ഞു: വിശ്വസ്ഥരായ നിവേദകർക്ക് എതിരായ ദുർബല ഹദീസുകൾ ഉദ്ധരിക്കുമായിരുന്നു. റാസി പറഞ്ഞു: അയാളുടെ ഹദീസ് എഴുതി വെക്കാമെങ്കിലും അവ പ്രാമാണികമല്ല. ദാറകുത്നി പറഞ്ഞു: ദുർബലൻ, നിവേദനങ്ങളിൽ ധാരാളം തെറ്റുകൾ സംഭവിക്കുന്ന വ്യക്തി. യഹ്‌യബ്‌നു മഈൻ പറഞ്ഞു: അയാളുടെ ഹദീസ് പ്രമാണമായി സ്വീകരിക്കപ്പെടില്ല, ദുർബലൻ. (http://hadith.islam-db.com/narrators/8368/يزيد-بن-أبي-زياد)

ചുരുക്കത്തിൽ വിമർശകർ ഉയർത്തിപ്പിടിക്കുന്ന ഹദീസ് ദുർബലമാണ്. മാത്രമല്ല, ഹദീസിന്റെ ആശയം വിമർശകർ ദുർവ്യാഖ്യാനിക്കുക കൂടി ചെയ്തിട്ടുണ്ട്. കാരണം, പ്രവാചകന്റെ (സ) പിതൃത്വമൊ അതിൽ കുറൈശികൾക്കുള്ള സംശയമൊ അല്ല ഈ (ദുർബല) ഹദീസിന്റെ ഉള്ളടക്കം. അതിനായി ആദ്യം നിവേദനത്തിന്റെ വിവർത്തനം കാണുക:

عن العباس بن عبد المطلب قال قلت… يا رسول الله إن قريشا جلسوا فتذاكروا أحسابهم بينهم فجعلوا مثلك كمثل نخلة في كبوة من الأرض فقال النبي صلى الله عليه وسلم إن الله خلق الخلق فجعلني من خيرهم من خير فرقهم وخير صليت ثم تخير القبائل فجعلني من خير قبيلة ثم تخير البيوت فجعلني من خير بيوتهم فأنا خيرهم نفسا وخيرهم بيتا

അബ്ബാസ്‌ ഇബ്‌നു അബ്ദുൽ മുത്വലിബിൽ (റ) നിന്ന്, അദ്ദേഹം പറഞ്ഞു: “ഞാന്‍ പ്രവാചകനോട് (സ) പറഞ്ഞു: അല്ലാഹുവിന്റെ ദൂതരേ, തീര്‍ച്ചയായും, കുറൈശ് ഗോത്രക്കാര്‍ ചേര്‍ന്ന് ഇരുന്ന്, അവരുടെ ‘അഹ്സാബി’നെ ( الأحساب ) സംബന്ധിച്ച് പരസ്പരം സംസാരിക്കുകയുണ്ടായി. എന്നാല്‍ താങ്കളെ അവര്‍ ഉപമിച്ചത്, പുറമ്പോക്കില്‍ നില്‍ക്കുന്ന ഒരു ഈന്തപ്പനമരത്തോട് ആണ്. (മറ്റൊരു നിവേദനത്തിൽ: ‘പുറമ്പോക്കിലെ തുളസി ചെടി പോലെയാണ് മുഹമ്മദ്’ എന്നും കാണാം: മജ്മഉസ്സവാഇദ്: 8/215 )

അന്നേരം പ്രവാചകന്‍ (ﷺ) പറഞ്ഞു: “തീര്‍ച്ചയായും അല്ലാഹു സൃഷ്ടികളെ സൃഷ്ടിച്ചപ്പോൾ അവരിൽ ഏറ്റവും ഉത്തമരിലാണ് എന്നെ ആക്കിയത്, അവരിൽ ഏറ്റവും ഉത്തമമായ വിഭാഗത്തിലും, ആ വിഭാഗത്തിലെ ഏറ്റവും മികച്ച രണ്ടു സംഘത്തിലും എന്നെ ഉൾപ്പെടുത്തി. പിന്നെ അവന്‍ ഗോത്രത്തില്‍ നിന്നും ഏറ്റവും നല്ല ഗോത്രത്തെ എനിക്കായി തിരഞ്ഞെടുത്തു, അതില്‍ നിന്നുള്ള കുടുംബങ്ങളില്‍ നിന്നും ഏറ്റവും മികച്ച കുടുബത്തെ തിരഞ്ഞെടുത്തു. ഞാന്‍ അവരുടെ ഇടയില്‍ ഏറ്റവും ഉത്തമനാണ്, അവരുടെ ഇടയിൽ ഏറ്റവും നല്ല കുടുംബത്തില്‍ നിന്നുള്ളവനും.” (തുർമുദി: 5/584: ഹദീസ് നമ്പർ: 3607)

കുറൈശികൾ കൂടിയിരുന്ന് പരസ്പരം ചർച്ച ചെയ്തത് അവർക്കിടയിലെയോ, പ്രവാചകന്റെയോ കുടുംബ പരമ്പരയെ (النسب) സംബന്ധിച്ചല്ല, അവർക്കിടയിലെ ‘അഹ്സാബി’നെ (മഹിമ الأحساب) സംബന്ധിച്ചാണ്. അഹ്സാബ് എന്നതിന്റെ ഏകവചനം ഹസബ് (الحسب) എന്നാണ്. ഹസബും നസബും രണ്ടും വ്യത്യസ്ഥമായ കാര്യങ്ങളാണ്. ഇത് അറബി ഭാഷാ ഗ്രന്ഥങ്ങളിൽ നിന്നും നമുക്ക് മനസ്സിലാക്കാം.

“ഒരു മനുഷ്യൻ തന്നിലും തന്റെ പ്രപിതാക്കന്മാരിലും അഭിമാന ഹേതുവായി പരിഗണിക്കുന്നവയാണ് ഹസബ്. ഒരാളുടെ ഹസബ് അയാളുടെ മതമോ ധനമോ സംഘശക്തിയോ ആകാം. പ്രവർത്തനങ്ങളിലൂടെയും കർമ്മങ്ങളിലൂടെയും നേടിയെടുക്കുന്ന മഹിമയാണ് ഹസബ്. പിതാക്കന്മാരെ കൂടാതെ തന്നെ ഹസബും കറമും (ഉദാരത) ഉണ്ടാകാം. എന്നാൽ ബഹുമാനവും മഹത്വവും പിതാക്കന്മാരിലൂടെയല്ലാതെ സിദ്ധിക്കില്ല. അഥവാ പിതൃത്വമോ കുടുംബ പരമ്പരയോ ആയി ബന്ധപ്പെട്ട ഒന്നല്ല ഹസബ്.” (മുഖ്താറുസ്സിഹാഹ്: 1/57, അന്നിഹായ ഫീ ഗരീബിൽ ഹദീസ്: 1/381)

പുറമ്പോക്കില്‍ നില്‍ക്കുന്ന ഒരു ഈന്തപ്പനമരത്തോട് ഉപമിച്ചു കൊണ്ട് മുഹമ്മദ് നബിയുടെﷺ പിതൃത്വവും കുടുംബ പരമ്പരയും അധിക്ഷേപിക്കുകയും നിഷേധിക്കുകയുമല്ല കുറൈശികൾ ചെയ്തത്. മറിച്ച് അവർക്കിടയിൽ പരസ്പരം നിലനിൽക്കുന്ന ഗോത്ര വിധ്വേഷമാണ് അവർ പ്രകടിപ്പിച്ചത്. ഹാശിം കുടുംബത്തെയാണ് അവർ പുറമ്പോക്കെന്ന് പറഞ്ഞ് ആക്ഷേപിച്ചത്. അതേസമയം മുഹമ്മദ് നബിയെ, അറബികളുടെ അടുക്കൽ ഏറ്റവും ആദരണീയമായ ഈന്തപ്പനമരത്തോടും തുളസി ചെടിയോടുമാണ് ഉപമിച്ചത്. ഇതിൽ നിന്ന് തന്നെ പ്രവാചകനെ ആക്ഷേപിക്കുകയോ അദ്ദേഹത്തിന്റെ പിതൃത്വത്തെ ചോദ്യം ചെയ്യുകയോ അല്ല എന്ന് വ്യക്തമായി മനസ്സിലാക്കാം.

മുഹമ്മദ് നബിയുടെﷺ പിതൃത്വവും കുടുംബ പരമ്പരയും കുറൈശികൾ അധിക്ഷേപിക്കുകയും നിഷേധിക്കുകയും ചെയ്തു എന്നത് വിചിത്ര വാദവും മൗഢ്യവുമാണ്. കാരണം സ്ഥാപിതവും സ്വീകാര്യയോഗ്യവുമായ ചരിത്ര നിവേദനങ്ങളും ഹദീസുകളും തെളിയിക്കുന്നത് നേർ വിപരീതമാണ്:

1. പ്രവാചകന്റെ(സ) ജനനവുമായി ബന്ധപ്പെട്ട ഒരു ദീർഘമായ ഹദീസിന്റെ ആശയ സംഗ്രഹം ഇപ്രകാരമാണ്:

ജൂതന്മാരിൽ നിന്ന് ഒരാൾ മക്കയിൽ കച്ചവട ആവശ്യാർത്ഥം താമസിക്കുകയുണ്ടായി. പ്രവാചകൻ (സ) ജനിച്ച രാത്രി അയാൾ കുറൈശികളുടെ സദസ്സിൽ വന്ന് ‘ഇന്ന് രാത്രി വല്ല കുഞ്ഞും പിറന്നോ’ എന്ന് ചോദിക്കുകയുണ്ടായി. വല്ല കുഞ്ഞും പിറന്നോ എന്ന് അന്വേഷിക്കാനായി ജൂതൻ കുറൈശികളോട് ആവശ്യപ്പെട്ടു, ഇന്ന് ഈ സമുദായത്തിൽ ഒരു പ്രവാചകൻ പിറക്കുമെന്നും ആ കുഞ്ഞിന്റെ മുതുകിൽ ഒരു അടയാളമുണ്ടാകുമെന്നും അവരോട് പറയുകയും ചെയ്തു. കുറൈശികൾ തങ്ങളുടെ വീടുകളിലേക്ക് മടങ്ങി പോവുകയും ഭാര്യമാരോട് കാര്യം പറയുകയും ചെയ്തു. അപ്പോൾ ഭാര്യമാർ പറഞ്ഞു: അബ്ദുല്ല ഇബ്‌നു അബ്ദുൽ മുത്വലിബിന് ഇന്ന് ഒരു കുഞ്ഞു പിറന്നിട്ടുണ്ട്. അവർ ആ കുഞ്ഞിന് മുഹമ്മദ് എന്ന് പേരിടുകയും ചെയ്തു. ഈ വിവരം കുറൈശികൾ ജൂതനെ അറിയിച്ചു. ഹിശാമിബ്‌നു വലീദിബ്‌നു മുഗീറ, മുസാഫിർ ഇബ്‌നു അബീ അംറ്, ഉബൈദ ഇബ്‌നുൽ ഹാരിസ് ഇബ്‌നു അബ്ദുൽ മുത്വലിബ്, ഉത്ബത്തിബ്‌നു റബീഅ തുടങ്ങി കുറൈശികളിലേയും ബനൂ മനാഫ് കുടുംബത്തിലേയും നേതാക്കളുമൊത്ത് ജൂതൻ ആമിനയുടെ വീട്ടിലേക്ക് പോയി. കുഞ്ഞിന്റെ മുതുകിൽ പ്രവാചകനുണ്ടാകുമെന്ന് പ്രവചിക്കപ്പെട്ട മുദ്ര /അടയാളം കണ്ട ജൂതൻ ബോധരഹിതനായി വീണു. ഉണർന്നപ്പോൾ കാര്യമന്വേഷിച്ച കുറൈശികളോട്, പ്രവാചകത്വം ഇസ്രാഈല്യരിൽ നിന്ന് കുറൈശികളിലേക്ക് നീക്കപ്പെട്ടല്ലൊ എന്ന് ജൂതൻ പരിതപിച്ചു. (അൽ മുസ്തദ്റക് അല സ്വഹീഹൈൻ: 2/656: ഹദീസ് നമ്പർ: 4177)

2. പ്രവാചകനും കുറൈശികളും തമ്മിൽ ഹുദൈബിയ സമാധാന കരാർ എഴുതവെ പ്രവാചകൻ (സ) അനുചരൻ അലിയോട് പറഞ്ഞു: അല്ലാഹുവിന്റെ ദൂതൻ മുഹമ്മദിൽ നിന്ന് എന്നെഴുതുക. അപ്പോൾ കുറൈശി നേതാവ് സുഹൈൽ ഇബ്‌നു അംറ് പറഞ്ഞു: നീ അല്ലാഹുവിന്റെ ദൂതനാണെന്ന് ഞങ്ങൾ അംഗീകരിച്ചിരുന്നുവെങ്കിൽ ഞങ്ങൾ നിന്നെ (മതത്തിൽ) പിന്തുടരുമായിരുന്നല്ലോ. അതുകൊണ്ട് നിന്റെ പേരും നിന്റെ പിതാവിന്റെ പേരും എഴുതുക. അപ്പോൾ പ്രവാചകൻ (സ) അലിയോട് (റ) പറഞ്ഞു: അബ്ദുല്ലയുടെ മകൻ മുഹമ്മദിൽ നിന്ന് എന്നെഴുതുക. (സ്വഹീഹു മുസ്‌ലിം: 3/1411: ഹദീസ് നമ്പർ: 1784)

3. അബ്‌സീനിയ ഭരിച്ചിരുന്ന ക്രിസ്ത്യൻ രാജാവ് നജ്ജാശിയോട് ജഅ്ഫർ (റ) പറഞ്ഞു: “രാജാവേ! ഞങ്ങള്‍ അജ്ഞരായ ഒരു ജനതയായിരുന്നു. ഞങ്ങള്‍ ബിംബങ്ങളെ ആരാധിക്കുകയും ശവം തിന്നുകയും മ്ലേച്ഛതകൾ പ്രവർത്തിക്കുകയും കുടുബബന്ധങ്ങള്‍ മുറിക്കുകയും അയല്‍ക്കാരെ ഉപദ്രവിക്കുകയും ഞങ്ങളിലെ ശക്തർ ദുർബലരുടെ ധനം ഭുജിക്കുകയും ചെയ്യുന്നവരായിരുന്നു. അങ്ങനെയിരിക്കെ അല്ലാഹു ഞങ്ങള്‍ക്ക് ഞങ്ങളില്‍നിന്നു തന്നെ ഒരു ദൈവദൂതനെ നിയോഗിച്ചു. അദ്ദേഹത്തിന്റെ കുടുംബ പരമ്പരയും സത്യസന്ധതയും വിശ്വസ്തതയും സൽസ്വഭാവവും ഞങ്ങള്‍ക്ക് നന്നായി അറിയാം. അദ്ദേഹം ഞങ്ങളെ ദൈവത്തിലേക്ക് ക്ഷണിച്ചു. ദൈവം ഏകനാണെന്ന് വിശ്വസിക്കാനും അവനെ മാത്രം ആരാധിക്കാനും, അവനു പുറമെ ഞങ്ങളും ഞങ്ങളുടെ പൂര്‍വ്വ പിതാക്കളും ആരാധിച്ചിരുന്ന ബിംബങ്ങളേയും കൽ പ്രതിമകളേയും മറ്റും ഉപേക്ഷിക്കുവാനും ഞങ്ങളോടു കല്‍പ്പിച്ചു. കൂടാതെ, സത്യം പറയുക, വിശ്വസിച്ചേൽപ്പിച്ചവ സത്യസന്ധമായി നിർവ്വഹിക്കുക, കുടുംബബന്ധം ചേര്‍ക്കുക, അയല്‍ക്കാർക്ക് നന്മ ചെയ്യുക എന്നെല്ലാം ഞങ്ങളോട് കൽപ്പിച്ചു. കൊലയും, രക്തച്ചൊരിച്ചിലും, വ്യഭിചാരവും, കളവും, അനാഥകളുടെ സമ്പത്ത് തട്ടിയെടുത്ത് ഭുജിക്കലും മാന്യവതികളായ സ്ത്രീകളെ സംബന്ധിച്ച് ദുരാരോപണങ്ങൾ ഉന്നയിക്കുന്നതുമെല്ലാം അദ്ദേഹം ഞങ്ങളോട് വിരോധിക്കുകയും ചെയ്തു. നമസ്‌ക്കരിക്കാനും വ്രതമനുഷ്ടിക്കാനും (പാവപ്പെട്ടവർക്ക് നിർബന്ധ ദാനമായ) സക്കാത്ത് നൽകാനും ഞങ്ങളോട് കൽപ്പിച്ചു. (സ്വഹീഹു ഇബ്‌നു ഖുസൈമ: 4/13: ഹദീസ് നമ്പർ: 2260, സീറത്തു ഇബ്‌നു ഇസ്ഹാക്: 213, മുസ്നദു അഹ്മദ്: 3/180, സീറത്തു ഇബ്‌നു ഹിശാം: 3/413, സ്വഹീഹു സീറത്തു ന്നബവിയ്യ: ത്വർഹൂനി: 1/340, ത്വബ്റാനി, ബസ്സാർ, ത്വയാലിസി)

മുകളിലെ ഹദീസിലെ نَعرِف نسبه وصدقه وأمانته وعفافه “അദ്ദേഹത്തിന്റെ കുടുംബ പരമ്പരയും സത്യസന്ധതയും വിശ്വസ്തതയും സൽസ്വഭാവവും ഞങ്ങള്‍ക്ക് നന്നായി അറിയാം….” എന്ന വരി പ്രത്യേകം ശ്രദ്ധ അർഹിക്കുന്നു.

4. പ്രവാചക കാലഘട്ടത്തിലെ റോമൻ ചക്രവർത്തി ഹിർക്കലിന്റെ ചരിത്രവും ഇതിന് തെളിവാണ്:

ആവിർഭാവ കാലഘട്ടത്തിൽ, ഇസ്‌ലാമിനെ സംബന്ധിച്ച് കേൾക്കുകയും അതിൽ കൗതുകം ജനിക്കുകയും ചെയ്ത ഹിർക്കൽ ചക്രവർത്തി ശാമിൽ കച്ചവടത്തിനായി വന്നു ചേർന്ന, പ്രവാചകന്റെ എതിരാളിയും കുറൈശി നേതാവുമായ അബൂസുസുഫ്‌യാനെ ചക്രവർത്തിയുടെ സന്നിധിലേക്ക് ക്ഷണിച്ചു വരുത്തി. സംഭവം അബൂസുഫ്‌യാൻ തന്നെ വിശദീകരിക്കുന്നു:

“ഞങ്ങൾ ഒരു കച്ചവട സംഘത്തിലായിരിക്കെ ഹിർക്കൽ ഒരു ദൂതനെ അയച്ചു അദ്ദേഹത്തിന്റെ അടുത്ത് ഹാജരാകാൻ കൽപ്പിച്ചു. കുറൈശികളും അല്ലാഹുവിന്റെ ദൂതനും അക്കാലത്ത് സമാധാന സന്ധിയിലേർപ്പെട്ടിരിക്കുകയായിരുന്നു. അങ്ങനെ അബൂസുഫ്‌യാനും കച്ചവട സംഘവും ഈലിയായിൽ ഹിർക്കലിന്റെ അടുത്തു ചെന്നു. ഹിർഖൽ അവരെ തന്റെ സദസ്സിലേക്ക് ക്ഷണിച്ചു. ചക്രവർത്തിക്ക് ചുറ്റും റോമിലെ പ്രധാനികളും പ്രമാണികളും ഹാജരുണ്ടായിരുന്നു. ചക്രവർത്തി തന്റെ പരിഭാഷകരേയും കൊണ്ടുവന്നു:

“നിങ്ങളിൽ ആർക്കാണ്, താൻ പ്രവാചകനാണെന്ന് വാദിക്കുന്ന ഈ വ്യക്തിയുമായി അടുത്ത കുടുംബ ബന്ധമുള്ളത് ?” എന്ന് ചക്രവർത്തി ചോദിച്ചു. “ഞാൻ പറഞ്ഞു: ഞാനാണ് ഈ കൂട്ടത്തിൽ അദ്ദേഹത്തോട് ഏറ്റവും അടുത്ത കുടുംബ ബന്ധമുള്ളയാൾ.” ഹിർക്കൽ പറഞ്ഞു; “അദ്ദേഹത്തെ എന്റെ അടുത്തേക്ക് കൊണ്ടു വരൂ, അദ്ദേഹത്തിന്റെ കൂട്ടാളികളെ അദ്ദേഹത്തിന്റെ പിറകിൽ നിർത്തൂ.” എന്നിട്ട് ഹിർക്കൽ പരിഭാഷകനോട് പറഞ്ഞു: “അവരോട് പറയൂ: ഞാൻ മുഹമ്മദെന്ന വ്യക്തിയെ പറ്റി അബൂസുഫ്‌യാനോട് ചിലത് ചോദിക്കും, അബൂസുഫ്‌യാൻ കളവ് പറഞ്ഞാൽ അദ്ദേഹം പറയുന്നത് കളവാണെന്ന് പറയണമെന്ന് അദ്ദേഹത്തിന്റെ കൂട്ടാളികളോട് പറയുക.” ശേഷം, ഹിർഖൽ ചക്രവർത്തി ചോദിച്ച ആദ്യത്തെ ചോദ്യം: “അദ്ദേഹത്തിന്റെ (മുഹമ്മദ് നബി (ﷺ)) കുടുംബ പരമ്പര എങ്ങനെയാണ്?” എന്നായിരുന്നു. ഞാൻ പറഞ്ഞു: “അദ്ദേഹം (സ) കുടുംബ മഹിമയുള്ള വ്യക്തിയാണ്.” (ദീർഘമായ സംഭാഷണത്തിന്റെ അവസാന ഭാഗത്തിൽ ഇപ്രകാരം കാണാം:) …ശേഷം ഹിർക്കൽ പരിഭാഷകനോട് പറഞ്ഞു: “അദ്ദേഹത്തോട് പറയുക: മുഹമ്മദിന്റെ കുടുംബമഹിമയെക്കുറിച്ച് ചോദിച്ചപ്പോള്‍ അദ്ദേഹം കുടുംബമഹിമയുള്ളവനാണെന്ന് താങ്കള്‍ പറഞ്ഞു. അപ്രകാരമാണ് പ്രവാചകന്മാരും, സാധാരണ ഗതിയിൽ അവർ കുടുംബ മഹിമയുള്ളവരായിരിക്കും….” (സ്വഹീഹുൽ ബുഖാരി: ഹദീസ് നമ്പർ: 7)

മുഹമ്മദിനെ(സ) ഗർഭം ധരിച്ചു എന്ന് പറയുന്നതിനപ്പുറം, ആമിനയ്ക്ക് എത്ര മാസമായിരുന്നു അബ്ദുല്ലയുടെ മരണം സംഭവിക്കുന്ന സമയം എന്ന് ഉറപ്പിക്കാവുന്ന തെളിവുകള്‍ ഒന്നും വന്നുകിട്ടിയിട്ടില്ല. അബ്ദുല്ല – ആമിന ദമ്പതികളുടേയും അബ്ദുൽമുത്തലിബ് – ഹാല ദമ്പതികളുടേയും വിവാഹം ഒരുമിച്ചാണ് നടന്നത്. അബ്ദുൽമുത്തലിബ് – ഹാല ദമ്പതികളുടെ ദാമ്പത്യം പിന്നീടും വര്‍ഷങ്ങള്‍ നീണ്ടുനിന്നു. ആയതിനാൽ അബ്ദു-മുത്തലിബ് ഹാല ദമ്പതിമാരുടെ മകന്‍ ഹംസ ബിന്‍ അബ്ദുൽ മുത്തലിബ്, അബ്ദുല്ല – ആമിന ദമ്പതിമാരുടെ പുത്രനായ മുഹമ്മദിനെക്കാളും ഇളയവന്‍ ആകണമല്ലൊ. കാരണം, അബ്ദുല്ല വിവാഹത്തിന് ശേഷം മാസങ്ങൾ മാത്രമെ ആമിനയുടെ കൂടെ ദാമ്പത്യം നടത്തിയിട്ടുള്ളു, ശേഷം മരണപ്പെട്ടു. പക്ഷെ ഹംസ മുഹമ്മദിനെക്കാള്‍ മൂന്ന്-നാലു വയസ്സിന് മുതിര്‍ന്നതാണ് എന്നതിലേയ്ക്കാണ് തെളിവുകൾ വിരല്‍ ചൂണ്ടുന്നത്. അപ്പോൾ അബ്ദുല്ല മരണപ്പെട്ടിട്ടും രണ്ടോ മൂന്നോ വര്‍ഷങ്ങള്‍ കഴിഞ്ഞാണ് ആമിന ഗര്‍ഭിണിയായത്‌ എന്നല്ലേ വരിക. ഇത് ആമിനയുടെ അവിഹിത ഗർഭത്തിലേക്കല്ലെ വിരൽ ചൂണ്ടുന്നത് ?

ആരോപണം തെളിവുകളുടെ പിൻബലമില്ലാത്തതും ചരിത്ര നിവേദനങ്ങൾ വളരെ ദുർബലവുമാണ്.

കാരണങ്ങൾ:

മുകളിൽ സൂചിപ്പിച്ച വിമർശകരുടെ ആരോപണത്തിൽ മൂന്ന് വാദങ്ങൾ ഉൾക്കൊള്ളുന്നു:

1. അബ്ദുല്ല – ആമിന ദമ്പതികളുടേയും, അബ്ദുൽ മുത്തലിബ് – ഹാല ദമ്പതികളുടേയും വിവാഹം ഒരുമിച്ചായിരുന്നു.

2. മുഹമ്മദിനേക്കാൾ(സ) മൂന്ന്-നാലു വയസ്സിന് മുതിര്‍ന്ന വ്യക്തിയാണ് ഹംസ (റ).

3. അബ്ദുല്ലയുടെ ജീവിത കാലഘട്ടത്തിൽ തന്നെ ആമിന മുഹമ്മദിനെ (സ) ഗർഭം ചുമന്നിട്ടുണ്ടോ എന്നതിന് വ്യക്തമായ തെളിവൊന്നുമില്ല എന്ന വ്യംഗ്യമായ സൂചന.

ഈ മൂന്ന് വാദവും തീർത്തും വാസ്തവ വിരുദ്ധവും കളവുമാണ്.

1. അബ്ദുല്ല – ആമിന ദമ്പതികളുടേയും, അബ്ദുൽ മുത്തലിബ് – ഹാല ദമ്പതികളുടേയും വിവാഹം ഒരേ ദിവസമായിരുന്നു എന്ന് സൂചിപ്പിക്കുന്ന നിവേദനങ്ങൾ വളരെ ദുർബലങ്ങളാണ് (ദഈഫ് ضعيف).

ഇബ്നു സഅ്ദ് തന്റെ ത്വബക്കാത്തുൽ കുബ്റായിൽ (1/94) ഉദ്ധരിച്ച ഈ നിവേദനത്തിന്റെ നിവേദക പരമ്പര (സനദ് السند) ഇപ്രകാരമാണ്:

قال حدثنا محمد بن عمر بن واقد الأسلمي قال حدثني عبد الله بن جعفر الزهري عن عمته أم بكر بنت المسور بن مخرمة عن أبيها قال وحدثني عمر بن محمد بن عمر بن علي بن أبي طالب عن يحيى بن شبل عن أبي جعفر محمد بن علي بن الحسين قالا…

മുഹമ്മദിബ്നു ഉമർ വാക്കിദുൽ അസ്ലമി നമ്മോട് പറഞ്ഞു – എന്നോട് അബ്ദുല്ലാഹിബ്നു ജഅ്ഫർ അസ്സുഹ്‌രി പറഞ്ഞു – അദ്ദേഹം തന്റെ പിതൃവ്യയായ ഉമ്മു ബകർ ബിൻത്ത് അൽ മിസ്വർ ഇബ്നു മഖ്‌റമയിൽ നിന്ന് – അദ്ദേഹം തന്റെ പിതാവിൽ നിന്ന് – എന്നോട് ഉമർ ഇബ്നു മുഹമ്മദിബ്നു ഉമറിബ്നു അലിയ്യിബ്നു അബീത്വാലിബ് പറഞ്ഞു – അദ്ദേഹം യഹ്‌യബ്നു ശിബലിൽ നിന്ന് – അദ്ദേഹം അബൂജഅ്ഫർ മഹമ്മദിബ്നു അലിയ്യിബ്നുൽ ഹുസൈനിൽ നിന്ന് – അവർ രണ്ടു പേരും പറഞ്ഞു: …..

പരമ്പരയിലെ മുഹമ്മദിബ്നു ഉമർ വാക്കിദുൽ അസ്‌ലമി (സത്യസന്ധതയിൽ) ദുർബലനാണ്. അഹ്മദിബ്നു ഹമ്പൽ പറഞ്ഞു: മുഹമ്മദിബ്നു ഉമർ വാക്കിദുൽ അസ്‌ലമി നുണയനാണ്; അയാൾ ഹദീസുകളിൽ കോട്ടിമാട്ടുമായിരുന്നു.

യഹ്‌യ പറഞ്ഞു: അയാൾ വിശ്വസ്ഥനല്ല. അയാളുടെ ഹദീസുകൾ എഴുതിവെക്കാൻ കൊള്ളാത്തത്രയും അവിശ്വസനീയമാണ്. ഇമാം ബുഖാരി, റാസി, നസാഈ എന്നിവർ പറഞ്ഞു: അയാൾ കളവു കൊണ്ട് ആരോപിതനാണ്. റാസി, നസാഈ എന്നിവർ പറഞ്ഞു: അയാൾ വ്യാജ ഹദീസുകൾ ഉണ്ടാക്കുന്ന വ്യക്തിയായിരുന്നു. ഇമാം ദാറക്കുത്നി പറഞ്ഞു: അയാളിൽ ദൗർബല്യമുണ്ട്. ഇസ്ഹാകിബ്നു റാഹൂയ പറഞ്ഞു: അയാൾ നുണയനാണ്. (അദ്ദുഅഫാഉ വൽ മത്റൂകീൻ : ഇബ്നുൽ ജൗസി: 3 / 87, അദ്ദുഅഫാഉ സ്സ്വഗീർ: ബുഖാരി: 334, അൽ ജർഹുവതഅദീൽ: അബൂഹാതിം: 8/21, അൽ കാമിൽ ഇബ്നു അദിയ്യ്: 7/ 481)

മറ്റൊരു നിവേദനത്തിന്റെ (അൽ മുസ്തദ്റക്ക്: 2:656) പരമ്പര (സനദ് السند) ഇപ്രകാരമാണ്:

4176 أخبرنا أبو جعفر محمد بن محمد بن عبد الله البغدادي حدثنا هاشم بن مرثد الطبراني حدثنا يعقوب بن محمد الزهري حدثنا عبد العزيز بن عمران حدثنا عبد الله بن جعفر عن أبي عون عن المسور بن مخرمة عن بن عباس عن أبيه قال قال عبد المطلب

അബൂ ജഅ്ഫർ മുഹമ്മദിബ്നു മുഹമ്മദിബ്നു അബ്ദുല്ല അൽ ബഗ്ദാദി നമ്മോട് പറഞ്ഞു – ഹാശിം ഇബ്നു മിർസദ് – യഅ്കൂബിബ്നു മുഹമ്മദ് അസ്സുഹ്‌രി നമ്മോട് പറഞ്ഞു – അബ്ദുൽ അസീസിബ്നു ഇംറാൻ നമ്മോട് പറഞ്ഞു – അബ്ദുല്ലാഹിബ്നു ജഅ്ഫർ – അബൂ ഔനിൽ നിന്ന് – അദ്ദേഹം മിസ്വറിബനു മഖ്റമയിൽ നിന്ന് – അദ്ദേഹം അബ്ബാസിൽ നിന്ന് – അദ്ദേഹം തന്റെ പിതാവായ അബ്ദുൽ മുത്വലിബിൽ നിന്ന്….

നിവേദക പരമ്പരയിൽ അബ്ദുൽ അസീസിബ്നു ഇംറാൻ എന്ന നിവേദകനുണ്ട്. അയാൾ വിശ്വസ്ഥനല്ലെന്ന് ഹദീസ് നിദാനശാസ്ത്ര പണ്ഡിതന്മാർ വ്യക്തമാക്കിയിട്ടുണ്ട്.

യഹ്‌യ പറഞ്ഞു: അയാൾ വിശ്വസ്ഥനല്ല. ഇമാം ബുഖാരി പറഞ്ഞു: അയാളുടെ ഹദീസുകൾ എഴുതിവെക്കാൻ കൊള്ളാത്തത്രയും അവിശ്വസനീയമാണ്. നസാഈ പറഞ്ഞു: അയാൾ കളവു കൊണ്ട് ആരോപിതനാണ്. ഇമാം ഇബ്നു ഹിബ്ബാൻ പറഞ്ഞു: അയാൾ വിശ്വസ്ഥരായ നിവേദകർക്കെതിരായി ദുർബല ഹദീസുകൾ ഉദ്ധരിക്കുന്ന വ്യക്തിയായിരുന്നു. ഇമാം തുർമുദി, ദാറക്കുത്നി എന്നിവർ പറഞ്ഞു: അയാൾ ദുർബലനാണ്. (അദ്ദുഅഫാഉ വൽ മത്റൂകീൻ: ഇബ്നുൽ ജൗസി: 2 / 111, അദ്ദുഅഫാഉ സ്സ്വഗീർ: ബുഖാരി: 223, അൽ ജർഹുവതഅദീൽ: അബൂഹാതിം: 5/391)

2. മുഹമ്മദിനേക്കാൾ(സ) മൂന്ന്-നാലു വയസ്സിന് മുതിര്‍ന്ന വ്യക്തിയാണ് ഹംസ (റ) എന്ന് സൂചിപ്പിക്കുന്ന നിവേദനത്തിന്റെ (ത്വബക്കാത്തുൽ കുബ്റാ: ഇബ്നു സഅ്ദ്: 3:10) നിവേദക പരമ്പര ഇപ്രകാരമാണ്:

قال أخبرنا محمد بن عمر قال حدثني موسى بن محمد بن إبراهيم عن أبيه قال

മുഹമ്മദിബ്നു ഉമർ വാക്കിദുൽ അസ്‌ലമി നമ്മെ അറിയിച്ചു – മൂസ ഇബ്നു മുഹമ്മദ് ഇബ്നു ഇബ്രാഹീം തന്റെ പിതാവിൽ നിന്ന് ഉദ്ധരിക്കുന്നു – അദ്ദേഹം പറഞ്ഞു:….

നിവേദക പരമ്പരയിലെ മുഹമ്മദിബ്നു ഉമർ വാക്കിദുൽ അസ്‌ലമി (സത്യസന്ധതയിൽ) ദുർബലനാണ് എന്ന് നാം മുമ്പ് സൂചിപ്പിച്ചു. പരമ്പരയിലെ മറ്റൊരു നിവേദകനായ മൂസ ഇബ്നു മുഹമ്മദ് ഇബ്നു ഇബ്രാഹീമും ദുർബലാണ്. ഇമാം അബൂ ഹാതിം പറഞ്ഞു: അയാൾ തന്റെ പിതാവിൽ നിന്ന് പിതാവ് പറഞ്ഞിട്ടില്ലാത്ത വ്യാജവർത്തമാനങ്ങൾ ഉദ്ധരിക്കുമായിരുന്നു…വിശ്വസ്ഥരായ നിവേദകർക്കെതിരായി ദുർബല ഹദീസുകൾ ഉദ്ധരിക്കുന്ന വ്യക്തിയായിരുന്നു… യാതൊരു വിധത്തിലും പ്രമാണികനല്ലാത്ത വ്യക്തി. (അൽ മജ്റൂഹീൻ: 2:241)

ഇമാം ഇബ്നു അബ്ദുൽ ബിർറ് പറഞ്ഞു: ഹംസ അല്ലാഹുവിന്റെ ദൂതനേക്കാൾ(സ) നാലു വയസ്സ് മുതിര്‍ന്നതാണ് എന്ന് സൂചിപ്പിക്കുന്ന നിവേദനമുണ്ട്. ഇത് സ്വീകാര്യയോഗ്യമല്ല. കാരണം സ്ഥാപിതമായ ഹദീസിൽ വന്നിരിക്കുന്നത് സുവൈബ ഹംസയേയും പ്രവാചകനേയും ഒരുമിച്ച് മുലയൂട്ടിയിട്ടുണ്ട് എന്നാണ്. (മുലയൂട്ടുന്ന പ്രായം രണ്ടു വയസ്സിന് താഴെയാണ്. അപ്പോൾ രണ്ടു പേർക്കും ഏകദേശം ഒരേ പ്രായമാണെന്ന് സ്ഥാപിതമാവുന്നു.) (അൽ ഇസ്തീആബ്: 1/369)

3. അബ്ദുല്ലയുടെ ജീവിത കാലഘട്ടത്തിൽ തന്നെ ആമിന മുഹമ്മദിനെ(സ) ഗർഭം ചുമന്നിട്ടുണ്ടോ എന്നതിന് വ്യക്തമായ തെളിവൊന്നുമില്ല എന്ന വ്യംഗ്യമായ സൂചന തീർത്തും കുതന്ത്രപരവും തെറ്റിദ്ധരിപ്പിക്കലുമാണ്. “ചരിത്രകാരനായ ഇബ്നു ഇസ്ഹാക് പറഞ്ഞു: അല്ലാഹുവിന്റെ തിരുദൂതനെ മാതാവ് ഗർഭം ചുമക്കുന്ന ഘട്ടത്തിലാണ് പിതാവ് അബ്ദുല്ല മരണപ്പെടുന്നത്.

ഇതാണ് ചരിത്രകാരന്മാരായ ഇബ്നു ഇസ്ഹാക് തറപ്പിച്ചു പറയുന്നത്. ഈ അഭിപ്രായത്തെ തന്നെയാണ് വാക്വിദി, ഇബ്നു സഅ്ദ്, ബലാദുരി എന്നിവർ മുന്തിക്കുന്നതും ദഹബി ശരിവെക്കുന്നതും. ഇബ്നുകസീർ പറഞ്ഞു: ഇത് വളരെ പ്രസിദ്ധമാണ്. ഇബ്നുൽ ജൗസി പറഞ്ഞു: (പ്രവാചകൻ തൊട്ടിലിൽ കിടക്കുന്ന പ്രായത്തിലാണ് പിതാവ് അബ്ദുല്ല മരണപ്പെടുന്നത് എന്ന് ചരിത്ര വിശാരദരിൽ ഒരു ന്യൂനപക്ഷത്തിന് അഭിപ്രായമുണ്ട് എന്നതൊഴിച്ചാൽ) പ്രവാചക ചരിത്രം രചിച്ച ഭൂരിഭാഗവും ഇതേ അഭിപ്രായക്കാരാണ്. ഇതിന് ഉപോൽബലകമായ നിവേദനം ഹാകിം ഉദ്ധരിക്കുകയും സ്വീകാര്യയോഗ്യമാണെന്ന് അഭിപ്രായപ്പെടുകയും ചെയ്തിരിക്കുന്നു. കൈസിബ്നു മഖ്റമ എന്ന പ്രവാചക ശിഷ്യനിൽ നിന്ന് തന്നെ ഇക്കാര്യം ഉദ്ധരിച്ച് ഉറപ്പിക്കുന്നുണ്ട് ഇമാം ദഹബി. ഇബ്നു ഇസ്ഹാക് അല്ലാത്ത ചരിത്രകാരന്മാർ പറയുന്നത് ആമിന പ്രവാചകനെയും കൊണ്ട് രണ്ട് മാസം ഗർഭിണിയായിരിക്കെയാണ് അബ്ദുല്ല മരണപ്പെട്ടത് എന്നാണ്.” (സുബുലുൽ ഹുദാ വർറശാദ്: അസ്സ്വാലിഹി അശ്ശാമി: 1/331)

സ്ത്രീകളെ തികഞ്ഞ അശ്ലീലതകള്‍ക്കു പ്രേരിപ്പിക്കുന്ന പല ഉപദേശങ്ങളും ഹദീഥ് ഗ്രന്ഥങ്ങളില്‍, നബിക്കു ലഭിച്ച ദൈവിക വെളിപാടുകളായി ഇന്നും രേഖപ്പെടുത്തപ്പെട്ടു കിടക്കുന്നതു കാണാം. സുഹൈലിന്റെ മകള്‍ സഹ്‌ല എന്ന സ്ത്രീയോട് സാലിം എന്ന പരപുരുഷനെ മുലയൂട്ടി മകനായി സ്വീകരിക്കാന്‍ ആവശ്യപ്പെട്ട ഹദീഫുകള്‍ സ്വഹീഹു മുസ്‌ലിമില്‍ വരെ കാണാം. ഈ പ്രവാചകോപദേശം സ്വീകരിക്കാന്‍ മാന്യതയുള്ള മുസ്‌ലിം സ്ത്രീകള്‍ക്കു സാധിക്കുമോ? മാത്രമല്ല പ്രവാചക പത്‌നി ആഇശ ഇത്തരത്തില്‍ പരപുരുഷന്മാരെ മുലയൂട്ടാറുണ്ടായിരുന്നുവെന്നും പലപ്പോഴും ആഇശയുടെ അരികില്‍ പരപുരുഷന്മാരെ കാണുക വഴി നബിക്കുതന്നെ അതില്‍ അനിഷ്ടമുണ്ടാവുകയും ചെയ്തിരുന്നു എന്ന് സ്വഹീഹ് മുസ്‌ലിമിലെ 1455-ാം ഹദീഥായി രേഖപ്പെടുത്തിയിട്ടുണ്ട്. സ്വയം ചെയ്തിരുന്നു എന്നു മാത്രമല്ല തന്റെ സഹോദര-സഹോദരി പുത്രിമാരോട് അപ്രകാരം ചെയ്യാന്‍ ആഇശ ഉപദേശിക്കുമായിരുന്നു എന്നു കൂടി ഹദീഥ് ഗ്രന്ഥങ്ങളില്‍ രേഖപ്പെടുത്തപ്പെട്ടിട്ടുണ്ട്. മാത്രമല്ല, കല്ലെറിയലിന്റെ വചനവും പത്ത് പ്രാവശ്യമാണ് മുതിര്‍ന്ന പുരുഷന്മാരുടെ മുലകുടിയെന്ന വചനവും (ആദ്യകാലത്ത് ക്വുര്‍ആനില്‍) അവതരിപ്പിക്കപ്പെട്ടിരുന്നെന്നും അതെഴുതിയ രേഖ എന്റെ തലയിണക്കടിയിലുണ്ടായിരുന്നെന്നും ദൈവദൂതന്‍ മരണപ്പെട്ടപ്പോള്‍, ഞങ്ങളെല്ലാം അതുമായി ബന്ധപ്പെട്ട തിരക്കുകള്‍ക്കടിയിലായിരുന്ന സന്ദര്‍ഭത്തില്‍ ഒരു ആട് അകത്ത് കടന്ന് അത് തിന്നുകളഞ്ഞു എന്നും ആഇശയില്‍ നിന്ന് സുനനു ഇബ്‌നുമാജയും മുസ്‌നദ് അഹ്‌മദും ഉദ്ദരിച്ചിട്ടുണ്ട്. സ്ത്രീകളെക്കൊണ്ട് പരപുരുഷന്മാര്‍ക്ക് മുലയൂട്ടുന്ന നടപടിയാണോ ഇസ്‌ലാം പഠിപ്പിച്ച ധാര്‍മ്മികത?

അസത്യങ്ങളും അര്‍ദ്ധസത്യങ്ങളും കോര്‍ത്തിണക്കി നെയ്‌തെടുത്ത ഒരു വ്യാജപ്രചരണമാണിത്. തെറിവിളികള്‍ക്കും പരമതനിന്ദക്കും ‘വിമര്‍ശനം’ എന്നുപേരിട്ട് വൈജ്ഞാനിക രംഗത്ത് കൃത്രിമ മേല്‍വിലാസമുണ്ടാക്കി വെറുപ്പുകച്ചവടം നടത്താനുള്ള കുടില വ്യഗ്രത മാത്രമാണ് പ്രസ്തുത ആരോപണങ്ങള്‍ക്കു പിന്നിലുള്ളത്. ‘ദൈവസ്‌നേഹം’ നെറ്റിയിലൊട്ടിച്ച് പരമത വിദ്വേഷം ഹൃദയത്തില്‍ ഒളിപ്പിച്ചുവെച്ച ചില മിഷണറി നുണ ഫാക്ടറികള്‍ നിര്‍മ്മിച്ച ഈ നബിനിന്ദാ പ്രചരണം, യുക്തിവാദികളും ഫെമിനിസ്റ്റുകളും ഫാഷിസ്റ്റുകളുമെല്ലാം മൊത്തമായും ചില്ലറയായും വിപണനം ചെയ്തുകൊണ്ടിരിക്കുകയാണ്. അതുകൊണ്ടുതന്നെ പ്രസ്തുത ആരോപണം ഇഴയടര്‍ത്തി പരിശോധിക്കേണ്ടിയിരിക്കുന്നു.

അറബികള്‍ക്കിടയില്‍ ബന്ധം സ്ഥാപിതമാകാനുള്ള കാരണങ്ങള്‍ നാലെണ്ണമായിരുന്നു; പ്രസവം, വിവാഹം, മുലകുടി, ദത്തെടുക്കല്‍. ഇതില്‍ ദത്തെടുക്കല്‍, ബന്ധം സ്ഥാപിതമാകാനുള്ള കാരണങ്ങളില്‍ നിന്നും ഇസ്‌ലാം ഒഴിവാക്കിയപ്പോഴാണ് സാലിമിന്റെ(റ) വിഷയത്തിലുള്ള പ്രശ്‌നം ഉണ്ടാകുന്നത്. അഥവാ അബൂഹുദൈഫഃ(റ)യുടെയും സഹ്‌ല(റ)യുടെയും ദത്തുപുത്രനായിരുന്നു സാലിം (റ). ഇസ്‌ലാം ദത്തുപുത്ര സമ്പ്രദായത്തിനു അറുതിവരുത്തിയപ്പോള്‍ ഇന്നലെവരെ തന്റെ മകനായി ജീവിച്ച സാലിം, സഹ്‌ല എന്ന വളര്‍ത്തു മാതാവിന് അന്യപുരുഷനായി മാറി. ഇതില്‍ തീവ്ര ദുഃഖമറിയിച്ചുകൊണ്ട് അവര്‍ പ്രവാചകനെ(സ) സമീപിക്കുകയുണ്ടായി. പ്രസ്തുത ചരിത്ര നിമിഷങ്ങള്‍ ഹദീഥ് ഗ്രന്ഥങ്ങളില്‍ നിന്നും നമുക്കു വായിച്ചെടുക്കാം. ആഇശ (റ) നിവേദനം: സുഹൈലിന്റെ മകള്‍ സഹ്‌ല ഒരിക്കല്‍ നബി(സ)യുടെ അരികില്‍ വന്നുപറഞ്ഞു. ‘അല്ലാഹുവിന്റെ ദൂതരേ, സാലിം എന്റെ അടുത്ത് പ്രവേശിക്കുന്നതില്‍ അബൂഹുദൈഫഃയുടെ (സഹ്‌ലയുടെ ഭര്‍ത്താവാണ് അബൂഹുദൈഫഃ) മുഖത്ത് വെറുപ്പുള്ളതായി തോന്നുന്നു. അപ്പോള്‍ നബി (സ) പറഞ്ഞു: ‘അയാള്‍ക്കു നീ മുലപ്പാല്‍ കൊടുക്കുക’. അവള്‍ ചോദിച്ചു ‘അയാള്‍ വലിയ മനുഷ്യനാണല്ലോ എങ്ങനെ ഞാന്‍ മുലപ്പാല്‍ കൊടുക്കും?!’. അപ്പോള്‍ നബി (സ) പുഞ്ചിരിച്ചുകൊണ്ട് പറഞ്ഞു: ‘അയാള്‍ വലിയ മനുഷ്യനാണെന്നു എനിക്കറിയാം. (സ്വഹീഹു മുസ്‌ലിം 1453)

മുസ്‌ലിമിന്റെ തന്നെ മറ്റൊരു നിവേദനം ഇപ്രകാരമാണ്. ”അബൂഹുദൈഫഃ(റ)യുടെ അടിമ സാലിം (റ), അബൂഹുദൈഫഃയുടെയും കുടുംബത്തിന്റെയും കൂടെ അവരുടെ വീട്ടിലായിരുന്നു താമസം. സുഹൈലിന്റെ മകള്‍ നബി(സ)യുടെ അടുത്ത് വന്നു പറഞ്ഞു: ‘തീര്‍ച്ചയായും സാലിമിന്നു സാധാരണ പുരുഷന്മാരാകുന്ന നിലയില്‍ ബുദ്ധിയും പ്രായവും തികഞ്ഞിട്ടുണ്ട്. അവന്‍ ഞങ്ങളുടെ അടുത്ത് കടന്നുവരാറുണ്ട്. അബൂഹുദൈഫഃയുടെ മനസ്സില്‍ അതില്‍ വെറുപ്പുള്ളതായി എനിക്ക് തോന്നുന്നു’. അപ്പോള്‍ നബി (സ) പറഞ്ഞു: ‘നീ അയാള്‍ക്ക് മുലപ്പാല്‍ കൊടുക്കുക. എന്നാല്‍ അയാള്‍ അടുത്ത ബന്ധുവായിത്തീരും. അബൂഹുദൈഫഃയുടെ മനസ്സില്‍ വെറുപ്പും ഇല്ലാതാകും’. അവള്‍ വീണ്ടും നബി(സ)യുടെ അരികെ വന്നു പറഞ്ഞു: ‘ഞാന്‍ അയാള്‍ക്ക് മുലപ്പാല്‍ കൊടുത്തു. അങ്ങനെ അബൂഹുദൈഫഃയുടെ മനസ്സില്‍ വെറുപ്പ് ഇല്ലാതാകുകയും ചെയ്തു’. (സ്വഹീഹു മുസ്‌ലിം)

ഈ ഹദീഥുകള്‍ മനസ്സിലാക്കിത്തരുന്ന കാര്യങ്ങളെ ഇങ്ങനെ സംഗ്രഹിക്കാം

1) അബൂഹുദൈഫഃ(റ)യുടെ മോചിത അടിമയായ സാലിമി(റ)നെ നന്നെ ചെറുപ്പത്തില്‍ തന്നെ അബൂഹുദൈഫ-സഹ്‌ല ദമ്പതിമാര്‍ ദത്തുപുത്രനായി സ്വീകരിക്കുകയും സ്വപുത്രന്റെ സ്ഥാനം നല്‍കി വളര്‍ത്തുകയും ചെയ്തു. 2) ഇസ്‌ലാം ദത്തുപുത്രസമ്പ്രദായം അവസാനിപ്പിച്ചതോടെ, മതപരമായി സാലിം(റ) ആ കുടുംബത്തിന് മകനല്ലാതായിത്തീരുകയും, വളര്‍ത്തു മാതാവാണെങ്കിലും സഹ്‌ല(റ)യെ സംബന്ധിച്ച് സാലിമി(റ)നുമേല്‍ അന്യപുരുഷന്റെ വിധി ബാധകമാവുകയും ചെയ്തു. 3) വളര്‍ത്തുപുത്രനാണെങ്കിലും മതപരമായി അന്യപുരുഷന്റെ വിധി ബാധകമായ ഒരാള്‍, തന്റെ ഭാര്യയെ സന്ദര്‍ശിക്കുന്നതും അവരുടെ അടുക്കല്‍ പ്രവേശിക്കുന്നതും ഭര്‍ത്താവായ അബൂഹുദൈഫഃ(റ)യില്‍ വെറുപ്പുളവാക്കി. 4) മകന്റെ സ്ഥാനം നല്‍കി താന്‍ വളര്‍ത്തിയ ഒരാളെ ഒരു നിമിഷം കൊണ്ട് അന്യനെപ്പോലെ ഒഴിവാക്കുവാന്‍ സഹ്‌ല(റ)യുടെ മാതൃഹൃദയം അവരെ അനുവദിച്ചില്ല. അതിനാല്‍ തങ്ങളുടെ കാര്യത്തില്‍ ഒരു പോംവഴി തേടിക്കൊണ്ട് നീറുന്ന ഹൃദയവുമായി അവള്‍ പ്രവാചക(സ)ന്റെ അരികിലെത്തി. 5) അവരുടെ പ്രശ്‌നത്തിനു പരിഹാരമായി ‘സാലിമിന് മുലപ്പാല്‍ നല്‍കാനും അങ്ങനെ സ്വപുത്രനെ പോലെ സ്വീകരിക്കുവാനും’ പ്രവാചകന്‍ (സ) അവര്‍ക്ക് ഇളവുനല്‍കി. അതോടെ അബൂഹുദൈഫഃ(റ)യുടെ മനസ്സില്‍നിന്നും ആ വെറുപ്പ് ഇല്ലാതാവുകയും ചെയ്തു.

സാലിം എന്ന മുതിര്‍ന്ന ആണ്‍കുട്ടിക്ക് മുലപ്പാല്‍ നല്‍കി മകനായി സ്വീകരിക്കുവാന്‍ സഹ്‌ലയോട് നിര്‍ദ്ദേശിച്ചതാണ്, പ്രവാചകനെ പെണ്‍വിരുദ്ധനും സ്ത്രീകളെ അശ്ലീലതകള്‍ക്കു പ്രേരിപ്പിച്ച വ്യക്തിത്വമായും മുദ്ര കുത്താന്‍ വിമര്‍ശകരെ പ്രേരിപ്പിക്കുന്ന ഘടകം. വസ്തുതകള്‍ പലതും മറച്ചുപിടിച്ചുകൊണ്ടാണ് ഈ വിമര്‍ശനം നെയ്‌തെടുത്തിരിക്കുന്നത്. ‘മുലപ്പാല്‍ നല്‍കുക’ എന്നു പറഞ്ഞതിനെ ‘പാല്‍ കുടിക്കാന്‍ സ്തനം നല്‍കുക’ എന്ന്, അശ്ലീല ഭാവനകള്‍ കൊണ്ട് ലൈംഗിക ഛായം പൂശുകയാണ് വാസ്തവത്തില്‍ വിമര്‍ശകന്മാര്‍ ചെയ്തിരിക്കുന്നത്. ‘ബാബു രിദ്വാഅത്തില്‍ കബീരി’ അഥവാ വലിയവരുടെ രിദ്വാഅഃ (മുലപ്പാല്‍ കുടിക്കല്‍) എന്ന അദ്ധ്യായങ്ങള്‍ക്കു കീഴിലാണ് പ്രസ്തുത ഹദീഥുകല്‍ ഉദ്ദരിക്കപ്പെട്ടിരിക്കുന്നത്. രിദ്വാഅഃ (الرضاعة) എന്ന പദം സ്ത്രീയുടെ മുലപ്പാല്‍ സ്തനത്തില്‍ നിന്ന് നേരിട്ട് ഈമ്പിക്കുടിക്കുന്ന രീതിക്കു മാത്രമാണോ അറബി ഭാഷയില്‍ ഉപയോഗിക്കുന്നത്? അല്ല എന്നതാണ് വസ്തുത. പക്ഷേ ആ വസ്തുതകളൊന്നും കാണാന്‍ വിമര്‍ശകര്‍ തയ്യാറല്ല. തയ്യാറായാല്‍ ഈ അശ്ലീല ഭാവനകള്‍ക്കൊന്നും നിലനില്‍പ്പുണ്ടാവില്ലെന്ന് അവര്‍ക്ക് നല്ലപോലെ അറിയാം.

എന്താണ് രിദ്വാഅഃ? (الرضاعة)

ഒരു സ്ത്രീയുടെ മുലപ്പാല്‍ മറ്റൊരാളുടെ അല്ലെങ്കില്‍ കുഞ്ഞിന്റെ വയറ്റില്‍ എത്തുക എന്നതിനെയാണ് ഇര്‍ദ്വാഅ് (الارضاع), രിദ്വാഅഃ (الرضاعة) എന്നൊക്കെ പറയുന്നത്. അതാകട്ടെ പല രീതിയിലും നടക്കാം. സ്ത്രീയുടെ മുലപ്പാല്‍ സ്തനത്തില്‍ നിന്ന് നേരിട്ട് ഈമ്പിക്കുടിക്കുന്ന രീതി അഥവാ ‘മസ്വ്’ (المص), സ്തനം സ്പര്‍ശിക്കാതെ കുട്ടിയുടെ വായിലേക്ക് പാല്‍ ചുരന്ന് ഒഴുക്കുന്ന രീതി അഥവാ ‘സ്വബ്ബ്’ (الصب), സ്തനം സ്പര്‍ശിക്കാതെ സ്ത്രീയുടെ മുലപ്പാല്‍ പിഴിഞ്ഞെടുത്ത് തളികയിലാക്കി കുട്ടിയുടെ തൊണ്ടയിലേക്ക് ചൊരിയുന്ന രീതി അഥവാ ‘വുജൂര്‍’ (الوجور), സ്തനം സ്പര്‍ശിക്കാതെ സ്ത്രീയുടെ മുലപ്പാല്‍ പിഴിഞ്ഞെടുത്ത് തളികയിലാക്കി കുട്ടിയുടെ നാസദ്വാരത്തില്‍ ഒഴിക്കുന്ന രീതി അഥവാ ‘സുഊത്വ്’ (السعوط), സ്തനം സ്പര്‍ശിക്കാതെ സ്ത്രീയുടെ മുലപ്പാല്‍ പൈപ്പിലൂടെയോ, സിറിഞ്ചിലൂടെയോ കുട്ടിക്കു നല്‍കുന്ന രീതി അഥവാ ‘ഹിക്‌നത്ത്’ (الحقنة). ഈ ഏത് രീതിയിലും മുലപ്പാല്‍ നല്‍കുന്നതിനും ഇസ്‌ലാമിലെ സാങ്കേതിക പദവും കര്‍മ്മശാസ്ത സംജ്ഞ്യയുമായ ‘രിദ്വാഅഃ’ എന്ന് പറയുമെന്ന് പൗരാണികരായ അറബി ഭാഷാ പണ്ഡിതരും സര്‍വ്വ മദ്ഹബുകളിലും പെട്ട കര്‍മ്മശാസ്ത്ര വിശാരദന്മാരും വ്യക്തമാക്കിയിട്ടുണ്ട്. (മുഖ്തസറു ഖലീല്‍: 162, മവാഹിബുല്‍ ജലീല്‍: 4/178, ശര്‍ഹുല്‍ കബീര്‍: 2/502, മന്‍ഹുല്‍ ജലീല്‍: 4/371, അല്‍ ഫവാകിഹുദ്ദവാനി: 2/54, മുഗ്നി അല്‍ മുഹ്താജ്: 3/414, ഫത്ഹുല്‍ വഹാബ്: 2/194, അസ്സിറാജുല്‍ വഹ്ഹാജ്: 460, അല്‍ മുബ്ദിഅ് 8/160, ശര്‍ഹു മുന്‍തഹല്‍ ഇറാദത്ത്: 3/213, കശ്ശാഫുല്‍ കനാഅ്- 5: 442, മതാലിബു ഉലിന്നുഹ- 5: 596, അല്‍ ബഹ്‌റു റാഇഖ്- 3: 238, മജ്മഉല്‍ അന്‍ഹുര്‍ : 1/551) (ഉദ്ദരണം: ഡോ. രിയാദ് മിശ്അലിന്റെ ലേഖനത്തില്‍ നിന്ന് Midad Al-Adab Refereed Quarterly journal : Vol.1 : Issue 1 : Article 10, ഇറാക് സര്‍വ്വകലാശാല)

പൗരാണിക ഭാഷാ പണ്ഡിതനായ ഇബ്‌നു മന്‍ളൂര്‍ (ജനനം: ഹിജ്‌റ 630) തന്റെ അറബി നിഘണ്ഡുവായ ‘ലിസാനുല്‍ അറബി’ല്‍ (2/92) പറഞ്ഞു: ”സ്തനത്തില്‍ നിന്ന് നേരിട്ട് മുലയൂട്ടുന്നത് വിവാഹബന്ധം നിഷിദ്ധമാക്കുന്നത് പോലെ തന്നെ സ്ത്രീയുടെ സ്തനത്തില്‍ നിന്ന് നേരിട്ടല്ലാതെ പാല്‍ ചുരന്നെടുത്ത് കുട്ടിയെ കുടിപ്പിക്കുന്നതും (രിദ്വാഅഃ ആയതിനാല്‍) വിവാഹബന്ധം നിഷിദ്ധമാക്കുന്നതാണ്. സ്തനത്തില്‍ നിന്ന് വേറിട്ടതിനാല്‍ അത് രിദ്വാഅഃ ആകാതിരിക്കുന്നില്ല.”

അപ്പോള്‍ ഭാഷാപരമായി പോലും രിദ്വാഅഃ എന്നു പറഞ്ഞാല്‍, സ്ത്രീയുടെ മുലപ്പാല്‍ സ്തനത്തില്‍ നിന്ന് നേരിട്ട് ഈമ്പി കുടിക്കുന്ന രീതിയാകണം എന്നില്ല. പ്രവാചകന്‍ (സ) സഹ്‌ലയോട് പറഞ്ഞതാകട്ടെ ‘അവന് മുലപ്പാല്‍ നല്‍കുക’ അര്‍ദ്വഈഹി (ارضعيه) എന്നാണ്. അല്ലാതെ നിന്റെ സ്തനം അവന് കുടിക്കാന്‍ കൊടുക്കുക എന്നല്ല. സഹ്‌ല-സാലിം സംഭവം കേവലം ഇസ്‌ലാംവിരോധികളുടെ മ്ലേഛമായ ഭാവനാവിലാസങ്ങള്‍ക്കപ്പുറം, യാതൊരു ആധികാരികതയുമില്ലാത്ത ദുരാരോപണം മാത്രമാണെന്ന് വ്യക്തമാക്കുന്ന തെളിവുകളാണ്, പൂര്‍വ്വകാലം മുതല്‍ മുസ്‌ലിം പണ്ഡിതന്മാര്‍ പ്രസ്തുത ഹദീഥുകള്‍ക്കു നല്‍കിയ വ്യാഖ്യാനം. അതിലൂടെ നമുക്കൊന്ന് കണ്ണോടിക്കാം.

ഇമാം ഇബ്‌നു കുതൈബ (ജനനം: ഹിജ്‌റ: 213) പറഞ്ഞു: ”പ്രവാചകന്‍ (സ) സഹ്‌ലയോട് പറഞ്ഞത് ‘അവന് മുലപ്പാല്‍ നല്‍കുക'(ارضعيه) എന്നാണ്. നിന്റെ സ്തനം അവന് കുടിക്കാന്‍ കൊടുക്കുക എന്ന് (പ്രവാചകന്‍) ഊദ്ദേശിച്ചിട്ടേ ഇല്ല. നിന്റെ പാല്‍ ചുരന്നെടുത്ത് തളികയിലാക്കി കുറച്ച് അവനെ കുടിപ്പിക്കുക എന്നേ അദ്ദേഹം ഉദ്ദേശിച്ചിട്ടുള്ളൂ. അതല്ലാതെ സ്തനത്തില്‍ നിന്ന് നേരിട്ട് കുടിക്കല്‍ അനുവദനീയമല്ല. കാരണം സഹ്‌ലയുടെ സ്തനത്തിലേക്ക് നോക്കുന്നത് തന്നെ സാലിമിന് നിഷിദ്ധമാണ്. പിന്നെയെങ്ങനെ നിഷിദ്ധമായ ഒരു കാര്യം പ്രവാചകന്‍ അദ്ദേഹത്തിന് അനുവദിച്ച് കൊടുക്കും!” (തഅ്‌വീലു മുക്തലിഫില്‍ ഹദീഥ്: 1/437) ഇതേ വ്യാഖ്യാനം തന്നെ ഇമാം ക്വാള്വീ ഇയാള്വും (ജനനം: ഹിജ്‌റ: 476) ആവര്‍ത്തിക്കുന്നതു കാണാം (ശര്‍ഹുന്നവവി അലാ സ്വഹീഹു മുസ്‌ലിം : 10/30,31)

ഇമാം മാവര്‍ദി (ജനനം : ഹിജ്‌റ : 364) പറയുന്നു: ”സാലിമിന്റെ വിഷയത്തില്‍ സ്തനത്തില്‍ നിന്ന് മുലപ്പാല്‍ ഊട്ടിയെന്ന പ്രസ്താവന ഒന്നും തന്നെയില്ല എന്നത് സുവിദിതമാണ്. അതുകൊണ്ട് തന്നെ പ്രവാചകന്‍ ഉദ്ദേശിച്ചത് ‘വുജൂര്‍’ (الوجور സ്തനം സ്പര്‍ശിക്കാതെ സ്ത്രീയുടെ മുലപ്പാല്‍ പിഴിഞ്ഞെടുത്ത് തളികയിലാക്കി കുട്ടിയുടെ തൊണ്ടയില്‍ ചൊരിയുന്ന രീതി) ആണെന്ന് സ്ഥാപിതമാകുന്നു” (അല്‍ ഹാവി അല്‍അല്‍ കബീര്‍ : 11/372)

ഇബ്‌നു അബ്ദുല്‍ ബിര്‍റ് (ജനനം: ഹിജ്‌റ: 368) പറഞ്ഞു: ”മുതിര്‍ന്നവരുടെ മുലകുടി എന്നാല്‍ പാല്‍ പാത്രത്തില്‍ ചുരന്ന് അതില്‍ നിന്ന് കുടിപ്പിക്കലാണ്. എന്നാല്‍ കുട്ടികള്‍ക്ക് നല്‍കുന്നതുപോലെ സ്തനത്തില്‍ നിന്ന് നേരിട്ട് കുടിപ്പിക്കല്‍ മുതിര്‍ന്നവരുടെ കാര്യത്തില്‍ നിഷിദ്ധമാണ് എന്ന് പണ്ഡിതസംഘം വ്യക്തമാക്കിയിട്ടുണ്ട്” (അത്തംഹീദ്: 8/257, അല്‍ ഇസ്തിദ്കാര്‍: 6/255)

ആധുനിക കാലഘട്ടത്തിലെ മുസ്‌ലിം പണ്ഡിതന്മാര്‍ ‘മുലയൂട്ടല്‍’ വിവാദവുമായി ബന്ധപ്പെട്ട ആരോപണങ്ങള്‍ക്ക് മറുപടി പറയുമ്പോള്‍ അതെല്ലാം ‘വ്യാഖ്യാന ഫാക്ടറികള്‍’ എന്നു പറഞ്ഞ് ഒഴിഞ്ഞുമാറുന്ന ഇസ്‌ലാംവിരോധികള്‍ക്ക്, നൂറ്റാണ്ടുകള്‍ക്കു മുമ്പേ പൂര്‍വ്വകാല മുസ്‌ലിം പണ്ഡിതന്മാര്‍ പ്രസ്തുത വിഷയത്തില്‍ നല്‍കിയ ഈ മറുപടികളെപ്പറ്റി എന്തു പറയാനുണ്ട്?!. മിഷണറി നുണഫാക്ടറികളില്‍ ‘മുലയൂട്ടല്‍’ വിവാദം നിര്‍മ്മിച്ചെടുക്കപ്പെടുന്നതിനും എത്രയോ നൂറ്റാണ്ടുകള്‍ക്ക് മുമ്പാണ് ഇമാം ഇബ്‌നു കുതൈബയും, ഇമാം ക്വാള്വീ ഇയാള്വും, ഇമാം മാവര്‍ദിയും, ഇബ്‌നു അബ്ദുല്‍ ബിര്‍റുമെക്കെ ഈ വിവാദങ്ങള്‍ക്ക് മറുപടി പറഞ്ഞതെന്ന വസ്തുതയെങ്കിലും ഇസ്‌ലാം വിരോധം കൊണ്ട് അന്ധത ബാധിച്ചവരുടെ കണ്ണുകള്‍ തുറപ്പിച്ചിരുന്നെങ്കില്‍!! പണ്ഡിത വ്യാഖ്യാനങ്ങള്‍ മാത്രമല്ല ചരിത്ര രേഖകളും പ്രസ്തുത ‘മുലപ്പാല്‍ നല്‍കല്‍’ അപ്രകാരമായിരുന്നെന്ന് വ്യക്തമാക്കുന്നുണ്ട്. ‘ഒരു പാത്രത്തില്‍ മുലപ്പാല്‍ ചുരന്നൊഴിച്ച് എല്ലാ ദിവസവും സാലിം കുടിച്ചു കൊണ്ടിരുന്നുവെന്നും അതിനു ശേഷമാണ് സഹ്‌ലയുടെ വീട്ടില്‍ പ്രവേശിച്ചത്’ എന്നും ചില ചരിത്ര നിവേദനങ്ങള്‍ കാണാം. (ത്വബകാത്തു ഇബ്‌നു സഅ്ദ്: 8/271, ശര്‍ഹു സര്‍ക്കാനി : 3/316)

അശ്ലീല ഭാവനകള്‍ കൊണ്ട് പ്രവാചക ജീവിതത്തിനുമേല്‍ ലൈംഗിക ഛായം പൂശുവാന്‍ വെമ്പല്‍ കൊള്ളുന്ന ഇസ്‌ലാംവിരോധികള്‍ നബി(സ)യില്‍ നിന്നും നിവേദനം ചെയ്യപ്പെട്ട ഈ ഹദീഥെങ്കിലും ഒന്നു പരിഗണിച്ചിരുന്നെങ്കില്‍ അവരുടെ വായകൊണ്ട് ഇത്ര ഗുരുതരമായ അപവാദം പറയാന്‍ ഒന്നു മടിക്കുമായിരുന്നു. നബി(സ) പറഞ്ഞു: ”നിന്റെ തലയില്‍ ഇരുമ്പിന്റെ സൂചികൊണ്ട് കുത്തി തറക്കുന്നതാണ് അനുവദനീയമല്ലാത്ത ഒരു സ്ത്രീയെ സ്പര്‍ശിക്കുന്നതിനേക്കാള്‍ നിനക്ക് നല്ലത്” (അല്‍ മുഅ്ജമുല്‍ കബീര്‍: ത്വബ്‌റാനി: 20:211) അന്യസ്ത്രീകളെ സ്പര്‍ശിക്കുന്നതിനേക്കാള്‍ നല്ലത് തലയില്‍ ഇരുമ്പുസൂചി കുത്തി തറക്കുന്നതാണെന്നു പഠിപ്പിച്ച ഒരു വിശുദ്ധ വ്യക്തിത്വത്തിനുമേല്‍, പരപുരുഷന്മാരെ മുലയൂട്ടാന്‍ സ്ത്രീകളെ പ്രേരിപ്പിച്ചെന്ന അപവാദം പറഞ്ഞു പ്രചരിപ്പിച്ചവര്‍ ദൈവത്തിന്റെ ‘കുഞ്ഞാടാ’വാന്‍ എന്തുകൊണ്ടും യോഗ്യരാണ്?!.

വിമര്‍ശകരുടെ കാപട്യം

വിമര്‍ശനവിധേയമായ ഹദീഥുകളുമായി ബന്ധപ്പെട്ട് മ്ലേച്ഛഭാവനകളുടെ മേച്ചില്‍പ്പുറങ്ങള്‍ തേടിയിറങ്ങിയ ഇസ്‌ലാംവിരോധികള്‍ ബോധപൂര്‍വ്വം മറച്ചുപിടിച്ച ചില വസ്തുതകള്‍ കൂടി നാം തിരിച്ചറിയേണ്ടിയിരിക്കുന്നു. 1) ഹദീഥുകളില്‍ പരാമര്‍ശിക്കപ്പെട്ട ഈ ഇളവ് കേവലം സാലിമിന്റെ വിഷയത്തില്‍ മാത്രം പരിമിതമാണ്. അതൊരു പൊതുവിധിയല്ലെന്നര്‍ത്ഥം. അതിനാല്‍ സഹ്‌ലയല്ലാത്ത ഒരു മുസ്‌ലിം സ്ത്രീക്കും ആ ഇളവ് ബാധകമല്ല. ‘മുഹമ്മദ് നബി(സ)യുടെ ഈ അദ്ധ്യാപനം പിന്‍പറ്റാന്‍ മാന്യതയുള്ള മുസ്‌ലിം സ്ത്രീകള്‍ക്കു സാധിക്കുമോ’ എന്നു ചോദിക്കുന്നവരുടെ ആശങ്കകള്‍ അടിസ്ഥാന രഹിതമാണ്. സ്വഹീഹു മുസ്‌ലിമില്‍ തന്നെ ആ ഇളവ് സാലിമിന് മാത്രമുള്ളതാണെന്ന് വ്യക്തമാക്കുന്ന നിവേദം കാണാവുന്നതാണ്. പ്രവാചക പത്‌നി ഉമ്മുസലമ (റ) പറയാറുണ്ടായിരുന്നു: ”ആ നിലക്ക് മുലകുടി ബന്ധത്തിലുള്ള (രണ്ടു വയസ്സിനുള്ളില്‍ മുലകുടി ബന്ധം സ്ഥാപിതമായിട്ടില്ലാത്ത) ഒരാളും നബി (സ) യുടെ ഭാര്യമാരുടെ അരികെ പ്രവേശിക്കുന്നതിനെ അവര്‍ വിസമ്മതിച്ചിരുന്നു. അവര്‍ ആഇശ(റ)യോട് പറഞ്ഞു: ‘ഇത് സാലിമിന്നു മാത്രമായി റസൂല്‍ (സ) പറഞ്ഞ ഒരു ഇളവാകുന്നു. അതിനാല്‍ ആ നിലക്ക് മുലകുടി ബന്ധമുള്ള ഒരാളും നമ്മുടെ അടുത്ത് പ്രവേശിക്കേണ്ടതില്ല. നമ്മളെ കാണുകയും ചെയ്യേണ്ട.” (സ്വഹീഹു മുസ്‌ലിം: 1454, മുസ്‌നദു അഹ്‌മദ്: 26660, സുനനു അബൂദാവൂദ്: 2061)

2) തന്റെ ഭാര്യയുടെ അടുത്ത് വളര്‍ത്തുപുത്രന്‍ സാലിം പ്രവേശിക്കുന്നത്- ദത്തുപുത്ര സമ്പ്രദായം അവസാനിപ്പിക്കപ്പെട്ടതോടെ- മതപരമായി അനുവദനീയമല്ലാതായി. അതുകൊണ്ടാണ് അബൂഹുദൈഫഃക്ക് സാലിമിന്റെ പ്രവേശനം നീരസമുള്ളതായി മാറിയത്. ആ നീരസം മാറാന്‍ വേണ്ടിയാണ് മുലപ്പാല്‍ നല്‍കി മാതൃബന്ധം സ്ഥാപിക്കുവാന്‍ പ്രവാചകന്‍ (സ) ഉപദേശിച്ചത്. അതേസമയം സ്തനങ്ങളില്‍ നിന്ന് നേരിട്ട് കുടിക്കാനായിരുന്നു നബി(സ)യുടെ ഉപദേശമെങ്കില്‍ അത് അബൂഹുദൈഫഃയുടെ നീരസവും മനഃപ്രയാസവും അധികരിപ്പിക്കുകയാണുണ്ടാവുക. ഈ വസ്തുത വിമര്‍ശകര്‍ ബോധപൂര്‍വ്വം മറച്ചുപിടിക്കുകയാണ്. പ്രവേശനം തന്നെ നീരസവും മനഃപ്രയാസവും സൃഷ്ടിക്കുന്ന ഒരാളെ കൊണ്ട് തന്നെ, സ്തനങ്ങളില്‍ നിന്ന് നേരിട്ട് മുലപ്പാല്‍ കുടിക്കുവാന്‍ കല്‍പ്പിക്കില്ലെന്ന് ഏതു വിഡ്ഢിക്കാണ് മനസ്സിലാകാതിരിക്കുക?!

‘മുലയൂട്ടല്‍’ ആഇശ(റ)യുടെ പേരിലെ പച്ചക്കള്ളങ്ങള്‍

അശ്ലീല ഭാവനകള്‍ കൊണ്ട് ഹദീഥുകളെ ദുഷിപ്പിച്ചവതരിപ്പിക്കുക മാത്രമല്ല, പച്ചകള്ളങ്ങള്‍ പോലും ഇസ്‌ലാമിനെതിരെ പടച്ചുണ്ടാക്കാന്‍ വരെ മടിയില്ലാതായിരിക്കുന്നു ഇസ്‌ലാംവിമര്‍ശകര്‍ക്ക്. ലജ്ജാവഹവും നിന്ദാപരവുമായ ഇത്തരം ഏര്‍പ്പാടുകളെ ‘സുവിശേഷ വേല’ എന്നുപേരിട്ടു വിളിക്കുന്നത് അല്പം കടന്ന കയ്യാണ്. ‘മുലയൂട്ടല്‍’ വിവാദവുമായി ബന്ധപ്പെട്ട് പ്രവാചക പത്‌നി ആഇശ(റ)ക്കെതിരെ മിഷണറി നുണഫാക്ടറികള്‍ ഉല്‍പ്പാദിപ്പിച്ച അത്തരം രണ്ടു കളവുകള്‍ നാം കാണുക.

‘പ്രവാചക പത്‌നി ആഇശ (റ) അത്തരത്തില്‍ പരപുരുഷന്മാരെ മുലയൂട്ടാറുണ്ടായിരുന്നുവെന്നും പലപ്പോഴും ആഇശയുടെ അരികില്‍ പരപുരുഷന്മാരെ കാണുക വഴി നബി(സ)ക്കുതന്നെ അതില്‍ അനിഷ്ടമുണ്ടാവുകയും ചെയ്തിരുന്നു എന്ന് സ്വഹീഹു മുസ്‌ലിമിലെ 1455-ാം ഹദീഥായി രേഖപ്പെടുത്തിയിട്ടുണ്ട്’. ഇതാണ് സോഷ്യല്‍ മീഡിയ വഴി ചില മിഷണറി നുണ ഫാക്ടറികള്‍ നിരന്തരം പ്രചരിപ്പിച്ചതും, യുക്തിവാദികളും ഫെമിനിസ്റ്റുകളും ഫാഷിസ്റ്റുകളുമടങ്ങുന്ന ഇസ്‌ലാംവിരോധികള്‍ ഏറ്റെടുത്തതുമായ ഒന്നാമത്തെ കളവ്. സ്വഹീഹു മുസ്‌ലിമിലെ ആ നിവേദനം നാം ആദ്യം കാണുക. ”മസ്‌റൂഖ് (റ) നിവേദനം. ആഇശ (റ) പറഞ്ഞതായി അദ്ദേഹം പറഞ്ഞു: റസൂല്‍ (സ) എന്റെ അടുത്ത് കടന്നുവന്നു. എന്റെ അരികെ ഒരാള്‍ ഇരിക്കുന്നുണ്ടായിരുന്നു? അത് നബി(സ)ക്ക് അതിയായ പ്രയാസമുണ്ടാക്കി. നബി(സ)യുടെ മുഖത്ത് ഞാന്‍ കോപം കണ്ടു. അപ്പോള്‍ ഞാന്‍ ചോദിച്ചു: ‘അല്ലാഹുവിന്റെ പ്രവാചകരെ, നിശ്ചയം ഇദ്ദേഹം മുലകുടി ബന്ധത്തില്‍ എന്റെ സഹോദരനാകുന്നു’. അപ്പോള്‍ നബി (സ) പറഞ്ഞു: നിങ്ങള്‍ മുലകുടി ബന്ധത്തിലുള്ള സഹോദരന്മാരെപ്പറ്റി ശരിക്കും നോക്കണം. വിശപ്പടങ്ങുന്ന നിലക്ക് മുല കുടിച്ചാല്‍ മാത്രമെ മുലകുടി ബന്ധം ഉണ്ടാവുകയുള്ളൂ”. (സ്വഹീഹു മുസ്‌ലിം: 1455) ഈ സംഭവത്തെ ദുര്‍വ്യാഖ്യാനിച്ചുകൊണ്ടാണ് ആഇശ (റ) പരപുരുഷന്മാരെ മുലയൂട്ടിയിരുന്നെന്ന് പച്ചകള്ളം ഇവര്‍ പ്രചരിപ്പിച്ചത്. ഹദീഥുകള്‍ നേര്‍ക്കുനേരെ ഒരാവര്‍ത്തി വായിച്ചാല്‍ തന്നെ അതൊരു ദുരാരോപണം മാത്രമാണെന്ന് മനസ്സിലാക്കാന്‍ കഴിയും. ‘അല്ലാഹുവിന്റെ പ്രവാചകരേ, നിശ്ചയം ഇദ്ദേഹം മുലകുടി ബന്ധത്തില്‍ എന്റെ സഹോദരനാകുന്നു’ എന്ന ആഇശ(റ)യുടെ വാക്കുകള്‍ സ്പഷ്ടമായും മനസ്സിലാക്കി തരുന്നത്; ആഇശ(റ)യും അവരുടെ സമീപത്തിരുന്ന വ്യക്തിയും കുഞ്ഞായിരുന്നപ്പോള്‍ ഒരേ സ്ത്രീയുടെ മുലപ്പാല്‍ കുടിച്ചിട്ടുണ്ട് എന്നതാണ്. അല്ലാതെ ആഇശ (റ) അദ്ദേഹത്തെ ‘മുലയൂട്ടി’ എന്നല്ല. ഒന്നുകില്‍ ആഇശ (റ) കൈകുഞ്ഞായിരിക്കുമ്പോള്‍ അദ്ദേഹത്തിന്റെ മാതാവിന്റെ മുലപ്പാല്‍ കുടിച്ചിട്ടുണ്ടാകും. അല്ലെങ്കില്‍ അദ്ദേഹം ശൈശവ ഘട്ടത്തില്‍ ആഇശ(റ)യുടെ മാതാവിന്റെ പാല്‍കുടിച്ചതുമാകാം. ഇതുരണ്ടുമല്ലെങ്കില്‍ അവര്‍ രണ്ടുപേരും മറ്റേതോ ഒരു സ്ത്രീയുടെ പാല്‍കുടിച്ചിട്ടുണ്ടാകാം. അതുകൊണ്ടാണ് ആഇശ(റ)ക്കും അദ്ദേഹത്തിനുമിടയില്‍ സഹോദര ബന്ധം സ്ഥാപിതമായത്. ഈ സംഭവത്തെ ദുര്‍വ്യാഖ്യാനിച്ചുകൊണ്ടാണ് ഈ ദുരാരോപണം ഇസ്‌ലാംവിരോധികള്‍ പ്രചരിപ്പിക്കുന്നത്. ആഇശ (റ) പ്രസവിക്കാത്ത ഒരു സ്ത്രീയായിരുന്നു. അതുകൊണ്ടുതന്നെ അവര്‍ക്ക് ഒരു കുഞ്ഞിനും മുലയൂട്ടാന്‍ സാധ്യവുമല്ല. ഈ വസ്തുതകളെല്ലാം അറിഞ്ഞിട്ടും വിമര്‍ശകര്‍ ഇത്തരം കള്ളപ്രചരണം നടത്തിക്കൊണ്ടിരിക്കുന്നത്, ഇസ്‌ലാംവിരോധം അവരുടെ മനഃസാക്ഷിയെപോലും കൊന്നുകളഞ്ഞിരിക്കുന്നു എന്നതിന്റെ സൂചനയാണ്. എങ്കില്‍പിന്നെ പ്രവാചകന്‍ കോപിച്ചതെന്തിനാണ്? അതിന്റെ ഉത്തരവും ഹദീഥിലുണ്ട്. ‘നിങ്ങള്‍ മുലകുടി ബന്ധത്തിലുള്ള സഹോദരന്മാരെപ്പറ്റി ശരിക്കും നോക്കണം. വിശപ്പടങ്ങുന്ന നിലക്ക് മുലകുടിച്ചാല്‍ മാത്രമേ മുലകുടി ബന്ധം ഉണ്ടാവുകയുള്ളു’ എന്ന നബി(സ)യുടെ വിശദീകരണം അതിന്റെ മറുപടിയാണ്. മുലപ്പാല്‍ കുടിക്കുന്നതിലൂടെ വിശപ്പടങ്ങുന്ന പ്രായത്തിലുള്ള മുലകുടിയെ മാത്രമേ ഇസ്‌ലാം – വിവാഹബന്ധം നിഷിദ്ധമായ- സഹോദരബന്ധമായി പരിഗണിക്കുകയുള്ളൂ. അത് രണ്ടു വയസ്സിനുള്ളില്‍ അഞ്ച് തവണയെങ്കിലും നടന്നിരിക്കണം. മുലകുടിയുമായി ബന്ധപ്പെട്ട ഈ നിബന്ധനകള്‍ എല്ലാം യോജിക്കുന്നതാണോ നിങ്ങള്‍ക്കിടയിലുള്ള സഹോദര ബന്ധങ്ങള്‍ എന്ന് നിങ്ങള്‍ ഉറപ്പു വരുത്തണമെന്ന കാര്‍ക്കശ്യമാണ് പ്രവാചക(സ)ന്റെ കോപത്തിനു പിന്നിലെ കാരണം. അല്ലാതെ ആഇശ(റ)യുടെ സമീപത്ത് അവരുടെ മുലകുടി ബന്ധത്തിലെ സഹോദരനെ കണ്ടപ്പോള്‍ എന്തെങ്കിലും സംശയം ഹൃദയത്തില്‍ വന്നുപോയതുകൊണ്ടല്ല. കാരണം ആഇശ (റ) അത്തരം ബന്ധുക്കള്‍ (മുലകുടിയിലൂടെയുള്ള ബന്ധുക്കള്‍) അവരെ സന്ദര്‍ശിക്കാന്‍ അനുവാദം തേടുമ്പോള്‍ വളരെ സൂക്ഷ്മതയോടെ മാത്രമേ അനുവാദം നല്‍കാറുള്ളൂ എന്ന് നബി(സ)ക്ക് അറിയാമായിരുന്നു. ഒരു സംഭവം നാം കാണുക: ”ആഇശ (റ) പറഞ്ഞു: മുലകുടി ബന്ധത്തിലൂടെയുള്ള എന്റെ പിതാമഹന്‍ എന്റെ അടുക്കല്‍ (വീട്ടില്‍)കയറാന്‍ സമ്മതം ചോദിച്ചു. അപ്പോള്‍ അല്ലാഹുവിന്റെ ദൂതരുടെ സമ്മതം കിട്ടാത്തതിനാല്‍ ഞാന്‍ വിസ്സമ്മതിച്ചു. അപ്പോള്‍ അല്ലാഹുവിന്റെ ദൂതന്‍ (സ) പറഞ്ഞു: നിന്റെ പിതാമഹനെ നീ അടുത്ത് പ്രവേശിപ്പിച്ചുകൊള്ളുക…” (സ്വഹീഹു മുസ്‌ലിം: 1445)

വിവാഹബന്ധം നിഷിദ്ധമായവര്‍ മാത്രമേ തന്റെ വീട്ടില്‍ പ്രവേശിക്കാവൂ എന്ന കാര്യത്തില്‍ അതീവ ശ്രദ്ധ പുലര്‍ത്തിയിരുന്ന ഒരു വ്യക്തിത്വമായിരുന്നു ആഇശ (റ). അല്ലാത്ത പുരഷന്മാരെ മറക്കു പിറകില്‍ നിന്നുകൊണ്ടല്ലാതെ അവര്‍ അഭിസംബോധന ചെയ്തിട്ടില്ല. അത്തരം ഒരു വ്യക്തിയെ പറ്റി ഇത്രമാത്രം ദുഷിച്ച കള്ളകഥകള്‍ പ്രചരിപ്പിക്കുവാന്‍ മിഷണറിമാര്‍ക്കല്ലാതെ മറ്റാര്‍ക്കു കഴിയും. ഇതിനെയെല്ലാം നാം ‘സുവിശേഷ വേല’ എന്നുതന്നെ വിളിക്കണോ?!. ദുഷിച്ച ഭാവനകള്‍കൊണ്ട് ഹദീഥുകളില്‍ നിന്നും അശ്ലീലതകള്‍ മെനഞ്ഞെടുത്ത ‘സുവിശേഷ വേല’യാണ് നാം മുകളില്‍ കണ്ടതെങ്കില്‍, വ്യാജതൂലികകള്‍ കൊണ്ട് ഹദീഥുകളില്‍ നേര്‍ക്കുനേരെ കൃത്രിമത്വം നടത്തുന്ന ‘സുവിശേഷ വേല’യാണ് ഇനി കാണാനുള്ളത്. വിഷയം ഒരു ദുര്‍ബലമായ ഹദീഥാണ്. മുസ്‌ലിംകള്‍ പ്രമാണമായി കാണാത്ത ഒരു വാറോല. പക്ഷെ അതില്‍പ്പോലും പച്ചയായ കാപട്യം കാണിക്കുവാന്‍ മിഷണറികള്‍ക്ക് മടിയില്ലെന്നതാണ് അതിശയകരമായ സംഗതി.

‘കല്ലെറിയലിന്റെ വചനവും പത്ത് പ്രാവശ്യമാണ് മുതിര്‍ന്ന പുരുഷന്മാര്‍ക്ക് മുലകുടിയെന്ന വചനവും (ആദ്യകാലത്ത് കുര്‍ആനില്‍) അവതരിപ്പിക്കപ്പെട്ടിരുന്നുവെന്നും, അതെഴുതിയ രേഖ തന്റെ തലയിണക്കടയില്‍ സൂക്ഷിച്ചുവെച്ചിരുന്നെന്നും പ്രവാചകന്‍ മരണപ്പെട്ടപ്പോള്‍, ഞങ്ങളെല്ലാം അതുമായി ബന്ധപ്പെട്ട തിരക്കുകള്‍ക്കിടയിലായിരുന്ന സന്ദര്‍ഭത്തില്‍ ഒരു ആട് അകത്ത് കടന്ന് അത് തിന്നുകളഞ്ഞെന്നും ആഇശയില്‍ നിന്നും സുനനു ഇബ്‌നു മാജയും മുസ്‌നദു അഹ്‌മദും നിവേദനം ചെയ്തിട്ടുണ്ടെന്നതാണ് വ്യാജമായ ആരോപണം. ആയതിനാല്‍ ദുര്‍ബലമായ ആ ഹദീഥ് നമുക്ക് ആദ്യം പരിശോധിക്കാം.

"ആഇശ (റ) പറഞ്ഞു: കല്ലെറിയലിന്റെ വചനവും പത്ത് പ്രാവശ്യമാണ് മുലകുടിയെന്ന വചനവും അവതരിക്കപ്പെട്ടിരുന്നു. അതെഴുതിയ രേഖ എന്റെ തലയിണക്കടിയിലുണ്ടായിരുന്നു. ദൈവദൂതന്‍ മരണപ്പെട്ടപ്പോള്‍, ഞങ്ങളെല്ലാം അതുമായി ബന്ധപ്പെട്ട തിരക്കുകള്‍ക്കിടയിലായിരുന്ന സന്ദര്‍ഭത്തില്‍ ഒരു ആട് അകത്ത് കടന്ന് അത് തിന്നു കളഞ്ഞു” (സുനനു ഇബ്‌നു മാജ: 1944, മുസ്‌നദു അഹ്‌മദ്: 43/343) ഹദീഥ് ദുര്‍ബലമാണ്. അതുകൊണ്ടുതന്നെ മുസ്‌ലിംകള്‍ക്ക് അത് പ്രമാണവുമല്ല. ഈ നിവേദനത്തില്‍ മുഹമ്മദ്ബ്‌നു ഇസ്ഹാഖ് എന്ന ഒരു നിവേദകനുണ്ട്. അദ്ദേഹത്തെ തെളിവിനു കൊള്ളില്ലെന്ന് ഹദീഥ് നിവേദനം ചെയ്ത അഹ്‌മദ് തന്നെ വ്യക്തമാക്കിയിട്ടുണ്ട്. അയാള്‍ ദുര്‍ബലനാണെന്ന് യഹ്‌യബിന്‍ മഈനും, ശക്തനല്ലെന്ന് നസാഇയും പറഞ്ഞിട്ടുണ്ട്. (തഹ്ദീബ് അത്തഹ്ദീബ് 9/45)

പക്ഷെ നമ്മുടെ പ്രശ്‌നം അതല്ല. ദുര്‍ബലമായ ആ നിവേദനത്തില്‍ പോലും പച്ചയായ കൃത്രിമത്വം നടത്തിയിരിക്കുകയാണ് ചില മിഷണറി നുണ ഫാക്ടറികള്‍. ‘കല്ലെറിയലിന്റെ വചനവും പത്ത് പ്രാവശ്യമാണ് മുലകുടിയെന്ന വചനവും അവതരിക്കപ്പെട്ടിരുന്നു’ എന്ന ഭാഗത്ത് മിഷണറിമാര്‍ എഴുതി പിടിപ്പിച്ചത് ‘കല്ലെറിയലിന്റെ വചനവും പത്ത് പ്രാവശ്യമാണ് മുതിര്‍ന്ന പുരുഷന്മാര്‍ക്ക് മുലകുടിയെന്ന വചനവും അവതരിപ്പിക്കപ്പെട്ടിരുന്നു എന്നാണ്. ‘മുതിര്‍ന്ന പുരുഷന്മാര്‍’ എന്നത് ‘സുവിശേഷകന്മാരു’ടെ കള്ളകോലാണ്. അവര്‍ കൃത്രിമമായി എഴുതിച്ചേര്‍ത്തു പ്രചരിപ്പിച്ചതാണെന്നര്‍ത്ഥം. എത്രമാത്രം ലജ്ജാവഹവും നിന്ദാപരവുമാണ് ഇവരുടെ പ്രവര്‍ത്തനങ്ങള്‍. ഇത്തരം ചവറുകളാണ്, ‘സയന്‍സ് ടെമ്പര്‍’ ഉണ്ടാക്കാന്‍ കൂട്ടയോട്ടം നടത്തുന്ന യുക്തിവാദികളും സ്ത്രീ ശാക്തീകരണം പത്യമാക്കിയ ഫെമിനിസ്റ്റുകളും സനാതനധര്‍മ്മം സ്ഥാപിക്കാന്‍ നെട്ടോട്ടമോടുന്ന ഫാഷിസ്റ്റുകളും കൊണ്ടുനടക്കുന്നതെന്നോര്‍ക്കുമ്പോള്‍ സഹതാപം തോന്നുന്നു. ‘ദൈവം സ്‌നേഹ’മാണെന്ന് നാഴികക്ക് നാല്‍പ്പത് വട്ടവും പാട്ടുപാടുന്നവര്‍ക്കെങ്കിലും, അവര്‍ പാടിയതിലും പറഞ്ഞതിലും തെല്ല് ആത്മാര്‍ത്ഥത ഉണ്ടായിരുന്നെങ്കില്‍ എത്ര നന്നായിരുന്നു.

‘മുലയൂട്ടല്‍’ ആഇശ(റ)യുടെ നിലപാടെന്തായിരുന്നു

വിവാഹബന്ധം നിഷിദ്ധമാക്കപ്പെടുന്ന ബന്ധങ്ങള്‍ സ്ഥാപിക്കാനായി ആഇശ (റ) തന്റെ സഹോദരി പുത്രിമാരോടും സഹോദര പുത്രിമാരോടും, മുതിര്‍ന്ന പുരുഷന്മാര്‍ക്ക് മുലപ്പാല്‍ നല്‍കുവാന്‍ ഉപദേശിച്ചിരുന്നു എന്നതാണ് വിഷയ സംബന്ധിയായ അടുത്ത വിമര്‍ശനം. വാസ്തവത്തില്‍ ആഇശ (റ) അപ്രകാരം ആരോടെങ്കിലും ഉപദേശിക്കുകയോ നിര്‍ദ്ദേശിക്കുകയോ ചെയ്തിരുന്നോ? പ്രസ്തുത വിഷയം ചര്‍ച്ച ചെയ്യും മുന്‍പ് ഇസ്‌ലാമില്‍ മുലകുടി ബന്ധം എപ്രകാരമാണ് സ്ഥാപിതമാകുന്നതെന്ന് നാം മനസ്സിലാക്കേണ്ടിയിരിക്കുന്നു. എങ്കില്‍ മാത്രമേ ആഇശ(റ)യുടെ നിലപാട് എന്തായിരുന്നു എന്ന് നമുക്ക് വേര്‍തിരിച്ചു മനസ്സിലാക്കാനാകൂ. രണ്ടു വയസ്സിനുള്ളില്‍, വിശപ്പ് അടങ്ങുംവിധം അഞ്ചു തവണയെങ്കിലും മുലപ്പാല്‍ നല്‍കിയാല്‍ മാത്രമാണ്, വിവാഹം നിഷിദ്ധമാക്കപ്പെടുന്ന ബന്ധം സ്ഥാപിതമാവുകയുള്ളൂ എന്നതാണ് ഈ വിഷയകമായി ഇസ്‌ലാമിക നിയമം. അഥവാ കുഞ്ഞുങ്ങള്‍ക്ക് മുലയൂട്ടുന്നതിലൂടെ മാത്രമേ മുലകുടി ബന്ധം സ്ഥാപിതമാകൂ എന്നതാണ് ഇസ്‌ലാമിക വിധി.

”മാതാക്കള്‍ തങ്ങളുടെ സന്താനങ്ങള്‍ക്ക് പൂര്‍ണ്ണമായ രണ്ടുകൊല്ലം മുല കൊടുക്കേണ്ടതാണ്. (കുട്ടിയുടെ) മുലകുടി പൂര്‍ണ്ണമാക്കണം എന്ന് ഉദ്ദേശിക്കുന്നവര്‍ക്കത്രെ ഇത്”. (ക്വുര്‍ആന്‍: 2:233) ”അവന്റെ മുലകുടി നിര്‍ത്തുന്നതാകട്ടെ രണ്ടുവര്‍ഷം കൊണ്ടുമാണ്” (ക്വുര്‍ആന്‍: 31:14) ”അവന്റെ ഗര്‍ഭകാലവും മുലകുടി നിര്‍ത്തലും കൂടി മുപ്പത് മാസക്കാലമാകുന്നു” (ക്വുര്‍ആന്‍: 46:15) ”വയറിനെ പുഷ്ടിപ്പിക്കുന്നതില്‍ അല്ലാതെ രിദ്വാഅ് (പാലൂട്ടല്‍) ഇല്ല” (ഇബ്‌നുമാജ: 1946, ഇബ്‌നുഹിബ്ബാന്‍: 4225) ”അസ്ഥിയെ ശക്തിപ്പെടുത്തുകയും മാംസം പുഷ്ടിപ്പെടുത്തുകയും ചെയ്യുന്നതല്ലാത്തത് രിദ്വാഅ് അല്ല” (സുനനുല്‍ ക്വുബ്‌റാ, ബൈഹകി: 15653) ”രണ്ടു വയസ്സിനുള്ളില്‍ ഊട്ടുന്നതല്ലാത്ത രിദ്വാഅ് ഇല്ല.” (സുനനുദ്ദാറക്വുത്‌നി : 4364) ”വിശപ്പടങ്ങുന്ന കുട്ടി മാത്രമാണ് (ബന്ധം സ്ഥാപിക്കുന്ന) രിദ്വാഅ് (മുലകുടി).” (സ്വഹീഹു മുസ്‌ലിം:32, അബൂദാവൂദ് : 12/8 നസാഈ : 26/51, ഇബ്‌നുമാജ : 9/37, അഹ്‌മദ്: 6/93) മുലപ്പാല്‍ കുടിക്കുന്നതിലൂടെ വിശപ്പടങ്ങുക കുഞ്ഞുങ്ങള്‍ക്കാണ്. അതുകൊണ്ടുതന്നെ മുലകുടിബന്ധം സ്ഥാപിതമാവുക കുഞ്ഞായിരിക്കുമ്പോള്‍ മുലപ്പാല്‍ കുടിക്കുന്നതിലൂടെ മാത്രമാണ് എന്ന് ഹദീഥിനെ വ്യാഖ്യാനിച്ച പണ്ഡിതന്മാരെല്ലാം വ്യക്തമാക്കുന്നുണ്ട്. (ശര്‍ഹു മുസ്‌ലിം: 1/434, ഫത്ഹുല്‍ ബാരി: 1/100, ഉംദത്തുല്‍ കാരി: 20/97, ഇര്‍ശാദുസ്സരി: 8/33, ഫൈദുല്‍ കദീര്‍: 3/360, മിര്‍ക്കാത്തുല്‍ മഫാത്തീഹ്: 3168, ശര്‍ഹു സര്‍ക്കാനി അലാല്‍ മുവത്വഃ: 3/374, ഹാശിയത്തു സിന്ദി അലാ സുനനി ഈബ്‌നുമാജ: 1/600, ഔനുല്‍ മഅ്ബൂദ്: 6/47) പ്രവാചകാനുചരന്മാരായ ഉമ്മര്‍, അലി, അബ്ദുല്ലാഹിബ്‌നു മസ്ഊദ്, അബ്ദുല്ലാഹിബ്‌നു അബ്ബാസ്, ഇബ്‌നു ഉമ്മര്‍, അബൂഹുറൈറ, ജാബിര്‍, പ്രവാചക പത്‌നിമാര്‍ തുടങ്ങി ഹസനുല്‍ ബസ്വരി, സുഹ്‌രി, ഇക്‌രിമ, ഔസാഈ, ശുഅ്ബി, കത്താദ, ഇബ്‌നു ശബ്‌റമ, ഇബ്‌നു അബീ ലൈല, ഹസനിബ്‌നു സ്വാലിഹ്, ഇബ്‌നു അബി ദിഅ്ബ്, സൗരി, ഇസ്ഹാക്, അബൂസൗര്‍, അബൂ ഉബൈദ്, ത്വബ്‌രി, ലൈസ് എന്നിങ്ങനെ പൗരാണിക കര്‍മ്മശാസ്ത്ര പണ്ഡിതരായ സര്‍വ്വരുടേയും അഭിപ്രായവും ഇതുതന്നെയാണ്. (മുഖ്തസറു ഇഖ്തിലാഫുല്‍ ഉലമ: 2/314, അല്‍ഹാവി അല്‍കബീര്‍: മാവര്‍ദി 11/366, മുഹല്ലാ: 10/17, മുഗ്നി: 11/319, സാദുല്‍ മആദ്: 5/577, തഫ്‌സീറു ഇബ്‌നു കസീര്‍ : 1/284, ഉംദത്തുല്‍ കാരി : 20/85) ഹനഫീ, മാലികീ, ശാഫിഈ, ഹമ്പലി കര്‍മ്മശാസ്ത്ര മദ്ഹബുകളുടെ പണ്ഡിതന്മാരുടെ അഭിപ്രായവും തഥൈവ. (അല്‍ മബ്‌സൂത്: 5/135, അല്‍ മുദവ്വനത്തുല്‍ കുബ്‌റാ: 5/407, അല്‍ ഉമ്മ്: 5/428, മുഗ്‌നി: 11/319)

അപ്പോള്‍ ഇസ്‌ലാമിന്റെ വിധി മുതിര്‍ന്നവര്‍ക്ക് മുലകുടി ഇല്ല എന്നതാണ്. എന്നാല്‍ സാലിമിന്റെ വിഷയത്തില്‍ ഉണ്ടായത് തീര്‍ത്തും അവരുടെ വിഷയത്തില്‍ മാത്രം ബാധകമായ ഒരു ഇളവു മാത്രമായിരുന്നു. അതും മുലപ്പാല്‍ സ്തനങ്ങളില്‍ നിന്നും നേരിട്ടു നല്‍കാതെ, പാല്‍ പിഴിഞ്ഞ് തളികയിലാക്കി കൊടുക്കുക മാത്രമാണുണ്ടായതെന്ന് നാം മുമ്പ് സൂചിപ്പിച്ചിട്ടുണ്ട്. എന്നാല്‍ ‘സാലിമിന്റെ വിഷയത്തിലെ പ്രവാചക നടപടി തെളിവ് പിടിച്ചുകൊണ്ട് ആഇശ (റ) തന്റെ വീട്ടില്‍ പ്രവേശിക്കാന്‍ താല്‍പര്യപ്പെടുന്ന മുതിര്‍ന്നവര്‍ക്ക് മുലപ്പാല്‍ നല്‍കാനായി തന്റെ സഹോദര പുത്രിമാരോടും, സഹോദരി പുത്രിമാരോടും നിര്‍ദ്ദേശിക്കാറുണ്ടായിരുന്നു എന്നും, അപ്രകാരം അവര്‍ (സഹോദര- സഹോദരി പുത്രിമാര്‍) അഞ്ചുതവണ മുലപ്പാല്‍ നല്‍കി മുലകുടിബന്ധം സ്ഥാപിതമായതിന് ശേഷമേ ആഇശ (റ) അവരെ തന്റെ വീട്ടില്‍ പ്രവേശിപ്പിക്കാറുള്ളൂ’ (സുനനു അബീദാവൂദ്: 2061) എന്ന നിവേദനത്തെ ദുര്‍വ്യാഖ്യാനിച്ചു കൊണ്ടാണ് ഇസ്‌ലാമില്‍ മുതിര്‍ന്ന പുരുഷന്മാര്‍ക്കും മുലകുടിയുണ്ടെന്ന വ്യാജപ്രചരണം ഇസ്‌ലാംവിരോധികള്‍ അഴിച്ചുവിട്ടു കൊണ്ടിരിക്കുന്നത്. എന്നാല്‍ ഇവിടെ നാം പ്രത്യേകം മനസ്സിലാക്കേണ്ടുന്ന ചില കാര്യങ്ങളെ ഇപ്രകാരം സംഗ്രഹിക്കാം:

ഒന്ന്: ആഇശ (റ) ആര്‍ക്കും മുലപ്പാല്‍ – സ്തനങ്ങളില്‍ നിന്ന് നേരിട്ടോ അല്ലാതെയോ കൊടുത്തിട്ടില്ല. ഒരിക്കലും പ്രസവിച്ചിട്ടില്ലാത്ത അവര്‍ക്ക് എങ്ങനെ മുലപ്പാലുണ്ടാകും? അതുകൊണ്ടാണ് മുലപ്പാല്‍ നല്‍കാനായി ആഇശ (റ) തന്റെ സഹോദരി പുത്രിമാരോടും സഹോദര പുത്രിമാരോടും നിര്‍ദ്ദേശിച്ചത്. ആഇശ (റ) മുതിര്‍ന്ന പുരുഷന്മാരെ മുലയൂട്ടിയെന്ന മിഷണറി പ്രചരണം കല്ലുവെച്ച കളവാണെന്ന് വ്യക്തം.

രണ്ട്: സാലിമിന് സഹ്‌ല മുലപ്പാല്‍ നല്‍കിയത് സ്തനങ്ങളില്‍ നിന്ന് നേരിട്ടല്ലെന്നും, ഒരു തളികയില്‍ പാല്‍ പിഴിഞ്ഞ് കുടിക്കാന്‍ കൊടുക്കുന്ന രീതി – ‘വുജൂര്‍’ (الوجور) – യാണ് അവലംബിച്ചിരുന്നതെന്നും നാം മുമ്പ് വ്യക്തമായ തെളിവുകളുടെ അടിസ്ഥാനത്തില്‍ സ്ഥാപിച്ചതാണ്. അതെ രീതിയില്‍ (‘വുജൂര്‍’) തന്നെ മുലപ്പാല്‍ കൊടുക്കാനാണ് ആഇശ (റ) തന്റെ സഹോദര പുത്രിമാരോടും, സഹോദരി പുത്രിമാരോടും നിര്‍ദ്ദേശിച്ചതെന്ന വസ്തുത മറച്ചുവെച്ചുകൊണ്ട്, ആഇശ(റ)യുടെ നിര്‍ദ്ദേശം – (സഹോദര, സഹോദരി പുത്രിമാരോട്) നേരിട്ട് സ്തനങ്ങളില്‍ നിന്നും മുലയൂട്ടുവാനായിരുന്നു എന്ന പച്ചക്കള്ളം പ്രചരിപ്പിക്കുകയാണ് ഇസ്‌ലാംവിമര്‍ശകര്‍ ചെയ്യുന്നത്. സാലിമിന്റെ വിഷയത്തിലെ പ്രവാചക നടപടി തെളിവ് പിടിച്ചുകൊണ്ട്, മുലകുടി പ്രായം (രണ്ടു വയസ്സ്) പിന്നിട്ട ഒരാള്‍ക്ക് മുലപ്പാല്‍ നല്‍കുക വഴി വിവാഹം നിഷിദ്ധമാക്കപ്പെടുന്ന ബന്ധം സ്ഥാപിക്കാമെന്ന് വിലയിരുത്തിയ ആഇശ (റ), പക്ഷെ ആ വിഷയത്തില്‍ പ്രവാചകന്‍ നിര്‍ദ്ദേശിച്ച മുലപ്പാല്‍ നല്കുന്ന രീതി (വുജൂര്‍) പരിഗണിച്ചില്ലെന്നത് കേവലം വിമര്‍ശകരുടെ ലൈംഗിക ഭാവന മാത്രമാണ്.

മൂന്ന്: ഇസ്‌ലാമില്‍ ബന്ധം സ്ഥാപിതമാക്കുന്നത് മൂന്ന് മാര്‍ഗങ്ങളിലൂടെയാണ്. പ്രസവം, വിവാഹം, മുലകുടി. ഈ ബന്ധങ്ങളില്‍ പെടാത്ത, വിവാഹബന്ധം നിഷിദ്ധമാക്കപ്പെട്ടുവെന്ന് സ്ഥിരപ്പെടാത്ത ഒരു അന്യപുരുഷനേയും തന്റെ വീട്ടില്‍ പ്രവേശിപ്പിക്കില്ലെന്ന ദൃഢനിശ്ചയം ആഇശ(റ)യുടെ കണിശമായ പാതിവ്രത്യത്തേയും പരിശുദ്ധിയേയുമാണ് തെളിയിക്കുന്നത്. അത്തരം മഹത്തരമായ ഒരു നിലപാടി നെയാണ്, തങ്ങളുടെ ദുഷിച്ച ലൈംഗിക ഭാവനയിലൂടെ അശ്ലീലതയുടെ ചായം പൂശാന്‍ മിഷണറി നുണ ഫാക്ടറികള്‍ ഒരുമ്പെട്ടതെന്നത് എത്രമാത്രം നെറികെട്ട ‘സുവിശേഷ വേല’യായിപ്പോയി.

നാല്: ‘തന്റെ വീട്ടില്‍ പ്രവേശിക്കാന്‍ താല്‍പര്യപ്പെടുന്ന മുതിര്‍ന്നവര്‍ക്ക് മുലപ്പാല്‍ നല്‍കാനായി അവര്‍ കല്‍പ്പിക്കുമായിരുന്നു’ എന്ന ഹദീഥില്‍ പരാമര്‍ശിക്കപ്പെട്ട ‘മുതിര്‍ന്നവര്‍’ (الكبير) എന്ന പദപ്രയോഗം യഥാര്‍ത്ഥത്തില്‍ മുതിര്‍ന്ന പുരുഷന്മാരെയല്ല ഉദ്ദേശിക്കുന്നത്. മറിച്ച്, മുലകുടിപ്രായം കഴിഞ്ഞ വലിയ കുട്ടികളെയാണ്. ഇത് ചില ഹദീഥുകളില്‍ വ്യക്തമായി തന്നെ പരാമര്‍ശിച്ചിട്ടുണ്ട്.

ഉമ്മുസലമ (റ) ആഇശ(റ)യോട് പറഞ്ഞു ”നിങ്ങളുടെ അടുക്കല്‍ മുലകുടി പ്രായം കഴിഞ്ഞ കുട്ടി – الغلام الايفع – പ്രവേശിക്കുന്നുണ്ടല്ലോ. (അവര്‍ക്ക് മുലപ്പാല്‍ നല്‍കിയാല്‍ മുലകുടിബന്ധം സ്ഥാപിതമാകുമെന്ന് വിശ്വസിക്കാത്തതുകൊണ്ട്) അവര്‍ എന്റെ അടുക്കല്‍ പ്രവേശിക്കുന്നത് ഞാനിഷ്ടപ്പെടുന്നില്ല. അപ്പോള്‍ അവര്‍ക്ക് മുലപ്പാല്‍ നല്‍കുക വഴി മുലകുടിബന്ധം സ്ഥാപിതമാകുമെന്നതിന് തെളിവായി ആഇശ (റ) സാലിമിന്റെ സംഭവം ഉദ്ധരിക്കുകയും ചെയ്തു…” (സ്വഹീഹു മുസ്‌ലിം : 1453, മുസ്‌നദു അഹ്‌മദ്: 25415) ഹദീഥില്‍ പരാമര്‍ശിക്കപ്പെട്ട, അല്‍ ഗുലാം അല്‍ അയ്ഫഅ് (الغلام الايفع) എന്നതുകൊണ്ട് വിവക്ഷിക്കുന്നത് പ്രായപൂര്‍ത്തിയാകാത്ത ആണ്‍കുട്ടിയെയാണെന്ന് ഇമാം നവവി (റ) ഹദീഥിനെ വ്യാഖ്യാനിക്കവെ വ്യക്തമാക്കുന്നുണ്ട്. (ശര്‍ഹു മുസ്‌ലിം: 10/33)

തന്റെ വീട്ടില്‍ പ്രവേശിക്കാന്‍ താല്‍പര്യപ്പെടുന്ന ‘പുരുഷന്മാര്‍ക്ക്’ മുലപ്പാല്‍ നല്‍കി മുലകുടിയിലെ ബന്ധുവാക്കാന്‍ ആഇശ (റ) ശ്രദ്ധിച്ചിരുന്നു എന്ന് സൂചിപ്പിക്കുന്ന ഹദീഥുകളിലെ ‘പുരുഷന്മാര്‍’ (الرجال) എന്നതുകൊണ്ട് ഉദ്ദേശിക്കുന്നതും മുലകുടി പ്രായം കഴിഞ്ഞ, എന്നാല്‍ പ്രായപൂര്‍ത്തിയായിട്ടില്ലാത്ത ആണ്‍കുട്ടികളെയാണ്. അല്ലാതെ വലിയ പുരുഷന്മാരെയല്ല. പുല്ലിംഗത്തെ (Male Gender) സൂചിപ്പിക്കാനായി രിജാല്‍ (الرجال) എന്ന് അറബിയില്‍ ഉപയോഗിക്കാറുണ്ട്. എന്ന് മാത്രമല്ല, ശാരീരികമായി പുരുഷനായി മാറിയിട്ടില്ലാത്ത ആണ്‍കുട്ടിയെ അവന്റെ ലിംഗഭേദം (Gender) പുല്ലിംഗമായതിനാല്‍ അവന്‍ ഭാവിയില്‍ പുരുഷനായി മാറും എന്നത് പരിഗണിച്ച് റജുല്‍ (الرجل) ‘പുരുഷന്‍’ എന്ന് അറബിഭാഷാ നിയമപ്രകാരം വിളിക്കാവുന്നതാണ്. ഇതിനെ അറബി അലങ്കാര ശാസ്ത്രത്തില്‍ (Rhetoric) ‘ഭാവിയില്‍ ആയി മാറുന്നതിനെ പരിഗണിച്ചുകൊണ്ടുള്ള പ്രയോഗം’ (اعتبار ما يكون) എന്നാണ് പറയുക. (അല്‍ മജാസുല്‍ മുര്‍സല്‍ ഫീ ലിസാനില്‍ അറബ്: അഹ്‌മദ് ഹിന്ദാവി അബ്ദുല്‍ ഗഫ്ഫാര്‍, കിത്താബു മിന്‍ഹാജുല്‍ വാള്വിനി ലില്‍ ബലാഗ: 3/300, ഗായത്തുല്‍ മുസൂല്‍ ഫീ ശര്‍ഹി ലുബ്ബുല്‍ ഉസൂല്‍: 1/51)

ഇത് കേവലം ഭാഷ ശാസ്ത്ര നിയമപ്രകാരമുള്ള ഒരു സമര്‍ത്ഥനമല്ല; മറിച്ച് ഹദീഥുകള്‍ വ്യക്തമായി സൂചിപ്പിച്ച ഒരു വിഷയം ഭാഷ പ്രയോഗങ്ങള്‍ക്കുപോലും അന്യമല്ലെന്ന ബോധ്യപ്പെടുത്തലാണ്. ആഇശ (റ) ഉദ്ദേശിച്ച ‘പുരുഷന്മാര്‍’ (الرجال) മുലകുടി പ്രായം പിന്നിട്ട, എന്നാല്‍ പ്രായപൂര്‍ത്തിയായിട്ടില്ലാത്ത ആണ്‍കുട്ടികളാണെന്ന് വ്യക്തമാക്കുന്ന മറ്റു ചരിത്ര നിവേദനങ്ങളും നമുക്ക് കാണാം.

(a) ”സാലിമിബ്‌നു അബ്ദുല്ലാഹിബ്‌നു ഉമര്‍ (സഹ്‌ല മുലപ്പാല്‍ നല്‍കിയ സാലിം അല്ല ഇത്) പറയുന്നു: ഞാന്‍ കുഞ്ഞായിരിക്കെ ആഇശ (റ) എന്നെ അവരുടെ സഹോദരി ഉമ്മുകുല്‍സും ബിന്‍ത് അബൂബക്കറിന്റെ അടുത്തേക്ക് അയച്ചു. എന്നിട്ടവരോട് ഇങ്ങനെ പറഞ്ഞു: ‘ഇവന് പത്തു തവണ മുലപ്പാല്‍ ഊട്ടുക എങ്കില്‍ ഇവന് എന്റെ അടുത്ത് (വീട്ടില്‍) പ്രവേശിക്കാമല്ലോ.’ സാലിം പറയുന്നു: ഉമ്മുകുല്‍സും എനിക്ക് മൂന്നു തവണ മുലപ്പാല്‍ തന്നു. പിന്നീടെനിക്ക് രോഗമായി. അതിനാല്‍ എനിക്ക് മൂന്നു തവണ മാത്രമേ മുലപ്പാല്‍ തരാന്‍ സാധിച്ചുള്ളൂ. ഉമ്മുകുല്‍സൂം എനിക്ക് പത്തു തവണ മുലപ്പാല്‍ തരാതിരുന്നത് കാരണത്താല്‍ (വലുതായപ്പോള്‍) ഞാന്‍ ആഇശയുടെ അടുത്ത് പ്രവേശിക്കില്ലായിരുന്നു” (മുവത്വഅ്: 2/603)

ചെറുപ്രായത്തില്‍ ഉമ്മുകുല്‍സൂമിന്റെ മുലപ്പാല്‍, ബന്ധം സ്ഥാപിതമാകുന്ന നിലയ്ക്ക് പൂര്‍ണമായും കുടിക്കാതിരിക്കുന്നത് മൂലം ആഇശ(റ)യുമായി മുലകുടിയിലെ ബന്ധം സ്ഥാപിതമായില്ല. അതു കാരണത്താല്‍ ആഇശ (റ) തന്നെ വലുതായപ്പോള്‍ അവരുടെ വീട്ടില്‍ പ്രവേശിപ്പിക്കില്ലായിരുന്നു എന്ന് ചുരുക്കം. വലിയ പുരുഷന്മാര്‍ മുലപ്പാല്‍ കുടിച്ചാല്‍ മുലകുടി ബന്ധം സ്ഥാപിതമാകും എന്നതായിരുന്നു ആഇശ(റ)യുടെ വാദമെങ്കില്‍ സാലിമിബ്‌നു അബ്ദുല്ലാഹിബ്‌നു ഉമ്മറിനെ വലിയ പുരുഷനായതിനുശേഷവും, സഹോദരിയുടെ മുലപ്പാല്‍ നല്‍കിയതിനുശേഷം തന്റെ വീട്ടില്‍ പ്രവേശിപ്പിക്കാമായിരുന്നില്ലേ? അപ്പോള്‍ കാര്യം വളരെ വ്യക്തമാണ്. മുലപ്പാല്‍ ഊട്ടപ്പെടുന്ന ‘രിജാല്‍’ (പുരുഷന്മാര്‍) എന്നതുകൊണ്ട് ആഇശ (റ) ഉദ്ദേശിച്ചത്, മുലകുടി പ്രായം പിന്നിട്ട, എന്നാല്‍ പ്രായപൂര്‍ത്തിയെത്തിയിട്ടില്ലാത്ത ആണ്‍കുട്ടികളെ മാത്രമാണ്.

(b) ”കുട്ടികള്‍ക്ക് (الصبيان) അവര്‍ പുരുഷന്മാരായാല്‍ തന്റെ അടുത്ത് പ്രവേശിക്കുന്നതിന് തടസ്സമാകാതിരിക്കാന്‍ – ചെറുപ്രായത്തില്‍ മുലപ്പാല്‍ ഊട്ടുവാന്‍ ആഇശ (റ) തന്റെ സഹോദരന്‍ അബ്ദുര്‍റഹ്‌മനിബ്‌നു അബൂബക്കറിനോട് കല്‍പ്പിക്കുമായിരുന്നു”. (ബദാഇഉ സ്വനാഇഅ്: 4/6, ശര്‍ഹു മുഖ്തസറു ത്വഹാവി: 3/325) ഇവിടെ ആഇശ (റ) ഉദ്ദേശിച്ച പ്രായം വളരെ വ്യക്തമാണ്. കുട്ടികള്‍ പുരുഷന്മാരായാല്‍ തന്റെ അടുത്ത് പ്രവേശിക്കുന്നതിന് തടസ്സമാകാതിരിക്കുവാന്‍- ചെറുപ്രായത്തില്‍ തന്നെ മുലപ്പാല്‍ ഊട്ടുവാനാണ് അവര്‍ നിര്‍ദേശിക്കുന്നത്. ഇവിടെയും, ‘പുരുഷന്മാര്‍’ എന്നതുകൊണ്ട് മുതിര്‍ന്ന പുരുഷന്മാരെയല്ല, പ്രായപൂര്‍ത്തിയെത്താത്ത എന്നാല്‍ മുലകുടി പ്രായം കഴിഞ്ഞ ആണ്‍കുട്ടികളെയാണ് അവര്‍ ഉദ്ദേശിച്ചതെന്ന് വ്യക്തം. ഹദീഥില്‍ ‘രിജാല്‍’ (പുരുഷന്മാര്‍) എന്നത് ലിംഗഭേദത്തെ കുറിക്കാനാണ്, പ്രായത്തെ കുറിക്കാനല്ല ആഇശ (റ) ഉപയോഗിച്ചതെന്ന് പൗരാണികവും പ്രാമാണികവുമായ അറബി ഡിക്ഷണറികളിലെല്ലാം വ്യക്തമാക്കപ്പെട്ടിട്ടുണ്ട്. (താജുല്‍ ഉറൂസ്: 29/34, അല്‍ കാമൂസുല്‍ മുഹീത്: 1/1297, ലിസാനുല്‍ അറബ്: 11/265, അല്‍ കാമില്‍ ഫില്ലുഗത്തി വല്‍ അദബ്: 1/100)

(c) വലിയ പുരുഷന്മാര്‍ക്ക് മുലപ്പാല്‍ നല്‍കുക വഴി ബന്ധം സ്ഥാപിതമാകും എന്നതല്ല, മറിച്ച് മുലകുടി പ്രായമായ രണ്ടു വയസ്സു കഴിഞ്ഞ മുതിര്‍ന്ന ആണ്‍കുട്ടികള്‍ക്ക് മുലപ്പാല്‍ നല്‍കുക വഴി മുലകുടിബന്ധം സ്ഥാപിതമാകും എന്ന് മാത്രമാണ് ആഇശയുടെ അഭിപ്രായം എന്നതിനാലാണ് അതിനെതിരെ, ആഇശ (റ)യുടെ അഭിപ്രായം തിരുത്തിക്കൊണ്ട് എതിരഭിപ്രായം പ്രകടിപ്പിച്ച ഉമ്മുസലമ (റ) ‘തൊട്ടിലില്‍ വെച്ച് തന്നെ മുലകുടിച്ചാലെ’ (حتي يرضع في المهد) മുലകുടി ബന്ധം സ്ഥാപിതമാകൂ എന്ന പദപ്രയോഗം തന്നെ ഉപയോഗിച്ചത്. (മുസ്‌നദു അഹ്‌മദ്: 6/270, മുസ്‌നദു ഇബ്‌നു ഉവാന: 3/122, സുനനു അബൂദാവൂദ്: 2/223) അതുകൊണ്ടു തന്നെ ഈ വിഷയത്തില്‍ ആഇശയുടെ ഒറ്റപ്പെട്ട അഭിപ്രായം, വലിയ പുരുഷന്മാര്‍ക്ക് മുലപ്പാല്‍ നല്‍കുക വഴി ബന്ധം സ്ഥാപിതമാകും എന്നല്ല മറിച്ച്, മുലകുടി പ്രായമായ രണ്ട് വയസ്സ് കഴിഞ്ഞ ആണ്‍കുട്ടികള്‍ക്കും മുലപ്പാല്‍ നല്‍കുക വഴി മുലകുടി ബന്ധം സ്ഥാപിതമാകും എന്നു മാത്രമാണ്. അതും തീര്‍ത്തും അവരുടെ ഒറ്റപ്പെട്ട ഒരഭിപ്രായം മാത്രമാണത്. അതുകൊണ്ടാണ് മറ്റു പ്രവാചകപത്‌നിമാരെല്ലാം അവരെ ആ വിഷയത്തില്‍ എതിര്‍ത്തതും തിരുത്തിയതും.

അതിനാല്‍ ആഇശ(റ)യുടെ ഈ ഒറ്റപ്പെട്ട അഭിപ്രായത്തെ ദുര്‍വ്യാഖ്യാനിച്ച് അശ്ലീലതകള്‍ വിതറുവാനുള്ള മിഷണറി നുണ ഫാക്ടറികളുടെ ഉദ്യമം വളരെ തരംതാഴ്ന്ന പ്രവര്‍ത്തനമായിപ്പോയി. ചെറിയ ആണ്‍കുട്ടികള്‍ക്കു മുലയൂട്ടുന്നതിലൂടെ, അതും സ്തനങ്ങളില്‍ നിന്ന് നേരിട്ടല്ലാതെ തളികയില്‍ പിഴിഞ്ഞ് കുടിപ്പിക്കപ്പെടുന്നതിലൂടെ (വുജൂര്‍) ബന്ധുക്കളായാല്‍ അവര്‍ക്ക് പ്രായപൂര്‍ത്തിയെത്തിയാലും അവരുടെ സാന്നിധ്യം അന്യപുരുഷ സാന്നിധ്യമാകില്ലല്ലോ എന്ന് കണക്കുകൂട്ടിയാണ് ഇത്തരമൊരു നയം ആഇശ (റ) സ്വീകരിച്ചത്.

‘മുലയൂട്ടല്‍’ വിവാദം ഇനിയും അവസാനിക്കാത്ത ‘സുവിശേഷ വേലകള്‍’

രണ്ടു വയസ്സിനുള്ളില്‍ വിശപ്പടങ്ങും വിധം അഞ്ചു തവണയെങ്കിലും മുലയൂട്ടിയാല്‍ മാത്രമേ, വിവാഹം നിഷിദ്ധമാക്കപ്പെടുന്ന ബന്ധം സ്ഥാപിതമാകൂ എന്ന കണിശമായ കര്‍മശാസ്ത്ര വിധി നിഷ്‌കര്‍ഷിച്ച ഒരു മതത്തെ അപകീര്‍ത്തിപ്പെടുത്താനും വൈകൃതവും പ്രാകൃതവുമായ ഒരു ധര്‍മ്മശാസ്ത്രമായി അതിനെ ചിത്രീകരിക്കുവാനും വേണ്ടി മിഷണറി നുണ ഫാക്ടറികള്‍ കൈവെച്ചത് ‘സഹ്‌ല-സാലിം മുലപ്പാലൂട്ടല്‍’ സംഭവമാണ്. സാലിമിന്റെ വിഷയത്തില്‍ മാത്രം, അദ്ദേഹത്തിന്റെ മാതാവ് സഹ്‌ലയുടെ തീവ്ര ദുഃഖത്തിന് ഒരു പോംവഴിയായി – ഇളവ് നല്‍കപ്പെട്ട ഒരു കാര്യമാണ് അതെന്ന് വ്യക്തമായിട്ടും ആ സംഭവത്തെ ഇത്രമേല്‍ മ്ലേച്ഛമായവതരിപ്പിച്ചു എന്നതു മാത്രമല്ല ‘മുലയൂട്ടല്‍’ വിവാദവുമായി ബന്ധപ്പെട്ട് ‘കുഞ്ഞാടുകള്‍’ ചെയ്ത ക്രൂരത. ഒരു കുഞ്ഞിനും ഒരിക്കലും മുലയൂട്ടാന്‍ സാധ്യമല്ലാത്ത, ഒരിക്കലും പ്രസവിച്ചിട്ടില്ലാത്ത പ്രവാചക പത്‌നി ആഇശ (റ) അത്തരത്തില്‍ ‘മുലയൂട്ടല്‍’ നടത്തിയിരുന്നു എന്ന പച്ചക്കള്ളം പടച്ചുണ്ടാക്കാന്‍ ഇസ്‌ലാമിന്റെ അടിസ്ഥാന പ്രമാണങ്ങളില്‍ ഒന്നായ ഹദീഥുകളില്‍ കൈവെച്ചു. എന്നിട്ടും അരിശം തീരാതെ ആഇശയുടെ പ്രസ്തുത വിഷയകമായ ഒറ്റപ്പെട്ട അഭിപ്രായത്തെ ഉയര്‍ത്തിക്കാട്ടി വീണ്ടും അശ്ലീലതകള്‍ക്ക് പഴുതു തപ്പി നടന്നു. അതും അവരുടെ അഭിപ്രായത്തെ വസ്തുതാവിരുദ്ധമായ നിലയില്‍ ദുര്‍വ്യാഖ്യാനിച്ചു കൊണ്ട്. ഇനിയും തീര്‍ന്നിട്ടില്ല ഇവരുടെ മഹത്തായ ‘സുവിശേഷ വേല’കള്‍. തന്റെ ഒറ്റപ്പെട്ട അഭിപ്രായത്തില്‍ നിന്നും അഥവാ മുലകുടി പ്രായം (രണ്ടു വയസ്സ്) കഴിഞ്ഞതും എന്നാല്‍ പ്രായപൂര്‍ത്തിയെത്തിയിട്ടില്ലാത്തതുമായ ആണ്‍കുട്ടികള്‍ക്ക് മുലപ്പാല്‍ തളികയില്‍ പിഴിഞ്ഞു കുടിപ്പിച്ചാല്‍ ‘ബന്ധം’ സ്ഥാപിതമാകും എന്ന അഭിപ്രായത്തില്‍ നിന്നും പിന്നീട് ആഇശ (റ) മടങ്ങിയിട്ടുണ്ട് എന്ന വസ്തുത ഈ ‘കുഞ്ഞാടുകള്‍’ മറച്ചുവെച്ചിട്ടുണ്ടെന്നതും കൂടി നാം അറിയണം. അപ്പോള്‍ നമുക്ക് മനസ്സിലാകും മിഷണറി പക്ഷം വിമര്‍ശനമാണോ അല്ല വിരോധമാണോ പ്രകടിപ്പിക്കുന്നതെന്ന്.

ഇമാം കാസാനി പറഞ്ഞു: ”എന്നാല്‍ ആഇശ (ഈ വിഷയത്തിലുള്ള) നയം അവര്‍ തിരുത്തുകയുണ്ടായി എന്ന് സൂചിപ്പിക്കുന്ന നിവേദനങ്ങള്‍ ഉദ്ദരിക്കപ്പെട്ടിട്ടുണ്ട്. അവര്‍ ഇപ്രകാരം പറഞ്ഞതായി നിവേദനം ചെയ്യപ്പെട്ടിരിക്കുന്നു: ‘രക്തവും മാംസവും മുളപ്പിക്കുന്നതല്ലാത്ത (കൈകുഞ്ഞായിരിക്കുമ്പോളല്ലാത്ത) മുലകുടി വിവാഹബന്ധം നിഷിദ്ധമാക്കില്ല (മുലകുടി ബന്ധം സ്ഥാപിതമാകില്ല)” (ബദാഇഉസ്വനാഇ: 4/6) ‘തൊട്ടിലില്‍ വെച്ചുതന്നെ മുല കുടിച്ചാലെ’ മുലകുടി ബന്ധം സ്ഥാപിതമാകൂ എന്നും (മുസ്‌നദു അഹമ്ദ്: 6/270, മുസ്‌നദു ഇബ്‌നു ഉവാന: 3/122, സുനനു അബൂദാവൂദ്: 2/223) ‘അല്ലാഹുവാണേ, സാലിമിന്റെ വിഷയത്തില്‍ മാത്രമായി (ഖാസ്) പ്രവാചകന്‍ അനുവദിച്ചു കൊടുത്ത ഇളവായിരുന്നു അത് (ആ ഇളവ് എല്ലാവര്‍ക്കും ബാധകമല്ല)’ (സ്വഹീഹു മുസ്‌ലിം: 1454, മുസ്‌നദു അഹ്‌മദ്: 26660, സുനനു അബൂദാവൂദ്: 2061) എന്നുമുള്ള പ്രവാചക പത്‌നി ഉമ്മുസലമ(റ)യുടെ തിരുത്ത് ആഇശ (റ) സ്വീകരിച്ചു എന്നും, തന്റെ ഒറ്റപ്പെട്ട പൂര്‍വ്വ അഭിപ്രായത്തില്‍ നിന്നും അവര്‍ മടങ്ങിയെന്നും ഹദീഥ് ഗ്രന്ഥങ്ങളില്‍ തന്നെ വ്യക്തമാക്കിയിരിക്കെ അതെല്ലാം മറച്ചുവച്ചുകൊണ്ട് ഇത്തരം കുത്സിത പ്രവര്‍ത്തനങ്ങള്‍ നടത്താന്‍ ‘ദൈവത്തിന്റെ കുഞ്ഞാടുകള്‍’ക്കെങ്ങിനെ കഴിഞ്ഞു. മിഷണറി നുണ ഫാക്ടറികള്‍ നിര്‍മ്മിച്ച ഈ നുണകഥയും പൊക്കിപ്പിടിച്ച് ഇപ്പോഴും യുക്തിവാദികളും ഫെമിനിസ്റ്റുകളും ഫാഷിസ്റ്റുകളുമടങ്ങുന്ന ഇസ്‌ലാംവിമര്‍ശകര്‍ ചോദിക്കുന്നു ‘പ്രവാചകന്റെ ഈ കല്പന നടപ്പിലാക്കുവാന്‍ ഈ കാലഘട്ടത്തിലെ മുസ്‌ലിം സ്ത്രീകള്‍ തയ്യാറാകുമോ? തങ്ങളുടെ ഭാര്യമാരെ കൊണ്ട് ഈ പ്രവാചക നിര്‍ദ്ദേശം നടപ്പില്‍ വരുത്തുവാന്‍ മുസ്‌ലിം പുരുഷന്മാര്‍ ഒരുക്കമാണോ?’ എന്ന്. പാവം! സാധുക്കള്‍ വല്ലതും അറിയുന്നുണ്ടോ ഇതു മിഷണറി ‘സുവിശേഷ വേല’ മാത്രമാണെന്ന കഥ.!!!

മുഹമ്മദ് നബി സ്ത്രീ പീഢകനായിരുന്നു എന്ന് വ്യക്തമാക്കുന്ന നിവേദനങ്ങള്‍ സ്വഹീഹുല്‍ ബുഖാരിയില്‍ തന്നെ കാണാം. ‘ശൗത്ത്’ എന്നു വിളിക്കപ്പെട്ടിരുന്ന ഒരു തോട്ടത്തില്‍ വെച്ച് ‘ജൗന്‍’ ഗോത്രത്തിലെ ഉമൈമ:ബിന്‍ത് ശറാഹീല്‍ എന്ന സ്ത്രീയെ പ്രവാചകന്‍ കടന്നുപിടിക്കാന്‍ ശ്രമിക്കുകയും അവര്‍ ശക്തമായി പ്രതിരോധിക്കുകയും ചെയ്ത സംഭവം ഹദീഥ് ഗ്രന്ഥങ്ങള്‍ തന്നെ റിപ്പോര്‍ട്ട് ചെയ്തിട്ടുണ്ട്. സ്ത്രീകളെ കടന്നുപിടിക്കാന്‍ പോലും മടിയില്ലാതിരുന്ന ഒരു വ്യക്തിയെ എങ്ങനെയാണ് മാനവികതയുടെ പ്രവാചകനായി വിലയിരുത്തുക?

ഇമാം ബുഖാരി തന്റെ സ്വഹീഹില്‍ വ്യത്യസ്ത പരമ്പരകളിലൂടെ ഉദ്ധരിച്ച ഒരു സംഭവത്തിന്റെ ഏതാനും ഭാഗങ്ങള്‍ അടര്‍ത്തിയെടുത്ത് പച്ചക്ക് ദുര്‍വ്യാഖാനിച്ചിരിക്കുകയാണിവിടെ. ഉമൈമ: ബിന്‍ത് ശറാഹീലുമായി ബന്ധപ്പെട്ട് ഇമാം ബുഖാരി അദ്ദേഹത്തിന്റെ സ്വഹീഹില്‍ ഉദ്ധരിച്ച നിവേദനങ്ങള്‍ വായിച്ചാല്‍ സംഭവം പ്രവാചക ജീവിതത്തിലെ ഒരു പുഴുകുത്തല്ലെന്നും മറിച്ച് പ്രവാചകന്റെ മഹാമനസ്‌കതയും സഹിഷ്ണുതയും കാരുണ്യവും അനുവാചകര്‍ക്ക് ബോധ്യപ്പെടുക മാത്രമാണുണ്ടാവുക എന്ന ഉത്തമബോധ്യമുള്ളതുകൊണ്ടു തന്നെ ഹദീഥുകളില്‍ നിന്നും തങ്ങള്‍ക്ക് കൈവെക്കാനൊക്കുന്നതു മാത്രം തിരഞ്ഞുപിടിച്ചു ദുര്‍വ്യാഖ്യാനിച്ചു ദുഷിപ്പിക്കുക എന്ന തന്ത്രമാണ് ഇസ്‌ലാംവിമര്‍ശകര്‍ ചെയ്തുകൊണ്ടിരിക്കുന്നത്. അതിനാല്‍ പ്രസ്തുത സംഭവവുമായി ബന്ധപ്പെട്ട് ഇമാം ബുഖാരി അദ്ദേഹത്തിന്റെ സ്വഹീഹില്‍ ഉദ്ധരിച്ച ഏതാനും ചില നിവേദനങ്ങള്‍ താഴെ കൊടുക്കുകയാണ്. ഒരൊറ്റ വായനയിലൂടെ തന്നെ ഏതൊരാള്‍ക്കും എളുപ്പം ഗ്രഹിക്കാന്‍ സാധിക്കുന്ന ഒരു വിഷയത്തെ പോലും ഇത്തരത്തില്‍ ദുര്‍വ്യാഖ്യാനിക്കുവാന്‍ ഇസ്‌ലാം വിമര്‍ശകര്‍ക്ക് യാതൊരു ലജ്ജയുമില്ലെന്നത് അവരുടെ സംസ്‌കാരത്തിന് നേരെയുള്ള ചൂണ്ടുവിരലാണ്.

”ഹംസത്തിബ്‌നു അബീ ഉസൈദ് നിവേദനം: അബൂ ഉസൈദ് (റ) പറഞ്ഞു: ഒരിക്കല്‍ പ്രവാചകനോടൊപ്പം(സ) ഞങ്ങള്‍ ഒരു യാത്ര പുറപ്പെട്ടു. ‘ശൗത്ത്’ എന്ന് വിളിക്കപ്പെടുന്ന ഒരു തോട്ടത്തെ ലക്ഷ്യമാക്കി ഞങ്ങള്‍ നീങ്ങി. ഞങ്ങള്‍ രണ്ട് തോട്ടങ്ങള്‍ക്കിടയിലെത്തിയപ്പോള്‍, അവിടെ ഞങ്ങളിരുന്നു. ഞങ്ങളോട് അവിടെ ഇരിക്കാൻ പ്രവാചകൻ (സ) പറഞ്ഞു. എന്നിട്ട് അദ്ദേഹം ആ തോട്ടത്തിലേക്ക് പോയി. ജൗന്‍ ഗോത്രത്തിലെ സ്ത്രീയെ അവിടേക്ക് കൊണ്ടുവരപ്പെട്ടിരുന്നു. ഈത്തപ്പന കൊണ്ടുണ്ടാക്കിയ വീട്ടിലായിരുന്നു അവര്‍. ഉമൈമ: ബിന്‍ത് ശറാഹീല്‍ എന്നായിരുന്നു അവരുടെ നാമം. അവരോടൊപ്പം അവരുടെ മുലകുടി ബന്ധത്തിലെ പോറ്റുമ്മയും ഉണ്ടായിരുന്നു. അവരുടെ അടുത്തേക്ക് അല്ലാഹുവിന്റെ ദൂതൻ (സ) പ്രവേശിക്കുകയും ‘നീ നിന്നെ എനിക്ക് സമര്‍പ്പിക്കുക’ എന്ന് പറയുകയും ചെയ്തു. അപ്പോള്‍ അവര്‍ പറഞ്ഞു: ‘ഒരു രാജ്ഞി അവരെ ഏതെങ്കിലും ഒരു സാധാരണക്കാരന് സമര്‍പ്പിക്കുമോ?’ അവരുടെ മേല്‍ കൈവെച്ച് അവരെ ശാന്തയാക്കാനായി തന്റെ കൈകള്‍ അദ്ദേഹം നീട്ടി. അപ്പോള്‍ അവര്‍ പറഞ്ഞു: ‘നിങ്ങളില്‍ നിന്നും ഞാൻ അല്ലാഹുവില്‍ ശരണം തേടുന്നു’. അപ്പോൾ പ്രവാചകൻ (സ) പറഞ്ഞു: ‘ശരണം തേടുവാന്‍ ഏറ്റവും അർഹനായവനിലാണ് നീ ശരണം തേടിയിരിക്കുന്നത്. പിന്നീട് അദ്ദേഹം ഞങ്ങളുടെ അടുത്തേക്ക് വന്നുകൊണ്ട് പറഞ്ഞു. “അബൂ ഉസൈദ്, അവര്‍ക്ക് രണ്ട് റാസിഖിയ്യാ വസ്ത്രങ്ങള്‍ നല്‍കുകയും അവരുടെ കുടുംബത്തിലേക്ക് അവരെ തിരിച്ചെത്തിക്കുകയും ചെയ്യുക.” (ബുഖാരി: 5255)

”പ്രവാചകൻ (സ) ഉമൈമ: ബിന്‍ത് ശറാഹീലിനെ വിവാഹം ചെയ്തു. അവരെ അദ്ദേഹത്തിന്റെ അടുത്തേക്ക് പ്രവേശിക്കപ്പെട്ടപ്പോള്‍, അദ്ദേഹം തന്റെ കൈകൾ നീട്ടി സ്വീകരിച്ചു. അവര്‍ക്കത് ഇഷ്ടപെടാത്തത് പോലെ അവര്‍ പ്രതികരിച്ചു. അപ്പോള്‍ അവര്‍ക്ക് തിരികെ സ്വഗൃഹത്തിലേക്ക് പോകാന്‍ യാത്രാ സൗകര്യങ്ങള്‍ ചെയ്യാനും, രണ്ട് റാസിഖിയ്യാ വസ്ത്രങ്ങള്‍ സമ്മാനമായി നല്‍കാനും പ്രവാചകൻ (സ) അബൂ ഉസൈദിനോട് കല്പിച്ചു.” (ബുഖാരി: 5256)

”ഇമാം ഔസാഇ (റ) പറഞ്ഞു: ഞാന്‍ സുഹ്‌രിയോട് ചോദിച്ചു: ‘പ്രവാചകൻ(സ)യുടെ ഭാര്യമാരില്‍ ആരാണ് അദ്ദേഹത്തില്‍ നിന്നും ശരണം തേടിയത്?’ അദ്ദേഹം പറഞ്ഞു: ‘എന്നോട് ആഇശ(റ)യില്‍ നിന്നും ഇപ്രകാരം ഉര്‍വ അറിയിക്കുകയുണ്ടായി. ജൗന്‍ ഗോത്രക്കാരിയെ പ്രവാചകന്റെ(സ) അരികിലേക്ക് (അദ്ദേഹത്തിന്റെ പത്‌നിയായി) ആനയിക്കപ്പെടുകയും അദ്ദേഹം അവളുടെ അരികിലേക്ക് ചെല്ലുകയും ചെയ്തപ്പോള്‍ അവള്‍ പറഞ്ഞു: ‘ഞാന്‍ താങ്കളില്‍ നിന്നും അല്ലാഹുവിനോട് ശരണം തേടുന്നു.’ അപ്പോള്‍ അദ്ദേഹം പറഞ്ഞു: ‘അതിമഹത്വമുള്ളവനിലാണ് നീ ശരണം തേടിയിരിക്കുന്നത്. നീ നിന്റെ കുടുംബത്തിലേക്ക് മടങ്ങിക്കൊള്ളുക.” (ബുഖാരി: 5254)

ഉമൈമ: ബിന്‍ത് ശറാഹീലീനെ പ്രവാചകൻ (സ) വിവാഹം ചെയ്തിരുന്നു എന്ന വസ്തുത മറച്ചു പിടിച്ചു കൊണ്ട് പ്രസ്തുത സംഭവം വിമര്‍ശകര്‍ അവതരിപ്പിക്കാറ് എന്ന് മുകളിലെ മൂന്ന് ഹദീസുകളും ഒരുമിച്ചു വെച്ച് വായിക്കുമ്പോൾ സുതരാം വ്യക്തമാവുന്നു. പ്രവാചകനെ പെണ്ണു പിടുത്തക്കാരനായും പരസ്ത്രീകളെ കടന്നുപിടിക്കുന്ന വ്യക്തിയായും താറടിക്കുകയാണ് ഈ ദുര്‍വ്യാഖ്യാന കസര്‍ത്ത് നടത്തുന്നതിന് പിന്നിലെ ചേതോവികാരം. താന്‍ വിവാഹം ചെയ്ത സ്ത്രീക്ക് തന്നോടൊപ്പം ജീവിക്കുവാന്‍ താല്‍പര്യമില്ലെന്നറിഞ്ഞപ്പോള്‍, നിര്‍ബന്ധിച്ച് കൂടെ താമസിപ്പിക്കാതെ മാന്യമായി അവരെ സ്വഗൃഹത്തിലേക്ക് യാത്രയാക്കുകയും വേര്‍പിരിയും മുമ്പ് അവര്‍ക്ക് സമ്മാനങ്ങള്‍ നല്‍കുകയും ചെയ്ത മാതൃകാപരമായ ഒരു നപടിയെ എത്ര നികൃഷ്ടമായാണ് ഇസ്‌ലാംവിമര്‍ശകര്‍ അവതരിപ്പിച്ചിരിക്കുന്നത്.

ﻻَ ﺗَﺤْﻤِﻠُﻮا اﻟﻨِّﺴَﺎءَ ﻋَﻠَﻰ ﻣَﺎ ﻳَﻜْﺮَﻫْﻦَ

“സ്ത്രീകളെ അവർക്ക് വെറുക്കുന്നത് ചെയ്യാൻ നിങ്ങൾ നിർബന്ധിക്കരുത്.” (മുസ്വന്നഫ് അബ്ദുർ റസാഖ്: 10320) എന്ന് അനുചരന്മാരെ പഠിപ്പിക്കുക മാത്രമല്ല കാരുണ്യ മൂർത്തിയായ പ്രവാചകൻ (സ) ചെയ്തത്, പ്രത്യുത ഉമൈമയോട് അനുവർത്തിച്ച നിലപാടിലൂടെ തന്റെ ആദർശനിഷ്ട സ്വജീവിതത്തിൽ പ്രാവർത്തികമായി തെളിയിക്കുക കൂടി അദ്ദേഹം ചെയ്തു.

പ്രവാചകന്‍ (സ) ഉദ്ദേശിച്ചിരുന്നെങ്കില്‍ അവരെ നിര്‍ബന്ധപൂര്‍വ്വം കൂടെ താമസിപ്പിക്കുവാന്‍ യാതൊരു തടസ്സവുമുണ്ടായിരുന്നില്ല. കാരണം അവിടുന്ന് ഇസ്‌ലാമിക സാമ്രാജ്യത്തിന്റെ അധിപനായിരുന്നു. രാജാക്കന്മാരും ചക്രവര്‍ത്തിമാരും ഒരു പെണ്ണിനെ ആഗ്രഹിച്ചു കഴിഞ്ഞാല്‍ അവളുടെ താല്‍പര്യം അന്വേഷിക്കുന്ന പതിവില്ലെന്ന് എല്ലാവര്‍ക്കുമറിയാവുന്ന വസ്തുതയാണ്. അതിനെതിരെ ഒരു ശബ്ദവുമവിടെ ഉയരുകയില്ല. ഇവിടെ പ്രവാചകന്‍ (സ) മാതൃകയാവുകയാണ്. താന്‍ വിവാഹം ചെയ്ത ഒരു സ്ത്രീക്ക് തന്നോടൊപ്പം ജീവിക്കുവാന്‍ താല്‍പര്യമില്ലെന്നറിഞ്ഞ നിമിഷം അവളെ ആശ്വസിപ്പിക്കുകയും സമാധാനിപ്പിക്കുകയും നിര്‍ഭയത്വത്തോടെ സ്വഗൃഹത്തിലേക്ക് മടങ്ങാന്‍ അവസരമൊരുക്കുകയും ചെയ്യുന്ന പ്രവാചകന്‍, ഒരു രാഷ്ട്രത്തിന്റെ ചോദ്യം ചെയ്യപ്പെടാത്ത ഭരണാധിപനാണെന്ന വസ്തുതയും ചേര്‍ത്തു മനസ്സിലാക്കുമ്പോള്‍ എത്രമാത്രം ആദരവും താല്‍പര്യവുമാണ് ആ വ്യക്തിത്വത്തിനോട് തോന്നേണ്ടത്. ഇസ്‌ലാം വിമര്‍ശകര്‍ക്ക് പക്ഷെ അത്തരം ഊഷ്മളമായ ചിന്തയും വികാരവുമൊന്നും ഉണ്ടാവുകയില്ല. കാരണം അവരുടെ ഹൃദയം കടുത്തു പോയിരിക്കുന്നു. ഊഷരമായ ചിന്തയും വികാരവുമാണ് അവരെ നയിക്കുന്നത്. വെറുപ്പും വിദ്വേഷവും മാത്രമാണ് അവരെ ഭരിക്കുന്നത്.

സമാധാനിപ്പിക്കാനായി -അതും സ്വന്തം ഭാര്യയുടെ മേൽ- കൈ വെക്കാൻ തുനിഞ്ഞതിനെ ‘കേറിപ്പിടിക്കലാക്കി’ ചിത്രീകരിക്കാനുള്ള ‘അപാരമായ കഴിവ് ‘ അഗമ്യഗമനത്തേയും, ബലാൽസംഗത്തേയും ശവരതിയേയുമെല്ലാം പ്രണയിക്കുന്നവരുടെ ‘ഹൈപ്പർ സെക്ഷ്വാലിറ്റിയുടെ'(Hypersexuality)ഭാഗമാണ്. ഇത്തരക്കാർക്ക് ഒരു തെറാപ്പിസ്റ്റിനെ സംഘടിപ്പിച്ച് കൊടുക്കുന്നതിന് പകരം സ്റ്റേജും, പേജും നൽകി സമൂഹത്തിലേക്ക് അഴിച്ചു വിടുന്നതാണ് ഭൗതികവാദികൾ മത വിശ്വാസികളോട് ചെയ്യുന്ന ഏറ്റവും വലിയ ദ്രോഹം.

ചരിത്രനിമിഷങ്ങളെ നാം തിരിച്ചൊന്ന് വിഭാവനം ചെയ്തു നോക്കൂ; ഉമൈമ: ബിന്‍ത് ശറാഹീലിന്റെ മനോഗതങ്ങളിലൂടെ. എന്തായിരിക്കും അവരുടെ ഹൃദയത്തില്‍ നിറഞ്ഞു നിന്ന വികാരങ്ങളും വിചാരങ്ങളും. തന്നെ വിവാഹം ചെയ്ത ആളോട്, അദ്ധേഹത്തെ തനിക്ക് ഇഷ്ടമല്ലെന്ന് അറിയിച്ചപ്പോള്‍ അവള്‍ കണ്ടത് ശാന്തവും മാന്യവുമായ പ്രതികരണമാണ്. ഒരു നിമിഷം പോലും അവളെ അദ്ദേഹത്തിന്റെ താല്‍പര്യങ്ങള്‍ക്കു വിധേയമാക്കാന്‍ ശ്രമിക്കാതെ, സമ്മാനങ്ങള്‍ നല്‍കി സ്വന്തം ഗൃഹത്തിലേക്ക് അവള്‍ക്ക് യാത്രാസൗകര്യമൊരുക്കിയ ഇസ്‌ലാമിക രാഷ്ട്രനായകനെ അവളുടെ ഹൃദയം എത്രമാത്രം ആദരിച്ചിട്ടുണ്ടാകും. തന്റെ നാട്ടിൽ തിരിച്ചെത്തിയ ഉമൈമ പ്രവാചകനുമായുള്ള ദാമ്പത്യ ജീവിതം ഉപേക്ഷിച്ചതിൽ പിന്നീട് ഖേദിക്കുകയും, ‘തനിക്ക് അമളി പറ്റി പോയി’ എന്ന് പറയുകയും ചെയ്തതായി ചരിത്രത്തിൽ തന്നെ കാണാം. (ഫത്ഹുൽ ബാരി: 9:314)

പക്ഷേ ഇസ്‌ലാം വിമര്‍ശകരുടെ വരണ്ട ഹൃദയങ്ങള്‍ക്ക് അത്തരം വിഭാവനങ്ങള്‍ അന്യമാണ്. ഹൃദയങ്ങളില്‍ അവര്‍ സൂക്ഷിച്ചിവെച്ചിരിക്കുന്ന ഇസ്‌ലാമിനോടുള്ള വെറുപ്പും വിദ്വേഷവും അവരെ എത്രമാത്രം ഊഷരവും വൃത്തിഹീനവുമായ മനോഗതിക്കാരാക്കിയിരുന്നു.! കഷ്ടം.!!

ബി (സ) യുടെ ജീവിതത്തിലെ ഏതെങ്കിലും ഒരു സംഭവം ഉയർത്തിക്കാണിച്ച് അദ്ദേഹം വർഗീയവാദിയായിരുന്നുവെന്ന് സ്ഥാപിക്കുവാൻ കൊടിയ നബി വിമർശകർക്കു പോലും കഴിഞ്ഞിട്ടില്ല.വർഗീയതയുടെ ലാഞ്ചനപോലുമില്ലാതെ ജീവിച്ചയാളായിരുന്നു പ്രവാചകനെന്ന് വ്യക്തമാക്കുന്ന നൂറുകണക്കിന് സംഭവങ്ങൾ ആ മഹത്ജീവിതത്തിൽ കാണാൻ കഴിയും.

ഹിജ്റയോടനുബന്ധിച്ച ഒരു സംഭവം നോക്കുക. അല്ലാഹുവിന്റെ മാര്ഗെത്തില്‍ ഇറങ്ങിത്തിരിച്ചവരോടൊപ്പം അല്ലാഹുവുണ്ടാകുമെന്ന പാഠം നൽകുന്ന സംഭവമാണ് ഹിജ്‌റ. മൂന്നുദിവസത്തെ ഗുഹാവാസത്തിനു ശേഷം, റബീഉല്‍ അവ്വല്‍ ഒന്നാം തിയ്യതി രാത്രിയാണ് യഥ്‌രിബ് ലക്ഷ്യമാക്കിയുള്ള നബിസംഘത്തിന്റെ യാത്രയാരംഭിച്ചത്. നബി (സ) യും അബൂബക്കറും (റ) അദ്ദേഹത്തിന്റെ ദാസനായ ആമിറുബ്‌നു ഫുഹൈറയും വഴികാട്ടിയായ അബ്ദുല്ലാഹിബ്‌നു ഉറൈഖത്ത് അല്‍ ലൈഥിയുമടങ്ങുന്ന നാലുപേരുടെ സംഘം എട്ടാം തിയ്യതി തിങ്കളാഴ്ച പകൽ സമയത്ത് യഥ്‌രിബിന്റെ കവാട നഗരിയായ ഖുബായില്‍ എത്തുന്നതിനിടയിലെ സംഭവങ്ങളിലൂടെ നബി (സ) നിരവധി പാഠങ്ങള്‍ ലോകത്തിന് നല്കു്കയുണ്ടായി. നബിസംഘത്തിലെ വഴികാട്ടിയെപ്പറ്റി ബുഖാരിയിലെ നിവേദനത്തില്‍ പറയുന്നത് 'സത്യനിഷേധികളായ ഖുറൈശികളുടെ മതത്തിലായിരുന്നു അയാള്‍'(സ്വഹീഹുല്‍ ബുഖാരി, കിതാബുഫദാഇലില്‍ അന്സാ്ര്‍) എന്നാണ്. കീല്‍ ഗോത്രക്കാരനും അബ്ദുബ്‌നു അദിയ്യിന്റെ സന്തതികളില്‍ പെട്ടയാളും വിദഗ്ധനായ വഴികാട്ടിയുമായിരുന്ന അദ്ദേഹമായിരുന്നു മക്കയില്‍ നിന്ന് യഥ്‌രിബിലെത്തുവോളം പ്രവാചകന് (സ) വഴികാട്ടിയായി കുടെയുണ്ടായിരുന്നതെന്ന യാഥാര്ഥ്യംക.

തങ്ങളുടെ പലായന വിവരവും ഥൗര്‍ ഗുഹയിലെ താമസവിവരവും മൂന്നു ദിവസങ്ങൾക്കുശേഷമുള്ള യാത്രാവിവരവുമെല്ലാം മുസ്‌ലിമല്ലാത്ത ഒരു വിശ്വസ്തനുമായി നേരത്തെതന്നെ പങ്കുവെക്കുന്നതില്‍ പ്രവാചകൻ (സ)യാതൊരുവിധ അനൗചിത്യവും ദർശിച്ചില്ല. അബ്ദുല്ലാഹിബ്‌നു ഉറൈഖത്താകട്ടെ തന്നെ ഏല്പിതച്ച ദൗത്യം ഭംഗിയായി നിര്വഹഹിക്കുകയും ചെയ്തു. യാത്രയിലോ യാത്രയ്ക്കു മുമ്പോ നബി (സ) യുടെ ശത്രുക്കളെ അറിയാതെ, അവര്ക്ക് പരിചയമില്ലാത്ത വഴികളിലൂടെ, തികച്ചും സുരക്ഷിതവുമായി മുഹമ്മദ് നബി (സ) യെ അദ്ദേഹം യഥ്‌രിബിലെത്തിച്ചു. നബി (സ) യെ ഒറ്റിക്കൊടുക്കുകയോ അപായപ്പെടുത്തുകയോ ചെയ്യാന്‍ അദ്ദേഹം സന്നദ്ധമായില്ല; അങ്ങനെ ചെയ്താല്‍ അദ്ദേഹത്തിന് നൂറ് ഒട്ടകം ലഭിക്കുമായിരുന്നിട്ടും നബി (സ) യോടുള്ള വാഗ്ദത്തം പാലിച്ചുകൊണ്ട് മുശ്‌രിക്കുകളുടെ പാരിതോഷികം വേണ്ടെന്നുവെക്കുകയാണ് ഇബ്‌നു ഉറൈഖത്ത് ചെയ്തത്. വിശ്വസ്തരാണെങ്കില്‍, അമുസ്‌ലിംകളെപ്പോലും സുപ്രധാനമായ ദൗത്യങ്ങള്‍ ഏൽപിക്കാമെന്ന് പഠിപ്പിക്കുന്നതാണ് അബ്ദുല്ലാഹിബ്‌നു ഉറൈഖത്തിനെ ഹിജ്‌റയിലെ വഴികാട്ടിയായി നിശ്ചയിച്ച മുഹമ്മദ് നബി (സ) യുടെ നടപടി. വഞ്ചിക്കുകയില്ലെന്നുറപ്പുള്ളവരെ, അവരുടെ മതം നോക്കാതെത്തന്നെ രഹസ്യങ്ങള്‍ വരെ ഏൽപിക്കാമെന്ന് സ്വന്തം ജീവിതത്തിലൂടെ പഠിപ്പിച്ച പ്രവാചകനില്‍ വർഗീതയവിദ്വേഷത്തിന്റെ കുഴലൂത്തുകാരനെ തിരയുന്നവര്‍ ഹിമപാളികൾക്കിടയില്‍ അഗ്നി അന്വേഷിക്കുന്നതുപോലുയുള്ള വൃഥാവ്യായാമമാണ് ചെയ്യുന്നത്.

മുഹമ്മദ് നബി(സ)യുടെ പ്രവാചകത്വത്തിന്റെ ദൃഷ്ടാന്തമെന്ന നിലയില്‍ അല്ലാഹു ചന്ദ്രനെ പിളര്‍ത്തിയെന്നും മക്കക്കാര്‍ അതിന് സാക്ഷികളായെന്നും പറയുന്ന ഇസ്‌ലാമിക പാരമ്പര്യങ്ങള്‍ അടിസ്ഥാനരഹിതവും അസ്വീകാര്യവുമാണ്. കാരണം, ചാന്ദ്രപിളര്‍പ്പ് ചരിത്രപരമായി സ്ഥാപിക്കപ്പെട്ടതോ ശാസ്ത്രീയമായി സംഭവ്യമോ അല്ല. ഇനി ചാന്ദ്രപ്പിളര്‍പ്പും പുനസമാഗമവും അത്യത്ഭുതകരമായി സംഭവിച്ചാല്‍ തന്നെ അതിന്റെ ഭൗതികാഘാതങ്ങള്‍ ചന്ദ്രോപരിതലത്തില്‍ ദൃശ്യമാകേണ്ടതാണ്. അങ്ങനെ യാതൊന്നും ഇന്നുവരെ കണ്ടെത്താനായിട്ടില്ല. പ്രവാചകന്റെ മഹത്വം വര്‍ധിപ്പിച്ചു കാണിക്കുന്നതിനുവേണ്ടി നിര്‍മിക്കപ്പെട്ട ഒരു വ്യാജകഥയാണ് ചാന്ദ്രപ്പിളര്‍പ്പിന്റേത് എന്നാണ് ഇതെല്ലാം വ്യക്തമാക്കുന്നത്. മിഷനറി വെബ്‌സൈറ്റുകളില്‍ സര്‍വസാധാരണമായ ഈ വാദങ്ങളില്‍ കഴമ്പില്ലേ?

 ഇല്ല. പ്രവാചകത്വത്തിന് തെളിവുകള്‍ ആവശ്യപ്പെട്ടുകൊണ്ടുള്ള മക്കന്‍ ബഹുദൈവാരാധകരുടെ അന്വേഷണങ്ങള്‍ക്കുള്ള മറുപടിയായും സത്യവിശ്വാസികള്‍ക്കുള്ള അനുഗ്രഹമായും അല്ലാഹു സംഭവിപ്പിച്ച നിരവധി അമാനുഷിക ദൃഷ്ടാന്തങ്ങള്‍കൊണ്ട് നിബിഢമാണ് നബിജീവിതമെന്ന് ചരിത്രം വ്യക്തമാക്കുന്നുണ്ട്. പ്രവാചകത്വത്തിന്റെ ഇരുപത്തിമൂന്ന് വര്‍ഷക്കാലയളവിനെ സമൃദ്ധമാക്കി ആകാശത്തുനിന്ന് ഭൂമിയിലേക്കവതരിപ്പിക്കപ്പെടുകയും ക്വുര്‍ആന്‍ ആയി ക്രോഡീകരിക്കപ്പെടുകയും ചെയ്ത ആറായിരത്തില്‍പരം ദിവ്യവചസ്സുകള്‍ തന്നെയായിരുന്നു അവയില്‍ ഏറ്റവും പ്രധാനപ്പെട്ടത്. ക്വുര്‍ആനിനു പുറമെ, അല്ലാഹുവില്‍നിന്ന് ലഭിച്ച വിസ്മയകരങ്ങളായ സഹായങ്ങളായും സംരക്ഷണങ്ങളായും കിറുകൃത്യമായ പ്രവചനപ്പുലര്‍ച്ചകളായും മനുഷ്യസാധ്യമല്ലാത്ത യാത്രാനുഭവങ്ങളായും പ്രകൃതിയിലും തീന്‍മേശയിലും മനുഷ്യശരീരങ്ങളിലും വരെ പ്രകടമായ അത്ഭുതങ്ങളായും ദിവ്യദൃഷ്ടാന്തങ്ങള്‍ മുഹമ്മദ് നബി(സ)യുടെ പ്രവാചകത്വത്തിന് സാക്ഷി പറഞ്ഞുകൊണ്ടേയിരിന്നിട്ടുണ്ട്.

അത്തരത്തില്‍ ചരിത്രം രേഖപ്പെടുത്തിയ അനേകം അത്ഭുത സംഭവങ്ങളിലൊന്നു മാത്രമാണ് ചന്ദ്രന്റെ പിളര്‍പ്പും പുനസമാഗമവും. പരിശുദ്ധ ക്വുര്‍ആന്‍ തന്നെ നേര്‍ക്കുനേരെ പരാമര്‍ശിച്ചിട്ടുള്ളതും അതുകൊണ്ടുതന്നെ മുസ്‌ലിം ലോകത്ത് സുപ്രസിദ്ധവുമാണ് ഈ സംഭവം. ചാന്ദ്രപിളര്‍പ്പിനെക്കുറിച്ച് ക്വുര്‍ആന്‍ സംസാരിച്ചതിന്റെ ആശയമിങ്ങനെയാണ്:

”ആ (അന്ത്യ) സമയം അടുത്തു. ചന്ദ്രന്‍ പിളരുകയും ചെയ്തു. ഏതൊരു ദൃഷ്ടാന്തം അവര്‍ കാണുകയാണെങ്കിലും അവര്‍ പിന്തിരിഞ്ഞു കളയുകയും, ഇത് നിലനിന്നു വരുന്ന ജാലവിദ്യയാകുന്നു എന്ന് അവര്‍ പറയുകയും ചെയ്യും. അവര്‍ നിഷേധിച്ചു തള്ളുകയും തങ്ങളുടെ തന്നിഷ്ടങ്ങളെ പിന്‍പറ്റുകയും ചെയ്തിരിക്കുന്നു. ഏതൊരു കാര്യവും ഒരു നിശ്ചിത സ്ഥാനം പ്രാപിക്കുന്നതാകുന്നു.” (54: 1-3)

അന്തിമ പ്രവാചകന്റെ ദൈവിക ദൃഷ്ടാന്തമെന്ന നിലയില്‍ ചന്ദ്രന്‍ പിളര്‍ന്നുവെന്നും പക്ഷേ എത്ര വലിയ അത്ഭുതങ്ങള്‍ കണ്ടാലും മാരണമോ ജാലവിദ്യയോ ആയി തള്ളുകയാണ് മക്കയിലെ പല സത്യനിഷേധികളുടെയും രീതിയെന്നും അത്തരക്കാര്‍ ഈ മഹാത്ഭുതത്തെയും ആ ഗണത്തില്‍പ്പെടുത്തി തള്ളിയെന്നുമുള്ള വിവരങ്ങള്‍ ചാന്ദ്രപിളര്‍പ്പിനെക്കുറിച്ച് ഈ ക്വുര്‍ആന്‍ വചനങ്ങള്‍ നല്‍കുന്നുണ്ട്. ക്വുര്‍ആന്‍ വചനങ്ങളെല്ലാം പ്രവാചകകാലഘട്ടത്തിലേതാണെന്ന കാര്യം ചരിത്രപരമായി അവിതര്‍ക്കിതമാംവിധം സ്ഥാപിക്കപ്പെട്ടതാണ്. ആകാശത്ത് ചന്ദ്രന്റെ പിളര്‍പ്പ് ദൃശ്യമാവുകയും അതിനെ ദൈവിക ദൃഷ്ടാന്തമായി പരിഗണിക്കാന്‍ വിസമ്മതിച്ച ചില മക്കക്കാര്‍ പ്രസ്തുത ദൃശ്യത്തിന് മറ്റു വ്യാഖ്യാനങ്ങള്‍ നല്‍കാന്‍ ശ്രമിക്കുകയും ചെയ്തുവെന്ന് പ്രഖ്യാപിക്കുന്ന ക്വുര്‍ആന്‍ വചനങ്ങള്‍ നബിനാവിലൂടെ മക്ക കേട്ടതാണ്; അദ്ദേഹത്തിന്റെ അനുചരന്‍മാര്‍ മറ്റെല്ലാ ക്വുര്‍ആന്‍ വചനങ്ങളെയും പോലെ സമകാലീനരായ അവിശ്വാസികള്‍ക്കിടയില്‍ നിരന്തരമായി അവ പാരായണം ചെയ്ത് പ്രഘോഷിച്ചുകൊണ്ടിരിക്കുകയും ചെയ്തിട്ടുണ്ട്. എന്നിട്ടും മക്കയിലെ ഒരു ബഹുദൈവാരാധകനും ഈ ക്വുര്‍ആന്‍ വചനങ്ങളെ ഒരിക്കല്‍ പോലും ചോദ്യം ചെയ്തു രംഗത്തുവന്നില്ല. അതിനര്‍ത്ഥം ചന്ദ്രന്‍ പിളര്‍ന്നതിന് ബഹുദൈവാരാധകരടക്കമുള്ള പല മക്കക്കാരും സാക്ഷിയായിയെന്നും അതിന്റെ സ്വഭാവത്തെക്കുറിച്ചുള്ള കുതര്‍ക്കങ്ങളുന്നയിക്കുക മാത്രമാണവര്‍ ചെയ്തതെന്നുമുള്ളതിനുള്ള അനിഷേധ്യമായ ചരിത്രരേഖകളായി പരാമൃഷ്ട ക്വുര്‍ആന്‍ വചനങ്ങള്‍ മാറുന്നുവെന്ന് തന്നെയാണ്.

മദീനയിലേക്കുള്ള പലായനത്തിന്റെ ഉദ്ദേശം അഞ്ചു വര്‍ഷം മുമ്പ് ഒരു പൗര്‍ണമി രാവിലാണ് നബി തിരുമേനി (സ) തന്റെ ചുറ്റുമുണ്ടായിരുന്നവര്‍ക്ക് ചന്ദ്രന്‍ പിളര്‍ന്നുമാറുന്നത് കാണിച്ചുകൊടുത്തത് എന്നാണ് ചരിത്രകാരന്‍മാരുടെ പൊതുവായ അഭിപ്രായം. ചാന്ദ്രപിളര്‍പ്പ് ദൃശ്യമായതിനുള്ള ചരിത്രരേഖ ക്വുര്‍ആന്‍ മാത്രമല്ല; പ്രത്യുത മക്കയില്‍ പ്രവാചകന്റെ സമകാലീനരായവരില്‍ നിന്നുള്ള വിശ്വസ്തമായ ധാരാളം നിവേദനങ്ങള്‍ കൂടിയാണ്. മുഹമ്മദ് നബി(സ)യുടെ കാലത്ത് ചന്ദ്രന്‍ രണ്ടുഭാഗങ്ങളായി പിളര്‍ന്നുമാറിയെന്നു പറയുന്ന ഒട്ടനവധി ഹദീഥുകളാണ് ആധികാരികതയില്‍ ആര്‍ക്കും സംശയമില്ലാത്ത അനേകം പരമ്പരകള്‍ വഴി ഏറ്റവും പ്രാമാണികമായ ഹദീഥ് സമാഹാരങ്ങളില്‍ തന്നെ രേഖപ്പെടുത്തപ്പെട്ടിരിക്കുന്നത്. പ്രവാചകകാലം മുതല്‍ക്കുള്ള ഓരോ തലമുറയിലും അനേകമാളുകള്‍ ഒരുമിച്ച് കൈമാറി വന്നുവെന്ന് (മുതവാതിര്‍) പറയാനാകുംവിധം വിഖ്യാതവും അനിഷേധ്യവുമാണ് ഈ ഹദീഥുകളിലെ സംഭവവിവരണങ്ങള്‍. സ്വഹീഹുല്‍ ബുഖാരിയിലെ കിതാബുത്തഫ്‌സീറിലും കിതാബുല്‍ മനാക്വിബില്‍ അന്‍സ്വാറിലും സ്വഹീഹുമുസ്‌ലിമിലെ ക്വിതാബുസ്വിഫാത്തില്‍ ക്വിയാമതി വല്‍ജന്നതി വന്നാറിലും ചന്ദ്രന്‍ രണ്ടായി പിളര്‍ന്നുനീങ്ങിയെന്ന് ഖണ്ഡിതമായി പ്രസ്താവിക്കുന്ന ഹദീഥുകളുണ്ട്. മക്കക്കാര്‍ പ്രവാചകനോട് ദൃഷ്ടാന്തങ്ങള്‍ക്കുവേണ്ടി ചോദിച്ചപ്പോഴാണ് നബി (സ) ചന്ദ്രന്‍ പിളരുന്നത് കാണിച്ചുകൊടുത്തതെന്ന് സൂചിപ്പിക്കുന്ന ഹദീഥുകള്‍ സ്വഹീഹുല്‍ ബുഖാരിയിലെ കിതാബുല്‍ മനാക്വിബില്‍ അന്‍സ്വാറിലും കിതാബു ഫദാഇലി അസ്‌വ്ഹാബിന്നബിയിലും കാണാന്‍ കഴിയും. സംഭവം നടക്കുമ്പോള്‍ നബി (സ) മക്ക നഗരത്തിന് തൊട്ടപ്പുറത്തുള്ള മിനായിലായിരുന്നുവെന്ന് സ്വഹീഹുല്‍ ബുഖാരിയിലെ കിതാബുല്‍ മനാക്വിബില്‍ അന്‍സ്വാറില്‍നിന്ന് മനസ്സിലാക്കാനാകും.

ചന്ദ്രന്‍ പിളര്‍ന്നുണ്ടായ രണ്ട് കഷ്ണങ്ങളില്‍ ഒന്ന് ഒരു പര്‍വതത്തിന്റെ ഒരു വശത്തും മറ്റേത് മറുവശത്തുമായി നിന്നുവെന്നും അപാരമായ വ്യക്തതയുണ്ടായിരുന്ന ആ പിളര്‍ന്നുമാറല്‍ ദൃശ്യത്തെ ചൂണ്ടി പ്രവാചകന്‍ (സ) കൂടെയുണ്ടായിരുന്നവരോട് ‘സാക്ഷിയായിക്കൊള്ളുക’ എന്നു പറഞ്ഞുവെന്നും സ്വഹീഹുല്‍ ബുഖാരിയിലെ കിതാബുത്തഫ്‌സീറിലെയും സ്വഹീഹുമുസ്‌ലിമിലെ ക്വിതാബുസ്വിഫാത്തില്‍ ക്വിയാമതി വല്‍ജന്നതി വന്നാറിലെയും ഹദീഥുകളിലുണ്ട്. വിസ്മയകരമായ ഈ അനുഭവങ്ങള്‍ക്ക് സാക്ഷിയായി പ്രവാചക സന്നിധിയിലുണ്ടായിരുന്നവരില്‍ താനുമുള്‍പ്പെട്ടിരുന്നതായി അബ്ദുല്ലാഹിബ്‌നു മസ്ഊദ് (റ) സാക്ഷ്യപ്പെടുത്തിയത് മുസ്‌ലിമിലെ ക്വിതാബുസ്വിഫാത്തില്‍ ക്വിയാമതി വല്‍ജന്നതി വന്നാറില്‍ ഉദ്ധരിക്കപ്പെട്ടിരിക്കുന്നു.

മുഹമ്മദ് നബി(സ)യുടെ ജീവിതകാലത്ത് തന്റെ പ്രവാചകത്വത്തിന് തെളിവായി അല്ലാഹു ചന്ദ്രനെ രണ്ടു ഭാഗമാക്കി മാറ്റിനിര്‍ത്തുന്നത് അദ്ദേഹം തന്റെ ചുറ്റുമുണ്ടായിരുന്ന ചിലര്‍ക്ക് കാണിച്ചുകൊടുത്തുവെന്ന ഇസ്‌ലാമിക പാരമ്പര്യത്തിന് ചരിത്രപരമായ അടിത്തറയില്ലെന്ന മിഷനറി വാദം ശുദ്ധ അസംബന്ധമാണെന്ന് ക്വുര്‍ആനും ഹദീഥുകളും പരിശോധിക്കുന്ന ആര്‍ക്കും മനസ്സിലാകുമെന്നാണ് പറഞ്ഞുവന്നതിന്റെ ചുരുക്കം. ഒട്ടനേകം അനിഷേധ്യമായ രേഖീകരണങ്ങളുള്ള ചാന്ദ്രപ്പിളര്‍പ്പ് നടന്നത് ചരിത്രത്തിന്റെ വെള്ളിവെളിച്ചത്തിലാണെന്ന് പറയുന്നതായിരിക്കും ശരി.

ഇങ്ങനെയൊരു സംഭവം കണ്ടതായി ഹിജാസിനു പുറത്തുള്ള ആരും രേഖപ്പെടുത്തിയിട്ടില്ലെന്നു വാദിച്ചാണ് ചില മിഷനറിമാര്‍ ചന്ദ്രന്‍ പിളര്‍ന്നതിന്റെ ചരിത്രപരത നിഷേധിക്കുവാന്‍ ശ്രമിക്കാറുള്ളത്. യഥാര്‍ത്ഥത്തില്‍ ചന്ദ്രന്റെ ദൃശ്യത ഒരു സമയത്തും ഭൂഗോളത്തില്‍ എല്ലാ ഭാഗത്തും ഒരു പോലെയല്ലെന്നും മക്കയുടെ ചക്രവാളം പങ്കിടുന്ന ഭൂപ്രദേശങ്ങളില്‍ തന്നെ മുന്‍പ്രഖ്യാപനമോ ആഗോളശ്രദ്ധ നേടിയ വിളംബരങ്ങളോ ഇല്ലാതെ ആകാശത്ത് പൊടുന്നനെ സംഭവിക്കുകയും ഏതാനും സമയത്തിനകം അവസാനിക്കുകയും ചെയ്ത ഒരു സംഭവം ശ്രദ്ധിക്കപ്പെടാതെ പോവുക തികച്ചും സ്വാഭാവികമാണെന്നും വാനനിരീക്ഷണം ആധുനിക കാലത്തേതുപോലെ വ്യവസ്ഥാപിതമായിത്തീരാത്ത ഒരു കാലത്ത് അസാധാരണമായ ഒരു ആകാശക്കാഴ്ചക്ക് നിരീക്ഷകക്കുറിപ്പുകളുടെ ആധിക്യമുണ്ടാകുമെന്ന് പ്രതീക്ഷിക്കുന്നത് ബാലിശമാണെന്നും മേഘപടലങ്ങളോ ഏതാനും സമയം എന്തു സംഭവിച്ചുവെന്ന് മനസ്സിലാകാതിരിക്കലോ മതി ഇത്തരമൊരു സന്ദര്‍ഭം കുറേ പ്രദേശങ്ങളുടെ ‘ചരിത്ര’ത്തില്‍ നിന്ന് ‘പുറത്താകാന്‍’ എന്നും കണ്ടതെല്ലാം എല്ലാ കാലത്തും എല്ലാ മനുഷ്യരും രേഖപ്പെടുത്താറില്ലെന്നും രേഖപ്പെടുത്തിയതെല്ലാം ചരിത്രഗവേഷകര്‍ കണ്ടെത്തിക്കഴിഞ്ഞിട്ടില്ലെന്നുമുള്ള ലളിതവസ്തുതകള്‍ മാത്രം കണക്കിലെടുത്താല്‍ മതി ഈ പരിശ്രമങ്ങള്‍ എന്തുമാത്രം അര്‍ത്ഥശൂന്യമാണെന്നു മനസ്സിലാകാന്‍.

മക്കയില്‍ സാക്ഷികളുണ്ടായതിന് അനിഷേധ്യമായ ചരിത്രരേഖകളുള്ള ഒരു ആകാശദൃശ്യത്തെ തള്ളിക്കളയുവാനുള്ള യാതൊരു ന്യായവും മിഷനറിമാരുന്നയിക്കുന്ന മറുവാദങ്ങളിലൊന്നുമില്ലെന്നു സാരം. പ്രവാചകകാലഘട്ടത്തിലെ ചാന്ദ്രപ്പിളര്‍പ്പിന് സാക്ഷിയാവുകയോ അതിന്റെ വാര്‍ത്തകളോട് പ്രതികരിക്കുകയോ ചെയ്ത ഇന്‍ഡ്യന്‍ രാജാക്കന്‍മാരെ സംബന്ധിച്ച പ്രചുരപ്രചാരമാര്‍ജ്ജിച്ച പാരമ്പര്യങ്ങളിലേക്ക് ഇവിടെ പ്രവേശിക്കാത്തത് അവയ്ക്ക് ചരിത്രപരത ഇല്ലെന്നുവന്നാലും ചാന്ദ്രപ്പിളര്‍പ്പ് ദൃശ്യം ഒരു ചരിത്രസംഭവമാണെന്ന യാഥാര്‍ത്ഥ്യത്തെ അതൊരിക്കലും ബാധിക്കാന്‍ പോകുന്നില്ല എന്നതിനാലാണ്.

ചാന്ദ്രപ്പിളര്‍പ്പ് ആകാശത്ത് മക്കക്കാര്‍ക്ക് ദൃശ്യമായിയെന്നുറപ്പിക്കുകയാണ് ചരിത്രത്തിന്റെ ദൗത്യം; ആ ദൗത്യമാണ് ചരിത്രശാസ്ത്രത്തിന്റെ വീക്ഷണത്തില്‍ നാം ചര്‍ച്ച ചെയ്ത രേഖകള്‍ നിര്‍വഹിക്കുന്നത്. പ്രസ്തുത ദൃശ്യം എങ്ങനെയുണ്ടായതാണെന്ന് പറയാന്‍ ചരിത്രത്തിന് കഴിഞ്ഞുകൊള്ളണമെന്നില്ല. ചന്ദ്രനെ അല്ലാഹു ഒരമാനുഷിക ദൃഷ്ടാന്തമെന്ന നിലയില്‍ പിളര്‍ത്തിയതാണെന്ന മുഹമ്മദ് നബി(സ)യുടെ വിശദീകരണം വിശ്വാസത്തിന്റെ മണ്ഡലത്തിലുള്ളതാണ്. മിഷനറിമാര്‍ക്ക് പരമാവധി ചെയ്യാനാവുക ആ വിശദീകരണത്തോട് വിയോജിക്കുകയാണ്; മക്കയിലെ ബഹുദൈവാരാധകര്‍ ചെയ്തതുപോലെ. ചന്ദ്രന്‍ പിളര്‍ന്നതായുള്ള ഒരു പ്രതീതി തങ്ങളുടെ കണ്ണുകള്‍ക്കുണ്ടാകുന്ന കണ്‍കെട്ടാണ് മുഹമ്മദ് നബി (സ) നടത്തിയതെന്നാണ് അവര്‍ ആരോപിച്ചത്. ഇത് ചാന്ദ്രപ്പിളര്‍പ്പിന്റെ കാര്യത്തില്‍ മാത്രമല്ല, ക്വുര്‍ആന്‍ അടക്കമുള്ള മുഴുവന്‍ നബിദൃഷ്ടാന്തങ്ങളുടെയും കാര്യത്തില്‍ അവരുയര്‍ത്തിയ നിലപാടാണ്. തങ്ങള്‍ നിഷേധിച്ചുതള്ളിയ പ്രവാചകനിലൂടെ അത്യത്ഭുകരമായ ദൃഷ്ടാന്തങ്ങള്‍ പ്രത്യക്ഷപ്പെടുന്നത് കണ്ടുണ്ടായ അമ്പരപ്പ് മറച്ചുവെക്കാന്‍ യാതൊരാത്മാര്‍ത്ഥതയുമില്ലാതെ അവര്‍ പറഞ്ഞുേനാക്കിയ ഒഴിവുകഴിവ് മാത്രമാണതെന്ന് ചരിത്രം സൂക്ഷ്മമായി പരിശോധിക്കുന്നവര്‍ക്കു മനസ്സിലാകും. കണ്‍കെട്ടിന്റെ പരിമിതശേഷികള്‍കൊണ്ട് സാധ്യമാകുന്നവയല്ല തങ്ങള്‍ സാക്ഷ്യം വഹിച്ചുകൊണ്ടിരുന്ന നബിയത്ഭുതങ്ങളെന്നും അവ ദൈവികമായ ഇടപെടലാകാനേ തരമുള്ളൂ എന്നും അവര്‍ക്ക് വ്യക്തമായിരുന്നു. ദുര്‍വാശിയില്‍ നിന്നുണ്ടായ കപട ആരോപണമായിരുന്നു ജാലവിദ്യയുടേത്. ചാന്ദ്രപിളര്‍പ്പ് സംഭവത്തിന്റെ വിശദാംശങ്ങള്‍ തന്നെ ഇക്കാര്യം വ്യക്തമാക്കുന്നുണ്ട്.

സംഭവം മാരണജന്യമായ വ്യാജപ്രതീതിയോ മറ്റോ ആണെന്ന് വരുത്താന്‍, നമ്മള്‍ മാത്രമേ (നബിയുടെ തൊട്ടടുത്തുള്ളവര്‍) ഇങ്ങനെ കാണുന്നുണ്ടായിരിക്കുകയുള്ളൂ എന്നും അങ്ങനെ സംഭവിക്കുന്നത് കണ്‍കെട്ടായതുകൊണ്ടാണെന്നും മറ്റാരെങ്കിലും കണ്ടതായി തെളിഞ്ഞാല്‍ മാത്രമേ ഇത് ജാലവിദ്യയല്ലെന്നു വരൂ എന്നും അവര്‍ പ്രഖ്യാപിച്ചതായി ജാമിഉത്തിര്‍മിദിയിലെ കിതാബു തഫ്‌സീറില്‍ ക്വുര്‍ആനി അന്‍ റസൂലില്ലാഹിയില്‍ രേഖപ്പെടുത്തിയിട്ടുള്ള ഹദീഥില്‍ വായിക്കാന്‍ കഴിയും. സിഹ്ര്‍ എന്നവര്‍ വിളിച്ചിരുന്ന പലതരം കണ്‍കെട്ടു വിദ്യകളുടെ പരിമിതിയെക്കുറിച്ചുള്ള ബോധ്യമാണ് ഈ വര്‍ത്തമാനത്തില്‍ നിഴലിക്കുന്നത്. പിന്നീടെന്ത് സംഭവിച്ചുവെന്ന് പ്രഗല്‍ഭനായ ആദ്യകാല ക്വുര്‍ആന്‍ വ്യാഖ്യാതാവും ചരിത്രകാരനുമായ അബൂ ജഅ്ഫര്‍ ഇബ്‌നു ജരീറുത്ത്വബ്‌രി തന്റെ ബൃഹദ് ക്വുര്‍ആന്‍ വ്യാഖ്യാനഗ്രന്ഥമായ ജാമിഉല്‍ ബയാനി അന്‍ തഅ്‌വീലി അയ്യുല്‍ ക്വുര്‍ആനില്‍ ചാന്ദ്രപ്പിളര്‍പ്പിനെ പരാമര്‍ശിക്കുന്ന ക്വുര്‍ആന്‍ വചനങ്ങളുടെ (54: 1-3) വിശദീകരണക്കുറിപ്പില്‍ ഉള്‍പ്പെടുത്തിയ നിവേദനത്തിലുണ്ട്. ചന്ദ്രന്‍ പിളര്‍ന്നത് കണ്ടുവോ എന്ന് ആ രാത്രി മക്കക്കു പുറത്തായിരുന്ന ഏതെങ്കിലും യാത്രക്കാരോട് ചോദിക്കാന്‍ അവര്‍ തീരുമാനിച്ചു. അങ്ങനെ ഒരു യാത്രാസംഘത്തോട് അവര്‍ അന്വേഷിച്ചു. ‘അതെ, ഞങ്ങള്‍ അത് കണ്ടു’വെന്നായിരുന്നു അവരുടെ മറുപടി. അപ്പോഴാണ് ഈ ക്വുര്‍ആന്‍ വചനങ്ങള്‍ അവതരിപ്പിക്കപ്പെട്ടത്. (കെയ്‌റോ, ദാറുല്‍ ഹിജ്ര്‍, 2001, Vol. 23, pp.106-7).

തങ്ങള്‍ കണ്ട ആകാശദൃശ്യം കണ്‍കെട്ടല്ലെന്ന് ഇതോടുകൂടി സത്യനിഷേധികള്‍ക്ക് ബോധ്യം വന്നുവെന്ന കാര്യമുറപ്പാണ്. കണ്‍കെട്ടിന് തങ്ങള്‍ തന്നെ വെച്ച മാനദണ്ഡങ്ങള്‍ ചാന്ദ്രപിളര്‍പ്പിനു ബാധകമല്ലെന്നു മനസ്സിലായിട്ടും പ്രസ്തുത വാദത്തിലവര്‍ ഉറച്ചുനിന്നത് സത്യത്തോട് വിമുഖരാകുമാറ് അവരില്‍ രൂഢമൂലമായിരുന്ന അഹങ്കാരം കൊണ്ടുമാത്രമാണെന്നര്‍ത്ഥം. ചന്ദ്രന്‍ പിളര്‍ന്നുനിന്ന ആകാശദൃശ്യം എന്ന അനിഷേധ്യമായ ചരിത്രസംഭവത്തെ മക്കന്‍ ബഹുദൈവാരാധകരുടെ കൂടെനിന്ന് മന്ത്രവാദവല്‍കരിക്കുവാനുള്ള ശ്രമങ്ങള്‍ ഇന്നും അഹങ്കാരത്തില്‍ നിന്നുമാത്രമാണ് നിര്‍ഗളിക്കുക; അതിനോട് സംവദിക്കാന്‍ ചരിത്രത്തിന്റെ ആയുധങ്ങള്‍ മാത്രം മതിയാകില്ല.

യഥാര്‍ത്ഥത്തില്‍, പ്രവാചകന്‍മാരിലൂടെ വെളിപ്പെടുന്ന അത്ഭുതസംഭവങ്ങളെ ജാലവിദ്യയോ മാരണമോ വഴിയുള്ള കണ്‍കെട്ടായി തള്ളിക്കളഞ്ഞ് പടച്ചവനു കാണിക്കാന്‍ കഴിയുന്നതൊക്കെ കാണിക്കാന്‍ കഴിയുന്ന സൂപ്പര്‍ പവറുകളായി കണ്‍കെട്ടുകാരെ അവതരിപ്പിക്കാന്‍ മുതിര്‍ന്ന അസംബന്ധ നാടകക്കാര്‍ ചരിത്രത്തിന്റെ എല്ലാ ഘട്ടങ്ങളിലുമുണ്ടായിട്ടുണ്ട്. കണ്‍കെട്ടുകാരുടെ ചെപ്പടിവിദ്യകളുമായി താരതമ്യം പോലുമില്ലാത്ത മഹാത്ഭുതങ്ങളാണ് സംഭവിക്കുന്നതെന്നു മനസ്സിലായിട്ടും അവയെ ഇപ്രകാരം തള്ളിക്കളഞ്ഞ നിര്‍ഭാഗ്യവാന്‍മാരെ സംബന്ധിച്ച് വിവിധ പ്രവാചകന്‍മാരെകുറിച്ച ബൈബിള്‍ കഥനങ്ങളില്‍ നിന്ന് മനസ്സിലാക്കിയിട്ടും നബി(സ)യുടെ കാര്യം വരുമ്പോള്‍ അതേ നിര്‍ഭാഗ്യത്തിലേക്ക് മുഖം കുത്തുന്ന ദുര്യോഗമാണ് മിഷനറിമാര്‍ക്ക് സംഭവിക്കുന്നത്.

വടി നിലത്തിട്ടാല്‍ സര്‍പ്പമാകുന്ന ദൈവികദൃഷ്ടാന്തവുമായി രാജസദസ്സിലേക്കു കടന്നുവന്ന മോശെയോടും അഹറോനോടും അത് കണ്‍കെട്ടാണെന്ന് വാദിക്കുകയും കൊട്ടാരത്തിലെ ആസ്ഥാന കണ്‍കെട്ടുകാരെക്കൊണ്ടുവന്ന് അവരുടെ കയറുകളും വടികളുംവെച്ച് ‘പ്രതീതി സര്‍പ്പങ്ങളെ’ ഉണ്ടാക്കിപ്പിക്കുകയും മോശെയുടെ സര്‍പ്പം തങ്ങളുടെ വ്യാജസര്‍പ്പങ്ങളെ വിഴുങ്ങുന്നതുകണ്ട് മോശെയുടേത് കണ്‍കെട്ടല്ലെന്നു കണ്‍കെട്ടുകാര്‍ക്കടക്കം ബോധ്യം വന്നിട്ടും ‘കണ്‍കെട്ടുവാദ’ത്തില്‍ ധാര്‍ഷ്ഠ്യത്തോടെ നിലനില്‍ക്കുകയും ചെയ്ത ഫറോവയെക്കുറിച്ച് ക്വുര്‍ആന്‍ വിശദമായും (20: 9-71) ബൈബിള്‍ അല്‍പം കൂടി ചുരുക്കിയും (പുറപ്പാട് 7: 1-13) -വിശദാംശങ്ങളില്‍ ചില വ്യത്യാസങ്ങളുണ്ടെങ്കിലും- വിശദീകരിക്കുന്നുണ്ട്. ഫറോവയുടെ അനന്തരഗാമിത്വമാണ് തങ്ങള്‍ക്കുവേണ്ടതെന്ന് തീരുമാനിക്കുവാന്‍ മിഷനറിമാര്‍ക്ക് തീര്‍ച്ചയായും സ്വാതന്ത്ര്യമുണ്ട്; പക്ഷേ അത് മോശെയുടെയും യേശുവിന്റെയും മേല്‍വിലാസത്തിലാകുമ്പോഴാണ് നമുക്ക് സങ്കടം! ഫറോവയുടെ മനഃശാസ്ത്രം കടമെടുക്കുന്നതു കൊണ്ടല്ലാതെ മറ്റെന്തുകൊണ്ടാണ്.

 പ്രപഞ്ചനാഥന്‍ മോശെക്കുവേണ്ടി ചെങ്കടല്‍ പിളര്‍ത്തിയതും (പുറപ്പാട് 14: 21-31) ജലം രക്തമാക്കി മാറ്റിയതും (പുറപ്പാട് 7: 14-25) അഹറോനുവേണ്ടി വെറും വടിയില്‍ തളിരും പൂവും കായുമുണ്ടാക്കിയതും (സംഖ്യ 17: 1-10) യോശുവക്കുവേണ്ടി പ്രളയകാലത്ത് ജോര്‍ദ്ദാന്‍ നദിയുടെ ഒഴുക്കു നിര്‍ത്തിയതും (യോശുവ 3: 1-17) ബാലാമിനോട് അദ്ദേഹത്തിന്റെ കഴുതയെക്കൊണ്ട് സംസാരിപ്പിച്ചതും (സംഖ്യ 22: 22-30) ഏലിശക്കുവേണ്ടി മരിച്ച കുട്ടിക്ക് ജീവന്‍ നല്‍കിയതും (1 രാജാക്കന്‍മാര്‍ 17: 5-24) അടക്കമുള്ള പഴയനിയമപ്രകാരമുള്ള അത്ഭുതങ്ങളിലും യേശുക്രിസ്തുവിന്റെ ജനനവും ജീവിതവും മരണവുമെല്ലാം അത്ഭുതങ്ങളില്‍ കുതിര്‍ത്തുനിര്‍ത്തുന്ന പുതിയ നിയമവിവരണങ്ങളിലും കണ്ണുമടച്ച് വിശ്വസിക്കുന്ന മിഷനറിമാര്‍ മുഹമ്മദ് നബി(സ)യെ വായിക്കുമ്പോള്‍ മാത്രം മുഴുവന്‍ പ്രവാചകന്‍മാരുടെയും ശത്രുപക്ഷത്തുനിന്ന കണ്‍കെട്ടു സിദ്ധാന്തത്തില്‍ അഭയം തേടുന്നത്. ബൈബിള്‍ കഥകളില്‍ വിവരിക്കപ്പെടുന്ന അത്ഭുതങ്ങളെക്കാള്‍ എന്ത് ‘അവിശ്വസനീത’യാണ് നബിജീവിതത്തിലെ ദൃഷ്ടാന്തങ്ങളില്‍ മിഷനറിമാര്‍ ‘കൂടുതലായി’ കാണുന്നത്?

ബൈബിള്‍, വസ്തുനിഷ്ഠമായി പറഞ്ഞാല്‍ ആധികാരികമായ ഒരു ചരിത്രസ്രോതസ്സല്ല. അതുകൊണ്ടുതന്നെ ബൈബിള്‍ വിവരണങ്ങളുടെ മാത്രം അടിസ്ഥാനത്തില്‍ ഒരത്ഭുതത്തെയും സ്ഥിരീകരിക്കാനാവില്ല. മിഷനറിമാര്‍ വിശ്വസിക്കുന്ന അത്ഭുതങ്ങള്‍ മിക്കതും യാതൊരു ചരിത്രപരതയുമില്ലാത്തതാണെന്ന് അതിനാല്‍ തന്നെ പറയാനാകും. എന്നാല്‍ ചന്ദ്രവിഭജനമടക്കമുള്ള നബിദൃഷ്ടാന്തങ്ങളുടെ കാര്യമതല്ലെന്ന് നാം കണ്ടു. ചരിത്രപരമായി സ്ഥാപിക്കാനാവാത്ത അത്ഭുതങ്ങളില്‍ വിശ്വസിക്കുന്നവര്‍ ചരിത്രം പൂര്‍ണമായി രേഖീകരിച്ച മുഹമ്മദ് നബി(സ)യുടെ ജീവിതത്തിലെ അത്ഭുതങ്ങളെ തള്ളിക്കളയുന്നതിലെ യുക്തി എന്താണ്?

അത്ഭുതങ്ങളെ സംബന്ധിച്ച വിശ്വാസിയുടെ സങ്കല്‍പത്തെ പരിഗണിക്കുന്ന ആര്‍ക്കും ചാന്ദ്രപിളര്‍പ്പുമായി ബന്ധപ്പെട്ട മിഷനറി വിമര്‍ശനങ്ങളുടെ അന്തസാര ശുന്യത ബോധ്യപ്പെടാതിരിക്കില്ല. ഒരു പ്രവാചകന്റെ കൈക്ക് സംഭവിച്ചുവെന്ന് ചരിത്രപരമായി സ്ഥിരീകരിക്കപ്പെടുന്നതോടുകൂടിത്തന്നെ പ്രസ്തുത അത്ഭുതങ്ങളെ ദൈവിക ദൃഷ്ടാന്തങ്ങള്‍ എന്ന നിലക്ക് മനസ്സിലാക്കുക എന്നതാണ് വിശ്വാസപരമായ സമീപനം. ‘അത്ഭുത’ങ്ങളുടെ മണ്ഡലം വിശ്വാസമാണെന്നു പറയുവാനുള്ള കാരണമതാണ്. ശാസ്ത്രത്തെ ഈ ചര്‍ച്ചയിലേക്കു വലിച്ചിഴക്കുവാനുള്ള മിഷനറി ശ്രമം ഏറ്റവും വലിയ അശ്ലീലമായിത്തീരുന്നതും അതുകൊണ്ടു തന്നെ. പ്രപഞ്ചവും പദാര്‍ത്ഥവും അവ സാധാരണഗതിയില്‍ പിന്തുടരുന്ന നിയമങ്ങള്‍വെച്ച് മനസ്സിലാക്കുകയാണ് ശാസ്ത്രത്തിന്റെ ധര്‍മം. അത്ഭുതങ്ങള്‍ ആ നിയമങ്ങളെ മറികടന്ന് സംഭവിക്കുന്ന കാര്യങ്ങളാണെന്നും അതുകൊണ്ടാണ് അവ അത്ഭുതങ്ങളാകുന്നതെന്നും മിഷനറിമാര്‍ക്കറിയില്ലേ? പിന്നെ അവയെ ‘ശാസ്ത്രീയമായി’ തെളിയിക്കാന്‍ ആവശ്യപ്പെടുന്നതിന്റെ ന്യായമെന്താണ്?

പ്രാപഞ്ചിക നിയമങ്ങളെ അതിലംഘിച്ചുകൊണ്ട് എപ്പോഴെങ്കിലും എന്തെങ്കിലും നടക്കുമോ എന്ന ചോദ്യത്തിന് ‘ഇല്ല’യെന്ന് തീര്‍ത്തും നിഷേധാത്മകമായി മറുപടി പറയുക ഭൗതികവാദികള്‍ മാത്രമാണ്. പ്രപഞ്ചത്തിനൊരു സ്രഷ്ടാവുണ്ടെന്ന വസ്തുത അവരംഗീകരിക്കുന്നില്ലെന്നതാണ് അതിന്റെ കാരണം. പ്രപഞ്ചസ്രഷ്ടാവില്‍ വിശ്വസിക്കുന്നവരെ സംബന്ധിച്ചടുത്തോളം അവനാണ് സൃഷ്ടിലോകത്തിന്റെ നിയാമകന്‍. ‘സാധാരണ’യായി പിന്തുടരപ്പെടുന്ന പ്രാപഞ്ചിക നിയമങ്ങള്‍ നിര്‍മിച്ചത് അവനാണ്. ആ നിയമങ്ങള്‍ പ്രകാരമല്ലാതെ ചില കാര്യങ്ങള്‍ സംഭവിപ്പിച്ച് ‘അസാധാരണ’മായ സ്ഥിതിവിശേഷങ്ങള്‍ സൃഷ്ടിക്കുക സര്‍വശക്തനായ അവനെ സംബന്ധിച്ചേടുത്തോളം തീര്‍ത്തും സാധ്യമാണ്. അതുകൊണ്ടാണ് പ്രവാചകന്‍മാര്‍ക്ക് സാക്ഷീകരണമായി പ്രപഞ്ചനാഥന്‍ സംവിധാനിക്കുന്ന അത്ഭുതങ്ങളില്‍ വിശ്വാസി സംശയലേശമന്യേ വിശ്വസിക്കുന്നത്. അതുകൊണ്ടുതന്നെയാണ് മിഷനറിമാര്‍ ബൈബിളിലെ അത്ഭുതകഥകളില്‍ വിശ്വസിക്കുന്നതും. മുകളിലുദ്ധരിച്ച ബൈബിള്‍ അത്ഭുതങ്ങളൊന്നും തന്നെ ശാസ്ത്രീയമായി വിശദീകരിക്കാനാവില്ല; ആകുമായിരുന്നെങ്കില്‍ അവ അത്ഭുതങ്ങളാവുകയും ചെയ്യുമായിരുന്നില്ല.

ശരീരത്തില്‍ തീയേറ്റാല്‍ ചൂടും പൊള്ളലുമുണ്ടാകുമെന്നേ ശാസ്ത്രത്തിന് പറയാനാകൂ. നംറൂദ് അഗ്നികുണ്ഠത്തിലേക്കെറിഞ്ഞ അബ്രഹാമിന് തീ തണുപ്പായത് ക്വുര്‍ആന്‍ പറയുന്നുണ്ട് (21: 68-70). വിഗ്രഹപൂജകനായ ബാബിലോണിയന്‍ രാജാവ് പ്രതിമാപൂജക്ക് വിസമ്മതിച്ച മൂന്ന് ജൂതപുരോഹിതന്‍മാരെ തീയിലെറിഞ്ഞപ്പോള്‍ അവര്‍ ഒരു പൊള്ളലുമേല്‍ക്കാതെ അഗ്നിനാളങ്ങളില്‍ സൈ്വരവിഹാരം നടത്തി പുറത്തുവന്ന കഥ ബൈബിളിലുണ്ട് (ദാനിയേല്‍ 3: 10-27). ഇവിടെ ശാസ്ത്രം നിസ്സഹായമാണ്. ദൈവം ഇടപെട്ടുവെന്നു കരുതുന്ന വിശ്വാസിയോടും ദൈവമില്ലെന്നതിനാല്‍ ഇത് അസംഭ്യവമാണെന്നു കരുതുന്ന അവിശ്വാസിയോടും ‘വിഷയം പരിധിക്കു പുറത്താണ്’ എന്നുമാത്രമേ ശാസ്ത്രത്തിന് പറയാന്‍ കഴിയൂ. ചന്ദ്രന്‍ പിളര്‍ന്നത് ‘ശാസ്ത്രീയമായി’ വിശദീകരിക്കാന്‍ ആവശ്യപ്പെടുന്ന മിഷനറിമാര്‍ മറക്കുന്നത് ഈ വസ്തുതയാണ്; അത്ഭുതങ്ങളെ സംബന്ധിച്ച് അവരും മുസ്‌ലിംകളുമെല്ലാം ഒരുപോലെയംഗീകരിക്കുന്ന പ്രാഥമിക വസ്തുതയെ. പ്രകൃതിനിയമങ്ങള്‍വെച്ച് സ്വാഭാവികമായി സംഭവിച്ചിട്ടില്ലാത്തുകൊണ്ടാണ് ചന്ദ്രന്റെ പിളര്‍പ്പ് ഒരു അത്ഭുതവും പ്രവാചകത്വത്തിനുള്ള തെളിവുമായതെന്നും അത്തരമൊരു പ്രതിഭാസത്തിന്റെ പ്രായോഗികതയെക്കുറിച്ചുള്ള സംശയം ദൈവം തമ്പുരാന്റെ ശക്തിവിശേഷങ്ങളെക്കുറിച്ചുള്ള അജ്ഞതയില്‍ നിന്നുണ്ടാകുന്നതാണെന്നും തിരിച്ചറിയാന്‍ മതവും ശാസ്ത്രവും എന്താണെന്നതിനെക്കുറിച്ച പ്രാഥമിക ധാരണകള്‍ മാത്രമേ ആവശ്യമുള്ളൂ.

അത്ഭുതങ്ങളില്‍ വിശ്വസിക്കുകയും അവ ശാസ്ത്രീയമായി വിശദീകരണക്ഷമമാവുകയില്ലെന്നു തിരിച്ചറിയുകയും ഏതെങ്കിലും നിയമങ്ങളുടെ നിര്‍ബന്ധിത വരുതിക്കുള്ളിലുള്ളവനല്ല പടച്ചവനെന്ന് മനസ്സിലാക്കുകയും ചെയ്യുമ്പോള്‍ തന്നെ ചാന്ദ്രപിളര്‍പ്പിനെ ശാസ്ത്രീയമായി നിഷേധിക്കുവാന്‍ ശ്രമിക്കുന്നത് അത് ഒരു വാനലോകാത്ഭുതമായതു കൊണ്ടാണെന്നാണ് ചില മിഷനറിമാര്‍ തങ്ങളുടെ ഊന്നലുകള്‍ കൊണ്ട് ധ്വനിപ്പിക്കുവാന്‍ ശ്രമിക്കാറുള്ളത്. വാനവും ചന്ദ്രനുമെല്ലാം സര്‍വശക്തന്റെ അധികാരപരിധിക്കു പുറത്താണെന്നാണോ ഇവര്‍ ധരിച്ചിരിക്കുന്നത്?

മണ്‍തരിയും പുല്‍ക്കൊടിയും മാത്രമല്ല, ഗ്രഹവും ഉപഗ്രഹവും നക്ഷത്രവുമെല്ലാം അവന്റേതു മാത്രമാണെന്നും അവയ്ക്കു നിയമങ്ങള്‍ നിശ്ചയിക്കുന്നതും ഇച്ഛിക്കുമ്പോള്‍ നിയമങ്ങള്‍ക്കു പുറത്തേക്ക് അവയെ കൊണ്ടുപോകുന്നതും അവന്‍ മാത്രമാണെന്നും എന്നാണ് ഇവരെല്ലാം തിരിച്ചറിയുക? ഭൂമിയും ചന്ദ്രനും സൂര്യനും ഗ്രഹങ്ങളുമെല്ലാമുള്‍ക്കൊള്ളുന്ന ഒരു സംവിധാനത്തില്‍ ചാന്ദ്രപിളര്‍പ്പ് സൃഷ്ടിക്കാവുന്ന ഭൗതിക ‘പ്രത്യാഘാത’ങ്ങളെക്കുറിച്ചാകുലപ്പെടുന്നവര്‍, ഇപ്പറഞ്ഞതൊക്കെയും അവന്റേതാണെന്നും ഭൗതികശാസ്ത്ര സമവാക്യങ്ങള്‍ പ്രകാരമുള്ള പ്രതിഫലനങ്ങള്‍ക്കിടം നല്‍കി മാത്രമല്ല, ‘ചുറ്റപാടുകളെ അറിയിക്കാതെ’യും കാര്യങ്ങള്‍ നടത്താന്‍ കഴിവുള്ളവനാണവന്‍ എന്നുമുള്ള സത്യങ്ങളാണ് മറന്നുപോകുന്നത്. ചന്ദ്രോപരിതലത്തില്‍ പിളര്‍പ്പിന്റെ അടയാളങ്ങള്‍ ശേഷിക്കുന്നുണ്ടോ എന്ന ചോദ്യമുയര്‍ത്തുന്നവര്‍, പിളര്‍ത്താന്‍ മാത്രമല്ല അടയാളങ്ങളൊന്നും ബാക്കിവെക്കാതെ ചേര്‍ക്കാനും പ്രപഞ്ചനാഥനു കഴിയുമെന്നും ചന്ദ്രോപരിതലത്തിലുള്ള എല്ലാ ‘അടയാള’ങ്ങളും നാം വായിച്ചുകഴിഞ്ഞിട്ടില്ലെന്നും ഓര്‍ക്കുന്നതില്‍ ദയനീയമായി പരാജയപ്പെടുന്നതെന്തുകൊണ്ടാണ്?

അന്ധമായ നബിവിരോധം എന്താണ് ഒരു അത്ഭുതം എന്നും ആരാണ് പടച്ചവന്‍ എന്നുമുള്ള അടിസ്ഥാന യാഥാര്‍ത്ഥ്യങ്ങളെപ്പോലും വിസ്മരിച്ചുകൊണ്ടുള്ള പരിമിതബുദ്ധിയുടെ ഉന്മാദത്തിലേക്കും ചോദ്യങ്ങളുടെ പ്രളയത്തിലേക്കും മിഷനറിമാരുടെ കൈപിടിച്ചുകൊണ്ടുപോകുന്നുണ്ടെങ്കില്‍, ഇതേ ഖഗോളശാസ്ത്ര’ജ്ഞാനം’ ബൈബിള്‍ വായിക്കാന്‍ കൂടി അവര്‍ക്കുപയോഗിക്കാവുന്നതാണ്! അഹറോന്റെയും മോശെയുടെയും മരണശേഷം മോവാബ് ദേശത്തുവെച്ച് ഇസ്രയേല്‍ സമൂഹത്തിന്റെ നായകത്വമേറ്റെടുത്തശേഷം അവരെയുംകൊണ്ട് ജോര്‍ദ്ദാന്‍ നദി മുറിച്ചുകടന്ന് വാഗ്ദത്ത കാനാന്‍ ദേശത്തെത്തിയ യോശുവ, അമോര്യ രാജാക്കന്‍മാരുമായി നടത്തിയ യുദ്ധത്തില്‍ സമയം കൂടുതല്‍ ലഭിക്കാന്‍ വേണ്ടി സൂര്യനെ ഗിബയോണ്‍ താഴ്‌വരക്കു മുകളിലും ചന്ദ്രനെ അയ്യലോണ്‍ താഴ്‌വരക്കുമുകളിലും ഒരു ‘ദിവസം’ മുഴുവന്‍ ‘നിശ്ചലമാക്കി’ നിര്‍ത്തിയതിനെക്കുറിച്ച് (യോശുവ 10: 12-14) അവരുടെ അഭിപ്രായമെന്താണ്?

യേശു കുരിശില്‍ തറക്കപ്പെട്ടതിന്റെ ‘ഫലമായി’ ‘സൂര്യപ്രകാശം’ നിലച്ച് രാജ്യം മുഴുക്കെ മണിക്കൂറുകളോളം കനത്ത ‘ഇരുട്ട് പരക്കുകയും’ ദേവാലയത്തിലെ തിരശ്ശീല കീറുകയും ഭൂമി വിറക്കുകയും പാറകള്‍ പിളരുകയും ചെയ്തതിലെ (മത്തായി 27: 45-51, മാര്‍ക്കോസ് 15-33, ലൂക്കോസ് 23-44) ‘കാര്യകാരണ ബന്ധ’ത്തെയും ‘ശാസ്ത്രീയത’യെയും കുറിച്ച് അവര്‍ക്കെന്താണ് പറയാനുള്ളത്? ഒടുവിലൊടുവില്‍ നിങ്ങള്‍ നബി(സ)യെത്തോല്‍പിക്കാന്‍ പറഞ്ഞെത്തുന്നത് ഭൗതികവാദത്തിന്റെ അരമനയിലാണോ?

മുഹമ്മദ് നബി(സ)യുടെ പ്രവാചകത്വത്തിന്റെ ആരംഭത്തെക്കുറിച്ച് മുസ്‌ലിംകള്‍ പറയുന്ന കഥ, വഹ്‌യിനെക്കുറിച്ച അദ്ദേഹത്തിന്റെ അവകാശവാദങ്ങള്‍ അടിസ്ഥാനരഹിതമാണെന്നാണ് വ്യക്തമാക്കുന്നുണ്ട്. ദൈവികമായ പ്രചോദനമുണ്ടാകുമ്പോള്‍ ഉണ്ടാകേണ്ട സമാധാനത്തിനും സന്തോഷത്തിനും പകരം ഭയവിഹ്വലനായി വീട്ടില്‍ തിരിച്ചെത്തുന്ന മുഹമ്മദിനെയാണ് വെളിപാട് തുടങ്ങിയെന്ന് പറയപ്പെടുന്ന ദിവസം ചരിത്രത്തില്‍ നാം കാണുന്നത്. ദൈവദൂതനായ ജിബ്‌രീല്‍ തന്നെയാണ് പ്രവാചകനടുക്കല്‍ വന്നിരുന്നതെങ്കില്‍ ഇതാകുമായിരുന്നില്ല അനുഭവം. അതിനാല്‍ മുഹമ്മദ് നബിയ്ക്ക് നാല്‍പതാം വയസ്സിലുണ്ടായിയെന്ന് പറയപ്പെടുന്ന വെളിപാട് ദൈവത്തില്‍ നിന്നല്ലെന്ന കാര്യം സ്പഷ്ടമാണ്. പ്രവാചകത്വാരംഭവുമായി ബന്ധപ്പെട്ട നിവേദനങ്ങളെ വിശകലനം ചെയ്തുകൊണ്ട് നബി (സ) അനുഭവിച്ച വെളിപാട് ദിവ്യമല്ലെന്ന് വരുത്തിത്തീര്‍ക്കാന്‍ വേണ്ടി നബിവിമര്‍ശകര്‍ ഉന്നയിക്കുന്ന പരാമൃഷ്ടവാദങ്ങള്‍ ന്യായമല്ലേ?

ല്ല. അന്തിമപ്രവാചകന് ദിവ്യവെളിപാടുകള്‍ കിട്ടിത്തുടങ്ങിയ സന്ദര്‍ഭത്തെക്കുറിച്ചുള്ള ചരിത്രരേഖകളൊന്നും പ്രസ്തുതവെളിപാടുകളുടെ ദൈവികതയെ സംശയാസ്പദമാക്കുന്ന യാതൊരു പരാമര്‍ശവും ഉള്‍ക്കൊള്ളുന്നില്ല. പ്രവാചകന്റെ നാല്‍പതാം വയസ്സിലുണ്ടായ പ്രവാചകത്വലബ്ധിയെയും വഹ്‌യിന്റെ ആരംഭത്തെയും കുറിച്ചുള്ള കുറ്റമറ്റ നിവേദകപരമ്പരയിലൂടെയുള്ള വിവരണമുള്ളത് സ്വഹീഹുല്‍ ബുഖാരിയിലാണ്. പ്രവാചകപത്‌നി ആഇശ (റ), തന്റെ സഹോദരീപുത്രന്‍ ഉര്‍വക്ക് ആദ്യവെളിപാടിന്റെ സമയത്തെ പ്രവാചകാനുഭവങ്ങള്‍ പ്രവാചകന്റെ തന്നെ ആത്മകഥനാപരമായ വാചകങ്ങളുദ്ധരിച്ചുകൊണ്ട് സ്വന്തം വാക്കുകളില്‍ വിശദീകരിച്ചുകൊടുത്തതാണ് ബുഖാരി തന്റെ സ്വഹീഹിന്റെ തുടക്കത്തില്‍ ഹദീഥായി രേഖപ്പെടുത്തിയിട്ടുള്ളത് (കിതാബുല്‍ വഹ്‌യ്). പ്രവാചകത്വത്തിന്റെ ആരംഭത്തെക്കുറിച്ച് പരാമൃഷ്ട ഹദീഥ് നല്‍കുന്നത് താഴെക്കൊടുത്തിരിക്കുന്ന വിവരങ്ങളാണ്.

1. മുഹമ്മദ് നബി(സ)ക്ക് പകല്‍വെളിച്ചം പോലെ സത്യമായിപ്പുലരുന്ന സ്വപ്‌നദര്‍ശനങ്ങള്‍ നിരന്തരമായി ഉണ്ടാകാന്‍ തുടങ്ങി. ഇതായിരുന്നു പ്രവാചകനുമായുള്ള ദൈവിക ആശയവിനിമയത്തിന്റെ ആരംഭം.

2. ഉറക്കത്തില്‍ താന്‍ കാണുന്ന സ്വപ്‌നങ്ങളുടെ പുലര്‍ച്ച പ്രവാചകനെ ചിന്താകുലനാക്കുകയും അദ്ദേഹം മക്കയിലെ ഒരു പര്‍വതത്തിനുമുകളിലുള്ള ഹിറാഗുഹയില്‍ ഏകാന്തനായി പ്രാര്‍ത്ഥനകളില്‍ മുഴുകിയിരിക്കുന്ന ശീലം ആരംഭിക്കുകയും ചെയ്തു. രണ്ടോ മൂന്നോ ദിവസങ്ങള്‍ക്കാവശ്യമായ ജീവിതവിഭവങ്ങളുമായി ഗുഹയില്‍ തങ്ങുകയും ശേഷം വീട്ടിലേക്ക് മടങ്ങി വീണ്ടും വിഭവശേഖരണം നടത്തി ഗുഹയിലേക്കുതന്നെ തിരിച്ചുപോവുകയുമായിരുന്നു പതിവ്.

3. ഇങ്ങനെ പ്രവാചകന്‍ ഗുഹയിലിരിക്കുന്ന ഒരു ദിവസമാണ് ക്വുര്‍ആന്‍ അവതരണത്തിന് സമാരംഭം കുറിച്ചുകൊണ്ട് ഒരു മലക്ക് പ്രവാചകനരികില്‍ പ്രത്യക്ഷപ്പെട്ടത്.

4. മലക്ക് പ്രവാചകനോട് ‘ഇക്വ്‌റഅ്’ (വായിക്കുക/ഓതുക) എന്ന് കല്‍പിച്ചു. ‘ഞാന്‍ വായിക്കാനറിയുന്നവനല്ല’ (മാ അന ബി ക്വാരിഅ്) എന്നായിരുന്നു നബി(സ)യുടെ മറുപടി. അപ്പോള്‍ മലക്ക് പ്രവാചകന് താങ്ങാന്‍ കഴിയുന്നതിലുമപ്പുറത്തുള്ള ശക്തിയോടെ അദ്ദേഹത്തെ പിടിച്ചുഞെരുക്കുകയും ശേഷം സ്വതന്ത്രനാക്കുകയും ചെയ്ത് ‘ഇക്വ്‌റഅ്’ എന്ന കല്‍പന ആവര്‍ത്തിച്ചു. വീണ്ടും പ്രവാചകന്‍ പഴയ മറുപടി തന്നെ പറഞ്ഞു. മലക്ക് വീണ്ടും പ്രവാചകനെ ഞെരുക്കുകയും സ്വതന്ത്രനാക്കുകയും കല്‍പന ആവര്‍ത്തിക്കുകയും ചെയ്തു. മൂന്നാം തവണയും ഇതേഘട്ടങ്ങള്‍ കടന്നപ്പോള്‍ മലക്ക് പരിശുദ്ധ ക്വുര്‍ആനില്‍ 96-ാം അധ്യായം സൂറത്തുല്‍ അലക്വിലെ ആദ്യത്തെ അഞ്ച് വചനങ്ങള്‍ പ്രവാചകനെ ഓതിക്കേള്‍പിച്ചു. ഇതായിരുന്നു ക്വുര്‍ആന്‍ അവതരണത്തിന്റെയും പ്രവാചകത്വത്തിന്റെയും തുടക്കം. പ്രസ്തുത വചനങ്ങളുടെ സാരം ഇപ്രകാരമാണ്: ”സൃഷ്ടിച്ചവനായ നിന്റെ രക്ഷിതാവിന്റെ നാമത്തില്‍ വായിക്കുക. മനുഷ്യനെ അവന്‍ ഭ്രൂണത്തില്‍ നിന്ന് സൃഷ്ടിച്ചിരിക്കുന്നു. നീ വായിക്കുക: നിന്റെ രക്ഷിതാവ് പേനകൊണ്ട് പഠിപ്പിച്ചവനായ ഏറ്റവും വലിയ ഔദാര്യവാനാകുന്നു. മനുഷ്യന് അറിയാത്തത് അവന്‍ പഠിപ്പിച്ചിരിക്കുന്നു.”

5. ഗുഹയില്‍ ഏകാന്തനായിരിക്കെ മലക്കുമായുണ്ടായ മുഖാമുഖവും തത്സസമയത്തെ അനുഭവങ്ങളും പ്രവാചകനെ ഭയവിഹ്വലനാക്കി. പരിഭ്രാന്തനായി അതിവേഗത്തില്‍ മിടിച്ചുകൊണ്ടിരിക്കുന്ന ഹൃദയവുമായി വീട്ടിലേക്ക് മടങ്ങിയെത്തിയ അദ്ദേഹം പത്‌നി ഖദീജ ബിന്‍ത് ഖുവയ്‌ലിദിനോട് (റ) തന്നെ പുതപ്പിക്കുവാന്‍ ആവശ്യപ്പെട്ടു. വിറ നിന്ന് സാധാരണ നില കൈവരിക്കുവോളം പ്രവാചകന്‍ (സ) പുതച്ചുമൂടി നിന്നു. 6. അനന്തരം പുതപ്പില്‍നിന്ന് പുറത്തുവന്ന പ്രവാചകന്‍ ഉണ്ടായ സംഭവങ്ങള്‍ വിവരിക്കുകയും തനിക്ക് ഭയം അനുഭവപ്പെടുന്നുവെന്ന് പറയുകയും ചെയ്തു. അപ്പോള്‍ ഖദീജ (റ) ‘ഇല്ല, അല്ലാഹുവാണ് സത്യം, അല്ലാഹു അങ്ങയെ നിന്ദിക്കുകയില്ല. കാരണം അങ്ങ് ബന്ധുക്കളുടെ കാര്യം ശ്രദ്ധിക്കുന്നു, ദുര്‍ബലരുടെ ഭാരങ്ങള്‍ ചുമക്കുന്നു, ദരിദ്രര്‍ക്കും അശരണര്‍ക്കും സംരക്ഷണമേകുന്നു, അതിഥികളെ ആദരിക്കുന്നു, പ്രയാസപ്പെടുന്നവരെ സഹായിക്കുന്നു’ എന്നുപറഞ്ഞുകൊണ്ട് പ്രവാചകനെ ആശ്വസിപ്പിച്ചു. 7. പ്രവാചകനെ ആശ്വസിപ്പിച്ചശേഷം ഖദീജ അദ്ദേഹത്തെയും കൂട്ടി തന്റെ ബന്ധുവും ക്രൈസ്തവ-ജൂതഗ്രന്ഥങ്ങളില്‍ പ്രാവിണ്യമുണ്ടായിരുന്ന പണ്ഡിതനുമായിരുന്ന വറക്വത്ബ്‌നു നൗഫലിനടുത്തേക്കുപോയി. വറക്വ കാഴ്ചശക്തിയടക്കം നഷ്ടപ്പെട്ട് അങ്ങേയറ്റം വൃദ്ധനായിത്തീര്‍ന്നിരുന്നു. പ്രവാചകനുണ്ടായ അനുഭവങ്ങള്‍ കേട്ട വറക്വ, മോശെ (മൂസ) പ്രവാചകന്റെയടുക്കലേക്ക് ദിവ്യവെളിപാടുമായി വന്ന മലക്ക് ജിബ്‌രീല്‍ തന്നെയാണ് ഹിറാ ഗുഹയില്‍ വന്നതെന്ന് പറയുകയും പ്രവാചകന്‍ തനിക്ക് ലഭിച്ചുതുടങ്ങിയ ദിവ്യവെളിപാടുകള്‍ പ്രബോധനം ചെയ്യാന്‍ തുടങ്ങുമ്പോള്‍ പൂര്‍വപ്രവാചകന്‍മാരെപ്പോലെ കടുത്ത എതിര്‍പ്പുകള്‍ നേരിടുമെന്നും മക്കയില്‍നിന്ന് പുറത്താക്കപ്പെടുമെന്നും അന്ന് ഒരു യുവാവായി നാട്ടിലുണ്ടാകണമെന്നും പ്രവാചകനെ പിന്തുണക്കാന്‍ കഴിയണമെന്നും താന്‍ ആഗ്രഹിച്ചുപോകുന്നുവെന്നും പറഞ്ഞു. പക്ഷേ ഈ കൂടിക്കാഴ്ച കഴിഞ്ഞ് അധികമാകും മുമ്പെ അദ്ദേഹം മരണപ്പെട്ടുപോയി.

പരിശുദ്ധ ക്വുര്‍ആനിന്റെ അവതരണസമയത്ത് പ്രവാചകനുണ്ടായ പരാമൃഷ്ടാനുഭവങ്ങളില്‍ അസംഗതമായിട്ടെന്താണുള്ളതെന്ന് വ്യക്തമാക്കേണ്ടത് വിമര്‍ശകര്‍ തന്നെയാണ്. ഭൗതികവാദികളായ നബിവിമര്‍ശകര്‍ക്ക് വഹ്‌യ് എന്ന ആശയത്തെ തന്നെ ഉള്‍ക്കൊള്ളാന്‍ കഴിയാതിരിക്കുക സ്വാഭാവികമാണ്. പ്രപഞ്ചത്തിന് ഒരു സ്രഷ്ടാവ് തന്നെയില്ലെന്ന് വിശ്വസിക്കുന്നവര്‍ക്ക് ആ സ്രഷ്ടാവ് മനുഷ്യര്‍ക്കായുളള ധാര്‍മികപദ്ധതി വെളിപാടുകള്‍ വഴി പ്രവാചകന്‍മാര്‍ക്കെത്തിച്ചു കൊടുക്കുന്നുവെന്ന ആശയത്തെ ഒരിക്കലും അംഗീകരിക്കാനാകില്ല. അവരുടെ നിഷേധത്തിന് വഹ്‌യിന്റെ സമയത്തെ പ്രവാചകാനുഭവങ്ങളെ വിശകലനം ചെയ്തുകൊണ്ട് മറുപടി പറയുന്നത് അര്‍ത്ഥശൂന്യമാണ്. ദൈവാസ്തിത്വത്തെക്കുറിച്ചും ദൈവിക മാര്‍ഗദര്‍ശനത്തിന്റെ അനിവാര്യതയെക്കുറിച്ചുമെല്ലാമുളള സംവാദങ്ങളാണ് അവരുമായി നടക്കേണ്ടത്. പടച്ചവനുണ്ടെന്നും വഹ്‌യ് എന്നൊരു സംവിധാനമുണ്ടെന്നും അംഗീകരിക്കുന്നവര്‍ക്ക് മാത്രമാണ് വഹ്‌യിന്റെ പ്രവാചകാനുഭവങ്ങളെ അപഗ്രഥിക്കേണ്ട ആവശ്യമുണ്ടാകുന്നത്.

മിഷനറിമാര്‍ തീര്‍ച്ചയായും രണ്ടാമതുപറഞ്ഞ ഗണത്തില്‍ വരുന്നവരാണ്. പ്രവാചകനുണ്ടായത് ദൈവിക വെളിപാടല്ലെന്ന് വരുത്തിത്തീര്‍ക്കാന്‍ ഹദീഥുകളിലുള്ള ഹിറാ വിവരണങ്ങളെ ഓറിയന്റലിസ്റ്റ് വ്യാഖാനങ്ങളെ ഉപജീവിച്ചുകൊണ്ട് അവര്‍ വിമര്‍ശനാത്മകമായി വിശകലനം ചെയ്തതിന് കയ്യും കണക്കുമില്ല. ആ വിമര്‍ശനങ്ങളെല്ലാം, സ്വാഭാവികമായും വഹ്‌യിന്റെ സമയത്ത് പ്രവാചകന്‍മാര്‍ക്കുണ്ടാകുന്ന അനുഭവങ്ങളെന്തൊക്കെയാണെന്ന് ബൈബിളുപയോഗിച്ച് വിശദീകരിച്ചശേഷം പ്രവാചകനുണ്ടായത് അത്തരത്തിലുള്ള അനുഭവങ്ങളൊന്നുമല്ലെന്ന് പറയാനാണ് അടിസ്ഥാനപരമായി ശ്രമിക്കുന്നത്. വഹ്‌യ് പ്രവാചകനില്‍ ഭൗതികമായി ഏതുതരം പ്രതിഫലനങ്ങളാണ് സൃഷ്ടിക്കുക എന്നതിനെ സംബന്ധിച്ച തെറ്റുപറ്റാത്ത ധാരണകള്‍ നാം സ്വരൂപിക്കേണ്ടത് ബൈബിളില്‍ നിന്നാണ് എന്ന നിലപാടാണ് ഈ വര്‍ത്തമാനത്തിന്റെ അടിത്തറ.

ബൈബിള്‍ നൂറുശതമാനവും ദൈവികമാണെന്നും അതില്‍ മനുഷ്യ നിഗമനങ്ങളിടം പിടിച്ചിട്ടില്ലെന്നും എഴുതപ്പെട്ട അതേ രീതിയില്‍ തന്നെ ബൈബിള്‍ പുസ്തകങ്ങള്‍ ഇപ്പോഴും നിലനില്‍ക്കുന്നുവെന്നും മിഷനറിമാര്‍ തെളിയിക്കുമ്പോള്‍ മാത്രമേ ബൈബിളുപയോഗിച്ച് മുഹമ്മദ് നബി(സ)യുടെ വഹ്‌യനുഭവങ്ങളെ നിരൂപണം ചെയ്യുന്നതിന് സാധൂകരണമുണ്ടാവുകയുള്ളൂ. ബൈബിളില്‍ വഹ്‌യിനെക്കുറിച്ച് ഏഴുതപ്പെട്ടിരിക്കുന്നതെന്നല്ലാം സത്യമാണെന്ന് വിശ്വസിക്കുന്നതില്‍ -ബൈബിളിന്റെ ദൈവികത തെളിയിക്കപ്പെടാത്തിടത്തോളം കാലം- യാതൊരു യുക്തിയുമില്ല. ഇതുപോലെത്തന്നെ പ്രധാനമാണ് എല്ലാ പ്രവാചകന്‍മാരുടെയും വെളിപാടനുഭവങ്ങള്‍ ഒരുപോലെയായിരിക്കണമെന്ന് ശഠിക്കുന്നത് അര്‍ത്ഥശൂന്യമാണെന്നതും. ദൈവം അവന്റെ പ്രവാചകന്‍മാരോട് സംസാരിക്കാന്‍ വ്യത്യസ്ത മാര്‍ഗങ്ങള്‍ സ്വീകരിക്കുകയും ആശയവിനിമയത്തിന് അവരെ വ്യത്യസ്ത അനുഭവങ്ങളിലൂടെ കൂട്ടിക്കൊണ്ടുപോവുകയും ചെയ്യുന്നതില്‍ എന്ത് അസാംഗത്യമാണുള്ളത്?

ചില പ്രവാചകന്‍മാരുടെ വെളിപാട് സ്വീകരണസമയത്തെ അനുഭവങ്ങള്‍ തന്നെ എല്ലാ പ്രവാചകന്‍മാര്‍ക്കും അതുപോലെ ആവര്‍ത്തിക്കുമെന്ന് ദൈവം പറയാത്തിടത്തോളം കാലം അത്തരം താരതമ്യങ്ങള്‍ തന്നെ അപ്രസക്തമാണെന്നതാണ് വസ്തുത. നബി(സ)യുടെ വ്യക്തിനിഷ്ഠമായ വെളിപാടനുഭവങ്ങളെ ബൈബിളിന്റെ മാത്രം അടിസ്ഥാനത്തില്‍ തള്ളിക്കളയാന്‍ ശ്രമിക്കുന്നത് തീര്‍ത്തും ബാലിശമായ ഒരു മിഷനറി സങ്കുചിതത്വമാണെന്ന് ചുരുക്കം.

ഇനി, ഹിറാ അനുഭവങ്ങളെ ബൈബിളുപയോഗിച്ച് വിശകലനം ചെയ്താല്‍ അവയുടെ ദൈവികതയെ നിഷേധിക്കേണ്ടി വരുമോ എന്ന് നാം പരിശോധിക്കുക. ബൈബിളിനെ അന്ധമായി പിന്‍പറ്റുന്നവര്‍ക്കുമാത്രം ബാധകമായ ഒരു ചര്‍ച്ചയാണ് ഇത്. എങ്കിലും നാം പരിശോധനയ്ക്ക് മുതിരുക. പ്രവാചകന് സത്യമായിപ്പുലരുന്ന സ്വപ്‌നദര്‍ശനങ്ങള്‍ ഉണ്ടായിത്തുടങ്ങിയെന്നും അതായിരുന്നു അല്ലാഹു അദ്ദേഹവുമായി സംവദിക്കാനാരംഭിച്ച രീതി എന്നുമാണ് ഹദീഥ് ഒന്നാമതായി പറയുന്നത്. ബൈബിളികമായ വീക്ഷണത്തില്‍ ഇതിന് എന്ത് കുഴപ്പമാണുള്ളത്? നേരായിപ്പുലരുന്ന സ്വപ്‌നങ്ങള്‍ ഉറക്കില്‍ പ്രവാചകന്‍മാരെ കാണിക്കുന്നത് ദൈവത്തിന്റെ രീതിയാണെന്നു തന്നെയല്ലേ ബൈബിളും പറയുന്നത്? ജോസഫും (ഉല്‍പത്തി 37: 5-8), ജേക്കബും (ഉല്‍പത്തി 28: 12-14), അബ്രഹാമും (ഉല്‍പത്തി 15: 1), സോളമനും (1 രാജാക്കന്‍മാര്‍ 8: 5) എല്ലാം ദൈവത്താല്‍ സത്യസ്വപ്‌നങ്ങള്‍ കാണിക്കപ്പെട്ടതിനെക്കുറിച്ചുള്ള ബൈബിള്‍ വചനങ്ങള്‍ ഉണ്ടെന്നിരിക്കെ മുഹമ്മദ് നബി(സ)ക്കുണ്ടായ സമാനമായ അനുഭവത്തെ ബൈബിളികമായ ഏത് പ്രതലമുപയോഗിച്ചാണ് മിഷനറിമാര്‍ക്ക് തള്ളിക്കളയേണ്ടി വരുന്നത്?

താന്‍ കാണാന്‍ തുടങ്ങിയ, അക്ഷരംപ്രതി പുലര്‍ന്നുകൊണ്ടിരിക്കുന്ന സ്വപ്‌നങ്ങള്‍ പ്രവാചകനെ (സ) അത്യധികം ചിന്താകുലനാക്കുകയും ഒറ്റയ്ക്കിരിക്കുവാന്‍ അദ്ദേഹം ഇഷ്ടപ്പെടുന്ന സ്ഥിതിയുണ്ടാക്കുകയും ചെയ്തതാണ് ഹിറയില്‍ പോയി പ്രാര്‍ത്ഥനകളില്‍ ശാന്തി കണ്ടെത്തുവാന്‍ ശ്രമിക്കുന്ന ഒരു സന്ദര്‍ഭം പ്രവാചകജീവിതത്തിലേക്ക് കൊണ്ടുവന്നത് എന്ന ചരിത്രവസ്തുത, തിരുനബി(സ)ക്ക് കാണിക്കപ്പെട്ട സ്വപ്‌നങ്ങള്‍ക്കുപിന്നില്‍ ജിബ്‌രീല്‍ മലക്കുമായി മുഖാമുഖം കാണാനുള്ള ഇടത്തിലേക്ക് അദ്ദേഹത്തെ നയിച്ചുകൊണ്ടുപോവുക എന്ന ദൈവിക പദ്ധതിയുണ്ടായിരുന്നുവെന്ന് സുതരാം വ്യക്തമാക്കുന്നുണ്ട്. അന്തിമവേദഗ്രന്ഥം അവതരിപ്പിച്ചു തുടങ്ങാനുള്ള സാഹചര്യം പ്രവാചകജീവിതത്തില്‍ സൃഷ്ടിക്കുവാനും പ്രവാചകത്വത്തിന്റെ മൂര്‍ത്തമായ ആരംഭം കുറിക്കുവാനും വേണ്ടി പടച്ചവന്‍ മുഹമ്മദ് നബി(സ)യുമായുള്ള സംവേദനത്തിന്റെ ആമുഖം സ്വപ്‌നങ്ങള്‍ വഴി നിര്‍വഹിച്ചുതുടങ്ങിയതില്‍ ശുദ്ധമായ ബൈബിള്‍ കണ്ണിലൂടെ നോക്കിയാല്‍ പോലും അസാധാരണമായി യാതൊന്നുമില്ലെന്ന് നാം കണ്ടു.

ഇനി ഹിറയിലേക്ക് വരാം. ഹിറാ ഗുഹയില്‍ വന്ന് മലക്ക് പ്രവാചകനെ ക്വുര്‍ആന്‍ വചനങ്ങള്‍ കേള്‍പിച്ച അനുഭവത്തെയാണ് ഓറിയന്റലിസ്റ്റുകളും മിഷനറിമാരും ഏറ്റവും പ്രധാനമായി ആക്രമിക്കാറുള്ളത്. മനഃശാസ്ത്രപരമായി പ്രസ്തുത സന്ദര്‍ഭത്തെ ഉപജീവിച്ചുകൊണ്ടുള്ള വിമര്‍ശനകാഠിന്യം തികച്ചും സ്വാഭാവികമാണ് എന്നുപറയുന്നതായിരിക്കും ശരി. കാരണം വെളിപാട് എന്നു പറയുമ്പോള്‍ അതിനെ കേവലം ഒരു ആന്തരിക പ്രചോദനമായി മാത്രം ചുരുക്കിയെഴുതാനാഗ്രഹിക്കുന്നവരാണ് മിഷനറിമാരെല്ലാം തന്നെ. തങ്ങള്‍ വേദഗ്രന്ഥമായി അവതരിപ്പിക്കുന്ന പഴയ-പുതിയ നിയമ പുസ്തകങ്ങളൊന്നും ദൈവമോ, ദൈവദൂതനോ ഏതെങ്കിലും പ്രവാചകനെ ഓതിക്കേള്‍പ്പിച്ചവയല്ലെന്നും പലരും പലകാലങ്ങളില്‍ സ്വന്തമായി എഴുതിയുണ്ടാക്കിയവയാണെന്നും ബോധ്യമുള്ളവര്‍ക്ക്, ദൈവം മനസ്സില്‍ സൃഷ്ടിക്കുന്ന ‘തോന്നലു’കള്‍ക്കനുസൃതമായി മനുഷ്യര്‍ സ്വന്തം വാചകങ്ങളില്‍ എഴുതിയുണ്ടാക്കുന്നവയാണ് വേദഗ്രന്ഥങ്ങള്‍ എന്ന് പറയേണ്ടി വരുന്നതില്‍ യാതൊരു അത്ഭുതത്തിനും വകയില്ല. ‘ദൈവനിവേശിതവും പരിശുദ്ധാത്മ പ്രചോദിതവുമാണ്, വാചകങ്ങള്‍ ദൈവികമല്ലെങ്കിലും വിശുദ്ധ ബൈബിള്‍’ എന്ന് സമാധാനിക്കുന്നവര്‍ക്കെങ്ങനെയാണ് തങ്ങള്‍ ശത്രുവായി കാണുന്ന മുഹമ്മദ് നബി(സ)ക്ക് ക്വുര്‍ആനിന്റെ പാഠം (text) ദൈവത്തിന്റെ മലക്ക് മൂര്‍ത്തമായി പ്രത്യക്ഷപ്പെട്ട് ഓതിക്കേള്‍പ്പിച്ചുതുടങ്ങി എന്ന വൃത്താന്തം അസ്വസ്ഥതയുണ്ടാക്കാതിരിക്കുക?

ക്വുര്‍ആനിലെ വചനങ്ങള്‍ നൂറുശതമാനം ദൈവികമാണെന്നും അവ അപ്പടി പ്രവാചകന് അവതരിപ്പിക്കപ്പെടുകയാണുണ്ടായതെന്നും വരുമ്പോള്‍ ബൈബിള്‍ ക്വുര്‍ആനുമായി താരതമ്യം പോലുമര്‍ഹിക്കുന്നില്ലെന്ന് തിരിച്ചറിയുന്നവരുടെ ആര്‍ക്കും പ്രവചിക്കാവുന്ന അസഹിഷ്ണുത മാത്രമാണ് ഹിറയില്‍ ചുറ്റിത്തിരിയുന്ന വിമര്‍ശകത്തൂലികകളുടെ മഷിയും ഊര്‍ജ്ജവുമെന്ന കാര്യം സ്പഷ്ടമാണ്. സത്യം സ്വാര്‍ത്ഥതാല്‍പര്യങ്ങള്‍ക്കെതിരായി വരുമ്പോള്‍ അസഹിഷ്ണുത കാണിക്കലല്ല മറിച്ച് അതിനോടുള്ള ശാത്രവം അവസാനിപ്പിക്കലാണ് രക്ഷയുടെ മാര്‍ഗമെന്ന് ഇവര്‍ തിരിച്ചറിഞ്ഞിരുന്നെങ്കില്‍ എന്ന് പ്രാര്‍ത്ഥിക്കുവാന്‍ മാത്രമേ നമുക്ക് കഴിയൂ. വെളിപാടിനെ ഒരു അന്തപ്രചോദനത്തിന്റെ മാത്രം തലമുള്ള മാനസികാനുഭൂതിയില്‍ ഒതുക്കി നിര്‍ത്തുകയല്ല അന്തിമപ്രവാചകന്റെ കാര്യത്തില്‍ പ്രപഞ്ചനാഥന്‍ ചെയ്തത്; മറിച്ച് അതിന്റെ ആദ്യ സന്ദര്‍ഭത്തില്‍ തന്നെ തികച്ചും മൂര്‍ത്തമായി തന്റെ മലക്കിനെ പ്രവാചകനുമുന്നില്‍ കൊണ്ടുചെന്നു നിര്‍ത്തുകയും ശാരീരികമായിത്തന്നെ പ്രവാചകനെ അതിശക്തമായി മലക്കിനെക്കൊണ്ട് സ്പര്‍ശിപ്പിക്കുകയും ശേഷം സ്ഫുടമായ വാചകങ്ങളിലുള്ള സംഭാഷണത്തിന് അവസരമൊരുക്കയും ചെയ്യുകയാണ്. അകത്തുനിന്ന് നിര്‍ഗളിക്കുന്ന യാതൊന്നുമല്ല, മറിച്ച് പുറത്തുനിന്ന് പ്രവാചകനിലേക്ക് പ്രവഹിക്കുന്നതാണ് വഹ്‌യ് എന്ന് ഇതിനേക്കാള്‍ വ്യക്തമായി ഭൗതിക പരിതസ്ഥിതികളുപയോഗിച്ച് പ്രതിഫലിപ്പിക്കാനാവുകയില്ല തന്നെ. വെളിപാടു സ്വീകരണത്തിന്റെ ഏറ്റവും ഉയര്‍ന്നതും സുവ്യക്തവും തീവ്രവുമായ വൈയക്തികാനുഭവങ്ങളാണ് പ്രവാചകശ്രേഷ്ഠനുണ്ടായതെന്ന് സാരം. നബിവിദ്വേഷത്താല്‍ ഉന്മാദം ബാധിച്ച തലച്ചോറുകള്‍ക്ക് പടച്ചവന്‍ പ്രവാചകനു നല്‍കിയ ഈ തുല്യതയില്ലാത്ത ആദരവിനെ തമസ്‌കരിക്കേണ്ടി വരുന്നതിന്റെ രാഷ്ട്രീയം യാതൊരു വിശദീകരണവുമില്ലാതെ തന്നെ വ്യക്തമാണ്.

ഹിറയില്‍വെച്ച് പ്രവാചകനുണ്ടായ അനുഭവങ്ങളെ നാം പരിഗണിക്കുക. പ്രവാചകത്വമോ വെളിപാടുകളോ ദീര്‍ഘകാലത്തേക്ക് ഉണ്ടായിട്ടില്ലാത്ത മക്കയിലെ അറബ് സമൂഹത്തില്‍ ജനിച്ചുവളര്‍ന്ന, വേദപുസ്തകങ്ങളുമായി യാതൊരു പരിചയവുമില്ലാത്ത സാധാരണക്കാരനായ ഒരു മനുഷ്യന്‍ ആള്‍പാര്‍പ്പില്ലാത്ത ഒരു കൂറ്റന്‍ പര്‍വതത്തിന്റെ ചെങ്കുത്തായ ചെരുവിലുള്ള ഹിറ എന്ന ചെറിയ ഗുഹയില്‍ ഒറ്റയ്ക്കിരിക്കുമ്പോള്‍ ഒരു ദിവസം പൊടുന്നനെ അല്ലാഹുവിന്റെ ദിവ്യസന്ദേശവുമായി അതിശക്തനായ ജിബ്‌രീല്‍ എന്ന മലക്ക് പ്രത്യക്ഷപ്പെടുകയും എഴുത്തോ വായനയോ അറിയാത്ത പ്രവാചകനോട് വായിക്കുവാനാജ്ഞാപിക്കുകയും വായിക്കാനറിയില്ലെന്നു പറഞ്ഞപ്പോള്‍ ആരും ഉലഞ്ഞുപോകുംവിധം ചേര്‍ത്തുപിടിച്ചമര്‍ത്തുകയും ഇത് രണ്ടുതവണ കൂടിയാവര്‍ത്തിച്ചശേഷം ഗാംഭീര്യം തുളുമ്പുന്ന അഞ്ച് ദിവ്യവചനങ്ങള്‍ ഓതികേള്‍പിക്കുകയും അപ്രത്യക്ഷനാവുകയും ചെയ്യുന്നു. പ്രവാചകന്‍ ഇത്തരമൊരനുഭവം ജീവിതത്തിലൊരിക്കലും പ്രതീക്ഷിച്ചിട്ടുപോലുമില്ലെന്ന കാര്യം കൂടി നമ്മളോര്‍ക്കണം. പ്രവാചകത്വത്തിന്റെയും വെളിപാടുകളുടെയും രീതിശാസ്ത്രവുമായുള്ള തികഞ്ഞ അപരിചിതത്വം, താന്‍ ഒരു പ്രവാചനായി നിയോഗിക്കപ്പെടുമെന്നോ തന്റെയടുക്കലേക്ക് മലക്ക് വരും എന്നോ ഉള്ള യാതൊരു വിചാരവുമില്ലാത്ത ഒരാളുടെ തയ്യാറെയുപ്പില്ലായ്മ. അതിഭീകരമായ ഏകാന്തത. ആരും ഭയന്നുപോകുംവിധമുള്ള ആശ്ലേഷം. ലോകര്‍ക്കു മുഴുവന്‍ ദിവ്യസന്ദേശമെത്തിക്കാനുള്ള ദൗത്യം ജഗന്നിയന്താവിനാല്‍ ഏല്‍പിക്കപ്പെടുന്നത് സൃഷ്ടിക്കുന്ന മനോവിസ്‌ഫോടനം. ദൈവത്തിന്റെ വചനങ്ങള്‍ അവന്റെ ദൂതനില്‍നിന്ന് നേരിട്ടുകേള്‍ക്കുന്നത് സൃഷ്ടിക്കുന്ന ശാരീരിക വ്യതിയാനങ്ങള്‍. ആരാണ് ഭയന്നുപോകാതിരിക്കുക? ആരാണ് വിറച്ചുപോകാതിരിക്കുക?

വിശുദ്ധ ക്വുര്‍ആനിലെ വചനങ്ങള്‍, അവയുടെ ആശയങ്ങളും പദങ്ങളും ഒരുപോലെ ദൈവത്തില്‍നിന്ന് നിര്‍ഗളിച്ചവയായതിനാല്‍, ശുദ്ധ മനസ്സുള്ള ആരിലും പ്രകമ്പനങ്ങള്‍ സൃഷ്ടിക്കുക തന്നെ ചെയ്യും. ക്വുര്‍ആന്‍ വചനങ്ങളുടെ കേവലമായ ശ്രുതി തന്നെ അതിന്റെ ദൈവികതയെക്കുറിച്ച ബോധമുള്ളവരുടെ മനസ്സില്‍ ആന്ദോളനങ്ങളുളവാക്കാന്‍ പോന്നതാണ്. ആശയങ്ങളറിഞ്ഞു കേള്‍ക്കുന്നവരില്‍ അത് ചെലുത്തുന്ന സ്വാധീനം അതിനേക്കാള്‍ ശക്തമാണ്. മനുഷ്യന്റെ നിസ്സാരതയും ദൈവത്തിന്റെ മഹത്വവും ദൈവതൃപ്തിക്കായുള്ള അധ്വാനങ്ങളില്‍ മനുഷ്യന്‍ വരുത്തുന്ന അലംഭാവത്തിന്റെ ഗൗരവവും ബോധ്യപ്പെടുത്തി ജീവിതത്തെ പൂര്‍ണമായി സംസ്‌കരിക്കുവാന്‍ ശേഷിയുള്ള അതിശക്തമായ ദൈവഭയം കേള്‍വിക്കാരനില്‍ നിറയ്ക്കുകയാണ് ഓരോ ക്വുര്‍ആന്‍ വചനവും ചെയ്യുന്നത്. സത്യവിശ്വാസികളില്‍ ക്വുര്‍ആന്‍ ശ്രവണം ഉണ്ടാക്കുന്ന ഫലത്തെക്കുറിച്ച് ക്വുര്‍ആന്‍ തന്നെ പറയുന്നതിപ്രകരമാണ്: ”അല്ലാഹുവെപ്പറ്റി പറയപ്പെട്ടാല്‍ ഹൃദയങ്ങള്‍ പേടിച്ചുനടുങ്ങുകയും അവന്റെ വചനങ്ങള്‍ വായിച്ചുകേള്‍പിക്കപ്പെട്ടാല്‍ വിശ്വാസം വര്‍ധിക്കുകയും തങ്ങളുടെ രക്ഷിതാവിന്റെ മേല്‍ ഭരമേല്‍പിക്കുകയും ചെയ്യുന്നവര്‍ മാത്രമാണ് സത്യവിശ്വാസികള്‍.” (ക്വുര്‍ആന്‍ 8: 2).

ഈ ഫലം സൃഷ്ടിക്കുന്ന ക്വുര്‍ആന്‍ വചനങ്ങള്‍, ദൈവത്തിന്റെ ഘനഗംഭീരമായ സംസാരം, ആകാശത്തുനിന്ന് ഭൂമിയിലേക്കിറങ്ങിവന്ന അതിശക്തനായ ഒരു മലക്കില്‍നിന്ന് മനുഷ്യരുടെ കൂട്ടത്തില്‍വെച്ച് ആദ്യമായി കേള്‍ക്കുകയാണ് മുഹമ്മദ് നബി (സ). ക്വുര്‍ആന്‍ അവതരണം പ്രവാചകനെ മാനസികമായും ശാരീരികമായും വിറപ്പിച്ച അനുഭവമായിത്തീരാതിരിക്കുക പിന്നെയെങ്ങനെയാണ്? ക്വുര്‍ആന്‍ അവതരണപ്രക്രിയയുടെ ഭാരത്തെക്കുറിച്ച് സുന്ദരമായ ഒരുപമയിലൂടെ ക്വുര്‍ആന്‍ തന്നെ വര്‍ണിക്കുന്നുണ്ട്: ”ഈ ക്വുര്‍ആനിനെ നാം (അല്ലാഹു) ഒരു പര്‍വതത്തിനുമുകളില്‍ അവതരിപ്പിച്ചിരുന്നെങ്കില്‍ അത് (പര്‍വതം) വിനീതമാകുന്നതും അല്ലാഹുവെപ്പറ്റിയുള്ള ഭയത്താല്‍ പൊട്ടിപ്പിളരുന്നതും നിനക്ക് കാണാമായിരുന്നു. ആ ഉദാഹരണങ്ങള്‍ നാം ജനങ്ങള്‍ക്കുേവണ്ടി വിവരിക്കുന്നു. അവര്‍ ചിന്തിക്കുവാന്‍വേണ്ടി.” (59: 21) വഹ്‌യ് പ്രവാചകനില്‍ സൃഷ്ടിച്ച വിഹ്വലത, തികച്ചും സ്വാഭാവികമായിരുന്നുവെന്നര്‍ത്ഥം. വഹ്‌യ് ലഭിച്ച സ്ഥലവും രീതിയും മുതല്‍ അതിന്റെ ഉള്ളടക്കവും ധ്വനികളും വരെ ആ വിഹ്വലതയില്‍ പ്രതിഫലിക്കുന്നുണ്ട്; ഇത്ര വലിയ പ്രകമ്പനങ്ങള്‍ സൃഷ്ടിക്കുന്ന ശാരീരിക-മാനസികാനുഭവങ്ങള്‍ സമ്മാനിക്കുന്ന ദൈവികബോധനപ്രക്രിയയെ താങ്ങാനുള്ള കരുത്ത് തനിക്കുണ്ടോ എന്ന ആവലാതി മുതല്‍ അന്തിമപ്രവാചകന്‍ എന്ന അതിഭയങ്കരമായ ഉത്തരവാദിത്തം ശിരസ്സാവഹിക്കാനുള്ള വലുപ്പം തനിക്കുണ്ടോ എന്ന ഭയപ്പാടുവരെ ആ വിറയലില്‍ പ്രതിധ്വനിക്കുന്നുണ്ട്.

പ്രവാചകന്‍ ഒരു മനുഷ്യനാണ് എന്ന വസ്തുത മാത്രമാണ് ഹിറാ സംഭവം തെളിയിക്കുന്നത്; നിഷ്‌കളങ്കനും ആത്മാര്‍ത്ഥതയുള്ളവനുമായ മനുഷ്യന്‍. പ്രവാചകത്വം ഏല്‍പിക്കുവാന്‍വേണ്ടി അതേ സാഹചര്യത്തിലുള്ള ഏതു മനുഷ്യനെ അതേ അനുഭവങ്ങളിലൂടെ കൊണ്ടുപോയാലും ഇതേ കാര്യങ്ങള്‍ തന്നെയാണ് സംഭവിക്കുക. മുഹമ്മദ് നബി (സ) മലക്കാണെന്നോ ദൈവമാണെന്നോ അല്ല മുസ്‌ലിംകള്‍ വിശ്വസിക്കുന്നത്; മനുഷ്യനാണെന്നു തന്നെയാണ്. ആ മനുഷ്യന് ദിവ്യബോധനങ്ങള്‍ ലഭിച്ചു എന്നാണ് മുസ്‌ലിംകള്‍ പറയുന്നത്. അതിനെ ഖണ്ഡിക്കുവാനുതകുന്ന യാതൊന്നും പ്രവാചകന്റെ ഹിറാ അനുഭവങ്ങളിലില്ല തന്നെ. ക്വുര്‍ആന്‍ പറയട്ടെ: ”( നബിയേ,) പറയുക: ഞാന്‍ നിങ്ങളെപ്പോലെയുള്ള ഒരു മനുഷ്യന്‍ മാത്രമാകുന്നു. നിങ്ങളുടെ ദൈവം ഏകദൈവം മാത്രമാണെന്ന് എനിക്ക് ബോധനം നല്‍കപ്പെടുന്നു. അതിനാല്‍ വല്ലവനും തന്റെ രക്ഷിതാവുമായി കണ്ടുമുട്ടണമെന്ന് ആഗ്രഹിക്കുന്നുവെങ്കില്‍ അവന്‍ സല്‍കര്‍മ്മം പ്രവര്‍ത്തിക്കുകയും, തന്റെ രക്ഷിതാവിനുള്ള ആരാധനയില്‍ യാതൊന്നിനെയും പങ്കുചേര്‍ക്കാതിരിക്കുകയും ചെയ്തുകൊള്ളട്ടെ.” (18: 110)

ജിബ്‌രീലിന്റെ ആഗമനം പ്രവാചകനില്‍ നിറച്ച ഭയവും അത് സൃഷ്ടിച്ച ശാരീരിക പ്രതിഫലനങ്ങളും ബൈബിള്‍ വിവരിക്കുന്ന പ്രവാചകാനുഭവങ്ങളുമായി ചേര്‍ന്നുപോകുന്നതല്ല എന്ന മിഷനറി വാദത്തിന്റെ (അങ്ങനെ ചേര്‍ന്നുപോകുക ഒരാളുടെ പ്രവാചകത്വം തെളിയിക്കുവാന്‍ ഒരു നിലയ്ക്കും ആവശ്യമല്ലെന്ന് നാം സൂചിപ്പിച്ചുകഴിഞ്ഞു) വസ്തുനിഷ്ഠതയാണ് ഇനി പരിശോധിക്കപ്പെടേണ്ടത്. ജിബ്‌രീല്‍ (ഗബ്രിയേല്‍) മലക്കുകളില്‍ ശക്തികൊണ്ടും ഗാംഭീര്യംകൊണ്ടും വേറിട്ടുനില്‍ക്കുന്നയാളാണെന്ന സങ്കല്‍പം ബൈബിള്‍ പ്രദാനം ചെയ്യുന്നുണ്ട്. പ്രശസ്തമായ ബൈബിള്‍ ഓണ്‍ലൈന്‍ വിജ്ഞാനകോശം www.newadvent.org പറയട്ടെ: ”ഗബ്രിയേല്‍, പേര് സൂചിപ്പിക്കുന്നതുപോലെ ദൈവത്തിന്റെ ശക്തിയുടെ ദൂതനാണ്. ഗബ്രിയേലിനെ പരാമര്‍ശിക്കുന്ന ബൈബിള്‍ വചനങ്ങളില്‍ മഹത്വത്തെയും ശക്തിയെയും അധികാരത്തെയും ബലത്തെയുമെല്ലാം സൂചിപ്പിക്കുന്ന പദങ്ങള്‍ നിരന്തരമായി ആവര്‍ത്തിക്കുന്നത് ശ്രദ്ധേയമാണ്… ജൂതന്‍മാര്‍ ഗബ്രിയേലിന്റെ ഈ വിശേഷണങ്ങളെ ആഴത്തില്‍ ഉള്‍ക്കൊണ്ടിട്ടുള്ളതുപോലെയാണ് തോന്നുന്നത്… സൊദോമിന്റെ നാശം ഗബ്രിയേലിന്റെ കൈകളിലൂടെയാണുണ്ടായതെന്ന് അവര്‍ മനസ്സിലാക്കുന്നു.” അതിശക്തനും ഗംഭീരഭാവമുള്ളയാളും ഒരു പ്രദേശത്തെയൊന്നടങ്കം നശിപ്പിക്കുവാന്‍ ദൈവം നിയോഗിച്ച ബലവാനുമെല്ലാമായാണ് ബൈബിള്‍ പഴയനിയമം ജിബ്‌രീലിനെ അവതരിപ്പിക്കുന്നതെന്ന് സാരം. ഹിറാ ഗുഹയില്‍ പ്രവാചകനുമുന്നില്‍ ജിബ്‌രീല്‍ പ്രത്യക്ഷപ്പെട്ടതിനെത്തുടര്‍ന്നുണ്ടായ സംഭവങ്ങളെ ന്യായീകരിക്കുക മാത്രമല്ലേ ഈ ബൈബിള്‍ വര്‍ണനകള്‍ ചെയ്യുന്നത്?

ഇനി ജിബ്‌രീല്‍ പ്രവാചകന്‍മാര്‍ക്ക് പ്രത്യക്ഷപ്പെട്ടതിനെ സംബന്ധിച്ച ബൈബിള്‍ വിവരണങ്ങളിലേക്കു വരാം. ഗബ്രിയേല്‍ ദൂതനുമായി ആശയവിനിമയം നടത്തിയ ഏറ്റവും ശ്രദ്ധേയനായ പഴയനിയമ കഥാപാത്രം ദാനിയേല്‍ ആണ്. എന്തായിരുന്നു ദാനിയേലിന്റെ അനുഭവം? ഭാവിയെക്കുറിച്ചുള്ള ഭീഷണമായ സുചനകളുള്‍ക്കൊള്ളുന്നുവെന്ന് തോന്നിപ്പിക്കുന്ന വിചിത്രദൃശ്യങ്ങളുള്ള ഒരു ദര്‍ശനം ദാനിയേലിനുണ്ടാകുന്നു. ദര്‍ശനത്തിന്റെ ആഘാതത്തില്‍ നില്‍ക്കവെ ഗബ്രിയേല്‍ മനുഷ്യരൂപത്തില്‍ ദാനിയേലിനു പ്രത്യക്ഷപ്പെട്ടതിനെക്കുറിച്ച് ബൈബിള്‍ പറയട്ടെ: ”ദാനിയേലായ ഞാന്‍ ഈ ദര്‍ശനം ഗ്രഹിക്കാന്‍ ശ്രമിച്ചുകൊണ്ടിരിക്കെ, ഇതാ എന്റെ മുമ്പില്‍ മനുഷ്യരൂപമുള്ള ഒരുവന്‍ നില്‍ക്കുന്നു. ഉലായ് തീരങ്ങളില്‍നിന്ന് ഒരുവന്‍ വിളിച്ചുപറയുന്നത് ഞാന്‍ കേട്ടു; ഗബ്രിയേല്‍, ദര്‍ശനം ഇവനെ ഗ്രഹിപ്പിക്കുക. ഞാന്‍ നിന്നിടത്തേക്ക് അവന്‍ (ഗബ്രിയേല്‍) വന്നു. അവന്‍ വന്നപ്പോള്‍ ഞാന്‍ ഭയവിഹ്വലനായി സാഷ്ടാംഗം വീണു… അവന്‍ എന്നോട് സംസാരിച്ചുകൊണ്ടിരിക്കുമ്പോള്‍ ഞാന്‍ മൂര്‍ഛിച്ചുവീണു. എന്നാല്‍ അവന്‍ എന്നെ തൊട്ട് എഴുന്നേല്‍പിച്ചു നിര്‍ത്തി.” (ദാനിയേല്‍ 8: 15-18)

മറ്റൊരിക്കല്‍ ഗബ്രിയേല്‍ ടൈഗ്രീസ് തീരത്തുവെച്ച് ദാനിയേലിനു പ്രത്യക്ഷപ്പെട്ടതിനെക്കുറിച്ച് പത്താം അധ്യായത്തിലുണ്ട്. അതിപ്രാകാരം: ”ഞാന്‍ കണ്ണുയര്‍ത്തി നോക്കിയപ്പോള്‍ ചണവസ്ത്രവും ഊഫാസിലെ സ്വര്‍ണം കൊണ്ടുള്ള അരപ്പട്ടയും ധരിച്ച ഒരുവനെ കണ്ടു. അവന്റെ ശരീരം ഗോമേദകം പോലെയും മുഖം മിന്നല്‍ പോലെയും കണ്ണുകള്‍ ജ്വലിക്കുന്ന ഒരു പന്തം പോലെയും ആയിരുന്നു. അവന്റെ കൈകാലുകള്‍ മിനുക്കിയ ഓടിന്റെ ഭംഗിയുള്ളവയും സ്വരം ജനക്കൂട്ടത്തിന്റെ ഇരമ്പല്‍ പോലെയും ആയിരുന്നു. ദാനിയേലായ ഞാന്‍ മാത്രം ഈ ദര്‍ശനം കണ്ടു; എന്നോടൊപ്പമുണ്ടായിരുന്നവര്‍ അതു കണ്ടില്ല. മഹാഭീതി പിടിപെട്ട് അവര്‍ ഓടിയൊളിച്ചു. അങ്ങനെ തനിച്ചായ ഞാന്‍ ഈ മഹാദര്‍ശനം കണ്ടു; എന്റെ ശക്തി ചോര്‍ന്നുപോയി. എന്റെ മുഖം തിരിച്ചറിയാന്‍ വയ്യാത്തവിധം മാറിപ്പോയി. എന്റെ ശക്തിയറ്റു. അപ്പോള്‍ ഞാന്‍ അവന്റെ സ്വരം കേട്ടു, അവന്റെ സ്വരം ശ്രവിച്ച ഞാന്‍ പ്രജ്ഞയറ്റ് നിലം പതിച്ചു. എന്നാല്‍, ഒരു കരം എന്നെ സ്പര്‍ശിച്ചു. അവന്‍ എന്നെ എഴുന്നേല്‍പിച്ചു. വിറയലോടെയാണെങ്കിലും മുട്ടും കയ്യും ഊന്നി ഞാന്‍ നിന്നു. അവന്‍ എന്നോട് പറഞ്ഞു: ഏറ്റവും പ്രിയങ്കരനായ ദാനിയേലേ, എഴുന്നേല്‍ക്കുക. ഞാന്‍ നിന്നോട് പറയുന്ന വാക്കുകള്‍ ശ്രദ്ധിച്ചുകേള്‍ക്കുക. എന്നെ നിന്റെയടുത്തേക്ക് അയച്ചിരിക്കുകയാണ്. അവന്‍ ഇതുപറഞ്ഞപ്പോള്‍ ഞാന്‍ വിറയലോടെ നിവര്‍ന്നുനിന്നു.” (ദാനിയേല്‍ 10: 5 – 11)

ഹിറയിലുണ്ടായ വെളിപാടിന്റെ ദൈവികതയെ നിഷേധിക്കാന്‍ ബൈബിളുപയോഗിച്ച് കഴിയില്ലെന്നും പ്രവാചകാനുഭവങ്ങളെ സാധൂകരിക്കുക മാത്രമാണ് ബൈബിള്‍ ഈ വിഷയത്തില്‍ ചെയ്യുന്നത് എന്നുമുള്ള കേവല വസ്തുതകളാണ് ഇവിടെ അനാവൃതമാകുന്നത്. ഗബ്രിയേലുമായുള്ള മനുഷ്യമുഖാമുഖത്തെക്കുറിച്ച് ബൈബിള്‍ പുതിയ നിയമവും ഇതേദിശയിലുള്ള സൂചനകള്‍ നല്‍കുന്നുണ്ട്. യോഹന്നാന്‍ സ്‌നാപകന്റെ പിതാവ് സെഖര്യാവിന്റെ അനുഭവങ്ങള്‍ ശ്രദ്ധിക്കുക: ”അപ്പോള്‍, കര്‍ത്താവിന്റെ ദൂതന്‍ ധൂപപീഠത്തിന്റെ വലതുവശത്ത് നില്‍ക്കുന്നതായി അവന് പ്രത്യക്ഷപ്പെട്ടു. അവനെക്കണ്ട് സഖറിയാ അസ്വസ്ഥനാവുകയും ഭയപ്പെടുകയും ചെയ്തു.” (ലൂക്കോസ് 1: 11, 12). പ്രവാചകന്‍ (സ) ഭയവിഹ്വലനായത് ചൂണ്ടിക്കാണിച്ച് അദ്ദേഹത്തിനുണ്ടായത് വഹ്‌യല്ലെന്നും വന്നത് മലക്കല്ലെന്നുമെല്ലാം സമര്‍ത്ഥിക്കുവാന്‍ പാടുപെടുന്ന മിഷനറിമാര്‍ തങ്ങളുടെ സ്വന്തം ബൈബിളിന്റെയടക്കം കഴുത്തിലാണ് ഈ അഭ്യാസപ്രകടനങ്ങള്‍ക്കിടയില്‍ കത്തിവെക്കുന്നതെന്നര്‍ത്ഥം.

ബൈബിള്‍ ശരിയായ രീതിയില്‍ വായിച്ചവര്‍ക്കൊന്നും മുഹമ്മദ് നബി(സ)യ്ക്ക് ഹിറാ ഗുഹയില്‍ വെച്ചുണ്ടായ വെളിപാടിനെ സാക്ഷീകരിക്കുവാനല്ലാതെ നിരാകരിക്കാന്‍ കഴിയില്ലെന്ന വസ്തുതയാണ് ഹദീഥിന്റെ അവസാന ഭാഗത്തുള്ള വറക്വത്ബ്‌നു നൗഫലിന്റെ വാചകങ്ങള്‍ തെളിയിക്കുന്നത്. ജൂത-ക്രിസ്തു ദര്‍ശനങ്ങളോട് ആഭിമുഖ്യവും അവരുടെ ഗ്രന്ഥങ്ങളില്‍ അഗാധപരിജ്ഞാനവുമുണ്ടായിരുന്ന വറക്വ, നബി(സ)യുടെ വെളിപാടനുഭവങ്ങള്‍ കേട്ടപ്പോള്‍ അത് ജിബ്‌രീല്‍ തന്നെയാണെന്ന് സാക്ഷ്യപ്പെടുത്തുകയും നബി(സ)ക്ക് പിന്തുണ പ്രഖ്യാപിക്കുകയുമാണ് ചെയ്തതെന്ന യാഥാര്‍ത്ഥ്യം മിഷനറിമാരുടെ കണ്ണുതുറപ്പിക്കേണ്ടതുണ്ട്. അതെ, ഖദീജ (റ) ആണയിട്ടു പറഞ്ഞതുപോലെ ബന്ധുക്കള്‍ക്കും ദുര്‍ബലര്‍ക്കും ദരിദ്രര്‍ക്കും അശരണര്‍ക്കും അതിഥികള്‍ക്കും പ്രയാസപ്പെടുന്നവര്‍ക്കും തണല്‍മരമായി നിന്നിരുന്ന മുഹമ്മദ് (സ) എന്ന നന്മകളുടെ ഉടല്‍ രൂപത്തെ പ്രവാചകനായി നിയോഗിക്കുവാന്‍ ജിബ്‌രീല്‍ എന്ന മലക്കു തന്നെയാണ് ഒന്നര സഹസ്രാബ്ദത്തോളം മുമ്പ് ഹിറാ ഗുഹയില്‍ പ്രത്യക്ഷപ്പെട്ടത്; മുഹമ്മദ് നബി (സ) ജിബ്‌രീലിനോട് സ്വയം സാക്ഷ്യപ്പെടുത്തിയതുപോലെ എഴുത്തോ വായനയോ അറിയാതിരുന്ന, വേദപുസ്തക പരിജ്ഞാനം അശേഷമില്ലാതിരുന്ന ആ സാധാരണക്കാരന്റെ നാവില്‍ നിന്ന് ലോകം വിസ്മയത്തോടുകൂടി ക്വുര്‍ആന്‍ കേട്ടത് ജിബ്‌രീല്‍ അദ്ദേഹത്തിനത് പഠിപ്പിച്ചുകൊടുത്തതുകൊണ്ടാണ്. ഹിറയില്‍ നിന്ന് പ്രസരിച്ച വെളിച്ചത്തിന് ബൈബിള്‍ മറയാകുമെന്ന് കരുതുന്നവര്‍ വിഡ്ഢികളുടെ സ്വര്‍ഗത്തിലാണെന്നു മാത്രമാണ് നമുക്ക് പറയാനുള്ളത്.

ഒരേ സമയം ഒൻപത് ഭാര്യമാരുണ്ടായിരുന്ന നബിയുടെ (സ) ദാമ്പത്യജീവിതം മാതൃകാപരമാണെന്ന് പറയുന്നതിൽ എന്ത് അർത്ഥമാണുള്ളത്?

ജീവിതത്തിലെവിടെയും കളങ്കത്തിന്റെ ലാഞ്ചനകളൊന്നുമില്ലാതെ ജീവിക്കുകയും തന്റെ വിശുദ്ധജീവിതം വഴി അവസാനനാളുവരെ ലോകത്തിന് മാതൃകയാക്കാന്‍പറ്റുന്ന പതിനായിരങ്ങളെ സൃഷ്ടിക്കുകയും ചെയ്ത നബി(സ)യുടെ ദാമ്പത്യജീവിതം അവസാന നാളുവരെയുള്ള മനുഷ്യർക്കെല്ലാം സംതൃപ്തദാമ്പത്യത്തിനുള്ള മാതൃകയാനിന്ന കാര്യത്തിൽ ആ ജീവിതത്തെ മുൻധാരണയില്ലാതെ പഠനവിധേയമാക്കിയവർക്കൊന്നും സംശയമുണ്ടാവില്ല.

നബിയുടെ (സ) ബഹുഭാര്യാത്വം നബിനിന്ദകരുടെ വിഷയമാകുന്നതും അതിനെ വൃത്തികെട്ട രൂപത്തിൽ ചിത്രീകരിക്കുന്നതും തങ്ങള്‍ രതിയിലേര്‍പ്പെടുന്നവരുടെ പൂര്‍ണമായ ഉത്തരവാദിത്വമേറ്റെടുക്കാന്‍ പുരുഷന് ബാധ്യതയുണ്ടെന്ന അദ്ദേഹത്തിന്റെ നിഷ്‌കൃഷ്ടമായ അനുശാസന നിന്ദകർക്കും അവരുടെ ഭൗതികാസ്വാദനത്തിന്റെ പ്രത്യയശാസ്ത്രത്തിനും അരോചകമാകുന്നതുകൊണ്ടാണ്. പെണ്ണിന്റെ ഉത്തരവാദിത്വങ്ങളൊന്നും ഏറ്റെടുക്കാതെ അവളില്‍നിന്ന് ലൈംഗികസുഖം നുണയാവുന്ന സാമൂഹ്യസാഹചര്യങ്ങള്‍ക്കാവശ്യമായ നിയമങ്ങള്‍ നിര്‍മിക്കുന്ന മുതലാളിത്ത ആണ്‍കോയ്മയുടെ കണ്ണട ധരിച്ചവര്‍ക്ക് പ്രവാചകന്റെ (സ) വിശുദ്ധജീവിതത്തിന്റെ അര്‍ത്ഥപൂര്‍ണിമ അറിയാന്‍ കഴിയില്ല. വിവാഹവും വ്യഭിചാരവും തമ്മിലുള്ള വ്യത്യാസമെന്താണെന്നുപോലും അവര്‍ക്ക് മനസ്സിലാവില്ല. മുഹമ്മദ് നബി(സ)ക്ക് ഒരേസമയം ഒന്‍പ്ത ഭാര്യമാരുണ്ടായിരുന്നുവെന്ന് നിന്ദാസ്വരത്തില്‍ പറയുന്നവര്‍ കരുതുന്നത് നിരവധി സ്ത്രീകളുമായി രമിക്കുകയും അവര്‍ക്കുള്ളത് അപ്പോള്‍തന്നെ കൊടുത്തുവിടുകയും ചെയ്യുന്ന വൃത്തികെട്ട സ്വതന്ത്രരതീ സംസ്‌കാരത്തിന് തുല്യമാണ് പ്രവാചകന്‍ (സ) തന്റെ ബഹുഭാര്യത്വത്തിലൂടെ പഠിപ്പിച്ച സംസ്‌കാരവുമെന്നാണ്. ഇണകളുടെയെല്ലാം പൂര്‍ണമായ ഉത്തരവാദിത്തമേറ്റെടുക്കുകയും അവര്‍ക്കെല്ലാം സംതൃപ്തമായ ദാമ്പത്യജീവിതം നല്‍കുകയും ചെയ്ത പ്രവാചകന്‍ ലോകത്തിനുപഠിപ്പിച്ച ധാര്‍മികതയെവിടെ? കൂടെ കിടന്നവളില്‍ മാംസം മാത്രം കാണുകയും അവളുടെ മാംസത്തിന് വില പറഞ്ഞുറപ്പിച്ച് അവളില്‍ സുഖമനുഭവിച്ച് അവളെ വലിച്ചെറിയുന്ന ആണ്‍കോയ്മാ ലൈംഗിക സംസ്‌കാരത്തിന്റെ ധാര്‍മികതയെവിടെ?

ഇണകളില്‍ നിന്നേ ലൈംഗികതയാസ്വദിക്കുവാന്‍ പാടുള്ളുവെന്ന് കരുതുന്നവര്‍ക്ക് മാത്രമാണ് നബി(സ)യുടെ ജീവിതത്തില്‍ മാതൃകയുള്ളതെന്നാണ് സ്വതന്ത്രരതിയുടെ ഭൂമികയില്‍ നിന്നുകൊണ്ട് നബി(സ)യെ തെറി പറയുകയും അദ്ദേഹത്തില്‍ മാതൃകയില്ലെന്ന് കരയുകയും ചെയ്യുന്നവരോട് ഒന്നാമതായി പറയാനുള്ളത്. ലൈംഗികസുഖം അനുഭവിക്കുകയും സ്വന്തം ഇണക്ക് അതിനുള്ള അവസരമുണ്ടാക്കുകയും ചെയ്യുന്നത് പുണ്യമാണെന്നും അതിന് പടച്ചവന്‍ പ്രതിഫലം നല്‍കുമെന്നും പഠിപ്പിച്ച ആത്മീയാചാര്യനാണ് മുഹമ്മദ് നബി(സ). സമ്പൂര്‍ണജീവിതത്തിന്റെ സവിശേഷമാതൃകയാണ് പ്രവാചകവ്യക്തിത്വത്തിലുടനീളം നമുക്ക് കാണാനാവുക.

ഏകഭാര്യത്വവും ബഹുഭാര്യത്വവും സ്വീകരിക്കുവാന്‍ സ്വാതന്ത്ര്യവും അവകാശവുമുള്ള മുസ്‌ലിംകള്‍ക്ക് ഈ രണ്ട് രംഗങ്ങളിലുമുള്ള പ്രവാചകമാതൃകയില്‍ നിന്ന് ഏറെ പഠിക്കുവാനുണ്ട്. തന്റെ ഇരുപത്തിയഞ്ചാമത്തെ വയസ്സില്‍ തന്നേക്കാള്‍ പ്രായക്കൂടുതലുള്ള ഒരു വിധവയെ വിവാഹം ചെയ്യുകയും നീണ്ട ഇരുപത്തിയഞ്ച് വര്‍ഷക്കാലം ആ ദാമ്പത്യം തുടരുകയും മക്കളുണ്ടാവുകയും പേരക്കുട്ടികളുണ്ടാവുകയുമെല്ലാം ചെയ്തിട്ടും ദാമ്പത്യജീവിതത്തിന്റെ മധുരിമ നിലനിര്‍ത്തിക്കൊണ്ട് തന്റെ ഇണയുടെ മരണം വരെ അവരോടൊപ്പം ജീവിക്കുകയും ചെയ്ത മുഹമ്മദ് (സ)-ഖദീജ (റ) ദാമ്പത്യത്തില്‍ ഏകഭാര്യത്വം സ്വീകരിക്കുന്ന മുസ്‌ലിംകള്‍ക്ക് കൃത്യമായ മാതൃകയുണ്ട്.

വൈധവ്യവും അനാഥത്വവും പേറേണ്ടി വരുന്ന പെണ്ണവസ്ഥകളില്‍ അവര്‍ക്ക് താങ്ങും തണലുമായിത്തീരുവാനും അതോടൊപ്പം ദാമ്പത്യജീവിതം ആസ്വദിക്കുവാനും അങ്ങനെ ബഹുഭാര്യത്വം ഉപയോഗിക്കാനാകുമെന്നും, എല്ലാ ഇണകള്‍ക്കും സംതൃപ്തമായ ദാമ്പത്യജീവിതം പ്രദാനം ചെയ്തുകൊണ്ട് മുസ്‌ലിം പുരുഷന് എങ്ങനെ നീതിമാനായ ഒരു ബഹുഭാര്യനായിത്തീരാമെന്നും പ്രവാചകന്റെ (സ) ബഹുഭാര്യത്വം നമുക്ക് മാതൃക കണിച്ചുതരുന്നുണ്ട്. ലൈംഗികസുഖമാസ്വദിക്കുന്നതിനുവേണ്ടി മാത്രമുള്ളതാണ് ഭാര്യ-ഭര്‍തൃ ജീവിതമെന്നു കരുതുന്നവര്‍ക്ക് പ്രവാചകന്റെ ഏകഭാര്യത്വത്തിലോ ബഹുഭാര്യത്വത്തിലോ മാതൃക കണ്ടെത്താനാവുകയില്ല. ഒന്നാണെങ്കിലും ഒന്നിലധികമാണെങ്കിലും ഇണകളുടെയെല്ലാം ഉത്തരവാദിത്തമേറ്റെടുക്കുകയും അവരുടെ അബലതകളില്‍ താങ്ങും തണലുമായി നില്‍ക്കുകയുമാണ് മുസ്‌ലിം പുരുഷന്റെ ഉത്തരവാദിത്തമെന്ന് പഠിപ്പിക്കപ്പെട്ടവര്‍ക്കു പ്രവാചകനില്‍ (സ) സമ്പൂര്‍ണമായ മാതൃക കാണാന്‍ കഴിയുകയും ചെയ്യും. പ്രവാചകജീവിതത്തിനല്ല നബിനിന്ദകരുടെ ഭൂമികയ്ക്കാണ് കുഴപ്പമെന്ന് സാരം.

ദുരിതമനുഭവിക്കുന്ന സഹോദരിമാരുടെ കരച്ചില്‍ കേള്‍ക്കാതെ അവരെ തെരുവിന് നല്‍കുന്നതാണ് ആധുനികതയെന്ന് കരുതുന്നവര്‍ പ്രയാസങ്ങളനുഭവിക്കുന്നവര്‍ക്ക് സ്വന്തം ജീവിതത്തിന്റെ തണലേകി സംരക്ഷിക്കണമെന്ന് സ്വന്തം മാതൃകയിലൂടെ കാണിച്ചുകൊടുത്ത അന്തിമപ്രവാചകനില്‍ (സ) നിന്ന് ഏറെ പഠിക്കേണ്ടതുണ്ട്. തന്റെ ആദ്യഭാര്യയായിരുന്ന ഖദീജ(റ)യുടെ വിയോഗാനന്തരമുള്ള ഓരോ പ്രവാചകവിവാഹങ്ങളിലും ദാമ്പത്യജീവിതമെന്നതിലുപരിയായ വലിയ ലക്ഷ്യങ്ങള്‍ കാണാന്‍ കഴിയും. വിധവയും മാതാവുമായ സൗദ ബിന്‍ത് സംഅയെന്ന അറുപതുകാരിയാണ് പ്രവാചകന്റെ (സ) ബഹുഭാര്യത്വജീവിതത്തിലേക്ക് ആദ്യമായി കടന്നുവന്ന സഖി. ഭര്‍തൃവിയോഗത്തിനുശേഷം അനാഥത്വത്തിന്റെ കയ്പുനീര്‍ കുടിച്ചുകൊണ്ടിരുന്ന, ആദ്യകാലത്ത് ഇസ്‌ലാം സ്വീകരിച്ച് ധൈര്യം കാണിച്ച ആ മാതൃകാവനിതയെ അവരുടെ വാര്‍ദ്ധക്യത്തിന്റെ തുടക്കത്തില്‍ തന്റെ സഖിയായി സ്വീകരിച്ച് മാതൃകയാവുകയായിരുന്നു നബി(സ). അബുസല്‍മയുടെ വിയോഗത്തിനുശേഷം പിഞ്ചുപൈതങ്ങളോടെ അവരുടെ മാതാവായ ഉമ്മുസല്‍മയെ ഏറ്റെടുക്കുകയും അനാഥത്വത്തില്‍നിന്ന് അവരെയും മക്കളെയും കരകയറ്റുകയും ചെയ്ത പ്രവാചകനില്‍ (സ) പറക്കമുറ്റാത്ത കുഞ്ഞുങ്ങളുണ്ടായിരിക്കെ മരണപ്പെടുന്നവരുടെ മക്കളെ സംരക്ഷിക്കേണ്ടത് എങ്ങനെയെന്ന വലിയ മാതൃകയുണ്ട്. തന്റെ സന്തതസഹചാരിയായിരുന്ന ഉമറി(റ)ന്റെ മകള്‍ ഹഫ്‌സ(റ)യെ അവരുടെ ഭര്‍ത്താവായ ഖുനൈസുബ്‌നു ഹുദൈഫ മരണപ്പെട്ടപ്പോള്‍ ഏറ്റെടുത്ത പ്രവാചകനില്‍ (സ) അനുയായികളുടെ സങ്കടങ്ങള്‍ക്ക് പ്രായോഗികപരിഹാരം നിര്‍ദ്ദേശിക്കുകയും അവരുടെ ബാധ്യതകളെ സ്വന്തം ഉത്തരവാദിത്വമായി ഏറ്റെടുക്കുകയും ചെയ്യുന്ന നേതാവിന്റെ മാതൃകയുണ്ട്.

ബദ്‌റില്‍ വെച്ച് മരണപ്പെട്ട ഭര്‍ത്താവിന്റെ വിയോഗത്താല്‍ സങ്കടമനുഭവിക്കുന്ന സൈനബ് ബിന്‍ത് ഖുസൈമയെ അവരുടെ തന്നെ ആവശ്യപ്രകാരം വിവാഹം ചെയ്തുകൊണ്ട് യുദ്ധത്തില്‍ മരണപ്പെട്ട മറ്റുള്ളവരുടെ വിധവകളെ ഏറ്റെടുക്കാന്‍ അനുചരന്‍മാരെ പ്രേരിപ്പിക്കുകയായിരുന്നു നബി(സ). ഇസ്‌ലാം സ്വീകരിച്ച ആദ്യപുരുഷനും തന്റെ സഖാക്കളില്‍ പ്രഥമഗണനീയനുമായ അബുബക്കറി(റ)ന്റെ മകള്‍ ആയിശ(റ)യുമായുള്ള വിവാഹത്തില്‍ കുടുംബബന്ധങ്ങള്‍ സൃഷ്ടിച്ചുകൊണ്ട് സൗഹൃദത്തെ ദൃഢീകരിക്കുന്നതെങ്ങനെയെന്ന മാതൃകയുണ്ട്. പ്രവാചകന്‍ (സ) സ്വീകരിച്ച ഒരേയൊരു കന്യകയായ ആയിശ (റ) ദാമ്പത്യബന്ധത്തില്‍ സ്വീകരിച്ചുപോന്ന പ്രവാചകമാതൃകകളുള്‍ക്കൊള്ളുന്ന നിരവധി ഹദീഥുകള്‍ നിവേദനം ചെയ്തുകൊണ്ട് പ്രസ്തുത വിവാഹത്തിലൂടെയുള്ള സാമൂഹികലക്ഷ്യം നിറവേറ്റി മാതൃകയാവുകയും ചെയ്തു. അബ്‌സീനിയായിലേക്ക് പാലായനം ചെയ്ത് തന്നോടൊപ്പം താങ്ങായി നിന്നിരുന്ന ഭര്‍ത്താവ് മതം മാറി മദ്യപനായിത്തീരുകയും മക്കയിലുള്ള പിതാവും സഹോദരങ്ങളും ഇസ്‌ലാമിനോടുള്ള ശത്രുത പരസ്യമായി പ്രഖ്യാപിക്കുകയും ചെയ്ത സാഹചര്യത്തില്‍ തികച്ചും അനാഥയും അഗതിയുമായിത്തീര്‍ന്ന ഉമ്മു ഹബീബ(റ)യെ മദീനാരാഷ്ട്രത്തിന്റെ തലവനായ പ്രവാചകന്‍ (സ) സ്വീകരിച്ചുകൊണ്ട് അശരണര്‍ക്ക് താങ്ങായിത്തീരുകയെന്ന ദൗത്യനിര്‍വഹണത്തിന് വിവാഹബന്ധത്തെ എങ്ങനെ ഉപയോഗിക്കാമെന്ന് പഠിപ്പിക്കുകയാണ് ചെയ്തത്.

പുതിയ സൗഹൃദങ്ങളിലൂടെ വ്യത്യസ്ത ഗോത്രങ്ങളുടെ ശാത്രവം കുറച്ചുകൊണ്ടുവരികയെന്ന ലക്ഷ്യമാണ് മൈമുന(റ)യുമായുള്ള വിവാഹത്തില്‍ നമുക്ക് കാണാനാവുക. യുദ്ധത്തില്‍ പിടിക്കപ്പെടുന്ന വനിതകളെ യുദ്ധത്തടവുകാരാക്കി അടിമകളാക്കുന്ന, അന്ന് നിലനിന്നിരുന്ന സമ്പ്രദായത്തില്‍ നിന്നുമാറി, അവരെ ഇണകളാക്കാനാവുന്നവര്‍ അങ്ങനെ ചെയ്യുകയാണ് വേണ്ടതെന്ന പാഠമാണ് ജുവൈരിയ(റ)യുമായും സ്വഫിയ്യ(റ)യുമായുള്ള വിവാഹങ്ങളിലൂടെ നബി (സ) പഠിപ്പിച്ചത്. ബനൂ മുസ്തലഖ് യുദ്ധത്തില്‍ പിടിക്കപ്പെട്ട ഗോത്രനേതാവിന്റെ മകളായ ജുവൈരിയ(റ)യെ സ്വതന്ത്രമാക്കി  വിവാഹം ചെയ്യുകവഴി അവരില്‍ നിന്ന് പിടിക്കപ്പെട്ടവരെയെല്ലാം സ്വതന്ത്രമാക്കുവാന്‍ മുസ്‌ലിം പടയാളികള്‍ സന്നദ്ധമാവുകയും അതുവഴി ആ ഗോത്രം മുഴുവന്‍ ഇസ്‌ലാം സ്വീകരിക്കുകയും ചെയ്തുവന്നതാണ് ചരിത്രം.

മാതാക്കളായും മക്കളായും തങ്ങള്‍ക്കിഷ്ടപ്പെട്ടവരെ വിളിക്കുകയും അവരുമായി ആ രൂപത്തിലുള്ള ബന്ധം പ്രഖ്യാപിക്കുകയും ചെയ്തിരുന്ന അജ്ഞാതകാലത്തെ സമ്പ്രദായത്തിന് അറുതി വരുത്തുകയെന്ന വലിയ ലക്ഷ്യത്തിനുവേണ്ടി അല്ലാഹുവിന്റെ നിര്‍ദ്ദേശപ്രകാരം നടന്നതാണ് സൈനബ് ബിന്‍ത് ജഹ്ശു (റ)മായുള്ള പ്രവാചകവിവാഹം.

ജീവിതത്തിന്റെ എല്ലാ രംഗങ്ങളിലുമെന്നതുപോലെ ഓരോ പ്രവാചകവിവാഹങ്ങളിലും വലിയ മാതൃകകളുണ്ടെന്ന വസ്തുതയാണ് ആ ജീവിതത്തെ വസ്തുനിഷ്ഠമായി പഠിച്ചവർക്കെല്ലാം മനസ്സിലാവുക.

വിഷയവുമായി ബന്ധപ്പെട്ട വീഡിയോ

ക്രൈസ്തവ വിമര്‍ശകരാണ് മുഹമ്മദ് നബിക്ക് ലഭിച്ച വെളിപാടുകള്‍ പിശാചില്‍നിന്നാണെന്ന ആരോപണം ഉന്നയിക്കുന്നത്. മുഹമ്മദി(സ)ന് ലഭിച്ച വെളിപാടുകള്‍ പിശാചുബാധയുടെ ഫലമായുണ്ടായതാണെന്ന് വരുത്തിത്തീര്‍ക്കാനാണ് സി.ഡി. ഫാണ്ടര്‍, ക്ലേയ്ര്‍ ടിസ്ഡാല്‍, ജോഷ്മാക്ഡവല്‍, ജോണ്‍ജില്‍ ക്രിസ്റ്റ്, ജി. നെഹ്ല്‍സ് തുടങ്ങിയ ക്രൈസ്തവ ഗ്രന്ഥകാരന്മാരെല്ലാം ശ്രമിച്ചിരിക്കുന്നത്. യേശുക്രിസ്തുവിന്റെ ക്രൂശീകര ണത്തെയും അതുമൂലമുള്ള പാപപരിഹാരത്തെയും നിഷേധിച്ചു കൊണ്ട് മനുഷ്യരാശിയെ പാപത്തിന്റെ ഗര്‍ത്തത്തില്‍തന്നെ തളച്ചിടുവാനുള്ള പി ശാചിന്റെ പരിശ്രമമാണ് ഖുര്‍ആനിന്റെ രചനക്കു പിന്നിലുള്ളതെന്ന് അവര്‍ വാദിക്കുന്നു. മനുഷ്യശരീരത്തില്‍ പിശാച് കയറിക്കൂടുമോ? പിശാചുബാധ കൊണ്ട് ഒരാള്‍ക്ക് രോഗങ്ങളുണ്ടാവുമോ? പിശാചുബാധിച്ച ഒരാള്‍ക്ക് വെളിപാടുണ്ടാവുമോ? തുടങ്ങിയ ചര്‍ച്ചകള്‍ ഇവിടെ അപ്രസക്തമാണ്. ബൈ ബിള്‍ പ്രകാരം പിശാചുബാധിച്ച ഒരാളില്‍ കാണപ്പെടുന്ന അസുഖങ്ങള്‍ എന്തെല്ലാമാണെന്ന് പരിശോധിക്കുക.

1. ബുദ്ധിഭ്രമത്താല്‍ അലറി വിളിക്കല്‍ (മാര്‍ക്കോസ് 1:24, ലൂക്കോസ് 9:39, യോഹന്നാന്‍ 10:20)

2. സ്വയം നശീകരണ പ്രവണത (മത്തായി 55:9, 18: 17, 15:32, മര്‍ക്കോസ് 5: 13, ലൂക്കോസ്, 8:33)

3. നഗ്‌നമായി നടക്കുന്നതിനുള്ള പ്രവണത (ലൂക്കോസ് 8:2, 8:35)

4. പിശാചിനാല്‍ തള്ളയിടപ്പെടുക (മത്തായി 17:15, മര്‍ക്കോസ് 1:26,9:18, 9:20,9:26)

5. മൂകത (മര്‍ക്കോസ് 9:25, 9:32, 12:22, ലൂക്കോസ് 11:14)

6. ബധിരത (മര്‍ക്കോസ് 9: 25)

7. അന്ധത (മത്തായി 12:22)

8. മറ്റാരും കാണാത്തത് കാണുകയും അറിയുകയും ചെയ്യുക (മര്‍ക്കോസ് 1:24, ലൂക്കോസ് 4:3, മത്തായി 8:29)

പിശാചുബാധിതനില്‍ കാണപ്പെടുന്നതെന്ന് ബൈബിള്‍ ഉദ്‌ഘോഷിക്കു ന്ന ലക്ഷണങ്ങളൊന്നും മുഹമ്മദി(സ)ല്‍ ഉണ്ടായിരുന്നതായി നമുക്ക് കാ ണാന്‍ കഴിയുന്നില്ല. ദൈവിക വെളിപാടുകള്‍ ലഭിക്കുമ്പോള്‍ അവ ഒരു മണിനാദം പോലെ തനിക്ക് അനുഭവപ്പെടാറുണ്ടെന്നും അതാണ് ഏറ്റവും പ്രയാസകരമായ വെളിപാടു രീതിയെന്നും മുഹമ്മദ്(സ) പറഞ്ഞതാണ് അദ്ദേഹത്തെ പിശാചുബാധിച്ചിരുന്നുവെന്നും പൈശാചിക വെളിപാടുകളാണ് ഖുര്‍ആനെന്നും വാദിക്കുന്നവരുടെ ഒരു തെളിവ്. വെളിപാട് ലഭിച്ചു കൊ ണ്ടിരുന്ന അതിശൈത്യമുള്ള ഒരു ദിവസം പ്രവാചകന്റെ നെറ്റിയില്‍ വിയര്‍പ്പുതുള്ളിയുണ്ടായിരുന്നതായി ഞാന്‍ കണ്ടുവെന്ന പ്രവാചകപത്‌നി ആഇശ(റ) യുടെ നിവേദനമാണ് മറ്റൊരു തെളിവ്. ഇവിടെ പ്രസക്തമായ ഒരു ചോദ്യമുണ്ട്. പിശാചുബാധിതന് ചെവിയില്‍ മണിയടിക്കുന്നതുപോലെ തോന്നുമെന്നോ അവന്റെ നെറ്റിത്തടം അതിശൈത്യമാണെങ്കിലും വിയര്‍പ്പുതുള്ളികളാല്‍ നിറയുമെന്നോ ബൈബിളില്‍ എവിടെയെങ്കിലുമുണ്ടോ? ഇല്ലെങ്കില്‍, പ്രവാചക(സ)നില്‍ പിശാചുബാധ ആരോപിക്കുവാന്‍ ബൈബിളിന്റെ അനുയായികള്‍ക്ക് എന്തടിസ്ഥാനമാണുള്ളത്?

പ്രവാചകന് ലഭിച്ച ദൈവിക സന്ദേശങ്ങള്‍ പിശാചുബാധയുടെ ഉല്‍പ ന്നങ്ങളാണെന്ന് പറയുന്നവര്‍ തങ്ങളുടെതന്നെ വിശുദ്ധന്മാരാണ് പിശാചു ബാധയേറ്റവരെന്ന് പറയാന്‍ നിര്‍ബന്ധിതരാവുമെന്നതാണ് വാസ്തവം.

യേശുവിന്റെ ജീവിതകാലമത്രയും അദ്ദേഹത്തെയും അദ്ദേഹം പഠിപ്പിച്ച ആശയങ്ങളെയും നശിപ്പിക്കുവാന്‍ വേണ്ടി അഹോരാത്രം പരിശ്രമിക്കുക യും (അപ്പോസ്തല പ്രവൃത്തികള്‍ 9:1, 26:10, 8:1) അദ്ദേഹത്തിനുശേഷം ക്രിസ്തു തനിക്ക് വെളിപ്പെട്ടിട്ടുണ്ടെന്ന് അവകാശവാദമുന്നയിക്കുകയും ചെയ്തയാളാണ് 'വിശുദ്ധ പൗലോസ്'. അദ്ദേഹത്തിന് ക്രിസ്തുദര്‍ശനം ലഭിച്ച രീതിയെക്കുറിച്ച് ബൈബിള്‍ വിവരിക്കുന്നത് കാണുക: ''പിന്നെ അയാള്‍ യാത്ര പുറപ്പെട്ട് ഡമാസ്‌കസിനെ സമീപിച്ചപ്പോള്‍, പെട്ടെന്ന് ആകാശത്തുനിന്ന് ഒരു പ്രകാശം അയാളുടെ ചുറ്റും മിന്നലൊളി പരത്തി. സാവൂള്‍ നിലം പതിച്ചു. 'സാവൂള്‍, സാവൂള്‍ നീ എന്നെ പീഡിപ്പിക്കുന്നത് എന്തിന്? എന്ന് തന്നോട് ചോദിക്കുന്ന ഒരു സ്വരം കേള്‍ക്കയായി. അപ്പോള്‍ അയാള്‍ ചോദിച്ചു: 'പ്രഭോ നീ ആരാണ്? അവന്‍ പറഞ്ഞു: നീ പീഡിപ്പിക്കുന്ന യേശുവാണ് ഞാന്‍. എഴുന്നേറ്റ് നഗരത്തില്‍ ചെല്ലുക. നീ ചെയ്യേണ്ടത് എന്തെന്ന് അവിടെ വെച്ച് നിനക്ക് അറിവ് കിട്ടും'. 'അയാളോടൊപ്പം യാത്ര ചെയ്തിരുന്ന ആളുകള്‍ സ്വരം കേട്ടെങ്കിലും ആരെയും കാണായ്കയാല്‍ വിസ്മയ സ്തബ്ധരായി നിന്നുപോയി. വീണുകിടന്നിടത്തുനിന്ന് സാവൂള്‍ എഴുന്നേറ്റു. കണ്ണുതുറന്നിട്ടും അയാള്‍ക്ക് ഒന്നും കാണാന്‍ കഴിഞ്ഞില്ല. അതിനാല്‍ അവര്‍ അയാളെ കൈക്കുപിടിച്ച് ഡമാസ്‌കസിലേക്കു കൊണ്ടുപോയി. മൂന്നു ദിവസത്തേക്ക് അയാള്‍ക്ക് കാഴ്ചയില്ലായിരുന്നു; അയാള്‍ തിന്നുകയോ കുടിക്കുകയോ ചെയ്തതുമില്ല'' (അപ്പോസ്തല പ്രവൃത്തികള്‍ 9:3-9)

നിലംപതിക്കുന്നതും കൂടെയുള്ളവര്‍ കാണാത്തത് കാണുന്നതും കേള്‍ ക്കാത്തത് കേള്‍ക്കുന്നതും കണ്ണു കാണാതാവുന്നതുമെല്ലാം പിശാചുബാധ യുടെ ലക്ഷണങ്ങളായി സുവിശേഷങ്ങളില്‍ സൂചിപ്പിക്കപ്പെട്ടിട്ടുള്ളത് നാം കണ്ടു. ക്രിസ്തുവിനെ താന്‍ കണ്ടുവെന്ന് പൗലോസ് അവകാശപ്പെട്ട സംഭവത്തില്‍ ഇതെല്ലാം അദ്ദേഹം അനുഭവിക്കുന്നുമുണ്ട്. പൗലോസിന് പിശാചുബാധയാണ് ഉണ്ടായതെന്ന് വാദിച്ചാല്‍ അത് അംഗീകരിക്കാന്‍ ക്രൈസ്തവ സമൂഹം സന്നദ്ധമാവുമോ? മുഹമ്മദി(സ)ന് പിശാചുബാധയായിരുന്നുവെന്ന് സമര്‍ഥിക്കുവാന്‍ ബൈബിളില്‍നിന്ന് ഒരു തെളിവെങ്കിലുമുദ്ധരിക്കാ ന്‍ ക്രൈസ്തവ വിമര്‍ശകര്‍ക്ക് കഴിയില്ല. അതേസമയം, നിലവിലുള്ള ക്രിസ്തുമതത്തിന്റെ സ്ഥാപകനായ പൗലോസിന് പിശാചുബാധയാണ് അനുഭ വപ്പെട്ടതെന്ന് ബൈബിള്‍ ഉപയോഗിച്ചു കൊണ്ട് സ്ഥാപിക്കാന്‍ ഒരാള്‍ക്ക് കഴിയും. അപ്പോള്‍ ആര്‍ക്കാണ് പിശാചുബാധ? ഇനി, മുഹമ്മദ് നബി(സ)ക്ക് പിശാച് ബാധിച്ചതുകൊണ്ടാണ് ഖുര്‍ആന്‍ എഴുതിയുണ്ടാക്കിയതെന്ന ക്രൈസ്തവവാദത്തിന്റെ ആണിക്കല്ല് പരിശോധിക്കുക. യേശുക്രിസ്തുവിന്റെ കുരിശുമരണത്തിലൂടെയുള്ള പാപപരിഹാരം എന്ന ആശയത്തെ വിമര്‍ശിക്കുന്നതുമൂലമാണല്ലോ ഖുര്‍ആന്‍ പിശാചിന്റെ സൃഷ്ടിയാണെന്ന് വാദിക്കുന്നത്.

എന്നാല്‍, യാഥാര്‍ഥ്യമെന്താണ്? യേശുക്രിസ്തു പരിശുദ്ധനായിരുന്നുവെന്ന് മുസ്‌ലിംകളും ക്രൈസ്തവരും വിശ്വസിക്കുന്നു. അദ്ദേഹം സര്‍വശക് തനാല്‍ നിയുക്തനായ വ്യക്തിയാണെന്ന് ഇരുകൂട്ടരും സമ്മതിക്കുന്നു. അദ്ദേഹത്തിന് പിശാചുബാധയുണ്ടായിട്ടില്ലെന്ന് ഇരുകക്ഷികളും പറയുന്നു. എങ്കില്‍, മുഹമ്മദ് നബി(സ)ക്കോ പൗലോസിനോ ആര്‍ക്കാണ് പിശാചില്‍നിന്ന് വെളിപാടുണ്ടായതെന്ന് പരിശോധിക്കാന്‍ നമുക്കെന്തുകൊണ്ട് യേശുക്രിസ്തുവിന്റെ ഉപദേശങ്ങളുമായി അവരുടെ ഉപദേശങ്ങളെ താരതമ്യം ചെയ്തുകൂടാ? പിശാചില്‍നിന്ന് വെളിപാടുണ്ടായ വ്യക്തി യേശുവിന്റെ ശത്രുവായിരിക്കുമല്ലോ. ഒരു ദൈവദൂതന്റെ ശത്രു അയാള്‍ പ്രബോധനം ചെയ്യുന്ന ആശയങ്ങളുടെ ശത്രുവായിരിക്കും എന്നോര്‍ക്കുക.

യേശു പറഞ്ഞു: നിയമത്തെ (തോറ)യോ പ്രവാചകന്മാരെയോ റദ്ദാക്കാനല്ല ഞാന്‍ വന്നത്' (മത്തായി 5:17). ഖുര്‍ആന്‍ പറയുന്നു: 'തീര്‍ച്ചയായും നാം തന്നെയാണ് തൗറാത്ത് അവ തരിപ്പിച്ചിരിക്കുന്നത്, അതില്‍ മാര്‍ഗദര്‍ശനവും പ്രകാശവുമുണ്ട്' (5:44).

'മര്‍യമിന്റെ മകന്‍ ഈസ പറഞ്ഞ സന്ദര്‍ഭം: ഇസ്രായേല്‍ സന്തതികളേ, എനിക്കുമുമ്പുള്ള തൗറാത്തിനെ സത്യപ്പെടുത്തുന്നവനായിക്കൊണ്ടും എനിക്ക് ശേഷം വരുന്ന അഹ്മദ് എന്നു പേരുള്ളൊരു ദൂതനെപ്പറ്റി സന്തോ ഷവാര്‍ത്ത അറിയിക്കുന്നവനായിക്കൊണ്ടും നിങ്ങളിലേക്ക് അല്ലാഹുവിന്റെ ദൂതനായി നിയോഗിക്കപ്പെട്ടവനാകുന്നു ഞാന്‍' (61:6).

പൗലോസ് എഴുതി: 'നിയമാനുഷ്ഠാനങ്ങളെ (തോറ) ആശ്രയിക്കുന്നവ രെല്ലാം ശാപഗ്രസ്തരാണ് (ഗലാത്തിയക്കാര്‍ 3:10).

'ക്രിസ്തു നിയമത്തിന്റെ ശാപത്തില്‍നിന്നു നമ്മെ മോചിപ്പിച്ചിരിക്കു ന്നു' (ഗലാത്തിയക്കാര്‍ 3:13). 'അവന്‍ (യേശു) തന്റെ ശരീരത്തില്‍, നിയമത്തെ അതിന്റെ കല്‍പന കളോടും അനുശാസനങ്ങളോടുംകൂടി റദ്ദാക്കി' (എഫേസോസുകാര്‍2:15)

ഞാന്‍ നിയമത്തെ റദ്ദാക്കാനല്ല വന്നതെന്ന് യേശു, ഖുര്‍ആനും അതുതന്നെ പറയുന്നു. പൗലോസാകട്ടെ യേശു നിയമത്തില്‍നിന്ന് ലോകത്തെ രക്ഷിക്കാനാണ് വന്നത് എന്നു സമര്‍ഥിക്കുന്നു. ആര്‍ക്കാണ് പിശാചിന്റെ വെളിപാട്?

യേശുക്രിസ്തു താന്‍ ദൈവമാണെന്ന് പഠിപ്പിച്ചില്ല (മര്‍ക്കോസ് 12:29, മത്തായി 4:10) ഇക്കാര്യം ഖുര്‍ആന്‍ അര്‍ഥശങ്കക്കിടയില്ലാത്തവണ്ണം വ്യക്ത മാക്കുന്നു (3:51), എന്നാല്‍ പൗലോസ് പറഞ്ഞതാകട്ടെ 'പ്രകൃത്യാതന്നെ ദൈവമായിരുന്നിട്ടും ദൈവത്തോടു തനിക്കുള്ള തുല്യതയെ, മുറുകെപ്പിടിച്ചുകൊണ്ടിരിക്കേണ്ട ഒരു കാര്യമായി അവന്‍ പരിഗണിച്ചില്ല? (ഫിലിപ്പിയര്‍ 2:6). 'അവന്‍ അദൃശ്യനായ ദൈവത്തിന്റെ പ്രതിരൂപമാണ്; സര്‍വസൃഷ്ടികളിലും ആദ്യജാതന്‍' (കൊളോസിയക്കാര്‍ 1:15) എന്നിങ്ങനെയാണ്. യേശുക്രിസ്തുവിന് സ്വയം താന്‍ ദൈവമാണെന്ന വെളിപാട് ലഭിച്ചിട്ടില്ല. അങ്ങനെ ലഭിച്ചിരുന്നുവെങ്കില്‍ അദ്ദേഹം അത് പറയുമായിരുന്നു. എന്നാല്‍, പൗലോസിന് യേശു ദൈവമായിരുന്നുവെന്ന് വെളിപാട് കിട്ടി. പ്രസ്തുത വെളിപാട് എവിടെനിന്നായിരിക്കണം?

അബ്രഹാമിനോട് ദൈവം ചെയ്ത ഉടമ്പടിയായിട്ടാണ് പരിച്‌ഛേദനാകര്‍മത്തെ ബൈബിള്‍ പരിചയപ്പെടുത്തുന്നത്. 'നീയും നിനക്കു ശേഷം തലമുറയായി നിന്റെ സന്തതികളും പാലിക്കേണ്ട ഉടമ്പടി'യെന്നു പറഞ്ഞുകൊണ്ടാണ് അബ്രഹാമിനോട് കര്‍ത്താവ് പരിച്‌ഛേദന ചെയ്യുന്നതിനുള്ള കല്‍പന നല്‍കുന്നത് (ഉല്‍പത്തി 17:9-14) കര്‍ത്താവ് മോശയോടു പറഞ്ഞ തായി ബൈബിള്‍ ഉദ്ധരിക്കുന്നു: 'എട്ടാം ദിവസം ശിശുവിന്റെ പരിച്‌ഛേ ദനം നടത്തണം (ലേവിയര്‍ 12:3) ഈ ദൈവിക കല്‍പന യേശുവും അനുസ രിച്ചിരുന്നു. 'എട്ടു ദിവസം പൂര്‍ത്തിയായപ്പോള്‍ ശിശുവിന് പരിച്‌ഛേദനം നടത്തി' (ലൂക്കോസ് 2:21). പരിച്‌ഛേദനം ചെയ്യേണ്ടതില്ലെന്ന് യേശു ആരോടും പറഞ്ഞില്ല. കാരണം അദ്ദേഹത്തിന് അത്തരത്തിലുള്ള ഒരു ബോധനം ലഭിച്ചിരുന്നില്ല. എന്നാല്‍ പൗലോസ് പറയുന്നത് കാണുക: 'പരിച്‌ഛേദനം സ്വീകരിക്കുന്നുവെങ്കില്‍ നിങ്ങള്‍ക്ക് ക്രിസ്തുവിനെക്കൊണ്ട് നേട്ടമില്ല' (ഗലാത്തിയക്കാര്‍ 5:2). ഈ വെളിപാട് പൗലോസിന് എവിടെനിന്ന് കിട്ടി? ദൈവത്തില്‍ നിന്നാകാന്‍ വഴിയില്ല. പിന്നെയോ?

പിശാചില്‍നിന്നാണ് മുഹമ്മദി(ല)ന് വെളിപാടുണ്ടായത് എന്നുപറയാ നുള്ള പ്രധാനപ്പെട്ട കാരണം കുരിശുമരണത്തെയും പാപപരിഹാരബലി യെയും ഖുര്‍ആന്‍ നിഷേധിക്കുന്നുവെന്നതാണല്ലോ. യേശുവിനെയും മാതാവിനെയും പുകഴ്ത്തുകയും ആദരിക്കുകയും ചെയ്യുന്ന ഒട്ടനവധി സൂക്ത ങ്ങള്‍ ഖുര്‍ആനിലുണ്ട്. ഖുര്‍ആനില്‍ പേരു പരാമര്‍ശിക്കപ്പെട്ട ഏക വനിത മര്‍യമാണെന്നോര്‍ക്കുക. യേശു ചെയ്തതായി ബൈബിളില്‍ പറയാത്ത കളിമണ്‍പക്ഷികളില്‍ ഊതി അവയ്ക്ക് ജീവനിടുക തുടങ്ങിയ അത്ഭുതങ്ങ ളെക്കുറിച്ച് ഖുര്‍ആന്‍ പ്രതിപാദിക്കുന്നുമുണ്ട് (3:49). തൊട്ടിലില്‍ വെച്ച് ഉണ്ണിയേശു സംസാരിച്ചതായുള്ള ഖുര്‍ആനിക പരാമര്‍ശം (19:30) ബൈബി ളിലൊരിടത്തും കാണുവാന്‍ സാധ്യമല്ല. യേശുവിന്റെ വിശുദ്ധ വ്യക്തിത്വത്തില്‍ കളങ്കമുണ്ടാക്കുന്ന യാതൊന്നും ഖുര്‍ആനിലില്ല. യോഹന്നാന്റെ സുവിശേഷ പ്രകാരം ക്രിസ്തുവിന്റെ ആദ്യത്തെ അത്ഭുതം കാനായിലെ കല്യാണവിരുന്നില്‍ വെച്ച് മദ്യം നിര്‍മിച്ചു നല്‍കിയതാണെന്ന കാര്യം പ്രത്യേകം പ്രസ്താവ്യമാണ് (യോഹന്നാന്‍ 2:1-11). ഖുര്‍ആനില്‍ ഇത്തരം യാതൊരു പരാമര്‍ശവുമില്ല.

'മരത്തില്‍ തൂക്കിക്കൊല്ലപ്പെടുന്നവന്‍ ദൈവത്താല്‍ ശപിക്കപ്പെട്ടവനാണ്' (ആവര്‍ത്തനം 21:23)എന്നാണ് ബൈബിളിന്റെ സിദ്ധാന്തം. കുരിശില്‍ തറക്കുക വഴി യേശുവിനെ ശപിക്കപ്പെട്ടവനായി മുദ്രയടിക്കുകയാണ് തങ്ങ ള്‍ ചെയ്തതെന്നാണ് യഹൂദര്‍ കരുതിയത്. പൗലോസ് പറയുന്നതും മറ്റൊന്നല്ല. 'മരത്തില്‍ തൂക്കപ്പെടുന്നവരെല്ലാം ശപിക്കപ്പെട്ടവര്‍ എന്ന് എഴുതിയിരിക്കുന്നതുപോലെ ക്രിസ്തു നമുക്കുവേണ്ടി ശാപമായിത്തീരുന്നു' (ഗലാത്യര്‍ 3:13). അപ്പോള്‍ ക്രൂശീകരണം യേശുവിനെ ശപിക്കപ്പെട്ടവനാക്കുകയാണ് ചെയ്യുന്നത്. ലോകത്തിനുവേണ്ടി യേശു ശാപമായിത്തീര്‍ന്നുവെന്ന വാദം ഖുര്‍ആന്‍ അംഗീകരിക്കുന്നില്ല. ശാപത്തിന്റെ മരക്കുരിശില്‍നിന്ന് തന്നെ രക്ഷിക്കേണമേയെന്ന ക്രിസ്തുവിന്റെ പ്രാര്‍ഥന (മത്തായി 26:39) ദൈവം കേട്ടില്ലെന്നു കരുതുന്നത് ദൈവിക കാരുണ്യത്തിന്റെ നിഷേധമല്ലാതെ മറ്റെന്താണ്? ശപിക്കപ്പെട്ട മരക്കുരിശില്‍നിന്ന് പടച്ചതമ്പുരാന്‍ യേശുവിനെ രക്ഷിച്ചുകൊണ്ട് യഹൂദന്മാരുടെ ഗൂഢാലോചനയെ തകര്‍ക്കുകയാണ് ചെയ്തത് എന്നാണ് ഖുര്‍ആന്‍ പഠിപ്പിക്കുന്നത് (4:157,158).

മരക്കുരിശില്‍ ക്രൂശിക്കുക വഴി യേശുവിനെ ശപിക്കപ്പെട്ടവനാക്കിയെന്ന് യഹൂദന്മാര്‍. മരക്കുരിശില്‍ മരിച്ച് യേശു ശപിക്കപ്പെട്ടവനായിത്തീര്‍ന്നുവെന്ന് പൗ ലോസ്. മരക്കുരിശില്‍നിന്ന് പരിശുദ്ധനായ യേശുവിനെ ദൈവം രക്ഷിച്ചുവെന്ന് ഖുര്‍ആന്‍.

ഏതാണ് പിശാചിന്റെ വെളിപാട്? യേശുവിനെ മഹത്വപ്പെടുത്തുന്നതോ അതല്ല ശാപഗ്രസ്തനാക്കുന്നതോ? ചുരുക്കത്തില്‍, ഖുര്‍ആന്‍ പൈശാചിക വെളിപാടാണെന്ന് സമര്‍ഥിക്കു വാന്‍ വേണ്ടി തെളിവുകള്‍ പരതുന്നവര്‍ കുഴിക്കുന്ന കുഴികളില്‍ തങ്ങള്‍തന്നെയാണ് വീഴുന്നത് എന്നുള്ളതാണ് യാഥാര്‍ഥ്യം.

യുക്തിവാദികളായ വിമര്‍ശകന്മാര്‍ പ്രധാനമായും ഉന്നയിക്കുന്ന ആരോ പണമാണ് മുഹമ്മദ്(സ) നബിക്ക് ഉന്മാദരോഗ (Schizophrenia) മായിരുന്നുവെന്നത്. ദൈവത്തിന്റെ അസ്തിത്വം അംഗീകരിക്കാത്തവരെ സംബന്ധിച്ചിടത്തോളം അവര്‍ക്ക് വെളിപാടുകളുടെ സത്യതയെക്കുറിച്ച് എത്രതന്നെ പറഞ്ഞാലും ഉള്‍ക്കൊള്ളാന്‍ കഴിയില്ല. അതുകൊണ്ടുതന്നെ നിരീശ്വരവാദികളോടുള്ള ചര്‍ച്ച തുടങ്ങേണ്ടത് ദൈവാസ്തിത്വത്തെക്കുറിച്ച് സംസാരിച്ചുകൊണ്ടാണ്. പടച്ചതമ്പുരാന്റെ അസ്തിത്വംതന്നെ അംഗീകരിക്കാത്തവരെ അവനില്‍നിന്നുള്ള വെളിപാടുകള്‍ സത്യസന്ധമാണെന്ന് സമ്മതിപ്പിക്കുന്ന തെങ്ങനെ?

ചോദ്യത്തിന്റെ രണ്ടാം ഭാഗമാണ് ആദ്യമായി ചര്‍ച്ച ചെയ്യപ്പെടേണ്ടത്. സമകാലികരാല്‍ മുഹമ്മദ് (സ) ഭ്രാന്തനെന്ന് വിളിക്കപ്പെട്ടിരുന്നുവോ? ഉണ്ടെ ങ്കില്‍ ഭ്രാന്തിന്റെ എന്തെല്ലാം ലക്ഷണങ്ങളുടെ അടിസ്ഥാനത്തിലാണ് അവര്‍ ഈ ആരോപണം ഉന്നയിച്ചത്?

നാല്‍പതു വയസ്സുവരെ സത്യസന്ധനും സര്‍വരാലും അംഗീകരിക്ക പ്പെട്ട വ്യക്തിത്വത്തിന്റെ ഉടമയുമായിരുന്നു മുഹമ്മദ് (സ) . സുദീര്‍ഘമായ ഈ കാലഘട്ടത്തിനിടയ്ക്ക് ആരെങ്കിലും അദ്ദേഹത്തില്‍ ഏതെങ്കിലും തരത്തിലുള്ള മാനസിക വിഭ്രാന്തി ആരോപിച്ചിട്ടില്ല.

പ്രവാചകത്വത്തിനുശേഷം അദ്ദേഹം ഭ്രാന്തനെന്ന് ആരോപിക്കപ്പെട്ടിരുന്നുവെന്നത് ശരിയാണ്. ഭ്രാന്ത നെന്ന് മാത്രമല്ല മുഹമ്മദ്(സ) അധിക്ഷേപിക്കപ്പെട്ടത്; ജ്യോല്‍സ്യന്‍, മാരണ ക്കാരന്‍, മാരണം ബാധിച്ചവന്‍, കവി എന്നിങ്ങനെയുള്ള അധിക്ഷേപങ്ങളെ ല്ലാം അദ്ദേഹത്തിനുനേരെ ഉന്നയിക്കപ്പെട്ടിരുന്നു. അദ്ദേഹത്തിന്റെ വ്യക്തിത്വത്തിലോ മാനസിക സംതുലനത്തിലോ വല്ല വ്യത്യാസവും പ്രകടമായതുകൊണ്ടാണോ അവര്‍ അങ്ങനെ അധിക്ഷേപിച്ചത്? ആണെന്ന് അവരാരുംതന്നെ വാദിച്ചിട്ടില്ല. അവരുടെ പ്രശ്‌നം ഖുര്‍ആനും അതുള്‍ക്കൊള്ളുന്ന ആശയങ്ങളുമായിരുന്നു. തങ്ങളുടെ പാരമ്പര്യ വിശ്വാസങ്ങള്‍ക്കെതിരെയാണ് മുഹമ്മദ്(സ) സംസാരിക്കുന്നത്. അദ്ദേഹം ദൈവികമാണെന്ന് പറഞ്ഞുകൊണ്ട് ഓതിക്കേള്‍പ്പിക്കുന്ന ഖുര്‍ആനിലേക്ക് ജനങ്ങള്‍ ആകൃഷ്ടരാവുക യും ചെയ്യുന്നു. മുഹമ്മദി(സ)നെ സ്വഭാവഹത്യ നടത്താതെ ജനങ്ങളെ അദ്ദേഹത്തില്‍നിന്ന് അകറ്റാന്‍ മറ്റു മാര്‍ഗങ്ങളൊന്നുമില്ലെന്ന് കണ്ട പാരമ്പ ര്യമതത്തിന്റെ കാവല്‍ക്കാര്‍ ബോധപൂര്‍വം കെട്ടിച്ചമച്ച സ്വഭാവഹത്യയായിരുന്നു ഇവയെല്ലാം.

മുഹമ്മദ്(സ) പ്രവാചകത്വം പരസ്യമായി പ്രഖ്യാപിച്ചകാലം. ഹജ്ജ് മാസം ആസന്നമായി. അറേബ്യയുടെ വിവിധ ഭാഗങ്ങളില്‍നിന്നും ഹജ്ജിനു വരുന്നവരോട് മുഹമ്മദ് (സ) മതപ്രബോധനം നടത്തുമെന്നും ഖുര്‍ആനിന്റെ വശ്യതയില്‍ അവര്‍ ആകൃഷ്ടരാവുമെന്നും മക്കയിലെ പ്രമാണിമാര്‍ ഭയന്നു. അവര്‍ യോഗം ചേര്‍ന്നു. ഹജ്ജിന് എത്തിച്ചേരുന്നവരോട് ആദ്യമേതന്നെ മുഹമ്മദി(സ)നെതിരെ പ്രചാരവേലകള്‍ നടത്താന്‍ തീരുമാനിച്ചു. മുഹമ്മദി(സ)നെ എങ്ങനെ വിശേഷിപ്പിക്കണം, എന്നതായി പിന്നീടുള്ള ചര്‍ച്ച. പലരും പല രൂപത്തില്‍ പറയുന്നത് തങ്ങളുടെ വിശ്വാസ്യത തകര്‍ക്കും. എല്ലാവര്‍ക്കും ഒരേ രൂപത്തില്‍ പറയാന്‍ പറ്റുന്ന ആരോപണമെന്ത്? ചിലര്‍ പറഞ്ഞു: ''നമുക്ക് മുഹമ്മദ് ഒരു ജ്യോല്‍സ്യനാണെന്ന് പറയാം''. പൗരപ്രമുഖനായ വലീദുബ്‌നുമുഗീറ പറഞ്ഞു: ''പറ്റില്ല, അല്ലാഹുവാണ് സത്യം അവ ന്‍ ജ്യോല്‍സ്യനല്ല. ജ്യോല്‍സ്യന്മാരെ നാം കണ്ടിട്ടുണ്ട്. മുഹമ്മദിന്റെ വാക്കുകള്‍ ജ്യോല്‍സ്യന്മാരുടെ പ്രവചനങ്ങളല്ല''. മറ്റു ചിലര്‍ പറഞ്ഞു: ''നമുക്ക് അവന്‍ ഭ്രാന്തനാണെന്ന് പറയാം''. വലീദ് പറഞ്ഞു: ''അവന്‍ ഭ്രാന്തനല്ല. ഭ്രാന്തന്മാരെ നാം കണ്ടിട്ടുണ്ട്. അവരുടെ ഭ്രാന്തമായ സംസാരങ്ങളോ ഗോഷ്ഠികളോ പിശാചുബാധയോ ഒന്നും അവനില്ല''. അവര്‍ പറഞ്ഞു: ''എങ്കില്‍ അവന്‍ കവിയാണെന്ന് പറയാം''. വലീദ് പ്രതിവചിച്ചു: ''അവന്‍ കവിയല്ല. കവിതയുടെ എല്ലാ ഇനങ്ങളും നമുക്കറിയാം. അവന്‍ പറയുന്ന ത് കവിതയല്ല''. ജനം പറഞ്ഞു: ''എങ്കില്‍ അവന്‍ മാരണക്കാരനാണെന്ന് പറയാം' വലീദ് പ്രതികരിച്ചു: ''അവന്‍ മാരണക്കാരനുമല്ല. മാരണക്കാരെ നമുക്കറിയാം. അവരുടെ കെട്ടുകളോ, ഊത്തുകളോ ഒന്നും അവന്‍ പ്രയോഗിക്കുന്നില്ല''. അവര്‍ ചോദിച്ചു: ''പിന്നെ എന്താണ് നിങ്ങളുടെ നിര്‍ദേശം?'' അദ്ദേഹം പറഞ്ഞു: ''തീര്‍ച്ചയായും അവന്റെ വചനങ്ങളില്‍ മാധുര്യമുണ്ട്. അതിന്റെ മൂലം വിസ്തൃതവും ശാഖകള്‍ ഫലസമൃദ്ധവുമാണ്. നിങ്ങള്‍ അവനെപ്പറ്റി എന്തു പറഞ്ഞാലും അതു നിരര്‍ഥകമാണെന്നു തെളിയും. പിതാവിനും മക്കള്‍ക്കുമിടയിലും ഭാര്യക്കും ഭര്‍ത്താവിനുമിടയിലും ജ്യേഷ്ഠനും അനുജനുമിടയിലും പിളര്‍പ്പുണ്ടാക്കുവാന്‍ വേണ്ടി വന്ന ജാലവിദ്യക്കാരനാണ് അവനെന്ന് പറയുന്നതാണ് നല്ലത്!'' ജനം ഇതംഗീകരിച്ചു. അവര്‍ പ്രചാര ണം തുടങ്ങി.

ഈ സംഭവം മനസ്സിലാക്കിത്തരുന്ന വസ്തുതയെന്താണ്? പ്രവാചകപ്ര ബോധനങ്ങളില്‍ നിന്ന് ജനങ്ങളെ പിന്തിരിപ്പിക്കുവാന്‍ വേണ്ടി ശത്രുക്കള്‍ മെനഞ്ഞെടുത്ത പലതരം ദുഷ്പ്രചാരണങ്ങളിലൊന്നു മാത്രമാണ് അദ്ദേ ഹം ഭ്രാന്തനാണെന്ന ആരോപണം. ഈ പ്രചാരണം നടത്തിയിരുന്നവര്‍ക്കു തന്നെ അതില്‍ വിശ്വാസമുണ്ടായിരുന്നില്ല. അതുകൊണ്ടുതന്നെ, അവരുടെ പ്രചാരണത്തെ ഒരു തെളിവായി സ്വീകരിക്കുന്നത് അബദ്ധമാണ്.

പ്രവാചകന്‍ ജീവിച്ചത് പതിനാലു നൂറ്റാണ്ടുകള്‍ക്കുമുമ്പാണ്. അദ്ദേഹ ത്തിന് ഉന്മാദരോഗമുണ്ടായിരുന്നുവോയെന്ന് വസ്തുനിഷ്ഠമായി പരിശോധിക്കുവാന്‍ ഇപ്പോള്‍ അദ്ദേഹം നമ്മുടെ മുന്നില്‍ ജീവിച്ചിരിക്കാത്തതിനാല്‍ ഇന്ന് നമുക്ക് കഴിയില്ല. അദ്ദേഹത്തിനുണ്ടായ വെളിപാടുകളും സ്വപ്‌നദര്‍ശ നങ്ങളുമാണ് പ്രവാചകന്‍ ഉന്മാദരോഗിയായിരുന്നുവെന്ന് വാദിക്കുന്നവര്‍ക്കുള്ള തെളിവ്. വെളിപാടുകള്‍ സ്വീകരിക്കുമ്പോള്‍ പ്രവാചകനില്‍ കാണ പ്പെട്ട ഭാവവ്യത്യാസങ്ങളെയും വഹ്‌യ് എങ്ങനെയാണെന്നുള്ള പ്രവാചക ന്റെ വിവരങ്ങളെയും വിശദീകരിക്കുന്ന ഹദീഥുകളുടെ വെളിച്ചത്തിലാണ് വിമര്‍ശകന്മാര്‍ ഈ വാദമുന്നയിക്കുന്നത്. ഉന്മാദരോഗത്തിന്റെ ലക്ഷണ ങ്ങള്‍ പ്രവാചകനില്‍ കാണപ്പെട്ടിരുന്നുവോയെന്ന് വസ്തുനിഷ്ഠമായി പരി ശോധിച്ചാല്‍ ഈ വാദത്തില്‍ യാതൊരു കഴമ്പുമില്ലെന്ന് സുതരാം വ്യക് തമാവും.

ഒന്ന്: ഉന്മാദരോഗികളുടെ സ്വഭാവം നിരന്തരം മാറിക്കൊണ്ടിരിക്കും. മറ്റുള്ളവരോടുള്ള പെരുമാറ്റത്തിലും സംസാരത്തിലുമെല്ലാം ഈ വൈരുധ്യം പ്രകടമായിരിക്കും.

മുഹമ്മദി(സ)ന്റെ ജീവിതവും സംസാരങ്ങളും പരിശോധിക്കുക.

യാതൊരു രീതിയിലുള്ള സ്വഭാവ വൈരുദ്ധ്യങ്ങളും അദ്ദേഹത്തില്‍ നമുക്ക് കാണാന്‍ കഴിയില്ല. മാറിക്കൊണ്ടിരിക്കുന്ന പെരുമാറ്റ രീതികളുടെയും പൂര്‍വാപരബന്ധമില്ലാത്ത സംസാരത്തിന്റെയും ഉടമസ്ഥനായിരുന്നു മുഹമ്മദ് നബി(ല)യെങ്കില്‍ അദ്ദേഹത്തിന് പരശ്ശതം അനുയായികളുണ്ടായതെങ്ങ നെ? സാധാരണയായി നാം മനസ്സിലാക്കുന്ന 'ദിവ്യന്‍'മാരുടെ അനുയായികളെപ്പോലെയായിരുന്നില്ല മുഹമ്മദി(സ)ന്റെ അനുചരന്മാര്‍. അദ്ദേഹത്തിന്റെ ഉപദേശങ്ങള്‍ പ്രാവര്‍ത്തികമാക്കുന്നതിനുവേണ്ടി മല്‍സരിക്കുകയായിരുന്നു അവര്‍. ഒരു ഉന്മാദരോഗിയുടെ വാക്കുകള്‍ അനുസരിക്കുവാന്‍ വേണ്ടി ജനസഹസ്രങ്ങള്‍ മല്‍സരിച്ചുവെന്ന് പറഞ്ഞാല്‍ അത് വിശ്വസിക്കാനാവുമോ?

രണ്ട്: ഉന്മാദരോഗികളുടെ പ്രതികരണങ്ങള്‍ വൈരുധ്യാത്മകമായിരിക്കും. സന്തോഷവേളയില്‍ പൊട്ടിക്കരയുകയും സന്താപവേളയില്‍ പൊട്ടിച്ചിരിക്കുകയും ചെയ്യും. വെറുതെ ചിരിക്കുകയും കരയുകയും ചെയ്യുന്ന സ്വഭാവവും കണ്ടുവരാറുണ്ട്.

മുഹമ്മദ് നബി(സ) യുടെ പ്രതികരണങ്ങള്‍ സമചിത്തതയോടുകൂടിയുള്ളതായിരുന്നു. ഒരു സംഭവം: പ്രവാചകന്‍(സ)ഒരു മരത്തണലില്‍ വിശ്രമിക്കുകയാണ്. പെട്ടെന്ന് ഊരിപ്പിടിച്ച വാളുമായി മുന്നില്‍ ഒരു കാട്ടാളന്‍ പ്രത്യക്ഷപ്പെ ട്ടു. അയാള്‍ ചോദിച്ചു: ''എന്നില്‍നിന്ന് നിന്നെ ഇപ്പോള്‍ ആര് രക്ഷിക്കും?'' പ്രവാചകന്‍ അക്ഷോഭ്യനായി മറുപടി പറഞ്ഞു: 'അല്ലാഹു'. ഈ മറുപടിയു ടെ ദൃഢത കേട്ട് കാട്ടാളന്റെ കൈയില്‍നിന്ന് വാള്‍ വീണുപോയി. (ബുഖാരി, മുസ്‌ലിം)

ഒരു ഉന്മാദരോഗിയില്‍നിന്ന് ദൃഢചിത്തതയോടുകൂടിയുള്ള ഇത്തരം പെരുമാറ്റങ്ങള്‍ പ്രതീക്ഷിക്കുവാന്‍ കഴിയുമോ?

മൂന്ന്: ഉന്മാദരോഗികള്‍ അന്തര്‍മുഖരായിരിക്കും. പുറമെയുള്ള ലോക ത്ത് നടക്കുന്ന സംഭവങ്ങളിലൊന്നും അവര്‍ക്ക് യാതൊരു താല്‍പര്യവും കാണുകയില്ല.

മുഹമ്മദ് നബി(സ)അന്തര്‍മുഖനായിരുന്നില്ല. തന്റെ ചുറ്റുപാടും നടക്കുന്ന സംഭവങ്ങള്‍ അതീവ താല്‍പര്യത്തോടെ നിരീക്ഷിക്കുകയും തന്റെ പങ്ക് ആവശ്യമെങ്കില്‍ നിര്‍വഹിക്കുകയും ചെയ്തിരുന്ന വ്യക്തിയാണദ്ദേ ഹം. ജനങ്ങള്‍ക്ക് ധാര്‍മിക നിര്‍ദേശങ്ങള്‍ നല്‍കുക മാത്രമല്ല, അവര്‍ക്ക് മാതൃകയായി ജീവിച്ച് കാണിച്ചുകൊടുക്കുകകൂടി ചെയ്ത വ്യക്തിയായിരുന്നു അദ്ദേഹം.

ലാമാര്‍ട്ടിന്‍ എഴുതി: 'തത്ത്വജ്ഞാനി, പ്രസംഗകന്‍, ദൈവദൂതന്‍, നിയമ നിര്‍മാതാവ്, പോരാളി, ആശയങ്ങളുടെ ജേതാവ്, അബദ്ധ സങ്കല്‍പങ്ങളില്‍നിന്ന് മുക്തമായ ആചാര വിശേഷങ്ങളുടെയും യുക്തിബന്ധുരമായ വിശ്വാസപ്രമാണങ്ങളുടെയും പുനഃസ്ഥാപകന്‍, ഇരുപത് ഭൗതിക സാമ്രാജ്യങ്ങളുടെ സ്ഥാപകന്‍ -അതായിരുന്നു മുഹമ്മദ്. മനുഷ്യത്വത്തിന്റെ എല്ലാ മാനദണ്ഡങ്ങളും വെച്ച് പരിഗണിക്കുമ്പോള്‍ നാം വ്യക്തമായും ചോദിച്ചേക്കാം. മുഹമ്മദിനേക്കാള്‍ മഹാനായ മറ്റു വല്ല മനുഷ്യനുമുണ്ടോ?''(Historie De La turquie., Vol, 2 Page 277)

അന്തര്‍മുഖനായ ഒരു ഉന്മാദരോഗിയെക്കുറിച്ച വിലയിരുത്തലാണോ ഇത്?

നാല്: ഉന്മാദരോഗികള്‍ക്ക് നിര്‍ണിതമായ എന്തെങ്കിലും ലക്ഷ്യത്തിനുവേണ്ടി വ്യവസ്ഥാപിതമായി പ്രവര്‍ത്തിക്കാന്‍ കഴിയില്ല. കാര്യമായി യാതൊന്നും ചെയ്യാനാവാത്ത ഇവര്‍ ശാരീരികമായും മാനസികമായും തളര്‍ന്നവരായിരിക്കും.

മുഹമ്മദ് നബി(സ) ജനങ്ങളെ സത്യമാര്‍ഗത്തിലേക്ക് നയിക്കുന്നതിനുവേണ്ടി അയക്കപ്പെട്ട ദൈവദൂതന്മാരില്‍ അന്തിമനായിരുന്നു. തന്നിലേല്‍പിക്കപ്പെട്ട ഉത്തരവാദിത്തം രണ്ടു ദശാബ്ദത്തിലധികം ഭംഗിയായി നിര്‍വഹിക്കാന്‍ അദ്ദേഹത്തിന് കഴിഞ്ഞു. ചിട്ടയോടുകൂടിയുള്ള പ്രബോധന പ്രവര്‍ ത്തനങ്ങള്‍ വഴി ജനസഹസ്രങ്ങളെ ദൈവികമതത്തിലേക്ക് ആകര്‍ഷിക്കുവാന്‍ മുഹമ്മദി(സ)ന് സാധിച്ചു. സാംസ്‌കാരിക രംഗത്ത് വട്ടപ്പൂജ്യത്തിലായിരുന്ന ഒരു ജനവിഭാഗത്തെ ലോകത്തിന് മുഴുവന്‍ മാതൃകയാക്കി പരിവര്‍ത്തിപ്പിക്കുവാന്‍ വേണ്ടിവന്നത് കേവലം ഇരുപത്തിമൂന്ന് വര്‍ഷങ്ങള്‍ മാത്രം. ലോകചരിത്രത്തെ ഏറ്റവുമധികം സ്വാധീനിച്ച വ്യക്തിയാണ് മുഹമ്മദ്(സ) എന്ന് ചരിത്രത്തെ നിഷ്പക്ഷമായി നോക്കിക്കണ്ടവരെല്ലാം അഭിപ്രായപ്പെട്ടിട്ടുണ്ട്.

ഇതെല്ലാം ഒരു ഉന്മാദരോഗിക്ക് കഴിയുന്നതാണെന്ന് പ്രസ്തുത രോഗ ത്തെക്കുറിച്ച് അല്‍പമെങ്കിലും അറിയുന്നവരാരെങ്കിലും സമ്മതിക്കുമോ?

അഞ്ച്: ഉന്മാദരോഗി അശരീരികള്‍ കേള്‍ക്കുകയും(Auditory Hallucination) മിഥ്യാഭ്രമത്തിലായിരിക്കുകയും (Delusion)  മായാദൃശ്യങ്ങള്‍ കാണുക യും(Hallucination) ചെയ്യും. ഈ അശരീരികളും മായാദൃശ്യങ്ങളും യാഥാര്‍ഥ്യവുമായി യാതൊരു ബന്ധവുമുള്ളതായിരിക്കില്ല.

മുഹമ്മദ് നബി(സ)ക്കുണ്ടായ വെളിപാടുകളും ദര്‍ശനങ്ങളും ഈ ഗണത്തില്‍ പെടുത്തിക്കൊണ്ടാണ് വിമര്‍ശകര്‍ അദ്ദേഹത്തില്‍ ഉന്മാദരോഗം ആരോപിക്കുന്നത്. ഉന്മാദരോഗത്തിന്റെ മറ്റു ലക്ഷണങ്ങളൊന്നും നബി(സ)യില്‍ ഉണ്ടായിരുന്നില്ലെന്ന് നാം മനസ്സിലാക്കി. അപ്പോള്‍ ഈ വെളിപാടുകളുടെ മാത്രം വെളിച്ചത്തില്‍ അദ്ദേഹം ഉന്മാദരോഗിയാണെന്ന് പറയുന്ന തെങ്ങനെ? ഉന്മാദരോഗിക്കുണ്ടാവുന്ന 'വെളിപാടു'കള്‍ അയാളുടെ രോഗത്തിന്റെ ലക്ഷണമാണ്. ഈ വെളിപാടുകള്‍ അയാളുടെ വൈയക്തിക മേഖലകളുമായി മാത്രം ബന്ധപ്പെട്ടതായിരിക്കും. എന്നാല്‍, മുഹമ്മദി(സ)നുണ്ടായ വെളിപാടുകളോ? ആ വെളിപാടുകള്‍ ഒരു ഉത്തമ സമൂഹത്തെ പടിപടിയായി വാര്‍ത്തെടുക്കുകയായിരുന്നു. ആദ്യം ദൈവബോധവും പര ലോകചിന്തയും ജനങ്ങളില്‍ വളര്‍ത്തി. ഘട്ടം ഘട്ടമായി സമൂഹത്തെ മുച്ചൂടും ബാധിച്ചിരുന്ന എല്ലാ തിന്മകളുടെയും അടിവേരറുത്തു. അങ്ങനെ ഒരു മാതൃകാ സമൂഹത്തിന്റെ സൃഷ്ടിക്ക് നിമിത്തമാകുവാന്‍ മുഹമ്മദി(സ)ന് ലഭിച്ച വെളിപാടുകള്‍ക്ക് കഴിഞ്ഞു. അത് സൃഷ്ടിച്ച വിപ്ലവം മഹത്തരമാണ്. ചരിത്രകാലത്ത് അതിനു തുല്യമായ മറ്റൊരു വിപ്ലവം നടന്നിട്ടില്ല.

ഉന്മാദരോഗി കേള്‍ക്കുന്ന അശരീരികള്‍ക്ക് ഒരു മാതൃകാ സമൂഹത്തിന്റെ സൃഷ്ടിക്കോ നിസ്തുലമായ ഒരു വിപ്ലവത്തിനോ നിമിത്തമാകുവാന്‍ കഴിയുമോ?

മുഹമ്മദി(സ)ന് ഉന്മാദരോഗമായിരുന്നുവെന്നും അദ്ദേഹം ശ്രവിച്ച അശ രീരികളാണ് ഖുര്‍ആനിലുള്ളതെന്നുമുള്ള വാദം പരിഗണനപോലും അര്‍ഹിക്കാത്ത ആരോപണം മാത്രമാണെന്നാണ് ഇതില്‍നിന്നെല്ലാം വ്യക്തമാകുന്നത്.

നങ്ങളെ ധാര്‍മികതയിലേക്ക് നയിക്കുന്ന ഗ്രന്ഥമാണ് ഖുര്‍ആന്‍. മദ്യ ത്തിലും മദിരാക്ഷിയിലും യുദ്ധങ്ങളിലും സായൂജ്യമടഞ്ഞിരുന്ന ഒരു സമൂഹ ത്തെ കേവലം 23 വര്‍ഷക്കാലം കൊണ്ട് ധാര്‍മികതയുടെ പ്രയോക്താക്കളും പ്രചാരകരുമാക്കിയ ഗ്രന്ഥമെന്ന ഖ്യാതി ഖുര്‍ആനിനു മാത്രം അവകാശപ്പെ ട്ടതാണ്. എന്നാല്‍ ധാര്‍മിക നവോത്ഥാനത്തിനുവേണ്ടി മുഹമ്മദ്(സ) രചി ച്ചുകൊണ്ട് ദൈവത്തില്‍ ആരോപിച്ച ഗ്രന്ഥമാണ് ഖുര്‍ആന്‍ എന്ന വാദ ഗതി അടിസ്ഥാന രഹിതമാണെന്ന് അത് ഒരാവര്‍ത്തി വായിക്കുന്ന ഏവര്‍ക്കും ബോധ്യമാവും. താഴെപ്പറയുന്ന വസ്തുതകള്‍ ശ്രദ്ധിക്കുക.

ഒന്ന്: സത്യസന്ധനായിരുന്നു മുഹമ്മദ് (സ) എന്ന കാര്യത്തില്‍ പക്ഷാ ന്തരമില്ല. അത്തരമൊരാള്‍ ധാര്‍മിക നവോത്ഥാനത്തിനുവേണ്ടി ദൈവത്തിന്റെ പേരില്‍ ഒരു പച്ചക്കള്ളം പറഞ്ഞുവെന്നു കരുതുന്നത് യുക്തി സഹമ ല്ല. ധാര്‍മിക നവോത്ഥാനത്തിനുവേണ്ടി ആത്മാര്‍ഥമായി പരിശ്രമിക്കുന്ന ഒരു വ്യക്തി അക്കാര്യത്തിനുവേണ്ടി സ്വന്തമായി ഒരു വലിയ അധര്‍മം ചെയ്യുകയെന്നത് അവിശ്വസനീയമാണ്. ദൈവത്തിന്റെ പേരില്‍ കളവ് പറയുന്നതിനേക്കാള്‍ വലിയ പാപമെന്താണ്?

രണ്ട്: പടച്ചവന്റെ പേരില്‍ കളവു പറയുകയും സ്വയം കൃതരചന കള്‍ ദൈവത്തിന്‍േറതാണെന്ന് വാദിക്കുകയും ചെയ്യുന്നവനാണ് ഏറ്റവും വലിയ അക്രമിയെന്നാണ് ഖുര്‍ആന്‍ പറയുന്നത്. ''അല്ലാഹുവിന്റെ പേരില്‍ കള്ളം കെട്ടിച്ചമയ്ക്കുകയോ, തനിക്ക് യാതൊരു ബോധനവും നല്‍കപ്പെടാതെ 'എനിക്ക് ബോധനം ലഭിച്ചിരിക്കുന്നു' എന്ന് പറയുകയോ ചെയ്തവനേ ക്കാളും അല്ലാഹു അവതരിപ്പിച്ചതുപോലെയുള്ളത് ഞാനും അവതരിപ്പിക്കാമെന്ന് പറഞ്ഞവനേക്കാളും വലിയ അക്രമി ആരുണ്ട്?''(6:93). ഖുര്‍ആന്‍ മുഹമ്മദി(സ)ന്റെ രചനയാണെങ്കില്‍ ഈ സൂക്തത്തില്‍ പറഞ്ഞ 'ഏറ്റവും വലിയ അക്രമി' അദ്ദേഹം തന്നെയായിരിക്കുമല്ലോ. തന്നെത്തന്നെ 'ഏറ്റവും വലിയ അക്രമി'യെന്ന് വിളിക്കുവാനും അതു രേഖപ്പെടുത്തുവാനും അദ്ദേ ഹം തയാറാകുമായിരുന്നുവോ?

മൂന്ന്: സ്വയംകൃത രചനകള്‍ നടത്തി അത് ദൈവത്തില്‍ ആരോപിക്കു ന്നവരെ ഖുര്‍ആന്‍ ശപിക്കുന്നുണ്ട്. ''എന്നാല്‍ സ്വന്തം കൈകള്‍ കൊണ്ട് ഗ്രന്ഥം എഴുതിയുണ്ടാക്കുകയും എന്നിട്ട് അത് അല്ലാഹുവില്‍നിന്ന് ലഭിച്ച താണെന്ന് പറയുകയും ചെയ്യുന്നവര്‍ക്ക് നാശം!''(2:79) ഖുര്‍ആന്‍ മുഹ മ്മദി(സ)ന്റെ സൃഷ്ടിയാണെങ്കില്‍ ഈ ശാപം അദ്ദേഹത്തിനുകൂടി ബാധക മാണല്ലോ. സ്വന്തമായി ഒരു രചന നിര്‍വഹിക്കുക. ആ രചനയില്‍ സ്വന്ത ത്തെത്തന്നെ ശപിക്കുക. ഇത് വിശ്വസനീയമാണോ?

നാല്: ഖുര്‍ആന്‍ ഒന്നിച്ച് അവതരിപ്പിക്കപ്പെട്ട ഗ്രന്ഥമല്ല. നീണ്ട ഇരുപ ത്തിമൂന്ന് വര്‍ഷങ്ങള്‍ക്കിടയില്‍ വ്യത്യസ്ത സാഹചര്യങ്ങളിലാണ് ഖുര്‍ആ ന്‍ സൂക്തങ്ങള്‍ അവതരിപ്പിക്കപ്പെട്ടത്. ഓരോ വിഷയങ്ങളിലും ജനങ്ങളുടെ ചോദ്യങ്ങള്‍ക്കുള്ള മറുപടിയായിട്ടാണ് ചില സന്ദര്‍ഭങ്ങളില്‍ ഖുര്‍ആന്‍ സൂക്തങ്ങള്‍ അവതരിപ്പിച്ചിരിക്കുന്നത്. ഖുര്‍ആനില്‍ പതിനഞ്ചോളം സ്ഥ ലങ്ങളില്‍ 'അവര്‍ നിന്നോട്...നെക്കുറിച്ചു ചോദിക്കുന്നു. പറയുക: ...' എന്ന ശൈലിയിലുള്ള സൂക്തങ്ങളുണ്ട്. ഓരോ വിഷയങ്ങളിലും പ്രവാചകനോട് അവര്‍ ചോദിച്ച സമയത്ത് അദ്ദേഹത്തിന് ഉത്തരം നല്‍കാന്‍ സാധിച്ചില്ലെ ന്നും പിന്നീട് ഖുര്‍ആന്‍ വാക്യം അവതരിപ്പിച്ചതിനുശേഷം മാത്രമാണ് അത് സാധിച്ചതെന്നുമാണല്ലോ ഇതില്‍നിന്ന് മനസ്സിലാവുന്നത്. ധാര്‍മിക നവോത്ഥാനം ലക്ഷ്യമാക്കിക്കൊണ്ടുള്ള പ്രവാചക(സ)ന്റെ രചനയായിരുന്നു ഖുര്‍ആനെങ്കില്‍ ജനം ചോദിച്ചപ്പോള്‍ ഉടന്‍തന്നെ അദ്ദേഹത്തിന് മറുപടി പറയാന്‍ കഴിയുമായിരുന്നു.

ഉദാഹരണത്തിന്, മദ്യത്തില്‍നിന്നും ചൂതാട്ടത്തില്‍നിന്നും ജനങ്ങളെ രക്ഷിക്കണമെന്നായിരുന്നു പ്രവാചകന്റെ ഉദ്ദശ്യമെങ്കില്‍ അവയെക്കുറിച്ച് ചോദിച്ച ഉടന്‍തന്നെ അവ പാപമാണ് എന്ന് അ ദ്ദേഹം മറുപടി പറയുമായിരുന്നു. എന്നാല്‍, അദ്ദേഹം ചെയ്തത് അതല്ല; സ്വയം മറുപടി പറയാതെ ദൈവിക വെളിപാട് പ്രതീക്ഷിക്കുകയായിരുന്നു. ദൈവവചനങ്ങള്‍ വെളിപ്പെട്ടതിനുശേഷമാണ് ഈ തിന്മകള്‍ക്കെതിരെയുള്ള നടപടികള്‍ അദ്ദേഹം സ്വീകരിച്ചത്.

അഞ്ച്: മുഹമ്മദ് നബി(സ)യെ തിരുത്തുന്ന ചില ഖുര്‍ആന്‍ സൂക്തങ്ങളുണ്ട്. ഖുറൈശി പ്രമുഖരുമായി സംസാരിച്ചുകൊണ്ടിരിക്കുമ്പോള്‍ കടന്നുവന്ന അന്ധനായ അബ്ദുല്ലാഹിബ്‌നുഉമ്മിമക്തൂമിനെ പ്രസന്നതയോടെ സ്വീകരിക്കാതിരുന്ന പ്രവാചക(സ)ന്റെ നടപടിയെ തിരുത്തിയ ഖുര്‍ആന്‍ സൂക്തങ്ങള്‍ (80:1-10) സുവിദിതമാണ്.

മറ്റൊരു സംഭവം: മുസ്‌ലിംകള്‍ക്ക് ഏറെ നാശനഷ്ടങ്ങള്‍ വിതച്ച ഉഹ്ദ് യുദ്ധത്തില്‍ പ്രവാചകന്റെ ശരീരത്തിലും ഒരുപാട് മുറിവുകള്‍ ഉണ്ടായി. യുദ്ധശേഷം അദ്ദേഹം അവിശ്വാസികളി ല്‍ ചിലരെ ശപിക്കുകയും 'അവരുടെ പ്രവാചകനെ മുറിപ്പെടുത്തിയ സമൂഹ മെങ്ങനെയാണ് നന്നാവുക?' എന്ന് ആത്മഗതം നടത്തുകയും ചെയ്തു. ഉടന്‍ ഖുര്‍ആന്‍ സൂക്തമവതരിച്ചു; പ്രവാചക(സ)നെ തിരുത്തിക്കൊണ്ട്. ''(നബിയേ), കാര്യത്തിന്റെ തീരുമാനത്തില്‍ നിനക്ക് യാതൊരവകാശവുമില്ല. അവന്‍ (അല്ലാഹു) ഒന്നുകില്‍ അവരുടെ പശ്ചാത്താപം സ്വീകരിച്ചേ ക്കാം. അല്ലെങ്കില്‍ അവരെ അവന്‍ ശിക്ഷിച്ചേക്കാം. തീര്‍ച്ചയായും അവര്‍ അക്രമികളാകുന്നു''(3:128)(തിര്‍മിദി,ഇബ്‌നുമാജ).

ഇതൊന്നും പ്രവാചകനി ല്‍ ബോധപൂര്‍വ്വം വന്ന തെറ്റുകളല്ല. താന്‍ സ്വീകരിച്ച നിലപാടുകളിലുണ്ടാ യ അബദ്ധം മാത്രം. എന്നിട്ടും അവ തിരുത്തുന്ന വചനങ്ങള്‍ ഖുര്‍ആനി ലുണ്ടായി. ജനങ്ങളെ ധര്‍മനിഷ്ഠരാക്കുവാന്‍ വേണ്ടി പ്രവാചകന്‍(സ) പടച്ച ഗ്രന്ഥമായിരുന്നു ഖുര്‍ആനെങ്കില്‍ അദ്ദേഹത്തിന്റെ നടപടികളെ വിമര്‍ശിക്കുന്ന സൂക്തങ്ങള്‍ ഖുര്‍ആനിലുണ്ടാവുമായിരുന്നുവോ?

ധികാരമോഹമെന്നാല്‍ എന്താണ്? രാജ്യത്തിന്റെ അധികാരം കൈ ക്കലാക്കി സുഖസമൃദ്ധമായ ജീവിതം നയിക്കാനുള്ള ആഗ്രഹം. പതിമൂന്ന് വര്‍ഷത്തെ കഷ്ടപ്പാടുകള്‍ക്കും പീഡനങ്ങള്‍ക്കും ശേഷം പലായനം ചെയ്തു മദീനയിലെത്തിയ പ്രവാചകന് അധികാരം ലഭിച്ചുവെന്നത് നേരാണ്. എന്നാല്‍, അദ്ദേഹത്തിന് അധികാരം സുഖലോലുപതയ്ക്കുള്ള മാര്‍ഗമായി രുന്നില്ല. ഭരണാധികാരിയായിരിക്കുമ്പോഴും ഈത്തപ്പനപ്പായയില്‍ അന്തിയുറങ്ങുകയും വസ്ത്രങ്ങള്‍ സ്വയം അലക്കുകയും പാദരക്ഷകള്‍ തുന്നുകയും ആടിനെ കറക്കുകയും ചെയ്യുന്ന മനുഷ്യനെ അധികാരമോഹിയെന്നു വിളിക്കാന്‍ ആര്‍ക്കാണ് സാധിക്കുക? അധികാരത്തിന്റെ പേരില്‍ ജനങ്ങളാല്‍ ആദരിക്കപ്പെടുകയും അവരില്‍ നിന്ന് ഉയര്‍ന്നുനില്‍ക്കുകയും ചെയ്യാന്‍ ആഗ്രഹിക്കുന്നവരാണ് അധികാരം മോഹിക്കുക. പ്രവാചക(സ)നാവട്ടെ ജനങ്ങളെ സേവിച്ച് ജനങ്ങളോടൊപ്പം ജീവിച്ചയാളായിരുന്നു. തന്നെ ബഹുമാനിച്ചുകൊണ്ട് ആളുകള്‍ എഴുന്നേറ്റു നില്‍ക്കുന്നതുപോലും അവിടുന്ന് ഇഷ്ടപ്പെട്ടിരുന്നില്ല.

അദ്ദേഹം ഉപദേശിച്ചു: ''ക്രിസ്ത്യാനികള്‍ മര്‍യമിന്റെ പുത്രനായ യേശുവിനെ പുകഴ്ത്തിയതുപോലെ എന്നെ നിങ്ങള്‍ പുകഴ്ത്തരുത്'' (ബുഖാരി, മുസ്‌ലിം). ഇതെല്ലാംതന്നെ മുഹമ്മദ് (സ) ഒരു അധികാര മോഹിയായിരുന്നില്ലെന്ന് വ്യക്തമാക്കുന്നു.

മാത്രവുമല്ല, തന്റെ പ്രബോധന പ്രവര്‍ത്തനങ്ങള്‍ നിര്‍ത്തിവെക്കുകയാ ണെങ്കില്‍, മക്കയിലെ പ്രയാസപൂര്‍ണമായ ആദ്യനാളുകളില്‍തന്നെ അധികാരം നല്‍കാമെന്ന് അദ്ദേഹം വാഗ്ദാനം ചെയ്യപ്പെട്ടിരുന്നു. സമൂഹത്തിലെ നേതാക്കന്മാരെല്ലാംകൂടി ഒരു ദിവസം മുഹമ്മദി(സ)ന്റെ അടുത്തുചെന്ന് അദ്ദേഹത്തെ വശീകരിക്കാനായി ആവത് ശ്രമിച്ചു നോക്കി. പക്ഷെ, നിരാശ മാത്രമായിരുന്നു ഫലം. മക്കയിലെ പ്രബോധന പ്രവര്‍ത്തനങ്ങളുടെ ആദ്യനാളുകളിലായിരുന്നു ഈ സംഭവം. ഖുര്‍ആന്‍ രചിച്ചുകൊണ്ട് താന്‍ ദൈവദൂതനാണെന്ന് വരുത്തിത്തീര്‍ത്ത് അധികാരം കൈക്കലാക്കുകയായിരുന്നു പ്രവാചക(ല)ന്റെ ലക്ഷ്യമെങ്കില്‍ പ്രയാസങ്ങള്‍ ഏറെയൊന്നും സഹിക്കാതെ അധികാരം തന്റെ കാല്‍ക്കീഴില്‍ വന്ന സമയത്ത് അദ്ദേഹം അത് സ്വീകരിക്കുവാന്‍ വൈമനസ്യം കാണിച്ചതെന്തിനാണ്? മുഹമ്മദ്അ(സ)ധികാരം കാംക്ഷിച്ചിരുന്നില്ലെന്ന് ഇതില്‍നിന്ന് സുതരാം വ്യക്തമാണ്. ഖുര്‍ആന്‍ കൊണ്ടുവന്നതിനു പിന്നില്‍ അധികാരമോഹമായിരുന്നില്ലെന്ന് സാരം.

നാഥനായി വളര്‍ന്ന മുഹമ്മദ്(സ) ചെറുപ്പത്തില്‍ ഒരുപാട് കഷ്ടപ്പാടുകള്‍ അനുഭവിച്ചിരിക്കാം. എന്നാല്‍, തന്റെ 25-ാം വയസ്സില്‍ നാല്‍പതുകാരിയായ കച്ചവടക്കാരി ഖദീജ( (റ)യെ വിവാഹം ചെയ്തതിനുശേഷം അദ്ദേ ഹത്തിന്റെ ജീവിതനിലവാരം സ്വാഭാവികമായും മെച്ചപ്പെട്ടതായി മാറിയിരിക്കണം. അത്യാവശ്യം നല്ല സാമ്പത്തിക ശേഷിയുണ്ടായിരുന്ന ഖദീജ( (റ)യുടെ ഭര്‍ത്താവായിരുന്ന അദ്ദേഹം സാമ്പത്തിക ക്ലേശങ്ങള്‍ അനുഭവിച്ചിരിക്കാനുള്ള സാധ്യത വിരളമാണ്.

ഖദീജയുമായുള്ള മുഹമ്മദി(സ) ന്റെ വിവാഹം നടന്നത് പ്രവാചകത്വം ലഭിക്കുന്നതിന് 15 വര്‍ഷങ്ങള്‍ക്കുമുമ്പാണ്. പതി നഞ്ച് വര്‍ഷം സാമ്പത്തികക്ലേശം കൂടാതെ ജീവിച്ചതിനുശേഷമാണ് താന്‍ പ്രവാചകനാണെന്നും ഖുര്‍ആന്‍ ദൈവവചനമാണെന്നുമുള്ള അവകാശവാദങ്ങളുമായി മുഹമ്മദ്(സ) രംഗപ്രവേശം ചെയ്യുന്നതെന്നര്‍ഥം. ഖുര്‍ആന്‍ ദൈവികമാണെന്ന് വാദിക്കുക വഴി ഭൗതികലാഭമാണ് അദ്ദേഹം ഇച്ഛിച്ചതെങ്കില്‍ ഈ വാദം ഉന്നയിച്ചതിനുശേഷം അദ്ദേഹത്തിന്റെ സാമ്പത്തിക സ്ഥിതി മെച്ചപ്പെട്ടിരിക്കണമല്ലോ.

എന്നാല്‍, എന്തായിരുന്നു സ്ഥിതി?

പ്രവാചകപത്‌നി ആഇശ(റ) പറയുന്നു: ''ഞങ്ങളുടെ വീട്ടില്‍ ഒന്നും പാചകം ചെയ്യാനില്ലാത്തതിനാല്‍ അടുപ്പു പുകയാതെ ഒന്നോ രണ്ടോ മാസ ങ്ങള്‍ കഴിഞ്ഞുപോകാറുണ്ടായിരുന്നു. ഈത്തപ്പഴവും വെള്ളവുമായിരുന്നു ഞങ്ങളുടെ ഉപജീവനം. ചിലപ്പോള്‍ മദീനത്തുകാര്‍ കൊണ്ടുവന്ന ആട്ടി ന്‍പാലും ഈത്തപ്പഴത്തോടു കൂടെയുണ്ടാവും''. (ബുഖാരി, മുസ്‌ലിം)

ആഇശ(റ) ഒരാളോട് പഴയകാര്യങ്ങള്‍ പറഞ്ഞുകൊണ്ടിരിക്കുകയാണ്. മദീനയിലേക്കുള്ള പലായനത്തിനുശേഷം പ്രവാചകനും കുടുംബവും സ ഹിച്ച പ്രയാസങ്ങളാണ് പ്രതിപാദ്യം. ഒരു രാത്രി തപ്പിത്തടഞ്ഞുകൊണ്ട് വീട്ടുജോലികള്‍ ചെയ്തകാര്യം അവര്‍ പറഞ്ഞു. അയാള്‍ ചോദിച്ചു: ''വിള ക്കില്ലായിരുന്നുവോ? അവര്‍ പ്രതിവചിച്ചു: ''വിളക്കു കത്തിക്കാനുള്ള എണ്ണ ഞങ്ങളുടെ പക്കലുണ്ടായിരുന്നുവെങ്കില്‍ വിശപ്പ് മാറ്റാന്‍ അത് കുടിക്കുമാ യിരുന്നു; കത്തിക്കുന്നതിനു പകരം''. (അഹ്മദ്, ത്വബ്‌റാനി)

ഇത് പ്രവാചകന്റെ ആദ്യകാലത്തെ മാത്രം അവസ്ഥയല്ല. മുഹമ്മദ്(സ) ശക്തമായ ഒരു സാമ്രാജ്യത്തിന്റെ ഭരണാധികാരിയായിരുന്നപ്പോഴും അദ്ദേ ഹത്തിന്റെ അവസ്ഥ ഇതില്‍നിന്ന് ഒട്ടും മെച്ചമായിരുന്നില്ല. ഇസ്‌ലാമിക സാമ്രാജ്യത്തിന്റെ അധിപന്റെ അന്തപുരത്തെക്കുറിച്ച് അദ്ദേഹത്തിന്റെ സഹചാരിയായിരുന്ന ഉമര്‍ (റ)തന്നെ പറയട്ടെ: ''പ്രവാചകന്റെ മുറിയില്‍ ഊറക്കിട്ട മൂന്ന് തോല്‍കഷ്ണങ്ങളും ഒരു മൂലയില്‍ അല്‍പം ബാര്‍ലിയുമല്ലാതെ മറ്റൊന്നുംതന്നെ ഞാന്‍ കണ്ടില്ല. ഞാന്‍ കരഞ്ഞുപോയി. പ്രവാചകന്‍ ചോദിച്ചു: 'എന്തിനാണ് താങ്കള്‍ കരയുന്നത്?' ഞാന്‍ പറഞ്ഞു: 'അല്ലാഹുവിന്റെ ദൂതരേ! ഞാനെങ്ങനെ കരയാതിരിക്കും? താങ്കളുടെ ശരീരത്തില്‍ ഈത്തപ്പനയോലകളുടെ പാട് ഞാന്‍ കാണുന്നു. ഈ മുറിയില്‍ എന്തെല്ലാമുണ്ടെന്നും ഞാനറിയുന്നു. അല്ലാഹുവിന്റെ ദൂതരേ! സമൃദ്ധമായ വിഭവങ്ങള്‍ക്കുവേണ്ടി അല്ലാഹുവിനോട് പ്രാര്‍ ഥിച്ചാലും. അവിശ്വാസികളും അല്ലാഹുവില്‍ പങ്കുചേര്‍ക്കുന്നവരുമായ പേര്‍ഷ്യക്കാരുടെയും റോമാക്കാരുടെയും രാജാക്കന്മാര്‍-സീസറും കൈസറുമെല്ലാം-അരുവികള്‍ ഒഴുകുന്ന തോട്ടങ്ങളില്‍ വസിക്കുമ്പോള്‍ അല്ലാഹുവിന്റെ തെരഞ്ഞെടുക്കപ്പെട്ട പ്രവാചകന്‍ ജീവിക്കുന്നത് ദാരുണമായ പട്ടിണിയില്‍!' എന്റെ ഈ സംസാരം കേട്ടപ്പോള്‍ തലയിണയില്‍ വിശ്രമിക്കുകയായിരുന്ന പ്രവാചകന്‍ എഴുന്നേറ്റിരുന്നു. എന്നിട്ടു പറഞ്ഞു: 'ഉമര്‍! താങ്കള്‍ ഈ വിഷയത്തില്‍ ഇനിയും സംശയാലുവാണോ? ഭൗതിക ജീവിതത്തിലെ സുഖസൗകര്യങ്ങളേക്കാള്‍ നല്ലത് മരണാനന്തര ജീവിതത്തിലെ സുഖസൗ കര്യങ്ങളാണ്. അവിശ്വാസികള്‍ അവരുടെ നന്മയുടെ വിഹിതം ഈ ജീവിതത്തില്‍ ആസ്വദിക്കുന്നു. നമ്മുടേതാകട്ടെ, മരണാനന്തര ജീവിതത്തിലേക്കുവേണ്ടി ബാക്കിവെച്ചിരിക്കുകയാണ്'. ഞാന്‍ അദ്ദേഹത്തോട് അപേക്ഷിച്ചു: 'ദൈവദൂതരെ! എനിക്കുവേണ്ടി മാപ്പിനപേക്ഷിച്ചാലും. എനിക്കു തെറ്റിപ്പോയി''.

ഖുര്‍ആന്‍ ഭൗതിക ലാഭങ്ങള്‍ക്കുവേണ്ടി പടച്ചുണ്ടാക്കിയ മുഹമ്മദി(സ) ന്റെ കൃതിയാണെന്ന വാദമാണിവിടെ തകരുന്നത്. ആകെ സ്വത്തായി ബാക്കിയുണ്ടായിരുന്ന ഏഴു ദീനാര്‍ മരണത്തിനുമുമ്പ് ദാനം ചെയ്യുകയും യഹൂദന് തന്റെ പടച്ചട്ട പണയം വെച്ചുകൊണ്ട് മരണപ്പെടുകയും ചെയ്ത മനുഷ്യന്‍ ധനമോഹിയായിരുന്നുവെന്ന് പറയുന്നത് അടിസ്ഥാനരഹിതമാണ്.

ഖുര്‍ആനിന്റെ രചനക്കുപിന്നില്‍ ധനമോഹമായിരുന്നുവെന്ന വാദം അടിസ്ഥാന രഹിതമാണെന്ന് ദി ന്യു കാത്തോലിക് എന്‍സൈക്ലോപീഡിയ പോലും സമ്മതിച്ചിട്ടുണ്ട്. ''മുഹമ്മദി(സ) ന്റെ മതവിപ്ലവത്തിനു പിന്നില്‍ ധനമോഹമായിരുന്നുവെന്ന ഒരു ധാരണ സൃഷ്ടിക്കപ്പെട്ടിട്ടുണ്ട്. വ്യക്തമാ യി അറിയപ്പെടുന്ന വസ്തുതകള്‍ ഈ ധാരണക്കെതിരാണ്'' (The New Catholic Encyclopedia Vol IX, Page 1001).

മുഹമ്മദ് നബി(സ) ജീവിച്ചത് ചരിത്രത്തിന്റെ വെളിച്ചത്തിലാണ്. അദ്ദേ ഹത്തിലൂടെയാണ് ലോകം ഖുര്‍ആന്‍ ശ്രവിച്ചത്. അതുകൊണ്ടുതന്നെ ഖുര്‍ആനിന്റെ ദൈവികത അംഗീകരിക്കാത്തവരെ സംബന്ധിച്ചിടത്തോളം അവര്‍ക്ക് പറയാനുള്ളത് ഇത് മുഹമ്മദി(സ)ന്റെ രചനയാണെന്നാണ്. ഈ വാദം വിശദമായി ചര്‍ച്ച ചെയ്യേണ്ടതാണ്. ചര്‍ച്ചയുടെ ആമുഖമായി നാം മനസ്സിലാക്കേണ്ട ചില വസ്തുതകളുണ്ട്. അവയുടെ അടിത്തറയില്‍നിന്നുകൊണ്ടായിരിക്കണം മുഹമ്മദ് നബി(സ)യില്‍ ഖുര്‍ആനിന്റെ കര്‍തൃത്വം ആരോപിക്കുന്നത്.

ഒന്ന്: നാല്‍പതു വയസ്സുവരെ അറബികള്‍ക്കിടയില്‍ സുസമ്മതനായ വ്യക്തിയായിരുന്നു മുഹമ്മദ്(സ). ഖുര്‍ആന്‍ ദൈവികമാണെന്നും അതിലെ വിധിവിലക്കുകള്‍ അനുസരിക്കേണ്ടതുണ്ടെന്നും പ്രബോധനം ചെയ്തതുകൊണ്ടാണ് അദ്ദേഹം വെറുക്കപ്പെട്ടവനായത്; ബഹിഷ്‌കരിക്കപ്പെട്ടത്; ജനിച്ച് വളര്‍ന്ന നാട്ടില്‍ നിന്ന് പലായനം ചെയ്യേണ്ടിവന്നത്.

രണ്ട്: സത്യസന്ധനായിരുന്നു മുഹമ്മദ്(സ)എന്ന കാര്യത്തില്‍ അദ്ദേഹ ത്തിന്റെ കഠിന ശത്രുക്കള്‍ക്കുപോലും അഭിപ്രായവ്യത്യാസമുണ്ടായിരുന്നി ല്ല. നാല്‍പതു വയസ്സുവരെ സത്യസന്ധനായി ജീവിച്ച അദ്ദേഹം ഒരു ദിവ സം പടച്ചതമ്പുരാന്റെ പേരില്‍ ഒരു പച്ചക്കള്ളം പറഞ്ഞുവെന്നും അത് പ്രചരിപ്പിക്കുന്നതിനുവേണ്ടി സ്വന്തം ജീവന്‍ തൃണവത്ഗണിച്ചുവെന്നും വിശ്വസിക്കുക പ്രയാസമാണ്.

മൂന്ന്: സാഹിത്യകാരന്മാര്‍ക്ക് അറേബ്യയില്‍ ഉന്നതമായ സ്ഥാനം നല്‍ കപ്പെട്ടിരുന്നു. ഖുര്‍ആന്‍ അത്യുന്നതമായ ഒരു സാഹിത്യ സൃഷ്ടിയാണെന്ന കാര്യത്തില്‍ ആര്‍ക്കും ഭിന്നാഭിപ്രായമുണ്ടായിരുന്നില്ല. അത് തന്‍േറതാണ് എന്ന് അവകാശപ്പെട്ടിരുന്നുവെങ്കില്‍ അദ്ദേഹത്തിന് അറബികള്‍ക്കിടയില്‍ ഉന്നതമായ സ്ഥാനമാനങ്ങള്‍ ലഭിക്കുമായിരുന്നു.

നാല്: മുഹമ്മദി(സ)ന്റെ ചില നടപടികളെ വിമര്‍ശിക്കുന്ന വാക്യങ്ങള്‍ ഖുര്‍ആനിലുണ്ട്. അഞ്ച്: മുഹമ്മദി(സ)നെ ശക്തമായി താക്കീത് ചെയ്യുന്ന വചനങ്ങളും ഖുര്‍ആനിലുണ്ട്. ഈ വസ്തുതകള്‍ മുന്നില്‍ വെച്ചുകൊണ്ടാണ് ഖുര്‍ആന്‍ മുഹമ്മദി(സ)ന്റെ സൃഷ്ടിയാണ് എന്ന വാദത്തിലെ ശരിയും തെറ്റും പരിശോധിക്കേ ണ്ടത്.

സാഹിത്യമൂല്യമുള്ള ഒരു സൃഷ്ടി നടത്തി അത് ദൈവത്തിന്റെ പേരില്‍ ആരോപിച്ചതാണെങ്കില്‍ അതിനു പിന്നില്‍ സ്വാര്‍ഥമായ വല്ല ലക്ഷ്യങ്ങളുമുണ്ടാവണമല്ലോ. അതെന്തായിരുന്നുവെന്നാണ് വിമര്‍ശകര്‍ ആദ്യം വ്യക്തമാക്കേണ്ടത്. അതിന്റെ അടിസ്ഥാനത്തിലാണ് ഈ വാദത്തിന്റെ സത്യത പരിശോധിക്കപ്പെടേണ്ടത്.